Transcript
Page 1: Standardized Testing Chapter 4 Brown

CHAPIER 4

STANDARDIZED TESTING

Every educated person has at some point been touched-if not deeply affectedshyby a standardized test For almost a century schools universities businesses and governments have looked to standardized measures for economical reliable and valid assessments of those who would enter continue in or exit their institutions Proponents of these large-scale instruments make strong claims for their usefulness when great numbers of people must be measured quickly and effectively Those claims are well supported by reams of research data that comprise construct valishydations of their efficacy And so we have become a world that abides by the reshysults of standardized tests as if they were sacrosanct

The rush to carry out standardized testing in every walk of life has not gone unchecked Some psychometricians have stood up in recent years to caution the public against reading too much into tests that require what may be a narrow band of specialized intelligence (Sternberg 1997 Gardner 2000 Kooo 2000) Organizations such as the National Center for Fair and Open Testing (wwwfairtestorg) have reminded us that standardization of assessment procedures creates an illusion of validity Strong claims from the giants of the testing industry they say have pulled the collective wool over the publics eyes and in the process have incorrectly marshyginalized thousands if not millions of children and adults worldwide These socioshyeconomic issues in standardized testing are discussed in Chapter 5

Whichever side is right -and both sides have legitimate cases-it is imporshytant for teachers to understand the educational institutions they are working in and an integral part of virtually all of those institutions is the use of standardized tests So it is important for you to understand what~~d~~4z~lt1 tests are ~y are not hgw to_interPret them and how to put them into a balanced perspective in which we strive to accurately assess all learners on all proposed objectivesWe can learn a great deal about many learners and their competencies through standardized forms of assessment But some of those learners and some of those objectives may not be adequately measured by a sit-down timed multiple-choice format that is likely to be decontextualized

This chapter has two goals to introduce the process of constructing valishydating admiistering and interpreting -sta-noartlizea-middottescs ofIarlguage and to

66

YAMITH
Typewriter
Brow H D (2004) Language assessment principles and classroom practices USA Pearson Longman
YAMITH
Typewriter
YAMITH
Typewriter
FOR EDUCATIONAL PURPOSES ONLY

CHAPTER4 Standardized Testing 67

___~--9~in~_YQ9~ith a variety of current standardized tests that claim to test overall language proficiency

It should be clear from these goals that in this chapter we are not focusing celshytrally on classroom-based assessment Dont forget however that stand~diz~d tests affect all classrooms and some of the practical steps that are involved in creating standardized tests are directly transferable to designing classroom tests

~

WHAT IS STANDARDIZATION

A standardized test presupposes certain standard objectives or criteria that are held constant across one form of the test to another The criteria in large-scale standardized tests are designed to apply to a broad band of competencies that are usually not exclusive to one particular curriculumA good standardized test is the product of a thorough process of empirical research and development It dictates standard procedures for administration and scoring And finally it is typical of a norm-referenced test the goal ofwhich is to place test-takers on a continuum across a range of scores and to differentiate test-takers by their relative ranking

Most elementary and secondary schools in the United States have standardized achievement tests to measure childrens mastery of the standards or competencies that have been prescribed for specified grade levels These tests vary by states counties and school districts but they all share the common objective of econOffishyicallarge-scale assessment Collegemiddot entrance exams such as the Scholastic Aptitude Test (SA~ are part of the educational experience of many high school seniors seeking further education The Graduate Record Exam (GRE~ is a required stanshydardized test for entry into many graduate school programs Tests like the Graduate Management Admission Test (GMA1) and the Law School Aptitude Test (LSAn speshycialize in particular diSCiplines One genre of standardized test that you may already be-familiar with is the Test of English as a Foreign Language (fOEFL ~produced by die Educational Testing Service (ETS) in the United States andor its British counshyterpart the International English Language Testing System (lELTS) which features standardized tests in affiliation with the University of Cambridge Local Examinations Syndicate (UCLES) They are all standardized because they specify a set of compeshytencies (or standards) for a given domain and through a process of construct valishydation they program a set of tasks that have been designed to measure those competencies

Many people are under the incorrect impression that all stanoardized tests conshysist of items that have predetermined responses presented1n a multiple-choice format While it is true that many standardized tests conform to a multiple-choice format by no means is multiple-choice a prerequisite characteristic It so happens that a multiple-chOice format provides the test producer with an objective means for determining correct and incorrect responses and therefore is the preferred mode for large-scale tests However standards are equally involved in certain humanshyscored tests of oral production and writing such as the Test of Spoken English (fSE~ and the Test ofWritten English (IWE~ both produced by ETS

68 CHAPTER 4 Standardized Testing

ADVANTAGES AND DISADVANTAGES OF STANDARDIZED TESTS

Advantages of standardized testing include foremost a re~gY-=1ade previously valishydated product that frees the teacher from having to spe~d hours creating a test Administration to large groups can be accomplished within reasonable time limits In the case ofmiddotinultiple-choice formats scoring procedures are streamlined (for either scal18Qble computerized scoring or hand-SCOring with a hole-punched grid) for fast turnaround time And for better or for worse there is often an air of face validity to such authoritative-looking instruments

Disadvantages center largely on the inp-Jl9priate use of such tests for example using an_QY~1llpr9tl~i~tl9_~est as an achievemenftesfsUnplY because of the convenience of the standardization A colleague told me-about a course director who after a frantic search for a last-minute placement test administered a multipleshychoice granunar achievement test even though the curriculum was mostly listening and speaking and involved few of the granunar points tested This instrunrent had the appearance and face validity of a good test when in reality it had no -content Validity whatsoever

Another disadvantage is the potential misunderstanding of the difference between directandindirect testing (see Chapter 2) Some standardized tests include tasks that do__-2QJrectly specify Q~~~~_~ the target r objpound~tive For example before 1996 the TOEFL included neither a written nor an oral production section yet statistics showed a reasonably strong correspondence between performance on the TOEFL and a students written and-to a lesser extent-oral production The comprehension-based TOEFL could therefore be claimed to be an indirect test of production A test of reading comprehension that proposes to me~e ability to read extensively and that engages test-takers in reading only short one or twoshyparagraph passages is anJndirect measure of extensive reading

Those who use standardized tests need to acknowledge both the advantages and limitations of indirect testing In the pre1996 TOEFL administrations the expense of giving a direct test of production was considerably reduced by offering only comprehension performance and showing through construct validation the appropriateness of conclusions about a test-takers production competence likewise short reading passages are easier to administer and if research validates the assumption that short reading passages indicate extensive reading ability then the use of the shorter passages is justified Yet the construct validation statistics that offer that support never offer a 100 percent probability of the relationship leaving room for some possibility that the indirect test is not valid for its targeted use

A more serious isslle lies in the assumption (alluded to above) that standardized tests correctly assess all learners equally well Wellestablished standardized tests usually demonstrate high correlations between performance on such tests and target objectives but correlations are not sufficient to demonstrate unequivocally the acquisition of criterion objectives by all test-takers Here is a non-language example In the United States some driverS iicense renewals require taking a papershyand-pencil multiple-choice test that covers signs safe speeds and distances lane

CHAPTER 4 Standardized Testingmiddot 69

changesand otherrules of the road Correlational statistics show a strong relationship between high scores on those tests and good driving records so people who do well on these tests are a safe bet to relicense Now an extremely high correlation (of pershyhaps SO or above) may be loosely interpreted to mean that a large majority of the drishyvers whose licenses are renewed by virtue of their having passed the little quiz are good behind-the-wheel drivers What about those few who do not fit the model That small minority of drivers could endanger the lives of the majority and is that a risk worth taking Motor vehicle registration departments in the United States seem to think so and thus avoid the high cost of behind-the-wheel driving tests

Are you willing to rely on a standardized test result in the case of all the learners in your class Of an applicant to your institution or of a potential degree candidate exiting your program These questions will be addressed more fully in Chapter 5 but for the moment think carefully about what has come to be known

as high-stakes testing in which standardized tests have become the only criterion for inclusion or exclusion The widespread acceptance and sometime misuse of this gate-keeping role of the testing industry has created a political educational and moral maelstrom

DEVELOPING A STANDARDIZED TEST

Whlle it is not likely that a classroom teacher with a team of test designers and researchers would be in a position to ~evelop a brand-new standardized test of large-scale proportions it is a yirtual certainty that some day you will be in a posishytion (a) to revise an existing test (b) toadapt or expand an existing test andor (c) to create a smaller-scale standardized test for a program you are teaching in And even if none of the above thre~ cases should ever apply to you it is of paramount importance to understand the process of the development of the standardized tests that have become ingrained in our educational institutions

How are standardized tests developed Where do test tasks and items come from How are they evaluated Who selects items and their arrangement in a test How do such items and tests- achieve consequential validity How are different forms of t~sts equated for diftlculty level Who sets norms and cutoff limits Are security add confidentiality an issue Are cultural and racial biases an issue in test development All these questions typify those that you might pose in an attempt to understand the process of test development

In the steps outlined below three different standardized- tests will be used to exemplify the process of standardized test design

(A) The Test of English as a Foreign Language (TOEFL) Educational Testing Service (ETS) (B) The English as a Second Language Placement Test (ESLP1) San Francisco State University (SFSU) (C)The Graduate Essay Test (Gm SFSU

70 CHAPTER 4 Standardized Testing

The first is a test of general language ability or profidencyThe second is a place ment test at a universityAnd the third is a gate-keeping essay test that all prospecshytive students must pass in order to take graduate-level courses As we look at the steps one by one you will see patterns that are consistent with those outlined in the previous two chapters for evaluating and developing a classroom test

1 Determine ihe purpose and objectives of the test

Most standardized tests are expected to provide high practicality in administration and scoring without unduly compromising validity The initial outlay of time and money for such a test is Significant but the test would be used repeatedly It is therefore imporshytant for its purpose and objectives to be stated specifically Lets look at the three tests

(A) The purpose of the TOEFL is to evaluate the English profiCiency of people whose native language is not English (TOEFL Test and Score Manual 2001 p 9) More specifically theTOEFL is designed to help institutions of higher learning make valid decisions concerning English language profiCiency lin terms of lthelr] own requirements (p 9) Most colleges and universities in the United States use TOEFL scores to admit or refuse international applicants for admission Various cut-off scores apply but most institutions require scores from 475 to 525 (paper-based) or from 150 to 195 (computer-based) in order to consider students for admissionThe high-stakes gate-keeping nature of the TOEFL is obvious

(B) The ESLPT referred to in Chapter 3 is designed to place already admitted students at San Francisco State University in an appropriate course in academic writing with the secondary goal of placing students into courses in oral production and grammar-editing While the tests primary purpose is to make placements another desirable objective is to provide teachers with some diagnostic information about their students on the first day or two of classThe ESLPT is locally designed by university faculty and staff

(C) The GEfanother test designed at SFSU is given to prospective graduate students-both native and non-native speakers-in all disciplines to determine whether their writing ability is sufficient to permit them to enter graduate-level courses in their programs It is offered at the beginning of each term Students who fail or marginally pass the GET are technically ineligible to take graduate courses in their field Instead they may elect to take a course in graduate-level writing of research papers A pass in that course is equivalent to passing the GET

As you can see the objectives of each of these tests are specific The content of each test must be designed to accomplish those particular ends TIlis first stage of goal-setting might be seen as one in which the consequential validity of the test is foreshymost in the mind of the developer each test has a specific gate-keeping function to perfonn therefore the criteria for entering those gates must be specified ~ccurately

2 Design test specifications

Now comes the hard part Decisions need to be made on how to go about structurshying the specifications of the test Before specs can be addressed a comprehensive

CHAPTER4 Standardized Testing 71

progratn of research must identify a set of constructs underlying the test itself This stage of laying the foundation stones can occupy weeks months or even years of effort Standardized tests that dont work are often the product of short-sighted conshystruct validation Lets look at the three tests again

(A) Construct validation for the TOEFL is carried out by the TOEFL staff at ETS under the guidance of a Policy Counell that works with a Committee of Examiners that is composed of appointed external university faculty linguists and assessment spedaJists Dozens of employees are involved in a complex process of reviewing curshyrentTOEFL specifications cOmmissioning and developing test tasks and items assemshybling forms of the test and performing ongoing exploratory research related to formulating new specs Reducing such a complex process to a set ofsimple steps runs the risk of gross overgeneralization but here is an idea of how aTOEFL is created

Because the TOEFL ismiddot a proficiency test the fIrst step in the developmental process is to define the construct of language proficiency First it should be made clear that many assessment specialists such as Bachman (1990) and Palmer (Bachman amp Palmer 1996) prefer the term ability to proficiency and thus speak of language abllity as the overarching concept The latter phrase is more conSistent they argue with our understanding that the specific components of language ability must be assessed separately Others such as the American Council on Teaching Foreign Languages (ACIFL) still prefer the term proficiency because it connotes more of a holistic unitary trait view of language ability (Lowe 1988) Most current views accept the ability argument and therefore strive to specify and assess the many components of language For the purposes of consistency in this book the term proficiency will nevertheless be retained with the above caveat

How you view language will make a difference in how you assess language proshyficiency After breaking language competence down into subsets of listening speaking reading and writing eapoundli~p~ifQ(m~I~~ Jll()4~ can be examined on a conshytiiiuum of linguistic units pho~~logy (p~onunciation) and orthography (spelling) words OeXicon) sentences (gtammar) discourse and pragmatic (sociolinguistic contextual functional cul~j features of language

How will the TOEFL sample from at) these possibilities Oral production tests can be tests of overall conversational fluency or pronunciation of a particular subset of phonolOgy and can take the form of imitation structured responses or free responses Listening comprehension tests can concentrate on a particular feature of language or on overalllistenins for general meaning Tests of realtling can cover the range of language units and can aim to test comprehension of long or short passhysages single sentences or even phrases and words Writing-tests can take on an open-ended form with free composition or be structured to elicit anything from correct spelling to discourse-level competence Are you overwhelmed yet

From the sea of potential performance modes that could be sampled in a test the developer must select a subset on some systematic basis To make a very long story short (and leaving out numerous controversies) the TOEFL had for many years included three types of performance in its organizationai specifications listening strucshyture and reading all of vtuch tested comprehension through standard multiple-choice

72 CHAPTER 4 Standardized Testing

tasks In 1996 a major step was taken to include written production in themiddot computershybased TOEFL by adding a slightly modified version of the already existing Test of Written English (TWE) In doing so some face validity and content validity were improved along with of course a significant increase in administrative expense Each of these four major sections is capsulized in the box below (adapted from the descripshytion of the current computer-based TOEFL at wwwroefLorg) Such descriptions are not strictly speaking specifications which are kept confidential by ETS Nevertheless they can give a sense of many of the constraints that are placed on themiddot design of actual TOEFL specifications

TOEFLlt8gt specifications

Listening Section The listening section measures the examinees ability to understand English as it is spoken in North America Conversational features of the language are bull stressed and the skills tested include vocabulary and idiomatic expression as well as speshycial grammatical constructions that are frequently used in spoken Engfish The stimulus material and questions are recorded in standard North American English

The listening section includes various stimuli such as dialogues short conversations academic discussions and mini-lectures and poses questions that test comprehension of main ideas the order ofa process supporting ideas important details and inferences as well as the ability to categorize topicsobjects

The test developers have taken advantage of the multimedia capability of the computer by using photos and graphics to create context and support the content of the lectures producing stimuli that more closely approximate Ureal-world situations in which people do more than just listen to voices The listening stimuli are often accompashynied by either context-setting or content-based visuals All dialogues conversations acashydemic discussions and mini-lectures include context visuals to establish the setting and role of the speakers Content-based visuals are often used to complement th~ topics of the mini-lectures

Structure-Section Themiddotstructure-section measures an examinees ability to recognize language that is appropriate for standard written English The language tested is formal rather than conversational The topics of the sentences are associated with general acadeshymic discourse so that individuals in specific fields of study or from specific national or linguistic groups have no particular advantage

Two types of questions are used questions in which examinees must (1) complete an incomplete sentence using one of four answers provided and (2) identify one of four unshyderlined words or phrases that would not be accepted in English The two question types are mixed randomly rather than being separated into two subsections as in the papershybased TOEFL test

Reading Section The reading section measures the ability to read and understand short passages similar in topic and style to academic texts used in North American colshyleges and universities Examinees read a variety of short passages on academic subjects and answer several questions about each passage Test items refer to what is stated or imshyplied in the passage as well as to words used in the passage To avoid creating an advanshyt~ t( 1 dull5- In 3n~ ont fidd of stud~ sufficient context is provided SO that no ~ -middott - ~tmiddot J ~ middoth - -~ rM~ ir~ to ~-er the if~(J(lS~ ~ t ~ ~ ~~~~-- Ih L_ 1_ 1 ~_

CHAPTER 4 Standardized Testing 73

The reading section consists of four to five passages of 250-350 words with 10-14 questions per passage This section is not computer-adaptive soexaminees can skip questions and return to previous questions The questions in this section assess the comshyprehension of main ideas inferences factual information stated in a passage pronoun referents and vocabulary (direct meaning synonym antonym) In all cases the questions can be answered by reading and understanding the passages This section consists of (1 ) traditional multiple-choice questions (2) questions that require examinees to click on a word phrase sentence or paragraph to answer and (3) questions that ask examinees to insert a sentence where it fits best

Writing Section The writing section measures the ability to write in English including the ability to generate organize and develop ideas to support those ideas with examples or evidence and to compose a response to one assigf)ed topic in standard written Enshyglish Because some examinees may not be accustomed to composing an essay on comshyputer they are given the choice of handwriting or typing the essay in the 30-minute time limit The rating scale for scoring the essay ranging from 0 to 6 is virtually the same as that of the Test of Written English [see Chapter 9 of this book] A score of 0 is given to papers that are blank simply copy the topic are written ina language other than English consist only of random keystroke characters or are written on a topic different from the one assigned

Each essay is rated independently by two trained certified readers Neither reader knows the rating assigned by the other An essay will receive the average of the two ratshyings unless there is a discrepancy of more than one point in that case a third reader will independently rate the essay The essay rating is incorporated into the StructureMriting scaled score~ and constitutes approximately 50 percent of that combined score

(B) The designing of the test specs for the ESLPT was a somewhat simpler task because the purpose is placement and the construct validation of the test consisted of an examination of the content of the ESL courses In fact in a recent revisiofi of the ESLPT (lmao et al 2000 Imao 2001) content validity (coupled with its attenshydant face validity) was the central theoretical issue to be considered The major issue centered on designing practical and reliable tasks and item response formats Having established the importance of designingESLPT tasks that simulated classroom tasks used in the courses the designers ultimately specified two writing production tasks (one a response to an essay that students read and the other a summary of another essay) and one multiple-choice grammar-editing taskThese specifications mirrored the readingbased process writing approach used in the courses

(C) Specifications for the GET arose out of the perceived need to provide a threshold of acceptable writing ability for all prospective graduate students at SFSU both native and non-native speakers of EnglishThe specifications for the GET are the skills of writing grammatically and rhetorically acceptable prose on a topic of some interest with clearly produced organization of ideas and logical development The GET is a direct test of writing ability in which test-takers must in a two-hour time period write an essay on a given topic

74 CHAPTER 4 Standardized Testing

3 Design select and arrange test tasksitems

Once specifications for a standardized test have been stipulated the sometimes never-ending task of designing selecting and arranging items beginS The spe~s act much like a blueprint in determining the number and types of items to be created Lets look at the three examples

(A) TOEFL test design specifies that each item be coded for content and statisshytical characteristics Content coding ensures that each examinee will receive test questions that assess a variety of skills (reading comprehending the main idea or understanding inferences) and cover a variety of subject matter without unduly biasing the content toward a subset of test-takers (for example in the listening secshytion involving an academic lecture the content must be universal enough for stushydents from many different academic fields of study) Statistical characteristics including the IRT equivalents of estimates of item ~~ility (IF) and the ability of an item to discriminate (ID) between higher orlower ability levels ate also coded

Items are then designed by a team who select and adapt items solicited from a bank of items that have been deposited by freemiddotlance writers and ErS staff Probes for the reading section for example are usually excerpts from authentic general or academic reading that are edited for linguistic difficulty culture bias or other topic biases Items are designed to test overall comprehension certain specific informashytion and inference

Consider the following sample of a reading selection and ten items based on it from a practice TOEFL (Phillips 2001pp423-424)

For hundreds of years in the early history of America pirates sailed through coastal washyters pillaging and plundering all in their path They stole from other ships andstole from coastal towns not content only to steal they destroyed everything they could not carry avay~ Some of the pirate ships amassed large treasures~ the fates of which are unknown leaving people of today to wonder at their whereabouts and to dream of one day coming across some lost treasure

One notoriously large treasure was on the pirate ship Whidah which sank in the washyters off Cape Cod during a strong storm in 1717 A hundred of the crew members went down with the ship along with its treasure of coins gold silver and jewels The treasure on board had an estimated value on todays market of more than 100 million dollars

The remains of the Whidah were discovered in 1984 by Barry Clifford who had spent years of painstaking research and tireless searching only finally to locate the ship about 500 yards from shore A considerable amount of treasure from the centuries-old ship has been recovered from its watery grave but there is clearly still a lot more out there Just as a reminder of what the waters off the coast have been protecting for hundreds of years occasional pieces of gold or silver or jewels still wash up on the beaches and lucky beach-goers find pieces of the treasure

11 ~ Thepa~e mainly diccus5eS

CH4PTER 4 Standardized Testing 75

(e) what really happened to the Whidahs pirates (D) why people go to the beach

12 It is NOT mentioned in the passage that pirates did which of the following (A) They killed lots of people (B) They robbed other ships (e) They took things from towns (D) They gathered big treasures

13 The word amassed in line 4 is closest in meaning to (A) sold (e) transported (B) hid (D) gathered

14 It is implied in the passage that the Whidahs crew (A) died (B) went diving (e) searched for the treasure (D) escaped with parts of the treasure

15 Which of the following is NOT mentioned as part of the treasure of the Whidah (A) Art objects (B) Coins (e) Gold and si Iver (D) Jewels

16 The word estimated in line 10 is closest in meaning to which of the following (A) Known (C) Approximate (B) Sold (D) Decided

17 The passage indicates that the cargo of theWhidah is worth about (A) $100000 (B) $1000000 (C) $10000000 (D) $100000000

18 The work that Barry Clifford did to locate the Whidah was NOT (A) successfu I (B) effortless (C) detailed (D) lengthy

19 It is mentioned in the passage that the treasure of the Whidah (A) is not very valuable (8) is all in museums (C) has not all been found (D) was taken to share by the pi rates

20 The paragraph following the passage most likely discusses (A) what Barry Clifford is doing today (8) the fate of the Whidahs crew (e) other storms in the area of Cape Cod (D) additional pieces that turn up from the Whidahs treasure

76 CHAPTER 4 Standardized Testing

As you can see items target the assessment of comprehension of the main idea (item 11) stated details (17 19) unstated details (12 15 18) implied details (14 20) and vocabulary in context (13 16) An argument could be made about the cultural schemata implied in a passage about pirate ships and you could engage in an angels on the head of a pin argument about the importance of picking cershytain vocabulary for emphasis but every test item is a sample of a larger domain and each of these fulfills its designated specification

Before any such items are released into a form of the TOEFL (or any validated standardized test) they are piloted and sCientifically selected to meet difficulty specshyifications within each subsection section and the test overall Furthermore those items are also selected to meet a desired discrimination index Both of these indices are important considerations in the design of a computer-adaptive test where pershyformance on one item determines the next one to be presented to the test-taker (See Chapter 3 for a complete treatment of multiple-choice item design)

(B)The selection of items in the ESLPT entailed two-entirel) different processes In the two subsections of -the test that elicit writing performance (summary of reading response to reading) the main hurdles were (a) selecting appropriate passhysages for test-takers to read (b) providing appropriate prompts and (c) processing data from pilot testing Passages have to conform to standards of content validity by being within the genre and the difficulty of the material used in the courses The prompt in each case (the section asking for a summary and the section asking for a response) has to be tailOred to fit the passage but a general template is used

[n the multiple-choice editing test that seeks to test grammar proofreading ability the first and easier task is to choose an appropriate essay within which to embed errors The more complicated task is to embed a specified number of errors from a previously determined taxonomy of error categories Those error categories came directly from student errors as perceived by their teachers (verb tenses verb agreeshyment logical connectors articles etc) The disttactors for each item were selected from actual errors that students make Itemsiti pilot versions were then coded fordifshyficulty and discrinlination indices after which final assembly of items could occur

(C) The GET prompts are designed by a faculty committee of examiners who are speCialists in the field of university academic writing The assumption is made that the topics are universally appealing and capable of yielding the intended product of an essay that requires an organized logical argument and conclusion No pilot testing of prompts is conducted The conditions for administration remain constant two-hour time limit sit-down context paper and pencil closed-book format Consider the following recent prompt

Graduate Essay Test sample prompt

In the Middletown Elementary School District the assistant superintendent has just been made superintendent in another district Her resignation leaves vacant the districts only administrative position ever held by a woman The School Board in response to strong

CHAPTER 4 Standardized Testing 77

arguments from the Teachers Association has urged that a woman be hired to replace her As a member of the hiring committee you must help choose her successor

Only one woman applicant meets the written qualifications for the job the two top male applicants are both more experienced than she

The hiring committee has asked each committee member to prepare a written statement to distribute before meeting together to discuss the issue Write a report that represents your position making it as logical and persuasive as possible

Some facts you may wish to draw on 1 Women make up more than 75 percent of classroom teachers but hold fewer than

10 percent of administrative positions in education Administrators salaries average 30 percent more than teachers salaries

2 The local Teachers Association is 89 percent women mostly under 40 In a heated debate on television a member of the National Organization of Women (NOW) and the chair of the Teachers Association threatened if a man is hired to bring a class-action suit against the district on behalf of all women teachers who cannot expect advancement because of discriminatory hiring practices

3 The local Lions Club which contributes heavily to school sports says hiring the less experienced woman would not be in the best interests of the schoolthe children or the teachers

The finalists for the position

1 Carole Gates Classroom teacher 10 years Teacher of the Year 1985 supervisor ofpractice teachers at Teachers College former president of Teachers Associ ati on Administrative Credential 1984 EdD degree 1986 assistant principal of Hoptown Elementary School 2 years

2 Spud Stonewall Principal of Middletown Elementary 15 years PhD in educational adminis~ration State Board of Education Committee for Improving Elementary School Curriculum 1982-present

3 Jim Henderson School Administrator 22 yearsgradesK-9-supports innovation in education Fair Bargaining Award 1981 former coach for winning collegiate basketball team 10 years

It is clear from such a prompt that the problem the test-takers must address is complex that there is sufficient information here for writing an essay and that testshytakers will be reasonably challenged to write a clear statement of opinion What also emerges from this prompt (and virtually any prompt that one might propose) is the potential cultural effect on the numerous international students who must take the GIIT Is it possible that such students who are not familiar with school systems in the United States with hiring procedures and perhaps with the politics of school board elections might be at a disadvantage in mounting their arguments within a two-hour time frame Some (such as Hosoya 2001) have strongly claimed such a bias

78 CHAPTER 4 Standardized Testing

4 Make appropriate evaluations of different kinds of items

In Chapter 3 the concepts of item facility (IF) item discrimination (ID) and disshytractor analysis were introduced As the discussion there showed such calculations provide useful infornlation for classroom tests but sometimes the time and effort involved in perfornling them may not be practical especially if the classroom-based test is a one-time test Yet for a standardized multiple-choice test that is designed to be marketed commercially andor administered a number of times andor adminisshytered in a different form these indices are a must

For other types of response formats namely production responses different forms of evaluation become importantThe principles of p-mpoundti~ality ~d poundabWty are prominent along with the concept o(JacjJjt Practicality issues in such items include the clarity of directions timing of the test ease of administration and how much time is required to score responses Reliability is a major player in instances where more than one scorer is employed and to a lesser extent when a single scorer has to evaluate tests over long spans of time that could lead to deterioration of stanshydards Facility is also a key to the validity and success of an item type ~irecshytions complex- language obscure topics fuz~Qata and culturally biased

~Jfiformatioifma~alliead to a highei1eVermiddotof diffiCidty than one desires (A) The IF ID and efficiency statistics of the multiple-choice items of current

forms of the TOEFL are not publicly available information For reasons of security and protection of patented copyrighted materials they must remain behind the closed doors of the ETS development staff Those statistics remain of paramount importance in the ongoing production ofTOEFL items and forms and are the founshydation stones for demonstrating the equatability of forms Statistical indices on retired forms of the TOEFL are available on request for research purposes

The essay portion of theTOEFL undergoes scrutiny for its practicality reliability and facility Special attention is given to reliabilIty since two human scorers must read each essay and every time a third reader becomes necessary (when the two readers disagree by more than one point) it costs ETS more money

(B) In the case of the open-ended responses on the two written tasks on the ESLPT a similar set of judgments must be made Some evaluative impressions of the effectiveness of prompts and passages are gained from informal student and scorer feedback In the developmental stage of the newly revised ESLPT both types of feedshyback were formally solicited through questiQnnaires and interviews That informashytion proved to be invaluable in the revisIon of prompts and stimulus reading passages After each administration now the teacher-scorers provide informal feedshyback on their perceptions of the effectiveness of the prompts and readings

The multiple-choice editing passage showed the value of statistical findings in determining the usefulness of items and pointing administrators toward revisions Following is a sample of the format used

CHAPTER 4- Standardized Testing 79

Multiple-choice editing passage

(1)EYer since supermarkets first appeared they have beentake over ~ world ABC 0

(2) Supermarkets have changed peoples life ~ yet and at the same time changes in ABC

peoples life ~ have encouraged the opening of supermarkets o

The task was to locate the error in each sentence Statistical tests on the experishymental version of this section revealed that a number of the 45 items were found to be of zero IF (no difficulty whatsoever) and of inconsequential discrimination power (some IDs of 15 and lower) Many distractors were of no consequence because they lured no one Such information led to a revision of numerous it~ms and their options eventually strengthening the effectiveness of this section

(C)The GET like its written counterparts in the ESLPT is a test ofwritten ability with a single prompt and therefore questions of practicality and J~~illy~are also largely observational No data are collected from students on their perceptions but the scorers have an opportunity to reflect on the validity ofa given topiC After one sitting a topic is retired which eliininates the possibility of improving a specific topiC but future framing of topics might benefit from scorers evaluations Inter-rater reliability is checked periodically and reader training sessions are modified if too many instances of unreliability appear

5 Specify scoring procedures and reporting formats - ---

A systematic assembly of test items in pre-selected arrangements and sequences all of which are validated to confo~ to an e~pected difficulty level should yield a test that can then be scored accurately and reported back to test-takers and institutions efficiently

(A) Of the three tests being exemplifled here the most straightforward scoring procedure comes from the TO~FL the one with the most complex issues of validashytion deSign and assembly Scores are calculated and reported fora) three sections of the TOEFL (the essay ratings are combined with the Structure and Written Expression score) and (b) a total score (range 40 to 300 on the computer-based TOEFL and 310 to 677 on the paper-and-pencil TOEFL) A separate score (c) for the Essay (range 0 to 6) is also provided on the examinees score record (see simulation of a score record on page 80)

80 CHAPTER 4 Standardized Testing

Facsimile of a TOEFLreg score report

TOEFL Scaled Scores Claudia Y Estudiante Peru ___

19 17 17 177 Listening Structure Writing Reading Total Score

Essay rati ng 30

The rating scale for the essay is virtually the same one that is used for the Test of Written English (see Chapter 9 for details) with a zero level added for no response copying the topic only writing completely off topic or not writing in English

(B) The ESLPT reports a score for each of themiddot essay sections but the rating scale differs between them because in one case the objective is to write a summary and in the other to write a response to a reading ~ch essayi~pd lgtY ~o readet~ ifhFfF js a discrepancy of more than one level a third reader1resolves the differenceThe ~ditiilg section is machine-scanned and -scored with a total score and ~th part-scores for each ofthe grammaticaVrhetorlcal sectionS From these data placement administrators have adequate information to make placements and teachers receive some diagnostic inforshymation on each student in their classes Students do not receive their essays back

(C) Each GET is read by two trained readers who give a score between 1 and 4 according to the following scale

Graduate Essay Test Scoring Guide

Please make no marks on the writers work Write your reader number and score on the front cover of each test booklet

4 Superior The opening establishes context purpose and point of view the body of the essay developsmiddot recommendations-logically and coherently The writer demonshystrates awareness of the complexities in the situation and provides analysis of the probJem offers compelling or common-sense reasons for recommendations made makes underlying assumptions explicit

The writer uses fluent and idiomatic English with few mechanical errors Style reshyveals syntactic maturity is dear and direct is not choppy or over-colloquial nor over-formal stuffy or unfocused Occasional spelling or punctuation errors may be easily attributed to hasty transcription under pressure

3 Competent After an opening that establishes context and purpose the paper unfolds with few lapses in coherence but may have somewhat less clear organization of less explicit transitions than a top-score paper It may have somewhat less compelling logic or slightly less-wellreasoned suggestions than a 4 paper though it will provide reasons for the recommendations made

The writer uses dear fluent and generally idiomatic English but may make minor or infrequent ESL errors (preposition errors dropped articles or verb endings etc) or repeat a single error (eg not punctuate possessive nouns) Occasional lapses of style are offSet by demonstrated mastery of syntax

CHAPTE84 Standardized Testing 81

2 Weak The writer makes somewhat simplistic suggestions not fully supported with reashysons fails to cite key facts offers little analysis of the problem or shows a limited grasp of the situation the given information is copied or listed withlittle integration into argument Points may be random or repetitious Writing may be badly focused with careless use of abstract language resulting in predication errors or illogical sentences

ESL andlor careless mechanical errors are frequent enough to be distracting OR sentences may be choppy style over-casual usage occasionally unidiomatic

1 Inadequate The essay may be disjointed incoherent or minimally developed The writer shows little grasp of the complex issues involved is unable to establish conshytext point of view or purpose in opening of paper or has a poor sense of audience Mechanical andor ESL errors or unidiomatic usages are frequent sentences may be ungrammatical OR correct but short and very simple

The two readers scores are added to yield a total possible score of 2 to 8 Test administrators recommend a score of 6 as the threshold for allowing a student to pursue graduate-level courses Anything below that is accompanied by a recomshymendation that the student either repeat the test or take a remedial course in gradshyuate writing offered in one of several different departments Students receive neither their essays nor any feedback other than the fmal score

6 Perform ongoing construct validation studies

From the above discussion it should be clear that no standardized instrument is expected to be used repeatedly without a ramporou~program of ongoing c~-sectmct valiltiatiOll Any standardized test once developed must be accompanied by sysshy~

tematic periodic corroboration of its effectiveness and by steps toward its improveshyment This rigor is especially true of tests that are produced in equated forms that is forms must be reliable across tests such that a score on a subsequent form of a test-has-the~same validityand-interpretability as its original

(A) The TOEFL program in cooperation with other tests produced by ETS has an impressive program of research Over the years dozens of TOEFL-sponsored research studies have appeared in the TOEFL Monograph Series An early example ofsuch a study was the seminal Duran et aI (1985) study TOEFLfrom a Communicative ViewpOint on Language Proficiency which examined the content characteristics of the TOEFL from a communicative perspective based on current research in applied linguistics and lanshyguage proficiency assessment More recent studies (such as Ginther 2001 Leacock amp Chodorow 2001 Powers et aI 2002) demonstrate an impressive array of scrutiny

(B) For approximately 20 years the ESLPT appeared to be placing students relishyably by means of an essay and a multiple-choice grammar and vocabulary test Over the years the security of the latter became s1lspect and the faculty administrators wished to see some content validity achieved in the process In the year 2000 that process began with a group of graduate students (Imao et aI 2000) in consl1ltation with faculty members and continued to fruition in the form of a new ESLPT reported in lmao (2002) The development of the new ESlPT involved a lengthy process of

82 CHAPTER 4 Standardized Testing

both content and construct validation along with facing such practical issues as scoring the written sections and a machine scorable multiple-choice answer sheet

The process of ongoing validation will no doubt continue as new forms of the editing section are created and as new prompts and reading passages are created for the writing section Such a validation process should also include consistent checks on placement accuracy and on face validity

(C) At this time there is little or no research to validate the GET itself For its conshy struct validation its administrators rely on a stockpile of research on university-level academic writing tests such as theTWEThe holistic scoring rubric and the topics and administrative conditions of the GET are to some extent patterned after that of the TWE In recent years some criticism of the GEf has come from international test-takers (Hosoya 2001) who posit that the topics and time limits of the GET among other facshytors work to the disadvantage of writers whose native language is not English These validity issues remain to be fully addressed in a comprehensive research study

I I

STANDARDIZED IANGUAGE PROFICIENCY TESTING

Tests of language profiCiency presuppose a comprehensive definition of the specific competencies that comprise overall language ability The specifications for the TOEFL provided an illustration of an operational definition of ability for assessment purposes This is not the only way to conceptualize the concept Swain (1990) offered a multidimensional view of profiCiency assessment by referring to three linshyguistic traits (grammar discourse and sociolinguistics) that can be assessed by means of oral multiple-choice and written responses (see Table 41) Swains conshyception was not meant to be an exhaustive analysis of ability but rather to serve as an operational framework for constructing proficiency assessments

Another defmition and conceptualization of profiCiency is suggested by the ACTFL association mentioned earlier ACfFL takes a holistic and more unitary view of proficiency in describing four levels superior advanced intermediate and noviceWithin each level descriptions of listening speaking reading and writing are provided as guidelines for assessment For example the ACfFL Guidelines describe the superior level of speaking as follows

ACTFL speaking guidelines summary superior-level

Superior-level speakers are characterized by the ability to

bull participate fully and effectively in conversations in formal and informal settings on topics related to practical needs and areas of professional andor scholarly interests

bull provide a structured argument to explain and defend opinions and develop effective hypotheses within extended discourse

bull discuss topics concretely and abstractly bull deal with a linguistically unfamiliar situation bull maintain a high degree of linguistic accuracy bull satisfy the linguistic demands of professional andor scholarly life

CHAPTER4 Standardized Testing 83

The other three ACfFL levels use the same parameters in describing progressively lower proficiencies across all four skills Such taxonomie~ have the advantage of considering a number of functions of linguistic discourse but the disadvantage at the lower levels of overly emphasizing test-takers deficiencies

Table 41 Traits of second language proficiency (Swain 1990 p 403)

Trait Grammar Discourse Sociolinguistic

focus on grammatical focus on textual focus on social accuracy within cohesion and appropriateness of sentences coherence language use

Method

Oral structured interview story telling and argumentationpersuasion

role-play ofspeech acts requests offers complaints

scored for accuracy of verbal morphology prepositions syntax

detailed rating for identification logical sequence and time orientation and global ratings for coherence

scored for ability to distinguish formal and informal register

Multiple-choice

sentence-level select the correct form exercise

paragraph-level select the coherent sentence exercise

speech act-Ievelselect the appropriate utterance exercise

(45 items) (29 items) (28 items)

involving verb morphology prepositionsan-d-uther items

Written composition

narrative and letter of persuasion

narrative and letter of persuasion

formal request letter and informal note

scored for accuracy of verb morphology prepositions syntax

detailed ratings much as for oral discourse and global rating for coherence

scored for the ability to distinguish formal and inforJ1lil1 register

FOUR STANDARDIZED lANGUAGE PROFICIENCY TESTS

We now tum to some of the better-known standardized tests of overall language ability or profiCiency to examine some of the typical formats used in commercially available tests We will not look at standardized tests of other specific skills here but that should not lead you to think by any means that proficiency is the only kind of test in the field that is standardized Three standardized oral production tests the

84 CHAPTER 4 Standardized Testing

Test of Spoken English (fSE) the Oral Proficiency Inventory (OPI) and PbonePassreg are discussed in Chapter 7 and the Test of Written English (WE) is covered in ChapterS

Four commercially produced standardized tests of English language proficiency are described briefly in this section the TOEFL the Michigan English Language Assessment Battery (MELAB) the International English Language Testing System (lELTS) and the Test of English for International Communication (fOEICreg) In an appendix to this chapter are sample items from each section of each test When you turn to that appendix use the following questions to help you evaluate these four tests and their subsections

1 What item types are included 2 How practical and reliable does each subsection of each test appear to be 3 Do the item types and tasks appropriately represent a conceptualizatio~ of

language proficiency (ability) That is can you evaluate their construct validity

4 Do the tasks achieve face validity 5 Are the tasks authentic 6 Is there some washback potential in the tasks

Test of English as a Foreign Language (TOEFL)

Producer Educational Testing Service (ETS) Objective To test overall proficiency (language ability) Primary market Almost exclusively US universities and colleges for admission

purposes Type Computer-based (CB) (and two sections are-computer-adaptive)

A traditional paper-based (PB) version is also available Response modes Multiple-choice responses essay Specifications See the box on pp 72-73 Time allocation Up to 4 hours (CB) 3 hours (PB) Internet access wwwtoeflorg

Comments In the North American context the TOEFL is the most widely used comshymercially available standardized test of proficiency Each year the TOEFL test is adminisshytered to approximately 800000 candidates in more than 200 countries It is highly respected because of the thorough program of ongoing research and development conshyducted by ETS The TOEFLs primary use is to set proficiency standards for international students seeking admission to English-speaking universities More than 4200 academic institutions government agencies scholarship programs and licensingcertification agenshycies in more than 80 countries use TOEFL scores By 2004 the TOEFL will include a secshytion on oral production

CHAPTER 4 Standardi~ed Testing 85

Michigan English Language Assessment Battery (MELAB)

Producer English language Institute University of Michigan Objective To test overall proficiency (language ability) Primary market Mostly US and Canadian language programs and colleges

some worldwide educational settings as well Type Paper-based Response modes Multiple-choice responses essay Time allocation 25 to 35 hours Internet access wwwlsaumicheduelimelabhtm

Specifications The MElAB consists of three sections Part 1 a 3D-minute impromptu essay is written on an assigned topic Part 2 a 25-minute multiple-choice listening comshyprehension test is delivered via tape recorder Part 3 is a 100-item 75-minute multipleshychoice test containing grammar doze reading vocabulary and reading comprehension An oral interview (speaking test) is optional

Comments The Ell at the University of Michigan has been producing the MELAB and its earlier incarnation (Michigan Test of English language Proficiency) since 1961 like the TOEFL it serves a North American audience but is also used internationally While its use is not as widespread as the TOEFL its validity is widely respected Because it is cheaper than the TOEFL and more easily obtained it is popular among language schools and institutes Many institutions and companies accept MElAB scores in lieu ofTOEFL scores

International English Language Testing System (IELTS)

Producer Jointly managed by The University of Cambridge local Examinations Syndicate (UClES) The British Council and lOP Education Australia

Objective To test overall proficiency (language ability) Primary-market Australian British Canadian and New Zealand academic

institutions and professional organizations American academic institutions are increasingly accepting IELTS for admissions purposes

1)rpe Computer-based (for the Reading and Writing sections) papershybased for the listening and Speaking modules

Response modes Multiple-choice responses essay oral production Time allocation 2 hours 45 minutes Internet access httpwwwieltsorgl

httpwwwudesorguk httpwwwbritishcouncilorg

Specifications Reading candidates choose between academic reading or general training reading (60 minutes) Writing the same option academic writing or general training writing (60 minutes) Listening four sections for all candidates (30 minutes) Speaking five sections for all candidates (1015 minutes)

86 CHAPTER 4 Standardized Testing

Comments The University of Cambridge local Examinations Syndicate (UCLES) has been producing English language tests since 1858 Now with three organizations cooperatshying to form the IELTS more than a million examinations are administered every year In 2002 a computer-based version of the Reading and Writing modules of the IELTS became available at selected centers around the world The other sections are administered locally by an examinet The paper-based IELTS remains an option for candidates The IELTS retains the distinct advantage of requiring all four skills in the test-takers performance

Test of English for International Communication (TOEICreg)

Producer The Chauncey Group International a subsidiary of Educational Testing Service

Objelttive To test overall proficiency (langlJage ability) Primary market Worldwide business commerce and industry contexts

(workplace settings) Type Computer-based and paper-based versions Response modes Multiple-choice responses Time allocation 2 hours Internet access httpwwwtoeiccom

Specifications Listening Comprehension 100 items administered by audiocassette Four types of task statements questions short conversations and short talks (approxishymately 45 minutes) Reading 100 items Three types of task cloze sentences error recogshynition and reading comprehension (75 minutes)

Comments The TOEIC has become a very widely used international test of English proficiency in workplace settings where English is required for job performance The conshytent includes many different employment settings such as conferences presentations sales ordering shipping schedules reservations (etters and memoranda It is approprishyate to use in educational settings where vocational or workplace English courses are being offered

sect sect sect sect sect

The construction of a valid standardized test is no minor accomplishment whether the instrument is large- or small-scale The designing of specifications alone as this chapter illustrates requires a sophisticated process of construct valishydation coupled with considerations of practicality Then the construction of items and scoringinterpretation procedures may require a lengthy period of trial and error with prototypes of the final form of the testWith painstaking attention to all the details of construction the end product can result in a cost-effective timeshysaving accurate instrument Your use of the results of such assessments can provide useful data on learners language abilities But your caution is warranted as well for all the reasons discussed in this chapter The next chapter will elaborate on what lies behind that need for a cautious approach to standardized assessment

CHAPTER4 Standardized Testing 87

EXERCISES

[Note (I) Individual work (G) Group or pair work (C) Whole-class discussion]

1 (C) Tell the class about the worst test experience youve ever had Briefly anamiddot lyze what made the experience so unbearable and try to come up with sugshygestions for improvement of the test andor its administrative conditions

2 (G) In pairs or small groups compile a brief list of pros and cons of standardshyized testing Cite illustrations of as many items in each list as possible Report your lists and examples to the rest o~ the class

3 (I) Select a standardized test that you are quite familiar with (probably a recent experience) Mentally evaluate that test using the five principles of practicality reliability validity authenticity and washback Report yourevaluashytion to the class

4 (G) The appendix to this chapter provides sample items from Jour different tests of language proficiency In groups one test for each group analyze your test for (a) content validity (b) face validity and (c) authenticity

5 (C) Do you think that the sample TOEFL reading passage about pirates (pages 74-75) and the Graduate EssayTest prompt (pages 76-77) about a school board hiring committee have any culture bias Discuss this and other cultural biases you have noticed in tests Is it possible to design a test that is completely free of culture bias

6 (CG) Compare the differences in conceptualization of language proficiency represented by Swains model the TOEFL and the ACfFL philosophy Which one best represents current thinking about communicative language ability What are the strengths and weaknesses of each approach

FORYOlIILEURTHER READING

Gronlund Norman E (1998) Assessment of student achievement Sixth Edition Boston Allyn and Bacon

Gronlunds classic also mentioned in Chapter 3 offers a concise overview of features of standardized tests offering definitions and examples of the statistical considerations in interpreting scores His approach is unbiased cleady written and accessible to those who might fear the mathematics of standardized testing

Phillips Deborah 2001 Long1nan introductory course for the TOEFL test White Plains NY Pearson Education

A careful examination of this or any other reputable preparation course for a standardized language test is well worth a students time Note especially how the book acquaints the user with the specifications of the test and offers a number of useful strategie~ that can be llsed in preparation for the test and during irs adn1inistration

88 CHAPTER 4 Standardized Testing

APPENDIX TO CHAPTER 4

Commercial Proficiency Tests Sample Items and Tasks

Test of English a~ a Foreign Language (TOEFLreg)

Listening r

Part A

In this section you will hear short conversations between two people In some ofthe conversations each person speaks only once In other conversations one or both of the people speak more than once Each conversation is followed by one questionabQlt it Each question in this part has four answer choices You should click on the best answer to each question Answer the questions on the basis of what is stated or implied by the speakers Here is an example On the computerscreen you will see

[man and woman talking]

On the recording you will hear

(woman) Hey wheres your sociology book (man) At home Why carry it around when were just going to be taking

a test (woman) Dont you remember Professor Smith said we could us it during

the test (man) Ohl no Well Ive still got an hour right Im so glad I ran into you

You wiII then see and hear the question before the answer choices appear

What will the man probably do next

o Begin studying for the sociology test o Explain the problem to his professor o Go home to get his textbook o Borrow the womans book

To choose an answer you will click on an oval The oval next to that answer will darken After you click on Next and Confirm Answer the next conversation will be presented

Part B

In this section you will hear several longer conversations and talks Each conversation or talk is followed by several questions The conversations talks and questions will not be repeated The conversations and talks are about a variety of topics You do not need speshycial knowledge of the topics to answer the questions correctly Rather you should answer each question on the basis of what is stated or implied by the speakers in the conversashytions or talks

For most of the questions you will need to click on the best of four possible answers Some questions will have special directions The special directions will appear in a box on the computer screen Here is an exampie ot a conversation and some questions

CHAPTER 4 Standardized Testing 89

Marine Biology (narrator) Listen to part of a discussion in a marine biology class

(professor) A few years ago our local government passed a number of strict environmental laws As a result Sunrise Beach looks nothing Ii ke it did ten years ago The water is cleaner and theres been a tremendous increase in all kinds of marine life which is why were going there on Thursday

(woman) I dont know if I agree that the water quality has improved I mean I was out there last weekend and it looked all brown It didnt seem too clean to me

(professor) Actually the color of the water doesnt always indicate whether its polluted The brown color you mentioned might be a result of pollution or it can mean a kind of brown algae is growing there Its called devils apron and it actually serves as food for whales

(man) So when does the water look blue (professor) Well water thats completely unpolluted is actually colorless But

it often looks bluish-green because the sunlight can penetrate deep down and thats the color thats reflected

(woman) But sometimes it looks really green Whats that about (professor) Ok well its the same principle as with devils apron the

water might be green because of different types of green algae there-gulfweed phytoplankton You all should finish reading about algae and plankton before we go In fact those are the types of living things Im going to ask you to be looking for when were there

Now get ready to answer the questions

What is the discussion mainly about

o The importance of protecting ocean environments o The reasons why ocean water appears to be different colors o The survival of whales in polluted water o The effect that colored ocean water has on algae

To choose an answer click on an oval The oval next to that answer will darken After you click on Next and Confirm Answer the next question will be presented

According to the professor what can make ocean water look browngt

o Pollution o Cloudy Skies o Sand o Algae

Click on 2 answers

To choose your answers you will click on the squares An XII wiii appear in each square

bullbullbullbullbullbullbull

90 CHAPTER 4 Standardized Testing

Structure and Written Expression This section measures the ability to recognize language that is appropriate for standard written English There are two types ofquestions in this section In the first type ofquestion there are incomplete sentences Beneath each sentence there are four words or phrases

Directions CIiSk on the one word or phrase that best completes the sentence

The colum~ine flower __ to nearly all of the United States can be raised from seed in almost any garden

native how native is how native is it is native

Time Help Confirm

After you click on Next and Confirm Answ~ the next question willbe presented

The second type of question has four underlined words or phrases You will choose the one underlined word or phrase that must be changed for the sentence to be correct

Directions Click on the one underlined word or phrase that must be changed for the senshytence to be correct

One of the most difficult problems in understanding sleep is determining what the funcshytions of sleep ~

lime Help Confirm

Clicking on an underlined word or phrase will darken it

Reading This section measures the ability to read and understand short passages similar in topic and style to those that students are likely to encounter in North American universities and colleges This section contains reading passages and questions about the passages There are several different types of questions in this section In the Reading section you will first have the opportunity to read the passage

The temperature of the Sun is over 10000 degrees Fahrenheit at the surface but it rises perhaps more than 270000000 at the center The Sun is so much hotter than the Earth that matter can exist only as a gasl except perhaps at the core In the core of the Sun the pressures are so great that despite the high temperature there may be a small solid core However no one really knows since the center of the Sun can never be directly observed ~ Solar astronomers do know that the Sun is divided into five general layers or zones Starting at the outside and going down into the Sun the zones are the corona chromoshysphere hotosphere convection zone and finally the core The first three zones are reshygarded as the Suns atmosphere But since the Sun has no solid surface it is hard to middottell where the atmosphere ends and the main body of the Sun begins

The Suns outermost layer begins about 10000 miles above the visible surface and goes outward for millions of miles This is the only part of the Sun that can be seen during an eclipse such as the one in February 1979 At any other time the corona can be seen

bullbullbullbullbullbullbull

bull bullbullbullbullbullbull

CHAPTER 4 Standardized Testing 91

only when special instruments are used on cameras and telescopes to block the light from the photosphere

The corona is a brilliant pearly white filmy light about as bright as the full Moon Its beautiful rays are a sensational sight during an eclipse The coronas rays flash out in a brilliant fan that has wispy spikelike rays near the Suns north and south poles The corona is generally thickest at the Suns equator The corona is made up of gases streamshying outward at tremendous speeds that reach a temperature of more than 2 million deshygrees Fahrenheit The gas thins out as it reaches the space around the planets By the time the gas of the corona reaches the Earth it has a relatively low density

When you have finished reading the passage you will use the mouse to click on Proceed Then the questions about the passage will be presented You are to choose the one best anshyswer to each question Answer all questions about the information in a passage on the basis ofwhat is stated or implied in that passage Most ofthe questions will be multiple-choice questions To answer these questions you will click on a choice below the question

With what topic is paragraph 2 mainly concerned

o How the Sun evolved o The structure of the Sun o Why scientists study the Sun o The distaflce of the Sun from the planets

Paragraph 2 is marked with an arrow (~)

You will see the next question after you click on Next

To answer some questions you will click on a word or phrase Here is an example

Look at the word one in the passage Click on the word or phrase in the bold text that one refers to To answer you can click on any part of the word or phrase in the passage Jour choice will darken to show which word you have chosen

The Suns outermost layer begins about 10000 miles above the visible surface and goes outward for millions of miles This is the only part of the Sun that can be seen durshying an eclipse such as the one in February 1979 At any other time the corona can be seen only when special instruments are used on cameras and telescopes to block the Iight from the photosphere

You will see the next question after you click on~ To answer some q~estions you will click on a sentence in the passage Here is an example

~ The corona is a brilliant pearly white filmy light about as bright as the full Moon Its beautiful rays are a sensational sight during an eclipse The coronas rays flash out in a brilliant fan that has wispy spikelike rays near the Suns north and south poles The corona is generally thickest at the Suns equator ~ The corona is made up of gases streaming outward at tremendous speeds that reach a temperature of more than 2 million degrees Fahrenheit The gas thins out as it reaches the space around the planets By the time the gas of the corona reaches the Earth it has a relatively low density

bull bullbullbullbullbullbull

92 CHAPTER 4 Standardized Testing

Click on the sentence in paragraph 4 or 5 in which the author compares the light of the Suns outermost layer to that ofanother astronomical body Paragraphs 4 and 5 are marked with arrows (~)

To answer some questions you will click on a square to add a sentence to the passage Here is an example -The following sentence can be added to paragraph 1

At the center of the Earths solar system lies the Sun

Where would it best fit in paragraph I Click on the square to add the sentence to the paragraph

D The temperature of the Sun is over 10000 degrees Fahrenheit at the surface but it rises to perhaps morethan 27000000deg at the center 0 The Sun is so much hotter than the Earth that matter can exist only as a gasi except p~rHapsatth~ c6relp the c~re of the ii Sun the pressures are so great that despite the high temperature there may be a small solid core D However no one really knows since the center of the Sun can never be directly observed D 0100

When you click on a square the sentence will appear in the passage at the place you have chosen You can read the sentence added to the paragraph to see if this is the best place to add it You can click on another square to change your answer The sentence will be added and shown in a dark box

Writing In this section you will have an opportunity to demonstrate your ability to write in Enshyglish This includes the ability to generate and organize ideas to support those ideas with examples or evidence and to compose in standard written English in response to an asshysigned topic You will have 30 minutes to write your essay on that topic You must write on the topic you are assigned An essay on any other topic will receive a score of 0 Read the topic below and then make any notes that will help you plan your response Begin typing your response in the box at the bottom of the screen or write your answer on the answer sheet provided to you

Following is a sample topic

Do you agree or disagree with the following statemenH

Teachers should make learning enjoyable and fun for their students

Use specific reasons and examples to support your opinion

CHAPTER 4 Standarczed Testing 93

Michigan English Language Assessment Battery (MELAB)

Composition The time limit for the composition is 30 minutes You must write on only one of the top~

ics below If you write about something else your composition paper will not be graded and you cannot be given a final score If you do not understand the topics ask the exam~ iner to explain or to translate them You may be asked to give your opinion ofsomething and explain why you believe this to describe something from your experience or to exshyplain a problem and offer possible solutions You should write at least one page Some sample topics are

1 What do you think is your countrys greatest problem Explain in detail and tell what you think can be done about it

2 What are the characteristics of a good teacher Explain and give examples 3 An optimist is someone who sees the good side of things A pessimist sees the

bad side Are you an optimist or a pessimist Relate a personal experience that shows this

4 In your opinion are the benefits of space exploration really worth the enormous costs Discuss

Most MELAB compositions are one or two pages long (about 200-300 words) If your paper is extremely short (less than 150 words) your composition will be given a lower score Before you begin writing you might want to take 2 or 3 minutes to plan your comshyposition and to make a short outline to organize your thoughts Such outlines will not be graded they are only to help you You should use the last 5 minutes to read through your composition and to make changes or corrections

Your composition will be graded on how clearly you express yourself in English and on the range of English you are able to use and your control in doing so This means your composition should be well organized your arguments should be fully developed and you should show a range ofgrammatical structures and broad vocabulary Compositions that consist only of very short sentences and very simple vocabulary cannot be given the

middothighest scores If errors are not frequent and if they do not confuse your meaning they will not lower your score very much

Listening Now you will hear a short lecture You may take notes during the lecture Following the lecture you will be asked some questions about it

Therell be a two-week exhibit of the paintings of the little-known master Laura Bernhart at the Claire Osmond Galleries starting on the fifteenth of the month and running through the thirtieth Bernharts known for her innovative designs in abstract expressionism Though a true original she declared a spiritual heritage from Salvador Dali the famous Spanish painter Since Bernhart lived a rather solitary life and died while only in her twenties few people are aware of her works This showing at the Osmond Galleries will provide many with an introduction to her works

10 Where is the exhibit a the Art Museum b the Dali Galleries c the Osmond Galleries

94 CHAPTER 4 Standardized Testing

11 What is Bernhart known for a her copies of Dalis paintings b the originality of her designs c her exhibitions

12 What will going to the exhibit allow most people to do a to see Saivador Dalis paintings b to see Bernharts works for the first time c to learn about Spanish art

Grammar

1 What did the teacher just tell you

She reminded our notebooksI a us to bring b that we bring c our bringing d we should bring

2 Is Bill a good dancer

Not really __ he tries very hard a in spite of h despite c even though d while

3 your clothes are all wet1

Yes I didnt come __ the rain soon enough a away to b over to c down with d in from

Cloze In years to come zoos will not only be places where animals are exhibited to the public but repositories where rare species can be saved from extinction (7) captive breeding The most powerful force (8) the future of many animals-and of zoos-is the decline of the wild (9) even zoo directors would argue that (10) are better places for animals than the fields and forest of their native (11) yet zoos may be the last chance for some creatures that would otherwise pass qUietly into oblivion

7 a through c from b of d damage

8 a bringing c to b that d influencing

9 a But c Not b So d Then

10 a where c even b zoos d wilds

11 alands c residence b life d field

CHAPTER 4 Standardized Testing 95

Vocabulary

12 Mark has a flair for writing a need b purpose c talent d dislike

13 Bill Collins launched his restaurant last June a moved b started c sold d bought

14 John will not accept the censure a burden b blame c credit d decision

15 I cant think of the answer Can you give me a __ a hint b token c taste d gaze

16 Because fewer people are taking expensive vacations the tourist industry is in a a choke b grope c grumble d slump

17 I disagree with a few of his opinions but __ we agree a deliberately b conclusively c essentially d immensely

Reading The influenza virus is a single molecule built from many millions of single atoms You must have heard of the viruses which are sometimes called living molecules While bacteria can be considered as a type of plant secreting pOisonous substances into the body of the organism they attack viruses are living organisms themselves We may conshysider them as regular chemical molecules since they have a strictly aefined atomic strucshyture but on the other hand we must also consider them as being alive since they are able to multiply in unlimited quantities

18 According to the passage bacteria are a poisons

b larger than viruses c very small d plants

96 CHAPTER 4 Standardized Testing

19 The writer says that viruses are alive because they a have a complex atomic structure b move c multiply d need warmth and light

20 The atomic structure of viruses a is -tJIariable b is strictly defined c cannot be analyzed chemically d is more complex than that of bacteria

International English Language Testing System (fELTS)

I

listening

The Listening Module has four sections The first two sections are concerned with social needs There is a conversation between two speakers and then a monologue For examshyple a conversation about travel arrangements or decisions on a night out and a speech about student services on a university campus or arrangements for meals during a confershyence The final two sections are concerned with situations related more closely to educashytional or training contexts For example conversation between a tutor and a student about an assignment or between three students planning a research project and a lecture or talk ofgeneral academic interest All the topics are ofgeneral interest and it makes no difference what subjects candidates study Tests and tasks become more difficult as the sections progress A range of English accents and dialects are used in the recording which reflects the international usage of IELTS

Academic Reading [A 7S0-word article on-th-e- topic of Wind Power in the US with a short glossary at the end]

Questions 1-5

Complete the summary below

Choose your answers from the box below the summary and write them in boxes 1-5 on your answer sheet Note There are more words or phrases than you will need to fill the gaps You may use any word or phrase more than once

Example The failure during the late 1970s and early 19805 of an attempt to establish a widespread wind power industry in the United States resulted largely from the (1) bull in oil prices during this period The industry is now experiencing a steady (2) due to improveshyments in technology and an increased awareness of the potential in the power of wind The wind turbines that are now being made based in part on the (3) of wide- ranging research in Europe are easier to manufacture and maintain than their predecesshysors This has led wind-turbine makers to be able to standardise and thus minimize (4) There has been growing (S) of the importance of wind power as an energy source

CHAPTER 4 Standardized Testing 97

criticism stability skepticism success operating costs decisions design costs fall effects production costs growth decline failure recognition results

Questions 6-1 0 Look at the following list of issues (Questions 6-10) and implications (A-C) Match each issue with one implication Write the appropriate letters A-C in boxes 6-10 on your anshyswer sheet

Example The current price of one wind-generated kilowatt Answer

6 The recent installation of systems taking advantage of economies of scale

7 The potential of meeting one fifth of current U5 energy requirements by wind power

8 The level of acceptance of current wind turbine technology

9 A comparison of costs between conventional and wind power sources

10 The view of wind power in the European Union

Implications

A provides evidence against claims that electricity produced from wind power is relatively expensive

B supports claims that wind power js an important source of energy

C opposes the view that wind power technology requires further-development

General Training Reading Read the passage on Daybreak trips by coach and look at the statements below On your answer sheet write

TRUE if the statement is true FALSE jf the statement is false

NOlGIVEN if the information is not given in the leaflet

1 MiIlers Coaches owns Cambridges Cam bus fleet

2 Premier is an older company than Millers

3 Most of the Daybreak coaches are less than 5 years old

4 Daybreak fares are more expensive than most of their competitors

5 Soft drinks and refreshments are served on most longer journeys

6 Smoking is permitted at the rear of the coach on longer journeys

7 Tickets must be bought in advance from an authorised Daybreak agent

6 Tickets and seats can be reserved by phoning the Daybreak Hotline

9 Daybreak passengers must join their coach at Cambridge Drummer Street

10 Daybreak cannot guarantee return times

98 CHAPTER 4 Standardized Testing

FROM CAMBRIDGE AND SURROUNDING AREA

SPRING IS INTHEAIR

Welcome to our Spring Daybreak programme which continues the tradition of offering unbeatable value for money day trips and tours All the excursions in this brochure will be operated by Pr~mier Travel Services Limited or Millers Coaches both companies are part of the CHLGroup owners of Cambridges Cambus fleet

WERE PROUD OF OUR TRADITION

Premier was established in 1936 the Company now offers the highest standards of coaching in todays competitive operating environment Miller has an enviable reputation stretching back over the past 20 years offering coach services at realistic prices Weve traveled a long way since our early days of pre-war seaside trips Now our fleet of 50 modern coaches (few are more than five years old) operate throughout Britain and Europe but were pleased to still maintain the high standards of quality and service the trademark of our founders nearly sixty years ago

EXCLUSIVE FEATURES

Admission-inclusive fares All Daybreak fares (unless specifically otherwise stated) include admission charges to the attractions shows and exhibits we visit Many full-day scenic tours are accompanied by a fully trained English Tourist Board Blue Badge guide or local experienced driverguide Some Daybreaks include lunch or afternoon tea Compare our admission inclusive fares and see how much you save Cheapest is not the best and value for money is guaranteed If you compare our bargain Daybreak fares beware--most of our competishytors do not offer an all-inclusive fare

SEAT RESERVATIONS

We value the freedom of choice so you can choose your seat when you book The seat reservation is guaranteed a-nd remains yours at all times when aboard the coach

NO SMOKING COMFORT

With the comfort of our passengers in mind coaches on all our Daybreaks are no smokshying throughout In the interests of fellow passengers comfort we kindly ask that smokers observe our no smoking policy On scenic tours and longer journeys ample refreshment stops are provided when of course smoking is permitted

YOUR QUESTIONS ANSWERED

Do I need to book Booking in advance is strongly recommended as all Daybreak tours are subject to demand Subject to availability stand-by tickets can be purchased from the driver

What ti me does the coach leave The coach departs from Cambridge Drummer Street (Bay 12 adjacent to public toilets) at the time shown There are many additional joining points indicated by departure codes in the brochure If you are joining at one of our less popular joining points you will be adshyvised of your pick-up time (normally by telephone) not less than 48 hours before deparshyture In this way we can minimize the length of pick-up routes and reduce journey times for the majority of passengers

CHAPTER 4 Standardized Testing 99

What time do we get back An approximate return time is shown for each excursion The tim~s shown serve as a guide but road conditions can sometimes cause delay If your arrival will be later than advertised your driver will try to allow for a telephone call during the return journey

Where can I board the coach All the Daybreaks in the brochure leave from Cambridge Drummer Street (Bay 12 adjashycent to public toilets) at the time shown Many Daybreaks offer additional pick-ups for pre-booked passengers within Cambridge and the surrounding area This facility must be requested at the time of booking

Academic Writing Writing Task 1 You should spend about 20 minutes on this task

The graph below shows the different modes of transport used to travel to and from work in one European city in 1950 1970 and 1990

[graph shown here]

Write a report for a university lecturer describing the information shown below You should write at least 150 words

Writing Task 2 You should spend about 40 minutes on this task

Present a written argument or case to an educated reader with no specialist knowledge of the folowing topic

It is inevitable that as technology develops so traditional cultures must be lost Technolshyogy and tradition are incompatible-you cannot have both together

To what extent do you agree or disagree with this statement Give reasons for your answer You should write at least 250 words You should use your own ideas knowlshyedge and experience and support your arguments with examples and relevant evidence

General Training Writing Writing Task 1 You should spend about 20 minutes on this task You rent a house through an agency The heating system has stopped working You phoned the agency a week ago but it has still not been mended Write a letter to the agency Explain the situation and teil them what you want them to do about it

You should write at least 150 words You do NOT need to write your own address

Begin your letter as follows

Dear - ___-I

Writing Task 2 You should spend about 40 minutes on this task As part ofa class assignment you have to write about the following topic

100 CHAPTER 4 Standardized Testing

Some businesses now say that no one can smoke cigarettes in any of their offices Some governments have banned smoking in all public placesThis is a good idea but it takes away some of our freedom

Do you agree or disagree Give reasons for your answer You should write at least 250 words

Speaking In each ofthe three parts of the speaking module a specific function is fulfilled In Part 1 the candidates answer general questions about themselves their homes or families their jobs or studies their interests and a range ofsimilar familiar topic areas This part lasts between four and five minutes In Part 2 the candidate is given a verbal prompt on a card and is asked to talk on a particular topic The candidate has one minute to prepare before speaking at length for between one and two minutes The examiner then asks one or two wind-down questions In Part 3 the examiner and candidate engage in a discusshysion of more abstract issues and concepts which are thematically linked to the topic prompt in Part 2 The discussion lasts between four and five minutes

All interviews are recorded on audiocassette Here is a sample ofa Part 2 topic

Describe a teacher who has greatly influenced you in your education

You shou Id say

where you met them what subject they taught what was special about them

and explain why this person influenced you so much

You will have to talk about the topic for 1 to 2 minutes You have 1 minute to think about what you are going to say You can make some notes if you wish

Test of English for International Communication (TOEICreg)

listening

Part 1 Photographs Directions For each question you will see a picture in your test book and you will hear four short statements The statements will be spoken just one time They will not be printed in your test book so you must listen carefully to understand what the speaker says When you hear the four statements look at the picture in your test book and choose the statement that best describes what you see in the picture Then on your answer sheet find the number of the question and mark your answer

[photograph of a scientist looking through a microscope]

You will hear Look at the picture marked number 1 in your test book

(A) Shes speaking into a microphone (B) Shes put on her glasses (C) She has both eyes open (D) Shes using a microscope

CHAPTER 4 Standardized Testing 101

Part 2 Question-Response Directions In this part of the test you will hear a question or statement spoken in Enshyglish followed by three responses also spoken in English The question or staten1ent and the responses will be spoken just one time They will not be printed in your test book so you must listen carefully to understand what the speakers say You are to choose the best response to each question or statement

Question 1 You will hear Ms Morikawa has worked here for a long time hasnt she

(A) At three oclock (B) No Ive lost my watch (C) More than ten years

Question 2 You will hear Which of these papers has a wider circulation

(A) The morning edition (B) Get more exercise (C) By messenger

Part 3 Short Conversations Directions In this part of the test you will hear short conversations between two people The conversations will not be printed in your test book You will hear the conversations only once so you must listen carefully to understand what the speakers say In your test book you will read a question about each conversation The question will be followed by four answers You are to choose the best answer to each question and mark it on your answer sheet

Question 1 (Man) We should think about finding another restaurant for lunch (Woman) Why The food and service here are great

(Man) Yes but the prices are going up every week

You will read Why is this man unhappy with the restaurant

(A) It is too noisy (B) It is too expensive (C) It is too crowded (D) It is too difficult to find

Question 2 (Woman A) How was Dr Borgs recent trip to Singapore (Woman B) She enjoyed the tour of the port very much (Woman A) They say its one of the most active in Asia

You will read 2 What did Dr Borg find interesting

(A) The tourist center (B) The airport (C) The musical performance (D) The harbor

Part 4 Short Talks Directions In this part of the test you vill hear several short talks Each will be spoken just one time They will not be printed in your test book so you must listen carefully to understand and remember what is said In your test book you will read two or more questions about each short talk The questions will be followed by four answers You are to choose the best answer to each question and mark it on your answer sheet

102 CHAPTER 4 Standardized Testing

You will hear Questions 1 and 2 refer to the following announcement

Good afternoon and welcome aboard Nordair Flight 857 from Copenhagen to Bangkok with intermediate stops in Dubai and Calcutta We are preparing for departure in a few minutes At this time your seat back should be returned to its full upright position and your seat belt s~ould be fastened OUf anticipated total flying time to Dubai is six hours and twenty-five minutes I hope you enjoy the flight You will hecJr Now read question 1 in your test book and answer it You will read 1 What is the final destination of the flight

(A) Bangkok (B) Copenhagen (C) Dubai (O) Calcutta

You will hear Now read question 2 in your test book and answer it You will read 2 What will happen in a few minutes

(A) The flight will land in Dubai I

(B) The passengers will board the plane (C) The plane will take off (0) The gate number will be announced

Reading In this section of the test you will have the chance to show how well you understand written English There are three parts to this section with special directions for each part

Part 4 Incomplete Sentences Directions This part of the test has incomplete sentences Four words or phrases marked (A) (8) (e) (D) are given beneath each sentence You are to choose the one word or phrase that best completes the sentence Then on your answer sheet find the number of the question and mark your answer

1 Mr Yangs trip will __ him away from the office for ten days (A) withdraw (B) continue (C) retain (0) keep

2 The company that Marie DuBois started now sells __ products throughout the world (A) its (B) it (C) theirs (D) them

3 If your shipment is not delivered __ Tuesday you can request a full refund for the merchandise (A) at (B) by (C) within (D) while

CHAPTER 4 Standardized Testing 103

Part 6 Error Recognition Directions In this part ofthe test each sentence has four words or phrases underlined The four underlined parts of the sentence are marked (A) (B) (C) (D) You are to identify the one underlined word or phrase that should be corrected or rewritten Then on your answer sheet find the number of the question and mark your answer

1 The pamphlet contains some importance information about the current exhibit ABC D

2 No matter how Jong it taking to finish the annual report it must be done properly ABC D

3 The popularity of jogging appears to have decreased since the past couple of years ABC D

Part 7 Reading Comprehension Directions The questions in this part of the test are based on a selection of reading mateshyrials such as notices letters) forms newspaper and magazine articles) and advertisements You are to choose the one best answer (A) (B) (C) or (OJ to each quesshytion Then on your ariswefsheelfindthe number of the qUestion andmcirkyour answer Answer all questions following each reading selection on thebasis of what is stated or implied in that selection

The Museum ofTechnology is a hands-on museum designed for people to experience science at w()rk~ Visitors are encouraged to use test and handle the objects o~ display Special demonstrations are scheduled for the first and second Wednesdays of each month at 1330 Open Tuesday-Friday 1200-1630 Saturday 1000-1730 and Sunday 11 00-1630

1 When during the month can visitors see special demonstrations (A) Every weekend (B) The first two Wednesdays (C) One afternoon a week (D) Every other Wednesday

Questions 2 and 3 refer to the followi ng notice

NOTICE If you are unable to work because of an extended illness or injury that is not workshyrelated you may be entitled to receive weekly benefits from your employer or the firms insurance company To claim benefits you must file a claim form within thirty days of the first day of your disability Before filing the claim you must ask your doctor to fill in the Doctors Statement on the claim form stating the period of disability

3 To whom is this notice addressed (A) Employers (8) Doctors (C) Employees (D) When paying the bill

4 When must the claim form be filed (A) On the first of the month (8) On the thirtieth of the month (C) On the first day ofdisabifity (D) Within 30 days of the start of disability

Page 2: Standardized Testing Chapter 4 Brown

CHAPTER4 Standardized Testing 67

___~--9~in~_YQ9~ith a variety of current standardized tests that claim to test overall language proficiency

It should be clear from these goals that in this chapter we are not focusing celshytrally on classroom-based assessment Dont forget however that stand~diz~d tests affect all classrooms and some of the practical steps that are involved in creating standardized tests are directly transferable to designing classroom tests

~

WHAT IS STANDARDIZATION

A standardized test presupposes certain standard objectives or criteria that are held constant across one form of the test to another The criteria in large-scale standardized tests are designed to apply to a broad band of competencies that are usually not exclusive to one particular curriculumA good standardized test is the product of a thorough process of empirical research and development It dictates standard procedures for administration and scoring And finally it is typical of a norm-referenced test the goal ofwhich is to place test-takers on a continuum across a range of scores and to differentiate test-takers by their relative ranking

Most elementary and secondary schools in the United States have standardized achievement tests to measure childrens mastery of the standards or competencies that have been prescribed for specified grade levels These tests vary by states counties and school districts but they all share the common objective of econOffishyicallarge-scale assessment Collegemiddot entrance exams such as the Scholastic Aptitude Test (SA~ are part of the educational experience of many high school seniors seeking further education The Graduate Record Exam (GRE~ is a required stanshydardized test for entry into many graduate school programs Tests like the Graduate Management Admission Test (GMA1) and the Law School Aptitude Test (LSAn speshycialize in particular diSCiplines One genre of standardized test that you may already be-familiar with is the Test of English as a Foreign Language (fOEFL ~produced by die Educational Testing Service (ETS) in the United States andor its British counshyterpart the International English Language Testing System (lELTS) which features standardized tests in affiliation with the University of Cambridge Local Examinations Syndicate (UCLES) They are all standardized because they specify a set of compeshytencies (or standards) for a given domain and through a process of construct valishydation they program a set of tasks that have been designed to measure those competencies

Many people are under the incorrect impression that all stanoardized tests conshysist of items that have predetermined responses presented1n a multiple-choice format While it is true that many standardized tests conform to a multiple-choice format by no means is multiple-choice a prerequisite characteristic It so happens that a multiple-chOice format provides the test producer with an objective means for determining correct and incorrect responses and therefore is the preferred mode for large-scale tests However standards are equally involved in certain humanshyscored tests of oral production and writing such as the Test of Spoken English (fSE~ and the Test ofWritten English (IWE~ both produced by ETS

68 CHAPTER 4 Standardized Testing

ADVANTAGES AND DISADVANTAGES OF STANDARDIZED TESTS

Advantages of standardized testing include foremost a re~gY-=1ade previously valishydated product that frees the teacher from having to spe~d hours creating a test Administration to large groups can be accomplished within reasonable time limits In the case ofmiddotinultiple-choice formats scoring procedures are streamlined (for either scal18Qble computerized scoring or hand-SCOring with a hole-punched grid) for fast turnaround time And for better or for worse there is often an air of face validity to such authoritative-looking instruments

Disadvantages center largely on the inp-Jl9priate use of such tests for example using an_QY~1llpr9tl~i~tl9_~est as an achievemenftesfsUnplY because of the convenience of the standardization A colleague told me-about a course director who after a frantic search for a last-minute placement test administered a multipleshychoice granunar achievement test even though the curriculum was mostly listening and speaking and involved few of the granunar points tested This instrunrent had the appearance and face validity of a good test when in reality it had no -content Validity whatsoever

Another disadvantage is the potential misunderstanding of the difference between directandindirect testing (see Chapter 2) Some standardized tests include tasks that do__-2QJrectly specify Q~~~~_~ the target r objpound~tive For example before 1996 the TOEFL included neither a written nor an oral production section yet statistics showed a reasonably strong correspondence between performance on the TOEFL and a students written and-to a lesser extent-oral production The comprehension-based TOEFL could therefore be claimed to be an indirect test of production A test of reading comprehension that proposes to me~e ability to read extensively and that engages test-takers in reading only short one or twoshyparagraph passages is anJndirect measure of extensive reading

Those who use standardized tests need to acknowledge both the advantages and limitations of indirect testing In the pre1996 TOEFL administrations the expense of giving a direct test of production was considerably reduced by offering only comprehension performance and showing through construct validation the appropriateness of conclusions about a test-takers production competence likewise short reading passages are easier to administer and if research validates the assumption that short reading passages indicate extensive reading ability then the use of the shorter passages is justified Yet the construct validation statistics that offer that support never offer a 100 percent probability of the relationship leaving room for some possibility that the indirect test is not valid for its targeted use

A more serious isslle lies in the assumption (alluded to above) that standardized tests correctly assess all learners equally well Wellestablished standardized tests usually demonstrate high correlations between performance on such tests and target objectives but correlations are not sufficient to demonstrate unequivocally the acquisition of criterion objectives by all test-takers Here is a non-language example In the United States some driverS iicense renewals require taking a papershyand-pencil multiple-choice test that covers signs safe speeds and distances lane

CHAPTER 4 Standardized Testingmiddot 69

changesand otherrules of the road Correlational statistics show a strong relationship between high scores on those tests and good driving records so people who do well on these tests are a safe bet to relicense Now an extremely high correlation (of pershyhaps SO or above) may be loosely interpreted to mean that a large majority of the drishyvers whose licenses are renewed by virtue of their having passed the little quiz are good behind-the-wheel drivers What about those few who do not fit the model That small minority of drivers could endanger the lives of the majority and is that a risk worth taking Motor vehicle registration departments in the United States seem to think so and thus avoid the high cost of behind-the-wheel driving tests

Are you willing to rely on a standardized test result in the case of all the learners in your class Of an applicant to your institution or of a potential degree candidate exiting your program These questions will be addressed more fully in Chapter 5 but for the moment think carefully about what has come to be known

as high-stakes testing in which standardized tests have become the only criterion for inclusion or exclusion The widespread acceptance and sometime misuse of this gate-keeping role of the testing industry has created a political educational and moral maelstrom

DEVELOPING A STANDARDIZED TEST

Whlle it is not likely that a classroom teacher with a team of test designers and researchers would be in a position to ~evelop a brand-new standardized test of large-scale proportions it is a yirtual certainty that some day you will be in a posishytion (a) to revise an existing test (b) toadapt or expand an existing test andor (c) to create a smaller-scale standardized test for a program you are teaching in And even if none of the above thre~ cases should ever apply to you it is of paramount importance to understand the process of the development of the standardized tests that have become ingrained in our educational institutions

How are standardized tests developed Where do test tasks and items come from How are they evaluated Who selects items and their arrangement in a test How do such items and tests- achieve consequential validity How are different forms of t~sts equated for diftlculty level Who sets norms and cutoff limits Are security add confidentiality an issue Are cultural and racial biases an issue in test development All these questions typify those that you might pose in an attempt to understand the process of test development

In the steps outlined below three different standardized- tests will be used to exemplify the process of standardized test design

(A) The Test of English as a Foreign Language (TOEFL) Educational Testing Service (ETS) (B) The English as a Second Language Placement Test (ESLP1) San Francisco State University (SFSU) (C)The Graduate Essay Test (Gm SFSU

70 CHAPTER 4 Standardized Testing

The first is a test of general language ability or profidencyThe second is a place ment test at a universityAnd the third is a gate-keeping essay test that all prospecshytive students must pass in order to take graduate-level courses As we look at the steps one by one you will see patterns that are consistent with those outlined in the previous two chapters for evaluating and developing a classroom test

1 Determine ihe purpose and objectives of the test

Most standardized tests are expected to provide high practicality in administration and scoring without unduly compromising validity The initial outlay of time and money for such a test is Significant but the test would be used repeatedly It is therefore imporshytant for its purpose and objectives to be stated specifically Lets look at the three tests

(A) The purpose of the TOEFL is to evaluate the English profiCiency of people whose native language is not English (TOEFL Test and Score Manual 2001 p 9) More specifically theTOEFL is designed to help institutions of higher learning make valid decisions concerning English language profiCiency lin terms of lthelr] own requirements (p 9) Most colleges and universities in the United States use TOEFL scores to admit or refuse international applicants for admission Various cut-off scores apply but most institutions require scores from 475 to 525 (paper-based) or from 150 to 195 (computer-based) in order to consider students for admissionThe high-stakes gate-keeping nature of the TOEFL is obvious

(B) The ESLPT referred to in Chapter 3 is designed to place already admitted students at San Francisco State University in an appropriate course in academic writing with the secondary goal of placing students into courses in oral production and grammar-editing While the tests primary purpose is to make placements another desirable objective is to provide teachers with some diagnostic information about their students on the first day or two of classThe ESLPT is locally designed by university faculty and staff

(C) The GEfanother test designed at SFSU is given to prospective graduate students-both native and non-native speakers-in all disciplines to determine whether their writing ability is sufficient to permit them to enter graduate-level courses in their programs It is offered at the beginning of each term Students who fail or marginally pass the GET are technically ineligible to take graduate courses in their field Instead they may elect to take a course in graduate-level writing of research papers A pass in that course is equivalent to passing the GET

As you can see the objectives of each of these tests are specific The content of each test must be designed to accomplish those particular ends TIlis first stage of goal-setting might be seen as one in which the consequential validity of the test is foreshymost in the mind of the developer each test has a specific gate-keeping function to perfonn therefore the criteria for entering those gates must be specified ~ccurately

2 Design test specifications

Now comes the hard part Decisions need to be made on how to go about structurshying the specifications of the test Before specs can be addressed a comprehensive

CHAPTER4 Standardized Testing 71

progratn of research must identify a set of constructs underlying the test itself This stage of laying the foundation stones can occupy weeks months or even years of effort Standardized tests that dont work are often the product of short-sighted conshystruct validation Lets look at the three tests again

(A) Construct validation for the TOEFL is carried out by the TOEFL staff at ETS under the guidance of a Policy Counell that works with a Committee of Examiners that is composed of appointed external university faculty linguists and assessment spedaJists Dozens of employees are involved in a complex process of reviewing curshyrentTOEFL specifications cOmmissioning and developing test tasks and items assemshybling forms of the test and performing ongoing exploratory research related to formulating new specs Reducing such a complex process to a set ofsimple steps runs the risk of gross overgeneralization but here is an idea of how aTOEFL is created

Because the TOEFL ismiddot a proficiency test the fIrst step in the developmental process is to define the construct of language proficiency First it should be made clear that many assessment specialists such as Bachman (1990) and Palmer (Bachman amp Palmer 1996) prefer the term ability to proficiency and thus speak of language abllity as the overarching concept The latter phrase is more conSistent they argue with our understanding that the specific components of language ability must be assessed separately Others such as the American Council on Teaching Foreign Languages (ACIFL) still prefer the term proficiency because it connotes more of a holistic unitary trait view of language ability (Lowe 1988) Most current views accept the ability argument and therefore strive to specify and assess the many components of language For the purposes of consistency in this book the term proficiency will nevertheless be retained with the above caveat

How you view language will make a difference in how you assess language proshyficiency After breaking language competence down into subsets of listening speaking reading and writing eapoundli~p~ifQ(m~I~~ Jll()4~ can be examined on a conshytiiiuum of linguistic units pho~~logy (p~onunciation) and orthography (spelling) words OeXicon) sentences (gtammar) discourse and pragmatic (sociolinguistic contextual functional cul~j features of language

How will the TOEFL sample from at) these possibilities Oral production tests can be tests of overall conversational fluency or pronunciation of a particular subset of phonolOgy and can take the form of imitation structured responses or free responses Listening comprehension tests can concentrate on a particular feature of language or on overalllistenins for general meaning Tests of realtling can cover the range of language units and can aim to test comprehension of long or short passhysages single sentences or even phrases and words Writing-tests can take on an open-ended form with free composition or be structured to elicit anything from correct spelling to discourse-level competence Are you overwhelmed yet

From the sea of potential performance modes that could be sampled in a test the developer must select a subset on some systematic basis To make a very long story short (and leaving out numerous controversies) the TOEFL had for many years included three types of performance in its organizationai specifications listening strucshyture and reading all of vtuch tested comprehension through standard multiple-choice

72 CHAPTER 4 Standardized Testing

tasks In 1996 a major step was taken to include written production in themiddot computershybased TOEFL by adding a slightly modified version of the already existing Test of Written English (TWE) In doing so some face validity and content validity were improved along with of course a significant increase in administrative expense Each of these four major sections is capsulized in the box below (adapted from the descripshytion of the current computer-based TOEFL at wwwroefLorg) Such descriptions are not strictly speaking specifications which are kept confidential by ETS Nevertheless they can give a sense of many of the constraints that are placed on themiddot design of actual TOEFL specifications

TOEFLlt8gt specifications

Listening Section The listening section measures the examinees ability to understand English as it is spoken in North America Conversational features of the language are bull stressed and the skills tested include vocabulary and idiomatic expression as well as speshycial grammatical constructions that are frequently used in spoken Engfish The stimulus material and questions are recorded in standard North American English

The listening section includes various stimuli such as dialogues short conversations academic discussions and mini-lectures and poses questions that test comprehension of main ideas the order ofa process supporting ideas important details and inferences as well as the ability to categorize topicsobjects

The test developers have taken advantage of the multimedia capability of the computer by using photos and graphics to create context and support the content of the lectures producing stimuli that more closely approximate Ureal-world situations in which people do more than just listen to voices The listening stimuli are often accompashynied by either context-setting or content-based visuals All dialogues conversations acashydemic discussions and mini-lectures include context visuals to establish the setting and role of the speakers Content-based visuals are often used to complement th~ topics of the mini-lectures

Structure-Section Themiddotstructure-section measures an examinees ability to recognize language that is appropriate for standard written English The language tested is formal rather than conversational The topics of the sentences are associated with general acadeshymic discourse so that individuals in specific fields of study or from specific national or linguistic groups have no particular advantage

Two types of questions are used questions in which examinees must (1) complete an incomplete sentence using one of four answers provided and (2) identify one of four unshyderlined words or phrases that would not be accepted in English The two question types are mixed randomly rather than being separated into two subsections as in the papershybased TOEFL test

Reading Section The reading section measures the ability to read and understand short passages similar in topic and style to academic texts used in North American colshyleges and universities Examinees read a variety of short passages on academic subjects and answer several questions about each passage Test items refer to what is stated or imshyplied in the passage as well as to words used in the passage To avoid creating an advanshyt~ t( 1 dull5- In 3n~ ont fidd of stud~ sufficient context is provided SO that no ~ -middott - ~tmiddot J ~ middoth - -~ rM~ ir~ to ~-er the if~(J(lS~ ~ t ~ ~ ~~~~-- Ih L_ 1_ 1 ~_

CHAPTER 4 Standardized Testing 73

The reading section consists of four to five passages of 250-350 words with 10-14 questions per passage This section is not computer-adaptive soexaminees can skip questions and return to previous questions The questions in this section assess the comshyprehension of main ideas inferences factual information stated in a passage pronoun referents and vocabulary (direct meaning synonym antonym) In all cases the questions can be answered by reading and understanding the passages This section consists of (1 ) traditional multiple-choice questions (2) questions that require examinees to click on a word phrase sentence or paragraph to answer and (3) questions that ask examinees to insert a sentence where it fits best

Writing Section The writing section measures the ability to write in English including the ability to generate organize and develop ideas to support those ideas with examples or evidence and to compose a response to one assigf)ed topic in standard written Enshyglish Because some examinees may not be accustomed to composing an essay on comshyputer they are given the choice of handwriting or typing the essay in the 30-minute time limit The rating scale for scoring the essay ranging from 0 to 6 is virtually the same as that of the Test of Written English [see Chapter 9 of this book] A score of 0 is given to papers that are blank simply copy the topic are written ina language other than English consist only of random keystroke characters or are written on a topic different from the one assigned

Each essay is rated independently by two trained certified readers Neither reader knows the rating assigned by the other An essay will receive the average of the two ratshyings unless there is a discrepancy of more than one point in that case a third reader will independently rate the essay The essay rating is incorporated into the StructureMriting scaled score~ and constitutes approximately 50 percent of that combined score

(B) The designing of the test specs for the ESLPT was a somewhat simpler task because the purpose is placement and the construct validation of the test consisted of an examination of the content of the ESL courses In fact in a recent revisiofi of the ESLPT (lmao et al 2000 Imao 2001) content validity (coupled with its attenshydant face validity) was the central theoretical issue to be considered The major issue centered on designing practical and reliable tasks and item response formats Having established the importance of designingESLPT tasks that simulated classroom tasks used in the courses the designers ultimately specified two writing production tasks (one a response to an essay that students read and the other a summary of another essay) and one multiple-choice grammar-editing taskThese specifications mirrored the readingbased process writing approach used in the courses

(C) Specifications for the GET arose out of the perceived need to provide a threshold of acceptable writing ability for all prospective graduate students at SFSU both native and non-native speakers of EnglishThe specifications for the GET are the skills of writing grammatically and rhetorically acceptable prose on a topic of some interest with clearly produced organization of ideas and logical development The GET is a direct test of writing ability in which test-takers must in a two-hour time period write an essay on a given topic

74 CHAPTER 4 Standardized Testing

3 Design select and arrange test tasksitems

Once specifications for a standardized test have been stipulated the sometimes never-ending task of designing selecting and arranging items beginS The spe~s act much like a blueprint in determining the number and types of items to be created Lets look at the three examples

(A) TOEFL test design specifies that each item be coded for content and statisshytical characteristics Content coding ensures that each examinee will receive test questions that assess a variety of skills (reading comprehending the main idea or understanding inferences) and cover a variety of subject matter without unduly biasing the content toward a subset of test-takers (for example in the listening secshytion involving an academic lecture the content must be universal enough for stushydents from many different academic fields of study) Statistical characteristics including the IRT equivalents of estimates of item ~~ility (IF) and the ability of an item to discriminate (ID) between higher orlower ability levels ate also coded

Items are then designed by a team who select and adapt items solicited from a bank of items that have been deposited by freemiddotlance writers and ErS staff Probes for the reading section for example are usually excerpts from authentic general or academic reading that are edited for linguistic difficulty culture bias or other topic biases Items are designed to test overall comprehension certain specific informashytion and inference

Consider the following sample of a reading selection and ten items based on it from a practice TOEFL (Phillips 2001pp423-424)

For hundreds of years in the early history of America pirates sailed through coastal washyters pillaging and plundering all in their path They stole from other ships andstole from coastal towns not content only to steal they destroyed everything they could not carry avay~ Some of the pirate ships amassed large treasures~ the fates of which are unknown leaving people of today to wonder at their whereabouts and to dream of one day coming across some lost treasure

One notoriously large treasure was on the pirate ship Whidah which sank in the washyters off Cape Cod during a strong storm in 1717 A hundred of the crew members went down with the ship along with its treasure of coins gold silver and jewels The treasure on board had an estimated value on todays market of more than 100 million dollars

The remains of the Whidah were discovered in 1984 by Barry Clifford who had spent years of painstaking research and tireless searching only finally to locate the ship about 500 yards from shore A considerable amount of treasure from the centuries-old ship has been recovered from its watery grave but there is clearly still a lot more out there Just as a reminder of what the waters off the coast have been protecting for hundreds of years occasional pieces of gold or silver or jewels still wash up on the beaches and lucky beach-goers find pieces of the treasure

11 ~ Thepa~e mainly diccus5eS

CH4PTER 4 Standardized Testing 75

(e) what really happened to the Whidahs pirates (D) why people go to the beach

12 It is NOT mentioned in the passage that pirates did which of the following (A) They killed lots of people (B) They robbed other ships (e) They took things from towns (D) They gathered big treasures

13 The word amassed in line 4 is closest in meaning to (A) sold (e) transported (B) hid (D) gathered

14 It is implied in the passage that the Whidahs crew (A) died (B) went diving (e) searched for the treasure (D) escaped with parts of the treasure

15 Which of the following is NOT mentioned as part of the treasure of the Whidah (A) Art objects (B) Coins (e) Gold and si Iver (D) Jewels

16 The word estimated in line 10 is closest in meaning to which of the following (A) Known (C) Approximate (B) Sold (D) Decided

17 The passage indicates that the cargo of theWhidah is worth about (A) $100000 (B) $1000000 (C) $10000000 (D) $100000000

18 The work that Barry Clifford did to locate the Whidah was NOT (A) successfu I (B) effortless (C) detailed (D) lengthy

19 It is mentioned in the passage that the treasure of the Whidah (A) is not very valuable (8) is all in museums (C) has not all been found (D) was taken to share by the pi rates

20 The paragraph following the passage most likely discusses (A) what Barry Clifford is doing today (8) the fate of the Whidahs crew (e) other storms in the area of Cape Cod (D) additional pieces that turn up from the Whidahs treasure

76 CHAPTER 4 Standardized Testing

As you can see items target the assessment of comprehension of the main idea (item 11) stated details (17 19) unstated details (12 15 18) implied details (14 20) and vocabulary in context (13 16) An argument could be made about the cultural schemata implied in a passage about pirate ships and you could engage in an angels on the head of a pin argument about the importance of picking cershytain vocabulary for emphasis but every test item is a sample of a larger domain and each of these fulfills its designated specification

Before any such items are released into a form of the TOEFL (or any validated standardized test) they are piloted and sCientifically selected to meet difficulty specshyifications within each subsection section and the test overall Furthermore those items are also selected to meet a desired discrimination index Both of these indices are important considerations in the design of a computer-adaptive test where pershyformance on one item determines the next one to be presented to the test-taker (See Chapter 3 for a complete treatment of multiple-choice item design)

(B)The selection of items in the ESLPT entailed two-entirel) different processes In the two subsections of -the test that elicit writing performance (summary of reading response to reading) the main hurdles were (a) selecting appropriate passhysages for test-takers to read (b) providing appropriate prompts and (c) processing data from pilot testing Passages have to conform to standards of content validity by being within the genre and the difficulty of the material used in the courses The prompt in each case (the section asking for a summary and the section asking for a response) has to be tailOred to fit the passage but a general template is used

[n the multiple-choice editing test that seeks to test grammar proofreading ability the first and easier task is to choose an appropriate essay within which to embed errors The more complicated task is to embed a specified number of errors from a previously determined taxonomy of error categories Those error categories came directly from student errors as perceived by their teachers (verb tenses verb agreeshyment logical connectors articles etc) The disttactors for each item were selected from actual errors that students make Itemsiti pilot versions were then coded fordifshyficulty and discrinlination indices after which final assembly of items could occur

(C) The GET prompts are designed by a faculty committee of examiners who are speCialists in the field of university academic writing The assumption is made that the topics are universally appealing and capable of yielding the intended product of an essay that requires an organized logical argument and conclusion No pilot testing of prompts is conducted The conditions for administration remain constant two-hour time limit sit-down context paper and pencil closed-book format Consider the following recent prompt

Graduate Essay Test sample prompt

In the Middletown Elementary School District the assistant superintendent has just been made superintendent in another district Her resignation leaves vacant the districts only administrative position ever held by a woman The School Board in response to strong

CHAPTER 4 Standardized Testing 77

arguments from the Teachers Association has urged that a woman be hired to replace her As a member of the hiring committee you must help choose her successor

Only one woman applicant meets the written qualifications for the job the two top male applicants are both more experienced than she

The hiring committee has asked each committee member to prepare a written statement to distribute before meeting together to discuss the issue Write a report that represents your position making it as logical and persuasive as possible

Some facts you may wish to draw on 1 Women make up more than 75 percent of classroom teachers but hold fewer than

10 percent of administrative positions in education Administrators salaries average 30 percent more than teachers salaries

2 The local Teachers Association is 89 percent women mostly under 40 In a heated debate on television a member of the National Organization of Women (NOW) and the chair of the Teachers Association threatened if a man is hired to bring a class-action suit against the district on behalf of all women teachers who cannot expect advancement because of discriminatory hiring practices

3 The local Lions Club which contributes heavily to school sports says hiring the less experienced woman would not be in the best interests of the schoolthe children or the teachers

The finalists for the position

1 Carole Gates Classroom teacher 10 years Teacher of the Year 1985 supervisor ofpractice teachers at Teachers College former president of Teachers Associ ati on Administrative Credential 1984 EdD degree 1986 assistant principal of Hoptown Elementary School 2 years

2 Spud Stonewall Principal of Middletown Elementary 15 years PhD in educational adminis~ration State Board of Education Committee for Improving Elementary School Curriculum 1982-present

3 Jim Henderson School Administrator 22 yearsgradesK-9-supports innovation in education Fair Bargaining Award 1981 former coach for winning collegiate basketball team 10 years

It is clear from such a prompt that the problem the test-takers must address is complex that there is sufficient information here for writing an essay and that testshytakers will be reasonably challenged to write a clear statement of opinion What also emerges from this prompt (and virtually any prompt that one might propose) is the potential cultural effect on the numerous international students who must take the GIIT Is it possible that such students who are not familiar with school systems in the United States with hiring procedures and perhaps with the politics of school board elections might be at a disadvantage in mounting their arguments within a two-hour time frame Some (such as Hosoya 2001) have strongly claimed such a bias

78 CHAPTER 4 Standardized Testing

4 Make appropriate evaluations of different kinds of items

In Chapter 3 the concepts of item facility (IF) item discrimination (ID) and disshytractor analysis were introduced As the discussion there showed such calculations provide useful infornlation for classroom tests but sometimes the time and effort involved in perfornling them may not be practical especially if the classroom-based test is a one-time test Yet for a standardized multiple-choice test that is designed to be marketed commercially andor administered a number of times andor adminisshytered in a different form these indices are a must

For other types of response formats namely production responses different forms of evaluation become importantThe principles of p-mpoundti~ality ~d poundabWty are prominent along with the concept o(JacjJjt Practicality issues in such items include the clarity of directions timing of the test ease of administration and how much time is required to score responses Reliability is a major player in instances where more than one scorer is employed and to a lesser extent when a single scorer has to evaluate tests over long spans of time that could lead to deterioration of stanshydards Facility is also a key to the validity and success of an item type ~irecshytions complex- language obscure topics fuz~Qata and culturally biased

~Jfiformatioifma~alliead to a highei1eVermiddotof diffiCidty than one desires (A) The IF ID and efficiency statistics of the multiple-choice items of current

forms of the TOEFL are not publicly available information For reasons of security and protection of patented copyrighted materials they must remain behind the closed doors of the ETS development staff Those statistics remain of paramount importance in the ongoing production ofTOEFL items and forms and are the founshydation stones for demonstrating the equatability of forms Statistical indices on retired forms of the TOEFL are available on request for research purposes

The essay portion of theTOEFL undergoes scrutiny for its practicality reliability and facility Special attention is given to reliabilIty since two human scorers must read each essay and every time a third reader becomes necessary (when the two readers disagree by more than one point) it costs ETS more money

(B) In the case of the open-ended responses on the two written tasks on the ESLPT a similar set of judgments must be made Some evaluative impressions of the effectiveness of prompts and passages are gained from informal student and scorer feedback In the developmental stage of the newly revised ESLPT both types of feedshyback were formally solicited through questiQnnaires and interviews That informashytion proved to be invaluable in the revisIon of prompts and stimulus reading passages After each administration now the teacher-scorers provide informal feedshyback on their perceptions of the effectiveness of the prompts and readings

The multiple-choice editing passage showed the value of statistical findings in determining the usefulness of items and pointing administrators toward revisions Following is a sample of the format used

CHAPTER 4- Standardized Testing 79

Multiple-choice editing passage

(1)EYer since supermarkets first appeared they have beentake over ~ world ABC 0

(2) Supermarkets have changed peoples life ~ yet and at the same time changes in ABC

peoples life ~ have encouraged the opening of supermarkets o

The task was to locate the error in each sentence Statistical tests on the experishymental version of this section revealed that a number of the 45 items were found to be of zero IF (no difficulty whatsoever) and of inconsequential discrimination power (some IDs of 15 and lower) Many distractors were of no consequence because they lured no one Such information led to a revision of numerous it~ms and their options eventually strengthening the effectiveness of this section

(C)The GET like its written counterparts in the ESLPT is a test ofwritten ability with a single prompt and therefore questions of practicality and J~~illy~are also largely observational No data are collected from students on their perceptions but the scorers have an opportunity to reflect on the validity ofa given topiC After one sitting a topic is retired which eliininates the possibility of improving a specific topiC but future framing of topics might benefit from scorers evaluations Inter-rater reliability is checked periodically and reader training sessions are modified if too many instances of unreliability appear

5 Specify scoring procedures and reporting formats - ---

A systematic assembly of test items in pre-selected arrangements and sequences all of which are validated to confo~ to an e~pected difficulty level should yield a test that can then be scored accurately and reported back to test-takers and institutions efficiently

(A) Of the three tests being exemplifled here the most straightforward scoring procedure comes from the TO~FL the one with the most complex issues of validashytion deSign and assembly Scores are calculated and reported fora) three sections of the TOEFL (the essay ratings are combined with the Structure and Written Expression score) and (b) a total score (range 40 to 300 on the computer-based TOEFL and 310 to 677 on the paper-and-pencil TOEFL) A separate score (c) for the Essay (range 0 to 6) is also provided on the examinees score record (see simulation of a score record on page 80)

80 CHAPTER 4 Standardized Testing

Facsimile of a TOEFLreg score report

TOEFL Scaled Scores Claudia Y Estudiante Peru ___

19 17 17 177 Listening Structure Writing Reading Total Score

Essay rati ng 30

The rating scale for the essay is virtually the same one that is used for the Test of Written English (see Chapter 9 for details) with a zero level added for no response copying the topic only writing completely off topic or not writing in English

(B) The ESLPT reports a score for each of themiddot essay sections but the rating scale differs between them because in one case the objective is to write a summary and in the other to write a response to a reading ~ch essayi~pd lgtY ~o readet~ ifhFfF js a discrepancy of more than one level a third reader1resolves the differenceThe ~ditiilg section is machine-scanned and -scored with a total score and ~th part-scores for each ofthe grammaticaVrhetorlcal sectionS From these data placement administrators have adequate information to make placements and teachers receive some diagnostic inforshymation on each student in their classes Students do not receive their essays back

(C) Each GET is read by two trained readers who give a score between 1 and 4 according to the following scale

Graduate Essay Test Scoring Guide

Please make no marks on the writers work Write your reader number and score on the front cover of each test booklet

4 Superior The opening establishes context purpose and point of view the body of the essay developsmiddot recommendations-logically and coherently The writer demonshystrates awareness of the complexities in the situation and provides analysis of the probJem offers compelling or common-sense reasons for recommendations made makes underlying assumptions explicit

The writer uses fluent and idiomatic English with few mechanical errors Style reshyveals syntactic maturity is dear and direct is not choppy or over-colloquial nor over-formal stuffy or unfocused Occasional spelling or punctuation errors may be easily attributed to hasty transcription under pressure

3 Competent After an opening that establishes context and purpose the paper unfolds with few lapses in coherence but may have somewhat less clear organization of less explicit transitions than a top-score paper It may have somewhat less compelling logic or slightly less-wellreasoned suggestions than a 4 paper though it will provide reasons for the recommendations made

The writer uses dear fluent and generally idiomatic English but may make minor or infrequent ESL errors (preposition errors dropped articles or verb endings etc) or repeat a single error (eg not punctuate possessive nouns) Occasional lapses of style are offSet by demonstrated mastery of syntax

CHAPTE84 Standardized Testing 81

2 Weak The writer makes somewhat simplistic suggestions not fully supported with reashysons fails to cite key facts offers little analysis of the problem or shows a limited grasp of the situation the given information is copied or listed withlittle integration into argument Points may be random or repetitious Writing may be badly focused with careless use of abstract language resulting in predication errors or illogical sentences

ESL andlor careless mechanical errors are frequent enough to be distracting OR sentences may be choppy style over-casual usage occasionally unidiomatic

1 Inadequate The essay may be disjointed incoherent or minimally developed The writer shows little grasp of the complex issues involved is unable to establish conshytext point of view or purpose in opening of paper or has a poor sense of audience Mechanical andor ESL errors or unidiomatic usages are frequent sentences may be ungrammatical OR correct but short and very simple

The two readers scores are added to yield a total possible score of 2 to 8 Test administrators recommend a score of 6 as the threshold for allowing a student to pursue graduate-level courses Anything below that is accompanied by a recomshymendation that the student either repeat the test or take a remedial course in gradshyuate writing offered in one of several different departments Students receive neither their essays nor any feedback other than the fmal score

6 Perform ongoing construct validation studies

From the above discussion it should be clear that no standardized instrument is expected to be used repeatedly without a ramporou~program of ongoing c~-sectmct valiltiatiOll Any standardized test once developed must be accompanied by sysshy~

tematic periodic corroboration of its effectiveness and by steps toward its improveshyment This rigor is especially true of tests that are produced in equated forms that is forms must be reliable across tests such that a score on a subsequent form of a test-has-the~same validityand-interpretability as its original

(A) The TOEFL program in cooperation with other tests produced by ETS has an impressive program of research Over the years dozens of TOEFL-sponsored research studies have appeared in the TOEFL Monograph Series An early example ofsuch a study was the seminal Duran et aI (1985) study TOEFLfrom a Communicative ViewpOint on Language Proficiency which examined the content characteristics of the TOEFL from a communicative perspective based on current research in applied linguistics and lanshyguage proficiency assessment More recent studies (such as Ginther 2001 Leacock amp Chodorow 2001 Powers et aI 2002) demonstrate an impressive array of scrutiny

(B) For approximately 20 years the ESLPT appeared to be placing students relishyably by means of an essay and a multiple-choice grammar and vocabulary test Over the years the security of the latter became s1lspect and the faculty administrators wished to see some content validity achieved in the process In the year 2000 that process began with a group of graduate students (Imao et aI 2000) in consl1ltation with faculty members and continued to fruition in the form of a new ESLPT reported in lmao (2002) The development of the new ESlPT involved a lengthy process of

82 CHAPTER 4 Standardized Testing

both content and construct validation along with facing such practical issues as scoring the written sections and a machine scorable multiple-choice answer sheet

The process of ongoing validation will no doubt continue as new forms of the editing section are created and as new prompts and reading passages are created for the writing section Such a validation process should also include consistent checks on placement accuracy and on face validity

(C) At this time there is little or no research to validate the GET itself For its conshy struct validation its administrators rely on a stockpile of research on university-level academic writing tests such as theTWEThe holistic scoring rubric and the topics and administrative conditions of the GET are to some extent patterned after that of the TWE In recent years some criticism of the GEf has come from international test-takers (Hosoya 2001) who posit that the topics and time limits of the GET among other facshytors work to the disadvantage of writers whose native language is not English These validity issues remain to be fully addressed in a comprehensive research study

I I

STANDARDIZED IANGUAGE PROFICIENCY TESTING

Tests of language profiCiency presuppose a comprehensive definition of the specific competencies that comprise overall language ability The specifications for the TOEFL provided an illustration of an operational definition of ability for assessment purposes This is not the only way to conceptualize the concept Swain (1990) offered a multidimensional view of profiCiency assessment by referring to three linshyguistic traits (grammar discourse and sociolinguistics) that can be assessed by means of oral multiple-choice and written responses (see Table 41) Swains conshyception was not meant to be an exhaustive analysis of ability but rather to serve as an operational framework for constructing proficiency assessments

Another defmition and conceptualization of profiCiency is suggested by the ACTFL association mentioned earlier ACfFL takes a holistic and more unitary view of proficiency in describing four levels superior advanced intermediate and noviceWithin each level descriptions of listening speaking reading and writing are provided as guidelines for assessment For example the ACfFL Guidelines describe the superior level of speaking as follows

ACTFL speaking guidelines summary superior-level

Superior-level speakers are characterized by the ability to

bull participate fully and effectively in conversations in formal and informal settings on topics related to practical needs and areas of professional andor scholarly interests

bull provide a structured argument to explain and defend opinions and develop effective hypotheses within extended discourse

bull discuss topics concretely and abstractly bull deal with a linguistically unfamiliar situation bull maintain a high degree of linguistic accuracy bull satisfy the linguistic demands of professional andor scholarly life

CHAPTER4 Standardized Testing 83

The other three ACfFL levels use the same parameters in describing progressively lower proficiencies across all four skills Such taxonomie~ have the advantage of considering a number of functions of linguistic discourse but the disadvantage at the lower levels of overly emphasizing test-takers deficiencies

Table 41 Traits of second language proficiency (Swain 1990 p 403)

Trait Grammar Discourse Sociolinguistic

focus on grammatical focus on textual focus on social accuracy within cohesion and appropriateness of sentences coherence language use

Method

Oral structured interview story telling and argumentationpersuasion

role-play ofspeech acts requests offers complaints

scored for accuracy of verbal morphology prepositions syntax

detailed rating for identification logical sequence and time orientation and global ratings for coherence

scored for ability to distinguish formal and informal register

Multiple-choice

sentence-level select the correct form exercise

paragraph-level select the coherent sentence exercise

speech act-Ievelselect the appropriate utterance exercise

(45 items) (29 items) (28 items)

involving verb morphology prepositionsan-d-uther items

Written composition

narrative and letter of persuasion

narrative and letter of persuasion

formal request letter and informal note

scored for accuracy of verb morphology prepositions syntax

detailed ratings much as for oral discourse and global rating for coherence

scored for the ability to distinguish formal and inforJ1lil1 register

FOUR STANDARDIZED lANGUAGE PROFICIENCY TESTS

We now tum to some of the better-known standardized tests of overall language ability or profiCiency to examine some of the typical formats used in commercially available tests We will not look at standardized tests of other specific skills here but that should not lead you to think by any means that proficiency is the only kind of test in the field that is standardized Three standardized oral production tests the

84 CHAPTER 4 Standardized Testing

Test of Spoken English (fSE) the Oral Proficiency Inventory (OPI) and PbonePassreg are discussed in Chapter 7 and the Test of Written English (WE) is covered in ChapterS

Four commercially produced standardized tests of English language proficiency are described briefly in this section the TOEFL the Michigan English Language Assessment Battery (MELAB) the International English Language Testing System (lELTS) and the Test of English for International Communication (fOEICreg) In an appendix to this chapter are sample items from each section of each test When you turn to that appendix use the following questions to help you evaluate these four tests and their subsections

1 What item types are included 2 How practical and reliable does each subsection of each test appear to be 3 Do the item types and tasks appropriately represent a conceptualizatio~ of

language proficiency (ability) That is can you evaluate their construct validity

4 Do the tasks achieve face validity 5 Are the tasks authentic 6 Is there some washback potential in the tasks

Test of English as a Foreign Language (TOEFL)

Producer Educational Testing Service (ETS) Objective To test overall proficiency (language ability) Primary market Almost exclusively US universities and colleges for admission

purposes Type Computer-based (CB) (and two sections are-computer-adaptive)

A traditional paper-based (PB) version is also available Response modes Multiple-choice responses essay Specifications See the box on pp 72-73 Time allocation Up to 4 hours (CB) 3 hours (PB) Internet access wwwtoeflorg

Comments In the North American context the TOEFL is the most widely used comshymercially available standardized test of proficiency Each year the TOEFL test is adminisshytered to approximately 800000 candidates in more than 200 countries It is highly respected because of the thorough program of ongoing research and development conshyducted by ETS The TOEFLs primary use is to set proficiency standards for international students seeking admission to English-speaking universities More than 4200 academic institutions government agencies scholarship programs and licensingcertification agenshycies in more than 80 countries use TOEFL scores By 2004 the TOEFL will include a secshytion on oral production

CHAPTER 4 Standardi~ed Testing 85

Michigan English Language Assessment Battery (MELAB)

Producer English language Institute University of Michigan Objective To test overall proficiency (language ability) Primary market Mostly US and Canadian language programs and colleges

some worldwide educational settings as well Type Paper-based Response modes Multiple-choice responses essay Time allocation 25 to 35 hours Internet access wwwlsaumicheduelimelabhtm

Specifications The MElAB consists of three sections Part 1 a 3D-minute impromptu essay is written on an assigned topic Part 2 a 25-minute multiple-choice listening comshyprehension test is delivered via tape recorder Part 3 is a 100-item 75-minute multipleshychoice test containing grammar doze reading vocabulary and reading comprehension An oral interview (speaking test) is optional

Comments The Ell at the University of Michigan has been producing the MELAB and its earlier incarnation (Michigan Test of English language Proficiency) since 1961 like the TOEFL it serves a North American audience but is also used internationally While its use is not as widespread as the TOEFL its validity is widely respected Because it is cheaper than the TOEFL and more easily obtained it is popular among language schools and institutes Many institutions and companies accept MElAB scores in lieu ofTOEFL scores

International English Language Testing System (IELTS)

Producer Jointly managed by The University of Cambridge local Examinations Syndicate (UClES) The British Council and lOP Education Australia

Objective To test overall proficiency (language ability) Primary-market Australian British Canadian and New Zealand academic

institutions and professional organizations American academic institutions are increasingly accepting IELTS for admissions purposes

1)rpe Computer-based (for the Reading and Writing sections) papershybased for the listening and Speaking modules

Response modes Multiple-choice responses essay oral production Time allocation 2 hours 45 minutes Internet access httpwwwieltsorgl

httpwwwudesorguk httpwwwbritishcouncilorg

Specifications Reading candidates choose between academic reading or general training reading (60 minutes) Writing the same option academic writing or general training writing (60 minutes) Listening four sections for all candidates (30 minutes) Speaking five sections for all candidates (1015 minutes)

86 CHAPTER 4 Standardized Testing

Comments The University of Cambridge local Examinations Syndicate (UCLES) has been producing English language tests since 1858 Now with three organizations cooperatshying to form the IELTS more than a million examinations are administered every year In 2002 a computer-based version of the Reading and Writing modules of the IELTS became available at selected centers around the world The other sections are administered locally by an examinet The paper-based IELTS remains an option for candidates The IELTS retains the distinct advantage of requiring all four skills in the test-takers performance

Test of English for International Communication (TOEICreg)

Producer The Chauncey Group International a subsidiary of Educational Testing Service

Objelttive To test overall proficiency (langlJage ability) Primary market Worldwide business commerce and industry contexts

(workplace settings) Type Computer-based and paper-based versions Response modes Multiple-choice responses Time allocation 2 hours Internet access httpwwwtoeiccom

Specifications Listening Comprehension 100 items administered by audiocassette Four types of task statements questions short conversations and short talks (approxishymately 45 minutes) Reading 100 items Three types of task cloze sentences error recogshynition and reading comprehension (75 minutes)

Comments The TOEIC has become a very widely used international test of English proficiency in workplace settings where English is required for job performance The conshytent includes many different employment settings such as conferences presentations sales ordering shipping schedules reservations (etters and memoranda It is approprishyate to use in educational settings where vocational or workplace English courses are being offered

sect sect sect sect sect

The construction of a valid standardized test is no minor accomplishment whether the instrument is large- or small-scale The designing of specifications alone as this chapter illustrates requires a sophisticated process of construct valishydation coupled with considerations of practicality Then the construction of items and scoringinterpretation procedures may require a lengthy period of trial and error with prototypes of the final form of the testWith painstaking attention to all the details of construction the end product can result in a cost-effective timeshysaving accurate instrument Your use of the results of such assessments can provide useful data on learners language abilities But your caution is warranted as well for all the reasons discussed in this chapter The next chapter will elaborate on what lies behind that need for a cautious approach to standardized assessment

CHAPTER4 Standardized Testing 87

EXERCISES

[Note (I) Individual work (G) Group or pair work (C) Whole-class discussion]

1 (C) Tell the class about the worst test experience youve ever had Briefly anamiddot lyze what made the experience so unbearable and try to come up with sugshygestions for improvement of the test andor its administrative conditions

2 (G) In pairs or small groups compile a brief list of pros and cons of standardshyized testing Cite illustrations of as many items in each list as possible Report your lists and examples to the rest o~ the class

3 (I) Select a standardized test that you are quite familiar with (probably a recent experience) Mentally evaluate that test using the five principles of practicality reliability validity authenticity and washback Report yourevaluashytion to the class

4 (G) The appendix to this chapter provides sample items from Jour different tests of language proficiency In groups one test for each group analyze your test for (a) content validity (b) face validity and (c) authenticity

5 (C) Do you think that the sample TOEFL reading passage about pirates (pages 74-75) and the Graduate EssayTest prompt (pages 76-77) about a school board hiring committee have any culture bias Discuss this and other cultural biases you have noticed in tests Is it possible to design a test that is completely free of culture bias

6 (CG) Compare the differences in conceptualization of language proficiency represented by Swains model the TOEFL and the ACfFL philosophy Which one best represents current thinking about communicative language ability What are the strengths and weaknesses of each approach

FORYOlIILEURTHER READING

Gronlund Norman E (1998) Assessment of student achievement Sixth Edition Boston Allyn and Bacon

Gronlunds classic also mentioned in Chapter 3 offers a concise overview of features of standardized tests offering definitions and examples of the statistical considerations in interpreting scores His approach is unbiased cleady written and accessible to those who might fear the mathematics of standardized testing

Phillips Deborah 2001 Long1nan introductory course for the TOEFL test White Plains NY Pearson Education

A careful examination of this or any other reputable preparation course for a standardized language test is well worth a students time Note especially how the book acquaints the user with the specifications of the test and offers a number of useful strategie~ that can be llsed in preparation for the test and during irs adn1inistration

88 CHAPTER 4 Standardized Testing

APPENDIX TO CHAPTER 4

Commercial Proficiency Tests Sample Items and Tasks

Test of English a~ a Foreign Language (TOEFLreg)

Listening r

Part A

In this section you will hear short conversations between two people In some ofthe conversations each person speaks only once In other conversations one or both of the people speak more than once Each conversation is followed by one questionabQlt it Each question in this part has four answer choices You should click on the best answer to each question Answer the questions on the basis of what is stated or implied by the speakers Here is an example On the computerscreen you will see

[man and woman talking]

On the recording you will hear

(woman) Hey wheres your sociology book (man) At home Why carry it around when were just going to be taking

a test (woman) Dont you remember Professor Smith said we could us it during

the test (man) Ohl no Well Ive still got an hour right Im so glad I ran into you

You wiII then see and hear the question before the answer choices appear

What will the man probably do next

o Begin studying for the sociology test o Explain the problem to his professor o Go home to get his textbook o Borrow the womans book

To choose an answer you will click on an oval The oval next to that answer will darken After you click on Next and Confirm Answer the next conversation will be presented

Part B

In this section you will hear several longer conversations and talks Each conversation or talk is followed by several questions The conversations talks and questions will not be repeated The conversations and talks are about a variety of topics You do not need speshycial knowledge of the topics to answer the questions correctly Rather you should answer each question on the basis of what is stated or implied by the speakers in the conversashytions or talks

For most of the questions you will need to click on the best of four possible answers Some questions will have special directions The special directions will appear in a box on the computer screen Here is an exampie ot a conversation and some questions

CHAPTER 4 Standardized Testing 89

Marine Biology (narrator) Listen to part of a discussion in a marine biology class

(professor) A few years ago our local government passed a number of strict environmental laws As a result Sunrise Beach looks nothing Ii ke it did ten years ago The water is cleaner and theres been a tremendous increase in all kinds of marine life which is why were going there on Thursday

(woman) I dont know if I agree that the water quality has improved I mean I was out there last weekend and it looked all brown It didnt seem too clean to me

(professor) Actually the color of the water doesnt always indicate whether its polluted The brown color you mentioned might be a result of pollution or it can mean a kind of brown algae is growing there Its called devils apron and it actually serves as food for whales

(man) So when does the water look blue (professor) Well water thats completely unpolluted is actually colorless But

it often looks bluish-green because the sunlight can penetrate deep down and thats the color thats reflected

(woman) But sometimes it looks really green Whats that about (professor) Ok well its the same principle as with devils apron the

water might be green because of different types of green algae there-gulfweed phytoplankton You all should finish reading about algae and plankton before we go In fact those are the types of living things Im going to ask you to be looking for when were there

Now get ready to answer the questions

What is the discussion mainly about

o The importance of protecting ocean environments o The reasons why ocean water appears to be different colors o The survival of whales in polluted water o The effect that colored ocean water has on algae

To choose an answer click on an oval The oval next to that answer will darken After you click on Next and Confirm Answer the next question will be presented

According to the professor what can make ocean water look browngt

o Pollution o Cloudy Skies o Sand o Algae

Click on 2 answers

To choose your answers you will click on the squares An XII wiii appear in each square

bullbullbullbullbullbullbull

90 CHAPTER 4 Standardized Testing

Structure and Written Expression This section measures the ability to recognize language that is appropriate for standard written English There are two types ofquestions in this section In the first type ofquestion there are incomplete sentences Beneath each sentence there are four words or phrases

Directions CIiSk on the one word or phrase that best completes the sentence

The colum~ine flower __ to nearly all of the United States can be raised from seed in almost any garden

native how native is how native is it is native

Time Help Confirm

After you click on Next and Confirm Answ~ the next question willbe presented

The second type of question has four underlined words or phrases You will choose the one underlined word or phrase that must be changed for the sentence to be correct

Directions Click on the one underlined word or phrase that must be changed for the senshytence to be correct

One of the most difficult problems in understanding sleep is determining what the funcshytions of sleep ~

lime Help Confirm

Clicking on an underlined word or phrase will darken it

Reading This section measures the ability to read and understand short passages similar in topic and style to those that students are likely to encounter in North American universities and colleges This section contains reading passages and questions about the passages There are several different types of questions in this section In the Reading section you will first have the opportunity to read the passage

The temperature of the Sun is over 10000 degrees Fahrenheit at the surface but it rises perhaps more than 270000000 at the center The Sun is so much hotter than the Earth that matter can exist only as a gasl except perhaps at the core In the core of the Sun the pressures are so great that despite the high temperature there may be a small solid core However no one really knows since the center of the Sun can never be directly observed ~ Solar astronomers do know that the Sun is divided into five general layers or zones Starting at the outside and going down into the Sun the zones are the corona chromoshysphere hotosphere convection zone and finally the core The first three zones are reshygarded as the Suns atmosphere But since the Sun has no solid surface it is hard to middottell where the atmosphere ends and the main body of the Sun begins

The Suns outermost layer begins about 10000 miles above the visible surface and goes outward for millions of miles This is the only part of the Sun that can be seen during an eclipse such as the one in February 1979 At any other time the corona can be seen

bullbullbullbullbullbullbull

bull bullbullbullbullbullbull

CHAPTER 4 Standardized Testing 91

only when special instruments are used on cameras and telescopes to block the light from the photosphere

The corona is a brilliant pearly white filmy light about as bright as the full Moon Its beautiful rays are a sensational sight during an eclipse The coronas rays flash out in a brilliant fan that has wispy spikelike rays near the Suns north and south poles The corona is generally thickest at the Suns equator The corona is made up of gases streamshying outward at tremendous speeds that reach a temperature of more than 2 million deshygrees Fahrenheit The gas thins out as it reaches the space around the planets By the time the gas of the corona reaches the Earth it has a relatively low density

When you have finished reading the passage you will use the mouse to click on Proceed Then the questions about the passage will be presented You are to choose the one best anshyswer to each question Answer all questions about the information in a passage on the basis ofwhat is stated or implied in that passage Most ofthe questions will be multiple-choice questions To answer these questions you will click on a choice below the question

With what topic is paragraph 2 mainly concerned

o How the Sun evolved o The structure of the Sun o Why scientists study the Sun o The distaflce of the Sun from the planets

Paragraph 2 is marked with an arrow (~)

You will see the next question after you click on Next

To answer some questions you will click on a word or phrase Here is an example

Look at the word one in the passage Click on the word or phrase in the bold text that one refers to To answer you can click on any part of the word or phrase in the passage Jour choice will darken to show which word you have chosen

The Suns outermost layer begins about 10000 miles above the visible surface and goes outward for millions of miles This is the only part of the Sun that can be seen durshying an eclipse such as the one in February 1979 At any other time the corona can be seen only when special instruments are used on cameras and telescopes to block the Iight from the photosphere

You will see the next question after you click on~ To answer some q~estions you will click on a sentence in the passage Here is an example

~ The corona is a brilliant pearly white filmy light about as bright as the full Moon Its beautiful rays are a sensational sight during an eclipse The coronas rays flash out in a brilliant fan that has wispy spikelike rays near the Suns north and south poles The corona is generally thickest at the Suns equator ~ The corona is made up of gases streaming outward at tremendous speeds that reach a temperature of more than 2 million degrees Fahrenheit The gas thins out as it reaches the space around the planets By the time the gas of the corona reaches the Earth it has a relatively low density

bull bullbullbullbullbullbull

92 CHAPTER 4 Standardized Testing

Click on the sentence in paragraph 4 or 5 in which the author compares the light of the Suns outermost layer to that ofanother astronomical body Paragraphs 4 and 5 are marked with arrows (~)

To answer some questions you will click on a square to add a sentence to the passage Here is an example -The following sentence can be added to paragraph 1

At the center of the Earths solar system lies the Sun

Where would it best fit in paragraph I Click on the square to add the sentence to the paragraph

D The temperature of the Sun is over 10000 degrees Fahrenheit at the surface but it rises to perhaps morethan 27000000deg at the center 0 The Sun is so much hotter than the Earth that matter can exist only as a gasi except p~rHapsatth~ c6relp the c~re of the ii Sun the pressures are so great that despite the high temperature there may be a small solid core D However no one really knows since the center of the Sun can never be directly observed D 0100

When you click on a square the sentence will appear in the passage at the place you have chosen You can read the sentence added to the paragraph to see if this is the best place to add it You can click on another square to change your answer The sentence will be added and shown in a dark box

Writing In this section you will have an opportunity to demonstrate your ability to write in Enshyglish This includes the ability to generate and organize ideas to support those ideas with examples or evidence and to compose in standard written English in response to an asshysigned topic You will have 30 minutes to write your essay on that topic You must write on the topic you are assigned An essay on any other topic will receive a score of 0 Read the topic below and then make any notes that will help you plan your response Begin typing your response in the box at the bottom of the screen or write your answer on the answer sheet provided to you

Following is a sample topic

Do you agree or disagree with the following statemenH

Teachers should make learning enjoyable and fun for their students

Use specific reasons and examples to support your opinion

CHAPTER 4 Standarczed Testing 93

Michigan English Language Assessment Battery (MELAB)

Composition The time limit for the composition is 30 minutes You must write on only one of the top~

ics below If you write about something else your composition paper will not be graded and you cannot be given a final score If you do not understand the topics ask the exam~ iner to explain or to translate them You may be asked to give your opinion ofsomething and explain why you believe this to describe something from your experience or to exshyplain a problem and offer possible solutions You should write at least one page Some sample topics are

1 What do you think is your countrys greatest problem Explain in detail and tell what you think can be done about it

2 What are the characteristics of a good teacher Explain and give examples 3 An optimist is someone who sees the good side of things A pessimist sees the

bad side Are you an optimist or a pessimist Relate a personal experience that shows this

4 In your opinion are the benefits of space exploration really worth the enormous costs Discuss

Most MELAB compositions are one or two pages long (about 200-300 words) If your paper is extremely short (less than 150 words) your composition will be given a lower score Before you begin writing you might want to take 2 or 3 minutes to plan your comshyposition and to make a short outline to organize your thoughts Such outlines will not be graded they are only to help you You should use the last 5 minutes to read through your composition and to make changes or corrections

Your composition will be graded on how clearly you express yourself in English and on the range of English you are able to use and your control in doing so This means your composition should be well organized your arguments should be fully developed and you should show a range ofgrammatical structures and broad vocabulary Compositions that consist only of very short sentences and very simple vocabulary cannot be given the

middothighest scores If errors are not frequent and if they do not confuse your meaning they will not lower your score very much

Listening Now you will hear a short lecture You may take notes during the lecture Following the lecture you will be asked some questions about it

Therell be a two-week exhibit of the paintings of the little-known master Laura Bernhart at the Claire Osmond Galleries starting on the fifteenth of the month and running through the thirtieth Bernharts known for her innovative designs in abstract expressionism Though a true original she declared a spiritual heritage from Salvador Dali the famous Spanish painter Since Bernhart lived a rather solitary life and died while only in her twenties few people are aware of her works This showing at the Osmond Galleries will provide many with an introduction to her works

10 Where is the exhibit a the Art Museum b the Dali Galleries c the Osmond Galleries

94 CHAPTER 4 Standardized Testing

11 What is Bernhart known for a her copies of Dalis paintings b the originality of her designs c her exhibitions

12 What will going to the exhibit allow most people to do a to see Saivador Dalis paintings b to see Bernharts works for the first time c to learn about Spanish art

Grammar

1 What did the teacher just tell you

She reminded our notebooksI a us to bring b that we bring c our bringing d we should bring

2 Is Bill a good dancer

Not really __ he tries very hard a in spite of h despite c even though d while

3 your clothes are all wet1

Yes I didnt come __ the rain soon enough a away to b over to c down with d in from

Cloze In years to come zoos will not only be places where animals are exhibited to the public but repositories where rare species can be saved from extinction (7) captive breeding The most powerful force (8) the future of many animals-and of zoos-is the decline of the wild (9) even zoo directors would argue that (10) are better places for animals than the fields and forest of their native (11) yet zoos may be the last chance for some creatures that would otherwise pass qUietly into oblivion

7 a through c from b of d damage

8 a bringing c to b that d influencing

9 a But c Not b So d Then

10 a where c even b zoos d wilds

11 alands c residence b life d field

CHAPTER 4 Standardized Testing 95

Vocabulary

12 Mark has a flair for writing a need b purpose c talent d dislike

13 Bill Collins launched his restaurant last June a moved b started c sold d bought

14 John will not accept the censure a burden b blame c credit d decision

15 I cant think of the answer Can you give me a __ a hint b token c taste d gaze

16 Because fewer people are taking expensive vacations the tourist industry is in a a choke b grope c grumble d slump

17 I disagree with a few of his opinions but __ we agree a deliberately b conclusively c essentially d immensely

Reading The influenza virus is a single molecule built from many millions of single atoms You must have heard of the viruses which are sometimes called living molecules While bacteria can be considered as a type of plant secreting pOisonous substances into the body of the organism they attack viruses are living organisms themselves We may conshysider them as regular chemical molecules since they have a strictly aefined atomic strucshyture but on the other hand we must also consider them as being alive since they are able to multiply in unlimited quantities

18 According to the passage bacteria are a poisons

b larger than viruses c very small d plants

96 CHAPTER 4 Standardized Testing

19 The writer says that viruses are alive because they a have a complex atomic structure b move c multiply d need warmth and light

20 The atomic structure of viruses a is -tJIariable b is strictly defined c cannot be analyzed chemically d is more complex than that of bacteria

International English Language Testing System (fELTS)

I

listening

The Listening Module has four sections The first two sections are concerned with social needs There is a conversation between two speakers and then a monologue For examshyple a conversation about travel arrangements or decisions on a night out and a speech about student services on a university campus or arrangements for meals during a confershyence The final two sections are concerned with situations related more closely to educashytional or training contexts For example conversation between a tutor and a student about an assignment or between three students planning a research project and a lecture or talk ofgeneral academic interest All the topics are ofgeneral interest and it makes no difference what subjects candidates study Tests and tasks become more difficult as the sections progress A range of English accents and dialects are used in the recording which reflects the international usage of IELTS

Academic Reading [A 7S0-word article on-th-e- topic of Wind Power in the US with a short glossary at the end]

Questions 1-5

Complete the summary below

Choose your answers from the box below the summary and write them in boxes 1-5 on your answer sheet Note There are more words or phrases than you will need to fill the gaps You may use any word or phrase more than once

Example The failure during the late 1970s and early 19805 of an attempt to establish a widespread wind power industry in the United States resulted largely from the (1) bull in oil prices during this period The industry is now experiencing a steady (2) due to improveshyments in technology and an increased awareness of the potential in the power of wind The wind turbines that are now being made based in part on the (3) of wide- ranging research in Europe are easier to manufacture and maintain than their predecesshysors This has led wind-turbine makers to be able to standardise and thus minimize (4) There has been growing (S) of the importance of wind power as an energy source

CHAPTER 4 Standardized Testing 97

criticism stability skepticism success operating costs decisions design costs fall effects production costs growth decline failure recognition results

Questions 6-1 0 Look at the following list of issues (Questions 6-10) and implications (A-C) Match each issue with one implication Write the appropriate letters A-C in boxes 6-10 on your anshyswer sheet

Example The current price of one wind-generated kilowatt Answer

6 The recent installation of systems taking advantage of economies of scale

7 The potential of meeting one fifth of current U5 energy requirements by wind power

8 The level of acceptance of current wind turbine technology

9 A comparison of costs between conventional and wind power sources

10 The view of wind power in the European Union

Implications

A provides evidence against claims that electricity produced from wind power is relatively expensive

B supports claims that wind power js an important source of energy

C opposes the view that wind power technology requires further-development

General Training Reading Read the passage on Daybreak trips by coach and look at the statements below On your answer sheet write

TRUE if the statement is true FALSE jf the statement is false

NOlGIVEN if the information is not given in the leaflet

1 MiIlers Coaches owns Cambridges Cam bus fleet

2 Premier is an older company than Millers

3 Most of the Daybreak coaches are less than 5 years old

4 Daybreak fares are more expensive than most of their competitors

5 Soft drinks and refreshments are served on most longer journeys

6 Smoking is permitted at the rear of the coach on longer journeys

7 Tickets must be bought in advance from an authorised Daybreak agent

6 Tickets and seats can be reserved by phoning the Daybreak Hotline

9 Daybreak passengers must join their coach at Cambridge Drummer Street

10 Daybreak cannot guarantee return times

98 CHAPTER 4 Standardized Testing

FROM CAMBRIDGE AND SURROUNDING AREA

SPRING IS INTHEAIR

Welcome to our Spring Daybreak programme which continues the tradition of offering unbeatable value for money day trips and tours All the excursions in this brochure will be operated by Pr~mier Travel Services Limited or Millers Coaches both companies are part of the CHLGroup owners of Cambridges Cambus fleet

WERE PROUD OF OUR TRADITION

Premier was established in 1936 the Company now offers the highest standards of coaching in todays competitive operating environment Miller has an enviable reputation stretching back over the past 20 years offering coach services at realistic prices Weve traveled a long way since our early days of pre-war seaside trips Now our fleet of 50 modern coaches (few are more than five years old) operate throughout Britain and Europe but were pleased to still maintain the high standards of quality and service the trademark of our founders nearly sixty years ago

EXCLUSIVE FEATURES

Admission-inclusive fares All Daybreak fares (unless specifically otherwise stated) include admission charges to the attractions shows and exhibits we visit Many full-day scenic tours are accompanied by a fully trained English Tourist Board Blue Badge guide or local experienced driverguide Some Daybreaks include lunch or afternoon tea Compare our admission inclusive fares and see how much you save Cheapest is not the best and value for money is guaranteed If you compare our bargain Daybreak fares beware--most of our competishytors do not offer an all-inclusive fare

SEAT RESERVATIONS

We value the freedom of choice so you can choose your seat when you book The seat reservation is guaranteed a-nd remains yours at all times when aboard the coach

NO SMOKING COMFORT

With the comfort of our passengers in mind coaches on all our Daybreaks are no smokshying throughout In the interests of fellow passengers comfort we kindly ask that smokers observe our no smoking policy On scenic tours and longer journeys ample refreshment stops are provided when of course smoking is permitted

YOUR QUESTIONS ANSWERED

Do I need to book Booking in advance is strongly recommended as all Daybreak tours are subject to demand Subject to availability stand-by tickets can be purchased from the driver

What ti me does the coach leave The coach departs from Cambridge Drummer Street (Bay 12 adjacent to public toilets) at the time shown There are many additional joining points indicated by departure codes in the brochure If you are joining at one of our less popular joining points you will be adshyvised of your pick-up time (normally by telephone) not less than 48 hours before deparshyture In this way we can minimize the length of pick-up routes and reduce journey times for the majority of passengers

CHAPTER 4 Standardized Testing 99

What time do we get back An approximate return time is shown for each excursion The tim~s shown serve as a guide but road conditions can sometimes cause delay If your arrival will be later than advertised your driver will try to allow for a telephone call during the return journey

Where can I board the coach All the Daybreaks in the brochure leave from Cambridge Drummer Street (Bay 12 adjashycent to public toilets) at the time shown Many Daybreaks offer additional pick-ups for pre-booked passengers within Cambridge and the surrounding area This facility must be requested at the time of booking

Academic Writing Writing Task 1 You should spend about 20 minutes on this task

The graph below shows the different modes of transport used to travel to and from work in one European city in 1950 1970 and 1990

[graph shown here]

Write a report for a university lecturer describing the information shown below You should write at least 150 words

Writing Task 2 You should spend about 40 minutes on this task

Present a written argument or case to an educated reader with no specialist knowledge of the folowing topic

It is inevitable that as technology develops so traditional cultures must be lost Technolshyogy and tradition are incompatible-you cannot have both together

To what extent do you agree or disagree with this statement Give reasons for your answer You should write at least 250 words You should use your own ideas knowlshyedge and experience and support your arguments with examples and relevant evidence

General Training Writing Writing Task 1 You should spend about 20 minutes on this task You rent a house through an agency The heating system has stopped working You phoned the agency a week ago but it has still not been mended Write a letter to the agency Explain the situation and teil them what you want them to do about it

You should write at least 150 words You do NOT need to write your own address

Begin your letter as follows

Dear - ___-I

Writing Task 2 You should spend about 40 minutes on this task As part ofa class assignment you have to write about the following topic

100 CHAPTER 4 Standardized Testing

Some businesses now say that no one can smoke cigarettes in any of their offices Some governments have banned smoking in all public placesThis is a good idea but it takes away some of our freedom

Do you agree or disagree Give reasons for your answer You should write at least 250 words

Speaking In each ofthe three parts of the speaking module a specific function is fulfilled In Part 1 the candidates answer general questions about themselves their homes or families their jobs or studies their interests and a range ofsimilar familiar topic areas This part lasts between four and five minutes In Part 2 the candidate is given a verbal prompt on a card and is asked to talk on a particular topic The candidate has one minute to prepare before speaking at length for between one and two minutes The examiner then asks one or two wind-down questions In Part 3 the examiner and candidate engage in a discusshysion of more abstract issues and concepts which are thematically linked to the topic prompt in Part 2 The discussion lasts between four and five minutes

All interviews are recorded on audiocassette Here is a sample ofa Part 2 topic

Describe a teacher who has greatly influenced you in your education

You shou Id say

where you met them what subject they taught what was special about them

and explain why this person influenced you so much

You will have to talk about the topic for 1 to 2 minutes You have 1 minute to think about what you are going to say You can make some notes if you wish

Test of English for International Communication (TOEICreg)

listening

Part 1 Photographs Directions For each question you will see a picture in your test book and you will hear four short statements The statements will be spoken just one time They will not be printed in your test book so you must listen carefully to understand what the speaker says When you hear the four statements look at the picture in your test book and choose the statement that best describes what you see in the picture Then on your answer sheet find the number of the question and mark your answer

[photograph of a scientist looking through a microscope]

You will hear Look at the picture marked number 1 in your test book

(A) Shes speaking into a microphone (B) Shes put on her glasses (C) She has both eyes open (D) Shes using a microscope

CHAPTER 4 Standardized Testing 101

Part 2 Question-Response Directions In this part of the test you will hear a question or statement spoken in Enshyglish followed by three responses also spoken in English The question or staten1ent and the responses will be spoken just one time They will not be printed in your test book so you must listen carefully to understand what the speakers say You are to choose the best response to each question or statement

Question 1 You will hear Ms Morikawa has worked here for a long time hasnt she

(A) At three oclock (B) No Ive lost my watch (C) More than ten years

Question 2 You will hear Which of these papers has a wider circulation

(A) The morning edition (B) Get more exercise (C) By messenger

Part 3 Short Conversations Directions In this part of the test you will hear short conversations between two people The conversations will not be printed in your test book You will hear the conversations only once so you must listen carefully to understand what the speakers say In your test book you will read a question about each conversation The question will be followed by four answers You are to choose the best answer to each question and mark it on your answer sheet

Question 1 (Man) We should think about finding another restaurant for lunch (Woman) Why The food and service here are great

(Man) Yes but the prices are going up every week

You will read Why is this man unhappy with the restaurant

(A) It is too noisy (B) It is too expensive (C) It is too crowded (D) It is too difficult to find

Question 2 (Woman A) How was Dr Borgs recent trip to Singapore (Woman B) She enjoyed the tour of the port very much (Woman A) They say its one of the most active in Asia

You will read 2 What did Dr Borg find interesting

(A) The tourist center (B) The airport (C) The musical performance (D) The harbor

Part 4 Short Talks Directions In this part of the test you vill hear several short talks Each will be spoken just one time They will not be printed in your test book so you must listen carefully to understand and remember what is said In your test book you will read two or more questions about each short talk The questions will be followed by four answers You are to choose the best answer to each question and mark it on your answer sheet

102 CHAPTER 4 Standardized Testing

You will hear Questions 1 and 2 refer to the following announcement

Good afternoon and welcome aboard Nordair Flight 857 from Copenhagen to Bangkok with intermediate stops in Dubai and Calcutta We are preparing for departure in a few minutes At this time your seat back should be returned to its full upright position and your seat belt s~ould be fastened OUf anticipated total flying time to Dubai is six hours and twenty-five minutes I hope you enjoy the flight You will hecJr Now read question 1 in your test book and answer it You will read 1 What is the final destination of the flight

(A) Bangkok (B) Copenhagen (C) Dubai (O) Calcutta

You will hear Now read question 2 in your test book and answer it You will read 2 What will happen in a few minutes

(A) The flight will land in Dubai I

(B) The passengers will board the plane (C) The plane will take off (0) The gate number will be announced

Reading In this section of the test you will have the chance to show how well you understand written English There are three parts to this section with special directions for each part

Part 4 Incomplete Sentences Directions This part of the test has incomplete sentences Four words or phrases marked (A) (8) (e) (D) are given beneath each sentence You are to choose the one word or phrase that best completes the sentence Then on your answer sheet find the number of the question and mark your answer

1 Mr Yangs trip will __ him away from the office for ten days (A) withdraw (B) continue (C) retain (0) keep

2 The company that Marie DuBois started now sells __ products throughout the world (A) its (B) it (C) theirs (D) them

3 If your shipment is not delivered __ Tuesday you can request a full refund for the merchandise (A) at (B) by (C) within (D) while

CHAPTER 4 Standardized Testing 103

Part 6 Error Recognition Directions In this part ofthe test each sentence has four words or phrases underlined The four underlined parts of the sentence are marked (A) (B) (C) (D) You are to identify the one underlined word or phrase that should be corrected or rewritten Then on your answer sheet find the number of the question and mark your answer

1 The pamphlet contains some importance information about the current exhibit ABC D

2 No matter how Jong it taking to finish the annual report it must be done properly ABC D

3 The popularity of jogging appears to have decreased since the past couple of years ABC D

Part 7 Reading Comprehension Directions The questions in this part of the test are based on a selection of reading mateshyrials such as notices letters) forms newspaper and magazine articles) and advertisements You are to choose the one best answer (A) (B) (C) or (OJ to each quesshytion Then on your ariswefsheelfindthe number of the qUestion andmcirkyour answer Answer all questions following each reading selection on thebasis of what is stated or implied in that selection

The Museum ofTechnology is a hands-on museum designed for people to experience science at w()rk~ Visitors are encouraged to use test and handle the objects o~ display Special demonstrations are scheduled for the first and second Wednesdays of each month at 1330 Open Tuesday-Friday 1200-1630 Saturday 1000-1730 and Sunday 11 00-1630

1 When during the month can visitors see special demonstrations (A) Every weekend (B) The first two Wednesdays (C) One afternoon a week (D) Every other Wednesday

Questions 2 and 3 refer to the followi ng notice

NOTICE If you are unable to work because of an extended illness or injury that is not workshyrelated you may be entitled to receive weekly benefits from your employer or the firms insurance company To claim benefits you must file a claim form within thirty days of the first day of your disability Before filing the claim you must ask your doctor to fill in the Doctors Statement on the claim form stating the period of disability

3 To whom is this notice addressed (A) Employers (8) Doctors (C) Employees (D) When paying the bill

4 When must the claim form be filed (A) On the first of the month (8) On the thirtieth of the month (C) On the first day ofdisabifity (D) Within 30 days of the start of disability

Page 3: Standardized Testing Chapter 4 Brown

68 CHAPTER 4 Standardized Testing

ADVANTAGES AND DISADVANTAGES OF STANDARDIZED TESTS

Advantages of standardized testing include foremost a re~gY-=1ade previously valishydated product that frees the teacher from having to spe~d hours creating a test Administration to large groups can be accomplished within reasonable time limits In the case ofmiddotinultiple-choice formats scoring procedures are streamlined (for either scal18Qble computerized scoring or hand-SCOring with a hole-punched grid) for fast turnaround time And for better or for worse there is often an air of face validity to such authoritative-looking instruments

Disadvantages center largely on the inp-Jl9priate use of such tests for example using an_QY~1llpr9tl~i~tl9_~est as an achievemenftesfsUnplY because of the convenience of the standardization A colleague told me-about a course director who after a frantic search for a last-minute placement test administered a multipleshychoice granunar achievement test even though the curriculum was mostly listening and speaking and involved few of the granunar points tested This instrunrent had the appearance and face validity of a good test when in reality it had no -content Validity whatsoever

Another disadvantage is the potential misunderstanding of the difference between directandindirect testing (see Chapter 2) Some standardized tests include tasks that do__-2QJrectly specify Q~~~~_~ the target r objpound~tive For example before 1996 the TOEFL included neither a written nor an oral production section yet statistics showed a reasonably strong correspondence between performance on the TOEFL and a students written and-to a lesser extent-oral production The comprehension-based TOEFL could therefore be claimed to be an indirect test of production A test of reading comprehension that proposes to me~e ability to read extensively and that engages test-takers in reading only short one or twoshyparagraph passages is anJndirect measure of extensive reading

Those who use standardized tests need to acknowledge both the advantages and limitations of indirect testing In the pre1996 TOEFL administrations the expense of giving a direct test of production was considerably reduced by offering only comprehension performance and showing through construct validation the appropriateness of conclusions about a test-takers production competence likewise short reading passages are easier to administer and if research validates the assumption that short reading passages indicate extensive reading ability then the use of the shorter passages is justified Yet the construct validation statistics that offer that support never offer a 100 percent probability of the relationship leaving room for some possibility that the indirect test is not valid for its targeted use

A more serious isslle lies in the assumption (alluded to above) that standardized tests correctly assess all learners equally well Wellestablished standardized tests usually demonstrate high correlations between performance on such tests and target objectives but correlations are not sufficient to demonstrate unequivocally the acquisition of criterion objectives by all test-takers Here is a non-language example In the United States some driverS iicense renewals require taking a papershyand-pencil multiple-choice test that covers signs safe speeds and distances lane

CHAPTER 4 Standardized Testingmiddot 69

changesand otherrules of the road Correlational statistics show a strong relationship between high scores on those tests and good driving records so people who do well on these tests are a safe bet to relicense Now an extremely high correlation (of pershyhaps SO or above) may be loosely interpreted to mean that a large majority of the drishyvers whose licenses are renewed by virtue of their having passed the little quiz are good behind-the-wheel drivers What about those few who do not fit the model That small minority of drivers could endanger the lives of the majority and is that a risk worth taking Motor vehicle registration departments in the United States seem to think so and thus avoid the high cost of behind-the-wheel driving tests

Are you willing to rely on a standardized test result in the case of all the learners in your class Of an applicant to your institution or of a potential degree candidate exiting your program These questions will be addressed more fully in Chapter 5 but for the moment think carefully about what has come to be known

as high-stakes testing in which standardized tests have become the only criterion for inclusion or exclusion The widespread acceptance and sometime misuse of this gate-keeping role of the testing industry has created a political educational and moral maelstrom

DEVELOPING A STANDARDIZED TEST

Whlle it is not likely that a classroom teacher with a team of test designers and researchers would be in a position to ~evelop a brand-new standardized test of large-scale proportions it is a yirtual certainty that some day you will be in a posishytion (a) to revise an existing test (b) toadapt or expand an existing test andor (c) to create a smaller-scale standardized test for a program you are teaching in And even if none of the above thre~ cases should ever apply to you it is of paramount importance to understand the process of the development of the standardized tests that have become ingrained in our educational institutions

How are standardized tests developed Where do test tasks and items come from How are they evaluated Who selects items and their arrangement in a test How do such items and tests- achieve consequential validity How are different forms of t~sts equated for diftlculty level Who sets norms and cutoff limits Are security add confidentiality an issue Are cultural and racial biases an issue in test development All these questions typify those that you might pose in an attempt to understand the process of test development

In the steps outlined below three different standardized- tests will be used to exemplify the process of standardized test design

(A) The Test of English as a Foreign Language (TOEFL) Educational Testing Service (ETS) (B) The English as a Second Language Placement Test (ESLP1) San Francisco State University (SFSU) (C)The Graduate Essay Test (Gm SFSU

70 CHAPTER 4 Standardized Testing

The first is a test of general language ability or profidencyThe second is a place ment test at a universityAnd the third is a gate-keeping essay test that all prospecshytive students must pass in order to take graduate-level courses As we look at the steps one by one you will see patterns that are consistent with those outlined in the previous two chapters for evaluating and developing a classroom test

1 Determine ihe purpose and objectives of the test

Most standardized tests are expected to provide high practicality in administration and scoring without unduly compromising validity The initial outlay of time and money for such a test is Significant but the test would be used repeatedly It is therefore imporshytant for its purpose and objectives to be stated specifically Lets look at the three tests

(A) The purpose of the TOEFL is to evaluate the English profiCiency of people whose native language is not English (TOEFL Test and Score Manual 2001 p 9) More specifically theTOEFL is designed to help institutions of higher learning make valid decisions concerning English language profiCiency lin terms of lthelr] own requirements (p 9) Most colleges and universities in the United States use TOEFL scores to admit or refuse international applicants for admission Various cut-off scores apply but most institutions require scores from 475 to 525 (paper-based) or from 150 to 195 (computer-based) in order to consider students for admissionThe high-stakes gate-keeping nature of the TOEFL is obvious

(B) The ESLPT referred to in Chapter 3 is designed to place already admitted students at San Francisco State University in an appropriate course in academic writing with the secondary goal of placing students into courses in oral production and grammar-editing While the tests primary purpose is to make placements another desirable objective is to provide teachers with some diagnostic information about their students on the first day or two of classThe ESLPT is locally designed by university faculty and staff

(C) The GEfanother test designed at SFSU is given to prospective graduate students-both native and non-native speakers-in all disciplines to determine whether their writing ability is sufficient to permit them to enter graduate-level courses in their programs It is offered at the beginning of each term Students who fail or marginally pass the GET are technically ineligible to take graduate courses in their field Instead they may elect to take a course in graduate-level writing of research papers A pass in that course is equivalent to passing the GET

As you can see the objectives of each of these tests are specific The content of each test must be designed to accomplish those particular ends TIlis first stage of goal-setting might be seen as one in which the consequential validity of the test is foreshymost in the mind of the developer each test has a specific gate-keeping function to perfonn therefore the criteria for entering those gates must be specified ~ccurately

2 Design test specifications

Now comes the hard part Decisions need to be made on how to go about structurshying the specifications of the test Before specs can be addressed a comprehensive

CHAPTER4 Standardized Testing 71

progratn of research must identify a set of constructs underlying the test itself This stage of laying the foundation stones can occupy weeks months or even years of effort Standardized tests that dont work are often the product of short-sighted conshystruct validation Lets look at the three tests again

(A) Construct validation for the TOEFL is carried out by the TOEFL staff at ETS under the guidance of a Policy Counell that works with a Committee of Examiners that is composed of appointed external university faculty linguists and assessment spedaJists Dozens of employees are involved in a complex process of reviewing curshyrentTOEFL specifications cOmmissioning and developing test tasks and items assemshybling forms of the test and performing ongoing exploratory research related to formulating new specs Reducing such a complex process to a set ofsimple steps runs the risk of gross overgeneralization but here is an idea of how aTOEFL is created

Because the TOEFL ismiddot a proficiency test the fIrst step in the developmental process is to define the construct of language proficiency First it should be made clear that many assessment specialists such as Bachman (1990) and Palmer (Bachman amp Palmer 1996) prefer the term ability to proficiency and thus speak of language abllity as the overarching concept The latter phrase is more conSistent they argue with our understanding that the specific components of language ability must be assessed separately Others such as the American Council on Teaching Foreign Languages (ACIFL) still prefer the term proficiency because it connotes more of a holistic unitary trait view of language ability (Lowe 1988) Most current views accept the ability argument and therefore strive to specify and assess the many components of language For the purposes of consistency in this book the term proficiency will nevertheless be retained with the above caveat

How you view language will make a difference in how you assess language proshyficiency After breaking language competence down into subsets of listening speaking reading and writing eapoundli~p~ifQ(m~I~~ Jll()4~ can be examined on a conshytiiiuum of linguistic units pho~~logy (p~onunciation) and orthography (spelling) words OeXicon) sentences (gtammar) discourse and pragmatic (sociolinguistic contextual functional cul~j features of language

How will the TOEFL sample from at) these possibilities Oral production tests can be tests of overall conversational fluency or pronunciation of a particular subset of phonolOgy and can take the form of imitation structured responses or free responses Listening comprehension tests can concentrate on a particular feature of language or on overalllistenins for general meaning Tests of realtling can cover the range of language units and can aim to test comprehension of long or short passhysages single sentences or even phrases and words Writing-tests can take on an open-ended form with free composition or be structured to elicit anything from correct spelling to discourse-level competence Are you overwhelmed yet

From the sea of potential performance modes that could be sampled in a test the developer must select a subset on some systematic basis To make a very long story short (and leaving out numerous controversies) the TOEFL had for many years included three types of performance in its organizationai specifications listening strucshyture and reading all of vtuch tested comprehension through standard multiple-choice

72 CHAPTER 4 Standardized Testing

tasks In 1996 a major step was taken to include written production in themiddot computershybased TOEFL by adding a slightly modified version of the already existing Test of Written English (TWE) In doing so some face validity and content validity were improved along with of course a significant increase in administrative expense Each of these four major sections is capsulized in the box below (adapted from the descripshytion of the current computer-based TOEFL at wwwroefLorg) Such descriptions are not strictly speaking specifications which are kept confidential by ETS Nevertheless they can give a sense of many of the constraints that are placed on themiddot design of actual TOEFL specifications

TOEFLlt8gt specifications

Listening Section The listening section measures the examinees ability to understand English as it is spoken in North America Conversational features of the language are bull stressed and the skills tested include vocabulary and idiomatic expression as well as speshycial grammatical constructions that are frequently used in spoken Engfish The stimulus material and questions are recorded in standard North American English

The listening section includes various stimuli such as dialogues short conversations academic discussions and mini-lectures and poses questions that test comprehension of main ideas the order ofa process supporting ideas important details and inferences as well as the ability to categorize topicsobjects

The test developers have taken advantage of the multimedia capability of the computer by using photos and graphics to create context and support the content of the lectures producing stimuli that more closely approximate Ureal-world situations in which people do more than just listen to voices The listening stimuli are often accompashynied by either context-setting or content-based visuals All dialogues conversations acashydemic discussions and mini-lectures include context visuals to establish the setting and role of the speakers Content-based visuals are often used to complement th~ topics of the mini-lectures

Structure-Section Themiddotstructure-section measures an examinees ability to recognize language that is appropriate for standard written English The language tested is formal rather than conversational The topics of the sentences are associated with general acadeshymic discourse so that individuals in specific fields of study or from specific national or linguistic groups have no particular advantage

Two types of questions are used questions in which examinees must (1) complete an incomplete sentence using one of four answers provided and (2) identify one of four unshyderlined words or phrases that would not be accepted in English The two question types are mixed randomly rather than being separated into two subsections as in the papershybased TOEFL test

Reading Section The reading section measures the ability to read and understand short passages similar in topic and style to academic texts used in North American colshyleges and universities Examinees read a variety of short passages on academic subjects and answer several questions about each passage Test items refer to what is stated or imshyplied in the passage as well as to words used in the passage To avoid creating an advanshyt~ t( 1 dull5- In 3n~ ont fidd of stud~ sufficient context is provided SO that no ~ -middott - ~tmiddot J ~ middoth - -~ rM~ ir~ to ~-er the if~(J(lS~ ~ t ~ ~ ~~~~-- Ih L_ 1_ 1 ~_

CHAPTER 4 Standardized Testing 73

The reading section consists of four to five passages of 250-350 words with 10-14 questions per passage This section is not computer-adaptive soexaminees can skip questions and return to previous questions The questions in this section assess the comshyprehension of main ideas inferences factual information stated in a passage pronoun referents and vocabulary (direct meaning synonym antonym) In all cases the questions can be answered by reading and understanding the passages This section consists of (1 ) traditional multiple-choice questions (2) questions that require examinees to click on a word phrase sentence or paragraph to answer and (3) questions that ask examinees to insert a sentence where it fits best

Writing Section The writing section measures the ability to write in English including the ability to generate organize and develop ideas to support those ideas with examples or evidence and to compose a response to one assigf)ed topic in standard written Enshyglish Because some examinees may not be accustomed to composing an essay on comshyputer they are given the choice of handwriting or typing the essay in the 30-minute time limit The rating scale for scoring the essay ranging from 0 to 6 is virtually the same as that of the Test of Written English [see Chapter 9 of this book] A score of 0 is given to papers that are blank simply copy the topic are written ina language other than English consist only of random keystroke characters or are written on a topic different from the one assigned

Each essay is rated independently by two trained certified readers Neither reader knows the rating assigned by the other An essay will receive the average of the two ratshyings unless there is a discrepancy of more than one point in that case a third reader will independently rate the essay The essay rating is incorporated into the StructureMriting scaled score~ and constitutes approximately 50 percent of that combined score

(B) The designing of the test specs for the ESLPT was a somewhat simpler task because the purpose is placement and the construct validation of the test consisted of an examination of the content of the ESL courses In fact in a recent revisiofi of the ESLPT (lmao et al 2000 Imao 2001) content validity (coupled with its attenshydant face validity) was the central theoretical issue to be considered The major issue centered on designing practical and reliable tasks and item response formats Having established the importance of designingESLPT tasks that simulated classroom tasks used in the courses the designers ultimately specified two writing production tasks (one a response to an essay that students read and the other a summary of another essay) and one multiple-choice grammar-editing taskThese specifications mirrored the readingbased process writing approach used in the courses

(C) Specifications for the GET arose out of the perceived need to provide a threshold of acceptable writing ability for all prospective graduate students at SFSU both native and non-native speakers of EnglishThe specifications for the GET are the skills of writing grammatically and rhetorically acceptable prose on a topic of some interest with clearly produced organization of ideas and logical development The GET is a direct test of writing ability in which test-takers must in a two-hour time period write an essay on a given topic

74 CHAPTER 4 Standardized Testing

3 Design select and arrange test tasksitems

Once specifications for a standardized test have been stipulated the sometimes never-ending task of designing selecting and arranging items beginS The spe~s act much like a blueprint in determining the number and types of items to be created Lets look at the three examples

(A) TOEFL test design specifies that each item be coded for content and statisshytical characteristics Content coding ensures that each examinee will receive test questions that assess a variety of skills (reading comprehending the main idea or understanding inferences) and cover a variety of subject matter without unduly biasing the content toward a subset of test-takers (for example in the listening secshytion involving an academic lecture the content must be universal enough for stushydents from many different academic fields of study) Statistical characteristics including the IRT equivalents of estimates of item ~~ility (IF) and the ability of an item to discriminate (ID) between higher orlower ability levels ate also coded

Items are then designed by a team who select and adapt items solicited from a bank of items that have been deposited by freemiddotlance writers and ErS staff Probes for the reading section for example are usually excerpts from authentic general or academic reading that are edited for linguistic difficulty culture bias or other topic biases Items are designed to test overall comprehension certain specific informashytion and inference

Consider the following sample of a reading selection and ten items based on it from a practice TOEFL (Phillips 2001pp423-424)

For hundreds of years in the early history of America pirates sailed through coastal washyters pillaging and plundering all in their path They stole from other ships andstole from coastal towns not content only to steal they destroyed everything they could not carry avay~ Some of the pirate ships amassed large treasures~ the fates of which are unknown leaving people of today to wonder at their whereabouts and to dream of one day coming across some lost treasure

One notoriously large treasure was on the pirate ship Whidah which sank in the washyters off Cape Cod during a strong storm in 1717 A hundred of the crew members went down with the ship along with its treasure of coins gold silver and jewels The treasure on board had an estimated value on todays market of more than 100 million dollars

The remains of the Whidah were discovered in 1984 by Barry Clifford who had spent years of painstaking research and tireless searching only finally to locate the ship about 500 yards from shore A considerable amount of treasure from the centuries-old ship has been recovered from its watery grave but there is clearly still a lot more out there Just as a reminder of what the waters off the coast have been protecting for hundreds of years occasional pieces of gold or silver or jewels still wash up on the beaches and lucky beach-goers find pieces of the treasure

11 ~ Thepa~e mainly diccus5eS

CH4PTER 4 Standardized Testing 75

(e) what really happened to the Whidahs pirates (D) why people go to the beach

12 It is NOT mentioned in the passage that pirates did which of the following (A) They killed lots of people (B) They robbed other ships (e) They took things from towns (D) They gathered big treasures

13 The word amassed in line 4 is closest in meaning to (A) sold (e) transported (B) hid (D) gathered

14 It is implied in the passage that the Whidahs crew (A) died (B) went diving (e) searched for the treasure (D) escaped with parts of the treasure

15 Which of the following is NOT mentioned as part of the treasure of the Whidah (A) Art objects (B) Coins (e) Gold and si Iver (D) Jewels

16 The word estimated in line 10 is closest in meaning to which of the following (A) Known (C) Approximate (B) Sold (D) Decided

17 The passage indicates that the cargo of theWhidah is worth about (A) $100000 (B) $1000000 (C) $10000000 (D) $100000000

18 The work that Barry Clifford did to locate the Whidah was NOT (A) successfu I (B) effortless (C) detailed (D) lengthy

19 It is mentioned in the passage that the treasure of the Whidah (A) is not very valuable (8) is all in museums (C) has not all been found (D) was taken to share by the pi rates

20 The paragraph following the passage most likely discusses (A) what Barry Clifford is doing today (8) the fate of the Whidahs crew (e) other storms in the area of Cape Cod (D) additional pieces that turn up from the Whidahs treasure

76 CHAPTER 4 Standardized Testing

As you can see items target the assessment of comprehension of the main idea (item 11) stated details (17 19) unstated details (12 15 18) implied details (14 20) and vocabulary in context (13 16) An argument could be made about the cultural schemata implied in a passage about pirate ships and you could engage in an angels on the head of a pin argument about the importance of picking cershytain vocabulary for emphasis but every test item is a sample of a larger domain and each of these fulfills its designated specification

Before any such items are released into a form of the TOEFL (or any validated standardized test) they are piloted and sCientifically selected to meet difficulty specshyifications within each subsection section and the test overall Furthermore those items are also selected to meet a desired discrimination index Both of these indices are important considerations in the design of a computer-adaptive test where pershyformance on one item determines the next one to be presented to the test-taker (See Chapter 3 for a complete treatment of multiple-choice item design)

(B)The selection of items in the ESLPT entailed two-entirel) different processes In the two subsections of -the test that elicit writing performance (summary of reading response to reading) the main hurdles were (a) selecting appropriate passhysages for test-takers to read (b) providing appropriate prompts and (c) processing data from pilot testing Passages have to conform to standards of content validity by being within the genre and the difficulty of the material used in the courses The prompt in each case (the section asking for a summary and the section asking for a response) has to be tailOred to fit the passage but a general template is used

[n the multiple-choice editing test that seeks to test grammar proofreading ability the first and easier task is to choose an appropriate essay within which to embed errors The more complicated task is to embed a specified number of errors from a previously determined taxonomy of error categories Those error categories came directly from student errors as perceived by their teachers (verb tenses verb agreeshyment logical connectors articles etc) The disttactors for each item were selected from actual errors that students make Itemsiti pilot versions were then coded fordifshyficulty and discrinlination indices after which final assembly of items could occur

(C) The GET prompts are designed by a faculty committee of examiners who are speCialists in the field of university academic writing The assumption is made that the topics are universally appealing and capable of yielding the intended product of an essay that requires an organized logical argument and conclusion No pilot testing of prompts is conducted The conditions for administration remain constant two-hour time limit sit-down context paper and pencil closed-book format Consider the following recent prompt

Graduate Essay Test sample prompt

In the Middletown Elementary School District the assistant superintendent has just been made superintendent in another district Her resignation leaves vacant the districts only administrative position ever held by a woman The School Board in response to strong

CHAPTER 4 Standardized Testing 77

arguments from the Teachers Association has urged that a woman be hired to replace her As a member of the hiring committee you must help choose her successor

Only one woman applicant meets the written qualifications for the job the two top male applicants are both more experienced than she

The hiring committee has asked each committee member to prepare a written statement to distribute before meeting together to discuss the issue Write a report that represents your position making it as logical and persuasive as possible

Some facts you may wish to draw on 1 Women make up more than 75 percent of classroom teachers but hold fewer than

10 percent of administrative positions in education Administrators salaries average 30 percent more than teachers salaries

2 The local Teachers Association is 89 percent women mostly under 40 In a heated debate on television a member of the National Organization of Women (NOW) and the chair of the Teachers Association threatened if a man is hired to bring a class-action suit against the district on behalf of all women teachers who cannot expect advancement because of discriminatory hiring practices

3 The local Lions Club which contributes heavily to school sports says hiring the less experienced woman would not be in the best interests of the schoolthe children or the teachers

The finalists for the position

1 Carole Gates Classroom teacher 10 years Teacher of the Year 1985 supervisor ofpractice teachers at Teachers College former president of Teachers Associ ati on Administrative Credential 1984 EdD degree 1986 assistant principal of Hoptown Elementary School 2 years

2 Spud Stonewall Principal of Middletown Elementary 15 years PhD in educational adminis~ration State Board of Education Committee for Improving Elementary School Curriculum 1982-present

3 Jim Henderson School Administrator 22 yearsgradesK-9-supports innovation in education Fair Bargaining Award 1981 former coach for winning collegiate basketball team 10 years

It is clear from such a prompt that the problem the test-takers must address is complex that there is sufficient information here for writing an essay and that testshytakers will be reasonably challenged to write a clear statement of opinion What also emerges from this prompt (and virtually any prompt that one might propose) is the potential cultural effect on the numerous international students who must take the GIIT Is it possible that such students who are not familiar with school systems in the United States with hiring procedures and perhaps with the politics of school board elections might be at a disadvantage in mounting their arguments within a two-hour time frame Some (such as Hosoya 2001) have strongly claimed such a bias

78 CHAPTER 4 Standardized Testing

4 Make appropriate evaluations of different kinds of items

In Chapter 3 the concepts of item facility (IF) item discrimination (ID) and disshytractor analysis were introduced As the discussion there showed such calculations provide useful infornlation for classroom tests but sometimes the time and effort involved in perfornling them may not be practical especially if the classroom-based test is a one-time test Yet for a standardized multiple-choice test that is designed to be marketed commercially andor administered a number of times andor adminisshytered in a different form these indices are a must

For other types of response formats namely production responses different forms of evaluation become importantThe principles of p-mpoundti~ality ~d poundabWty are prominent along with the concept o(JacjJjt Practicality issues in such items include the clarity of directions timing of the test ease of administration and how much time is required to score responses Reliability is a major player in instances where more than one scorer is employed and to a lesser extent when a single scorer has to evaluate tests over long spans of time that could lead to deterioration of stanshydards Facility is also a key to the validity and success of an item type ~irecshytions complex- language obscure topics fuz~Qata and culturally biased

~Jfiformatioifma~alliead to a highei1eVermiddotof diffiCidty than one desires (A) The IF ID and efficiency statistics of the multiple-choice items of current

forms of the TOEFL are not publicly available information For reasons of security and protection of patented copyrighted materials they must remain behind the closed doors of the ETS development staff Those statistics remain of paramount importance in the ongoing production ofTOEFL items and forms and are the founshydation stones for demonstrating the equatability of forms Statistical indices on retired forms of the TOEFL are available on request for research purposes

The essay portion of theTOEFL undergoes scrutiny for its practicality reliability and facility Special attention is given to reliabilIty since two human scorers must read each essay and every time a third reader becomes necessary (when the two readers disagree by more than one point) it costs ETS more money

(B) In the case of the open-ended responses on the two written tasks on the ESLPT a similar set of judgments must be made Some evaluative impressions of the effectiveness of prompts and passages are gained from informal student and scorer feedback In the developmental stage of the newly revised ESLPT both types of feedshyback were formally solicited through questiQnnaires and interviews That informashytion proved to be invaluable in the revisIon of prompts and stimulus reading passages After each administration now the teacher-scorers provide informal feedshyback on their perceptions of the effectiveness of the prompts and readings

The multiple-choice editing passage showed the value of statistical findings in determining the usefulness of items and pointing administrators toward revisions Following is a sample of the format used

CHAPTER 4- Standardized Testing 79

Multiple-choice editing passage

(1)EYer since supermarkets first appeared they have beentake over ~ world ABC 0

(2) Supermarkets have changed peoples life ~ yet and at the same time changes in ABC

peoples life ~ have encouraged the opening of supermarkets o

The task was to locate the error in each sentence Statistical tests on the experishymental version of this section revealed that a number of the 45 items were found to be of zero IF (no difficulty whatsoever) and of inconsequential discrimination power (some IDs of 15 and lower) Many distractors were of no consequence because they lured no one Such information led to a revision of numerous it~ms and their options eventually strengthening the effectiveness of this section

(C)The GET like its written counterparts in the ESLPT is a test ofwritten ability with a single prompt and therefore questions of practicality and J~~illy~are also largely observational No data are collected from students on their perceptions but the scorers have an opportunity to reflect on the validity ofa given topiC After one sitting a topic is retired which eliininates the possibility of improving a specific topiC but future framing of topics might benefit from scorers evaluations Inter-rater reliability is checked periodically and reader training sessions are modified if too many instances of unreliability appear

5 Specify scoring procedures and reporting formats - ---

A systematic assembly of test items in pre-selected arrangements and sequences all of which are validated to confo~ to an e~pected difficulty level should yield a test that can then be scored accurately and reported back to test-takers and institutions efficiently

(A) Of the three tests being exemplifled here the most straightforward scoring procedure comes from the TO~FL the one with the most complex issues of validashytion deSign and assembly Scores are calculated and reported fora) three sections of the TOEFL (the essay ratings are combined with the Structure and Written Expression score) and (b) a total score (range 40 to 300 on the computer-based TOEFL and 310 to 677 on the paper-and-pencil TOEFL) A separate score (c) for the Essay (range 0 to 6) is also provided on the examinees score record (see simulation of a score record on page 80)

80 CHAPTER 4 Standardized Testing

Facsimile of a TOEFLreg score report

TOEFL Scaled Scores Claudia Y Estudiante Peru ___

19 17 17 177 Listening Structure Writing Reading Total Score

Essay rati ng 30

The rating scale for the essay is virtually the same one that is used for the Test of Written English (see Chapter 9 for details) with a zero level added for no response copying the topic only writing completely off topic or not writing in English

(B) The ESLPT reports a score for each of themiddot essay sections but the rating scale differs between them because in one case the objective is to write a summary and in the other to write a response to a reading ~ch essayi~pd lgtY ~o readet~ ifhFfF js a discrepancy of more than one level a third reader1resolves the differenceThe ~ditiilg section is machine-scanned and -scored with a total score and ~th part-scores for each ofthe grammaticaVrhetorlcal sectionS From these data placement administrators have adequate information to make placements and teachers receive some diagnostic inforshymation on each student in their classes Students do not receive their essays back

(C) Each GET is read by two trained readers who give a score between 1 and 4 according to the following scale

Graduate Essay Test Scoring Guide

Please make no marks on the writers work Write your reader number and score on the front cover of each test booklet

4 Superior The opening establishes context purpose and point of view the body of the essay developsmiddot recommendations-logically and coherently The writer demonshystrates awareness of the complexities in the situation and provides analysis of the probJem offers compelling or common-sense reasons for recommendations made makes underlying assumptions explicit

The writer uses fluent and idiomatic English with few mechanical errors Style reshyveals syntactic maturity is dear and direct is not choppy or over-colloquial nor over-formal stuffy or unfocused Occasional spelling or punctuation errors may be easily attributed to hasty transcription under pressure

3 Competent After an opening that establishes context and purpose the paper unfolds with few lapses in coherence but may have somewhat less clear organization of less explicit transitions than a top-score paper It may have somewhat less compelling logic or slightly less-wellreasoned suggestions than a 4 paper though it will provide reasons for the recommendations made

The writer uses dear fluent and generally idiomatic English but may make minor or infrequent ESL errors (preposition errors dropped articles or verb endings etc) or repeat a single error (eg not punctuate possessive nouns) Occasional lapses of style are offSet by demonstrated mastery of syntax

CHAPTE84 Standardized Testing 81

2 Weak The writer makes somewhat simplistic suggestions not fully supported with reashysons fails to cite key facts offers little analysis of the problem or shows a limited grasp of the situation the given information is copied or listed withlittle integration into argument Points may be random or repetitious Writing may be badly focused with careless use of abstract language resulting in predication errors or illogical sentences

ESL andlor careless mechanical errors are frequent enough to be distracting OR sentences may be choppy style over-casual usage occasionally unidiomatic

1 Inadequate The essay may be disjointed incoherent or minimally developed The writer shows little grasp of the complex issues involved is unable to establish conshytext point of view or purpose in opening of paper or has a poor sense of audience Mechanical andor ESL errors or unidiomatic usages are frequent sentences may be ungrammatical OR correct but short and very simple

The two readers scores are added to yield a total possible score of 2 to 8 Test administrators recommend a score of 6 as the threshold for allowing a student to pursue graduate-level courses Anything below that is accompanied by a recomshymendation that the student either repeat the test or take a remedial course in gradshyuate writing offered in one of several different departments Students receive neither their essays nor any feedback other than the fmal score

6 Perform ongoing construct validation studies

From the above discussion it should be clear that no standardized instrument is expected to be used repeatedly without a ramporou~program of ongoing c~-sectmct valiltiatiOll Any standardized test once developed must be accompanied by sysshy~

tematic periodic corroboration of its effectiveness and by steps toward its improveshyment This rigor is especially true of tests that are produced in equated forms that is forms must be reliable across tests such that a score on a subsequent form of a test-has-the~same validityand-interpretability as its original

(A) The TOEFL program in cooperation with other tests produced by ETS has an impressive program of research Over the years dozens of TOEFL-sponsored research studies have appeared in the TOEFL Monograph Series An early example ofsuch a study was the seminal Duran et aI (1985) study TOEFLfrom a Communicative ViewpOint on Language Proficiency which examined the content characteristics of the TOEFL from a communicative perspective based on current research in applied linguistics and lanshyguage proficiency assessment More recent studies (such as Ginther 2001 Leacock amp Chodorow 2001 Powers et aI 2002) demonstrate an impressive array of scrutiny

(B) For approximately 20 years the ESLPT appeared to be placing students relishyably by means of an essay and a multiple-choice grammar and vocabulary test Over the years the security of the latter became s1lspect and the faculty administrators wished to see some content validity achieved in the process In the year 2000 that process began with a group of graduate students (Imao et aI 2000) in consl1ltation with faculty members and continued to fruition in the form of a new ESLPT reported in lmao (2002) The development of the new ESlPT involved a lengthy process of

82 CHAPTER 4 Standardized Testing

both content and construct validation along with facing such practical issues as scoring the written sections and a machine scorable multiple-choice answer sheet

The process of ongoing validation will no doubt continue as new forms of the editing section are created and as new prompts and reading passages are created for the writing section Such a validation process should also include consistent checks on placement accuracy and on face validity

(C) At this time there is little or no research to validate the GET itself For its conshy struct validation its administrators rely on a stockpile of research on university-level academic writing tests such as theTWEThe holistic scoring rubric and the topics and administrative conditions of the GET are to some extent patterned after that of the TWE In recent years some criticism of the GEf has come from international test-takers (Hosoya 2001) who posit that the topics and time limits of the GET among other facshytors work to the disadvantage of writers whose native language is not English These validity issues remain to be fully addressed in a comprehensive research study

I I

STANDARDIZED IANGUAGE PROFICIENCY TESTING

Tests of language profiCiency presuppose a comprehensive definition of the specific competencies that comprise overall language ability The specifications for the TOEFL provided an illustration of an operational definition of ability for assessment purposes This is not the only way to conceptualize the concept Swain (1990) offered a multidimensional view of profiCiency assessment by referring to three linshyguistic traits (grammar discourse and sociolinguistics) that can be assessed by means of oral multiple-choice and written responses (see Table 41) Swains conshyception was not meant to be an exhaustive analysis of ability but rather to serve as an operational framework for constructing proficiency assessments

Another defmition and conceptualization of profiCiency is suggested by the ACTFL association mentioned earlier ACfFL takes a holistic and more unitary view of proficiency in describing four levels superior advanced intermediate and noviceWithin each level descriptions of listening speaking reading and writing are provided as guidelines for assessment For example the ACfFL Guidelines describe the superior level of speaking as follows

ACTFL speaking guidelines summary superior-level

Superior-level speakers are characterized by the ability to

bull participate fully and effectively in conversations in formal and informal settings on topics related to practical needs and areas of professional andor scholarly interests

bull provide a structured argument to explain and defend opinions and develop effective hypotheses within extended discourse

bull discuss topics concretely and abstractly bull deal with a linguistically unfamiliar situation bull maintain a high degree of linguistic accuracy bull satisfy the linguistic demands of professional andor scholarly life

CHAPTER4 Standardized Testing 83

The other three ACfFL levels use the same parameters in describing progressively lower proficiencies across all four skills Such taxonomie~ have the advantage of considering a number of functions of linguistic discourse but the disadvantage at the lower levels of overly emphasizing test-takers deficiencies

Table 41 Traits of second language proficiency (Swain 1990 p 403)

Trait Grammar Discourse Sociolinguistic

focus on grammatical focus on textual focus on social accuracy within cohesion and appropriateness of sentences coherence language use

Method

Oral structured interview story telling and argumentationpersuasion

role-play ofspeech acts requests offers complaints

scored for accuracy of verbal morphology prepositions syntax

detailed rating for identification logical sequence and time orientation and global ratings for coherence

scored for ability to distinguish formal and informal register

Multiple-choice

sentence-level select the correct form exercise

paragraph-level select the coherent sentence exercise

speech act-Ievelselect the appropriate utterance exercise

(45 items) (29 items) (28 items)

involving verb morphology prepositionsan-d-uther items

Written composition

narrative and letter of persuasion

narrative and letter of persuasion

formal request letter and informal note

scored for accuracy of verb morphology prepositions syntax

detailed ratings much as for oral discourse and global rating for coherence

scored for the ability to distinguish formal and inforJ1lil1 register

FOUR STANDARDIZED lANGUAGE PROFICIENCY TESTS

We now tum to some of the better-known standardized tests of overall language ability or profiCiency to examine some of the typical formats used in commercially available tests We will not look at standardized tests of other specific skills here but that should not lead you to think by any means that proficiency is the only kind of test in the field that is standardized Three standardized oral production tests the

84 CHAPTER 4 Standardized Testing

Test of Spoken English (fSE) the Oral Proficiency Inventory (OPI) and PbonePassreg are discussed in Chapter 7 and the Test of Written English (WE) is covered in ChapterS

Four commercially produced standardized tests of English language proficiency are described briefly in this section the TOEFL the Michigan English Language Assessment Battery (MELAB) the International English Language Testing System (lELTS) and the Test of English for International Communication (fOEICreg) In an appendix to this chapter are sample items from each section of each test When you turn to that appendix use the following questions to help you evaluate these four tests and their subsections

1 What item types are included 2 How practical and reliable does each subsection of each test appear to be 3 Do the item types and tasks appropriately represent a conceptualizatio~ of

language proficiency (ability) That is can you evaluate their construct validity

4 Do the tasks achieve face validity 5 Are the tasks authentic 6 Is there some washback potential in the tasks

Test of English as a Foreign Language (TOEFL)

Producer Educational Testing Service (ETS) Objective To test overall proficiency (language ability) Primary market Almost exclusively US universities and colleges for admission

purposes Type Computer-based (CB) (and two sections are-computer-adaptive)

A traditional paper-based (PB) version is also available Response modes Multiple-choice responses essay Specifications See the box on pp 72-73 Time allocation Up to 4 hours (CB) 3 hours (PB) Internet access wwwtoeflorg

Comments In the North American context the TOEFL is the most widely used comshymercially available standardized test of proficiency Each year the TOEFL test is adminisshytered to approximately 800000 candidates in more than 200 countries It is highly respected because of the thorough program of ongoing research and development conshyducted by ETS The TOEFLs primary use is to set proficiency standards for international students seeking admission to English-speaking universities More than 4200 academic institutions government agencies scholarship programs and licensingcertification agenshycies in more than 80 countries use TOEFL scores By 2004 the TOEFL will include a secshytion on oral production

CHAPTER 4 Standardi~ed Testing 85

Michigan English Language Assessment Battery (MELAB)

Producer English language Institute University of Michigan Objective To test overall proficiency (language ability) Primary market Mostly US and Canadian language programs and colleges

some worldwide educational settings as well Type Paper-based Response modes Multiple-choice responses essay Time allocation 25 to 35 hours Internet access wwwlsaumicheduelimelabhtm

Specifications The MElAB consists of three sections Part 1 a 3D-minute impromptu essay is written on an assigned topic Part 2 a 25-minute multiple-choice listening comshyprehension test is delivered via tape recorder Part 3 is a 100-item 75-minute multipleshychoice test containing grammar doze reading vocabulary and reading comprehension An oral interview (speaking test) is optional

Comments The Ell at the University of Michigan has been producing the MELAB and its earlier incarnation (Michigan Test of English language Proficiency) since 1961 like the TOEFL it serves a North American audience but is also used internationally While its use is not as widespread as the TOEFL its validity is widely respected Because it is cheaper than the TOEFL and more easily obtained it is popular among language schools and institutes Many institutions and companies accept MElAB scores in lieu ofTOEFL scores

International English Language Testing System (IELTS)

Producer Jointly managed by The University of Cambridge local Examinations Syndicate (UClES) The British Council and lOP Education Australia

Objective To test overall proficiency (language ability) Primary-market Australian British Canadian and New Zealand academic

institutions and professional organizations American academic institutions are increasingly accepting IELTS for admissions purposes

1)rpe Computer-based (for the Reading and Writing sections) papershybased for the listening and Speaking modules

Response modes Multiple-choice responses essay oral production Time allocation 2 hours 45 minutes Internet access httpwwwieltsorgl

httpwwwudesorguk httpwwwbritishcouncilorg

Specifications Reading candidates choose between academic reading or general training reading (60 minutes) Writing the same option academic writing or general training writing (60 minutes) Listening four sections for all candidates (30 minutes) Speaking five sections for all candidates (1015 minutes)

86 CHAPTER 4 Standardized Testing

Comments The University of Cambridge local Examinations Syndicate (UCLES) has been producing English language tests since 1858 Now with three organizations cooperatshying to form the IELTS more than a million examinations are administered every year In 2002 a computer-based version of the Reading and Writing modules of the IELTS became available at selected centers around the world The other sections are administered locally by an examinet The paper-based IELTS remains an option for candidates The IELTS retains the distinct advantage of requiring all four skills in the test-takers performance

Test of English for International Communication (TOEICreg)

Producer The Chauncey Group International a subsidiary of Educational Testing Service

Objelttive To test overall proficiency (langlJage ability) Primary market Worldwide business commerce and industry contexts

(workplace settings) Type Computer-based and paper-based versions Response modes Multiple-choice responses Time allocation 2 hours Internet access httpwwwtoeiccom

Specifications Listening Comprehension 100 items administered by audiocassette Four types of task statements questions short conversations and short talks (approxishymately 45 minutes) Reading 100 items Three types of task cloze sentences error recogshynition and reading comprehension (75 minutes)

Comments The TOEIC has become a very widely used international test of English proficiency in workplace settings where English is required for job performance The conshytent includes many different employment settings such as conferences presentations sales ordering shipping schedules reservations (etters and memoranda It is approprishyate to use in educational settings where vocational or workplace English courses are being offered

sect sect sect sect sect

The construction of a valid standardized test is no minor accomplishment whether the instrument is large- or small-scale The designing of specifications alone as this chapter illustrates requires a sophisticated process of construct valishydation coupled with considerations of practicality Then the construction of items and scoringinterpretation procedures may require a lengthy period of trial and error with prototypes of the final form of the testWith painstaking attention to all the details of construction the end product can result in a cost-effective timeshysaving accurate instrument Your use of the results of such assessments can provide useful data on learners language abilities But your caution is warranted as well for all the reasons discussed in this chapter The next chapter will elaborate on what lies behind that need for a cautious approach to standardized assessment

CHAPTER4 Standardized Testing 87

EXERCISES

[Note (I) Individual work (G) Group or pair work (C) Whole-class discussion]

1 (C) Tell the class about the worst test experience youve ever had Briefly anamiddot lyze what made the experience so unbearable and try to come up with sugshygestions for improvement of the test andor its administrative conditions

2 (G) In pairs or small groups compile a brief list of pros and cons of standardshyized testing Cite illustrations of as many items in each list as possible Report your lists and examples to the rest o~ the class

3 (I) Select a standardized test that you are quite familiar with (probably a recent experience) Mentally evaluate that test using the five principles of practicality reliability validity authenticity and washback Report yourevaluashytion to the class

4 (G) The appendix to this chapter provides sample items from Jour different tests of language proficiency In groups one test for each group analyze your test for (a) content validity (b) face validity and (c) authenticity

5 (C) Do you think that the sample TOEFL reading passage about pirates (pages 74-75) and the Graduate EssayTest prompt (pages 76-77) about a school board hiring committee have any culture bias Discuss this and other cultural biases you have noticed in tests Is it possible to design a test that is completely free of culture bias

6 (CG) Compare the differences in conceptualization of language proficiency represented by Swains model the TOEFL and the ACfFL philosophy Which one best represents current thinking about communicative language ability What are the strengths and weaknesses of each approach

FORYOlIILEURTHER READING

Gronlund Norman E (1998) Assessment of student achievement Sixth Edition Boston Allyn and Bacon

Gronlunds classic also mentioned in Chapter 3 offers a concise overview of features of standardized tests offering definitions and examples of the statistical considerations in interpreting scores His approach is unbiased cleady written and accessible to those who might fear the mathematics of standardized testing

Phillips Deborah 2001 Long1nan introductory course for the TOEFL test White Plains NY Pearson Education

A careful examination of this or any other reputable preparation course for a standardized language test is well worth a students time Note especially how the book acquaints the user with the specifications of the test and offers a number of useful strategie~ that can be llsed in preparation for the test and during irs adn1inistration

88 CHAPTER 4 Standardized Testing

APPENDIX TO CHAPTER 4

Commercial Proficiency Tests Sample Items and Tasks

Test of English a~ a Foreign Language (TOEFLreg)

Listening r

Part A

In this section you will hear short conversations between two people In some ofthe conversations each person speaks only once In other conversations one or both of the people speak more than once Each conversation is followed by one questionabQlt it Each question in this part has four answer choices You should click on the best answer to each question Answer the questions on the basis of what is stated or implied by the speakers Here is an example On the computerscreen you will see

[man and woman talking]

On the recording you will hear

(woman) Hey wheres your sociology book (man) At home Why carry it around when were just going to be taking

a test (woman) Dont you remember Professor Smith said we could us it during

the test (man) Ohl no Well Ive still got an hour right Im so glad I ran into you

You wiII then see and hear the question before the answer choices appear

What will the man probably do next

o Begin studying for the sociology test o Explain the problem to his professor o Go home to get his textbook o Borrow the womans book

To choose an answer you will click on an oval The oval next to that answer will darken After you click on Next and Confirm Answer the next conversation will be presented

Part B

In this section you will hear several longer conversations and talks Each conversation or talk is followed by several questions The conversations talks and questions will not be repeated The conversations and talks are about a variety of topics You do not need speshycial knowledge of the topics to answer the questions correctly Rather you should answer each question on the basis of what is stated or implied by the speakers in the conversashytions or talks

For most of the questions you will need to click on the best of four possible answers Some questions will have special directions The special directions will appear in a box on the computer screen Here is an exampie ot a conversation and some questions

CHAPTER 4 Standardized Testing 89

Marine Biology (narrator) Listen to part of a discussion in a marine biology class

(professor) A few years ago our local government passed a number of strict environmental laws As a result Sunrise Beach looks nothing Ii ke it did ten years ago The water is cleaner and theres been a tremendous increase in all kinds of marine life which is why were going there on Thursday

(woman) I dont know if I agree that the water quality has improved I mean I was out there last weekend and it looked all brown It didnt seem too clean to me

(professor) Actually the color of the water doesnt always indicate whether its polluted The brown color you mentioned might be a result of pollution or it can mean a kind of brown algae is growing there Its called devils apron and it actually serves as food for whales

(man) So when does the water look blue (professor) Well water thats completely unpolluted is actually colorless But

it often looks bluish-green because the sunlight can penetrate deep down and thats the color thats reflected

(woman) But sometimes it looks really green Whats that about (professor) Ok well its the same principle as with devils apron the

water might be green because of different types of green algae there-gulfweed phytoplankton You all should finish reading about algae and plankton before we go In fact those are the types of living things Im going to ask you to be looking for when were there

Now get ready to answer the questions

What is the discussion mainly about

o The importance of protecting ocean environments o The reasons why ocean water appears to be different colors o The survival of whales in polluted water o The effect that colored ocean water has on algae

To choose an answer click on an oval The oval next to that answer will darken After you click on Next and Confirm Answer the next question will be presented

According to the professor what can make ocean water look browngt

o Pollution o Cloudy Skies o Sand o Algae

Click on 2 answers

To choose your answers you will click on the squares An XII wiii appear in each square

bullbullbullbullbullbullbull

90 CHAPTER 4 Standardized Testing

Structure and Written Expression This section measures the ability to recognize language that is appropriate for standard written English There are two types ofquestions in this section In the first type ofquestion there are incomplete sentences Beneath each sentence there are four words or phrases

Directions CIiSk on the one word or phrase that best completes the sentence

The colum~ine flower __ to nearly all of the United States can be raised from seed in almost any garden

native how native is how native is it is native

Time Help Confirm

After you click on Next and Confirm Answ~ the next question willbe presented

The second type of question has four underlined words or phrases You will choose the one underlined word or phrase that must be changed for the sentence to be correct

Directions Click on the one underlined word or phrase that must be changed for the senshytence to be correct

One of the most difficult problems in understanding sleep is determining what the funcshytions of sleep ~

lime Help Confirm

Clicking on an underlined word or phrase will darken it

Reading This section measures the ability to read and understand short passages similar in topic and style to those that students are likely to encounter in North American universities and colleges This section contains reading passages and questions about the passages There are several different types of questions in this section In the Reading section you will first have the opportunity to read the passage

The temperature of the Sun is over 10000 degrees Fahrenheit at the surface but it rises perhaps more than 270000000 at the center The Sun is so much hotter than the Earth that matter can exist only as a gasl except perhaps at the core In the core of the Sun the pressures are so great that despite the high temperature there may be a small solid core However no one really knows since the center of the Sun can never be directly observed ~ Solar astronomers do know that the Sun is divided into five general layers or zones Starting at the outside and going down into the Sun the zones are the corona chromoshysphere hotosphere convection zone and finally the core The first three zones are reshygarded as the Suns atmosphere But since the Sun has no solid surface it is hard to middottell where the atmosphere ends and the main body of the Sun begins

The Suns outermost layer begins about 10000 miles above the visible surface and goes outward for millions of miles This is the only part of the Sun that can be seen during an eclipse such as the one in February 1979 At any other time the corona can be seen

bullbullbullbullbullbullbull

bull bullbullbullbullbullbull

CHAPTER 4 Standardized Testing 91

only when special instruments are used on cameras and telescopes to block the light from the photosphere

The corona is a brilliant pearly white filmy light about as bright as the full Moon Its beautiful rays are a sensational sight during an eclipse The coronas rays flash out in a brilliant fan that has wispy spikelike rays near the Suns north and south poles The corona is generally thickest at the Suns equator The corona is made up of gases streamshying outward at tremendous speeds that reach a temperature of more than 2 million deshygrees Fahrenheit The gas thins out as it reaches the space around the planets By the time the gas of the corona reaches the Earth it has a relatively low density

When you have finished reading the passage you will use the mouse to click on Proceed Then the questions about the passage will be presented You are to choose the one best anshyswer to each question Answer all questions about the information in a passage on the basis ofwhat is stated or implied in that passage Most ofthe questions will be multiple-choice questions To answer these questions you will click on a choice below the question

With what topic is paragraph 2 mainly concerned

o How the Sun evolved o The structure of the Sun o Why scientists study the Sun o The distaflce of the Sun from the planets

Paragraph 2 is marked with an arrow (~)

You will see the next question after you click on Next

To answer some questions you will click on a word or phrase Here is an example

Look at the word one in the passage Click on the word or phrase in the bold text that one refers to To answer you can click on any part of the word or phrase in the passage Jour choice will darken to show which word you have chosen

The Suns outermost layer begins about 10000 miles above the visible surface and goes outward for millions of miles This is the only part of the Sun that can be seen durshying an eclipse such as the one in February 1979 At any other time the corona can be seen only when special instruments are used on cameras and telescopes to block the Iight from the photosphere

You will see the next question after you click on~ To answer some q~estions you will click on a sentence in the passage Here is an example

~ The corona is a brilliant pearly white filmy light about as bright as the full Moon Its beautiful rays are a sensational sight during an eclipse The coronas rays flash out in a brilliant fan that has wispy spikelike rays near the Suns north and south poles The corona is generally thickest at the Suns equator ~ The corona is made up of gases streaming outward at tremendous speeds that reach a temperature of more than 2 million degrees Fahrenheit The gas thins out as it reaches the space around the planets By the time the gas of the corona reaches the Earth it has a relatively low density

bull bullbullbullbullbullbull

92 CHAPTER 4 Standardized Testing

Click on the sentence in paragraph 4 or 5 in which the author compares the light of the Suns outermost layer to that ofanother astronomical body Paragraphs 4 and 5 are marked with arrows (~)

To answer some questions you will click on a square to add a sentence to the passage Here is an example -The following sentence can be added to paragraph 1

At the center of the Earths solar system lies the Sun

Where would it best fit in paragraph I Click on the square to add the sentence to the paragraph

D The temperature of the Sun is over 10000 degrees Fahrenheit at the surface but it rises to perhaps morethan 27000000deg at the center 0 The Sun is so much hotter than the Earth that matter can exist only as a gasi except p~rHapsatth~ c6relp the c~re of the ii Sun the pressures are so great that despite the high temperature there may be a small solid core D However no one really knows since the center of the Sun can never be directly observed D 0100

When you click on a square the sentence will appear in the passage at the place you have chosen You can read the sentence added to the paragraph to see if this is the best place to add it You can click on another square to change your answer The sentence will be added and shown in a dark box

Writing In this section you will have an opportunity to demonstrate your ability to write in Enshyglish This includes the ability to generate and organize ideas to support those ideas with examples or evidence and to compose in standard written English in response to an asshysigned topic You will have 30 minutes to write your essay on that topic You must write on the topic you are assigned An essay on any other topic will receive a score of 0 Read the topic below and then make any notes that will help you plan your response Begin typing your response in the box at the bottom of the screen or write your answer on the answer sheet provided to you

Following is a sample topic

Do you agree or disagree with the following statemenH

Teachers should make learning enjoyable and fun for their students

Use specific reasons and examples to support your opinion

CHAPTER 4 Standarczed Testing 93

Michigan English Language Assessment Battery (MELAB)

Composition The time limit for the composition is 30 minutes You must write on only one of the top~

ics below If you write about something else your composition paper will not be graded and you cannot be given a final score If you do not understand the topics ask the exam~ iner to explain or to translate them You may be asked to give your opinion ofsomething and explain why you believe this to describe something from your experience or to exshyplain a problem and offer possible solutions You should write at least one page Some sample topics are

1 What do you think is your countrys greatest problem Explain in detail and tell what you think can be done about it

2 What are the characteristics of a good teacher Explain and give examples 3 An optimist is someone who sees the good side of things A pessimist sees the

bad side Are you an optimist or a pessimist Relate a personal experience that shows this

4 In your opinion are the benefits of space exploration really worth the enormous costs Discuss

Most MELAB compositions are one or two pages long (about 200-300 words) If your paper is extremely short (less than 150 words) your composition will be given a lower score Before you begin writing you might want to take 2 or 3 minutes to plan your comshyposition and to make a short outline to organize your thoughts Such outlines will not be graded they are only to help you You should use the last 5 minutes to read through your composition and to make changes or corrections

Your composition will be graded on how clearly you express yourself in English and on the range of English you are able to use and your control in doing so This means your composition should be well organized your arguments should be fully developed and you should show a range ofgrammatical structures and broad vocabulary Compositions that consist only of very short sentences and very simple vocabulary cannot be given the

middothighest scores If errors are not frequent and if they do not confuse your meaning they will not lower your score very much

Listening Now you will hear a short lecture You may take notes during the lecture Following the lecture you will be asked some questions about it

Therell be a two-week exhibit of the paintings of the little-known master Laura Bernhart at the Claire Osmond Galleries starting on the fifteenth of the month and running through the thirtieth Bernharts known for her innovative designs in abstract expressionism Though a true original she declared a spiritual heritage from Salvador Dali the famous Spanish painter Since Bernhart lived a rather solitary life and died while only in her twenties few people are aware of her works This showing at the Osmond Galleries will provide many with an introduction to her works

10 Where is the exhibit a the Art Museum b the Dali Galleries c the Osmond Galleries

94 CHAPTER 4 Standardized Testing

11 What is Bernhart known for a her copies of Dalis paintings b the originality of her designs c her exhibitions

12 What will going to the exhibit allow most people to do a to see Saivador Dalis paintings b to see Bernharts works for the first time c to learn about Spanish art

Grammar

1 What did the teacher just tell you

She reminded our notebooksI a us to bring b that we bring c our bringing d we should bring

2 Is Bill a good dancer

Not really __ he tries very hard a in spite of h despite c even though d while

3 your clothes are all wet1

Yes I didnt come __ the rain soon enough a away to b over to c down with d in from

Cloze In years to come zoos will not only be places where animals are exhibited to the public but repositories where rare species can be saved from extinction (7) captive breeding The most powerful force (8) the future of many animals-and of zoos-is the decline of the wild (9) even zoo directors would argue that (10) are better places for animals than the fields and forest of their native (11) yet zoos may be the last chance for some creatures that would otherwise pass qUietly into oblivion

7 a through c from b of d damage

8 a bringing c to b that d influencing

9 a But c Not b So d Then

10 a where c even b zoos d wilds

11 alands c residence b life d field

CHAPTER 4 Standardized Testing 95

Vocabulary

12 Mark has a flair for writing a need b purpose c talent d dislike

13 Bill Collins launched his restaurant last June a moved b started c sold d bought

14 John will not accept the censure a burden b blame c credit d decision

15 I cant think of the answer Can you give me a __ a hint b token c taste d gaze

16 Because fewer people are taking expensive vacations the tourist industry is in a a choke b grope c grumble d slump

17 I disagree with a few of his opinions but __ we agree a deliberately b conclusively c essentially d immensely

Reading The influenza virus is a single molecule built from many millions of single atoms You must have heard of the viruses which are sometimes called living molecules While bacteria can be considered as a type of plant secreting pOisonous substances into the body of the organism they attack viruses are living organisms themselves We may conshysider them as regular chemical molecules since they have a strictly aefined atomic strucshyture but on the other hand we must also consider them as being alive since they are able to multiply in unlimited quantities

18 According to the passage bacteria are a poisons

b larger than viruses c very small d plants

96 CHAPTER 4 Standardized Testing

19 The writer says that viruses are alive because they a have a complex atomic structure b move c multiply d need warmth and light

20 The atomic structure of viruses a is -tJIariable b is strictly defined c cannot be analyzed chemically d is more complex than that of bacteria

International English Language Testing System (fELTS)

I

listening

The Listening Module has four sections The first two sections are concerned with social needs There is a conversation between two speakers and then a monologue For examshyple a conversation about travel arrangements or decisions on a night out and a speech about student services on a university campus or arrangements for meals during a confershyence The final two sections are concerned with situations related more closely to educashytional or training contexts For example conversation between a tutor and a student about an assignment or between three students planning a research project and a lecture or talk ofgeneral academic interest All the topics are ofgeneral interest and it makes no difference what subjects candidates study Tests and tasks become more difficult as the sections progress A range of English accents and dialects are used in the recording which reflects the international usage of IELTS

Academic Reading [A 7S0-word article on-th-e- topic of Wind Power in the US with a short glossary at the end]

Questions 1-5

Complete the summary below

Choose your answers from the box below the summary and write them in boxes 1-5 on your answer sheet Note There are more words or phrases than you will need to fill the gaps You may use any word or phrase more than once

Example The failure during the late 1970s and early 19805 of an attempt to establish a widespread wind power industry in the United States resulted largely from the (1) bull in oil prices during this period The industry is now experiencing a steady (2) due to improveshyments in technology and an increased awareness of the potential in the power of wind The wind turbines that are now being made based in part on the (3) of wide- ranging research in Europe are easier to manufacture and maintain than their predecesshysors This has led wind-turbine makers to be able to standardise and thus minimize (4) There has been growing (S) of the importance of wind power as an energy source

CHAPTER 4 Standardized Testing 97

criticism stability skepticism success operating costs decisions design costs fall effects production costs growth decline failure recognition results

Questions 6-1 0 Look at the following list of issues (Questions 6-10) and implications (A-C) Match each issue with one implication Write the appropriate letters A-C in boxes 6-10 on your anshyswer sheet

Example The current price of one wind-generated kilowatt Answer

6 The recent installation of systems taking advantage of economies of scale

7 The potential of meeting one fifth of current U5 energy requirements by wind power

8 The level of acceptance of current wind turbine technology

9 A comparison of costs between conventional and wind power sources

10 The view of wind power in the European Union

Implications

A provides evidence against claims that electricity produced from wind power is relatively expensive

B supports claims that wind power js an important source of energy

C opposes the view that wind power technology requires further-development

General Training Reading Read the passage on Daybreak trips by coach and look at the statements below On your answer sheet write

TRUE if the statement is true FALSE jf the statement is false

NOlGIVEN if the information is not given in the leaflet

1 MiIlers Coaches owns Cambridges Cam bus fleet

2 Premier is an older company than Millers

3 Most of the Daybreak coaches are less than 5 years old

4 Daybreak fares are more expensive than most of their competitors

5 Soft drinks and refreshments are served on most longer journeys

6 Smoking is permitted at the rear of the coach on longer journeys

7 Tickets must be bought in advance from an authorised Daybreak agent

6 Tickets and seats can be reserved by phoning the Daybreak Hotline

9 Daybreak passengers must join their coach at Cambridge Drummer Street

10 Daybreak cannot guarantee return times

98 CHAPTER 4 Standardized Testing

FROM CAMBRIDGE AND SURROUNDING AREA

SPRING IS INTHEAIR

Welcome to our Spring Daybreak programme which continues the tradition of offering unbeatable value for money day trips and tours All the excursions in this brochure will be operated by Pr~mier Travel Services Limited or Millers Coaches both companies are part of the CHLGroup owners of Cambridges Cambus fleet

WERE PROUD OF OUR TRADITION

Premier was established in 1936 the Company now offers the highest standards of coaching in todays competitive operating environment Miller has an enviable reputation stretching back over the past 20 years offering coach services at realistic prices Weve traveled a long way since our early days of pre-war seaside trips Now our fleet of 50 modern coaches (few are more than five years old) operate throughout Britain and Europe but were pleased to still maintain the high standards of quality and service the trademark of our founders nearly sixty years ago

EXCLUSIVE FEATURES

Admission-inclusive fares All Daybreak fares (unless specifically otherwise stated) include admission charges to the attractions shows and exhibits we visit Many full-day scenic tours are accompanied by a fully trained English Tourist Board Blue Badge guide or local experienced driverguide Some Daybreaks include lunch or afternoon tea Compare our admission inclusive fares and see how much you save Cheapest is not the best and value for money is guaranteed If you compare our bargain Daybreak fares beware--most of our competishytors do not offer an all-inclusive fare

SEAT RESERVATIONS

We value the freedom of choice so you can choose your seat when you book The seat reservation is guaranteed a-nd remains yours at all times when aboard the coach

NO SMOKING COMFORT

With the comfort of our passengers in mind coaches on all our Daybreaks are no smokshying throughout In the interests of fellow passengers comfort we kindly ask that smokers observe our no smoking policy On scenic tours and longer journeys ample refreshment stops are provided when of course smoking is permitted

YOUR QUESTIONS ANSWERED

Do I need to book Booking in advance is strongly recommended as all Daybreak tours are subject to demand Subject to availability stand-by tickets can be purchased from the driver

What ti me does the coach leave The coach departs from Cambridge Drummer Street (Bay 12 adjacent to public toilets) at the time shown There are many additional joining points indicated by departure codes in the brochure If you are joining at one of our less popular joining points you will be adshyvised of your pick-up time (normally by telephone) not less than 48 hours before deparshyture In this way we can minimize the length of pick-up routes and reduce journey times for the majority of passengers

CHAPTER 4 Standardized Testing 99

What time do we get back An approximate return time is shown for each excursion The tim~s shown serve as a guide but road conditions can sometimes cause delay If your arrival will be later than advertised your driver will try to allow for a telephone call during the return journey

Where can I board the coach All the Daybreaks in the brochure leave from Cambridge Drummer Street (Bay 12 adjashycent to public toilets) at the time shown Many Daybreaks offer additional pick-ups for pre-booked passengers within Cambridge and the surrounding area This facility must be requested at the time of booking

Academic Writing Writing Task 1 You should spend about 20 minutes on this task

The graph below shows the different modes of transport used to travel to and from work in one European city in 1950 1970 and 1990

[graph shown here]

Write a report for a university lecturer describing the information shown below You should write at least 150 words

Writing Task 2 You should spend about 40 minutes on this task

Present a written argument or case to an educated reader with no specialist knowledge of the folowing topic

It is inevitable that as technology develops so traditional cultures must be lost Technolshyogy and tradition are incompatible-you cannot have both together

To what extent do you agree or disagree with this statement Give reasons for your answer You should write at least 250 words You should use your own ideas knowlshyedge and experience and support your arguments with examples and relevant evidence

General Training Writing Writing Task 1 You should spend about 20 minutes on this task You rent a house through an agency The heating system has stopped working You phoned the agency a week ago but it has still not been mended Write a letter to the agency Explain the situation and teil them what you want them to do about it

You should write at least 150 words You do NOT need to write your own address

Begin your letter as follows

Dear - ___-I

Writing Task 2 You should spend about 40 minutes on this task As part ofa class assignment you have to write about the following topic

100 CHAPTER 4 Standardized Testing

Some businesses now say that no one can smoke cigarettes in any of their offices Some governments have banned smoking in all public placesThis is a good idea but it takes away some of our freedom

Do you agree or disagree Give reasons for your answer You should write at least 250 words

Speaking In each ofthe three parts of the speaking module a specific function is fulfilled In Part 1 the candidates answer general questions about themselves their homes or families their jobs or studies their interests and a range ofsimilar familiar topic areas This part lasts between four and five minutes In Part 2 the candidate is given a verbal prompt on a card and is asked to talk on a particular topic The candidate has one minute to prepare before speaking at length for between one and two minutes The examiner then asks one or two wind-down questions In Part 3 the examiner and candidate engage in a discusshysion of more abstract issues and concepts which are thematically linked to the topic prompt in Part 2 The discussion lasts between four and five minutes

All interviews are recorded on audiocassette Here is a sample ofa Part 2 topic

Describe a teacher who has greatly influenced you in your education

You shou Id say

where you met them what subject they taught what was special about them

and explain why this person influenced you so much

You will have to talk about the topic for 1 to 2 minutes You have 1 minute to think about what you are going to say You can make some notes if you wish

Test of English for International Communication (TOEICreg)

listening

Part 1 Photographs Directions For each question you will see a picture in your test book and you will hear four short statements The statements will be spoken just one time They will not be printed in your test book so you must listen carefully to understand what the speaker says When you hear the four statements look at the picture in your test book and choose the statement that best describes what you see in the picture Then on your answer sheet find the number of the question and mark your answer

[photograph of a scientist looking through a microscope]

You will hear Look at the picture marked number 1 in your test book

(A) Shes speaking into a microphone (B) Shes put on her glasses (C) She has both eyes open (D) Shes using a microscope

CHAPTER 4 Standardized Testing 101

Part 2 Question-Response Directions In this part of the test you will hear a question or statement spoken in Enshyglish followed by three responses also spoken in English The question or staten1ent and the responses will be spoken just one time They will not be printed in your test book so you must listen carefully to understand what the speakers say You are to choose the best response to each question or statement

Question 1 You will hear Ms Morikawa has worked here for a long time hasnt she

(A) At three oclock (B) No Ive lost my watch (C) More than ten years

Question 2 You will hear Which of these papers has a wider circulation

(A) The morning edition (B) Get more exercise (C) By messenger

Part 3 Short Conversations Directions In this part of the test you will hear short conversations between two people The conversations will not be printed in your test book You will hear the conversations only once so you must listen carefully to understand what the speakers say In your test book you will read a question about each conversation The question will be followed by four answers You are to choose the best answer to each question and mark it on your answer sheet

Question 1 (Man) We should think about finding another restaurant for lunch (Woman) Why The food and service here are great

(Man) Yes but the prices are going up every week

You will read Why is this man unhappy with the restaurant

(A) It is too noisy (B) It is too expensive (C) It is too crowded (D) It is too difficult to find

Question 2 (Woman A) How was Dr Borgs recent trip to Singapore (Woman B) She enjoyed the tour of the port very much (Woman A) They say its one of the most active in Asia

You will read 2 What did Dr Borg find interesting

(A) The tourist center (B) The airport (C) The musical performance (D) The harbor

Part 4 Short Talks Directions In this part of the test you vill hear several short talks Each will be spoken just one time They will not be printed in your test book so you must listen carefully to understand and remember what is said In your test book you will read two or more questions about each short talk The questions will be followed by four answers You are to choose the best answer to each question and mark it on your answer sheet

102 CHAPTER 4 Standardized Testing

You will hear Questions 1 and 2 refer to the following announcement

Good afternoon and welcome aboard Nordair Flight 857 from Copenhagen to Bangkok with intermediate stops in Dubai and Calcutta We are preparing for departure in a few minutes At this time your seat back should be returned to its full upright position and your seat belt s~ould be fastened OUf anticipated total flying time to Dubai is six hours and twenty-five minutes I hope you enjoy the flight You will hecJr Now read question 1 in your test book and answer it You will read 1 What is the final destination of the flight

(A) Bangkok (B) Copenhagen (C) Dubai (O) Calcutta

You will hear Now read question 2 in your test book and answer it You will read 2 What will happen in a few minutes

(A) The flight will land in Dubai I

(B) The passengers will board the plane (C) The plane will take off (0) The gate number will be announced

Reading In this section of the test you will have the chance to show how well you understand written English There are three parts to this section with special directions for each part

Part 4 Incomplete Sentences Directions This part of the test has incomplete sentences Four words or phrases marked (A) (8) (e) (D) are given beneath each sentence You are to choose the one word or phrase that best completes the sentence Then on your answer sheet find the number of the question and mark your answer

1 Mr Yangs trip will __ him away from the office for ten days (A) withdraw (B) continue (C) retain (0) keep

2 The company that Marie DuBois started now sells __ products throughout the world (A) its (B) it (C) theirs (D) them

3 If your shipment is not delivered __ Tuesday you can request a full refund for the merchandise (A) at (B) by (C) within (D) while

CHAPTER 4 Standardized Testing 103

Part 6 Error Recognition Directions In this part ofthe test each sentence has four words or phrases underlined The four underlined parts of the sentence are marked (A) (B) (C) (D) You are to identify the one underlined word or phrase that should be corrected or rewritten Then on your answer sheet find the number of the question and mark your answer

1 The pamphlet contains some importance information about the current exhibit ABC D

2 No matter how Jong it taking to finish the annual report it must be done properly ABC D

3 The popularity of jogging appears to have decreased since the past couple of years ABC D

Part 7 Reading Comprehension Directions The questions in this part of the test are based on a selection of reading mateshyrials such as notices letters) forms newspaper and magazine articles) and advertisements You are to choose the one best answer (A) (B) (C) or (OJ to each quesshytion Then on your ariswefsheelfindthe number of the qUestion andmcirkyour answer Answer all questions following each reading selection on thebasis of what is stated or implied in that selection

The Museum ofTechnology is a hands-on museum designed for people to experience science at w()rk~ Visitors are encouraged to use test and handle the objects o~ display Special demonstrations are scheduled for the first and second Wednesdays of each month at 1330 Open Tuesday-Friday 1200-1630 Saturday 1000-1730 and Sunday 11 00-1630

1 When during the month can visitors see special demonstrations (A) Every weekend (B) The first two Wednesdays (C) One afternoon a week (D) Every other Wednesday

Questions 2 and 3 refer to the followi ng notice

NOTICE If you are unable to work because of an extended illness or injury that is not workshyrelated you may be entitled to receive weekly benefits from your employer or the firms insurance company To claim benefits you must file a claim form within thirty days of the first day of your disability Before filing the claim you must ask your doctor to fill in the Doctors Statement on the claim form stating the period of disability

3 To whom is this notice addressed (A) Employers (8) Doctors (C) Employees (D) When paying the bill

4 When must the claim form be filed (A) On the first of the month (8) On the thirtieth of the month (C) On the first day ofdisabifity (D) Within 30 days of the start of disability

Page 4: Standardized Testing Chapter 4 Brown

CHAPTER 4 Standardized Testingmiddot 69

changesand otherrules of the road Correlational statistics show a strong relationship between high scores on those tests and good driving records so people who do well on these tests are a safe bet to relicense Now an extremely high correlation (of pershyhaps SO or above) may be loosely interpreted to mean that a large majority of the drishyvers whose licenses are renewed by virtue of their having passed the little quiz are good behind-the-wheel drivers What about those few who do not fit the model That small minority of drivers could endanger the lives of the majority and is that a risk worth taking Motor vehicle registration departments in the United States seem to think so and thus avoid the high cost of behind-the-wheel driving tests

Are you willing to rely on a standardized test result in the case of all the learners in your class Of an applicant to your institution or of a potential degree candidate exiting your program These questions will be addressed more fully in Chapter 5 but for the moment think carefully about what has come to be known

as high-stakes testing in which standardized tests have become the only criterion for inclusion or exclusion The widespread acceptance and sometime misuse of this gate-keeping role of the testing industry has created a political educational and moral maelstrom

DEVELOPING A STANDARDIZED TEST

Whlle it is not likely that a classroom teacher with a team of test designers and researchers would be in a position to ~evelop a brand-new standardized test of large-scale proportions it is a yirtual certainty that some day you will be in a posishytion (a) to revise an existing test (b) toadapt or expand an existing test andor (c) to create a smaller-scale standardized test for a program you are teaching in And even if none of the above thre~ cases should ever apply to you it is of paramount importance to understand the process of the development of the standardized tests that have become ingrained in our educational institutions

How are standardized tests developed Where do test tasks and items come from How are they evaluated Who selects items and their arrangement in a test How do such items and tests- achieve consequential validity How are different forms of t~sts equated for diftlculty level Who sets norms and cutoff limits Are security add confidentiality an issue Are cultural and racial biases an issue in test development All these questions typify those that you might pose in an attempt to understand the process of test development

In the steps outlined below three different standardized- tests will be used to exemplify the process of standardized test design

(A) The Test of English as a Foreign Language (TOEFL) Educational Testing Service (ETS) (B) The English as a Second Language Placement Test (ESLP1) San Francisco State University (SFSU) (C)The Graduate Essay Test (Gm SFSU

70 CHAPTER 4 Standardized Testing

The first is a test of general language ability or profidencyThe second is a place ment test at a universityAnd the third is a gate-keeping essay test that all prospecshytive students must pass in order to take graduate-level courses As we look at the steps one by one you will see patterns that are consistent with those outlined in the previous two chapters for evaluating and developing a classroom test

1 Determine ihe purpose and objectives of the test

Most standardized tests are expected to provide high practicality in administration and scoring without unduly compromising validity The initial outlay of time and money for such a test is Significant but the test would be used repeatedly It is therefore imporshytant for its purpose and objectives to be stated specifically Lets look at the three tests

(A) The purpose of the TOEFL is to evaluate the English profiCiency of people whose native language is not English (TOEFL Test and Score Manual 2001 p 9) More specifically theTOEFL is designed to help institutions of higher learning make valid decisions concerning English language profiCiency lin terms of lthelr] own requirements (p 9) Most colleges and universities in the United States use TOEFL scores to admit or refuse international applicants for admission Various cut-off scores apply but most institutions require scores from 475 to 525 (paper-based) or from 150 to 195 (computer-based) in order to consider students for admissionThe high-stakes gate-keeping nature of the TOEFL is obvious

(B) The ESLPT referred to in Chapter 3 is designed to place already admitted students at San Francisco State University in an appropriate course in academic writing with the secondary goal of placing students into courses in oral production and grammar-editing While the tests primary purpose is to make placements another desirable objective is to provide teachers with some diagnostic information about their students on the first day or two of classThe ESLPT is locally designed by university faculty and staff

(C) The GEfanother test designed at SFSU is given to prospective graduate students-both native and non-native speakers-in all disciplines to determine whether their writing ability is sufficient to permit them to enter graduate-level courses in their programs It is offered at the beginning of each term Students who fail or marginally pass the GET are technically ineligible to take graduate courses in their field Instead they may elect to take a course in graduate-level writing of research papers A pass in that course is equivalent to passing the GET

As you can see the objectives of each of these tests are specific The content of each test must be designed to accomplish those particular ends TIlis first stage of goal-setting might be seen as one in which the consequential validity of the test is foreshymost in the mind of the developer each test has a specific gate-keeping function to perfonn therefore the criteria for entering those gates must be specified ~ccurately

2 Design test specifications

Now comes the hard part Decisions need to be made on how to go about structurshying the specifications of the test Before specs can be addressed a comprehensive

CHAPTER4 Standardized Testing 71

progratn of research must identify a set of constructs underlying the test itself This stage of laying the foundation stones can occupy weeks months or even years of effort Standardized tests that dont work are often the product of short-sighted conshystruct validation Lets look at the three tests again

(A) Construct validation for the TOEFL is carried out by the TOEFL staff at ETS under the guidance of a Policy Counell that works with a Committee of Examiners that is composed of appointed external university faculty linguists and assessment spedaJists Dozens of employees are involved in a complex process of reviewing curshyrentTOEFL specifications cOmmissioning and developing test tasks and items assemshybling forms of the test and performing ongoing exploratory research related to formulating new specs Reducing such a complex process to a set ofsimple steps runs the risk of gross overgeneralization but here is an idea of how aTOEFL is created

Because the TOEFL ismiddot a proficiency test the fIrst step in the developmental process is to define the construct of language proficiency First it should be made clear that many assessment specialists such as Bachman (1990) and Palmer (Bachman amp Palmer 1996) prefer the term ability to proficiency and thus speak of language abllity as the overarching concept The latter phrase is more conSistent they argue with our understanding that the specific components of language ability must be assessed separately Others such as the American Council on Teaching Foreign Languages (ACIFL) still prefer the term proficiency because it connotes more of a holistic unitary trait view of language ability (Lowe 1988) Most current views accept the ability argument and therefore strive to specify and assess the many components of language For the purposes of consistency in this book the term proficiency will nevertheless be retained with the above caveat

How you view language will make a difference in how you assess language proshyficiency After breaking language competence down into subsets of listening speaking reading and writing eapoundli~p~ifQ(m~I~~ Jll()4~ can be examined on a conshytiiiuum of linguistic units pho~~logy (p~onunciation) and orthography (spelling) words OeXicon) sentences (gtammar) discourse and pragmatic (sociolinguistic contextual functional cul~j features of language

How will the TOEFL sample from at) these possibilities Oral production tests can be tests of overall conversational fluency or pronunciation of a particular subset of phonolOgy and can take the form of imitation structured responses or free responses Listening comprehension tests can concentrate on a particular feature of language or on overalllistenins for general meaning Tests of realtling can cover the range of language units and can aim to test comprehension of long or short passhysages single sentences or even phrases and words Writing-tests can take on an open-ended form with free composition or be structured to elicit anything from correct spelling to discourse-level competence Are you overwhelmed yet

From the sea of potential performance modes that could be sampled in a test the developer must select a subset on some systematic basis To make a very long story short (and leaving out numerous controversies) the TOEFL had for many years included three types of performance in its organizationai specifications listening strucshyture and reading all of vtuch tested comprehension through standard multiple-choice

72 CHAPTER 4 Standardized Testing

tasks In 1996 a major step was taken to include written production in themiddot computershybased TOEFL by adding a slightly modified version of the already existing Test of Written English (TWE) In doing so some face validity and content validity were improved along with of course a significant increase in administrative expense Each of these four major sections is capsulized in the box below (adapted from the descripshytion of the current computer-based TOEFL at wwwroefLorg) Such descriptions are not strictly speaking specifications which are kept confidential by ETS Nevertheless they can give a sense of many of the constraints that are placed on themiddot design of actual TOEFL specifications

TOEFLlt8gt specifications

Listening Section The listening section measures the examinees ability to understand English as it is spoken in North America Conversational features of the language are bull stressed and the skills tested include vocabulary and idiomatic expression as well as speshycial grammatical constructions that are frequently used in spoken Engfish The stimulus material and questions are recorded in standard North American English

The listening section includes various stimuli such as dialogues short conversations academic discussions and mini-lectures and poses questions that test comprehension of main ideas the order ofa process supporting ideas important details and inferences as well as the ability to categorize topicsobjects

The test developers have taken advantage of the multimedia capability of the computer by using photos and graphics to create context and support the content of the lectures producing stimuli that more closely approximate Ureal-world situations in which people do more than just listen to voices The listening stimuli are often accompashynied by either context-setting or content-based visuals All dialogues conversations acashydemic discussions and mini-lectures include context visuals to establish the setting and role of the speakers Content-based visuals are often used to complement th~ topics of the mini-lectures

Structure-Section Themiddotstructure-section measures an examinees ability to recognize language that is appropriate for standard written English The language tested is formal rather than conversational The topics of the sentences are associated with general acadeshymic discourse so that individuals in specific fields of study or from specific national or linguistic groups have no particular advantage

Two types of questions are used questions in which examinees must (1) complete an incomplete sentence using one of four answers provided and (2) identify one of four unshyderlined words or phrases that would not be accepted in English The two question types are mixed randomly rather than being separated into two subsections as in the papershybased TOEFL test

Reading Section The reading section measures the ability to read and understand short passages similar in topic and style to academic texts used in North American colshyleges and universities Examinees read a variety of short passages on academic subjects and answer several questions about each passage Test items refer to what is stated or imshyplied in the passage as well as to words used in the passage To avoid creating an advanshyt~ t( 1 dull5- In 3n~ ont fidd of stud~ sufficient context is provided SO that no ~ -middott - ~tmiddot J ~ middoth - -~ rM~ ir~ to ~-er the if~(J(lS~ ~ t ~ ~ ~~~~-- Ih L_ 1_ 1 ~_

CHAPTER 4 Standardized Testing 73

The reading section consists of four to five passages of 250-350 words with 10-14 questions per passage This section is not computer-adaptive soexaminees can skip questions and return to previous questions The questions in this section assess the comshyprehension of main ideas inferences factual information stated in a passage pronoun referents and vocabulary (direct meaning synonym antonym) In all cases the questions can be answered by reading and understanding the passages This section consists of (1 ) traditional multiple-choice questions (2) questions that require examinees to click on a word phrase sentence or paragraph to answer and (3) questions that ask examinees to insert a sentence where it fits best

Writing Section The writing section measures the ability to write in English including the ability to generate organize and develop ideas to support those ideas with examples or evidence and to compose a response to one assigf)ed topic in standard written Enshyglish Because some examinees may not be accustomed to composing an essay on comshyputer they are given the choice of handwriting or typing the essay in the 30-minute time limit The rating scale for scoring the essay ranging from 0 to 6 is virtually the same as that of the Test of Written English [see Chapter 9 of this book] A score of 0 is given to papers that are blank simply copy the topic are written ina language other than English consist only of random keystroke characters or are written on a topic different from the one assigned

Each essay is rated independently by two trained certified readers Neither reader knows the rating assigned by the other An essay will receive the average of the two ratshyings unless there is a discrepancy of more than one point in that case a third reader will independently rate the essay The essay rating is incorporated into the StructureMriting scaled score~ and constitutes approximately 50 percent of that combined score

(B) The designing of the test specs for the ESLPT was a somewhat simpler task because the purpose is placement and the construct validation of the test consisted of an examination of the content of the ESL courses In fact in a recent revisiofi of the ESLPT (lmao et al 2000 Imao 2001) content validity (coupled with its attenshydant face validity) was the central theoretical issue to be considered The major issue centered on designing practical and reliable tasks and item response formats Having established the importance of designingESLPT tasks that simulated classroom tasks used in the courses the designers ultimately specified two writing production tasks (one a response to an essay that students read and the other a summary of another essay) and one multiple-choice grammar-editing taskThese specifications mirrored the readingbased process writing approach used in the courses

(C) Specifications for the GET arose out of the perceived need to provide a threshold of acceptable writing ability for all prospective graduate students at SFSU both native and non-native speakers of EnglishThe specifications for the GET are the skills of writing grammatically and rhetorically acceptable prose on a topic of some interest with clearly produced organization of ideas and logical development The GET is a direct test of writing ability in which test-takers must in a two-hour time period write an essay on a given topic

74 CHAPTER 4 Standardized Testing

3 Design select and arrange test tasksitems

Once specifications for a standardized test have been stipulated the sometimes never-ending task of designing selecting and arranging items beginS The spe~s act much like a blueprint in determining the number and types of items to be created Lets look at the three examples

(A) TOEFL test design specifies that each item be coded for content and statisshytical characteristics Content coding ensures that each examinee will receive test questions that assess a variety of skills (reading comprehending the main idea or understanding inferences) and cover a variety of subject matter without unduly biasing the content toward a subset of test-takers (for example in the listening secshytion involving an academic lecture the content must be universal enough for stushydents from many different academic fields of study) Statistical characteristics including the IRT equivalents of estimates of item ~~ility (IF) and the ability of an item to discriminate (ID) between higher orlower ability levels ate also coded

Items are then designed by a team who select and adapt items solicited from a bank of items that have been deposited by freemiddotlance writers and ErS staff Probes for the reading section for example are usually excerpts from authentic general or academic reading that are edited for linguistic difficulty culture bias or other topic biases Items are designed to test overall comprehension certain specific informashytion and inference

Consider the following sample of a reading selection and ten items based on it from a practice TOEFL (Phillips 2001pp423-424)

For hundreds of years in the early history of America pirates sailed through coastal washyters pillaging and plundering all in their path They stole from other ships andstole from coastal towns not content only to steal they destroyed everything they could not carry avay~ Some of the pirate ships amassed large treasures~ the fates of which are unknown leaving people of today to wonder at their whereabouts and to dream of one day coming across some lost treasure

One notoriously large treasure was on the pirate ship Whidah which sank in the washyters off Cape Cod during a strong storm in 1717 A hundred of the crew members went down with the ship along with its treasure of coins gold silver and jewels The treasure on board had an estimated value on todays market of more than 100 million dollars

The remains of the Whidah were discovered in 1984 by Barry Clifford who had spent years of painstaking research and tireless searching only finally to locate the ship about 500 yards from shore A considerable amount of treasure from the centuries-old ship has been recovered from its watery grave but there is clearly still a lot more out there Just as a reminder of what the waters off the coast have been protecting for hundreds of years occasional pieces of gold or silver or jewels still wash up on the beaches and lucky beach-goers find pieces of the treasure

11 ~ Thepa~e mainly diccus5eS

CH4PTER 4 Standardized Testing 75

(e) what really happened to the Whidahs pirates (D) why people go to the beach

12 It is NOT mentioned in the passage that pirates did which of the following (A) They killed lots of people (B) They robbed other ships (e) They took things from towns (D) They gathered big treasures

13 The word amassed in line 4 is closest in meaning to (A) sold (e) transported (B) hid (D) gathered

14 It is implied in the passage that the Whidahs crew (A) died (B) went diving (e) searched for the treasure (D) escaped with parts of the treasure

15 Which of the following is NOT mentioned as part of the treasure of the Whidah (A) Art objects (B) Coins (e) Gold and si Iver (D) Jewels

16 The word estimated in line 10 is closest in meaning to which of the following (A) Known (C) Approximate (B) Sold (D) Decided

17 The passage indicates that the cargo of theWhidah is worth about (A) $100000 (B) $1000000 (C) $10000000 (D) $100000000

18 The work that Barry Clifford did to locate the Whidah was NOT (A) successfu I (B) effortless (C) detailed (D) lengthy

19 It is mentioned in the passage that the treasure of the Whidah (A) is not very valuable (8) is all in museums (C) has not all been found (D) was taken to share by the pi rates

20 The paragraph following the passage most likely discusses (A) what Barry Clifford is doing today (8) the fate of the Whidahs crew (e) other storms in the area of Cape Cod (D) additional pieces that turn up from the Whidahs treasure

76 CHAPTER 4 Standardized Testing

As you can see items target the assessment of comprehension of the main idea (item 11) stated details (17 19) unstated details (12 15 18) implied details (14 20) and vocabulary in context (13 16) An argument could be made about the cultural schemata implied in a passage about pirate ships and you could engage in an angels on the head of a pin argument about the importance of picking cershytain vocabulary for emphasis but every test item is a sample of a larger domain and each of these fulfills its designated specification

Before any such items are released into a form of the TOEFL (or any validated standardized test) they are piloted and sCientifically selected to meet difficulty specshyifications within each subsection section and the test overall Furthermore those items are also selected to meet a desired discrimination index Both of these indices are important considerations in the design of a computer-adaptive test where pershyformance on one item determines the next one to be presented to the test-taker (See Chapter 3 for a complete treatment of multiple-choice item design)

(B)The selection of items in the ESLPT entailed two-entirel) different processes In the two subsections of -the test that elicit writing performance (summary of reading response to reading) the main hurdles were (a) selecting appropriate passhysages for test-takers to read (b) providing appropriate prompts and (c) processing data from pilot testing Passages have to conform to standards of content validity by being within the genre and the difficulty of the material used in the courses The prompt in each case (the section asking for a summary and the section asking for a response) has to be tailOred to fit the passage but a general template is used

[n the multiple-choice editing test that seeks to test grammar proofreading ability the first and easier task is to choose an appropriate essay within which to embed errors The more complicated task is to embed a specified number of errors from a previously determined taxonomy of error categories Those error categories came directly from student errors as perceived by their teachers (verb tenses verb agreeshyment logical connectors articles etc) The disttactors for each item were selected from actual errors that students make Itemsiti pilot versions were then coded fordifshyficulty and discrinlination indices after which final assembly of items could occur

(C) The GET prompts are designed by a faculty committee of examiners who are speCialists in the field of university academic writing The assumption is made that the topics are universally appealing and capable of yielding the intended product of an essay that requires an organized logical argument and conclusion No pilot testing of prompts is conducted The conditions for administration remain constant two-hour time limit sit-down context paper and pencil closed-book format Consider the following recent prompt

Graduate Essay Test sample prompt

In the Middletown Elementary School District the assistant superintendent has just been made superintendent in another district Her resignation leaves vacant the districts only administrative position ever held by a woman The School Board in response to strong

CHAPTER 4 Standardized Testing 77

arguments from the Teachers Association has urged that a woman be hired to replace her As a member of the hiring committee you must help choose her successor

Only one woman applicant meets the written qualifications for the job the two top male applicants are both more experienced than she

The hiring committee has asked each committee member to prepare a written statement to distribute before meeting together to discuss the issue Write a report that represents your position making it as logical and persuasive as possible

Some facts you may wish to draw on 1 Women make up more than 75 percent of classroom teachers but hold fewer than

10 percent of administrative positions in education Administrators salaries average 30 percent more than teachers salaries

2 The local Teachers Association is 89 percent women mostly under 40 In a heated debate on television a member of the National Organization of Women (NOW) and the chair of the Teachers Association threatened if a man is hired to bring a class-action suit against the district on behalf of all women teachers who cannot expect advancement because of discriminatory hiring practices

3 The local Lions Club which contributes heavily to school sports says hiring the less experienced woman would not be in the best interests of the schoolthe children or the teachers

The finalists for the position

1 Carole Gates Classroom teacher 10 years Teacher of the Year 1985 supervisor ofpractice teachers at Teachers College former president of Teachers Associ ati on Administrative Credential 1984 EdD degree 1986 assistant principal of Hoptown Elementary School 2 years

2 Spud Stonewall Principal of Middletown Elementary 15 years PhD in educational adminis~ration State Board of Education Committee for Improving Elementary School Curriculum 1982-present

3 Jim Henderson School Administrator 22 yearsgradesK-9-supports innovation in education Fair Bargaining Award 1981 former coach for winning collegiate basketball team 10 years

It is clear from such a prompt that the problem the test-takers must address is complex that there is sufficient information here for writing an essay and that testshytakers will be reasonably challenged to write a clear statement of opinion What also emerges from this prompt (and virtually any prompt that one might propose) is the potential cultural effect on the numerous international students who must take the GIIT Is it possible that such students who are not familiar with school systems in the United States with hiring procedures and perhaps with the politics of school board elections might be at a disadvantage in mounting their arguments within a two-hour time frame Some (such as Hosoya 2001) have strongly claimed such a bias

78 CHAPTER 4 Standardized Testing

4 Make appropriate evaluations of different kinds of items

In Chapter 3 the concepts of item facility (IF) item discrimination (ID) and disshytractor analysis were introduced As the discussion there showed such calculations provide useful infornlation for classroom tests but sometimes the time and effort involved in perfornling them may not be practical especially if the classroom-based test is a one-time test Yet for a standardized multiple-choice test that is designed to be marketed commercially andor administered a number of times andor adminisshytered in a different form these indices are a must

For other types of response formats namely production responses different forms of evaluation become importantThe principles of p-mpoundti~ality ~d poundabWty are prominent along with the concept o(JacjJjt Practicality issues in such items include the clarity of directions timing of the test ease of administration and how much time is required to score responses Reliability is a major player in instances where more than one scorer is employed and to a lesser extent when a single scorer has to evaluate tests over long spans of time that could lead to deterioration of stanshydards Facility is also a key to the validity and success of an item type ~irecshytions complex- language obscure topics fuz~Qata and culturally biased

~Jfiformatioifma~alliead to a highei1eVermiddotof diffiCidty than one desires (A) The IF ID and efficiency statistics of the multiple-choice items of current

forms of the TOEFL are not publicly available information For reasons of security and protection of patented copyrighted materials they must remain behind the closed doors of the ETS development staff Those statistics remain of paramount importance in the ongoing production ofTOEFL items and forms and are the founshydation stones for demonstrating the equatability of forms Statistical indices on retired forms of the TOEFL are available on request for research purposes

The essay portion of theTOEFL undergoes scrutiny for its practicality reliability and facility Special attention is given to reliabilIty since two human scorers must read each essay and every time a third reader becomes necessary (when the two readers disagree by more than one point) it costs ETS more money

(B) In the case of the open-ended responses on the two written tasks on the ESLPT a similar set of judgments must be made Some evaluative impressions of the effectiveness of prompts and passages are gained from informal student and scorer feedback In the developmental stage of the newly revised ESLPT both types of feedshyback were formally solicited through questiQnnaires and interviews That informashytion proved to be invaluable in the revisIon of prompts and stimulus reading passages After each administration now the teacher-scorers provide informal feedshyback on their perceptions of the effectiveness of the prompts and readings

The multiple-choice editing passage showed the value of statistical findings in determining the usefulness of items and pointing administrators toward revisions Following is a sample of the format used

CHAPTER 4- Standardized Testing 79

Multiple-choice editing passage

(1)EYer since supermarkets first appeared they have beentake over ~ world ABC 0

(2) Supermarkets have changed peoples life ~ yet and at the same time changes in ABC

peoples life ~ have encouraged the opening of supermarkets o

The task was to locate the error in each sentence Statistical tests on the experishymental version of this section revealed that a number of the 45 items were found to be of zero IF (no difficulty whatsoever) and of inconsequential discrimination power (some IDs of 15 and lower) Many distractors were of no consequence because they lured no one Such information led to a revision of numerous it~ms and their options eventually strengthening the effectiveness of this section

(C)The GET like its written counterparts in the ESLPT is a test ofwritten ability with a single prompt and therefore questions of practicality and J~~illy~are also largely observational No data are collected from students on their perceptions but the scorers have an opportunity to reflect on the validity ofa given topiC After one sitting a topic is retired which eliininates the possibility of improving a specific topiC but future framing of topics might benefit from scorers evaluations Inter-rater reliability is checked periodically and reader training sessions are modified if too many instances of unreliability appear

5 Specify scoring procedures and reporting formats - ---

A systematic assembly of test items in pre-selected arrangements and sequences all of which are validated to confo~ to an e~pected difficulty level should yield a test that can then be scored accurately and reported back to test-takers and institutions efficiently

(A) Of the three tests being exemplifled here the most straightforward scoring procedure comes from the TO~FL the one with the most complex issues of validashytion deSign and assembly Scores are calculated and reported fora) three sections of the TOEFL (the essay ratings are combined with the Structure and Written Expression score) and (b) a total score (range 40 to 300 on the computer-based TOEFL and 310 to 677 on the paper-and-pencil TOEFL) A separate score (c) for the Essay (range 0 to 6) is also provided on the examinees score record (see simulation of a score record on page 80)

80 CHAPTER 4 Standardized Testing

Facsimile of a TOEFLreg score report

TOEFL Scaled Scores Claudia Y Estudiante Peru ___

19 17 17 177 Listening Structure Writing Reading Total Score

Essay rati ng 30

The rating scale for the essay is virtually the same one that is used for the Test of Written English (see Chapter 9 for details) with a zero level added for no response copying the topic only writing completely off topic or not writing in English

(B) The ESLPT reports a score for each of themiddot essay sections but the rating scale differs between them because in one case the objective is to write a summary and in the other to write a response to a reading ~ch essayi~pd lgtY ~o readet~ ifhFfF js a discrepancy of more than one level a third reader1resolves the differenceThe ~ditiilg section is machine-scanned and -scored with a total score and ~th part-scores for each ofthe grammaticaVrhetorlcal sectionS From these data placement administrators have adequate information to make placements and teachers receive some diagnostic inforshymation on each student in their classes Students do not receive their essays back

(C) Each GET is read by two trained readers who give a score between 1 and 4 according to the following scale

Graduate Essay Test Scoring Guide

Please make no marks on the writers work Write your reader number and score on the front cover of each test booklet

4 Superior The opening establishes context purpose and point of view the body of the essay developsmiddot recommendations-logically and coherently The writer demonshystrates awareness of the complexities in the situation and provides analysis of the probJem offers compelling or common-sense reasons for recommendations made makes underlying assumptions explicit

The writer uses fluent and idiomatic English with few mechanical errors Style reshyveals syntactic maturity is dear and direct is not choppy or over-colloquial nor over-formal stuffy or unfocused Occasional spelling or punctuation errors may be easily attributed to hasty transcription under pressure

3 Competent After an opening that establishes context and purpose the paper unfolds with few lapses in coherence but may have somewhat less clear organization of less explicit transitions than a top-score paper It may have somewhat less compelling logic or slightly less-wellreasoned suggestions than a 4 paper though it will provide reasons for the recommendations made

The writer uses dear fluent and generally idiomatic English but may make minor or infrequent ESL errors (preposition errors dropped articles or verb endings etc) or repeat a single error (eg not punctuate possessive nouns) Occasional lapses of style are offSet by demonstrated mastery of syntax

CHAPTE84 Standardized Testing 81

2 Weak The writer makes somewhat simplistic suggestions not fully supported with reashysons fails to cite key facts offers little analysis of the problem or shows a limited grasp of the situation the given information is copied or listed withlittle integration into argument Points may be random or repetitious Writing may be badly focused with careless use of abstract language resulting in predication errors or illogical sentences

ESL andlor careless mechanical errors are frequent enough to be distracting OR sentences may be choppy style over-casual usage occasionally unidiomatic

1 Inadequate The essay may be disjointed incoherent or minimally developed The writer shows little grasp of the complex issues involved is unable to establish conshytext point of view or purpose in opening of paper or has a poor sense of audience Mechanical andor ESL errors or unidiomatic usages are frequent sentences may be ungrammatical OR correct but short and very simple

The two readers scores are added to yield a total possible score of 2 to 8 Test administrators recommend a score of 6 as the threshold for allowing a student to pursue graduate-level courses Anything below that is accompanied by a recomshymendation that the student either repeat the test or take a remedial course in gradshyuate writing offered in one of several different departments Students receive neither their essays nor any feedback other than the fmal score

6 Perform ongoing construct validation studies

From the above discussion it should be clear that no standardized instrument is expected to be used repeatedly without a ramporou~program of ongoing c~-sectmct valiltiatiOll Any standardized test once developed must be accompanied by sysshy~

tematic periodic corroboration of its effectiveness and by steps toward its improveshyment This rigor is especially true of tests that are produced in equated forms that is forms must be reliable across tests such that a score on a subsequent form of a test-has-the~same validityand-interpretability as its original

(A) The TOEFL program in cooperation with other tests produced by ETS has an impressive program of research Over the years dozens of TOEFL-sponsored research studies have appeared in the TOEFL Monograph Series An early example ofsuch a study was the seminal Duran et aI (1985) study TOEFLfrom a Communicative ViewpOint on Language Proficiency which examined the content characteristics of the TOEFL from a communicative perspective based on current research in applied linguistics and lanshyguage proficiency assessment More recent studies (such as Ginther 2001 Leacock amp Chodorow 2001 Powers et aI 2002) demonstrate an impressive array of scrutiny

(B) For approximately 20 years the ESLPT appeared to be placing students relishyably by means of an essay and a multiple-choice grammar and vocabulary test Over the years the security of the latter became s1lspect and the faculty administrators wished to see some content validity achieved in the process In the year 2000 that process began with a group of graduate students (Imao et aI 2000) in consl1ltation with faculty members and continued to fruition in the form of a new ESLPT reported in lmao (2002) The development of the new ESlPT involved a lengthy process of

82 CHAPTER 4 Standardized Testing

both content and construct validation along with facing such practical issues as scoring the written sections and a machine scorable multiple-choice answer sheet

The process of ongoing validation will no doubt continue as new forms of the editing section are created and as new prompts and reading passages are created for the writing section Such a validation process should also include consistent checks on placement accuracy and on face validity

(C) At this time there is little or no research to validate the GET itself For its conshy struct validation its administrators rely on a stockpile of research on university-level academic writing tests such as theTWEThe holistic scoring rubric and the topics and administrative conditions of the GET are to some extent patterned after that of the TWE In recent years some criticism of the GEf has come from international test-takers (Hosoya 2001) who posit that the topics and time limits of the GET among other facshytors work to the disadvantage of writers whose native language is not English These validity issues remain to be fully addressed in a comprehensive research study

I I

STANDARDIZED IANGUAGE PROFICIENCY TESTING

Tests of language profiCiency presuppose a comprehensive definition of the specific competencies that comprise overall language ability The specifications for the TOEFL provided an illustration of an operational definition of ability for assessment purposes This is not the only way to conceptualize the concept Swain (1990) offered a multidimensional view of profiCiency assessment by referring to three linshyguistic traits (grammar discourse and sociolinguistics) that can be assessed by means of oral multiple-choice and written responses (see Table 41) Swains conshyception was not meant to be an exhaustive analysis of ability but rather to serve as an operational framework for constructing proficiency assessments

Another defmition and conceptualization of profiCiency is suggested by the ACTFL association mentioned earlier ACfFL takes a holistic and more unitary view of proficiency in describing four levels superior advanced intermediate and noviceWithin each level descriptions of listening speaking reading and writing are provided as guidelines for assessment For example the ACfFL Guidelines describe the superior level of speaking as follows

ACTFL speaking guidelines summary superior-level

Superior-level speakers are characterized by the ability to

bull participate fully and effectively in conversations in formal and informal settings on topics related to practical needs and areas of professional andor scholarly interests

bull provide a structured argument to explain and defend opinions and develop effective hypotheses within extended discourse

bull discuss topics concretely and abstractly bull deal with a linguistically unfamiliar situation bull maintain a high degree of linguistic accuracy bull satisfy the linguistic demands of professional andor scholarly life

CHAPTER4 Standardized Testing 83

The other three ACfFL levels use the same parameters in describing progressively lower proficiencies across all four skills Such taxonomie~ have the advantage of considering a number of functions of linguistic discourse but the disadvantage at the lower levels of overly emphasizing test-takers deficiencies

Table 41 Traits of second language proficiency (Swain 1990 p 403)

Trait Grammar Discourse Sociolinguistic

focus on grammatical focus on textual focus on social accuracy within cohesion and appropriateness of sentences coherence language use

Method

Oral structured interview story telling and argumentationpersuasion

role-play ofspeech acts requests offers complaints

scored for accuracy of verbal morphology prepositions syntax

detailed rating for identification logical sequence and time orientation and global ratings for coherence

scored for ability to distinguish formal and informal register

Multiple-choice

sentence-level select the correct form exercise

paragraph-level select the coherent sentence exercise

speech act-Ievelselect the appropriate utterance exercise

(45 items) (29 items) (28 items)

involving verb morphology prepositionsan-d-uther items

Written composition

narrative and letter of persuasion

narrative and letter of persuasion

formal request letter and informal note

scored for accuracy of verb morphology prepositions syntax

detailed ratings much as for oral discourse and global rating for coherence

scored for the ability to distinguish formal and inforJ1lil1 register

FOUR STANDARDIZED lANGUAGE PROFICIENCY TESTS

We now tum to some of the better-known standardized tests of overall language ability or profiCiency to examine some of the typical formats used in commercially available tests We will not look at standardized tests of other specific skills here but that should not lead you to think by any means that proficiency is the only kind of test in the field that is standardized Three standardized oral production tests the

84 CHAPTER 4 Standardized Testing

Test of Spoken English (fSE) the Oral Proficiency Inventory (OPI) and PbonePassreg are discussed in Chapter 7 and the Test of Written English (WE) is covered in ChapterS

Four commercially produced standardized tests of English language proficiency are described briefly in this section the TOEFL the Michigan English Language Assessment Battery (MELAB) the International English Language Testing System (lELTS) and the Test of English for International Communication (fOEICreg) In an appendix to this chapter are sample items from each section of each test When you turn to that appendix use the following questions to help you evaluate these four tests and their subsections

1 What item types are included 2 How practical and reliable does each subsection of each test appear to be 3 Do the item types and tasks appropriately represent a conceptualizatio~ of

language proficiency (ability) That is can you evaluate their construct validity

4 Do the tasks achieve face validity 5 Are the tasks authentic 6 Is there some washback potential in the tasks

Test of English as a Foreign Language (TOEFL)

Producer Educational Testing Service (ETS) Objective To test overall proficiency (language ability) Primary market Almost exclusively US universities and colleges for admission

purposes Type Computer-based (CB) (and two sections are-computer-adaptive)

A traditional paper-based (PB) version is also available Response modes Multiple-choice responses essay Specifications See the box on pp 72-73 Time allocation Up to 4 hours (CB) 3 hours (PB) Internet access wwwtoeflorg

Comments In the North American context the TOEFL is the most widely used comshymercially available standardized test of proficiency Each year the TOEFL test is adminisshytered to approximately 800000 candidates in more than 200 countries It is highly respected because of the thorough program of ongoing research and development conshyducted by ETS The TOEFLs primary use is to set proficiency standards for international students seeking admission to English-speaking universities More than 4200 academic institutions government agencies scholarship programs and licensingcertification agenshycies in more than 80 countries use TOEFL scores By 2004 the TOEFL will include a secshytion on oral production

CHAPTER 4 Standardi~ed Testing 85

Michigan English Language Assessment Battery (MELAB)

Producer English language Institute University of Michigan Objective To test overall proficiency (language ability) Primary market Mostly US and Canadian language programs and colleges

some worldwide educational settings as well Type Paper-based Response modes Multiple-choice responses essay Time allocation 25 to 35 hours Internet access wwwlsaumicheduelimelabhtm

Specifications The MElAB consists of three sections Part 1 a 3D-minute impromptu essay is written on an assigned topic Part 2 a 25-minute multiple-choice listening comshyprehension test is delivered via tape recorder Part 3 is a 100-item 75-minute multipleshychoice test containing grammar doze reading vocabulary and reading comprehension An oral interview (speaking test) is optional

Comments The Ell at the University of Michigan has been producing the MELAB and its earlier incarnation (Michigan Test of English language Proficiency) since 1961 like the TOEFL it serves a North American audience but is also used internationally While its use is not as widespread as the TOEFL its validity is widely respected Because it is cheaper than the TOEFL and more easily obtained it is popular among language schools and institutes Many institutions and companies accept MElAB scores in lieu ofTOEFL scores

International English Language Testing System (IELTS)

Producer Jointly managed by The University of Cambridge local Examinations Syndicate (UClES) The British Council and lOP Education Australia

Objective To test overall proficiency (language ability) Primary-market Australian British Canadian and New Zealand academic

institutions and professional organizations American academic institutions are increasingly accepting IELTS for admissions purposes

1)rpe Computer-based (for the Reading and Writing sections) papershybased for the listening and Speaking modules

Response modes Multiple-choice responses essay oral production Time allocation 2 hours 45 minutes Internet access httpwwwieltsorgl

httpwwwudesorguk httpwwwbritishcouncilorg

Specifications Reading candidates choose between academic reading or general training reading (60 minutes) Writing the same option academic writing or general training writing (60 minutes) Listening four sections for all candidates (30 minutes) Speaking five sections for all candidates (1015 minutes)

86 CHAPTER 4 Standardized Testing

Comments The University of Cambridge local Examinations Syndicate (UCLES) has been producing English language tests since 1858 Now with three organizations cooperatshying to form the IELTS more than a million examinations are administered every year In 2002 a computer-based version of the Reading and Writing modules of the IELTS became available at selected centers around the world The other sections are administered locally by an examinet The paper-based IELTS remains an option for candidates The IELTS retains the distinct advantage of requiring all four skills in the test-takers performance

Test of English for International Communication (TOEICreg)

Producer The Chauncey Group International a subsidiary of Educational Testing Service

Objelttive To test overall proficiency (langlJage ability) Primary market Worldwide business commerce and industry contexts

(workplace settings) Type Computer-based and paper-based versions Response modes Multiple-choice responses Time allocation 2 hours Internet access httpwwwtoeiccom

Specifications Listening Comprehension 100 items administered by audiocassette Four types of task statements questions short conversations and short talks (approxishymately 45 minutes) Reading 100 items Three types of task cloze sentences error recogshynition and reading comprehension (75 minutes)

Comments The TOEIC has become a very widely used international test of English proficiency in workplace settings where English is required for job performance The conshytent includes many different employment settings such as conferences presentations sales ordering shipping schedules reservations (etters and memoranda It is approprishyate to use in educational settings where vocational or workplace English courses are being offered

sect sect sect sect sect

The construction of a valid standardized test is no minor accomplishment whether the instrument is large- or small-scale The designing of specifications alone as this chapter illustrates requires a sophisticated process of construct valishydation coupled with considerations of practicality Then the construction of items and scoringinterpretation procedures may require a lengthy period of trial and error with prototypes of the final form of the testWith painstaking attention to all the details of construction the end product can result in a cost-effective timeshysaving accurate instrument Your use of the results of such assessments can provide useful data on learners language abilities But your caution is warranted as well for all the reasons discussed in this chapter The next chapter will elaborate on what lies behind that need for a cautious approach to standardized assessment

CHAPTER4 Standardized Testing 87

EXERCISES

[Note (I) Individual work (G) Group or pair work (C) Whole-class discussion]

1 (C) Tell the class about the worst test experience youve ever had Briefly anamiddot lyze what made the experience so unbearable and try to come up with sugshygestions for improvement of the test andor its administrative conditions

2 (G) In pairs or small groups compile a brief list of pros and cons of standardshyized testing Cite illustrations of as many items in each list as possible Report your lists and examples to the rest o~ the class

3 (I) Select a standardized test that you are quite familiar with (probably a recent experience) Mentally evaluate that test using the five principles of practicality reliability validity authenticity and washback Report yourevaluashytion to the class

4 (G) The appendix to this chapter provides sample items from Jour different tests of language proficiency In groups one test for each group analyze your test for (a) content validity (b) face validity and (c) authenticity

5 (C) Do you think that the sample TOEFL reading passage about pirates (pages 74-75) and the Graduate EssayTest prompt (pages 76-77) about a school board hiring committee have any culture bias Discuss this and other cultural biases you have noticed in tests Is it possible to design a test that is completely free of culture bias

6 (CG) Compare the differences in conceptualization of language proficiency represented by Swains model the TOEFL and the ACfFL philosophy Which one best represents current thinking about communicative language ability What are the strengths and weaknesses of each approach

FORYOlIILEURTHER READING

Gronlund Norman E (1998) Assessment of student achievement Sixth Edition Boston Allyn and Bacon

Gronlunds classic also mentioned in Chapter 3 offers a concise overview of features of standardized tests offering definitions and examples of the statistical considerations in interpreting scores His approach is unbiased cleady written and accessible to those who might fear the mathematics of standardized testing

Phillips Deborah 2001 Long1nan introductory course for the TOEFL test White Plains NY Pearson Education

A careful examination of this or any other reputable preparation course for a standardized language test is well worth a students time Note especially how the book acquaints the user with the specifications of the test and offers a number of useful strategie~ that can be llsed in preparation for the test and during irs adn1inistration

88 CHAPTER 4 Standardized Testing

APPENDIX TO CHAPTER 4

Commercial Proficiency Tests Sample Items and Tasks

Test of English a~ a Foreign Language (TOEFLreg)

Listening r

Part A

In this section you will hear short conversations between two people In some ofthe conversations each person speaks only once In other conversations one or both of the people speak more than once Each conversation is followed by one questionabQlt it Each question in this part has four answer choices You should click on the best answer to each question Answer the questions on the basis of what is stated or implied by the speakers Here is an example On the computerscreen you will see

[man and woman talking]

On the recording you will hear

(woman) Hey wheres your sociology book (man) At home Why carry it around when were just going to be taking

a test (woman) Dont you remember Professor Smith said we could us it during

the test (man) Ohl no Well Ive still got an hour right Im so glad I ran into you

You wiII then see and hear the question before the answer choices appear

What will the man probably do next

o Begin studying for the sociology test o Explain the problem to his professor o Go home to get his textbook o Borrow the womans book

To choose an answer you will click on an oval The oval next to that answer will darken After you click on Next and Confirm Answer the next conversation will be presented

Part B

In this section you will hear several longer conversations and talks Each conversation or talk is followed by several questions The conversations talks and questions will not be repeated The conversations and talks are about a variety of topics You do not need speshycial knowledge of the topics to answer the questions correctly Rather you should answer each question on the basis of what is stated or implied by the speakers in the conversashytions or talks

For most of the questions you will need to click on the best of four possible answers Some questions will have special directions The special directions will appear in a box on the computer screen Here is an exampie ot a conversation and some questions

CHAPTER 4 Standardized Testing 89

Marine Biology (narrator) Listen to part of a discussion in a marine biology class

(professor) A few years ago our local government passed a number of strict environmental laws As a result Sunrise Beach looks nothing Ii ke it did ten years ago The water is cleaner and theres been a tremendous increase in all kinds of marine life which is why were going there on Thursday

(woman) I dont know if I agree that the water quality has improved I mean I was out there last weekend and it looked all brown It didnt seem too clean to me

(professor) Actually the color of the water doesnt always indicate whether its polluted The brown color you mentioned might be a result of pollution or it can mean a kind of brown algae is growing there Its called devils apron and it actually serves as food for whales

(man) So when does the water look blue (professor) Well water thats completely unpolluted is actually colorless But

it often looks bluish-green because the sunlight can penetrate deep down and thats the color thats reflected

(woman) But sometimes it looks really green Whats that about (professor) Ok well its the same principle as with devils apron the

water might be green because of different types of green algae there-gulfweed phytoplankton You all should finish reading about algae and plankton before we go In fact those are the types of living things Im going to ask you to be looking for when were there

Now get ready to answer the questions

What is the discussion mainly about

o The importance of protecting ocean environments o The reasons why ocean water appears to be different colors o The survival of whales in polluted water o The effect that colored ocean water has on algae

To choose an answer click on an oval The oval next to that answer will darken After you click on Next and Confirm Answer the next question will be presented

According to the professor what can make ocean water look browngt

o Pollution o Cloudy Skies o Sand o Algae

Click on 2 answers

To choose your answers you will click on the squares An XII wiii appear in each square

bullbullbullbullbullbullbull

90 CHAPTER 4 Standardized Testing

Structure and Written Expression This section measures the ability to recognize language that is appropriate for standard written English There are two types ofquestions in this section In the first type ofquestion there are incomplete sentences Beneath each sentence there are four words or phrases

Directions CIiSk on the one word or phrase that best completes the sentence

The colum~ine flower __ to nearly all of the United States can be raised from seed in almost any garden

native how native is how native is it is native

Time Help Confirm

After you click on Next and Confirm Answ~ the next question willbe presented

The second type of question has four underlined words or phrases You will choose the one underlined word or phrase that must be changed for the sentence to be correct

Directions Click on the one underlined word or phrase that must be changed for the senshytence to be correct

One of the most difficult problems in understanding sleep is determining what the funcshytions of sleep ~

lime Help Confirm

Clicking on an underlined word or phrase will darken it

Reading This section measures the ability to read and understand short passages similar in topic and style to those that students are likely to encounter in North American universities and colleges This section contains reading passages and questions about the passages There are several different types of questions in this section In the Reading section you will first have the opportunity to read the passage

The temperature of the Sun is over 10000 degrees Fahrenheit at the surface but it rises perhaps more than 270000000 at the center The Sun is so much hotter than the Earth that matter can exist only as a gasl except perhaps at the core In the core of the Sun the pressures are so great that despite the high temperature there may be a small solid core However no one really knows since the center of the Sun can never be directly observed ~ Solar astronomers do know that the Sun is divided into five general layers or zones Starting at the outside and going down into the Sun the zones are the corona chromoshysphere hotosphere convection zone and finally the core The first three zones are reshygarded as the Suns atmosphere But since the Sun has no solid surface it is hard to middottell where the atmosphere ends and the main body of the Sun begins

The Suns outermost layer begins about 10000 miles above the visible surface and goes outward for millions of miles This is the only part of the Sun that can be seen during an eclipse such as the one in February 1979 At any other time the corona can be seen

bullbullbullbullbullbullbull

bull bullbullbullbullbullbull

CHAPTER 4 Standardized Testing 91

only when special instruments are used on cameras and telescopes to block the light from the photosphere

The corona is a brilliant pearly white filmy light about as bright as the full Moon Its beautiful rays are a sensational sight during an eclipse The coronas rays flash out in a brilliant fan that has wispy spikelike rays near the Suns north and south poles The corona is generally thickest at the Suns equator The corona is made up of gases streamshying outward at tremendous speeds that reach a temperature of more than 2 million deshygrees Fahrenheit The gas thins out as it reaches the space around the planets By the time the gas of the corona reaches the Earth it has a relatively low density

When you have finished reading the passage you will use the mouse to click on Proceed Then the questions about the passage will be presented You are to choose the one best anshyswer to each question Answer all questions about the information in a passage on the basis ofwhat is stated or implied in that passage Most ofthe questions will be multiple-choice questions To answer these questions you will click on a choice below the question

With what topic is paragraph 2 mainly concerned

o How the Sun evolved o The structure of the Sun o Why scientists study the Sun o The distaflce of the Sun from the planets

Paragraph 2 is marked with an arrow (~)

You will see the next question after you click on Next

To answer some questions you will click on a word or phrase Here is an example

Look at the word one in the passage Click on the word or phrase in the bold text that one refers to To answer you can click on any part of the word or phrase in the passage Jour choice will darken to show which word you have chosen

The Suns outermost layer begins about 10000 miles above the visible surface and goes outward for millions of miles This is the only part of the Sun that can be seen durshying an eclipse such as the one in February 1979 At any other time the corona can be seen only when special instruments are used on cameras and telescopes to block the Iight from the photosphere

You will see the next question after you click on~ To answer some q~estions you will click on a sentence in the passage Here is an example

~ The corona is a brilliant pearly white filmy light about as bright as the full Moon Its beautiful rays are a sensational sight during an eclipse The coronas rays flash out in a brilliant fan that has wispy spikelike rays near the Suns north and south poles The corona is generally thickest at the Suns equator ~ The corona is made up of gases streaming outward at tremendous speeds that reach a temperature of more than 2 million degrees Fahrenheit The gas thins out as it reaches the space around the planets By the time the gas of the corona reaches the Earth it has a relatively low density

bull bullbullbullbullbullbull

92 CHAPTER 4 Standardized Testing

Click on the sentence in paragraph 4 or 5 in which the author compares the light of the Suns outermost layer to that ofanother astronomical body Paragraphs 4 and 5 are marked with arrows (~)

To answer some questions you will click on a square to add a sentence to the passage Here is an example -The following sentence can be added to paragraph 1

At the center of the Earths solar system lies the Sun

Where would it best fit in paragraph I Click on the square to add the sentence to the paragraph

D The temperature of the Sun is over 10000 degrees Fahrenheit at the surface but it rises to perhaps morethan 27000000deg at the center 0 The Sun is so much hotter than the Earth that matter can exist only as a gasi except p~rHapsatth~ c6relp the c~re of the ii Sun the pressures are so great that despite the high temperature there may be a small solid core D However no one really knows since the center of the Sun can never be directly observed D 0100

When you click on a square the sentence will appear in the passage at the place you have chosen You can read the sentence added to the paragraph to see if this is the best place to add it You can click on another square to change your answer The sentence will be added and shown in a dark box

Writing In this section you will have an opportunity to demonstrate your ability to write in Enshyglish This includes the ability to generate and organize ideas to support those ideas with examples or evidence and to compose in standard written English in response to an asshysigned topic You will have 30 minutes to write your essay on that topic You must write on the topic you are assigned An essay on any other topic will receive a score of 0 Read the topic below and then make any notes that will help you plan your response Begin typing your response in the box at the bottom of the screen or write your answer on the answer sheet provided to you

Following is a sample topic

Do you agree or disagree with the following statemenH

Teachers should make learning enjoyable and fun for their students

Use specific reasons and examples to support your opinion

CHAPTER 4 Standarczed Testing 93

Michigan English Language Assessment Battery (MELAB)

Composition The time limit for the composition is 30 minutes You must write on only one of the top~

ics below If you write about something else your composition paper will not be graded and you cannot be given a final score If you do not understand the topics ask the exam~ iner to explain or to translate them You may be asked to give your opinion ofsomething and explain why you believe this to describe something from your experience or to exshyplain a problem and offer possible solutions You should write at least one page Some sample topics are

1 What do you think is your countrys greatest problem Explain in detail and tell what you think can be done about it

2 What are the characteristics of a good teacher Explain and give examples 3 An optimist is someone who sees the good side of things A pessimist sees the

bad side Are you an optimist or a pessimist Relate a personal experience that shows this

4 In your opinion are the benefits of space exploration really worth the enormous costs Discuss

Most MELAB compositions are one or two pages long (about 200-300 words) If your paper is extremely short (less than 150 words) your composition will be given a lower score Before you begin writing you might want to take 2 or 3 minutes to plan your comshyposition and to make a short outline to organize your thoughts Such outlines will not be graded they are only to help you You should use the last 5 minutes to read through your composition and to make changes or corrections

Your composition will be graded on how clearly you express yourself in English and on the range of English you are able to use and your control in doing so This means your composition should be well organized your arguments should be fully developed and you should show a range ofgrammatical structures and broad vocabulary Compositions that consist only of very short sentences and very simple vocabulary cannot be given the

middothighest scores If errors are not frequent and if they do not confuse your meaning they will not lower your score very much

Listening Now you will hear a short lecture You may take notes during the lecture Following the lecture you will be asked some questions about it

Therell be a two-week exhibit of the paintings of the little-known master Laura Bernhart at the Claire Osmond Galleries starting on the fifteenth of the month and running through the thirtieth Bernharts known for her innovative designs in abstract expressionism Though a true original she declared a spiritual heritage from Salvador Dali the famous Spanish painter Since Bernhart lived a rather solitary life and died while only in her twenties few people are aware of her works This showing at the Osmond Galleries will provide many with an introduction to her works

10 Where is the exhibit a the Art Museum b the Dali Galleries c the Osmond Galleries

94 CHAPTER 4 Standardized Testing

11 What is Bernhart known for a her copies of Dalis paintings b the originality of her designs c her exhibitions

12 What will going to the exhibit allow most people to do a to see Saivador Dalis paintings b to see Bernharts works for the first time c to learn about Spanish art

Grammar

1 What did the teacher just tell you

She reminded our notebooksI a us to bring b that we bring c our bringing d we should bring

2 Is Bill a good dancer

Not really __ he tries very hard a in spite of h despite c even though d while

3 your clothes are all wet1

Yes I didnt come __ the rain soon enough a away to b over to c down with d in from

Cloze In years to come zoos will not only be places where animals are exhibited to the public but repositories where rare species can be saved from extinction (7) captive breeding The most powerful force (8) the future of many animals-and of zoos-is the decline of the wild (9) even zoo directors would argue that (10) are better places for animals than the fields and forest of their native (11) yet zoos may be the last chance for some creatures that would otherwise pass qUietly into oblivion

7 a through c from b of d damage

8 a bringing c to b that d influencing

9 a But c Not b So d Then

10 a where c even b zoos d wilds

11 alands c residence b life d field

CHAPTER 4 Standardized Testing 95

Vocabulary

12 Mark has a flair for writing a need b purpose c talent d dislike

13 Bill Collins launched his restaurant last June a moved b started c sold d bought

14 John will not accept the censure a burden b blame c credit d decision

15 I cant think of the answer Can you give me a __ a hint b token c taste d gaze

16 Because fewer people are taking expensive vacations the tourist industry is in a a choke b grope c grumble d slump

17 I disagree with a few of his opinions but __ we agree a deliberately b conclusively c essentially d immensely

Reading The influenza virus is a single molecule built from many millions of single atoms You must have heard of the viruses which are sometimes called living molecules While bacteria can be considered as a type of plant secreting pOisonous substances into the body of the organism they attack viruses are living organisms themselves We may conshysider them as regular chemical molecules since they have a strictly aefined atomic strucshyture but on the other hand we must also consider them as being alive since they are able to multiply in unlimited quantities

18 According to the passage bacteria are a poisons

b larger than viruses c very small d plants

96 CHAPTER 4 Standardized Testing

19 The writer says that viruses are alive because they a have a complex atomic structure b move c multiply d need warmth and light

20 The atomic structure of viruses a is -tJIariable b is strictly defined c cannot be analyzed chemically d is more complex than that of bacteria

International English Language Testing System (fELTS)

I

listening

The Listening Module has four sections The first two sections are concerned with social needs There is a conversation between two speakers and then a monologue For examshyple a conversation about travel arrangements or decisions on a night out and a speech about student services on a university campus or arrangements for meals during a confershyence The final two sections are concerned with situations related more closely to educashytional or training contexts For example conversation between a tutor and a student about an assignment or between three students planning a research project and a lecture or talk ofgeneral academic interest All the topics are ofgeneral interest and it makes no difference what subjects candidates study Tests and tasks become more difficult as the sections progress A range of English accents and dialects are used in the recording which reflects the international usage of IELTS

Academic Reading [A 7S0-word article on-th-e- topic of Wind Power in the US with a short glossary at the end]

Questions 1-5

Complete the summary below

Choose your answers from the box below the summary and write them in boxes 1-5 on your answer sheet Note There are more words or phrases than you will need to fill the gaps You may use any word or phrase more than once

Example The failure during the late 1970s and early 19805 of an attempt to establish a widespread wind power industry in the United States resulted largely from the (1) bull in oil prices during this period The industry is now experiencing a steady (2) due to improveshyments in technology and an increased awareness of the potential in the power of wind The wind turbines that are now being made based in part on the (3) of wide- ranging research in Europe are easier to manufacture and maintain than their predecesshysors This has led wind-turbine makers to be able to standardise and thus minimize (4) There has been growing (S) of the importance of wind power as an energy source

CHAPTER 4 Standardized Testing 97

criticism stability skepticism success operating costs decisions design costs fall effects production costs growth decline failure recognition results

Questions 6-1 0 Look at the following list of issues (Questions 6-10) and implications (A-C) Match each issue with one implication Write the appropriate letters A-C in boxes 6-10 on your anshyswer sheet

Example The current price of one wind-generated kilowatt Answer

6 The recent installation of systems taking advantage of economies of scale

7 The potential of meeting one fifth of current U5 energy requirements by wind power

8 The level of acceptance of current wind turbine technology

9 A comparison of costs between conventional and wind power sources

10 The view of wind power in the European Union

Implications

A provides evidence against claims that electricity produced from wind power is relatively expensive

B supports claims that wind power js an important source of energy

C opposes the view that wind power technology requires further-development

General Training Reading Read the passage on Daybreak trips by coach and look at the statements below On your answer sheet write

TRUE if the statement is true FALSE jf the statement is false

NOlGIVEN if the information is not given in the leaflet

1 MiIlers Coaches owns Cambridges Cam bus fleet

2 Premier is an older company than Millers

3 Most of the Daybreak coaches are less than 5 years old

4 Daybreak fares are more expensive than most of their competitors

5 Soft drinks and refreshments are served on most longer journeys

6 Smoking is permitted at the rear of the coach on longer journeys

7 Tickets must be bought in advance from an authorised Daybreak agent

6 Tickets and seats can be reserved by phoning the Daybreak Hotline

9 Daybreak passengers must join their coach at Cambridge Drummer Street

10 Daybreak cannot guarantee return times

98 CHAPTER 4 Standardized Testing

FROM CAMBRIDGE AND SURROUNDING AREA

SPRING IS INTHEAIR

Welcome to our Spring Daybreak programme which continues the tradition of offering unbeatable value for money day trips and tours All the excursions in this brochure will be operated by Pr~mier Travel Services Limited or Millers Coaches both companies are part of the CHLGroup owners of Cambridges Cambus fleet

WERE PROUD OF OUR TRADITION

Premier was established in 1936 the Company now offers the highest standards of coaching in todays competitive operating environment Miller has an enviable reputation stretching back over the past 20 years offering coach services at realistic prices Weve traveled a long way since our early days of pre-war seaside trips Now our fleet of 50 modern coaches (few are more than five years old) operate throughout Britain and Europe but were pleased to still maintain the high standards of quality and service the trademark of our founders nearly sixty years ago

EXCLUSIVE FEATURES

Admission-inclusive fares All Daybreak fares (unless specifically otherwise stated) include admission charges to the attractions shows and exhibits we visit Many full-day scenic tours are accompanied by a fully trained English Tourist Board Blue Badge guide or local experienced driverguide Some Daybreaks include lunch or afternoon tea Compare our admission inclusive fares and see how much you save Cheapest is not the best and value for money is guaranteed If you compare our bargain Daybreak fares beware--most of our competishytors do not offer an all-inclusive fare

SEAT RESERVATIONS

We value the freedom of choice so you can choose your seat when you book The seat reservation is guaranteed a-nd remains yours at all times when aboard the coach

NO SMOKING COMFORT

With the comfort of our passengers in mind coaches on all our Daybreaks are no smokshying throughout In the interests of fellow passengers comfort we kindly ask that smokers observe our no smoking policy On scenic tours and longer journeys ample refreshment stops are provided when of course smoking is permitted

YOUR QUESTIONS ANSWERED

Do I need to book Booking in advance is strongly recommended as all Daybreak tours are subject to demand Subject to availability stand-by tickets can be purchased from the driver

What ti me does the coach leave The coach departs from Cambridge Drummer Street (Bay 12 adjacent to public toilets) at the time shown There are many additional joining points indicated by departure codes in the brochure If you are joining at one of our less popular joining points you will be adshyvised of your pick-up time (normally by telephone) not less than 48 hours before deparshyture In this way we can minimize the length of pick-up routes and reduce journey times for the majority of passengers

CHAPTER 4 Standardized Testing 99

What time do we get back An approximate return time is shown for each excursion The tim~s shown serve as a guide but road conditions can sometimes cause delay If your arrival will be later than advertised your driver will try to allow for a telephone call during the return journey

Where can I board the coach All the Daybreaks in the brochure leave from Cambridge Drummer Street (Bay 12 adjashycent to public toilets) at the time shown Many Daybreaks offer additional pick-ups for pre-booked passengers within Cambridge and the surrounding area This facility must be requested at the time of booking

Academic Writing Writing Task 1 You should spend about 20 minutes on this task

The graph below shows the different modes of transport used to travel to and from work in one European city in 1950 1970 and 1990

[graph shown here]

Write a report for a university lecturer describing the information shown below You should write at least 150 words

Writing Task 2 You should spend about 40 minutes on this task

Present a written argument or case to an educated reader with no specialist knowledge of the folowing topic

It is inevitable that as technology develops so traditional cultures must be lost Technolshyogy and tradition are incompatible-you cannot have both together

To what extent do you agree or disagree with this statement Give reasons for your answer You should write at least 250 words You should use your own ideas knowlshyedge and experience and support your arguments with examples and relevant evidence

General Training Writing Writing Task 1 You should spend about 20 minutes on this task You rent a house through an agency The heating system has stopped working You phoned the agency a week ago but it has still not been mended Write a letter to the agency Explain the situation and teil them what you want them to do about it

You should write at least 150 words You do NOT need to write your own address

Begin your letter as follows

Dear - ___-I

Writing Task 2 You should spend about 40 minutes on this task As part ofa class assignment you have to write about the following topic

100 CHAPTER 4 Standardized Testing

Some businesses now say that no one can smoke cigarettes in any of their offices Some governments have banned smoking in all public placesThis is a good idea but it takes away some of our freedom

Do you agree or disagree Give reasons for your answer You should write at least 250 words

Speaking In each ofthe three parts of the speaking module a specific function is fulfilled In Part 1 the candidates answer general questions about themselves their homes or families their jobs or studies their interests and a range ofsimilar familiar topic areas This part lasts between four and five minutes In Part 2 the candidate is given a verbal prompt on a card and is asked to talk on a particular topic The candidate has one minute to prepare before speaking at length for between one and two minutes The examiner then asks one or two wind-down questions In Part 3 the examiner and candidate engage in a discusshysion of more abstract issues and concepts which are thematically linked to the topic prompt in Part 2 The discussion lasts between four and five minutes

All interviews are recorded on audiocassette Here is a sample ofa Part 2 topic

Describe a teacher who has greatly influenced you in your education

You shou Id say

where you met them what subject they taught what was special about them

and explain why this person influenced you so much

You will have to talk about the topic for 1 to 2 minutes You have 1 minute to think about what you are going to say You can make some notes if you wish

Test of English for International Communication (TOEICreg)

listening

Part 1 Photographs Directions For each question you will see a picture in your test book and you will hear four short statements The statements will be spoken just one time They will not be printed in your test book so you must listen carefully to understand what the speaker says When you hear the four statements look at the picture in your test book and choose the statement that best describes what you see in the picture Then on your answer sheet find the number of the question and mark your answer

[photograph of a scientist looking through a microscope]

You will hear Look at the picture marked number 1 in your test book

(A) Shes speaking into a microphone (B) Shes put on her glasses (C) She has both eyes open (D) Shes using a microscope

CHAPTER 4 Standardized Testing 101

Part 2 Question-Response Directions In this part of the test you will hear a question or statement spoken in Enshyglish followed by three responses also spoken in English The question or staten1ent and the responses will be spoken just one time They will not be printed in your test book so you must listen carefully to understand what the speakers say You are to choose the best response to each question or statement

Question 1 You will hear Ms Morikawa has worked here for a long time hasnt she

(A) At three oclock (B) No Ive lost my watch (C) More than ten years

Question 2 You will hear Which of these papers has a wider circulation

(A) The morning edition (B) Get more exercise (C) By messenger

Part 3 Short Conversations Directions In this part of the test you will hear short conversations between two people The conversations will not be printed in your test book You will hear the conversations only once so you must listen carefully to understand what the speakers say In your test book you will read a question about each conversation The question will be followed by four answers You are to choose the best answer to each question and mark it on your answer sheet

Question 1 (Man) We should think about finding another restaurant for lunch (Woman) Why The food and service here are great

(Man) Yes but the prices are going up every week

You will read Why is this man unhappy with the restaurant

(A) It is too noisy (B) It is too expensive (C) It is too crowded (D) It is too difficult to find

Question 2 (Woman A) How was Dr Borgs recent trip to Singapore (Woman B) She enjoyed the tour of the port very much (Woman A) They say its one of the most active in Asia

You will read 2 What did Dr Borg find interesting

(A) The tourist center (B) The airport (C) The musical performance (D) The harbor

Part 4 Short Talks Directions In this part of the test you vill hear several short talks Each will be spoken just one time They will not be printed in your test book so you must listen carefully to understand and remember what is said In your test book you will read two or more questions about each short talk The questions will be followed by four answers You are to choose the best answer to each question and mark it on your answer sheet

102 CHAPTER 4 Standardized Testing

You will hear Questions 1 and 2 refer to the following announcement

Good afternoon and welcome aboard Nordair Flight 857 from Copenhagen to Bangkok with intermediate stops in Dubai and Calcutta We are preparing for departure in a few minutes At this time your seat back should be returned to its full upright position and your seat belt s~ould be fastened OUf anticipated total flying time to Dubai is six hours and twenty-five minutes I hope you enjoy the flight You will hecJr Now read question 1 in your test book and answer it You will read 1 What is the final destination of the flight

(A) Bangkok (B) Copenhagen (C) Dubai (O) Calcutta

You will hear Now read question 2 in your test book and answer it You will read 2 What will happen in a few minutes

(A) The flight will land in Dubai I

(B) The passengers will board the plane (C) The plane will take off (0) The gate number will be announced

Reading In this section of the test you will have the chance to show how well you understand written English There are three parts to this section with special directions for each part

Part 4 Incomplete Sentences Directions This part of the test has incomplete sentences Four words or phrases marked (A) (8) (e) (D) are given beneath each sentence You are to choose the one word or phrase that best completes the sentence Then on your answer sheet find the number of the question and mark your answer

1 Mr Yangs trip will __ him away from the office for ten days (A) withdraw (B) continue (C) retain (0) keep

2 The company that Marie DuBois started now sells __ products throughout the world (A) its (B) it (C) theirs (D) them

3 If your shipment is not delivered __ Tuesday you can request a full refund for the merchandise (A) at (B) by (C) within (D) while

CHAPTER 4 Standardized Testing 103

Part 6 Error Recognition Directions In this part ofthe test each sentence has four words or phrases underlined The four underlined parts of the sentence are marked (A) (B) (C) (D) You are to identify the one underlined word or phrase that should be corrected or rewritten Then on your answer sheet find the number of the question and mark your answer

1 The pamphlet contains some importance information about the current exhibit ABC D

2 No matter how Jong it taking to finish the annual report it must be done properly ABC D

3 The popularity of jogging appears to have decreased since the past couple of years ABC D

Part 7 Reading Comprehension Directions The questions in this part of the test are based on a selection of reading mateshyrials such as notices letters) forms newspaper and magazine articles) and advertisements You are to choose the one best answer (A) (B) (C) or (OJ to each quesshytion Then on your ariswefsheelfindthe number of the qUestion andmcirkyour answer Answer all questions following each reading selection on thebasis of what is stated or implied in that selection

The Museum ofTechnology is a hands-on museum designed for people to experience science at w()rk~ Visitors are encouraged to use test and handle the objects o~ display Special demonstrations are scheduled for the first and second Wednesdays of each month at 1330 Open Tuesday-Friday 1200-1630 Saturday 1000-1730 and Sunday 11 00-1630

1 When during the month can visitors see special demonstrations (A) Every weekend (B) The first two Wednesdays (C) One afternoon a week (D) Every other Wednesday

Questions 2 and 3 refer to the followi ng notice

NOTICE If you are unable to work because of an extended illness or injury that is not workshyrelated you may be entitled to receive weekly benefits from your employer or the firms insurance company To claim benefits you must file a claim form within thirty days of the first day of your disability Before filing the claim you must ask your doctor to fill in the Doctors Statement on the claim form stating the period of disability

3 To whom is this notice addressed (A) Employers (8) Doctors (C) Employees (D) When paying the bill

4 When must the claim form be filed (A) On the first of the month (8) On the thirtieth of the month (C) On the first day ofdisabifity (D) Within 30 days of the start of disability

Page 5: Standardized Testing Chapter 4 Brown

70 CHAPTER 4 Standardized Testing

The first is a test of general language ability or profidencyThe second is a place ment test at a universityAnd the third is a gate-keeping essay test that all prospecshytive students must pass in order to take graduate-level courses As we look at the steps one by one you will see patterns that are consistent with those outlined in the previous two chapters for evaluating and developing a classroom test

1 Determine ihe purpose and objectives of the test

Most standardized tests are expected to provide high practicality in administration and scoring without unduly compromising validity The initial outlay of time and money for such a test is Significant but the test would be used repeatedly It is therefore imporshytant for its purpose and objectives to be stated specifically Lets look at the three tests

(A) The purpose of the TOEFL is to evaluate the English profiCiency of people whose native language is not English (TOEFL Test and Score Manual 2001 p 9) More specifically theTOEFL is designed to help institutions of higher learning make valid decisions concerning English language profiCiency lin terms of lthelr] own requirements (p 9) Most colleges and universities in the United States use TOEFL scores to admit or refuse international applicants for admission Various cut-off scores apply but most institutions require scores from 475 to 525 (paper-based) or from 150 to 195 (computer-based) in order to consider students for admissionThe high-stakes gate-keeping nature of the TOEFL is obvious

(B) The ESLPT referred to in Chapter 3 is designed to place already admitted students at San Francisco State University in an appropriate course in academic writing with the secondary goal of placing students into courses in oral production and grammar-editing While the tests primary purpose is to make placements another desirable objective is to provide teachers with some diagnostic information about their students on the first day or two of classThe ESLPT is locally designed by university faculty and staff

(C) The GEfanother test designed at SFSU is given to prospective graduate students-both native and non-native speakers-in all disciplines to determine whether their writing ability is sufficient to permit them to enter graduate-level courses in their programs It is offered at the beginning of each term Students who fail or marginally pass the GET are technically ineligible to take graduate courses in their field Instead they may elect to take a course in graduate-level writing of research papers A pass in that course is equivalent to passing the GET

As you can see the objectives of each of these tests are specific The content of each test must be designed to accomplish those particular ends TIlis first stage of goal-setting might be seen as one in which the consequential validity of the test is foreshymost in the mind of the developer each test has a specific gate-keeping function to perfonn therefore the criteria for entering those gates must be specified ~ccurately

2 Design test specifications

Now comes the hard part Decisions need to be made on how to go about structurshying the specifications of the test Before specs can be addressed a comprehensive

CHAPTER4 Standardized Testing 71

progratn of research must identify a set of constructs underlying the test itself This stage of laying the foundation stones can occupy weeks months or even years of effort Standardized tests that dont work are often the product of short-sighted conshystruct validation Lets look at the three tests again

(A) Construct validation for the TOEFL is carried out by the TOEFL staff at ETS under the guidance of a Policy Counell that works with a Committee of Examiners that is composed of appointed external university faculty linguists and assessment spedaJists Dozens of employees are involved in a complex process of reviewing curshyrentTOEFL specifications cOmmissioning and developing test tasks and items assemshybling forms of the test and performing ongoing exploratory research related to formulating new specs Reducing such a complex process to a set ofsimple steps runs the risk of gross overgeneralization but here is an idea of how aTOEFL is created

Because the TOEFL ismiddot a proficiency test the fIrst step in the developmental process is to define the construct of language proficiency First it should be made clear that many assessment specialists such as Bachman (1990) and Palmer (Bachman amp Palmer 1996) prefer the term ability to proficiency and thus speak of language abllity as the overarching concept The latter phrase is more conSistent they argue with our understanding that the specific components of language ability must be assessed separately Others such as the American Council on Teaching Foreign Languages (ACIFL) still prefer the term proficiency because it connotes more of a holistic unitary trait view of language ability (Lowe 1988) Most current views accept the ability argument and therefore strive to specify and assess the many components of language For the purposes of consistency in this book the term proficiency will nevertheless be retained with the above caveat

How you view language will make a difference in how you assess language proshyficiency After breaking language competence down into subsets of listening speaking reading and writing eapoundli~p~ifQ(m~I~~ Jll()4~ can be examined on a conshytiiiuum of linguistic units pho~~logy (p~onunciation) and orthography (spelling) words OeXicon) sentences (gtammar) discourse and pragmatic (sociolinguistic contextual functional cul~j features of language

How will the TOEFL sample from at) these possibilities Oral production tests can be tests of overall conversational fluency or pronunciation of a particular subset of phonolOgy and can take the form of imitation structured responses or free responses Listening comprehension tests can concentrate on a particular feature of language or on overalllistenins for general meaning Tests of realtling can cover the range of language units and can aim to test comprehension of long or short passhysages single sentences or even phrases and words Writing-tests can take on an open-ended form with free composition or be structured to elicit anything from correct spelling to discourse-level competence Are you overwhelmed yet

From the sea of potential performance modes that could be sampled in a test the developer must select a subset on some systematic basis To make a very long story short (and leaving out numerous controversies) the TOEFL had for many years included three types of performance in its organizationai specifications listening strucshyture and reading all of vtuch tested comprehension through standard multiple-choice

72 CHAPTER 4 Standardized Testing

tasks In 1996 a major step was taken to include written production in themiddot computershybased TOEFL by adding a slightly modified version of the already existing Test of Written English (TWE) In doing so some face validity and content validity were improved along with of course a significant increase in administrative expense Each of these four major sections is capsulized in the box below (adapted from the descripshytion of the current computer-based TOEFL at wwwroefLorg) Such descriptions are not strictly speaking specifications which are kept confidential by ETS Nevertheless they can give a sense of many of the constraints that are placed on themiddot design of actual TOEFL specifications

TOEFLlt8gt specifications

Listening Section The listening section measures the examinees ability to understand English as it is spoken in North America Conversational features of the language are bull stressed and the skills tested include vocabulary and idiomatic expression as well as speshycial grammatical constructions that are frequently used in spoken Engfish The stimulus material and questions are recorded in standard North American English

The listening section includes various stimuli such as dialogues short conversations academic discussions and mini-lectures and poses questions that test comprehension of main ideas the order ofa process supporting ideas important details and inferences as well as the ability to categorize topicsobjects

The test developers have taken advantage of the multimedia capability of the computer by using photos and graphics to create context and support the content of the lectures producing stimuli that more closely approximate Ureal-world situations in which people do more than just listen to voices The listening stimuli are often accompashynied by either context-setting or content-based visuals All dialogues conversations acashydemic discussions and mini-lectures include context visuals to establish the setting and role of the speakers Content-based visuals are often used to complement th~ topics of the mini-lectures

Structure-Section Themiddotstructure-section measures an examinees ability to recognize language that is appropriate for standard written English The language tested is formal rather than conversational The topics of the sentences are associated with general acadeshymic discourse so that individuals in specific fields of study or from specific national or linguistic groups have no particular advantage

Two types of questions are used questions in which examinees must (1) complete an incomplete sentence using one of four answers provided and (2) identify one of four unshyderlined words or phrases that would not be accepted in English The two question types are mixed randomly rather than being separated into two subsections as in the papershybased TOEFL test

Reading Section The reading section measures the ability to read and understand short passages similar in topic and style to academic texts used in North American colshyleges and universities Examinees read a variety of short passages on academic subjects and answer several questions about each passage Test items refer to what is stated or imshyplied in the passage as well as to words used in the passage To avoid creating an advanshyt~ t( 1 dull5- In 3n~ ont fidd of stud~ sufficient context is provided SO that no ~ -middott - ~tmiddot J ~ middoth - -~ rM~ ir~ to ~-er the if~(J(lS~ ~ t ~ ~ ~~~~-- Ih L_ 1_ 1 ~_

CHAPTER 4 Standardized Testing 73

The reading section consists of four to five passages of 250-350 words with 10-14 questions per passage This section is not computer-adaptive soexaminees can skip questions and return to previous questions The questions in this section assess the comshyprehension of main ideas inferences factual information stated in a passage pronoun referents and vocabulary (direct meaning synonym antonym) In all cases the questions can be answered by reading and understanding the passages This section consists of (1 ) traditional multiple-choice questions (2) questions that require examinees to click on a word phrase sentence or paragraph to answer and (3) questions that ask examinees to insert a sentence where it fits best

Writing Section The writing section measures the ability to write in English including the ability to generate organize and develop ideas to support those ideas with examples or evidence and to compose a response to one assigf)ed topic in standard written Enshyglish Because some examinees may not be accustomed to composing an essay on comshyputer they are given the choice of handwriting or typing the essay in the 30-minute time limit The rating scale for scoring the essay ranging from 0 to 6 is virtually the same as that of the Test of Written English [see Chapter 9 of this book] A score of 0 is given to papers that are blank simply copy the topic are written ina language other than English consist only of random keystroke characters or are written on a topic different from the one assigned

Each essay is rated independently by two trained certified readers Neither reader knows the rating assigned by the other An essay will receive the average of the two ratshyings unless there is a discrepancy of more than one point in that case a third reader will independently rate the essay The essay rating is incorporated into the StructureMriting scaled score~ and constitutes approximately 50 percent of that combined score

(B) The designing of the test specs for the ESLPT was a somewhat simpler task because the purpose is placement and the construct validation of the test consisted of an examination of the content of the ESL courses In fact in a recent revisiofi of the ESLPT (lmao et al 2000 Imao 2001) content validity (coupled with its attenshydant face validity) was the central theoretical issue to be considered The major issue centered on designing practical and reliable tasks and item response formats Having established the importance of designingESLPT tasks that simulated classroom tasks used in the courses the designers ultimately specified two writing production tasks (one a response to an essay that students read and the other a summary of another essay) and one multiple-choice grammar-editing taskThese specifications mirrored the readingbased process writing approach used in the courses

(C) Specifications for the GET arose out of the perceived need to provide a threshold of acceptable writing ability for all prospective graduate students at SFSU both native and non-native speakers of EnglishThe specifications for the GET are the skills of writing grammatically and rhetorically acceptable prose on a topic of some interest with clearly produced organization of ideas and logical development The GET is a direct test of writing ability in which test-takers must in a two-hour time period write an essay on a given topic

74 CHAPTER 4 Standardized Testing

3 Design select and arrange test tasksitems

Once specifications for a standardized test have been stipulated the sometimes never-ending task of designing selecting and arranging items beginS The spe~s act much like a blueprint in determining the number and types of items to be created Lets look at the three examples

(A) TOEFL test design specifies that each item be coded for content and statisshytical characteristics Content coding ensures that each examinee will receive test questions that assess a variety of skills (reading comprehending the main idea or understanding inferences) and cover a variety of subject matter without unduly biasing the content toward a subset of test-takers (for example in the listening secshytion involving an academic lecture the content must be universal enough for stushydents from many different academic fields of study) Statistical characteristics including the IRT equivalents of estimates of item ~~ility (IF) and the ability of an item to discriminate (ID) between higher orlower ability levels ate also coded

Items are then designed by a team who select and adapt items solicited from a bank of items that have been deposited by freemiddotlance writers and ErS staff Probes for the reading section for example are usually excerpts from authentic general or academic reading that are edited for linguistic difficulty culture bias or other topic biases Items are designed to test overall comprehension certain specific informashytion and inference

Consider the following sample of a reading selection and ten items based on it from a practice TOEFL (Phillips 2001pp423-424)

For hundreds of years in the early history of America pirates sailed through coastal washyters pillaging and plundering all in their path They stole from other ships andstole from coastal towns not content only to steal they destroyed everything they could not carry avay~ Some of the pirate ships amassed large treasures~ the fates of which are unknown leaving people of today to wonder at their whereabouts and to dream of one day coming across some lost treasure

One notoriously large treasure was on the pirate ship Whidah which sank in the washyters off Cape Cod during a strong storm in 1717 A hundred of the crew members went down with the ship along with its treasure of coins gold silver and jewels The treasure on board had an estimated value on todays market of more than 100 million dollars

The remains of the Whidah were discovered in 1984 by Barry Clifford who had spent years of painstaking research and tireless searching only finally to locate the ship about 500 yards from shore A considerable amount of treasure from the centuries-old ship has been recovered from its watery grave but there is clearly still a lot more out there Just as a reminder of what the waters off the coast have been protecting for hundreds of years occasional pieces of gold or silver or jewels still wash up on the beaches and lucky beach-goers find pieces of the treasure

11 ~ Thepa~e mainly diccus5eS

CH4PTER 4 Standardized Testing 75

(e) what really happened to the Whidahs pirates (D) why people go to the beach

12 It is NOT mentioned in the passage that pirates did which of the following (A) They killed lots of people (B) They robbed other ships (e) They took things from towns (D) They gathered big treasures

13 The word amassed in line 4 is closest in meaning to (A) sold (e) transported (B) hid (D) gathered

14 It is implied in the passage that the Whidahs crew (A) died (B) went diving (e) searched for the treasure (D) escaped with parts of the treasure

15 Which of the following is NOT mentioned as part of the treasure of the Whidah (A) Art objects (B) Coins (e) Gold and si Iver (D) Jewels

16 The word estimated in line 10 is closest in meaning to which of the following (A) Known (C) Approximate (B) Sold (D) Decided

17 The passage indicates that the cargo of theWhidah is worth about (A) $100000 (B) $1000000 (C) $10000000 (D) $100000000

18 The work that Barry Clifford did to locate the Whidah was NOT (A) successfu I (B) effortless (C) detailed (D) lengthy

19 It is mentioned in the passage that the treasure of the Whidah (A) is not very valuable (8) is all in museums (C) has not all been found (D) was taken to share by the pi rates

20 The paragraph following the passage most likely discusses (A) what Barry Clifford is doing today (8) the fate of the Whidahs crew (e) other storms in the area of Cape Cod (D) additional pieces that turn up from the Whidahs treasure

76 CHAPTER 4 Standardized Testing

As you can see items target the assessment of comprehension of the main idea (item 11) stated details (17 19) unstated details (12 15 18) implied details (14 20) and vocabulary in context (13 16) An argument could be made about the cultural schemata implied in a passage about pirate ships and you could engage in an angels on the head of a pin argument about the importance of picking cershytain vocabulary for emphasis but every test item is a sample of a larger domain and each of these fulfills its designated specification

Before any such items are released into a form of the TOEFL (or any validated standardized test) they are piloted and sCientifically selected to meet difficulty specshyifications within each subsection section and the test overall Furthermore those items are also selected to meet a desired discrimination index Both of these indices are important considerations in the design of a computer-adaptive test where pershyformance on one item determines the next one to be presented to the test-taker (See Chapter 3 for a complete treatment of multiple-choice item design)

(B)The selection of items in the ESLPT entailed two-entirel) different processes In the two subsections of -the test that elicit writing performance (summary of reading response to reading) the main hurdles were (a) selecting appropriate passhysages for test-takers to read (b) providing appropriate prompts and (c) processing data from pilot testing Passages have to conform to standards of content validity by being within the genre and the difficulty of the material used in the courses The prompt in each case (the section asking for a summary and the section asking for a response) has to be tailOred to fit the passage but a general template is used

[n the multiple-choice editing test that seeks to test grammar proofreading ability the first and easier task is to choose an appropriate essay within which to embed errors The more complicated task is to embed a specified number of errors from a previously determined taxonomy of error categories Those error categories came directly from student errors as perceived by their teachers (verb tenses verb agreeshyment logical connectors articles etc) The disttactors for each item were selected from actual errors that students make Itemsiti pilot versions were then coded fordifshyficulty and discrinlination indices after which final assembly of items could occur

(C) The GET prompts are designed by a faculty committee of examiners who are speCialists in the field of university academic writing The assumption is made that the topics are universally appealing and capable of yielding the intended product of an essay that requires an organized logical argument and conclusion No pilot testing of prompts is conducted The conditions for administration remain constant two-hour time limit sit-down context paper and pencil closed-book format Consider the following recent prompt

Graduate Essay Test sample prompt

In the Middletown Elementary School District the assistant superintendent has just been made superintendent in another district Her resignation leaves vacant the districts only administrative position ever held by a woman The School Board in response to strong

CHAPTER 4 Standardized Testing 77

arguments from the Teachers Association has urged that a woman be hired to replace her As a member of the hiring committee you must help choose her successor

Only one woman applicant meets the written qualifications for the job the two top male applicants are both more experienced than she

The hiring committee has asked each committee member to prepare a written statement to distribute before meeting together to discuss the issue Write a report that represents your position making it as logical and persuasive as possible

Some facts you may wish to draw on 1 Women make up more than 75 percent of classroom teachers but hold fewer than

10 percent of administrative positions in education Administrators salaries average 30 percent more than teachers salaries

2 The local Teachers Association is 89 percent women mostly under 40 In a heated debate on television a member of the National Organization of Women (NOW) and the chair of the Teachers Association threatened if a man is hired to bring a class-action suit against the district on behalf of all women teachers who cannot expect advancement because of discriminatory hiring practices

3 The local Lions Club which contributes heavily to school sports says hiring the less experienced woman would not be in the best interests of the schoolthe children or the teachers

The finalists for the position

1 Carole Gates Classroom teacher 10 years Teacher of the Year 1985 supervisor ofpractice teachers at Teachers College former president of Teachers Associ ati on Administrative Credential 1984 EdD degree 1986 assistant principal of Hoptown Elementary School 2 years

2 Spud Stonewall Principal of Middletown Elementary 15 years PhD in educational adminis~ration State Board of Education Committee for Improving Elementary School Curriculum 1982-present

3 Jim Henderson School Administrator 22 yearsgradesK-9-supports innovation in education Fair Bargaining Award 1981 former coach for winning collegiate basketball team 10 years

It is clear from such a prompt that the problem the test-takers must address is complex that there is sufficient information here for writing an essay and that testshytakers will be reasonably challenged to write a clear statement of opinion What also emerges from this prompt (and virtually any prompt that one might propose) is the potential cultural effect on the numerous international students who must take the GIIT Is it possible that such students who are not familiar with school systems in the United States with hiring procedures and perhaps with the politics of school board elections might be at a disadvantage in mounting their arguments within a two-hour time frame Some (such as Hosoya 2001) have strongly claimed such a bias

78 CHAPTER 4 Standardized Testing

4 Make appropriate evaluations of different kinds of items

In Chapter 3 the concepts of item facility (IF) item discrimination (ID) and disshytractor analysis were introduced As the discussion there showed such calculations provide useful infornlation for classroom tests but sometimes the time and effort involved in perfornling them may not be practical especially if the classroom-based test is a one-time test Yet for a standardized multiple-choice test that is designed to be marketed commercially andor administered a number of times andor adminisshytered in a different form these indices are a must

For other types of response formats namely production responses different forms of evaluation become importantThe principles of p-mpoundti~ality ~d poundabWty are prominent along with the concept o(JacjJjt Practicality issues in such items include the clarity of directions timing of the test ease of administration and how much time is required to score responses Reliability is a major player in instances where more than one scorer is employed and to a lesser extent when a single scorer has to evaluate tests over long spans of time that could lead to deterioration of stanshydards Facility is also a key to the validity and success of an item type ~irecshytions complex- language obscure topics fuz~Qata and culturally biased

~Jfiformatioifma~alliead to a highei1eVermiddotof diffiCidty than one desires (A) The IF ID and efficiency statistics of the multiple-choice items of current

forms of the TOEFL are not publicly available information For reasons of security and protection of patented copyrighted materials they must remain behind the closed doors of the ETS development staff Those statistics remain of paramount importance in the ongoing production ofTOEFL items and forms and are the founshydation stones for demonstrating the equatability of forms Statistical indices on retired forms of the TOEFL are available on request for research purposes

The essay portion of theTOEFL undergoes scrutiny for its practicality reliability and facility Special attention is given to reliabilIty since two human scorers must read each essay and every time a third reader becomes necessary (when the two readers disagree by more than one point) it costs ETS more money

(B) In the case of the open-ended responses on the two written tasks on the ESLPT a similar set of judgments must be made Some evaluative impressions of the effectiveness of prompts and passages are gained from informal student and scorer feedback In the developmental stage of the newly revised ESLPT both types of feedshyback were formally solicited through questiQnnaires and interviews That informashytion proved to be invaluable in the revisIon of prompts and stimulus reading passages After each administration now the teacher-scorers provide informal feedshyback on their perceptions of the effectiveness of the prompts and readings

The multiple-choice editing passage showed the value of statistical findings in determining the usefulness of items and pointing administrators toward revisions Following is a sample of the format used

CHAPTER 4- Standardized Testing 79

Multiple-choice editing passage

(1)EYer since supermarkets first appeared they have beentake over ~ world ABC 0

(2) Supermarkets have changed peoples life ~ yet and at the same time changes in ABC

peoples life ~ have encouraged the opening of supermarkets o

The task was to locate the error in each sentence Statistical tests on the experishymental version of this section revealed that a number of the 45 items were found to be of zero IF (no difficulty whatsoever) and of inconsequential discrimination power (some IDs of 15 and lower) Many distractors were of no consequence because they lured no one Such information led to a revision of numerous it~ms and their options eventually strengthening the effectiveness of this section

(C)The GET like its written counterparts in the ESLPT is a test ofwritten ability with a single prompt and therefore questions of practicality and J~~illy~are also largely observational No data are collected from students on their perceptions but the scorers have an opportunity to reflect on the validity ofa given topiC After one sitting a topic is retired which eliininates the possibility of improving a specific topiC but future framing of topics might benefit from scorers evaluations Inter-rater reliability is checked periodically and reader training sessions are modified if too many instances of unreliability appear

5 Specify scoring procedures and reporting formats - ---

A systematic assembly of test items in pre-selected arrangements and sequences all of which are validated to confo~ to an e~pected difficulty level should yield a test that can then be scored accurately and reported back to test-takers and institutions efficiently

(A) Of the three tests being exemplifled here the most straightforward scoring procedure comes from the TO~FL the one with the most complex issues of validashytion deSign and assembly Scores are calculated and reported fora) three sections of the TOEFL (the essay ratings are combined with the Structure and Written Expression score) and (b) a total score (range 40 to 300 on the computer-based TOEFL and 310 to 677 on the paper-and-pencil TOEFL) A separate score (c) for the Essay (range 0 to 6) is also provided on the examinees score record (see simulation of a score record on page 80)

80 CHAPTER 4 Standardized Testing

Facsimile of a TOEFLreg score report

TOEFL Scaled Scores Claudia Y Estudiante Peru ___

19 17 17 177 Listening Structure Writing Reading Total Score

Essay rati ng 30

The rating scale for the essay is virtually the same one that is used for the Test of Written English (see Chapter 9 for details) with a zero level added for no response copying the topic only writing completely off topic or not writing in English

(B) The ESLPT reports a score for each of themiddot essay sections but the rating scale differs between them because in one case the objective is to write a summary and in the other to write a response to a reading ~ch essayi~pd lgtY ~o readet~ ifhFfF js a discrepancy of more than one level a third reader1resolves the differenceThe ~ditiilg section is machine-scanned and -scored with a total score and ~th part-scores for each ofthe grammaticaVrhetorlcal sectionS From these data placement administrators have adequate information to make placements and teachers receive some diagnostic inforshymation on each student in their classes Students do not receive their essays back

(C) Each GET is read by two trained readers who give a score between 1 and 4 according to the following scale

Graduate Essay Test Scoring Guide

Please make no marks on the writers work Write your reader number and score on the front cover of each test booklet

4 Superior The opening establishes context purpose and point of view the body of the essay developsmiddot recommendations-logically and coherently The writer demonshystrates awareness of the complexities in the situation and provides analysis of the probJem offers compelling or common-sense reasons for recommendations made makes underlying assumptions explicit

The writer uses fluent and idiomatic English with few mechanical errors Style reshyveals syntactic maturity is dear and direct is not choppy or over-colloquial nor over-formal stuffy or unfocused Occasional spelling or punctuation errors may be easily attributed to hasty transcription under pressure

3 Competent After an opening that establishes context and purpose the paper unfolds with few lapses in coherence but may have somewhat less clear organization of less explicit transitions than a top-score paper It may have somewhat less compelling logic or slightly less-wellreasoned suggestions than a 4 paper though it will provide reasons for the recommendations made

The writer uses dear fluent and generally idiomatic English but may make minor or infrequent ESL errors (preposition errors dropped articles or verb endings etc) or repeat a single error (eg not punctuate possessive nouns) Occasional lapses of style are offSet by demonstrated mastery of syntax

CHAPTE84 Standardized Testing 81

2 Weak The writer makes somewhat simplistic suggestions not fully supported with reashysons fails to cite key facts offers little analysis of the problem or shows a limited grasp of the situation the given information is copied or listed withlittle integration into argument Points may be random or repetitious Writing may be badly focused with careless use of abstract language resulting in predication errors or illogical sentences

ESL andlor careless mechanical errors are frequent enough to be distracting OR sentences may be choppy style over-casual usage occasionally unidiomatic

1 Inadequate The essay may be disjointed incoherent or minimally developed The writer shows little grasp of the complex issues involved is unable to establish conshytext point of view or purpose in opening of paper or has a poor sense of audience Mechanical andor ESL errors or unidiomatic usages are frequent sentences may be ungrammatical OR correct but short and very simple

The two readers scores are added to yield a total possible score of 2 to 8 Test administrators recommend a score of 6 as the threshold for allowing a student to pursue graduate-level courses Anything below that is accompanied by a recomshymendation that the student either repeat the test or take a remedial course in gradshyuate writing offered in one of several different departments Students receive neither their essays nor any feedback other than the fmal score

6 Perform ongoing construct validation studies

From the above discussion it should be clear that no standardized instrument is expected to be used repeatedly without a ramporou~program of ongoing c~-sectmct valiltiatiOll Any standardized test once developed must be accompanied by sysshy~

tematic periodic corroboration of its effectiveness and by steps toward its improveshyment This rigor is especially true of tests that are produced in equated forms that is forms must be reliable across tests such that a score on a subsequent form of a test-has-the~same validityand-interpretability as its original

(A) The TOEFL program in cooperation with other tests produced by ETS has an impressive program of research Over the years dozens of TOEFL-sponsored research studies have appeared in the TOEFL Monograph Series An early example ofsuch a study was the seminal Duran et aI (1985) study TOEFLfrom a Communicative ViewpOint on Language Proficiency which examined the content characteristics of the TOEFL from a communicative perspective based on current research in applied linguistics and lanshyguage proficiency assessment More recent studies (such as Ginther 2001 Leacock amp Chodorow 2001 Powers et aI 2002) demonstrate an impressive array of scrutiny

(B) For approximately 20 years the ESLPT appeared to be placing students relishyably by means of an essay and a multiple-choice grammar and vocabulary test Over the years the security of the latter became s1lspect and the faculty administrators wished to see some content validity achieved in the process In the year 2000 that process began with a group of graduate students (Imao et aI 2000) in consl1ltation with faculty members and continued to fruition in the form of a new ESLPT reported in lmao (2002) The development of the new ESlPT involved a lengthy process of

82 CHAPTER 4 Standardized Testing

both content and construct validation along with facing such practical issues as scoring the written sections and a machine scorable multiple-choice answer sheet

The process of ongoing validation will no doubt continue as new forms of the editing section are created and as new prompts and reading passages are created for the writing section Such a validation process should also include consistent checks on placement accuracy and on face validity

(C) At this time there is little or no research to validate the GET itself For its conshy struct validation its administrators rely on a stockpile of research on university-level academic writing tests such as theTWEThe holistic scoring rubric and the topics and administrative conditions of the GET are to some extent patterned after that of the TWE In recent years some criticism of the GEf has come from international test-takers (Hosoya 2001) who posit that the topics and time limits of the GET among other facshytors work to the disadvantage of writers whose native language is not English These validity issues remain to be fully addressed in a comprehensive research study

I I

STANDARDIZED IANGUAGE PROFICIENCY TESTING

Tests of language profiCiency presuppose a comprehensive definition of the specific competencies that comprise overall language ability The specifications for the TOEFL provided an illustration of an operational definition of ability for assessment purposes This is not the only way to conceptualize the concept Swain (1990) offered a multidimensional view of profiCiency assessment by referring to three linshyguistic traits (grammar discourse and sociolinguistics) that can be assessed by means of oral multiple-choice and written responses (see Table 41) Swains conshyception was not meant to be an exhaustive analysis of ability but rather to serve as an operational framework for constructing proficiency assessments

Another defmition and conceptualization of profiCiency is suggested by the ACTFL association mentioned earlier ACfFL takes a holistic and more unitary view of proficiency in describing four levels superior advanced intermediate and noviceWithin each level descriptions of listening speaking reading and writing are provided as guidelines for assessment For example the ACfFL Guidelines describe the superior level of speaking as follows

ACTFL speaking guidelines summary superior-level

Superior-level speakers are characterized by the ability to

bull participate fully and effectively in conversations in formal and informal settings on topics related to practical needs and areas of professional andor scholarly interests

bull provide a structured argument to explain and defend opinions and develop effective hypotheses within extended discourse

bull discuss topics concretely and abstractly bull deal with a linguistically unfamiliar situation bull maintain a high degree of linguistic accuracy bull satisfy the linguistic demands of professional andor scholarly life

CHAPTER4 Standardized Testing 83

The other three ACfFL levels use the same parameters in describing progressively lower proficiencies across all four skills Such taxonomie~ have the advantage of considering a number of functions of linguistic discourse but the disadvantage at the lower levels of overly emphasizing test-takers deficiencies

Table 41 Traits of second language proficiency (Swain 1990 p 403)

Trait Grammar Discourse Sociolinguistic

focus on grammatical focus on textual focus on social accuracy within cohesion and appropriateness of sentences coherence language use

Method

Oral structured interview story telling and argumentationpersuasion

role-play ofspeech acts requests offers complaints

scored for accuracy of verbal morphology prepositions syntax

detailed rating for identification logical sequence and time orientation and global ratings for coherence

scored for ability to distinguish formal and informal register

Multiple-choice

sentence-level select the correct form exercise

paragraph-level select the coherent sentence exercise

speech act-Ievelselect the appropriate utterance exercise

(45 items) (29 items) (28 items)

involving verb morphology prepositionsan-d-uther items

Written composition

narrative and letter of persuasion

narrative and letter of persuasion

formal request letter and informal note

scored for accuracy of verb morphology prepositions syntax

detailed ratings much as for oral discourse and global rating for coherence

scored for the ability to distinguish formal and inforJ1lil1 register

FOUR STANDARDIZED lANGUAGE PROFICIENCY TESTS

We now tum to some of the better-known standardized tests of overall language ability or profiCiency to examine some of the typical formats used in commercially available tests We will not look at standardized tests of other specific skills here but that should not lead you to think by any means that proficiency is the only kind of test in the field that is standardized Three standardized oral production tests the

84 CHAPTER 4 Standardized Testing

Test of Spoken English (fSE) the Oral Proficiency Inventory (OPI) and PbonePassreg are discussed in Chapter 7 and the Test of Written English (WE) is covered in ChapterS

Four commercially produced standardized tests of English language proficiency are described briefly in this section the TOEFL the Michigan English Language Assessment Battery (MELAB) the International English Language Testing System (lELTS) and the Test of English for International Communication (fOEICreg) In an appendix to this chapter are sample items from each section of each test When you turn to that appendix use the following questions to help you evaluate these four tests and their subsections

1 What item types are included 2 How practical and reliable does each subsection of each test appear to be 3 Do the item types and tasks appropriately represent a conceptualizatio~ of

language proficiency (ability) That is can you evaluate their construct validity

4 Do the tasks achieve face validity 5 Are the tasks authentic 6 Is there some washback potential in the tasks

Test of English as a Foreign Language (TOEFL)

Producer Educational Testing Service (ETS) Objective To test overall proficiency (language ability) Primary market Almost exclusively US universities and colleges for admission

purposes Type Computer-based (CB) (and two sections are-computer-adaptive)

A traditional paper-based (PB) version is also available Response modes Multiple-choice responses essay Specifications See the box on pp 72-73 Time allocation Up to 4 hours (CB) 3 hours (PB) Internet access wwwtoeflorg

Comments In the North American context the TOEFL is the most widely used comshymercially available standardized test of proficiency Each year the TOEFL test is adminisshytered to approximately 800000 candidates in more than 200 countries It is highly respected because of the thorough program of ongoing research and development conshyducted by ETS The TOEFLs primary use is to set proficiency standards for international students seeking admission to English-speaking universities More than 4200 academic institutions government agencies scholarship programs and licensingcertification agenshycies in more than 80 countries use TOEFL scores By 2004 the TOEFL will include a secshytion on oral production

CHAPTER 4 Standardi~ed Testing 85

Michigan English Language Assessment Battery (MELAB)

Producer English language Institute University of Michigan Objective To test overall proficiency (language ability) Primary market Mostly US and Canadian language programs and colleges

some worldwide educational settings as well Type Paper-based Response modes Multiple-choice responses essay Time allocation 25 to 35 hours Internet access wwwlsaumicheduelimelabhtm

Specifications The MElAB consists of three sections Part 1 a 3D-minute impromptu essay is written on an assigned topic Part 2 a 25-minute multiple-choice listening comshyprehension test is delivered via tape recorder Part 3 is a 100-item 75-minute multipleshychoice test containing grammar doze reading vocabulary and reading comprehension An oral interview (speaking test) is optional

Comments The Ell at the University of Michigan has been producing the MELAB and its earlier incarnation (Michigan Test of English language Proficiency) since 1961 like the TOEFL it serves a North American audience but is also used internationally While its use is not as widespread as the TOEFL its validity is widely respected Because it is cheaper than the TOEFL and more easily obtained it is popular among language schools and institutes Many institutions and companies accept MElAB scores in lieu ofTOEFL scores

International English Language Testing System (IELTS)

Producer Jointly managed by The University of Cambridge local Examinations Syndicate (UClES) The British Council and lOP Education Australia

Objective To test overall proficiency (language ability) Primary-market Australian British Canadian and New Zealand academic

institutions and professional organizations American academic institutions are increasingly accepting IELTS for admissions purposes

1)rpe Computer-based (for the Reading and Writing sections) papershybased for the listening and Speaking modules

Response modes Multiple-choice responses essay oral production Time allocation 2 hours 45 minutes Internet access httpwwwieltsorgl

httpwwwudesorguk httpwwwbritishcouncilorg

Specifications Reading candidates choose between academic reading or general training reading (60 minutes) Writing the same option academic writing or general training writing (60 minutes) Listening four sections for all candidates (30 minutes) Speaking five sections for all candidates (1015 minutes)

86 CHAPTER 4 Standardized Testing

Comments The University of Cambridge local Examinations Syndicate (UCLES) has been producing English language tests since 1858 Now with three organizations cooperatshying to form the IELTS more than a million examinations are administered every year In 2002 a computer-based version of the Reading and Writing modules of the IELTS became available at selected centers around the world The other sections are administered locally by an examinet The paper-based IELTS remains an option for candidates The IELTS retains the distinct advantage of requiring all four skills in the test-takers performance

Test of English for International Communication (TOEICreg)

Producer The Chauncey Group International a subsidiary of Educational Testing Service

Objelttive To test overall proficiency (langlJage ability) Primary market Worldwide business commerce and industry contexts

(workplace settings) Type Computer-based and paper-based versions Response modes Multiple-choice responses Time allocation 2 hours Internet access httpwwwtoeiccom

Specifications Listening Comprehension 100 items administered by audiocassette Four types of task statements questions short conversations and short talks (approxishymately 45 minutes) Reading 100 items Three types of task cloze sentences error recogshynition and reading comprehension (75 minutes)

Comments The TOEIC has become a very widely used international test of English proficiency in workplace settings where English is required for job performance The conshytent includes many different employment settings such as conferences presentations sales ordering shipping schedules reservations (etters and memoranda It is approprishyate to use in educational settings where vocational or workplace English courses are being offered

sect sect sect sect sect

The construction of a valid standardized test is no minor accomplishment whether the instrument is large- or small-scale The designing of specifications alone as this chapter illustrates requires a sophisticated process of construct valishydation coupled with considerations of practicality Then the construction of items and scoringinterpretation procedures may require a lengthy period of trial and error with prototypes of the final form of the testWith painstaking attention to all the details of construction the end product can result in a cost-effective timeshysaving accurate instrument Your use of the results of such assessments can provide useful data on learners language abilities But your caution is warranted as well for all the reasons discussed in this chapter The next chapter will elaborate on what lies behind that need for a cautious approach to standardized assessment

CHAPTER4 Standardized Testing 87

EXERCISES

[Note (I) Individual work (G) Group or pair work (C) Whole-class discussion]

1 (C) Tell the class about the worst test experience youve ever had Briefly anamiddot lyze what made the experience so unbearable and try to come up with sugshygestions for improvement of the test andor its administrative conditions

2 (G) In pairs or small groups compile a brief list of pros and cons of standardshyized testing Cite illustrations of as many items in each list as possible Report your lists and examples to the rest o~ the class

3 (I) Select a standardized test that you are quite familiar with (probably a recent experience) Mentally evaluate that test using the five principles of practicality reliability validity authenticity and washback Report yourevaluashytion to the class

4 (G) The appendix to this chapter provides sample items from Jour different tests of language proficiency In groups one test for each group analyze your test for (a) content validity (b) face validity and (c) authenticity

5 (C) Do you think that the sample TOEFL reading passage about pirates (pages 74-75) and the Graduate EssayTest prompt (pages 76-77) about a school board hiring committee have any culture bias Discuss this and other cultural biases you have noticed in tests Is it possible to design a test that is completely free of culture bias

6 (CG) Compare the differences in conceptualization of language proficiency represented by Swains model the TOEFL and the ACfFL philosophy Which one best represents current thinking about communicative language ability What are the strengths and weaknesses of each approach

FORYOlIILEURTHER READING

Gronlund Norman E (1998) Assessment of student achievement Sixth Edition Boston Allyn and Bacon

Gronlunds classic also mentioned in Chapter 3 offers a concise overview of features of standardized tests offering definitions and examples of the statistical considerations in interpreting scores His approach is unbiased cleady written and accessible to those who might fear the mathematics of standardized testing

Phillips Deborah 2001 Long1nan introductory course for the TOEFL test White Plains NY Pearson Education

A careful examination of this or any other reputable preparation course for a standardized language test is well worth a students time Note especially how the book acquaints the user with the specifications of the test and offers a number of useful strategie~ that can be llsed in preparation for the test and during irs adn1inistration

88 CHAPTER 4 Standardized Testing

APPENDIX TO CHAPTER 4

Commercial Proficiency Tests Sample Items and Tasks

Test of English a~ a Foreign Language (TOEFLreg)

Listening r

Part A

In this section you will hear short conversations between two people In some ofthe conversations each person speaks only once In other conversations one or both of the people speak more than once Each conversation is followed by one questionabQlt it Each question in this part has four answer choices You should click on the best answer to each question Answer the questions on the basis of what is stated or implied by the speakers Here is an example On the computerscreen you will see

[man and woman talking]

On the recording you will hear

(woman) Hey wheres your sociology book (man) At home Why carry it around when were just going to be taking

a test (woman) Dont you remember Professor Smith said we could us it during

the test (man) Ohl no Well Ive still got an hour right Im so glad I ran into you

You wiII then see and hear the question before the answer choices appear

What will the man probably do next

o Begin studying for the sociology test o Explain the problem to his professor o Go home to get his textbook o Borrow the womans book

To choose an answer you will click on an oval The oval next to that answer will darken After you click on Next and Confirm Answer the next conversation will be presented

Part B

In this section you will hear several longer conversations and talks Each conversation or talk is followed by several questions The conversations talks and questions will not be repeated The conversations and talks are about a variety of topics You do not need speshycial knowledge of the topics to answer the questions correctly Rather you should answer each question on the basis of what is stated or implied by the speakers in the conversashytions or talks

For most of the questions you will need to click on the best of four possible answers Some questions will have special directions The special directions will appear in a box on the computer screen Here is an exampie ot a conversation and some questions

CHAPTER 4 Standardized Testing 89

Marine Biology (narrator) Listen to part of a discussion in a marine biology class

(professor) A few years ago our local government passed a number of strict environmental laws As a result Sunrise Beach looks nothing Ii ke it did ten years ago The water is cleaner and theres been a tremendous increase in all kinds of marine life which is why were going there on Thursday

(woman) I dont know if I agree that the water quality has improved I mean I was out there last weekend and it looked all brown It didnt seem too clean to me

(professor) Actually the color of the water doesnt always indicate whether its polluted The brown color you mentioned might be a result of pollution or it can mean a kind of brown algae is growing there Its called devils apron and it actually serves as food for whales

(man) So when does the water look blue (professor) Well water thats completely unpolluted is actually colorless But

it often looks bluish-green because the sunlight can penetrate deep down and thats the color thats reflected

(woman) But sometimes it looks really green Whats that about (professor) Ok well its the same principle as with devils apron the

water might be green because of different types of green algae there-gulfweed phytoplankton You all should finish reading about algae and plankton before we go In fact those are the types of living things Im going to ask you to be looking for when were there

Now get ready to answer the questions

What is the discussion mainly about

o The importance of protecting ocean environments o The reasons why ocean water appears to be different colors o The survival of whales in polluted water o The effect that colored ocean water has on algae

To choose an answer click on an oval The oval next to that answer will darken After you click on Next and Confirm Answer the next question will be presented

According to the professor what can make ocean water look browngt

o Pollution o Cloudy Skies o Sand o Algae

Click on 2 answers

To choose your answers you will click on the squares An XII wiii appear in each square

bullbullbullbullbullbullbull

90 CHAPTER 4 Standardized Testing

Structure and Written Expression This section measures the ability to recognize language that is appropriate for standard written English There are two types ofquestions in this section In the first type ofquestion there are incomplete sentences Beneath each sentence there are four words or phrases

Directions CIiSk on the one word or phrase that best completes the sentence

The colum~ine flower __ to nearly all of the United States can be raised from seed in almost any garden

native how native is how native is it is native

Time Help Confirm

After you click on Next and Confirm Answ~ the next question willbe presented

The second type of question has four underlined words or phrases You will choose the one underlined word or phrase that must be changed for the sentence to be correct

Directions Click on the one underlined word or phrase that must be changed for the senshytence to be correct

One of the most difficult problems in understanding sleep is determining what the funcshytions of sleep ~

lime Help Confirm

Clicking on an underlined word or phrase will darken it

Reading This section measures the ability to read and understand short passages similar in topic and style to those that students are likely to encounter in North American universities and colleges This section contains reading passages and questions about the passages There are several different types of questions in this section In the Reading section you will first have the opportunity to read the passage

The temperature of the Sun is over 10000 degrees Fahrenheit at the surface but it rises perhaps more than 270000000 at the center The Sun is so much hotter than the Earth that matter can exist only as a gasl except perhaps at the core In the core of the Sun the pressures are so great that despite the high temperature there may be a small solid core However no one really knows since the center of the Sun can never be directly observed ~ Solar astronomers do know that the Sun is divided into five general layers or zones Starting at the outside and going down into the Sun the zones are the corona chromoshysphere hotosphere convection zone and finally the core The first three zones are reshygarded as the Suns atmosphere But since the Sun has no solid surface it is hard to middottell where the atmosphere ends and the main body of the Sun begins

The Suns outermost layer begins about 10000 miles above the visible surface and goes outward for millions of miles This is the only part of the Sun that can be seen during an eclipse such as the one in February 1979 At any other time the corona can be seen

bullbullbullbullbullbullbull

bull bullbullbullbullbullbull

CHAPTER 4 Standardized Testing 91

only when special instruments are used on cameras and telescopes to block the light from the photosphere

The corona is a brilliant pearly white filmy light about as bright as the full Moon Its beautiful rays are a sensational sight during an eclipse The coronas rays flash out in a brilliant fan that has wispy spikelike rays near the Suns north and south poles The corona is generally thickest at the Suns equator The corona is made up of gases streamshying outward at tremendous speeds that reach a temperature of more than 2 million deshygrees Fahrenheit The gas thins out as it reaches the space around the planets By the time the gas of the corona reaches the Earth it has a relatively low density

When you have finished reading the passage you will use the mouse to click on Proceed Then the questions about the passage will be presented You are to choose the one best anshyswer to each question Answer all questions about the information in a passage on the basis ofwhat is stated or implied in that passage Most ofthe questions will be multiple-choice questions To answer these questions you will click on a choice below the question

With what topic is paragraph 2 mainly concerned

o How the Sun evolved o The structure of the Sun o Why scientists study the Sun o The distaflce of the Sun from the planets

Paragraph 2 is marked with an arrow (~)

You will see the next question after you click on Next

To answer some questions you will click on a word or phrase Here is an example

Look at the word one in the passage Click on the word or phrase in the bold text that one refers to To answer you can click on any part of the word or phrase in the passage Jour choice will darken to show which word you have chosen

The Suns outermost layer begins about 10000 miles above the visible surface and goes outward for millions of miles This is the only part of the Sun that can be seen durshying an eclipse such as the one in February 1979 At any other time the corona can be seen only when special instruments are used on cameras and telescopes to block the Iight from the photosphere

You will see the next question after you click on~ To answer some q~estions you will click on a sentence in the passage Here is an example

~ The corona is a brilliant pearly white filmy light about as bright as the full Moon Its beautiful rays are a sensational sight during an eclipse The coronas rays flash out in a brilliant fan that has wispy spikelike rays near the Suns north and south poles The corona is generally thickest at the Suns equator ~ The corona is made up of gases streaming outward at tremendous speeds that reach a temperature of more than 2 million degrees Fahrenheit The gas thins out as it reaches the space around the planets By the time the gas of the corona reaches the Earth it has a relatively low density

bull bullbullbullbullbullbull

92 CHAPTER 4 Standardized Testing

Click on the sentence in paragraph 4 or 5 in which the author compares the light of the Suns outermost layer to that ofanother astronomical body Paragraphs 4 and 5 are marked with arrows (~)

To answer some questions you will click on a square to add a sentence to the passage Here is an example -The following sentence can be added to paragraph 1

At the center of the Earths solar system lies the Sun

Where would it best fit in paragraph I Click on the square to add the sentence to the paragraph

D The temperature of the Sun is over 10000 degrees Fahrenheit at the surface but it rises to perhaps morethan 27000000deg at the center 0 The Sun is so much hotter than the Earth that matter can exist only as a gasi except p~rHapsatth~ c6relp the c~re of the ii Sun the pressures are so great that despite the high temperature there may be a small solid core D However no one really knows since the center of the Sun can never be directly observed D 0100

When you click on a square the sentence will appear in the passage at the place you have chosen You can read the sentence added to the paragraph to see if this is the best place to add it You can click on another square to change your answer The sentence will be added and shown in a dark box

Writing In this section you will have an opportunity to demonstrate your ability to write in Enshyglish This includes the ability to generate and organize ideas to support those ideas with examples or evidence and to compose in standard written English in response to an asshysigned topic You will have 30 minutes to write your essay on that topic You must write on the topic you are assigned An essay on any other topic will receive a score of 0 Read the topic below and then make any notes that will help you plan your response Begin typing your response in the box at the bottom of the screen or write your answer on the answer sheet provided to you

Following is a sample topic

Do you agree or disagree with the following statemenH

Teachers should make learning enjoyable and fun for their students

Use specific reasons and examples to support your opinion

CHAPTER 4 Standarczed Testing 93

Michigan English Language Assessment Battery (MELAB)

Composition The time limit for the composition is 30 minutes You must write on only one of the top~

ics below If you write about something else your composition paper will not be graded and you cannot be given a final score If you do not understand the topics ask the exam~ iner to explain or to translate them You may be asked to give your opinion ofsomething and explain why you believe this to describe something from your experience or to exshyplain a problem and offer possible solutions You should write at least one page Some sample topics are

1 What do you think is your countrys greatest problem Explain in detail and tell what you think can be done about it

2 What are the characteristics of a good teacher Explain and give examples 3 An optimist is someone who sees the good side of things A pessimist sees the

bad side Are you an optimist or a pessimist Relate a personal experience that shows this

4 In your opinion are the benefits of space exploration really worth the enormous costs Discuss

Most MELAB compositions are one or two pages long (about 200-300 words) If your paper is extremely short (less than 150 words) your composition will be given a lower score Before you begin writing you might want to take 2 or 3 minutes to plan your comshyposition and to make a short outline to organize your thoughts Such outlines will not be graded they are only to help you You should use the last 5 minutes to read through your composition and to make changes or corrections

Your composition will be graded on how clearly you express yourself in English and on the range of English you are able to use and your control in doing so This means your composition should be well organized your arguments should be fully developed and you should show a range ofgrammatical structures and broad vocabulary Compositions that consist only of very short sentences and very simple vocabulary cannot be given the

middothighest scores If errors are not frequent and if they do not confuse your meaning they will not lower your score very much

Listening Now you will hear a short lecture You may take notes during the lecture Following the lecture you will be asked some questions about it

Therell be a two-week exhibit of the paintings of the little-known master Laura Bernhart at the Claire Osmond Galleries starting on the fifteenth of the month and running through the thirtieth Bernharts known for her innovative designs in abstract expressionism Though a true original she declared a spiritual heritage from Salvador Dali the famous Spanish painter Since Bernhart lived a rather solitary life and died while only in her twenties few people are aware of her works This showing at the Osmond Galleries will provide many with an introduction to her works

10 Where is the exhibit a the Art Museum b the Dali Galleries c the Osmond Galleries

94 CHAPTER 4 Standardized Testing

11 What is Bernhart known for a her copies of Dalis paintings b the originality of her designs c her exhibitions

12 What will going to the exhibit allow most people to do a to see Saivador Dalis paintings b to see Bernharts works for the first time c to learn about Spanish art

Grammar

1 What did the teacher just tell you

She reminded our notebooksI a us to bring b that we bring c our bringing d we should bring

2 Is Bill a good dancer

Not really __ he tries very hard a in spite of h despite c even though d while

3 your clothes are all wet1

Yes I didnt come __ the rain soon enough a away to b over to c down with d in from

Cloze In years to come zoos will not only be places where animals are exhibited to the public but repositories where rare species can be saved from extinction (7) captive breeding The most powerful force (8) the future of many animals-and of zoos-is the decline of the wild (9) even zoo directors would argue that (10) are better places for animals than the fields and forest of their native (11) yet zoos may be the last chance for some creatures that would otherwise pass qUietly into oblivion

7 a through c from b of d damage

8 a bringing c to b that d influencing

9 a But c Not b So d Then

10 a where c even b zoos d wilds

11 alands c residence b life d field

CHAPTER 4 Standardized Testing 95

Vocabulary

12 Mark has a flair for writing a need b purpose c talent d dislike

13 Bill Collins launched his restaurant last June a moved b started c sold d bought

14 John will not accept the censure a burden b blame c credit d decision

15 I cant think of the answer Can you give me a __ a hint b token c taste d gaze

16 Because fewer people are taking expensive vacations the tourist industry is in a a choke b grope c grumble d slump

17 I disagree with a few of his opinions but __ we agree a deliberately b conclusively c essentially d immensely

Reading The influenza virus is a single molecule built from many millions of single atoms You must have heard of the viruses which are sometimes called living molecules While bacteria can be considered as a type of plant secreting pOisonous substances into the body of the organism they attack viruses are living organisms themselves We may conshysider them as regular chemical molecules since they have a strictly aefined atomic strucshyture but on the other hand we must also consider them as being alive since they are able to multiply in unlimited quantities

18 According to the passage bacteria are a poisons

b larger than viruses c very small d plants

96 CHAPTER 4 Standardized Testing

19 The writer says that viruses are alive because they a have a complex atomic structure b move c multiply d need warmth and light

20 The atomic structure of viruses a is -tJIariable b is strictly defined c cannot be analyzed chemically d is more complex than that of bacteria

International English Language Testing System (fELTS)

I

listening

The Listening Module has four sections The first two sections are concerned with social needs There is a conversation between two speakers and then a monologue For examshyple a conversation about travel arrangements or decisions on a night out and a speech about student services on a university campus or arrangements for meals during a confershyence The final two sections are concerned with situations related more closely to educashytional or training contexts For example conversation between a tutor and a student about an assignment or between three students planning a research project and a lecture or talk ofgeneral academic interest All the topics are ofgeneral interest and it makes no difference what subjects candidates study Tests and tasks become more difficult as the sections progress A range of English accents and dialects are used in the recording which reflects the international usage of IELTS

Academic Reading [A 7S0-word article on-th-e- topic of Wind Power in the US with a short glossary at the end]

Questions 1-5

Complete the summary below

Choose your answers from the box below the summary and write them in boxes 1-5 on your answer sheet Note There are more words or phrases than you will need to fill the gaps You may use any word or phrase more than once

Example The failure during the late 1970s and early 19805 of an attempt to establish a widespread wind power industry in the United States resulted largely from the (1) bull in oil prices during this period The industry is now experiencing a steady (2) due to improveshyments in technology and an increased awareness of the potential in the power of wind The wind turbines that are now being made based in part on the (3) of wide- ranging research in Europe are easier to manufacture and maintain than their predecesshysors This has led wind-turbine makers to be able to standardise and thus minimize (4) There has been growing (S) of the importance of wind power as an energy source

CHAPTER 4 Standardized Testing 97

criticism stability skepticism success operating costs decisions design costs fall effects production costs growth decline failure recognition results

Questions 6-1 0 Look at the following list of issues (Questions 6-10) and implications (A-C) Match each issue with one implication Write the appropriate letters A-C in boxes 6-10 on your anshyswer sheet

Example The current price of one wind-generated kilowatt Answer

6 The recent installation of systems taking advantage of economies of scale

7 The potential of meeting one fifth of current U5 energy requirements by wind power

8 The level of acceptance of current wind turbine technology

9 A comparison of costs between conventional and wind power sources

10 The view of wind power in the European Union

Implications

A provides evidence against claims that electricity produced from wind power is relatively expensive

B supports claims that wind power js an important source of energy

C opposes the view that wind power technology requires further-development

General Training Reading Read the passage on Daybreak trips by coach and look at the statements below On your answer sheet write

TRUE if the statement is true FALSE jf the statement is false

NOlGIVEN if the information is not given in the leaflet

1 MiIlers Coaches owns Cambridges Cam bus fleet

2 Premier is an older company than Millers

3 Most of the Daybreak coaches are less than 5 years old

4 Daybreak fares are more expensive than most of their competitors

5 Soft drinks and refreshments are served on most longer journeys

6 Smoking is permitted at the rear of the coach on longer journeys

7 Tickets must be bought in advance from an authorised Daybreak agent

6 Tickets and seats can be reserved by phoning the Daybreak Hotline

9 Daybreak passengers must join their coach at Cambridge Drummer Street

10 Daybreak cannot guarantee return times

98 CHAPTER 4 Standardized Testing

FROM CAMBRIDGE AND SURROUNDING AREA

SPRING IS INTHEAIR

Welcome to our Spring Daybreak programme which continues the tradition of offering unbeatable value for money day trips and tours All the excursions in this brochure will be operated by Pr~mier Travel Services Limited or Millers Coaches both companies are part of the CHLGroup owners of Cambridges Cambus fleet

WERE PROUD OF OUR TRADITION

Premier was established in 1936 the Company now offers the highest standards of coaching in todays competitive operating environment Miller has an enviable reputation stretching back over the past 20 years offering coach services at realistic prices Weve traveled a long way since our early days of pre-war seaside trips Now our fleet of 50 modern coaches (few are more than five years old) operate throughout Britain and Europe but were pleased to still maintain the high standards of quality and service the trademark of our founders nearly sixty years ago

EXCLUSIVE FEATURES

Admission-inclusive fares All Daybreak fares (unless specifically otherwise stated) include admission charges to the attractions shows and exhibits we visit Many full-day scenic tours are accompanied by a fully trained English Tourist Board Blue Badge guide or local experienced driverguide Some Daybreaks include lunch or afternoon tea Compare our admission inclusive fares and see how much you save Cheapest is not the best and value for money is guaranteed If you compare our bargain Daybreak fares beware--most of our competishytors do not offer an all-inclusive fare

SEAT RESERVATIONS

We value the freedom of choice so you can choose your seat when you book The seat reservation is guaranteed a-nd remains yours at all times when aboard the coach

NO SMOKING COMFORT

With the comfort of our passengers in mind coaches on all our Daybreaks are no smokshying throughout In the interests of fellow passengers comfort we kindly ask that smokers observe our no smoking policy On scenic tours and longer journeys ample refreshment stops are provided when of course smoking is permitted

YOUR QUESTIONS ANSWERED

Do I need to book Booking in advance is strongly recommended as all Daybreak tours are subject to demand Subject to availability stand-by tickets can be purchased from the driver

What ti me does the coach leave The coach departs from Cambridge Drummer Street (Bay 12 adjacent to public toilets) at the time shown There are many additional joining points indicated by departure codes in the brochure If you are joining at one of our less popular joining points you will be adshyvised of your pick-up time (normally by telephone) not less than 48 hours before deparshyture In this way we can minimize the length of pick-up routes and reduce journey times for the majority of passengers

CHAPTER 4 Standardized Testing 99

What time do we get back An approximate return time is shown for each excursion The tim~s shown serve as a guide but road conditions can sometimes cause delay If your arrival will be later than advertised your driver will try to allow for a telephone call during the return journey

Where can I board the coach All the Daybreaks in the brochure leave from Cambridge Drummer Street (Bay 12 adjashycent to public toilets) at the time shown Many Daybreaks offer additional pick-ups for pre-booked passengers within Cambridge and the surrounding area This facility must be requested at the time of booking

Academic Writing Writing Task 1 You should spend about 20 minutes on this task

The graph below shows the different modes of transport used to travel to and from work in one European city in 1950 1970 and 1990

[graph shown here]

Write a report for a university lecturer describing the information shown below You should write at least 150 words

Writing Task 2 You should spend about 40 minutes on this task

Present a written argument or case to an educated reader with no specialist knowledge of the folowing topic

It is inevitable that as technology develops so traditional cultures must be lost Technolshyogy and tradition are incompatible-you cannot have both together

To what extent do you agree or disagree with this statement Give reasons for your answer You should write at least 250 words You should use your own ideas knowlshyedge and experience and support your arguments with examples and relevant evidence

General Training Writing Writing Task 1 You should spend about 20 minutes on this task You rent a house through an agency The heating system has stopped working You phoned the agency a week ago but it has still not been mended Write a letter to the agency Explain the situation and teil them what you want them to do about it

You should write at least 150 words You do NOT need to write your own address

Begin your letter as follows

Dear - ___-I

Writing Task 2 You should spend about 40 minutes on this task As part ofa class assignment you have to write about the following topic

100 CHAPTER 4 Standardized Testing

Some businesses now say that no one can smoke cigarettes in any of their offices Some governments have banned smoking in all public placesThis is a good idea but it takes away some of our freedom

Do you agree or disagree Give reasons for your answer You should write at least 250 words

Speaking In each ofthe three parts of the speaking module a specific function is fulfilled In Part 1 the candidates answer general questions about themselves their homes or families their jobs or studies their interests and a range ofsimilar familiar topic areas This part lasts between four and five minutes In Part 2 the candidate is given a verbal prompt on a card and is asked to talk on a particular topic The candidate has one minute to prepare before speaking at length for between one and two minutes The examiner then asks one or two wind-down questions In Part 3 the examiner and candidate engage in a discusshysion of more abstract issues and concepts which are thematically linked to the topic prompt in Part 2 The discussion lasts between four and five minutes

All interviews are recorded on audiocassette Here is a sample ofa Part 2 topic

Describe a teacher who has greatly influenced you in your education

You shou Id say

where you met them what subject they taught what was special about them

and explain why this person influenced you so much

You will have to talk about the topic for 1 to 2 minutes You have 1 minute to think about what you are going to say You can make some notes if you wish

Test of English for International Communication (TOEICreg)

listening

Part 1 Photographs Directions For each question you will see a picture in your test book and you will hear four short statements The statements will be spoken just one time They will not be printed in your test book so you must listen carefully to understand what the speaker says When you hear the four statements look at the picture in your test book and choose the statement that best describes what you see in the picture Then on your answer sheet find the number of the question and mark your answer

[photograph of a scientist looking through a microscope]

You will hear Look at the picture marked number 1 in your test book

(A) Shes speaking into a microphone (B) Shes put on her glasses (C) She has both eyes open (D) Shes using a microscope

CHAPTER 4 Standardized Testing 101

Part 2 Question-Response Directions In this part of the test you will hear a question or statement spoken in Enshyglish followed by three responses also spoken in English The question or staten1ent and the responses will be spoken just one time They will not be printed in your test book so you must listen carefully to understand what the speakers say You are to choose the best response to each question or statement

Question 1 You will hear Ms Morikawa has worked here for a long time hasnt she

(A) At three oclock (B) No Ive lost my watch (C) More than ten years

Question 2 You will hear Which of these papers has a wider circulation

(A) The morning edition (B) Get more exercise (C) By messenger

Part 3 Short Conversations Directions In this part of the test you will hear short conversations between two people The conversations will not be printed in your test book You will hear the conversations only once so you must listen carefully to understand what the speakers say In your test book you will read a question about each conversation The question will be followed by four answers You are to choose the best answer to each question and mark it on your answer sheet

Question 1 (Man) We should think about finding another restaurant for lunch (Woman) Why The food and service here are great

(Man) Yes but the prices are going up every week

You will read Why is this man unhappy with the restaurant

(A) It is too noisy (B) It is too expensive (C) It is too crowded (D) It is too difficult to find

Question 2 (Woman A) How was Dr Borgs recent trip to Singapore (Woman B) She enjoyed the tour of the port very much (Woman A) They say its one of the most active in Asia

You will read 2 What did Dr Borg find interesting

(A) The tourist center (B) The airport (C) The musical performance (D) The harbor

Part 4 Short Talks Directions In this part of the test you vill hear several short talks Each will be spoken just one time They will not be printed in your test book so you must listen carefully to understand and remember what is said In your test book you will read two or more questions about each short talk The questions will be followed by four answers You are to choose the best answer to each question and mark it on your answer sheet

102 CHAPTER 4 Standardized Testing

You will hear Questions 1 and 2 refer to the following announcement

Good afternoon and welcome aboard Nordair Flight 857 from Copenhagen to Bangkok with intermediate stops in Dubai and Calcutta We are preparing for departure in a few minutes At this time your seat back should be returned to its full upright position and your seat belt s~ould be fastened OUf anticipated total flying time to Dubai is six hours and twenty-five minutes I hope you enjoy the flight You will hecJr Now read question 1 in your test book and answer it You will read 1 What is the final destination of the flight

(A) Bangkok (B) Copenhagen (C) Dubai (O) Calcutta

You will hear Now read question 2 in your test book and answer it You will read 2 What will happen in a few minutes

(A) The flight will land in Dubai I

(B) The passengers will board the plane (C) The plane will take off (0) The gate number will be announced

Reading In this section of the test you will have the chance to show how well you understand written English There are three parts to this section with special directions for each part

Part 4 Incomplete Sentences Directions This part of the test has incomplete sentences Four words or phrases marked (A) (8) (e) (D) are given beneath each sentence You are to choose the one word or phrase that best completes the sentence Then on your answer sheet find the number of the question and mark your answer

1 Mr Yangs trip will __ him away from the office for ten days (A) withdraw (B) continue (C) retain (0) keep

2 The company that Marie DuBois started now sells __ products throughout the world (A) its (B) it (C) theirs (D) them

3 If your shipment is not delivered __ Tuesday you can request a full refund for the merchandise (A) at (B) by (C) within (D) while

CHAPTER 4 Standardized Testing 103

Part 6 Error Recognition Directions In this part ofthe test each sentence has four words or phrases underlined The four underlined parts of the sentence are marked (A) (B) (C) (D) You are to identify the one underlined word or phrase that should be corrected or rewritten Then on your answer sheet find the number of the question and mark your answer

1 The pamphlet contains some importance information about the current exhibit ABC D

2 No matter how Jong it taking to finish the annual report it must be done properly ABC D

3 The popularity of jogging appears to have decreased since the past couple of years ABC D

Part 7 Reading Comprehension Directions The questions in this part of the test are based on a selection of reading mateshyrials such as notices letters) forms newspaper and magazine articles) and advertisements You are to choose the one best answer (A) (B) (C) or (OJ to each quesshytion Then on your ariswefsheelfindthe number of the qUestion andmcirkyour answer Answer all questions following each reading selection on thebasis of what is stated or implied in that selection

The Museum ofTechnology is a hands-on museum designed for people to experience science at w()rk~ Visitors are encouraged to use test and handle the objects o~ display Special demonstrations are scheduled for the first and second Wednesdays of each month at 1330 Open Tuesday-Friday 1200-1630 Saturday 1000-1730 and Sunday 11 00-1630

1 When during the month can visitors see special demonstrations (A) Every weekend (B) The first two Wednesdays (C) One afternoon a week (D) Every other Wednesday

Questions 2 and 3 refer to the followi ng notice

NOTICE If you are unable to work because of an extended illness or injury that is not workshyrelated you may be entitled to receive weekly benefits from your employer or the firms insurance company To claim benefits you must file a claim form within thirty days of the first day of your disability Before filing the claim you must ask your doctor to fill in the Doctors Statement on the claim form stating the period of disability

3 To whom is this notice addressed (A) Employers (8) Doctors (C) Employees (D) When paying the bill

4 When must the claim form be filed (A) On the first of the month (8) On the thirtieth of the month (C) On the first day ofdisabifity (D) Within 30 days of the start of disability

Page 6: Standardized Testing Chapter 4 Brown

CHAPTER4 Standardized Testing 71

progratn of research must identify a set of constructs underlying the test itself This stage of laying the foundation stones can occupy weeks months or even years of effort Standardized tests that dont work are often the product of short-sighted conshystruct validation Lets look at the three tests again

(A) Construct validation for the TOEFL is carried out by the TOEFL staff at ETS under the guidance of a Policy Counell that works with a Committee of Examiners that is composed of appointed external university faculty linguists and assessment spedaJists Dozens of employees are involved in a complex process of reviewing curshyrentTOEFL specifications cOmmissioning and developing test tasks and items assemshybling forms of the test and performing ongoing exploratory research related to formulating new specs Reducing such a complex process to a set ofsimple steps runs the risk of gross overgeneralization but here is an idea of how aTOEFL is created

Because the TOEFL ismiddot a proficiency test the fIrst step in the developmental process is to define the construct of language proficiency First it should be made clear that many assessment specialists such as Bachman (1990) and Palmer (Bachman amp Palmer 1996) prefer the term ability to proficiency and thus speak of language abllity as the overarching concept The latter phrase is more conSistent they argue with our understanding that the specific components of language ability must be assessed separately Others such as the American Council on Teaching Foreign Languages (ACIFL) still prefer the term proficiency because it connotes more of a holistic unitary trait view of language ability (Lowe 1988) Most current views accept the ability argument and therefore strive to specify and assess the many components of language For the purposes of consistency in this book the term proficiency will nevertheless be retained with the above caveat

How you view language will make a difference in how you assess language proshyficiency After breaking language competence down into subsets of listening speaking reading and writing eapoundli~p~ifQ(m~I~~ Jll()4~ can be examined on a conshytiiiuum of linguistic units pho~~logy (p~onunciation) and orthography (spelling) words OeXicon) sentences (gtammar) discourse and pragmatic (sociolinguistic contextual functional cul~j features of language

How will the TOEFL sample from at) these possibilities Oral production tests can be tests of overall conversational fluency or pronunciation of a particular subset of phonolOgy and can take the form of imitation structured responses or free responses Listening comprehension tests can concentrate on a particular feature of language or on overalllistenins for general meaning Tests of realtling can cover the range of language units and can aim to test comprehension of long or short passhysages single sentences or even phrases and words Writing-tests can take on an open-ended form with free composition or be structured to elicit anything from correct spelling to discourse-level competence Are you overwhelmed yet

From the sea of potential performance modes that could be sampled in a test the developer must select a subset on some systematic basis To make a very long story short (and leaving out numerous controversies) the TOEFL had for many years included three types of performance in its organizationai specifications listening strucshyture and reading all of vtuch tested comprehension through standard multiple-choice

72 CHAPTER 4 Standardized Testing

tasks In 1996 a major step was taken to include written production in themiddot computershybased TOEFL by adding a slightly modified version of the already existing Test of Written English (TWE) In doing so some face validity and content validity were improved along with of course a significant increase in administrative expense Each of these four major sections is capsulized in the box below (adapted from the descripshytion of the current computer-based TOEFL at wwwroefLorg) Such descriptions are not strictly speaking specifications which are kept confidential by ETS Nevertheless they can give a sense of many of the constraints that are placed on themiddot design of actual TOEFL specifications

TOEFLlt8gt specifications

Listening Section The listening section measures the examinees ability to understand English as it is spoken in North America Conversational features of the language are bull stressed and the skills tested include vocabulary and idiomatic expression as well as speshycial grammatical constructions that are frequently used in spoken Engfish The stimulus material and questions are recorded in standard North American English

The listening section includes various stimuli such as dialogues short conversations academic discussions and mini-lectures and poses questions that test comprehension of main ideas the order ofa process supporting ideas important details and inferences as well as the ability to categorize topicsobjects

The test developers have taken advantage of the multimedia capability of the computer by using photos and graphics to create context and support the content of the lectures producing stimuli that more closely approximate Ureal-world situations in which people do more than just listen to voices The listening stimuli are often accompashynied by either context-setting or content-based visuals All dialogues conversations acashydemic discussions and mini-lectures include context visuals to establish the setting and role of the speakers Content-based visuals are often used to complement th~ topics of the mini-lectures

Structure-Section Themiddotstructure-section measures an examinees ability to recognize language that is appropriate for standard written English The language tested is formal rather than conversational The topics of the sentences are associated with general acadeshymic discourse so that individuals in specific fields of study or from specific national or linguistic groups have no particular advantage

Two types of questions are used questions in which examinees must (1) complete an incomplete sentence using one of four answers provided and (2) identify one of four unshyderlined words or phrases that would not be accepted in English The two question types are mixed randomly rather than being separated into two subsections as in the papershybased TOEFL test

Reading Section The reading section measures the ability to read and understand short passages similar in topic and style to academic texts used in North American colshyleges and universities Examinees read a variety of short passages on academic subjects and answer several questions about each passage Test items refer to what is stated or imshyplied in the passage as well as to words used in the passage To avoid creating an advanshyt~ t( 1 dull5- In 3n~ ont fidd of stud~ sufficient context is provided SO that no ~ -middott - ~tmiddot J ~ middoth - -~ rM~ ir~ to ~-er the if~(J(lS~ ~ t ~ ~ ~~~~-- Ih L_ 1_ 1 ~_

CHAPTER 4 Standardized Testing 73

The reading section consists of four to five passages of 250-350 words with 10-14 questions per passage This section is not computer-adaptive soexaminees can skip questions and return to previous questions The questions in this section assess the comshyprehension of main ideas inferences factual information stated in a passage pronoun referents and vocabulary (direct meaning synonym antonym) In all cases the questions can be answered by reading and understanding the passages This section consists of (1 ) traditional multiple-choice questions (2) questions that require examinees to click on a word phrase sentence or paragraph to answer and (3) questions that ask examinees to insert a sentence where it fits best

Writing Section The writing section measures the ability to write in English including the ability to generate organize and develop ideas to support those ideas with examples or evidence and to compose a response to one assigf)ed topic in standard written Enshyglish Because some examinees may not be accustomed to composing an essay on comshyputer they are given the choice of handwriting or typing the essay in the 30-minute time limit The rating scale for scoring the essay ranging from 0 to 6 is virtually the same as that of the Test of Written English [see Chapter 9 of this book] A score of 0 is given to papers that are blank simply copy the topic are written ina language other than English consist only of random keystroke characters or are written on a topic different from the one assigned

Each essay is rated independently by two trained certified readers Neither reader knows the rating assigned by the other An essay will receive the average of the two ratshyings unless there is a discrepancy of more than one point in that case a third reader will independently rate the essay The essay rating is incorporated into the StructureMriting scaled score~ and constitutes approximately 50 percent of that combined score

(B) The designing of the test specs for the ESLPT was a somewhat simpler task because the purpose is placement and the construct validation of the test consisted of an examination of the content of the ESL courses In fact in a recent revisiofi of the ESLPT (lmao et al 2000 Imao 2001) content validity (coupled with its attenshydant face validity) was the central theoretical issue to be considered The major issue centered on designing practical and reliable tasks and item response formats Having established the importance of designingESLPT tasks that simulated classroom tasks used in the courses the designers ultimately specified two writing production tasks (one a response to an essay that students read and the other a summary of another essay) and one multiple-choice grammar-editing taskThese specifications mirrored the readingbased process writing approach used in the courses

(C) Specifications for the GET arose out of the perceived need to provide a threshold of acceptable writing ability for all prospective graduate students at SFSU both native and non-native speakers of EnglishThe specifications for the GET are the skills of writing grammatically and rhetorically acceptable prose on a topic of some interest with clearly produced organization of ideas and logical development The GET is a direct test of writing ability in which test-takers must in a two-hour time period write an essay on a given topic

74 CHAPTER 4 Standardized Testing

3 Design select and arrange test tasksitems

Once specifications for a standardized test have been stipulated the sometimes never-ending task of designing selecting and arranging items beginS The spe~s act much like a blueprint in determining the number and types of items to be created Lets look at the three examples

(A) TOEFL test design specifies that each item be coded for content and statisshytical characteristics Content coding ensures that each examinee will receive test questions that assess a variety of skills (reading comprehending the main idea or understanding inferences) and cover a variety of subject matter without unduly biasing the content toward a subset of test-takers (for example in the listening secshytion involving an academic lecture the content must be universal enough for stushydents from many different academic fields of study) Statistical characteristics including the IRT equivalents of estimates of item ~~ility (IF) and the ability of an item to discriminate (ID) between higher orlower ability levels ate also coded

Items are then designed by a team who select and adapt items solicited from a bank of items that have been deposited by freemiddotlance writers and ErS staff Probes for the reading section for example are usually excerpts from authentic general or academic reading that are edited for linguistic difficulty culture bias or other topic biases Items are designed to test overall comprehension certain specific informashytion and inference

Consider the following sample of a reading selection and ten items based on it from a practice TOEFL (Phillips 2001pp423-424)

For hundreds of years in the early history of America pirates sailed through coastal washyters pillaging and plundering all in their path They stole from other ships andstole from coastal towns not content only to steal they destroyed everything they could not carry avay~ Some of the pirate ships amassed large treasures~ the fates of which are unknown leaving people of today to wonder at their whereabouts and to dream of one day coming across some lost treasure

One notoriously large treasure was on the pirate ship Whidah which sank in the washyters off Cape Cod during a strong storm in 1717 A hundred of the crew members went down with the ship along with its treasure of coins gold silver and jewels The treasure on board had an estimated value on todays market of more than 100 million dollars

The remains of the Whidah were discovered in 1984 by Barry Clifford who had spent years of painstaking research and tireless searching only finally to locate the ship about 500 yards from shore A considerable amount of treasure from the centuries-old ship has been recovered from its watery grave but there is clearly still a lot more out there Just as a reminder of what the waters off the coast have been protecting for hundreds of years occasional pieces of gold or silver or jewels still wash up on the beaches and lucky beach-goers find pieces of the treasure

11 ~ Thepa~e mainly diccus5eS

CH4PTER 4 Standardized Testing 75

(e) what really happened to the Whidahs pirates (D) why people go to the beach

12 It is NOT mentioned in the passage that pirates did which of the following (A) They killed lots of people (B) They robbed other ships (e) They took things from towns (D) They gathered big treasures

13 The word amassed in line 4 is closest in meaning to (A) sold (e) transported (B) hid (D) gathered

14 It is implied in the passage that the Whidahs crew (A) died (B) went diving (e) searched for the treasure (D) escaped with parts of the treasure

15 Which of the following is NOT mentioned as part of the treasure of the Whidah (A) Art objects (B) Coins (e) Gold and si Iver (D) Jewels

16 The word estimated in line 10 is closest in meaning to which of the following (A) Known (C) Approximate (B) Sold (D) Decided

17 The passage indicates that the cargo of theWhidah is worth about (A) $100000 (B) $1000000 (C) $10000000 (D) $100000000

18 The work that Barry Clifford did to locate the Whidah was NOT (A) successfu I (B) effortless (C) detailed (D) lengthy

19 It is mentioned in the passage that the treasure of the Whidah (A) is not very valuable (8) is all in museums (C) has not all been found (D) was taken to share by the pi rates

20 The paragraph following the passage most likely discusses (A) what Barry Clifford is doing today (8) the fate of the Whidahs crew (e) other storms in the area of Cape Cod (D) additional pieces that turn up from the Whidahs treasure

76 CHAPTER 4 Standardized Testing

As you can see items target the assessment of comprehension of the main idea (item 11) stated details (17 19) unstated details (12 15 18) implied details (14 20) and vocabulary in context (13 16) An argument could be made about the cultural schemata implied in a passage about pirate ships and you could engage in an angels on the head of a pin argument about the importance of picking cershytain vocabulary for emphasis but every test item is a sample of a larger domain and each of these fulfills its designated specification

Before any such items are released into a form of the TOEFL (or any validated standardized test) they are piloted and sCientifically selected to meet difficulty specshyifications within each subsection section and the test overall Furthermore those items are also selected to meet a desired discrimination index Both of these indices are important considerations in the design of a computer-adaptive test where pershyformance on one item determines the next one to be presented to the test-taker (See Chapter 3 for a complete treatment of multiple-choice item design)

(B)The selection of items in the ESLPT entailed two-entirel) different processes In the two subsections of -the test that elicit writing performance (summary of reading response to reading) the main hurdles were (a) selecting appropriate passhysages for test-takers to read (b) providing appropriate prompts and (c) processing data from pilot testing Passages have to conform to standards of content validity by being within the genre and the difficulty of the material used in the courses The prompt in each case (the section asking for a summary and the section asking for a response) has to be tailOred to fit the passage but a general template is used

[n the multiple-choice editing test that seeks to test grammar proofreading ability the first and easier task is to choose an appropriate essay within which to embed errors The more complicated task is to embed a specified number of errors from a previously determined taxonomy of error categories Those error categories came directly from student errors as perceived by their teachers (verb tenses verb agreeshyment logical connectors articles etc) The disttactors for each item were selected from actual errors that students make Itemsiti pilot versions were then coded fordifshyficulty and discrinlination indices after which final assembly of items could occur

(C) The GET prompts are designed by a faculty committee of examiners who are speCialists in the field of university academic writing The assumption is made that the topics are universally appealing and capable of yielding the intended product of an essay that requires an organized logical argument and conclusion No pilot testing of prompts is conducted The conditions for administration remain constant two-hour time limit sit-down context paper and pencil closed-book format Consider the following recent prompt

Graduate Essay Test sample prompt

In the Middletown Elementary School District the assistant superintendent has just been made superintendent in another district Her resignation leaves vacant the districts only administrative position ever held by a woman The School Board in response to strong

CHAPTER 4 Standardized Testing 77

arguments from the Teachers Association has urged that a woman be hired to replace her As a member of the hiring committee you must help choose her successor

Only one woman applicant meets the written qualifications for the job the two top male applicants are both more experienced than she

The hiring committee has asked each committee member to prepare a written statement to distribute before meeting together to discuss the issue Write a report that represents your position making it as logical and persuasive as possible

Some facts you may wish to draw on 1 Women make up more than 75 percent of classroom teachers but hold fewer than

10 percent of administrative positions in education Administrators salaries average 30 percent more than teachers salaries

2 The local Teachers Association is 89 percent women mostly under 40 In a heated debate on television a member of the National Organization of Women (NOW) and the chair of the Teachers Association threatened if a man is hired to bring a class-action suit against the district on behalf of all women teachers who cannot expect advancement because of discriminatory hiring practices

3 The local Lions Club which contributes heavily to school sports says hiring the less experienced woman would not be in the best interests of the schoolthe children or the teachers

The finalists for the position

1 Carole Gates Classroom teacher 10 years Teacher of the Year 1985 supervisor ofpractice teachers at Teachers College former president of Teachers Associ ati on Administrative Credential 1984 EdD degree 1986 assistant principal of Hoptown Elementary School 2 years

2 Spud Stonewall Principal of Middletown Elementary 15 years PhD in educational adminis~ration State Board of Education Committee for Improving Elementary School Curriculum 1982-present

3 Jim Henderson School Administrator 22 yearsgradesK-9-supports innovation in education Fair Bargaining Award 1981 former coach for winning collegiate basketball team 10 years

It is clear from such a prompt that the problem the test-takers must address is complex that there is sufficient information here for writing an essay and that testshytakers will be reasonably challenged to write a clear statement of opinion What also emerges from this prompt (and virtually any prompt that one might propose) is the potential cultural effect on the numerous international students who must take the GIIT Is it possible that such students who are not familiar with school systems in the United States with hiring procedures and perhaps with the politics of school board elections might be at a disadvantage in mounting their arguments within a two-hour time frame Some (such as Hosoya 2001) have strongly claimed such a bias

78 CHAPTER 4 Standardized Testing

4 Make appropriate evaluations of different kinds of items

In Chapter 3 the concepts of item facility (IF) item discrimination (ID) and disshytractor analysis were introduced As the discussion there showed such calculations provide useful infornlation for classroom tests but sometimes the time and effort involved in perfornling them may not be practical especially if the classroom-based test is a one-time test Yet for a standardized multiple-choice test that is designed to be marketed commercially andor administered a number of times andor adminisshytered in a different form these indices are a must

For other types of response formats namely production responses different forms of evaluation become importantThe principles of p-mpoundti~ality ~d poundabWty are prominent along with the concept o(JacjJjt Practicality issues in such items include the clarity of directions timing of the test ease of administration and how much time is required to score responses Reliability is a major player in instances where more than one scorer is employed and to a lesser extent when a single scorer has to evaluate tests over long spans of time that could lead to deterioration of stanshydards Facility is also a key to the validity and success of an item type ~irecshytions complex- language obscure topics fuz~Qata and culturally biased

~Jfiformatioifma~alliead to a highei1eVermiddotof diffiCidty than one desires (A) The IF ID and efficiency statistics of the multiple-choice items of current

forms of the TOEFL are not publicly available information For reasons of security and protection of patented copyrighted materials they must remain behind the closed doors of the ETS development staff Those statistics remain of paramount importance in the ongoing production ofTOEFL items and forms and are the founshydation stones for demonstrating the equatability of forms Statistical indices on retired forms of the TOEFL are available on request for research purposes

The essay portion of theTOEFL undergoes scrutiny for its practicality reliability and facility Special attention is given to reliabilIty since two human scorers must read each essay and every time a third reader becomes necessary (when the two readers disagree by more than one point) it costs ETS more money

(B) In the case of the open-ended responses on the two written tasks on the ESLPT a similar set of judgments must be made Some evaluative impressions of the effectiveness of prompts and passages are gained from informal student and scorer feedback In the developmental stage of the newly revised ESLPT both types of feedshyback were formally solicited through questiQnnaires and interviews That informashytion proved to be invaluable in the revisIon of prompts and stimulus reading passages After each administration now the teacher-scorers provide informal feedshyback on their perceptions of the effectiveness of the prompts and readings

The multiple-choice editing passage showed the value of statistical findings in determining the usefulness of items and pointing administrators toward revisions Following is a sample of the format used

CHAPTER 4- Standardized Testing 79

Multiple-choice editing passage

(1)EYer since supermarkets first appeared they have beentake over ~ world ABC 0

(2) Supermarkets have changed peoples life ~ yet and at the same time changes in ABC

peoples life ~ have encouraged the opening of supermarkets o

The task was to locate the error in each sentence Statistical tests on the experishymental version of this section revealed that a number of the 45 items were found to be of zero IF (no difficulty whatsoever) and of inconsequential discrimination power (some IDs of 15 and lower) Many distractors were of no consequence because they lured no one Such information led to a revision of numerous it~ms and their options eventually strengthening the effectiveness of this section

(C)The GET like its written counterparts in the ESLPT is a test ofwritten ability with a single prompt and therefore questions of practicality and J~~illy~are also largely observational No data are collected from students on their perceptions but the scorers have an opportunity to reflect on the validity ofa given topiC After one sitting a topic is retired which eliininates the possibility of improving a specific topiC but future framing of topics might benefit from scorers evaluations Inter-rater reliability is checked periodically and reader training sessions are modified if too many instances of unreliability appear

5 Specify scoring procedures and reporting formats - ---

A systematic assembly of test items in pre-selected arrangements and sequences all of which are validated to confo~ to an e~pected difficulty level should yield a test that can then be scored accurately and reported back to test-takers and institutions efficiently

(A) Of the three tests being exemplifled here the most straightforward scoring procedure comes from the TO~FL the one with the most complex issues of validashytion deSign and assembly Scores are calculated and reported fora) three sections of the TOEFL (the essay ratings are combined with the Structure and Written Expression score) and (b) a total score (range 40 to 300 on the computer-based TOEFL and 310 to 677 on the paper-and-pencil TOEFL) A separate score (c) for the Essay (range 0 to 6) is also provided on the examinees score record (see simulation of a score record on page 80)

80 CHAPTER 4 Standardized Testing

Facsimile of a TOEFLreg score report

TOEFL Scaled Scores Claudia Y Estudiante Peru ___

19 17 17 177 Listening Structure Writing Reading Total Score

Essay rati ng 30

The rating scale for the essay is virtually the same one that is used for the Test of Written English (see Chapter 9 for details) with a zero level added for no response copying the topic only writing completely off topic or not writing in English

(B) The ESLPT reports a score for each of themiddot essay sections but the rating scale differs between them because in one case the objective is to write a summary and in the other to write a response to a reading ~ch essayi~pd lgtY ~o readet~ ifhFfF js a discrepancy of more than one level a third reader1resolves the differenceThe ~ditiilg section is machine-scanned and -scored with a total score and ~th part-scores for each ofthe grammaticaVrhetorlcal sectionS From these data placement administrators have adequate information to make placements and teachers receive some diagnostic inforshymation on each student in their classes Students do not receive their essays back

(C) Each GET is read by two trained readers who give a score between 1 and 4 according to the following scale

Graduate Essay Test Scoring Guide

Please make no marks on the writers work Write your reader number and score on the front cover of each test booklet

4 Superior The opening establishes context purpose and point of view the body of the essay developsmiddot recommendations-logically and coherently The writer demonshystrates awareness of the complexities in the situation and provides analysis of the probJem offers compelling or common-sense reasons for recommendations made makes underlying assumptions explicit

The writer uses fluent and idiomatic English with few mechanical errors Style reshyveals syntactic maturity is dear and direct is not choppy or over-colloquial nor over-formal stuffy or unfocused Occasional spelling or punctuation errors may be easily attributed to hasty transcription under pressure

3 Competent After an opening that establishes context and purpose the paper unfolds with few lapses in coherence but may have somewhat less clear organization of less explicit transitions than a top-score paper It may have somewhat less compelling logic or slightly less-wellreasoned suggestions than a 4 paper though it will provide reasons for the recommendations made

The writer uses dear fluent and generally idiomatic English but may make minor or infrequent ESL errors (preposition errors dropped articles or verb endings etc) or repeat a single error (eg not punctuate possessive nouns) Occasional lapses of style are offSet by demonstrated mastery of syntax

CHAPTE84 Standardized Testing 81

2 Weak The writer makes somewhat simplistic suggestions not fully supported with reashysons fails to cite key facts offers little analysis of the problem or shows a limited grasp of the situation the given information is copied or listed withlittle integration into argument Points may be random or repetitious Writing may be badly focused with careless use of abstract language resulting in predication errors or illogical sentences

ESL andlor careless mechanical errors are frequent enough to be distracting OR sentences may be choppy style over-casual usage occasionally unidiomatic

1 Inadequate The essay may be disjointed incoherent or minimally developed The writer shows little grasp of the complex issues involved is unable to establish conshytext point of view or purpose in opening of paper or has a poor sense of audience Mechanical andor ESL errors or unidiomatic usages are frequent sentences may be ungrammatical OR correct but short and very simple

The two readers scores are added to yield a total possible score of 2 to 8 Test administrators recommend a score of 6 as the threshold for allowing a student to pursue graduate-level courses Anything below that is accompanied by a recomshymendation that the student either repeat the test or take a remedial course in gradshyuate writing offered in one of several different departments Students receive neither their essays nor any feedback other than the fmal score

6 Perform ongoing construct validation studies

From the above discussion it should be clear that no standardized instrument is expected to be used repeatedly without a ramporou~program of ongoing c~-sectmct valiltiatiOll Any standardized test once developed must be accompanied by sysshy~

tematic periodic corroboration of its effectiveness and by steps toward its improveshyment This rigor is especially true of tests that are produced in equated forms that is forms must be reliable across tests such that a score on a subsequent form of a test-has-the~same validityand-interpretability as its original

(A) The TOEFL program in cooperation with other tests produced by ETS has an impressive program of research Over the years dozens of TOEFL-sponsored research studies have appeared in the TOEFL Monograph Series An early example ofsuch a study was the seminal Duran et aI (1985) study TOEFLfrom a Communicative ViewpOint on Language Proficiency which examined the content characteristics of the TOEFL from a communicative perspective based on current research in applied linguistics and lanshyguage proficiency assessment More recent studies (such as Ginther 2001 Leacock amp Chodorow 2001 Powers et aI 2002) demonstrate an impressive array of scrutiny

(B) For approximately 20 years the ESLPT appeared to be placing students relishyably by means of an essay and a multiple-choice grammar and vocabulary test Over the years the security of the latter became s1lspect and the faculty administrators wished to see some content validity achieved in the process In the year 2000 that process began with a group of graduate students (Imao et aI 2000) in consl1ltation with faculty members and continued to fruition in the form of a new ESLPT reported in lmao (2002) The development of the new ESlPT involved a lengthy process of

82 CHAPTER 4 Standardized Testing

both content and construct validation along with facing such practical issues as scoring the written sections and a machine scorable multiple-choice answer sheet

The process of ongoing validation will no doubt continue as new forms of the editing section are created and as new prompts and reading passages are created for the writing section Such a validation process should also include consistent checks on placement accuracy and on face validity

(C) At this time there is little or no research to validate the GET itself For its conshy struct validation its administrators rely on a stockpile of research on university-level academic writing tests such as theTWEThe holistic scoring rubric and the topics and administrative conditions of the GET are to some extent patterned after that of the TWE In recent years some criticism of the GEf has come from international test-takers (Hosoya 2001) who posit that the topics and time limits of the GET among other facshytors work to the disadvantage of writers whose native language is not English These validity issues remain to be fully addressed in a comprehensive research study

I I

STANDARDIZED IANGUAGE PROFICIENCY TESTING

Tests of language profiCiency presuppose a comprehensive definition of the specific competencies that comprise overall language ability The specifications for the TOEFL provided an illustration of an operational definition of ability for assessment purposes This is not the only way to conceptualize the concept Swain (1990) offered a multidimensional view of profiCiency assessment by referring to three linshyguistic traits (grammar discourse and sociolinguistics) that can be assessed by means of oral multiple-choice and written responses (see Table 41) Swains conshyception was not meant to be an exhaustive analysis of ability but rather to serve as an operational framework for constructing proficiency assessments

Another defmition and conceptualization of profiCiency is suggested by the ACTFL association mentioned earlier ACfFL takes a holistic and more unitary view of proficiency in describing four levels superior advanced intermediate and noviceWithin each level descriptions of listening speaking reading and writing are provided as guidelines for assessment For example the ACfFL Guidelines describe the superior level of speaking as follows

ACTFL speaking guidelines summary superior-level

Superior-level speakers are characterized by the ability to

bull participate fully and effectively in conversations in formal and informal settings on topics related to practical needs and areas of professional andor scholarly interests

bull provide a structured argument to explain and defend opinions and develop effective hypotheses within extended discourse

bull discuss topics concretely and abstractly bull deal with a linguistically unfamiliar situation bull maintain a high degree of linguistic accuracy bull satisfy the linguistic demands of professional andor scholarly life

CHAPTER4 Standardized Testing 83

The other three ACfFL levels use the same parameters in describing progressively lower proficiencies across all four skills Such taxonomie~ have the advantage of considering a number of functions of linguistic discourse but the disadvantage at the lower levels of overly emphasizing test-takers deficiencies

Table 41 Traits of second language proficiency (Swain 1990 p 403)

Trait Grammar Discourse Sociolinguistic

focus on grammatical focus on textual focus on social accuracy within cohesion and appropriateness of sentences coherence language use

Method

Oral structured interview story telling and argumentationpersuasion

role-play ofspeech acts requests offers complaints

scored for accuracy of verbal morphology prepositions syntax

detailed rating for identification logical sequence and time orientation and global ratings for coherence

scored for ability to distinguish formal and informal register

Multiple-choice

sentence-level select the correct form exercise

paragraph-level select the coherent sentence exercise

speech act-Ievelselect the appropriate utterance exercise

(45 items) (29 items) (28 items)

involving verb morphology prepositionsan-d-uther items

Written composition

narrative and letter of persuasion

narrative and letter of persuasion

formal request letter and informal note

scored for accuracy of verb morphology prepositions syntax

detailed ratings much as for oral discourse and global rating for coherence

scored for the ability to distinguish formal and inforJ1lil1 register

FOUR STANDARDIZED lANGUAGE PROFICIENCY TESTS

We now tum to some of the better-known standardized tests of overall language ability or profiCiency to examine some of the typical formats used in commercially available tests We will not look at standardized tests of other specific skills here but that should not lead you to think by any means that proficiency is the only kind of test in the field that is standardized Three standardized oral production tests the

84 CHAPTER 4 Standardized Testing

Test of Spoken English (fSE) the Oral Proficiency Inventory (OPI) and PbonePassreg are discussed in Chapter 7 and the Test of Written English (WE) is covered in ChapterS

Four commercially produced standardized tests of English language proficiency are described briefly in this section the TOEFL the Michigan English Language Assessment Battery (MELAB) the International English Language Testing System (lELTS) and the Test of English for International Communication (fOEICreg) In an appendix to this chapter are sample items from each section of each test When you turn to that appendix use the following questions to help you evaluate these four tests and their subsections

1 What item types are included 2 How practical and reliable does each subsection of each test appear to be 3 Do the item types and tasks appropriately represent a conceptualizatio~ of

language proficiency (ability) That is can you evaluate their construct validity

4 Do the tasks achieve face validity 5 Are the tasks authentic 6 Is there some washback potential in the tasks

Test of English as a Foreign Language (TOEFL)

Producer Educational Testing Service (ETS) Objective To test overall proficiency (language ability) Primary market Almost exclusively US universities and colleges for admission

purposes Type Computer-based (CB) (and two sections are-computer-adaptive)

A traditional paper-based (PB) version is also available Response modes Multiple-choice responses essay Specifications See the box on pp 72-73 Time allocation Up to 4 hours (CB) 3 hours (PB) Internet access wwwtoeflorg

Comments In the North American context the TOEFL is the most widely used comshymercially available standardized test of proficiency Each year the TOEFL test is adminisshytered to approximately 800000 candidates in more than 200 countries It is highly respected because of the thorough program of ongoing research and development conshyducted by ETS The TOEFLs primary use is to set proficiency standards for international students seeking admission to English-speaking universities More than 4200 academic institutions government agencies scholarship programs and licensingcertification agenshycies in more than 80 countries use TOEFL scores By 2004 the TOEFL will include a secshytion on oral production

CHAPTER 4 Standardi~ed Testing 85

Michigan English Language Assessment Battery (MELAB)

Producer English language Institute University of Michigan Objective To test overall proficiency (language ability) Primary market Mostly US and Canadian language programs and colleges

some worldwide educational settings as well Type Paper-based Response modes Multiple-choice responses essay Time allocation 25 to 35 hours Internet access wwwlsaumicheduelimelabhtm

Specifications The MElAB consists of three sections Part 1 a 3D-minute impromptu essay is written on an assigned topic Part 2 a 25-minute multiple-choice listening comshyprehension test is delivered via tape recorder Part 3 is a 100-item 75-minute multipleshychoice test containing grammar doze reading vocabulary and reading comprehension An oral interview (speaking test) is optional

Comments The Ell at the University of Michigan has been producing the MELAB and its earlier incarnation (Michigan Test of English language Proficiency) since 1961 like the TOEFL it serves a North American audience but is also used internationally While its use is not as widespread as the TOEFL its validity is widely respected Because it is cheaper than the TOEFL and more easily obtained it is popular among language schools and institutes Many institutions and companies accept MElAB scores in lieu ofTOEFL scores

International English Language Testing System (IELTS)

Producer Jointly managed by The University of Cambridge local Examinations Syndicate (UClES) The British Council and lOP Education Australia

Objective To test overall proficiency (language ability) Primary-market Australian British Canadian and New Zealand academic

institutions and professional organizations American academic institutions are increasingly accepting IELTS for admissions purposes

1)rpe Computer-based (for the Reading and Writing sections) papershybased for the listening and Speaking modules

Response modes Multiple-choice responses essay oral production Time allocation 2 hours 45 minutes Internet access httpwwwieltsorgl

httpwwwudesorguk httpwwwbritishcouncilorg

Specifications Reading candidates choose between academic reading or general training reading (60 minutes) Writing the same option academic writing or general training writing (60 minutes) Listening four sections for all candidates (30 minutes) Speaking five sections for all candidates (1015 minutes)

86 CHAPTER 4 Standardized Testing

Comments The University of Cambridge local Examinations Syndicate (UCLES) has been producing English language tests since 1858 Now with three organizations cooperatshying to form the IELTS more than a million examinations are administered every year In 2002 a computer-based version of the Reading and Writing modules of the IELTS became available at selected centers around the world The other sections are administered locally by an examinet The paper-based IELTS remains an option for candidates The IELTS retains the distinct advantage of requiring all four skills in the test-takers performance

Test of English for International Communication (TOEICreg)

Producer The Chauncey Group International a subsidiary of Educational Testing Service

Objelttive To test overall proficiency (langlJage ability) Primary market Worldwide business commerce and industry contexts

(workplace settings) Type Computer-based and paper-based versions Response modes Multiple-choice responses Time allocation 2 hours Internet access httpwwwtoeiccom

Specifications Listening Comprehension 100 items administered by audiocassette Four types of task statements questions short conversations and short talks (approxishymately 45 minutes) Reading 100 items Three types of task cloze sentences error recogshynition and reading comprehension (75 minutes)

Comments The TOEIC has become a very widely used international test of English proficiency in workplace settings where English is required for job performance The conshytent includes many different employment settings such as conferences presentations sales ordering shipping schedules reservations (etters and memoranda It is approprishyate to use in educational settings where vocational or workplace English courses are being offered

sect sect sect sect sect

The construction of a valid standardized test is no minor accomplishment whether the instrument is large- or small-scale The designing of specifications alone as this chapter illustrates requires a sophisticated process of construct valishydation coupled with considerations of practicality Then the construction of items and scoringinterpretation procedures may require a lengthy period of trial and error with prototypes of the final form of the testWith painstaking attention to all the details of construction the end product can result in a cost-effective timeshysaving accurate instrument Your use of the results of such assessments can provide useful data on learners language abilities But your caution is warranted as well for all the reasons discussed in this chapter The next chapter will elaborate on what lies behind that need for a cautious approach to standardized assessment

CHAPTER4 Standardized Testing 87

EXERCISES

[Note (I) Individual work (G) Group or pair work (C) Whole-class discussion]

1 (C) Tell the class about the worst test experience youve ever had Briefly anamiddot lyze what made the experience so unbearable and try to come up with sugshygestions for improvement of the test andor its administrative conditions

2 (G) In pairs or small groups compile a brief list of pros and cons of standardshyized testing Cite illustrations of as many items in each list as possible Report your lists and examples to the rest o~ the class

3 (I) Select a standardized test that you are quite familiar with (probably a recent experience) Mentally evaluate that test using the five principles of practicality reliability validity authenticity and washback Report yourevaluashytion to the class

4 (G) The appendix to this chapter provides sample items from Jour different tests of language proficiency In groups one test for each group analyze your test for (a) content validity (b) face validity and (c) authenticity

5 (C) Do you think that the sample TOEFL reading passage about pirates (pages 74-75) and the Graduate EssayTest prompt (pages 76-77) about a school board hiring committee have any culture bias Discuss this and other cultural biases you have noticed in tests Is it possible to design a test that is completely free of culture bias

6 (CG) Compare the differences in conceptualization of language proficiency represented by Swains model the TOEFL and the ACfFL philosophy Which one best represents current thinking about communicative language ability What are the strengths and weaknesses of each approach

FORYOlIILEURTHER READING

Gronlund Norman E (1998) Assessment of student achievement Sixth Edition Boston Allyn and Bacon

Gronlunds classic also mentioned in Chapter 3 offers a concise overview of features of standardized tests offering definitions and examples of the statistical considerations in interpreting scores His approach is unbiased cleady written and accessible to those who might fear the mathematics of standardized testing

Phillips Deborah 2001 Long1nan introductory course for the TOEFL test White Plains NY Pearson Education

A careful examination of this or any other reputable preparation course for a standardized language test is well worth a students time Note especially how the book acquaints the user with the specifications of the test and offers a number of useful strategie~ that can be llsed in preparation for the test and during irs adn1inistration

88 CHAPTER 4 Standardized Testing

APPENDIX TO CHAPTER 4

Commercial Proficiency Tests Sample Items and Tasks

Test of English a~ a Foreign Language (TOEFLreg)

Listening r

Part A

In this section you will hear short conversations between two people In some ofthe conversations each person speaks only once In other conversations one or both of the people speak more than once Each conversation is followed by one questionabQlt it Each question in this part has four answer choices You should click on the best answer to each question Answer the questions on the basis of what is stated or implied by the speakers Here is an example On the computerscreen you will see

[man and woman talking]

On the recording you will hear

(woman) Hey wheres your sociology book (man) At home Why carry it around when were just going to be taking

a test (woman) Dont you remember Professor Smith said we could us it during

the test (man) Ohl no Well Ive still got an hour right Im so glad I ran into you

You wiII then see and hear the question before the answer choices appear

What will the man probably do next

o Begin studying for the sociology test o Explain the problem to his professor o Go home to get his textbook o Borrow the womans book

To choose an answer you will click on an oval The oval next to that answer will darken After you click on Next and Confirm Answer the next conversation will be presented

Part B

In this section you will hear several longer conversations and talks Each conversation or talk is followed by several questions The conversations talks and questions will not be repeated The conversations and talks are about a variety of topics You do not need speshycial knowledge of the topics to answer the questions correctly Rather you should answer each question on the basis of what is stated or implied by the speakers in the conversashytions or talks

For most of the questions you will need to click on the best of four possible answers Some questions will have special directions The special directions will appear in a box on the computer screen Here is an exampie ot a conversation and some questions

CHAPTER 4 Standardized Testing 89

Marine Biology (narrator) Listen to part of a discussion in a marine biology class

(professor) A few years ago our local government passed a number of strict environmental laws As a result Sunrise Beach looks nothing Ii ke it did ten years ago The water is cleaner and theres been a tremendous increase in all kinds of marine life which is why were going there on Thursday

(woman) I dont know if I agree that the water quality has improved I mean I was out there last weekend and it looked all brown It didnt seem too clean to me

(professor) Actually the color of the water doesnt always indicate whether its polluted The brown color you mentioned might be a result of pollution or it can mean a kind of brown algae is growing there Its called devils apron and it actually serves as food for whales

(man) So when does the water look blue (professor) Well water thats completely unpolluted is actually colorless But

it often looks bluish-green because the sunlight can penetrate deep down and thats the color thats reflected

(woman) But sometimes it looks really green Whats that about (professor) Ok well its the same principle as with devils apron the

water might be green because of different types of green algae there-gulfweed phytoplankton You all should finish reading about algae and plankton before we go In fact those are the types of living things Im going to ask you to be looking for when were there

Now get ready to answer the questions

What is the discussion mainly about

o The importance of protecting ocean environments o The reasons why ocean water appears to be different colors o The survival of whales in polluted water o The effect that colored ocean water has on algae

To choose an answer click on an oval The oval next to that answer will darken After you click on Next and Confirm Answer the next question will be presented

According to the professor what can make ocean water look browngt

o Pollution o Cloudy Skies o Sand o Algae

Click on 2 answers

To choose your answers you will click on the squares An XII wiii appear in each square

bullbullbullbullbullbullbull

90 CHAPTER 4 Standardized Testing

Structure and Written Expression This section measures the ability to recognize language that is appropriate for standard written English There are two types ofquestions in this section In the first type ofquestion there are incomplete sentences Beneath each sentence there are four words or phrases

Directions CIiSk on the one word or phrase that best completes the sentence

The colum~ine flower __ to nearly all of the United States can be raised from seed in almost any garden

native how native is how native is it is native

Time Help Confirm

After you click on Next and Confirm Answ~ the next question willbe presented

The second type of question has four underlined words or phrases You will choose the one underlined word or phrase that must be changed for the sentence to be correct

Directions Click on the one underlined word or phrase that must be changed for the senshytence to be correct

One of the most difficult problems in understanding sleep is determining what the funcshytions of sleep ~

lime Help Confirm

Clicking on an underlined word or phrase will darken it

Reading This section measures the ability to read and understand short passages similar in topic and style to those that students are likely to encounter in North American universities and colleges This section contains reading passages and questions about the passages There are several different types of questions in this section In the Reading section you will first have the opportunity to read the passage

The temperature of the Sun is over 10000 degrees Fahrenheit at the surface but it rises perhaps more than 270000000 at the center The Sun is so much hotter than the Earth that matter can exist only as a gasl except perhaps at the core In the core of the Sun the pressures are so great that despite the high temperature there may be a small solid core However no one really knows since the center of the Sun can never be directly observed ~ Solar astronomers do know that the Sun is divided into five general layers or zones Starting at the outside and going down into the Sun the zones are the corona chromoshysphere hotosphere convection zone and finally the core The first three zones are reshygarded as the Suns atmosphere But since the Sun has no solid surface it is hard to middottell where the atmosphere ends and the main body of the Sun begins

The Suns outermost layer begins about 10000 miles above the visible surface and goes outward for millions of miles This is the only part of the Sun that can be seen during an eclipse such as the one in February 1979 At any other time the corona can be seen

bullbullbullbullbullbullbull

bull bullbullbullbullbullbull

CHAPTER 4 Standardized Testing 91

only when special instruments are used on cameras and telescopes to block the light from the photosphere

The corona is a brilliant pearly white filmy light about as bright as the full Moon Its beautiful rays are a sensational sight during an eclipse The coronas rays flash out in a brilliant fan that has wispy spikelike rays near the Suns north and south poles The corona is generally thickest at the Suns equator The corona is made up of gases streamshying outward at tremendous speeds that reach a temperature of more than 2 million deshygrees Fahrenheit The gas thins out as it reaches the space around the planets By the time the gas of the corona reaches the Earth it has a relatively low density

When you have finished reading the passage you will use the mouse to click on Proceed Then the questions about the passage will be presented You are to choose the one best anshyswer to each question Answer all questions about the information in a passage on the basis ofwhat is stated or implied in that passage Most ofthe questions will be multiple-choice questions To answer these questions you will click on a choice below the question

With what topic is paragraph 2 mainly concerned

o How the Sun evolved o The structure of the Sun o Why scientists study the Sun o The distaflce of the Sun from the planets

Paragraph 2 is marked with an arrow (~)

You will see the next question after you click on Next

To answer some questions you will click on a word or phrase Here is an example

Look at the word one in the passage Click on the word or phrase in the bold text that one refers to To answer you can click on any part of the word or phrase in the passage Jour choice will darken to show which word you have chosen

The Suns outermost layer begins about 10000 miles above the visible surface and goes outward for millions of miles This is the only part of the Sun that can be seen durshying an eclipse such as the one in February 1979 At any other time the corona can be seen only when special instruments are used on cameras and telescopes to block the Iight from the photosphere

You will see the next question after you click on~ To answer some q~estions you will click on a sentence in the passage Here is an example

~ The corona is a brilliant pearly white filmy light about as bright as the full Moon Its beautiful rays are a sensational sight during an eclipse The coronas rays flash out in a brilliant fan that has wispy spikelike rays near the Suns north and south poles The corona is generally thickest at the Suns equator ~ The corona is made up of gases streaming outward at tremendous speeds that reach a temperature of more than 2 million degrees Fahrenheit The gas thins out as it reaches the space around the planets By the time the gas of the corona reaches the Earth it has a relatively low density

bull bullbullbullbullbullbull

92 CHAPTER 4 Standardized Testing

Click on the sentence in paragraph 4 or 5 in which the author compares the light of the Suns outermost layer to that ofanother astronomical body Paragraphs 4 and 5 are marked with arrows (~)

To answer some questions you will click on a square to add a sentence to the passage Here is an example -The following sentence can be added to paragraph 1

At the center of the Earths solar system lies the Sun

Where would it best fit in paragraph I Click on the square to add the sentence to the paragraph

D The temperature of the Sun is over 10000 degrees Fahrenheit at the surface but it rises to perhaps morethan 27000000deg at the center 0 The Sun is so much hotter than the Earth that matter can exist only as a gasi except p~rHapsatth~ c6relp the c~re of the ii Sun the pressures are so great that despite the high temperature there may be a small solid core D However no one really knows since the center of the Sun can never be directly observed D 0100

When you click on a square the sentence will appear in the passage at the place you have chosen You can read the sentence added to the paragraph to see if this is the best place to add it You can click on another square to change your answer The sentence will be added and shown in a dark box

Writing In this section you will have an opportunity to demonstrate your ability to write in Enshyglish This includes the ability to generate and organize ideas to support those ideas with examples or evidence and to compose in standard written English in response to an asshysigned topic You will have 30 minutes to write your essay on that topic You must write on the topic you are assigned An essay on any other topic will receive a score of 0 Read the topic below and then make any notes that will help you plan your response Begin typing your response in the box at the bottom of the screen or write your answer on the answer sheet provided to you

Following is a sample topic

Do you agree or disagree with the following statemenH

Teachers should make learning enjoyable and fun for their students

Use specific reasons and examples to support your opinion

CHAPTER 4 Standarczed Testing 93

Michigan English Language Assessment Battery (MELAB)

Composition The time limit for the composition is 30 minutes You must write on only one of the top~

ics below If you write about something else your composition paper will not be graded and you cannot be given a final score If you do not understand the topics ask the exam~ iner to explain or to translate them You may be asked to give your opinion ofsomething and explain why you believe this to describe something from your experience or to exshyplain a problem and offer possible solutions You should write at least one page Some sample topics are

1 What do you think is your countrys greatest problem Explain in detail and tell what you think can be done about it

2 What are the characteristics of a good teacher Explain and give examples 3 An optimist is someone who sees the good side of things A pessimist sees the

bad side Are you an optimist or a pessimist Relate a personal experience that shows this

4 In your opinion are the benefits of space exploration really worth the enormous costs Discuss

Most MELAB compositions are one or two pages long (about 200-300 words) If your paper is extremely short (less than 150 words) your composition will be given a lower score Before you begin writing you might want to take 2 or 3 minutes to plan your comshyposition and to make a short outline to organize your thoughts Such outlines will not be graded they are only to help you You should use the last 5 minutes to read through your composition and to make changes or corrections

Your composition will be graded on how clearly you express yourself in English and on the range of English you are able to use and your control in doing so This means your composition should be well organized your arguments should be fully developed and you should show a range ofgrammatical structures and broad vocabulary Compositions that consist only of very short sentences and very simple vocabulary cannot be given the

middothighest scores If errors are not frequent and if they do not confuse your meaning they will not lower your score very much

Listening Now you will hear a short lecture You may take notes during the lecture Following the lecture you will be asked some questions about it

Therell be a two-week exhibit of the paintings of the little-known master Laura Bernhart at the Claire Osmond Galleries starting on the fifteenth of the month and running through the thirtieth Bernharts known for her innovative designs in abstract expressionism Though a true original she declared a spiritual heritage from Salvador Dali the famous Spanish painter Since Bernhart lived a rather solitary life and died while only in her twenties few people are aware of her works This showing at the Osmond Galleries will provide many with an introduction to her works

10 Where is the exhibit a the Art Museum b the Dali Galleries c the Osmond Galleries

94 CHAPTER 4 Standardized Testing

11 What is Bernhart known for a her copies of Dalis paintings b the originality of her designs c her exhibitions

12 What will going to the exhibit allow most people to do a to see Saivador Dalis paintings b to see Bernharts works for the first time c to learn about Spanish art

Grammar

1 What did the teacher just tell you

She reminded our notebooksI a us to bring b that we bring c our bringing d we should bring

2 Is Bill a good dancer

Not really __ he tries very hard a in spite of h despite c even though d while

3 your clothes are all wet1

Yes I didnt come __ the rain soon enough a away to b over to c down with d in from

Cloze In years to come zoos will not only be places where animals are exhibited to the public but repositories where rare species can be saved from extinction (7) captive breeding The most powerful force (8) the future of many animals-and of zoos-is the decline of the wild (9) even zoo directors would argue that (10) are better places for animals than the fields and forest of their native (11) yet zoos may be the last chance for some creatures that would otherwise pass qUietly into oblivion

7 a through c from b of d damage

8 a bringing c to b that d influencing

9 a But c Not b So d Then

10 a where c even b zoos d wilds

11 alands c residence b life d field

CHAPTER 4 Standardized Testing 95

Vocabulary

12 Mark has a flair for writing a need b purpose c talent d dislike

13 Bill Collins launched his restaurant last June a moved b started c sold d bought

14 John will not accept the censure a burden b blame c credit d decision

15 I cant think of the answer Can you give me a __ a hint b token c taste d gaze

16 Because fewer people are taking expensive vacations the tourist industry is in a a choke b grope c grumble d slump

17 I disagree with a few of his opinions but __ we agree a deliberately b conclusively c essentially d immensely

Reading The influenza virus is a single molecule built from many millions of single atoms You must have heard of the viruses which are sometimes called living molecules While bacteria can be considered as a type of plant secreting pOisonous substances into the body of the organism they attack viruses are living organisms themselves We may conshysider them as regular chemical molecules since they have a strictly aefined atomic strucshyture but on the other hand we must also consider them as being alive since they are able to multiply in unlimited quantities

18 According to the passage bacteria are a poisons

b larger than viruses c very small d plants

96 CHAPTER 4 Standardized Testing

19 The writer says that viruses are alive because they a have a complex atomic structure b move c multiply d need warmth and light

20 The atomic structure of viruses a is -tJIariable b is strictly defined c cannot be analyzed chemically d is more complex than that of bacteria

International English Language Testing System (fELTS)

I

listening

The Listening Module has four sections The first two sections are concerned with social needs There is a conversation between two speakers and then a monologue For examshyple a conversation about travel arrangements or decisions on a night out and a speech about student services on a university campus or arrangements for meals during a confershyence The final two sections are concerned with situations related more closely to educashytional or training contexts For example conversation between a tutor and a student about an assignment or between three students planning a research project and a lecture or talk ofgeneral academic interest All the topics are ofgeneral interest and it makes no difference what subjects candidates study Tests and tasks become more difficult as the sections progress A range of English accents and dialects are used in the recording which reflects the international usage of IELTS

Academic Reading [A 7S0-word article on-th-e- topic of Wind Power in the US with a short glossary at the end]

Questions 1-5

Complete the summary below

Choose your answers from the box below the summary and write them in boxes 1-5 on your answer sheet Note There are more words or phrases than you will need to fill the gaps You may use any word or phrase more than once

Example The failure during the late 1970s and early 19805 of an attempt to establish a widespread wind power industry in the United States resulted largely from the (1) bull in oil prices during this period The industry is now experiencing a steady (2) due to improveshyments in technology and an increased awareness of the potential in the power of wind The wind turbines that are now being made based in part on the (3) of wide- ranging research in Europe are easier to manufacture and maintain than their predecesshysors This has led wind-turbine makers to be able to standardise and thus minimize (4) There has been growing (S) of the importance of wind power as an energy source

CHAPTER 4 Standardized Testing 97

criticism stability skepticism success operating costs decisions design costs fall effects production costs growth decline failure recognition results

Questions 6-1 0 Look at the following list of issues (Questions 6-10) and implications (A-C) Match each issue with one implication Write the appropriate letters A-C in boxes 6-10 on your anshyswer sheet

Example The current price of one wind-generated kilowatt Answer

6 The recent installation of systems taking advantage of economies of scale

7 The potential of meeting one fifth of current U5 energy requirements by wind power

8 The level of acceptance of current wind turbine technology

9 A comparison of costs between conventional and wind power sources

10 The view of wind power in the European Union

Implications

A provides evidence against claims that electricity produced from wind power is relatively expensive

B supports claims that wind power js an important source of energy

C opposes the view that wind power technology requires further-development

General Training Reading Read the passage on Daybreak trips by coach and look at the statements below On your answer sheet write

TRUE if the statement is true FALSE jf the statement is false

NOlGIVEN if the information is not given in the leaflet

1 MiIlers Coaches owns Cambridges Cam bus fleet

2 Premier is an older company than Millers

3 Most of the Daybreak coaches are less than 5 years old

4 Daybreak fares are more expensive than most of their competitors

5 Soft drinks and refreshments are served on most longer journeys

6 Smoking is permitted at the rear of the coach on longer journeys

7 Tickets must be bought in advance from an authorised Daybreak agent

6 Tickets and seats can be reserved by phoning the Daybreak Hotline

9 Daybreak passengers must join their coach at Cambridge Drummer Street

10 Daybreak cannot guarantee return times

98 CHAPTER 4 Standardized Testing

FROM CAMBRIDGE AND SURROUNDING AREA

SPRING IS INTHEAIR

Welcome to our Spring Daybreak programme which continues the tradition of offering unbeatable value for money day trips and tours All the excursions in this brochure will be operated by Pr~mier Travel Services Limited or Millers Coaches both companies are part of the CHLGroup owners of Cambridges Cambus fleet

WERE PROUD OF OUR TRADITION

Premier was established in 1936 the Company now offers the highest standards of coaching in todays competitive operating environment Miller has an enviable reputation stretching back over the past 20 years offering coach services at realistic prices Weve traveled a long way since our early days of pre-war seaside trips Now our fleet of 50 modern coaches (few are more than five years old) operate throughout Britain and Europe but were pleased to still maintain the high standards of quality and service the trademark of our founders nearly sixty years ago

EXCLUSIVE FEATURES

Admission-inclusive fares All Daybreak fares (unless specifically otherwise stated) include admission charges to the attractions shows and exhibits we visit Many full-day scenic tours are accompanied by a fully trained English Tourist Board Blue Badge guide or local experienced driverguide Some Daybreaks include lunch or afternoon tea Compare our admission inclusive fares and see how much you save Cheapest is not the best and value for money is guaranteed If you compare our bargain Daybreak fares beware--most of our competishytors do not offer an all-inclusive fare

SEAT RESERVATIONS

We value the freedom of choice so you can choose your seat when you book The seat reservation is guaranteed a-nd remains yours at all times when aboard the coach

NO SMOKING COMFORT

With the comfort of our passengers in mind coaches on all our Daybreaks are no smokshying throughout In the interests of fellow passengers comfort we kindly ask that smokers observe our no smoking policy On scenic tours and longer journeys ample refreshment stops are provided when of course smoking is permitted

YOUR QUESTIONS ANSWERED

Do I need to book Booking in advance is strongly recommended as all Daybreak tours are subject to demand Subject to availability stand-by tickets can be purchased from the driver

What ti me does the coach leave The coach departs from Cambridge Drummer Street (Bay 12 adjacent to public toilets) at the time shown There are many additional joining points indicated by departure codes in the brochure If you are joining at one of our less popular joining points you will be adshyvised of your pick-up time (normally by telephone) not less than 48 hours before deparshyture In this way we can minimize the length of pick-up routes and reduce journey times for the majority of passengers

CHAPTER 4 Standardized Testing 99

What time do we get back An approximate return time is shown for each excursion The tim~s shown serve as a guide but road conditions can sometimes cause delay If your arrival will be later than advertised your driver will try to allow for a telephone call during the return journey

Where can I board the coach All the Daybreaks in the brochure leave from Cambridge Drummer Street (Bay 12 adjashycent to public toilets) at the time shown Many Daybreaks offer additional pick-ups for pre-booked passengers within Cambridge and the surrounding area This facility must be requested at the time of booking

Academic Writing Writing Task 1 You should spend about 20 minutes on this task

The graph below shows the different modes of transport used to travel to and from work in one European city in 1950 1970 and 1990

[graph shown here]

Write a report for a university lecturer describing the information shown below You should write at least 150 words

Writing Task 2 You should spend about 40 minutes on this task

Present a written argument or case to an educated reader with no specialist knowledge of the folowing topic

It is inevitable that as technology develops so traditional cultures must be lost Technolshyogy and tradition are incompatible-you cannot have both together

To what extent do you agree or disagree with this statement Give reasons for your answer You should write at least 250 words You should use your own ideas knowlshyedge and experience and support your arguments with examples and relevant evidence

General Training Writing Writing Task 1 You should spend about 20 minutes on this task You rent a house through an agency The heating system has stopped working You phoned the agency a week ago but it has still not been mended Write a letter to the agency Explain the situation and teil them what you want them to do about it

You should write at least 150 words You do NOT need to write your own address

Begin your letter as follows

Dear - ___-I

Writing Task 2 You should spend about 40 minutes on this task As part ofa class assignment you have to write about the following topic

100 CHAPTER 4 Standardized Testing

Some businesses now say that no one can smoke cigarettes in any of their offices Some governments have banned smoking in all public placesThis is a good idea but it takes away some of our freedom

Do you agree or disagree Give reasons for your answer You should write at least 250 words

Speaking In each ofthe three parts of the speaking module a specific function is fulfilled In Part 1 the candidates answer general questions about themselves their homes or families their jobs or studies their interests and a range ofsimilar familiar topic areas This part lasts between four and five minutes In Part 2 the candidate is given a verbal prompt on a card and is asked to talk on a particular topic The candidate has one minute to prepare before speaking at length for between one and two minutes The examiner then asks one or two wind-down questions In Part 3 the examiner and candidate engage in a discusshysion of more abstract issues and concepts which are thematically linked to the topic prompt in Part 2 The discussion lasts between four and five minutes

All interviews are recorded on audiocassette Here is a sample ofa Part 2 topic

Describe a teacher who has greatly influenced you in your education

You shou Id say

where you met them what subject they taught what was special about them

and explain why this person influenced you so much

You will have to talk about the topic for 1 to 2 minutes You have 1 minute to think about what you are going to say You can make some notes if you wish

Test of English for International Communication (TOEICreg)

listening

Part 1 Photographs Directions For each question you will see a picture in your test book and you will hear four short statements The statements will be spoken just one time They will not be printed in your test book so you must listen carefully to understand what the speaker says When you hear the four statements look at the picture in your test book and choose the statement that best describes what you see in the picture Then on your answer sheet find the number of the question and mark your answer

[photograph of a scientist looking through a microscope]

You will hear Look at the picture marked number 1 in your test book

(A) Shes speaking into a microphone (B) Shes put on her glasses (C) She has both eyes open (D) Shes using a microscope

CHAPTER 4 Standardized Testing 101

Part 2 Question-Response Directions In this part of the test you will hear a question or statement spoken in Enshyglish followed by three responses also spoken in English The question or staten1ent and the responses will be spoken just one time They will not be printed in your test book so you must listen carefully to understand what the speakers say You are to choose the best response to each question or statement

Question 1 You will hear Ms Morikawa has worked here for a long time hasnt she

(A) At three oclock (B) No Ive lost my watch (C) More than ten years

Question 2 You will hear Which of these papers has a wider circulation

(A) The morning edition (B) Get more exercise (C) By messenger

Part 3 Short Conversations Directions In this part of the test you will hear short conversations between two people The conversations will not be printed in your test book You will hear the conversations only once so you must listen carefully to understand what the speakers say In your test book you will read a question about each conversation The question will be followed by four answers You are to choose the best answer to each question and mark it on your answer sheet

Question 1 (Man) We should think about finding another restaurant for lunch (Woman) Why The food and service here are great

(Man) Yes but the prices are going up every week

You will read Why is this man unhappy with the restaurant

(A) It is too noisy (B) It is too expensive (C) It is too crowded (D) It is too difficult to find

Question 2 (Woman A) How was Dr Borgs recent trip to Singapore (Woman B) She enjoyed the tour of the port very much (Woman A) They say its one of the most active in Asia

You will read 2 What did Dr Borg find interesting

(A) The tourist center (B) The airport (C) The musical performance (D) The harbor

Part 4 Short Talks Directions In this part of the test you vill hear several short talks Each will be spoken just one time They will not be printed in your test book so you must listen carefully to understand and remember what is said In your test book you will read two or more questions about each short talk The questions will be followed by four answers You are to choose the best answer to each question and mark it on your answer sheet

102 CHAPTER 4 Standardized Testing

You will hear Questions 1 and 2 refer to the following announcement

Good afternoon and welcome aboard Nordair Flight 857 from Copenhagen to Bangkok with intermediate stops in Dubai and Calcutta We are preparing for departure in a few minutes At this time your seat back should be returned to its full upright position and your seat belt s~ould be fastened OUf anticipated total flying time to Dubai is six hours and twenty-five minutes I hope you enjoy the flight You will hecJr Now read question 1 in your test book and answer it You will read 1 What is the final destination of the flight

(A) Bangkok (B) Copenhagen (C) Dubai (O) Calcutta

You will hear Now read question 2 in your test book and answer it You will read 2 What will happen in a few minutes

(A) The flight will land in Dubai I

(B) The passengers will board the plane (C) The plane will take off (0) The gate number will be announced

Reading In this section of the test you will have the chance to show how well you understand written English There are three parts to this section with special directions for each part

Part 4 Incomplete Sentences Directions This part of the test has incomplete sentences Four words or phrases marked (A) (8) (e) (D) are given beneath each sentence You are to choose the one word or phrase that best completes the sentence Then on your answer sheet find the number of the question and mark your answer

1 Mr Yangs trip will __ him away from the office for ten days (A) withdraw (B) continue (C) retain (0) keep

2 The company that Marie DuBois started now sells __ products throughout the world (A) its (B) it (C) theirs (D) them

3 If your shipment is not delivered __ Tuesday you can request a full refund for the merchandise (A) at (B) by (C) within (D) while

CHAPTER 4 Standardized Testing 103

Part 6 Error Recognition Directions In this part ofthe test each sentence has four words or phrases underlined The four underlined parts of the sentence are marked (A) (B) (C) (D) You are to identify the one underlined word or phrase that should be corrected or rewritten Then on your answer sheet find the number of the question and mark your answer

1 The pamphlet contains some importance information about the current exhibit ABC D

2 No matter how Jong it taking to finish the annual report it must be done properly ABC D

3 The popularity of jogging appears to have decreased since the past couple of years ABC D

Part 7 Reading Comprehension Directions The questions in this part of the test are based on a selection of reading mateshyrials such as notices letters) forms newspaper and magazine articles) and advertisements You are to choose the one best answer (A) (B) (C) or (OJ to each quesshytion Then on your ariswefsheelfindthe number of the qUestion andmcirkyour answer Answer all questions following each reading selection on thebasis of what is stated or implied in that selection

The Museum ofTechnology is a hands-on museum designed for people to experience science at w()rk~ Visitors are encouraged to use test and handle the objects o~ display Special demonstrations are scheduled for the first and second Wednesdays of each month at 1330 Open Tuesday-Friday 1200-1630 Saturday 1000-1730 and Sunday 11 00-1630

1 When during the month can visitors see special demonstrations (A) Every weekend (B) The first two Wednesdays (C) One afternoon a week (D) Every other Wednesday

Questions 2 and 3 refer to the followi ng notice

NOTICE If you are unable to work because of an extended illness or injury that is not workshyrelated you may be entitled to receive weekly benefits from your employer or the firms insurance company To claim benefits you must file a claim form within thirty days of the first day of your disability Before filing the claim you must ask your doctor to fill in the Doctors Statement on the claim form stating the period of disability

3 To whom is this notice addressed (A) Employers (8) Doctors (C) Employees (D) When paying the bill

4 When must the claim form be filed (A) On the first of the month (8) On the thirtieth of the month (C) On the first day ofdisabifity (D) Within 30 days of the start of disability

Page 7: Standardized Testing Chapter 4 Brown

72 CHAPTER 4 Standardized Testing

tasks In 1996 a major step was taken to include written production in themiddot computershybased TOEFL by adding a slightly modified version of the already existing Test of Written English (TWE) In doing so some face validity and content validity were improved along with of course a significant increase in administrative expense Each of these four major sections is capsulized in the box below (adapted from the descripshytion of the current computer-based TOEFL at wwwroefLorg) Such descriptions are not strictly speaking specifications which are kept confidential by ETS Nevertheless they can give a sense of many of the constraints that are placed on themiddot design of actual TOEFL specifications

TOEFLlt8gt specifications

Listening Section The listening section measures the examinees ability to understand English as it is spoken in North America Conversational features of the language are bull stressed and the skills tested include vocabulary and idiomatic expression as well as speshycial grammatical constructions that are frequently used in spoken Engfish The stimulus material and questions are recorded in standard North American English

The listening section includes various stimuli such as dialogues short conversations academic discussions and mini-lectures and poses questions that test comprehension of main ideas the order ofa process supporting ideas important details and inferences as well as the ability to categorize topicsobjects

The test developers have taken advantage of the multimedia capability of the computer by using photos and graphics to create context and support the content of the lectures producing stimuli that more closely approximate Ureal-world situations in which people do more than just listen to voices The listening stimuli are often accompashynied by either context-setting or content-based visuals All dialogues conversations acashydemic discussions and mini-lectures include context visuals to establish the setting and role of the speakers Content-based visuals are often used to complement th~ topics of the mini-lectures

Structure-Section Themiddotstructure-section measures an examinees ability to recognize language that is appropriate for standard written English The language tested is formal rather than conversational The topics of the sentences are associated with general acadeshymic discourse so that individuals in specific fields of study or from specific national or linguistic groups have no particular advantage

Two types of questions are used questions in which examinees must (1) complete an incomplete sentence using one of four answers provided and (2) identify one of four unshyderlined words or phrases that would not be accepted in English The two question types are mixed randomly rather than being separated into two subsections as in the papershybased TOEFL test

Reading Section The reading section measures the ability to read and understand short passages similar in topic and style to academic texts used in North American colshyleges and universities Examinees read a variety of short passages on academic subjects and answer several questions about each passage Test items refer to what is stated or imshyplied in the passage as well as to words used in the passage To avoid creating an advanshyt~ t( 1 dull5- In 3n~ ont fidd of stud~ sufficient context is provided SO that no ~ -middott - ~tmiddot J ~ middoth - -~ rM~ ir~ to ~-er the if~(J(lS~ ~ t ~ ~ ~~~~-- Ih L_ 1_ 1 ~_

CHAPTER 4 Standardized Testing 73

The reading section consists of four to five passages of 250-350 words with 10-14 questions per passage This section is not computer-adaptive soexaminees can skip questions and return to previous questions The questions in this section assess the comshyprehension of main ideas inferences factual information stated in a passage pronoun referents and vocabulary (direct meaning synonym antonym) In all cases the questions can be answered by reading and understanding the passages This section consists of (1 ) traditional multiple-choice questions (2) questions that require examinees to click on a word phrase sentence or paragraph to answer and (3) questions that ask examinees to insert a sentence where it fits best

Writing Section The writing section measures the ability to write in English including the ability to generate organize and develop ideas to support those ideas with examples or evidence and to compose a response to one assigf)ed topic in standard written Enshyglish Because some examinees may not be accustomed to composing an essay on comshyputer they are given the choice of handwriting or typing the essay in the 30-minute time limit The rating scale for scoring the essay ranging from 0 to 6 is virtually the same as that of the Test of Written English [see Chapter 9 of this book] A score of 0 is given to papers that are blank simply copy the topic are written ina language other than English consist only of random keystroke characters or are written on a topic different from the one assigned

Each essay is rated independently by two trained certified readers Neither reader knows the rating assigned by the other An essay will receive the average of the two ratshyings unless there is a discrepancy of more than one point in that case a third reader will independently rate the essay The essay rating is incorporated into the StructureMriting scaled score~ and constitutes approximately 50 percent of that combined score

(B) The designing of the test specs for the ESLPT was a somewhat simpler task because the purpose is placement and the construct validation of the test consisted of an examination of the content of the ESL courses In fact in a recent revisiofi of the ESLPT (lmao et al 2000 Imao 2001) content validity (coupled with its attenshydant face validity) was the central theoretical issue to be considered The major issue centered on designing practical and reliable tasks and item response formats Having established the importance of designingESLPT tasks that simulated classroom tasks used in the courses the designers ultimately specified two writing production tasks (one a response to an essay that students read and the other a summary of another essay) and one multiple-choice grammar-editing taskThese specifications mirrored the readingbased process writing approach used in the courses

(C) Specifications for the GET arose out of the perceived need to provide a threshold of acceptable writing ability for all prospective graduate students at SFSU both native and non-native speakers of EnglishThe specifications for the GET are the skills of writing grammatically and rhetorically acceptable prose on a topic of some interest with clearly produced organization of ideas and logical development The GET is a direct test of writing ability in which test-takers must in a two-hour time period write an essay on a given topic

74 CHAPTER 4 Standardized Testing

3 Design select and arrange test tasksitems

Once specifications for a standardized test have been stipulated the sometimes never-ending task of designing selecting and arranging items beginS The spe~s act much like a blueprint in determining the number and types of items to be created Lets look at the three examples

(A) TOEFL test design specifies that each item be coded for content and statisshytical characteristics Content coding ensures that each examinee will receive test questions that assess a variety of skills (reading comprehending the main idea or understanding inferences) and cover a variety of subject matter without unduly biasing the content toward a subset of test-takers (for example in the listening secshytion involving an academic lecture the content must be universal enough for stushydents from many different academic fields of study) Statistical characteristics including the IRT equivalents of estimates of item ~~ility (IF) and the ability of an item to discriminate (ID) between higher orlower ability levels ate also coded

Items are then designed by a team who select and adapt items solicited from a bank of items that have been deposited by freemiddotlance writers and ErS staff Probes for the reading section for example are usually excerpts from authentic general or academic reading that are edited for linguistic difficulty culture bias or other topic biases Items are designed to test overall comprehension certain specific informashytion and inference

Consider the following sample of a reading selection and ten items based on it from a practice TOEFL (Phillips 2001pp423-424)

For hundreds of years in the early history of America pirates sailed through coastal washyters pillaging and plundering all in their path They stole from other ships andstole from coastal towns not content only to steal they destroyed everything they could not carry avay~ Some of the pirate ships amassed large treasures~ the fates of which are unknown leaving people of today to wonder at their whereabouts and to dream of one day coming across some lost treasure

One notoriously large treasure was on the pirate ship Whidah which sank in the washyters off Cape Cod during a strong storm in 1717 A hundred of the crew members went down with the ship along with its treasure of coins gold silver and jewels The treasure on board had an estimated value on todays market of more than 100 million dollars

The remains of the Whidah were discovered in 1984 by Barry Clifford who had spent years of painstaking research and tireless searching only finally to locate the ship about 500 yards from shore A considerable amount of treasure from the centuries-old ship has been recovered from its watery grave but there is clearly still a lot more out there Just as a reminder of what the waters off the coast have been protecting for hundreds of years occasional pieces of gold or silver or jewels still wash up on the beaches and lucky beach-goers find pieces of the treasure

11 ~ Thepa~e mainly diccus5eS

CH4PTER 4 Standardized Testing 75

(e) what really happened to the Whidahs pirates (D) why people go to the beach

12 It is NOT mentioned in the passage that pirates did which of the following (A) They killed lots of people (B) They robbed other ships (e) They took things from towns (D) They gathered big treasures

13 The word amassed in line 4 is closest in meaning to (A) sold (e) transported (B) hid (D) gathered

14 It is implied in the passage that the Whidahs crew (A) died (B) went diving (e) searched for the treasure (D) escaped with parts of the treasure

15 Which of the following is NOT mentioned as part of the treasure of the Whidah (A) Art objects (B) Coins (e) Gold and si Iver (D) Jewels

16 The word estimated in line 10 is closest in meaning to which of the following (A) Known (C) Approximate (B) Sold (D) Decided

17 The passage indicates that the cargo of theWhidah is worth about (A) $100000 (B) $1000000 (C) $10000000 (D) $100000000

18 The work that Barry Clifford did to locate the Whidah was NOT (A) successfu I (B) effortless (C) detailed (D) lengthy

19 It is mentioned in the passage that the treasure of the Whidah (A) is not very valuable (8) is all in museums (C) has not all been found (D) was taken to share by the pi rates

20 The paragraph following the passage most likely discusses (A) what Barry Clifford is doing today (8) the fate of the Whidahs crew (e) other storms in the area of Cape Cod (D) additional pieces that turn up from the Whidahs treasure

76 CHAPTER 4 Standardized Testing

As you can see items target the assessment of comprehension of the main idea (item 11) stated details (17 19) unstated details (12 15 18) implied details (14 20) and vocabulary in context (13 16) An argument could be made about the cultural schemata implied in a passage about pirate ships and you could engage in an angels on the head of a pin argument about the importance of picking cershytain vocabulary for emphasis but every test item is a sample of a larger domain and each of these fulfills its designated specification

Before any such items are released into a form of the TOEFL (or any validated standardized test) they are piloted and sCientifically selected to meet difficulty specshyifications within each subsection section and the test overall Furthermore those items are also selected to meet a desired discrimination index Both of these indices are important considerations in the design of a computer-adaptive test where pershyformance on one item determines the next one to be presented to the test-taker (See Chapter 3 for a complete treatment of multiple-choice item design)

(B)The selection of items in the ESLPT entailed two-entirel) different processes In the two subsections of -the test that elicit writing performance (summary of reading response to reading) the main hurdles were (a) selecting appropriate passhysages for test-takers to read (b) providing appropriate prompts and (c) processing data from pilot testing Passages have to conform to standards of content validity by being within the genre and the difficulty of the material used in the courses The prompt in each case (the section asking for a summary and the section asking for a response) has to be tailOred to fit the passage but a general template is used

[n the multiple-choice editing test that seeks to test grammar proofreading ability the first and easier task is to choose an appropriate essay within which to embed errors The more complicated task is to embed a specified number of errors from a previously determined taxonomy of error categories Those error categories came directly from student errors as perceived by their teachers (verb tenses verb agreeshyment logical connectors articles etc) The disttactors for each item were selected from actual errors that students make Itemsiti pilot versions were then coded fordifshyficulty and discrinlination indices after which final assembly of items could occur

(C) The GET prompts are designed by a faculty committee of examiners who are speCialists in the field of university academic writing The assumption is made that the topics are universally appealing and capable of yielding the intended product of an essay that requires an organized logical argument and conclusion No pilot testing of prompts is conducted The conditions for administration remain constant two-hour time limit sit-down context paper and pencil closed-book format Consider the following recent prompt

Graduate Essay Test sample prompt

In the Middletown Elementary School District the assistant superintendent has just been made superintendent in another district Her resignation leaves vacant the districts only administrative position ever held by a woman The School Board in response to strong

CHAPTER 4 Standardized Testing 77

arguments from the Teachers Association has urged that a woman be hired to replace her As a member of the hiring committee you must help choose her successor

Only one woman applicant meets the written qualifications for the job the two top male applicants are both more experienced than she

The hiring committee has asked each committee member to prepare a written statement to distribute before meeting together to discuss the issue Write a report that represents your position making it as logical and persuasive as possible

Some facts you may wish to draw on 1 Women make up more than 75 percent of classroom teachers but hold fewer than

10 percent of administrative positions in education Administrators salaries average 30 percent more than teachers salaries

2 The local Teachers Association is 89 percent women mostly under 40 In a heated debate on television a member of the National Organization of Women (NOW) and the chair of the Teachers Association threatened if a man is hired to bring a class-action suit against the district on behalf of all women teachers who cannot expect advancement because of discriminatory hiring practices

3 The local Lions Club which contributes heavily to school sports says hiring the less experienced woman would not be in the best interests of the schoolthe children or the teachers

The finalists for the position

1 Carole Gates Classroom teacher 10 years Teacher of the Year 1985 supervisor ofpractice teachers at Teachers College former president of Teachers Associ ati on Administrative Credential 1984 EdD degree 1986 assistant principal of Hoptown Elementary School 2 years

2 Spud Stonewall Principal of Middletown Elementary 15 years PhD in educational adminis~ration State Board of Education Committee for Improving Elementary School Curriculum 1982-present

3 Jim Henderson School Administrator 22 yearsgradesK-9-supports innovation in education Fair Bargaining Award 1981 former coach for winning collegiate basketball team 10 years

It is clear from such a prompt that the problem the test-takers must address is complex that there is sufficient information here for writing an essay and that testshytakers will be reasonably challenged to write a clear statement of opinion What also emerges from this prompt (and virtually any prompt that one might propose) is the potential cultural effect on the numerous international students who must take the GIIT Is it possible that such students who are not familiar with school systems in the United States with hiring procedures and perhaps with the politics of school board elections might be at a disadvantage in mounting their arguments within a two-hour time frame Some (such as Hosoya 2001) have strongly claimed such a bias

78 CHAPTER 4 Standardized Testing

4 Make appropriate evaluations of different kinds of items

In Chapter 3 the concepts of item facility (IF) item discrimination (ID) and disshytractor analysis were introduced As the discussion there showed such calculations provide useful infornlation for classroom tests but sometimes the time and effort involved in perfornling them may not be practical especially if the classroom-based test is a one-time test Yet for a standardized multiple-choice test that is designed to be marketed commercially andor administered a number of times andor adminisshytered in a different form these indices are a must

For other types of response formats namely production responses different forms of evaluation become importantThe principles of p-mpoundti~ality ~d poundabWty are prominent along with the concept o(JacjJjt Practicality issues in such items include the clarity of directions timing of the test ease of administration and how much time is required to score responses Reliability is a major player in instances where more than one scorer is employed and to a lesser extent when a single scorer has to evaluate tests over long spans of time that could lead to deterioration of stanshydards Facility is also a key to the validity and success of an item type ~irecshytions complex- language obscure topics fuz~Qata and culturally biased

~Jfiformatioifma~alliead to a highei1eVermiddotof diffiCidty than one desires (A) The IF ID and efficiency statistics of the multiple-choice items of current

forms of the TOEFL are not publicly available information For reasons of security and protection of patented copyrighted materials they must remain behind the closed doors of the ETS development staff Those statistics remain of paramount importance in the ongoing production ofTOEFL items and forms and are the founshydation stones for demonstrating the equatability of forms Statistical indices on retired forms of the TOEFL are available on request for research purposes

The essay portion of theTOEFL undergoes scrutiny for its practicality reliability and facility Special attention is given to reliabilIty since two human scorers must read each essay and every time a third reader becomes necessary (when the two readers disagree by more than one point) it costs ETS more money

(B) In the case of the open-ended responses on the two written tasks on the ESLPT a similar set of judgments must be made Some evaluative impressions of the effectiveness of prompts and passages are gained from informal student and scorer feedback In the developmental stage of the newly revised ESLPT both types of feedshyback were formally solicited through questiQnnaires and interviews That informashytion proved to be invaluable in the revisIon of prompts and stimulus reading passages After each administration now the teacher-scorers provide informal feedshyback on their perceptions of the effectiveness of the prompts and readings

The multiple-choice editing passage showed the value of statistical findings in determining the usefulness of items and pointing administrators toward revisions Following is a sample of the format used

CHAPTER 4- Standardized Testing 79

Multiple-choice editing passage

(1)EYer since supermarkets first appeared they have beentake over ~ world ABC 0

(2) Supermarkets have changed peoples life ~ yet and at the same time changes in ABC

peoples life ~ have encouraged the opening of supermarkets o

The task was to locate the error in each sentence Statistical tests on the experishymental version of this section revealed that a number of the 45 items were found to be of zero IF (no difficulty whatsoever) and of inconsequential discrimination power (some IDs of 15 and lower) Many distractors were of no consequence because they lured no one Such information led to a revision of numerous it~ms and their options eventually strengthening the effectiveness of this section

(C)The GET like its written counterparts in the ESLPT is a test ofwritten ability with a single prompt and therefore questions of practicality and J~~illy~are also largely observational No data are collected from students on their perceptions but the scorers have an opportunity to reflect on the validity ofa given topiC After one sitting a topic is retired which eliininates the possibility of improving a specific topiC but future framing of topics might benefit from scorers evaluations Inter-rater reliability is checked periodically and reader training sessions are modified if too many instances of unreliability appear

5 Specify scoring procedures and reporting formats - ---

A systematic assembly of test items in pre-selected arrangements and sequences all of which are validated to confo~ to an e~pected difficulty level should yield a test that can then be scored accurately and reported back to test-takers and institutions efficiently

(A) Of the three tests being exemplifled here the most straightforward scoring procedure comes from the TO~FL the one with the most complex issues of validashytion deSign and assembly Scores are calculated and reported fora) three sections of the TOEFL (the essay ratings are combined with the Structure and Written Expression score) and (b) a total score (range 40 to 300 on the computer-based TOEFL and 310 to 677 on the paper-and-pencil TOEFL) A separate score (c) for the Essay (range 0 to 6) is also provided on the examinees score record (see simulation of a score record on page 80)

80 CHAPTER 4 Standardized Testing

Facsimile of a TOEFLreg score report

TOEFL Scaled Scores Claudia Y Estudiante Peru ___

19 17 17 177 Listening Structure Writing Reading Total Score

Essay rati ng 30

The rating scale for the essay is virtually the same one that is used for the Test of Written English (see Chapter 9 for details) with a zero level added for no response copying the topic only writing completely off topic or not writing in English

(B) The ESLPT reports a score for each of themiddot essay sections but the rating scale differs between them because in one case the objective is to write a summary and in the other to write a response to a reading ~ch essayi~pd lgtY ~o readet~ ifhFfF js a discrepancy of more than one level a third reader1resolves the differenceThe ~ditiilg section is machine-scanned and -scored with a total score and ~th part-scores for each ofthe grammaticaVrhetorlcal sectionS From these data placement administrators have adequate information to make placements and teachers receive some diagnostic inforshymation on each student in their classes Students do not receive their essays back

(C) Each GET is read by two trained readers who give a score between 1 and 4 according to the following scale

Graduate Essay Test Scoring Guide

Please make no marks on the writers work Write your reader number and score on the front cover of each test booklet

4 Superior The opening establishes context purpose and point of view the body of the essay developsmiddot recommendations-logically and coherently The writer demonshystrates awareness of the complexities in the situation and provides analysis of the probJem offers compelling or common-sense reasons for recommendations made makes underlying assumptions explicit

The writer uses fluent and idiomatic English with few mechanical errors Style reshyveals syntactic maturity is dear and direct is not choppy or over-colloquial nor over-formal stuffy or unfocused Occasional spelling or punctuation errors may be easily attributed to hasty transcription under pressure

3 Competent After an opening that establishes context and purpose the paper unfolds with few lapses in coherence but may have somewhat less clear organization of less explicit transitions than a top-score paper It may have somewhat less compelling logic or slightly less-wellreasoned suggestions than a 4 paper though it will provide reasons for the recommendations made

The writer uses dear fluent and generally idiomatic English but may make minor or infrequent ESL errors (preposition errors dropped articles or verb endings etc) or repeat a single error (eg not punctuate possessive nouns) Occasional lapses of style are offSet by demonstrated mastery of syntax

CHAPTE84 Standardized Testing 81

2 Weak The writer makes somewhat simplistic suggestions not fully supported with reashysons fails to cite key facts offers little analysis of the problem or shows a limited grasp of the situation the given information is copied or listed withlittle integration into argument Points may be random or repetitious Writing may be badly focused with careless use of abstract language resulting in predication errors or illogical sentences

ESL andlor careless mechanical errors are frequent enough to be distracting OR sentences may be choppy style over-casual usage occasionally unidiomatic

1 Inadequate The essay may be disjointed incoherent or minimally developed The writer shows little grasp of the complex issues involved is unable to establish conshytext point of view or purpose in opening of paper or has a poor sense of audience Mechanical andor ESL errors or unidiomatic usages are frequent sentences may be ungrammatical OR correct but short and very simple

The two readers scores are added to yield a total possible score of 2 to 8 Test administrators recommend a score of 6 as the threshold for allowing a student to pursue graduate-level courses Anything below that is accompanied by a recomshymendation that the student either repeat the test or take a remedial course in gradshyuate writing offered in one of several different departments Students receive neither their essays nor any feedback other than the fmal score

6 Perform ongoing construct validation studies

From the above discussion it should be clear that no standardized instrument is expected to be used repeatedly without a ramporou~program of ongoing c~-sectmct valiltiatiOll Any standardized test once developed must be accompanied by sysshy~

tematic periodic corroboration of its effectiveness and by steps toward its improveshyment This rigor is especially true of tests that are produced in equated forms that is forms must be reliable across tests such that a score on a subsequent form of a test-has-the~same validityand-interpretability as its original

(A) The TOEFL program in cooperation with other tests produced by ETS has an impressive program of research Over the years dozens of TOEFL-sponsored research studies have appeared in the TOEFL Monograph Series An early example ofsuch a study was the seminal Duran et aI (1985) study TOEFLfrom a Communicative ViewpOint on Language Proficiency which examined the content characteristics of the TOEFL from a communicative perspective based on current research in applied linguistics and lanshyguage proficiency assessment More recent studies (such as Ginther 2001 Leacock amp Chodorow 2001 Powers et aI 2002) demonstrate an impressive array of scrutiny

(B) For approximately 20 years the ESLPT appeared to be placing students relishyably by means of an essay and a multiple-choice grammar and vocabulary test Over the years the security of the latter became s1lspect and the faculty administrators wished to see some content validity achieved in the process In the year 2000 that process began with a group of graduate students (Imao et aI 2000) in consl1ltation with faculty members and continued to fruition in the form of a new ESLPT reported in lmao (2002) The development of the new ESlPT involved a lengthy process of

82 CHAPTER 4 Standardized Testing

both content and construct validation along with facing such practical issues as scoring the written sections and a machine scorable multiple-choice answer sheet

The process of ongoing validation will no doubt continue as new forms of the editing section are created and as new prompts and reading passages are created for the writing section Such a validation process should also include consistent checks on placement accuracy and on face validity

(C) At this time there is little or no research to validate the GET itself For its conshy struct validation its administrators rely on a stockpile of research on university-level academic writing tests such as theTWEThe holistic scoring rubric and the topics and administrative conditions of the GET are to some extent patterned after that of the TWE In recent years some criticism of the GEf has come from international test-takers (Hosoya 2001) who posit that the topics and time limits of the GET among other facshytors work to the disadvantage of writers whose native language is not English These validity issues remain to be fully addressed in a comprehensive research study

I I

STANDARDIZED IANGUAGE PROFICIENCY TESTING

Tests of language profiCiency presuppose a comprehensive definition of the specific competencies that comprise overall language ability The specifications for the TOEFL provided an illustration of an operational definition of ability for assessment purposes This is not the only way to conceptualize the concept Swain (1990) offered a multidimensional view of profiCiency assessment by referring to three linshyguistic traits (grammar discourse and sociolinguistics) that can be assessed by means of oral multiple-choice and written responses (see Table 41) Swains conshyception was not meant to be an exhaustive analysis of ability but rather to serve as an operational framework for constructing proficiency assessments

Another defmition and conceptualization of profiCiency is suggested by the ACTFL association mentioned earlier ACfFL takes a holistic and more unitary view of proficiency in describing four levels superior advanced intermediate and noviceWithin each level descriptions of listening speaking reading and writing are provided as guidelines for assessment For example the ACfFL Guidelines describe the superior level of speaking as follows

ACTFL speaking guidelines summary superior-level

Superior-level speakers are characterized by the ability to

bull participate fully and effectively in conversations in formal and informal settings on topics related to practical needs and areas of professional andor scholarly interests

bull provide a structured argument to explain and defend opinions and develop effective hypotheses within extended discourse

bull discuss topics concretely and abstractly bull deal with a linguistically unfamiliar situation bull maintain a high degree of linguistic accuracy bull satisfy the linguistic demands of professional andor scholarly life

CHAPTER4 Standardized Testing 83

The other three ACfFL levels use the same parameters in describing progressively lower proficiencies across all four skills Such taxonomie~ have the advantage of considering a number of functions of linguistic discourse but the disadvantage at the lower levels of overly emphasizing test-takers deficiencies

Table 41 Traits of second language proficiency (Swain 1990 p 403)

Trait Grammar Discourse Sociolinguistic

focus on grammatical focus on textual focus on social accuracy within cohesion and appropriateness of sentences coherence language use

Method

Oral structured interview story telling and argumentationpersuasion

role-play ofspeech acts requests offers complaints

scored for accuracy of verbal morphology prepositions syntax

detailed rating for identification logical sequence and time orientation and global ratings for coherence

scored for ability to distinguish formal and informal register

Multiple-choice

sentence-level select the correct form exercise

paragraph-level select the coherent sentence exercise

speech act-Ievelselect the appropriate utterance exercise

(45 items) (29 items) (28 items)

involving verb morphology prepositionsan-d-uther items

Written composition

narrative and letter of persuasion

narrative and letter of persuasion

formal request letter and informal note

scored for accuracy of verb morphology prepositions syntax

detailed ratings much as for oral discourse and global rating for coherence

scored for the ability to distinguish formal and inforJ1lil1 register

FOUR STANDARDIZED lANGUAGE PROFICIENCY TESTS

We now tum to some of the better-known standardized tests of overall language ability or profiCiency to examine some of the typical formats used in commercially available tests We will not look at standardized tests of other specific skills here but that should not lead you to think by any means that proficiency is the only kind of test in the field that is standardized Three standardized oral production tests the

84 CHAPTER 4 Standardized Testing

Test of Spoken English (fSE) the Oral Proficiency Inventory (OPI) and PbonePassreg are discussed in Chapter 7 and the Test of Written English (WE) is covered in ChapterS

Four commercially produced standardized tests of English language proficiency are described briefly in this section the TOEFL the Michigan English Language Assessment Battery (MELAB) the International English Language Testing System (lELTS) and the Test of English for International Communication (fOEICreg) In an appendix to this chapter are sample items from each section of each test When you turn to that appendix use the following questions to help you evaluate these four tests and their subsections

1 What item types are included 2 How practical and reliable does each subsection of each test appear to be 3 Do the item types and tasks appropriately represent a conceptualizatio~ of

language proficiency (ability) That is can you evaluate their construct validity

4 Do the tasks achieve face validity 5 Are the tasks authentic 6 Is there some washback potential in the tasks

Test of English as a Foreign Language (TOEFL)

Producer Educational Testing Service (ETS) Objective To test overall proficiency (language ability) Primary market Almost exclusively US universities and colleges for admission

purposes Type Computer-based (CB) (and two sections are-computer-adaptive)

A traditional paper-based (PB) version is also available Response modes Multiple-choice responses essay Specifications See the box on pp 72-73 Time allocation Up to 4 hours (CB) 3 hours (PB) Internet access wwwtoeflorg

Comments In the North American context the TOEFL is the most widely used comshymercially available standardized test of proficiency Each year the TOEFL test is adminisshytered to approximately 800000 candidates in more than 200 countries It is highly respected because of the thorough program of ongoing research and development conshyducted by ETS The TOEFLs primary use is to set proficiency standards for international students seeking admission to English-speaking universities More than 4200 academic institutions government agencies scholarship programs and licensingcertification agenshycies in more than 80 countries use TOEFL scores By 2004 the TOEFL will include a secshytion on oral production

CHAPTER 4 Standardi~ed Testing 85

Michigan English Language Assessment Battery (MELAB)

Producer English language Institute University of Michigan Objective To test overall proficiency (language ability) Primary market Mostly US and Canadian language programs and colleges

some worldwide educational settings as well Type Paper-based Response modes Multiple-choice responses essay Time allocation 25 to 35 hours Internet access wwwlsaumicheduelimelabhtm

Specifications The MElAB consists of three sections Part 1 a 3D-minute impromptu essay is written on an assigned topic Part 2 a 25-minute multiple-choice listening comshyprehension test is delivered via tape recorder Part 3 is a 100-item 75-minute multipleshychoice test containing grammar doze reading vocabulary and reading comprehension An oral interview (speaking test) is optional

Comments The Ell at the University of Michigan has been producing the MELAB and its earlier incarnation (Michigan Test of English language Proficiency) since 1961 like the TOEFL it serves a North American audience but is also used internationally While its use is not as widespread as the TOEFL its validity is widely respected Because it is cheaper than the TOEFL and more easily obtained it is popular among language schools and institutes Many institutions and companies accept MElAB scores in lieu ofTOEFL scores

International English Language Testing System (IELTS)

Producer Jointly managed by The University of Cambridge local Examinations Syndicate (UClES) The British Council and lOP Education Australia

Objective To test overall proficiency (language ability) Primary-market Australian British Canadian and New Zealand academic

institutions and professional organizations American academic institutions are increasingly accepting IELTS for admissions purposes

1)rpe Computer-based (for the Reading and Writing sections) papershybased for the listening and Speaking modules

Response modes Multiple-choice responses essay oral production Time allocation 2 hours 45 minutes Internet access httpwwwieltsorgl

httpwwwudesorguk httpwwwbritishcouncilorg

Specifications Reading candidates choose between academic reading or general training reading (60 minutes) Writing the same option academic writing or general training writing (60 minutes) Listening four sections for all candidates (30 minutes) Speaking five sections for all candidates (1015 minutes)

86 CHAPTER 4 Standardized Testing

Comments The University of Cambridge local Examinations Syndicate (UCLES) has been producing English language tests since 1858 Now with three organizations cooperatshying to form the IELTS more than a million examinations are administered every year In 2002 a computer-based version of the Reading and Writing modules of the IELTS became available at selected centers around the world The other sections are administered locally by an examinet The paper-based IELTS remains an option for candidates The IELTS retains the distinct advantage of requiring all four skills in the test-takers performance

Test of English for International Communication (TOEICreg)

Producer The Chauncey Group International a subsidiary of Educational Testing Service

Objelttive To test overall proficiency (langlJage ability) Primary market Worldwide business commerce and industry contexts

(workplace settings) Type Computer-based and paper-based versions Response modes Multiple-choice responses Time allocation 2 hours Internet access httpwwwtoeiccom

Specifications Listening Comprehension 100 items administered by audiocassette Four types of task statements questions short conversations and short talks (approxishymately 45 minutes) Reading 100 items Three types of task cloze sentences error recogshynition and reading comprehension (75 minutes)

Comments The TOEIC has become a very widely used international test of English proficiency in workplace settings where English is required for job performance The conshytent includes many different employment settings such as conferences presentations sales ordering shipping schedules reservations (etters and memoranda It is approprishyate to use in educational settings where vocational or workplace English courses are being offered

sect sect sect sect sect

The construction of a valid standardized test is no minor accomplishment whether the instrument is large- or small-scale The designing of specifications alone as this chapter illustrates requires a sophisticated process of construct valishydation coupled with considerations of practicality Then the construction of items and scoringinterpretation procedures may require a lengthy period of trial and error with prototypes of the final form of the testWith painstaking attention to all the details of construction the end product can result in a cost-effective timeshysaving accurate instrument Your use of the results of such assessments can provide useful data on learners language abilities But your caution is warranted as well for all the reasons discussed in this chapter The next chapter will elaborate on what lies behind that need for a cautious approach to standardized assessment

CHAPTER4 Standardized Testing 87

EXERCISES

[Note (I) Individual work (G) Group or pair work (C) Whole-class discussion]

1 (C) Tell the class about the worst test experience youve ever had Briefly anamiddot lyze what made the experience so unbearable and try to come up with sugshygestions for improvement of the test andor its administrative conditions

2 (G) In pairs or small groups compile a brief list of pros and cons of standardshyized testing Cite illustrations of as many items in each list as possible Report your lists and examples to the rest o~ the class

3 (I) Select a standardized test that you are quite familiar with (probably a recent experience) Mentally evaluate that test using the five principles of practicality reliability validity authenticity and washback Report yourevaluashytion to the class

4 (G) The appendix to this chapter provides sample items from Jour different tests of language proficiency In groups one test for each group analyze your test for (a) content validity (b) face validity and (c) authenticity

5 (C) Do you think that the sample TOEFL reading passage about pirates (pages 74-75) and the Graduate EssayTest prompt (pages 76-77) about a school board hiring committee have any culture bias Discuss this and other cultural biases you have noticed in tests Is it possible to design a test that is completely free of culture bias

6 (CG) Compare the differences in conceptualization of language proficiency represented by Swains model the TOEFL and the ACfFL philosophy Which one best represents current thinking about communicative language ability What are the strengths and weaknesses of each approach

FORYOlIILEURTHER READING

Gronlund Norman E (1998) Assessment of student achievement Sixth Edition Boston Allyn and Bacon

Gronlunds classic also mentioned in Chapter 3 offers a concise overview of features of standardized tests offering definitions and examples of the statistical considerations in interpreting scores His approach is unbiased cleady written and accessible to those who might fear the mathematics of standardized testing

Phillips Deborah 2001 Long1nan introductory course for the TOEFL test White Plains NY Pearson Education

A careful examination of this or any other reputable preparation course for a standardized language test is well worth a students time Note especially how the book acquaints the user with the specifications of the test and offers a number of useful strategie~ that can be llsed in preparation for the test and during irs adn1inistration

88 CHAPTER 4 Standardized Testing

APPENDIX TO CHAPTER 4

Commercial Proficiency Tests Sample Items and Tasks

Test of English a~ a Foreign Language (TOEFLreg)

Listening r

Part A

In this section you will hear short conversations between two people In some ofthe conversations each person speaks only once In other conversations one or both of the people speak more than once Each conversation is followed by one questionabQlt it Each question in this part has four answer choices You should click on the best answer to each question Answer the questions on the basis of what is stated or implied by the speakers Here is an example On the computerscreen you will see

[man and woman talking]

On the recording you will hear

(woman) Hey wheres your sociology book (man) At home Why carry it around when were just going to be taking

a test (woman) Dont you remember Professor Smith said we could us it during

the test (man) Ohl no Well Ive still got an hour right Im so glad I ran into you

You wiII then see and hear the question before the answer choices appear

What will the man probably do next

o Begin studying for the sociology test o Explain the problem to his professor o Go home to get his textbook o Borrow the womans book

To choose an answer you will click on an oval The oval next to that answer will darken After you click on Next and Confirm Answer the next conversation will be presented

Part B

In this section you will hear several longer conversations and talks Each conversation or talk is followed by several questions The conversations talks and questions will not be repeated The conversations and talks are about a variety of topics You do not need speshycial knowledge of the topics to answer the questions correctly Rather you should answer each question on the basis of what is stated or implied by the speakers in the conversashytions or talks

For most of the questions you will need to click on the best of four possible answers Some questions will have special directions The special directions will appear in a box on the computer screen Here is an exampie ot a conversation and some questions

CHAPTER 4 Standardized Testing 89

Marine Biology (narrator) Listen to part of a discussion in a marine biology class

(professor) A few years ago our local government passed a number of strict environmental laws As a result Sunrise Beach looks nothing Ii ke it did ten years ago The water is cleaner and theres been a tremendous increase in all kinds of marine life which is why were going there on Thursday

(woman) I dont know if I agree that the water quality has improved I mean I was out there last weekend and it looked all brown It didnt seem too clean to me

(professor) Actually the color of the water doesnt always indicate whether its polluted The brown color you mentioned might be a result of pollution or it can mean a kind of brown algae is growing there Its called devils apron and it actually serves as food for whales

(man) So when does the water look blue (professor) Well water thats completely unpolluted is actually colorless But

it often looks bluish-green because the sunlight can penetrate deep down and thats the color thats reflected

(woman) But sometimes it looks really green Whats that about (professor) Ok well its the same principle as with devils apron the

water might be green because of different types of green algae there-gulfweed phytoplankton You all should finish reading about algae and plankton before we go In fact those are the types of living things Im going to ask you to be looking for when were there

Now get ready to answer the questions

What is the discussion mainly about

o The importance of protecting ocean environments o The reasons why ocean water appears to be different colors o The survival of whales in polluted water o The effect that colored ocean water has on algae

To choose an answer click on an oval The oval next to that answer will darken After you click on Next and Confirm Answer the next question will be presented

According to the professor what can make ocean water look browngt

o Pollution o Cloudy Skies o Sand o Algae

Click on 2 answers

To choose your answers you will click on the squares An XII wiii appear in each square

bullbullbullbullbullbullbull

90 CHAPTER 4 Standardized Testing

Structure and Written Expression This section measures the ability to recognize language that is appropriate for standard written English There are two types ofquestions in this section In the first type ofquestion there are incomplete sentences Beneath each sentence there are four words or phrases

Directions CIiSk on the one word or phrase that best completes the sentence

The colum~ine flower __ to nearly all of the United States can be raised from seed in almost any garden

native how native is how native is it is native

Time Help Confirm

After you click on Next and Confirm Answ~ the next question willbe presented

The second type of question has four underlined words or phrases You will choose the one underlined word or phrase that must be changed for the sentence to be correct

Directions Click on the one underlined word or phrase that must be changed for the senshytence to be correct

One of the most difficult problems in understanding sleep is determining what the funcshytions of sleep ~

lime Help Confirm

Clicking on an underlined word or phrase will darken it

Reading This section measures the ability to read and understand short passages similar in topic and style to those that students are likely to encounter in North American universities and colleges This section contains reading passages and questions about the passages There are several different types of questions in this section In the Reading section you will first have the opportunity to read the passage

The temperature of the Sun is over 10000 degrees Fahrenheit at the surface but it rises perhaps more than 270000000 at the center The Sun is so much hotter than the Earth that matter can exist only as a gasl except perhaps at the core In the core of the Sun the pressures are so great that despite the high temperature there may be a small solid core However no one really knows since the center of the Sun can never be directly observed ~ Solar astronomers do know that the Sun is divided into five general layers or zones Starting at the outside and going down into the Sun the zones are the corona chromoshysphere hotosphere convection zone and finally the core The first three zones are reshygarded as the Suns atmosphere But since the Sun has no solid surface it is hard to middottell where the atmosphere ends and the main body of the Sun begins

The Suns outermost layer begins about 10000 miles above the visible surface and goes outward for millions of miles This is the only part of the Sun that can be seen during an eclipse such as the one in February 1979 At any other time the corona can be seen

bullbullbullbullbullbullbull

bull bullbullbullbullbullbull

CHAPTER 4 Standardized Testing 91

only when special instruments are used on cameras and telescopes to block the light from the photosphere

The corona is a brilliant pearly white filmy light about as bright as the full Moon Its beautiful rays are a sensational sight during an eclipse The coronas rays flash out in a brilliant fan that has wispy spikelike rays near the Suns north and south poles The corona is generally thickest at the Suns equator The corona is made up of gases streamshying outward at tremendous speeds that reach a temperature of more than 2 million deshygrees Fahrenheit The gas thins out as it reaches the space around the planets By the time the gas of the corona reaches the Earth it has a relatively low density

When you have finished reading the passage you will use the mouse to click on Proceed Then the questions about the passage will be presented You are to choose the one best anshyswer to each question Answer all questions about the information in a passage on the basis ofwhat is stated or implied in that passage Most ofthe questions will be multiple-choice questions To answer these questions you will click on a choice below the question

With what topic is paragraph 2 mainly concerned

o How the Sun evolved o The structure of the Sun o Why scientists study the Sun o The distaflce of the Sun from the planets

Paragraph 2 is marked with an arrow (~)

You will see the next question after you click on Next

To answer some questions you will click on a word or phrase Here is an example

Look at the word one in the passage Click on the word or phrase in the bold text that one refers to To answer you can click on any part of the word or phrase in the passage Jour choice will darken to show which word you have chosen

The Suns outermost layer begins about 10000 miles above the visible surface and goes outward for millions of miles This is the only part of the Sun that can be seen durshying an eclipse such as the one in February 1979 At any other time the corona can be seen only when special instruments are used on cameras and telescopes to block the Iight from the photosphere

You will see the next question after you click on~ To answer some q~estions you will click on a sentence in the passage Here is an example

~ The corona is a brilliant pearly white filmy light about as bright as the full Moon Its beautiful rays are a sensational sight during an eclipse The coronas rays flash out in a brilliant fan that has wispy spikelike rays near the Suns north and south poles The corona is generally thickest at the Suns equator ~ The corona is made up of gases streaming outward at tremendous speeds that reach a temperature of more than 2 million degrees Fahrenheit The gas thins out as it reaches the space around the planets By the time the gas of the corona reaches the Earth it has a relatively low density

bull bullbullbullbullbullbull

92 CHAPTER 4 Standardized Testing

Click on the sentence in paragraph 4 or 5 in which the author compares the light of the Suns outermost layer to that ofanother astronomical body Paragraphs 4 and 5 are marked with arrows (~)

To answer some questions you will click on a square to add a sentence to the passage Here is an example -The following sentence can be added to paragraph 1

At the center of the Earths solar system lies the Sun

Where would it best fit in paragraph I Click on the square to add the sentence to the paragraph

D The temperature of the Sun is over 10000 degrees Fahrenheit at the surface but it rises to perhaps morethan 27000000deg at the center 0 The Sun is so much hotter than the Earth that matter can exist only as a gasi except p~rHapsatth~ c6relp the c~re of the ii Sun the pressures are so great that despite the high temperature there may be a small solid core D However no one really knows since the center of the Sun can never be directly observed D 0100

When you click on a square the sentence will appear in the passage at the place you have chosen You can read the sentence added to the paragraph to see if this is the best place to add it You can click on another square to change your answer The sentence will be added and shown in a dark box

Writing In this section you will have an opportunity to demonstrate your ability to write in Enshyglish This includes the ability to generate and organize ideas to support those ideas with examples or evidence and to compose in standard written English in response to an asshysigned topic You will have 30 minutes to write your essay on that topic You must write on the topic you are assigned An essay on any other topic will receive a score of 0 Read the topic below and then make any notes that will help you plan your response Begin typing your response in the box at the bottom of the screen or write your answer on the answer sheet provided to you

Following is a sample topic

Do you agree or disagree with the following statemenH

Teachers should make learning enjoyable and fun for their students

Use specific reasons and examples to support your opinion

CHAPTER 4 Standarczed Testing 93

Michigan English Language Assessment Battery (MELAB)

Composition The time limit for the composition is 30 minutes You must write on only one of the top~

ics below If you write about something else your composition paper will not be graded and you cannot be given a final score If you do not understand the topics ask the exam~ iner to explain or to translate them You may be asked to give your opinion ofsomething and explain why you believe this to describe something from your experience or to exshyplain a problem and offer possible solutions You should write at least one page Some sample topics are

1 What do you think is your countrys greatest problem Explain in detail and tell what you think can be done about it

2 What are the characteristics of a good teacher Explain and give examples 3 An optimist is someone who sees the good side of things A pessimist sees the

bad side Are you an optimist or a pessimist Relate a personal experience that shows this

4 In your opinion are the benefits of space exploration really worth the enormous costs Discuss

Most MELAB compositions are one or two pages long (about 200-300 words) If your paper is extremely short (less than 150 words) your composition will be given a lower score Before you begin writing you might want to take 2 or 3 minutes to plan your comshyposition and to make a short outline to organize your thoughts Such outlines will not be graded they are only to help you You should use the last 5 minutes to read through your composition and to make changes or corrections

Your composition will be graded on how clearly you express yourself in English and on the range of English you are able to use and your control in doing so This means your composition should be well organized your arguments should be fully developed and you should show a range ofgrammatical structures and broad vocabulary Compositions that consist only of very short sentences and very simple vocabulary cannot be given the

middothighest scores If errors are not frequent and if they do not confuse your meaning they will not lower your score very much

Listening Now you will hear a short lecture You may take notes during the lecture Following the lecture you will be asked some questions about it

Therell be a two-week exhibit of the paintings of the little-known master Laura Bernhart at the Claire Osmond Galleries starting on the fifteenth of the month and running through the thirtieth Bernharts known for her innovative designs in abstract expressionism Though a true original she declared a spiritual heritage from Salvador Dali the famous Spanish painter Since Bernhart lived a rather solitary life and died while only in her twenties few people are aware of her works This showing at the Osmond Galleries will provide many with an introduction to her works

10 Where is the exhibit a the Art Museum b the Dali Galleries c the Osmond Galleries

94 CHAPTER 4 Standardized Testing

11 What is Bernhart known for a her copies of Dalis paintings b the originality of her designs c her exhibitions

12 What will going to the exhibit allow most people to do a to see Saivador Dalis paintings b to see Bernharts works for the first time c to learn about Spanish art

Grammar

1 What did the teacher just tell you

She reminded our notebooksI a us to bring b that we bring c our bringing d we should bring

2 Is Bill a good dancer

Not really __ he tries very hard a in spite of h despite c even though d while

3 your clothes are all wet1

Yes I didnt come __ the rain soon enough a away to b over to c down with d in from

Cloze In years to come zoos will not only be places where animals are exhibited to the public but repositories where rare species can be saved from extinction (7) captive breeding The most powerful force (8) the future of many animals-and of zoos-is the decline of the wild (9) even zoo directors would argue that (10) are better places for animals than the fields and forest of their native (11) yet zoos may be the last chance for some creatures that would otherwise pass qUietly into oblivion

7 a through c from b of d damage

8 a bringing c to b that d influencing

9 a But c Not b So d Then

10 a where c even b zoos d wilds

11 alands c residence b life d field

CHAPTER 4 Standardized Testing 95

Vocabulary

12 Mark has a flair for writing a need b purpose c talent d dislike

13 Bill Collins launched his restaurant last June a moved b started c sold d bought

14 John will not accept the censure a burden b blame c credit d decision

15 I cant think of the answer Can you give me a __ a hint b token c taste d gaze

16 Because fewer people are taking expensive vacations the tourist industry is in a a choke b grope c grumble d slump

17 I disagree with a few of his opinions but __ we agree a deliberately b conclusively c essentially d immensely

Reading The influenza virus is a single molecule built from many millions of single atoms You must have heard of the viruses which are sometimes called living molecules While bacteria can be considered as a type of plant secreting pOisonous substances into the body of the organism they attack viruses are living organisms themselves We may conshysider them as regular chemical molecules since they have a strictly aefined atomic strucshyture but on the other hand we must also consider them as being alive since they are able to multiply in unlimited quantities

18 According to the passage bacteria are a poisons

b larger than viruses c very small d plants

96 CHAPTER 4 Standardized Testing

19 The writer says that viruses are alive because they a have a complex atomic structure b move c multiply d need warmth and light

20 The atomic structure of viruses a is -tJIariable b is strictly defined c cannot be analyzed chemically d is more complex than that of bacteria

International English Language Testing System (fELTS)

I

listening

The Listening Module has four sections The first two sections are concerned with social needs There is a conversation between two speakers and then a monologue For examshyple a conversation about travel arrangements or decisions on a night out and a speech about student services on a university campus or arrangements for meals during a confershyence The final two sections are concerned with situations related more closely to educashytional or training contexts For example conversation between a tutor and a student about an assignment or between three students planning a research project and a lecture or talk ofgeneral academic interest All the topics are ofgeneral interest and it makes no difference what subjects candidates study Tests and tasks become more difficult as the sections progress A range of English accents and dialects are used in the recording which reflects the international usage of IELTS

Academic Reading [A 7S0-word article on-th-e- topic of Wind Power in the US with a short glossary at the end]

Questions 1-5

Complete the summary below

Choose your answers from the box below the summary and write them in boxes 1-5 on your answer sheet Note There are more words or phrases than you will need to fill the gaps You may use any word or phrase more than once

Example The failure during the late 1970s and early 19805 of an attempt to establish a widespread wind power industry in the United States resulted largely from the (1) bull in oil prices during this period The industry is now experiencing a steady (2) due to improveshyments in technology and an increased awareness of the potential in the power of wind The wind turbines that are now being made based in part on the (3) of wide- ranging research in Europe are easier to manufacture and maintain than their predecesshysors This has led wind-turbine makers to be able to standardise and thus minimize (4) There has been growing (S) of the importance of wind power as an energy source

CHAPTER 4 Standardized Testing 97

criticism stability skepticism success operating costs decisions design costs fall effects production costs growth decline failure recognition results

Questions 6-1 0 Look at the following list of issues (Questions 6-10) and implications (A-C) Match each issue with one implication Write the appropriate letters A-C in boxes 6-10 on your anshyswer sheet

Example The current price of one wind-generated kilowatt Answer

6 The recent installation of systems taking advantage of economies of scale

7 The potential of meeting one fifth of current U5 energy requirements by wind power

8 The level of acceptance of current wind turbine technology

9 A comparison of costs between conventional and wind power sources

10 The view of wind power in the European Union

Implications

A provides evidence against claims that electricity produced from wind power is relatively expensive

B supports claims that wind power js an important source of energy

C opposes the view that wind power technology requires further-development

General Training Reading Read the passage on Daybreak trips by coach and look at the statements below On your answer sheet write

TRUE if the statement is true FALSE jf the statement is false

NOlGIVEN if the information is not given in the leaflet

1 MiIlers Coaches owns Cambridges Cam bus fleet

2 Premier is an older company than Millers

3 Most of the Daybreak coaches are less than 5 years old

4 Daybreak fares are more expensive than most of their competitors

5 Soft drinks and refreshments are served on most longer journeys

6 Smoking is permitted at the rear of the coach on longer journeys

7 Tickets must be bought in advance from an authorised Daybreak agent

6 Tickets and seats can be reserved by phoning the Daybreak Hotline

9 Daybreak passengers must join their coach at Cambridge Drummer Street

10 Daybreak cannot guarantee return times

98 CHAPTER 4 Standardized Testing

FROM CAMBRIDGE AND SURROUNDING AREA

SPRING IS INTHEAIR

Welcome to our Spring Daybreak programme which continues the tradition of offering unbeatable value for money day trips and tours All the excursions in this brochure will be operated by Pr~mier Travel Services Limited or Millers Coaches both companies are part of the CHLGroup owners of Cambridges Cambus fleet

WERE PROUD OF OUR TRADITION

Premier was established in 1936 the Company now offers the highest standards of coaching in todays competitive operating environment Miller has an enviable reputation stretching back over the past 20 years offering coach services at realistic prices Weve traveled a long way since our early days of pre-war seaside trips Now our fleet of 50 modern coaches (few are more than five years old) operate throughout Britain and Europe but were pleased to still maintain the high standards of quality and service the trademark of our founders nearly sixty years ago

EXCLUSIVE FEATURES

Admission-inclusive fares All Daybreak fares (unless specifically otherwise stated) include admission charges to the attractions shows and exhibits we visit Many full-day scenic tours are accompanied by a fully trained English Tourist Board Blue Badge guide or local experienced driverguide Some Daybreaks include lunch or afternoon tea Compare our admission inclusive fares and see how much you save Cheapest is not the best and value for money is guaranteed If you compare our bargain Daybreak fares beware--most of our competishytors do not offer an all-inclusive fare

SEAT RESERVATIONS

We value the freedom of choice so you can choose your seat when you book The seat reservation is guaranteed a-nd remains yours at all times when aboard the coach

NO SMOKING COMFORT

With the comfort of our passengers in mind coaches on all our Daybreaks are no smokshying throughout In the interests of fellow passengers comfort we kindly ask that smokers observe our no smoking policy On scenic tours and longer journeys ample refreshment stops are provided when of course smoking is permitted

YOUR QUESTIONS ANSWERED

Do I need to book Booking in advance is strongly recommended as all Daybreak tours are subject to demand Subject to availability stand-by tickets can be purchased from the driver

What ti me does the coach leave The coach departs from Cambridge Drummer Street (Bay 12 adjacent to public toilets) at the time shown There are many additional joining points indicated by departure codes in the brochure If you are joining at one of our less popular joining points you will be adshyvised of your pick-up time (normally by telephone) not less than 48 hours before deparshyture In this way we can minimize the length of pick-up routes and reduce journey times for the majority of passengers

CHAPTER 4 Standardized Testing 99

What time do we get back An approximate return time is shown for each excursion The tim~s shown serve as a guide but road conditions can sometimes cause delay If your arrival will be later than advertised your driver will try to allow for a telephone call during the return journey

Where can I board the coach All the Daybreaks in the brochure leave from Cambridge Drummer Street (Bay 12 adjashycent to public toilets) at the time shown Many Daybreaks offer additional pick-ups for pre-booked passengers within Cambridge and the surrounding area This facility must be requested at the time of booking

Academic Writing Writing Task 1 You should spend about 20 minutes on this task

The graph below shows the different modes of transport used to travel to and from work in one European city in 1950 1970 and 1990

[graph shown here]

Write a report for a university lecturer describing the information shown below You should write at least 150 words

Writing Task 2 You should spend about 40 minutes on this task

Present a written argument or case to an educated reader with no specialist knowledge of the folowing topic

It is inevitable that as technology develops so traditional cultures must be lost Technolshyogy and tradition are incompatible-you cannot have both together

To what extent do you agree or disagree with this statement Give reasons for your answer You should write at least 250 words You should use your own ideas knowlshyedge and experience and support your arguments with examples and relevant evidence

General Training Writing Writing Task 1 You should spend about 20 minutes on this task You rent a house through an agency The heating system has stopped working You phoned the agency a week ago but it has still not been mended Write a letter to the agency Explain the situation and teil them what you want them to do about it

You should write at least 150 words You do NOT need to write your own address

Begin your letter as follows

Dear - ___-I

Writing Task 2 You should spend about 40 minutes on this task As part ofa class assignment you have to write about the following topic

100 CHAPTER 4 Standardized Testing

Some businesses now say that no one can smoke cigarettes in any of their offices Some governments have banned smoking in all public placesThis is a good idea but it takes away some of our freedom

Do you agree or disagree Give reasons for your answer You should write at least 250 words

Speaking In each ofthe three parts of the speaking module a specific function is fulfilled In Part 1 the candidates answer general questions about themselves their homes or families their jobs or studies their interests and a range ofsimilar familiar topic areas This part lasts between four and five minutes In Part 2 the candidate is given a verbal prompt on a card and is asked to talk on a particular topic The candidate has one minute to prepare before speaking at length for between one and two minutes The examiner then asks one or two wind-down questions In Part 3 the examiner and candidate engage in a discusshysion of more abstract issues and concepts which are thematically linked to the topic prompt in Part 2 The discussion lasts between four and five minutes

All interviews are recorded on audiocassette Here is a sample ofa Part 2 topic

Describe a teacher who has greatly influenced you in your education

You shou Id say

where you met them what subject they taught what was special about them

and explain why this person influenced you so much

You will have to talk about the topic for 1 to 2 minutes You have 1 minute to think about what you are going to say You can make some notes if you wish

Test of English for International Communication (TOEICreg)

listening

Part 1 Photographs Directions For each question you will see a picture in your test book and you will hear four short statements The statements will be spoken just one time They will not be printed in your test book so you must listen carefully to understand what the speaker says When you hear the four statements look at the picture in your test book and choose the statement that best describes what you see in the picture Then on your answer sheet find the number of the question and mark your answer

[photograph of a scientist looking through a microscope]

You will hear Look at the picture marked number 1 in your test book

(A) Shes speaking into a microphone (B) Shes put on her glasses (C) She has both eyes open (D) Shes using a microscope

CHAPTER 4 Standardized Testing 101

Part 2 Question-Response Directions In this part of the test you will hear a question or statement spoken in Enshyglish followed by three responses also spoken in English The question or staten1ent and the responses will be spoken just one time They will not be printed in your test book so you must listen carefully to understand what the speakers say You are to choose the best response to each question or statement

Question 1 You will hear Ms Morikawa has worked here for a long time hasnt she

(A) At three oclock (B) No Ive lost my watch (C) More than ten years

Question 2 You will hear Which of these papers has a wider circulation

(A) The morning edition (B) Get more exercise (C) By messenger

Part 3 Short Conversations Directions In this part of the test you will hear short conversations between two people The conversations will not be printed in your test book You will hear the conversations only once so you must listen carefully to understand what the speakers say In your test book you will read a question about each conversation The question will be followed by four answers You are to choose the best answer to each question and mark it on your answer sheet

Question 1 (Man) We should think about finding another restaurant for lunch (Woman) Why The food and service here are great

(Man) Yes but the prices are going up every week

You will read Why is this man unhappy with the restaurant

(A) It is too noisy (B) It is too expensive (C) It is too crowded (D) It is too difficult to find

Question 2 (Woman A) How was Dr Borgs recent trip to Singapore (Woman B) She enjoyed the tour of the port very much (Woman A) They say its one of the most active in Asia

You will read 2 What did Dr Borg find interesting

(A) The tourist center (B) The airport (C) The musical performance (D) The harbor

Part 4 Short Talks Directions In this part of the test you vill hear several short talks Each will be spoken just one time They will not be printed in your test book so you must listen carefully to understand and remember what is said In your test book you will read two or more questions about each short talk The questions will be followed by four answers You are to choose the best answer to each question and mark it on your answer sheet

102 CHAPTER 4 Standardized Testing

You will hear Questions 1 and 2 refer to the following announcement

Good afternoon and welcome aboard Nordair Flight 857 from Copenhagen to Bangkok with intermediate stops in Dubai and Calcutta We are preparing for departure in a few minutes At this time your seat back should be returned to its full upright position and your seat belt s~ould be fastened OUf anticipated total flying time to Dubai is six hours and twenty-five minutes I hope you enjoy the flight You will hecJr Now read question 1 in your test book and answer it You will read 1 What is the final destination of the flight

(A) Bangkok (B) Copenhagen (C) Dubai (O) Calcutta

You will hear Now read question 2 in your test book and answer it You will read 2 What will happen in a few minutes

(A) The flight will land in Dubai I

(B) The passengers will board the plane (C) The plane will take off (0) The gate number will be announced

Reading In this section of the test you will have the chance to show how well you understand written English There are three parts to this section with special directions for each part

Part 4 Incomplete Sentences Directions This part of the test has incomplete sentences Four words or phrases marked (A) (8) (e) (D) are given beneath each sentence You are to choose the one word or phrase that best completes the sentence Then on your answer sheet find the number of the question and mark your answer

1 Mr Yangs trip will __ him away from the office for ten days (A) withdraw (B) continue (C) retain (0) keep

2 The company that Marie DuBois started now sells __ products throughout the world (A) its (B) it (C) theirs (D) them

3 If your shipment is not delivered __ Tuesday you can request a full refund for the merchandise (A) at (B) by (C) within (D) while

CHAPTER 4 Standardized Testing 103

Part 6 Error Recognition Directions In this part ofthe test each sentence has four words or phrases underlined The four underlined parts of the sentence are marked (A) (B) (C) (D) You are to identify the one underlined word or phrase that should be corrected or rewritten Then on your answer sheet find the number of the question and mark your answer

1 The pamphlet contains some importance information about the current exhibit ABC D

2 No matter how Jong it taking to finish the annual report it must be done properly ABC D

3 The popularity of jogging appears to have decreased since the past couple of years ABC D

Part 7 Reading Comprehension Directions The questions in this part of the test are based on a selection of reading mateshyrials such as notices letters) forms newspaper and magazine articles) and advertisements You are to choose the one best answer (A) (B) (C) or (OJ to each quesshytion Then on your ariswefsheelfindthe number of the qUestion andmcirkyour answer Answer all questions following each reading selection on thebasis of what is stated or implied in that selection

The Museum ofTechnology is a hands-on museum designed for people to experience science at w()rk~ Visitors are encouraged to use test and handle the objects o~ display Special demonstrations are scheduled for the first and second Wednesdays of each month at 1330 Open Tuesday-Friday 1200-1630 Saturday 1000-1730 and Sunday 11 00-1630

1 When during the month can visitors see special demonstrations (A) Every weekend (B) The first two Wednesdays (C) One afternoon a week (D) Every other Wednesday

Questions 2 and 3 refer to the followi ng notice

NOTICE If you are unable to work because of an extended illness or injury that is not workshyrelated you may be entitled to receive weekly benefits from your employer or the firms insurance company To claim benefits you must file a claim form within thirty days of the first day of your disability Before filing the claim you must ask your doctor to fill in the Doctors Statement on the claim form stating the period of disability

3 To whom is this notice addressed (A) Employers (8) Doctors (C) Employees (D) When paying the bill

4 When must the claim form be filed (A) On the first of the month (8) On the thirtieth of the month (C) On the first day ofdisabifity (D) Within 30 days of the start of disability

Page 8: Standardized Testing Chapter 4 Brown

CHAPTER 4 Standardized Testing 73

The reading section consists of four to five passages of 250-350 words with 10-14 questions per passage This section is not computer-adaptive soexaminees can skip questions and return to previous questions The questions in this section assess the comshyprehension of main ideas inferences factual information stated in a passage pronoun referents and vocabulary (direct meaning synonym antonym) In all cases the questions can be answered by reading and understanding the passages This section consists of (1 ) traditional multiple-choice questions (2) questions that require examinees to click on a word phrase sentence or paragraph to answer and (3) questions that ask examinees to insert a sentence where it fits best

Writing Section The writing section measures the ability to write in English including the ability to generate organize and develop ideas to support those ideas with examples or evidence and to compose a response to one assigf)ed topic in standard written Enshyglish Because some examinees may not be accustomed to composing an essay on comshyputer they are given the choice of handwriting or typing the essay in the 30-minute time limit The rating scale for scoring the essay ranging from 0 to 6 is virtually the same as that of the Test of Written English [see Chapter 9 of this book] A score of 0 is given to papers that are blank simply copy the topic are written ina language other than English consist only of random keystroke characters or are written on a topic different from the one assigned

Each essay is rated independently by two trained certified readers Neither reader knows the rating assigned by the other An essay will receive the average of the two ratshyings unless there is a discrepancy of more than one point in that case a third reader will independently rate the essay The essay rating is incorporated into the StructureMriting scaled score~ and constitutes approximately 50 percent of that combined score

(B) The designing of the test specs for the ESLPT was a somewhat simpler task because the purpose is placement and the construct validation of the test consisted of an examination of the content of the ESL courses In fact in a recent revisiofi of the ESLPT (lmao et al 2000 Imao 2001) content validity (coupled with its attenshydant face validity) was the central theoretical issue to be considered The major issue centered on designing practical and reliable tasks and item response formats Having established the importance of designingESLPT tasks that simulated classroom tasks used in the courses the designers ultimately specified two writing production tasks (one a response to an essay that students read and the other a summary of another essay) and one multiple-choice grammar-editing taskThese specifications mirrored the readingbased process writing approach used in the courses

(C) Specifications for the GET arose out of the perceived need to provide a threshold of acceptable writing ability for all prospective graduate students at SFSU both native and non-native speakers of EnglishThe specifications for the GET are the skills of writing grammatically and rhetorically acceptable prose on a topic of some interest with clearly produced organization of ideas and logical development The GET is a direct test of writing ability in which test-takers must in a two-hour time period write an essay on a given topic

74 CHAPTER 4 Standardized Testing

3 Design select and arrange test tasksitems

Once specifications for a standardized test have been stipulated the sometimes never-ending task of designing selecting and arranging items beginS The spe~s act much like a blueprint in determining the number and types of items to be created Lets look at the three examples

(A) TOEFL test design specifies that each item be coded for content and statisshytical characteristics Content coding ensures that each examinee will receive test questions that assess a variety of skills (reading comprehending the main idea or understanding inferences) and cover a variety of subject matter without unduly biasing the content toward a subset of test-takers (for example in the listening secshytion involving an academic lecture the content must be universal enough for stushydents from many different academic fields of study) Statistical characteristics including the IRT equivalents of estimates of item ~~ility (IF) and the ability of an item to discriminate (ID) between higher orlower ability levels ate also coded

Items are then designed by a team who select and adapt items solicited from a bank of items that have been deposited by freemiddotlance writers and ErS staff Probes for the reading section for example are usually excerpts from authentic general or academic reading that are edited for linguistic difficulty culture bias or other topic biases Items are designed to test overall comprehension certain specific informashytion and inference

Consider the following sample of a reading selection and ten items based on it from a practice TOEFL (Phillips 2001pp423-424)

For hundreds of years in the early history of America pirates sailed through coastal washyters pillaging and plundering all in their path They stole from other ships andstole from coastal towns not content only to steal they destroyed everything they could not carry avay~ Some of the pirate ships amassed large treasures~ the fates of which are unknown leaving people of today to wonder at their whereabouts and to dream of one day coming across some lost treasure

One notoriously large treasure was on the pirate ship Whidah which sank in the washyters off Cape Cod during a strong storm in 1717 A hundred of the crew members went down with the ship along with its treasure of coins gold silver and jewels The treasure on board had an estimated value on todays market of more than 100 million dollars

The remains of the Whidah were discovered in 1984 by Barry Clifford who had spent years of painstaking research and tireless searching only finally to locate the ship about 500 yards from shore A considerable amount of treasure from the centuries-old ship has been recovered from its watery grave but there is clearly still a lot more out there Just as a reminder of what the waters off the coast have been protecting for hundreds of years occasional pieces of gold or silver or jewels still wash up on the beaches and lucky beach-goers find pieces of the treasure

11 ~ Thepa~e mainly diccus5eS

CH4PTER 4 Standardized Testing 75

(e) what really happened to the Whidahs pirates (D) why people go to the beach

12 It is NOT mentioned in the passage that pirates did which of the following (A) They killed lots of people (B) They robbed other ships (e) They took things from towns (D) They gathered big treasures

13 The word amassed in line 4 is closest in meaning to (A) sold (e) transported (B) hid (D) gathered

14 It is implied in the passage that the Whidahs crew (A) died (B) went diving (e) searched for the treasure (D) escaped with parts of the treasure

15 Which of the following is NOT mentioned as part of the treasure of the Whidah (A) Art objects (B) Coins (e) Gold and si Iver (D) Jewels

16 The word estimated in line 10 is closest in meaning to which of the following (A) Known (C) Approximate (B) Sold (D) Decided

17 The passage indicates that the cargo of theWhidah is worth about (A) $100000 (B) $1000000 (C) $10000000 (D) $100000000

18 The work that Barry Clifford did to locate the Whidah was NOT (A) successfu I (B) effortless (C) detailed (D) lengthy

19 It is mentioned in the passage that the treasure of the Whidah (A) is not very valuable (8) is all in museums (C) has not all been found (D) was taken to share by the pi rates

20 The paragraph following the passage most likely discusses (A) what Barry Clifford is doing today (8) the fate of the Whidahs crew (e) other storms in the area of Cape Cod (D) additional pieces that turn up from the Whidahs treasure

76 CHAPTER 4 Standardized Testing

As you can see items target the assessment of comprehension of the main idea (item 11) stated details (17 19) unstated details (12 15 18) implied details (14 20) and vocabulary in context (13 16) An argument could be made about the cultural schemata implied in a passage about pirate ships and you could engage in an angels on the head of a pin argument about the importance of picking cershytain vocabulary for emphasis but every test item is a sample of a larger domain and each of these fulfills its designated specification

Before any such items are released into a form of the TOEFL (or any validated standardized test) they are piloted and sCientifically selected to meet difficulty specshyifications within each subsection section and the test overall Furthermore those items are also selected to meet a desired discrimination index Both of these indices are important considerations in the design of a computer-adaptive test where pershyformance on one item determines the next one to be presented to the test-taker (See Chapter 3 for a complete treatment of multiple-choice item design)

(B)The selection of items in the ESLPT entailed two-entirel) different processes In the two subsections of -the test that elicit writing performance (summary of reading response to reading) the main hurdles were (a) selecting appropriate passhysages for test-takers to read (b) providing appropriate prompts and (c) processing data from pilot testing Passages have to conform to standards of content validity by being within the genre and the difficulty of the material used in the courses The prompt in each case (the section asking for a summary and the section asking for a response) has to be tailOred to fit the passage but a general template is used

[n the multiple-choice editing test that seeks to test grammar proofreading ability the first and easier task is to choose an appropriate essay within which to embed errors The more complicated task is to embed a specified number of errors from a previously determined taxonomy of error categories Those error categories came directly from student errors as perceived by their teachers (verb tenses verb agreeshyment logical connectors articles etc) The disttactors for each item were selected from actual errors that students make Itemsiti pilot versions were then coded fordifshyficulty and discrinlination indices after which final assembly of items could occur

(C) The GET prompts are designed by a faculty committee of examiners who are speCialists in the field of university academic writing The assumption is made that the topics are universally appealing and capable of yielding the intended product of an essay that requires an organized logical argument and conclusion No pilot testing of prompts is conducted The conditions for administration remain constant two-hour time limit sit-down context paper and pencil closed-book format Consider the following recent prompt

Graduate Essay Test sample prompt

In the Middletown Elementary School District the assistant superintendent has just been made superintendent in another district Her resignation leaves vacant the districts only administrative position ever held by a woman The School Board in response to strong

CHAPTER 4 Standardized Testing 77

arguments from the Teachers Association has urged that a woman be hired to replace her As a member of the hiring committee you must help choose her successor

Only one woman applicant meets the written qualifications for the job the two top male applicants are both more experienced than she

The hiring committee has asked each committee member to prepare a written statement to distribute before meeting together to discuss the issue Write a report that represents your position making it as logical and persuasive as possible

Some facts you may wish to draw on 1 Women make up more than 75 percent of classroom teachers but hold fewer than

10 percent of administrative positions in education Administrators salaries average 30 percent more than teachers salaries

2 The local Teachers Association is 89 percent women mostly under 40 In a heated debate on television a member of the National Organization of Women (NOW) and the chair of the Teachers Association threatened if a man is hired to bring a class-action suit against the district on behalf of all women teachers who cannot expect advancement because of discriminatory hiring practices

3 The local Lions Club which contributes heavily to school sports says hiring the less experienced woman would not be in the best interests of the schoolthe children or the teachers

The finalists for the position

1 Carole Gates Classroom teacher 10 years Teacher of the Year 1985 supervisor ofpractice teachers at Teachers College former president of Teachers Associ ati on Administrative Credential 1984 EdD degree 1986 assistant principal of Hoptown Elementary School 2 years

2 Spud Stonewall Principal of Middletown Elementary 15 years PhD in educational adminis~ration State Board of Education Committee for Improving Elementary School Curriculum 1982-present

3 Jim Henderson School Administrator 22 yearsgradesK-9-supports innovation in education Fair Bargaining Award 1981 former coach for winning collegiate basketball team 10 years

It is clear from such a prompt that the problem the test-takers must address is complex that there is sufficient information here for writing an essay and that testshytakers will be reasonably challenged to write a clear statement of opinion What also emerges from this prompt (and virtually any prompt that one might propose) is the potential cultural effect on the numerous international students who must take the GIIT Is it possible that such students who are not familiar with school systems in the United States with hiring procedures and perhaps with the politics of school board elections might be at a disadvantage in mounting their arguments within a two-hour time frame Some (such as Hosoya 2001) have strongly claimed such a bias

78 CHAPTER 4 Standardized Testing

4 Make appropriate evaluations of different kinds of items

In Chapter 3 the concepts of item facility (IF) item discrimination (ID) and disshytractor analysis were introduced As the discussion there showed such calculations provide useful infornlation for classroom tests but sometimes the time and effort involved in perfornling them may not be practical especially if the classroom-based test is a one-time test Yet for a standardized multiple-choice test that is designed to be marketed commercially andor administered a number of times andor adminisshytered in a different form these indices are a must

For other types of response formats namely production responses different forms of evaluation become importantThe principles of p-mpoundti~ality ~d poundabWty are prominent along with the concept o(JacjJjt Practicality issues in such items include the clarity of directions timing of the test ease of administration and how much time is required to score responses Reliability is a major player in instances where more than one scorer is employed and to a lesser extent when a single scorer has to evaluate tests over long spans of time that could lead to deterioration of stanshydards Facility is also a key to the validity and success of an item type ~irecshytions complex- language obscure topics fuz~Qata and culturally biased

~Jfiformatioifma~alliead to a highei1eVermiddotof diffiCidty than one desires (A) The IF ID and efficiency statistics of the multiple-choice items of current

forms of the TOEFL are not publicly available information For reasons of security and protection of patented copyrighted materials they must remain behind the closed doors of the ETS development staff Those statistics remain of paramount importance in the ongoing production ofTOEFL items and forms and are the founshydation stones for demonstrating the equatability of forms Statistical indices on retired forms of the TOEFL are available on request for research purposes

The essay portion of theTOEFL undergoes scrutiny for its practicality reliability and facility Special attention is given to reliabilIty since two human scorers must read each essay and every time a third reader becomes necessary (when the two readers disagree by more than one point) it costs ETS more money

(B) In the case of the open-ended responses on the two written tasks on the ESLPT a similar set of judgments must be made Some evaluative impressions of the effectiveness of prompts and passages are gained from informal student and scorer feedback In the developmental stage of the newly revised ESLPT both types of feedshyback were formally solicited through questiQnnaires and interviews That informashytion proved to be invaluable in the revisIon of prompts and stimulus reading passages After each administration now the teacher-scorers provide informal feedshyback on their perceptions of the effectiveness of the prompts and readings

The multiple-choice editing passage showed the value of statistical findings in determining the usefulness of items and pointing administrators toward revisions Following is a sample of the format used

CHAPTER 4- Standardized Testing 79

Multiple-choice editing passage

(1)EYer since supermarkets first appeared they have beentake over ~ world ABC 0

(2) Supermarkets have changed peoples life ~ yet and at the same time changes in ABC

peoples life ~ have encouraged the opening of supermarkets o

The task was to locate the error in each sentence Statistical tests on the experishymental version of this section revealed that a number of the 45 items were found to be of zero IF (no difficulty whatsoever) and of inconsequential discrimination power (some IDs of 15 and lower) Many distractors were of no consequence because they lured no one Such information led to a revision of numerous it~ms and their options eventually strengthening the effectiveness of this section

(C)The GET like its written counterparts in the ESLPT is a test ofwritten ability with a single prompt and therefore questions of practicality and J~~illy~are also largely observational No data are collected from students on their perceptions but the scorers have an opportunity to reflect on the validity ofa given topiC After one sitting a topic is retired which eliininates the possibility of improving a specific topiC but future framing of topics might benefit from scorers evaluations Inter-rater reliability is checked periodically and reader training sessions are modified if too many instances of unreliability appear

5 Specify scoring procedures and reporting formats - ---

A systematic assembly of test items in pre-selected arrangements and sequences all of which are validated to confo~ to an e~pected difficulty level should yield a test that can then be scored accurately and reported back to test-takers and institutions efficiently

(A) Of the three tests being exemplifled here the most straightforward scoring procedure comes from the TO~FL the one with the most complex issues of validashytion deSign and assembly Scores are calculated and reported fora) three sections of the TOEFL (the essay ratings are combined with the Structure and Written Expression score) and (b) a total score (range 40 to 300 on the computer-based TOEFL and 310 to 677 on the paper-and-pencil TOEFL) A separate score (c) for the Essay (range 0 to 6) is also provided on the examinees score record (see simulation of a score record on page 80)

80 CHAPTER 4 Standardized Testing

Facsimile of a TOEFLreg score report

TOEFL Scaled Scores Claudia Y Estudiante Peru ___

19 17 17 177 Listening Structure Writing Reading Total Score

Essay rati ng 30

The rating scale for the essay is virtually the same one that is used for the Test of Written English (see Chapter 9 for details) with a zero level added for no response copying the topic only writing completely off topic or not writing in English

(B) The ESLPT reports a score for each of themiddot essay sections but the rating scale differs between them because in one case the objective is to write a summary and in the other to write a response to a reading ~ch essayi~pd lgtY ~o readet~ ifhFfF js a discrepancy of more than one level a third reader1resolves the differenceThe ~ditiilg section is machine-scanned and -scored with a total score and ~th part-scores for each ofthe grammaticaVrhetorlcal sectionS From these data placement administrators have adequate information to make placements and teachers receive some diagnostic inforshymation on each student in their classes Students do not receive their essays back

(C) Each GET is read by two trained readers who give a score between 1 and 4 according to the following scale

Graduate Essay Test Scoring Guide

Please make no marks on the writers work Write your reader number and score on the front cover of each test booklet

4 Superior The opening establishes context purpose and point of view the body of the essay developsmiddot recommendations-logically and coherently The writer demonshystrates awareness of the complexities in the situation and provides analysis of the probJem offers compelling or common-sense reasons for recommendations made makes underlying assumptions explicit

The writer uses fluent and idiomatic English with few mechanical errors Style reshyveals syntactic maturity is dear and direct is not choppy or over-colloquial nor over-formal stuffy or unfocused Occasional spelling or punctuation errors may be easily attributed to hasty transcription under pressure

3 Competent After an opening that establishes context and purpose the paper unfolds with few lapses in coherence but may have somewhat less clear organization of less explicit transitions than a top-score paper It may have somewhat less compelling logic or slightly less-wellreasoned suggestions than a 4 paper though it will provide reasons for the recommendations made

The writer uses dear fluent and generally idiomatic English but may make minor or infrequent ESL errors (preposition errors dropped articles or verb endings etc) or repeat a single error (eg not punctuate possessive nouns) Occasional lapses of style are offSet by demonstrated mastery of syntax

CHAPTE84 Standardized Testing 81

2 Weak The writer makes somewhat simplistic suggestions not fully supported with reashysons fails to cite key facts offers little analysis of the problem or shows a limited grasp of the situation the given information is copied or listed withlittle integration into argument Points may be random or repetitious Writing may be badly focused with careless use of abstract language resulting in predication errors or illogical sentences

ESL andlor careless mechanical errors are frequent enough to be distracting OR sentences may be choppy style over-casual usage occasionally unidiomatic

1 Inadequate The essay may be disjointed incoherent or minimally developed The writer shows little grasp of the complex issues involved is unable to establish conshytext point of view or purpose in opening of paper or has a poor sense of audience Mechanical andor ESL errors or unidiomatic usages are frequent sentences may be ungrammatical OR correct but short and very simple

The two readers scores are added to yield a total possible score of 2 to 8 Test administrators recommend a score of 6 as the threshold for allowing a student to pursue graduate-level courses Anything below that is accompanied by a recomshymendation that the student either repeat the test or take a remedial course in gradshyuate writing offered in one of several different departments Students receive neither their essays nor any feedback other than the fmal score

6 Perform ongoing construct validation studies

From the above discussion it should be clear that no standardized instrument is expected to be used repeatedly without a ramporou~program of ongoing c~-sectmct valiltiatiOll Any standardized test once developed must be accompanied by sysshy~

tematic periodic corroboration of its effectiveness and by steps toward its improveshyment This rigor is especially true of tests that are produced in equated forms that is forms must be reliable across tests such that a score on a subsequent form of a test-has-the~same validityand-interpretability as its original

(A) The TOEFL program in cooperation with other tests produced by ETS has an impressive program of research Over the years dozens of TOEFL-sponsored research studies have appeared in the TOEFL Monograph Series An early example ofsuch a study was the seminal Duran et aI (1985) study TOEFLfrom a Communicative ViewpOint on Language Proficiency which examined the content characteristics of the TOEFL from a communicative perspective based on current research in applied linguistics and lanshyguage proficiency assessment More recent studies (such as Ginther 2001 Leacock amp Chodorow 2001 Powers et aI 2002) demonstrate an impressive array of scrutiny

(B) For approximately 20 years the ESLPT appeared to be placing students relishyably by means of an essay and a multiple-choice grammar and vocabulary test Over the years the security of the latter became s1lspect and the faculty administrators wished to see some content validity achieved in the process In the year 2000 that process began with a group of graduate students (Imao et aI 2000) in consl1ltation with faculty members and continued to fruition in the form of a new ESLPT reported in lmao (2002) The development of the new ESlPT involved a lengthy process of

82 CHAPTER 4 Standardized Testing

both content and construct validation along with facing such practical issues as scoring the written sections and a machine scorable multiple-choice answer sheet

The process of ongoing validation will no doubt continue as new forms of the editing section are created and as new prompts and reading passages are created for the writing section Such a validation process should also include consistent checks on placement accuracy and on face validity

(C) At this time there is little or no research to validate the GET itself For its conshy struct validation its administrators rely on a stockpile of research on university-level academic writing tests such as theTWEThe holistic scoring rubric and the topics and administrative conditions of the GET are to some extent patterned after that of the TWE In recent years some criticism of the GEf has come from international test-takers (Hosoya 2001) who posit that the topics and time limits of the GET among other facshytors work to the disadvantage of writers whose native language is not English These validity issues remain to be fully addressed in a comprehensive research study

I I

STANDARDIZED IANGUAGE PROFICIENCY TESTING

Tests of language profiCiency presuppose a comprehensive definition of the specific competencies that comprise overall language ability The specifications for the TOEFL provided an illustration of an operational definition of ability for assessment purposes This is not the only way to conceptualize the concept Swain (1990) offered a multidimensional view of profiCiency assessment by referring to three linshyguistic traits (grammar discourse and sociolinguistics) that can be assessed by means of oral multiple-choice and written responses (see Table 41) Swains conshyception was not meant to be an exhaustive analysis of ability but rather to serve as an operational framework for constructing proficiency assessments

Another defmition and conceptualization of profiCiency is suggested by the ACTFL association mentioned earlier ACfFL takes a holistic and more unitary view of proficiency in describing four levels superior advanced intermediate and noviceWithin each level descriptions of listening speaking reading and writing are provided as guidelines for assessment For example the ACfFL Guidelines describe the superior level of speaking as follows

ACTFL speaking guidelines summary superior-level

Superior-level speakers are characterized by the ability to

bull participate fully and effectively in conversations in formal and informal settings on topics related to practical needs and areas of professional andor scholarly interests

bull provide a structured argument to explain and defend opinions and develop effective hypotheses within extended discourse

bull discuss topics concretely and abstractly bull deal with a linguistically unfamiliar situation bull maintain a high degree of linguistic accuracy bull satisfy the linguistic demands of professional andor scholarly life

CHAPTER4 Standardized Testing 83

The other three ACfFL levels use the same parameters in describing progressively lower proficiencies across all four skills Such taxonomie~ have the advantage of considering a number of functions of linguistic discourse but the disadvantage at the lower levels of overly emphasizing test-takers deficiencies

Table 41 Traits of second language proficiency (Swain 1990 p 403)

Trait Grammar Discourse Sociolinguistic

focus on grammatical focus on textual focus on social accuracy within cohesion and appropriateness of sentences coherence language use

Method

Oral structured interview story telling and argumentationpersuasion

role-play ofspeech acts requests offers complaints

scored for accuracy of verbal morphology prepositions syntax

detailed rating for identification logical sequence and time orientation and global ratings for coherence

scored for ability to distinguish formal and informal register

Multiple-choice

sentence-level select the correct form exercise

paragraph-level select the coherent sentence exercise

speech act-Ievelselect the appropriate utterance exercise

(45 items) (29 items) (28 items)

involving verb morphology prepositionsan-d-uther items

Written composition

narrative and letter of persuasion

narrative and letter of persuasion

formal request letter and informal note

scored for accuracy of verb morphology prepositions syntax

detailed ratings much as for oral discourse and global rating for coherence

scored for the ability to distinguish formal and inforJ1lil1 register

FOUR STANDARDIZED lANGUAGE PROFICIENCY TESTS

We now tum to some of the better-known standardized tests of overall language ability or profiCiency to examine some of the typical formats used in commercially available tests We will not look at standardized tests of other specific skills here but that should not lead you to think by any means that proficiency is the only kind of test in the field that is standardized Three standardized oral production tests the

84 CHAPTER 4 Standardized Testing

Test of Spoken English (fSE) the Oral Proficiency Inventory (OPI) and PbonePassreg are discussed in Chapter 7 and the Test of Written English (WE) is covered in ChapterS

Four commercially produced standardized tests of English language proficiency are described briefly in this section the TOEFL the Michigan English Language Assessment Battery (MELAB) the International English Language Testing System (lELTS) and the Test of English for International Communication (fOEICreg) In an appendix to this chapter are sample items from each section of each test When you turn to that appendix use the following questions to help you evaluate these four tests and their subsections

1 What item types are included 2 How practical and reliable does each subsection of each test appear to be 3 Do the item types and tasks appropriately represent a conceptualizatio~ of

language proficiency (ability) That is can you evaluate their construct validity

4 Do the tasks achieve face validity 5 Are the tasks authentic 6 Is there some washback potential in the tasks

Test of English as a Foreign Language (TOEFL)

Producer Educational Testing Service (ETS) Objective To test overall proficiency (language ability) Primary market Almost exclusively US universities and colleges for admission

purposes Type Computer-based (CB) (and two sections are-computer-adaptive)

A traditional paper-based (PB) version is also available Response modes Multiple-choice responses essay Specifications See the box on pp 72-73 Time allocation Up to 4 hours (CB) 3 hours (PB) Internet access wwwtoeflorg

Comments In the North American context the TOEFL is the most widely used comshymercially available standardized test of proficiency Each year the TOEFL test is adminisshytered to approximately 800000 candidates in more than 200 countries It is highly respected because of the thorough program of ongoing research and development conshyducted by ETS The TOEFLs primary use is to set proficiency standards for international students seeking admission to English-speaking universities More than 4200 academic institutions government agencies scholarship programs and licensingcertification agenshycies in more than 80 countries use TOEFL scores By 2004 the TOEFL will include a secshytion on oral production

CHAPTER 4 Standardi~ed Testing 85

Michigan English Language Assessment Battery (MELAB)

Producer English language Institute University of Michigan Objective To test overall proficiency (language ability) Primary market Mostly US and Canadian language programs and colleges

some worldwide educational settings as well Type Paper-based Response modes Multiple-choice responses essay Time allocation 25 to 35 hours Internet access wwwlsaumicheduelimelabhtm

Specifications The MElAB consists of three sections Part 1 a 3D-minute impromptu essay is written on an assigned topic Part 2 a 25-minute multiple-choice listening comshyprehension test is delivered via tape recorder Part 3 is a 100-item 75-minute multipleshychoice test containing grammar doze reading vocabulary and reading comprehension An oral interview (speaking test) is optional

Comments The Ell at the University of Michigan has been producing the MELAB and its earlier incarnation (Michigan Test of English language Proficiency) since 1961 like the TOEFL it serves a North American audience but is also used internationally While its use is not as widespread as the TOEFL its validity is widely respected Because it is cheaper than the TOEFL and more easily obtained it is popular among language schools and institutes Many institutions and companies accept MElAB scores in lieu ofTOEFL scores

International English Language Testing System (IELTS)

Producer Jointly managed by The University of Cambridge local Examinations Syndicate (UClES) The British Council and lOP Education Australia

Objective To test overall proficiency (language ability) Primary-market Australian British Canadian and New Zealand academic

institutions and professional organizations American academic institutions are increasingly accepting IELTS for admissions purposes

1)rpe Computer-based (for the Reading and Writing sections) papershybased for the listening and Speaking modules

Response modes Multiple-choice responses essay oral production Time allocation 2 hours 45 minutes Internet access httpwwwieltsorgl

httpwwwudesorguk httpwwwbritishcouncilorg

Specifications Reading candidates choose between academic reading or general training reading (60 minutes) Writing the same option academic writing or general training writing (60 minutes) Listening four sections for all candidates (30 minutes) Speaking five sections for all candidates (1015 minutes)

86 CHAPTER 4 Standardized Testing

Comments The University of Cambridge local Examinations Syndicate (UCLES) has been producing English language tests since 1858 Now with three organizations cooperatshying to form the IELTS more than a million examinations are administered every year In 2002 a computer-based version of the Reading and Writing modules of the IELTS became available at selected centers around the world The other sections are administered locally by an examinet The paper-based IELTS remains an option for candidates The IELTS retains the distinct advantage of requiring all four skills in the test-takers performance

Test of English for International Communication (TOEICreg)

Producer The Chauncey Group International a subsidiary of Educational Testing Service

Objelttive To test overall proficiency (langlJage ability) Primary market Worldwide business commerce and industry contexts

(workplace settings) Type Computer-based and paper-based versions Response modes Multiple-choice responses Time allocation 2 hours Internet access httpwwwtoeiccom

Specifications Listening Comprehension 100 items administered by audiocassette Four types of task statements questions short conversations and short talks (approxishymately 45 minutes) Reading 100 items Three types of task cloze sentences error recogshynition and reading comprehension (75 minutes)

Comments The TOEIC has become a very widely used international test of English proficiency in workplace settings where English is required for job performance The conshytent includes many different employment settings such as conferences presentations sales ordering shipping schedules reservations (etters and memoranda It is approprishyate to use in educational settings where vocational or workplace English courses are being offered

sect sect sect sect sect

The construction of a valid standardized test is no minor accomplishment whether the instrument is large- or small-scale The designing of specifications alone as this chapter illustrates requires a sophisticated process of construct valishydation coupled with considerations of practicality Then the construction of items and scoringinterpretation procedures may require a lengthy period of trial and error with prototypes of the final form of the testWith painstaking attention to all the details of construction the end product can result in a cost-effective timeshysaving accurate instrument Your use of the results of such assessments can provide useful data on learners language abilities But your caution is warranted as well for all the reasons discussed in this chapter The next chapter will elaborate on what lies behind that need for a cautious approach to standardized assessment

CHAPTER4 Standardized Testing 87

EXERCISES

[Note (I) Individual work (G) Group or pair work (C) Whole-class discussion]

1 (C) Tell the class about the worst test experience youve ever had Briefly anamiddot lyze what made the experience so unbearable and try to come up with sugshygestions for improvement of the test andor its administrative conditions

2 (G) In pairs or small groups compile a brief list of pros and cons of standardshyized testing Cite illustrations of as many items in each list as possible Report your lists and examples to the rest o~ the class

3 (I) Select a standardized test that you are quite familiar with (probably a recent experience) Mentally evaluate that test using the five principles of practicality reliability validity authenticity and washback Report yourevaluashytion to the class

4 (G) The appendix to this chapter provides sample items from Jour different tests of language proficiency In groups one test for each group analyze your test for (a) content validity (b) face validity and (c) authenticity

5 (C) Do you think that the sample TOEFL reading passage about pirates (pages 74-75) and the Graduate EssayTest prompt (pages 76-77) about a school board hiring committee have any culture bias Discuss this and other cultural biases you have noticed in tests Is it possible to design a test that is completely free of culture bias

6 (CG) Compare the differences in conceptualization of language proficiency represented by Swains model the TOEFL and the ACfFL philosophy Which one best represents current thinking about communicative language ability What are the strengths and weaknesses of each approach

FORYOlIILEURTHER READING

Gronlund Norman E (1998) Assessment of student achievement Sixth Edition Boston Allyn and Bacon

Gronlunds classic also mentioned in Chapter 3 offers a concise overview of features of standardized tests offering definitions and examples of the statistical considerations in interpreting scores His approach is unbiased cleady written and accessible to those who might fear the mathematics of standardized testing

Phillips Deborah 2001 Long1nan introductory course for the TOEFL test White Plains NY Pearson Education

A careful examination of this or any other reputable preparation course for a standardized language test is well worth a students time Note especially how the book acquaints the user with the specifications of the test and offers a number of useful strategie~ that can be llsed in preparation for the test and during irs adn1inistration

88 CHAPTER 4 Standardized Testing

APPENDIX TO CHAPTER 4

Commercial Proficiency Tests Sample Items and Tasks

Test of English a~ a Foreign Language (TOEFLreg)

Listening r

Part A

In this section you will hear short conversations between two people In some ofthe conversations each person speaks only once In other conversations one or both of the people speak more than once Each conversation is followed by one questionabQlt it Each question in this part has four answer choices You should click on the best answer to each question Answer the questions on the basis of what is stated or implied by the speakers Here is an example On the computerscreen you will see

[man and woman talking]

On the recording you will hear

(woman) Hey wheres your sociology book (man) At home Why carry it around when were just going to be taking

a test (woman) Dont you remember Professor Smith said we could us it during

the test (man) Ohl no Well Ive still got an hour right Im so glad I ran into you

You wiII then see and hear the question before the answer choices appear

What will the man probably do next

o Begin studying for the sociology test o Explain the problem to his professor o Go home to get his textbook o Borrow the womans book

To choose an answer you will click on an oval The oval next to that answer will darken After you click on Next and Confirm Answer the next conversation will be presented

Part B

In this section you will hear several longer conversations and talks Each conversation or talk is followed by several questions The conversations talks and questions will not be repeated The conversations and talks are about a variety of topics You do not need speshycial knowledge of the topics to answer the questions correctly Rather you should answer each question on the basis of what is stated or implied by the speakers in the conversashytions or talks

For most of the questions you will need to click on the best of four possible answers Some questions will have special directions The special directions will appear in a box on the computer screen Here is an exampie ot a conversation and some questions

CHAPTER 4 Standardized Testing 89

Marine Biology (narrator) Listen to part of a discussion in a marine biology class

(professor) A few years ago our local government passed a number of strict environmental laws As a result Sunrise Beach looks nothing Ii ke it did ten years ago The water is cleaner and theres been a tremendous increase in all kinds of marine life which is why were going there on Thursday

(woman) I dont know if I agree that the water quality has improved I mean I was out there last weekend and it looked all brown It didnt seem too clean to me

(professor) Actually the color of the water doesnt always indicate whether its polluted The brown color you mentioned might be a result of pollution or it can mean a kind of brown algae is growing there Its called devils apron and it actually serves as food for whales

(man) So when does the water look blue (professor) Well water thats completely unpolluted is actually colorless But

it often looks bluish-green because the sunlight can penetrate deep down and thats the color thats reflected

(woman) But sometimes it looks really green Whats that about (professor) Ok well its the same principle as with devils apron the

water might be green because of different types of green algae there-gulfweed phytoplankton You all should finish reading about algae and plankton before we go In fact those are the types of living things Im going to ask you to be looking for when were there

Now get ready to answer the questions

What is the discussion mainly about

o The importance of protecting ocean environments o The reasons why ocean water appears to be different colors o The survival of whales in polluted water o The effect that colored ocean water has on algae

To choose an answer click on an oval The oval next to that answer will darken After you click on Next and Confirm Answer the next question will be presented

According to the professor what can make ocean water look browngt

o Pollution o Cloudy Skies o Sand o Algae

Click on 2 answers

To choose your answers you will click on the squares An XII wiii appear in each square

bullbullbullbullbullbullbull

90 CHAPTER 4 Standardized Testing

Structure and Written Expression This section measures the ability to recognize language that is appropriate for standard written English There are two types ofquestions in this section In the first type ofquestion there are incomplete sentences Beneath each sentence there are four words or phrases

Directions CIiSk on the one word or phrase that best completes the sentence

The colum~ine flower __ to nearly all of the United States can be raised from seed in almost any garden

native how native is how native is it is native

Time Help Confirm

After you click on Next and Confirm Answ~ the next question willbe presented

The second type of question has four underlined words or phrases You will choose the one underlined word or phrase that must be changed for the sentence to be correct

Directions Click on the one underlined word or phrase that must be changed for the senshytence to be correct

One of the most difficult problems in understanding sleep is determining what the funcshytions of sleep ~

lime Help Confirm

Clicking on an underlined word or phrase will darken it

Reading This section measures the ability to read and understand short passages similar in topic and style to those that students are likely to encounter in North American universities and colleges This section contains reading passages and questions about the passages There are several different types of questions in this section In the Reading section you will first have the opportunity to read the passage

The temperature of the Sun is over 10000 degrees Fahrenheit at the surface but it rises perhaps more than 270000000 at the center The Sun is so much hotter than the Earth that matter can exist only as a gasl except perhaps at the core In the core of the Sun the pressures are so great that despite the high temperature there may be a small solid core However no one really knows since the center of the Sun can never be directly observed ~ Solar astronomers do know that the Sun is divided into five general layers or zones Starting at the outside and going down into the Sun the zones are the corona chromoshysphere hotosphere convection zone and finally the core The first three zones are reshygarded as the Suns atmosphere But since the Sun has no solid surface it is hard to middottell where the atmosphere ends and the main body of the Sun begins

The Suns outermost layer begins about 10000 miles above the visible surface and goes outward for millions of miles This is the only part of the Sun that can be seen during an eclipse such as the one in February 1979 At any other time the corona can be seen

bullbullbullbullbullbullbull

bull bullbullbullbullbullbull

CHAPTER 4 Standardized Testing 91

only when special instruments are used on cameras and telescopes to block the light from the photosphere

The corona is a brilliant pearly white filmy light about as bright as the full Moon Its beautiful rays are a sensational sight during an eclipse The coronas rays flash out in a brilliant fan that has wispy spikelike rays near the Suns north and south poles The corona is generally thickest at the Suns equator The corona is made up of gases streamshying outward at tremendous speeds that reach a temperature of more than 2 million deshygrees Fahrenheit The gas thins out as it reaches the space around the planets By the time the gas of the corona reaches the Earth it has a relatively low density

When you have finished reading the passage you will use the mouse to click on Proceed Then the questions about the passage will be presented You are to choose the one best anshyswer to each question Answer all questions about the information in a passage on the basis ofwhat is stated or implied in that passage Most ofthe questions will be multiple-choice questions To answer these questions you will click on a choice below the question

With what topic is paragraph 2 mainly concerned

o How the Sun evolved o The structure of the Sun o Why scientists study the Sun o The distaflce of the Sun from the planets

Paragraph 2 is marked with an arrow (~)

You will see the next question after you click on Next

To answer some questions you will click on a word or phrase Here is an example

Look at the word one in the passage Click on the word or phrase in the bold text that one refers to To answer you can click on any part of the word or phrase in the passage Jour choice will darken to show which word you have chosen

The Suns outermost layer begins about 10000 miles above the visible surface and goes outward for millions of miles This is the only part of the Sun that can be seen durshying an eclipse such as the one in February 1979 At any other time the corona can be seen only when special instruments are used on cameras and telescopes to block the Iight from the photosphere

You will see the next question after you click on~ To answer some q~estions you will click on a sentence in the passage Here is an example

~ The corona is a brilliant pearly white filmy light about as bright as the full Moon Its beautiful rays are a sensational sight during an eclipse The coronas rays flash out in a brilliant fan that has wispy spikelike rays near the Suns north and south poles The corona is generally thickest at the Suns equator ~ The corona is made up of gases streaming outward at tremendous speeds that reach a temperature of more than 2 million degrees Fahrenheit The gas thins out as it reaches the space around the planets By the time the gas of the corona reaches the Earth it has a relatively low density

bull bullbullbullbullbullbull

92 CHAPTER 4 Standardized Testing

Click on the sentence in paragraph 4 or 5 in which the author compares the light of the Suns outermost layer to that ofanother astronomical body Paragraphs 4 and 5 are marked with arrows (~)

To answer some questions you will click on a square to add a sentence to the passage Here is an example -The following sentence can be added to paragraph 1

At the center of the Earths solar system lies the Sun

Where would it best fit in paragraph I Click on the square to add the sentence to the paragraph

D The temperature of the Sun is over 10000 degrees Fahrenheit at the surface but it rises to perhaps morethan 27000000deg at the center 0 The Sun is so much hotter than the Earth that matter can exist only as a gasi except p~rHapsatth~ c6relp the c~re of the ii Sun the pressures are so great that despite the high temperature there may be a small solid core D However no one really knows since the center of the Sun can never be directly observed D 0100

When you click on a square the sentence will appear in the passage at the place you have chosen You can read the sentence added to the paragraph to see if this is the best place to add it You can click on another square to change your answer The sentence will be added and shown in a dark box

Writing In this section you will have an opportunity to demonstrate your ability to write in Enshyglish This includes the ability to generate and organize ideas to support those ideas with examples or evidence and to compose in standard written English in response to an asshysigned topic You will have 30 minutes to write your essay on that topic You must write on the topic you are assigned An essay on any other topic will receive a score of 0 Read the topic below and then make any notes that will help you plan your response Begin typing your response in the box at the bottom of the screen or write your answer on the answer sheet provided to you

Following is a sample topic

Do you agree or disagree with the following statemenH

Teachers should make learning enjoyable and fun for their students

Use specific reasons and examples to support your opinion

CHAPTER 4 Standarczed Testing 93

Michigan English Language Assessment Battery (MELAB)

Composition The time limit for the composition is 30 minutes You must write on only one of the top~

ics below If you write about something else your composition paper will not be graded and you cannot be given a final score If you do not understand the topics ask the exam~ iner to explain or to translate them You may be asked to give your opinion ofsomething and explain why you believe this to describe something from your experience or to exshyplain a problem and offer possible solutions You should write at least one page Some sample topics are

1 What do you think is your countrys greatest problem Explain in detail and tell what you think can be done about it

2 What are the characteristics of a good teacher Explain and give examples 3 An optimist is someone who sees the good side of things A pessimist sees the

bad side Are you an optimist or a pessimist Relate a personal experience that shows this

4 In your opinion are the benefits of space exploration really worth the enormous costs Discuss

Most MELAB compositions are one or two pages long (about 200-300 words) If your paper is extremely short (less than 150 words) your composition will be given a lower score Before you begin writing you might want to take 2 or 3 minutes to plan your comshyposition and to make a short outline to organize your thoughts Such outlines will not be graded they are only to help you You should use the last 5 minutes to read through your composition and to make changes or corrections

Your composition will be graded on how clearly you express yourself in English and on the range of English you are able to use and your control in doing so This means your composition should be well organized your arguments should be fully developed and you should show a range ofgrammatical structures and broad vocabulary Compositions that consist only of very short sentences and very simple vocabulary cannot be given the

middothighest scores If errors are not frequent and if they do not confuse your meaning they will not lower your score very much

Listening Now you will hear a short lecture You may take notes during the lecture Following the lecture you will be asked some questions about it

Therell be a two-week exhibit of the paintings of the little-known master Laura Bernhart at the Claire Osmond Galleries starting on the fifteenth of the month and running through the thirtieth Bernharts known for her innovative designs in abstract expressionism Though a true original she declared a spiritual heritage from Salvador Dali the famous Spanish painter Since Bernhart lived a rather solitary life and died while only in her twenties few people are aware of her works This showing at the Osmond Galleries will provide many with an introduction to her works

10 Where is the exhibit a the Art Museum b the Dali Galleries c the Osmond Galleries

94 CHAPTER 4 Standardized Testing

11 What is Bernhart known for a her copies of Dalis paintings b the originality of her designs c her exhibitions

12 What will going to the exhibit allow most people to do a to see Saivador Dalis paintings b to see Bernharts works for the first time c to learn about Spanish art

Grammar

1 What did the teacher just tell you

She reminded our notebooksI a us to bring b that we bring c our bringing d we should bring

2 Is Bill a good dancer

Not really __ he tries very hard a in spite of h despite c even though d while

3 your clothes are all wet1

Yes I didnt come __ the rain soon enough a away to b over to c down with d in from

Cloze In years to come zoos will not only be places where animals are exhibited to the public but repositories where rare species can be saved from extinction (7) captive breeding The most powerful force (8) the future of many animals-and of zoos-is the decline of the wild (9) even zoo directors would argue that (10) are better places for animals than the fields and forest of their native (11) yet zoos may be the last chance for some creatures that would otherwise pass qUietly into oblivion

7 a through c from b of d damage

8 a bringing c to b that d influencing

9 a But c Not b So d Then

10 a where c even b zoos d wilds

11 alands c residence b life d field

CHAPTER 4 Standardized Testing 95

Vocabulary

12 Mark has a flair for writing a need b purpose c talent d dislike

13 Bill Collins launched his restaurant last June a moved b started c sold d bought

14 John will not accept the censure a burden b blame c credit d decision

15 I cant think of the answer Can you give me a __ a hint b token c taste d gaze

16 Because fewer people are taking expensive vacations the tourist industry is in a a choke b grope c grumble d slump

17 I disagree with a few of his opinions but __ we agree a deliberately b conclusively c essentially d immensely

Reading The influenza virus is a single molecule built from many millions of single atoms You must have heard of the viruses which are sometimes called living molecules While bacteria can be considered as a type of plant secreting pOisonous substances into the body of the organism they attack viruses are living organisms themselves We may conshysider them as regular chemical molecules since they have a strictly aefined atomic strucshyture but on the other hand we must also consider them as being alive since they are able to multiply in unlimited quantities

18 According to the passage bacteria are a poisons

b larger than viruses c very small d plants

96 CHAPTER 4 Standardized Testing

19 The writer says that viruses are alive because they a have a complex atomic structure b move c multiply d need warmth and light

20 The atomic structure of viruses a is -tJIariable b is strictly defined c cannot be analyzed chemically d is more complex than that of bacteria

International English Language Testing System (fELTS)

I

listening

The Listening Module has four sections The first two sections are concerned with social needs There is a conversation between two speakers and then a monologue For examshyple a conversation about travel arrangements or decisions on a night out and a speech about student services on a university campus or arrangements for meals during a confershyence The final two sections are concerned with situations related more closely to educashytional or training contexts For example conversation between a tutor and a student about an assignment or between three students planning a research project and a lecture or talk ofgeneral academic interest All the topics are ofgeneral interest and it makes no difference what subjects candidates study Tests and tasks become more difficult as the sections progress A range of English accents and dialects are used in the recording which reflects the international usage of IELTS

Academic Reading [A 7S0-word article on-th-e- topic of Wind Power in the US with a short glossary at the end]

Questions 1-5

Complete the summary below

Choose your answers from the box below the summary and write them in boxes 1-5 on your answer sheet Note There are more words or phrases than you will need to fill the gaps You may use any word or phrase more than once

Example The failure during the late 1970s and early 19805 of an attempt to establish a widespread wind power industry in the United States resulted largely from the (1) bull in oil prices during this period The industry is now experiencing a steady (2) due to improveshyments in technology and an increased awareness of the potential in the power of wind The wind turbines that are now being made based in part on the (3) of wide- ranging research in Europe are easier to manufacture and maintain than their predecesshysors This has led wind-turbine makers to be able to standardise and thus minimize (4) There has been growing (S) of the importance of wind power as an energy source

CHAPTER 4 Standardized Testing 97

criticism stability skepticism success operating costs decisions design costs fall effects production costs growth decline failure recognition results

Questions 6-1 0 Look at the following list of issues (Questions 6-10) and implications (A-C) Match each issue with one implication Write the appropriate letters A-C in boxes 6-10 on your anshyswer sheet

Example The current price of one wind-generated kilowatt Answer

6 The recent installation of systems taking advantage of economies of scale

7 The potential of meeting one fifth of current U5 energy requirements by wind power

8 The level of acceptance of current wind turbine technology

9 A comparison of costs between conventional and wind power sources

10 The view of wind power in the European Union

Implications

A provides evidence against claims that electricity produced from wind power is relatively expensive

B supports claims that wind power js an important source of energy

C opposes the view that wind power technology requires further-development

General Training Reading Read the passage on Daybreak trips by coach and look at the statements below On your answer sheet write

TRUE if the statement is true FALSE jf the statement is false

NOlGIVEN if the information is not given in the leaflet

1 MiIlers Coaches owns Cambridges Cam bus fleet

2 Premier is an older company than Millers

3 Most of the Daybreak coaches are less than 5 years old

4 Daybreak fares are more expensive than most of their competitors

5 Soft drinks and refreshments are served on most longer journeys

6 Smoking is permitted at the rear of the coach on longer journeys

7 Tickets must be bought in advance from an authorised Daybreak agent

6 Tickets and seats can be reserved by phoning the Daybreak Hotline

9 Daybreak passengers must join their coach at Cambridge Drummer Street

10 Daybreak cannot guarantee return times

98 CHAPTER 4 Standardized Testing

FROM CAMBRIDGE AND SURROUNDING AREA

SPRING IS INTHEAIR

Welcome to our Spring Daybreak programme which continues the tradition of offering unbeatable value for money day trips and tours All the excursions in this brochure will be operated by Pr~mier Travel Services Limited or Millers Coaches both companies are part of the CHLGroup owners of Cambridges Cambus fleet

WERE PROUD OF OUR TRADITION

Premier was established in 1936 the Company now offers the highest standards of coaching in todays competitive operating environment Miller has an enviable reputation stretching back over the past 20 years offering coach services at realistic prices Weve traveled a long way since our early days of pre-war seaside trips Now our fleet of 50 modern coaches (few are more than five years old) operate throughout Britain and Europe but were pleased to still maintain the high standards of quality and service the trademark of our founders nearly sixty years ago

EXCLUSIVE FEATURES

Admission-inclusive fares All Daybreak fares (unless specifically otherwise stated) include admission charges to the attractions shows and exhibits we visit Many full-day scenic tours are accompanied by a fully trained English Tourist Board Blue Badge guide or local experienced driverguide Some Daybreaks include lunch or afternoon tea Compare our admission inclusive fares and see how much you save Cheapest is not the best and value for money is guaranteed If you compare our bargain Daybreak fares beware--most of our competishytors do not offer an all-inclusive fare

SEAT RESERVATIONS

We value the freedom of choice so you can choose your seat when you book The seat reservation is guaranteed a-nd remains yours at all times when aboard the coach

NO SMOKING COMFORT

With the comfort of our passengers in mind coaches on all our Daybreaks are no smokshying throughout In the interests of fellow passengers comfort we kindly ask that smokers observe our no smoking policy On scenic tours and longer journeys ample refreshment stops are provided when of course smoking is permitted

YOUR QUESTIONS ANSWERED

Do I need to book Booking in advance is strongly recommended as all Daybreak tours are subject to demand Subject to availability stand-by tickets can be purchased from the driver

What ti me does the coach leave The coach departs from Cambridge Drummer Street (Bay 12 adjacent to public toilets) at the time shown There are many additional joining points indicated by departure codes in the brochure If you are joining at one of our less popular joining points you will be adshyvised of your pick-up time (normally by telephone) not less than 48 hours before deparshyture In this way we can minimize the length of pick-up routes and reduce journey times for the majority of passengers

CHAPTER 4 Standardized Testing 99

What time do we get back An approximate return time is shown for each excursion The tim~s shown serve as a guide but road conditions can sometimes cause delay If your arrival will be later than advertised your driver will try to allow for a telephone call during the return journey

Where can I board the coach All the Daybreaks in the brochure leave from Cambridge Drummer Street (Bay 12 adjashycent to public toilets) at the time shown Many Daybreaks offer additional pick-ups for pre-booked passengers within Cambridge and the surrounding area This facility must be requested at the time of booking

Academic Writing Writing Task 1 You should spend about 20 minutes on this task

The graph below shows the different modes of transport used to travel to and from work in one European city in 1950 1970 and 1990

[graph shown here]

Write a report for a university lecturer describing the information shown below You should write at least 150 words

Writing Task 2 You should spend about 40 minutes on this task

Present a written argument or case to an educated reader with no specialist knowledge of the folowing topic

It is inevitable that as technology develops so traditional cultures must be lost Technolshyogy and tradition are incompatible-you cannot have both together

To what extent do you agree or disagree with this statement Give reasons for your answer You should write at least 250 words You should use your own ideas knowlshyedge and experience and support your arguments with examples and relevant evidence

General Training Writing Writing Task 1 You should spend about 20 minutes on this task You rent a house through an agency The heating system has stopped working You phoned the agency a week ago but it has still not been mended Write a letter to the agency Explain the situation and teil them what you want them to do about it

You should write at least 150 words You do NOT need to write your own address

Begin your letter as follows

Dear - ___-I

Writing Task 2 You should spend about 40 minutes on this task As part ofa class assignment you have to write about the following topic

100 CHAPTER 4 Standardized Testing

Some businesses now say that no one can smoke cigarettes in any of their offices Some governments have banned smoking in all public placesThis is a good idea but it takes away some of our freedom

Do you agree or disagree Give reasons for your answer You should write at least 250 words

Speaking In each ofthe three parts of the speaking module a specific function is fulfilled In Part 1 the candidates answer general questions about themselves their homes or families their jobs or studies their interests and a range ofsimilar familiar topic areas This part lasts between four and five minutes In Part 2 the candidate is given a verbal prompt on a card and is asked to talk on a particular topic The candidate has one minute to prepare before speaking at length for between one and two minutes The examiner then asks one or two wind-down questions In Part 3 the examiner and candidate engage in a discusshysion of more abstract issues and concepts which are thematically linked to the topic prompt in Part 2 The discussion lasts between four and five minutes

All interviews are recorded on audiocassette Here is a sample ofa Part 2 topic

Describe a teacher who has greatly influenced you in your education

You shou Id say

where you met them what subject they taught what was special about them

and explain why this person influenced you so much

You will have to talk about the topic for 1 to 2 minutes You have 1 minute to think about what you are going to say You can make some notes if you wish

Test of English for International Communication (TOEICreg)

listening

Part 1 Photographs Directions For each question you will see a picture in your test book and you will hear four short statements The statements will be spoken just one time They will not be printed in your test book so you must listen carefully to understand what the speaker says When you hear the four statements look at the picture in your test book and choose the statement that best describes what you see in the picture Then on your answer sheet find the number of the question and mark your answer

[photograph of a scientist looking through a microscope]

You will hear Look at the picture marked number 1 in your test book

(A) Shes speaking into a microphone (B) Shes put on her glasses (C) She has both eyes open (D) Shes using a microscope

CHAPTER 4 Standardized Testing 101

Part 2 Question-Response Directions In this part of the test you will hear a question or statement spoken in Enshyglish followed by three responses also spoken in English The question or staten1ent and the responses will be spoken just one time They will not be printed in your test book so you must listen carefully to understand what the speakers say You are to choose the best response to each question or statement

Question 1 You will hear Ms Morikawa has worked here for a long time hasnt she

(A) At three oclock (B) No Ive lost my watch (C) More than ten years

Question 2 You will hear Which of these papers has a wider circulation

(A) The morning edition (B) Get more exercise (C) By messenger

Part 3 Short Conversations Directions In this part of the test you will hear short conversations between two people The conversations will not be printed in your test book You will hear the conversations only once so you must listen carefully to understand what the speakers say In your test book you will read a question about each conversation The question will be followed by four answers You are to choose the best answer to each question and mark it on your answer sheet

Question 1 (Man) We should think about finding another restaurant for lunch (Woman) Why The food and service here are great

(Man) Yes but the prices are going up every week

You will read Why is this man unhappy with the restaurant

(A) It is too noisy (B) It is too expensive (C) It is too crowded (D) It is too difficult to find

Question 2 (Woman A) How was Dr Borgs recent trip to Singapore (Woman B) She enjoyed the tour of the port very much (Woman A) They say its one of the most active in Asia

You will read 2 What did Dr Borg find interesting

(A) The tourist center (B) The airport (C) The musical performance (D) The harbor

Part 4 Short Talks Directions In this part of the test you vill hear several short talks Each will be spoken just one time They will not be printed in your test book so you must listen carefully to understand and remember what is said In your test book you will read two or more questions about each short talk The questions will be followed by four answers You are to choose the best answer to each question and mark it on your answer sheet

102 CHAPTER 4 Standardized Testing

You will hear Questions 1 and 2 refer to the following announcement

Good afternoon and welcome aboard Nordair Flight 857 from Copenhagen to Bangkok with intermediate stops in Dubai and Calcutta We are preparing for departure in a few minutes At this time your seat back should be returned to its full upright position and your seat belt s~ould be fastened OUf anticipated total flying time to Dubai is six hours and twenty-five minutes I hope you enjoy the flight You will hecJr Now read question 1 in your test book and answer it You will read 1 What is the final destination of the flight

(A) Bangkok (B) Copenhagen (C) Dubai (O) Calcutta

You will hear Now read question 2 in your test book and answer it You will read 2 What will happen in a few minutes

(A) The flight will land in Dubai I

(B) The passengers will board the plane (C) The plane will take off (0) The gate number will be announced

Reading In this section of the test you will have the chance to show how well you understand written English There are three parts to this section with special directions for each part

Part 4 Incomplete Sentences Directions This part of the test has incomplete sentences Four words or phrases marked (A) (8) (e) (D) are given beneath each sentence You are to choose the one word or phrase that best completes the sentence Then on your answer sheet find the number of the question and mark your answer

1 Mr Yangs trip will __ him away from the office for ten days (A) withdraw (B) continue (C) retain (0) keep

2 The company that Marie DuBois started now sells __ products throughout the world (A) its (B) it (C) theirs (D) them

3 If your shipment is not delivered __ Tuesday you can request a full refund for the merchandise (A) at (B) by (C) within (D) while

CHAPTER 4 Standardized Testing 103

Part 6 Error Recognition Directions In this part ofthe test each sentence has four words or phrases underlined The four underlined parts of the sentence are marked (A) (B) (C) (D) You are to identify the one underlined word or phrase that should be corrected or rewritten Then on your answer sheet find the number of the question and mark your answer

1 The pamphlet contains some importance information about the current exhibit ABC D

2 No matter how Jong it taking to finish the annual report it must be done properly ABC D

3 The popularity of jogging appears to have decreased since the past couple of years ABC D

Part 7 Reading Comprehension Directions The questions in this part of the test are based on a selection of reading mateshyrials such as notices letters) forms newspaper and magazine articles) and advertisements You are to choose the one best answer (A) (B) (C) or (OJ to each quesshytion Then on your ariswefsheelfindthe number of the qUestion andmcirkyour answer Answer all questions following each reading selection on thebasis of what is stated or implied in that selection

The Museum ofTechnology is a hands-on museum designed for people to experience science at w()rk~ Visitors are encouraged to use test and handle the objects o~ display Special demonstrations are scheduled for the first and second Wednesdays of each month at 1330 Open Tuesday-Friday 1200-1630 Saturday 1000-1730 and Sunday 11 00-1630

1 When during the month can visitors see special demonstrations (A) Every weekend (B) The first two Wednesdays (C) One afternoon a week (D) Every other Wednesday

Questions 2 and 3 refer to the followi ng notice

NOTICE If you are unable to work because of an extended illness or injury that is not workshyrelated you may be entitled to receive weekly benefits from your employer or the firms insurance company To claim benefits you must file a claim form within thirty days of the first day of your disability Before filing the claim you must ask your doctor to fill in the Doctors Statement on the claim form stating the period of disability

3 To whom is this notice addressed (A) Employers (8) Doctors (C) Employees (D) When paying the bill

4 When must the claim form be filed (A) On the first of the month (8) On the thirtieth of the month (C) On the first day ofdisabifity (D) Within 30 days of the start of disability

Page 9: Standardized Testing Chapter 4 Brown

74 CHAPTER 4 Standardized Testing

3 Design select and arrange test tasksitems

Once specifications for a standardized test have been stipulated the sometimes never-ending task of designing selecting and arranging items beginS The spe~s act much like a blueprint in determining the number and types of items to be created Lets look at the three examples

(A) TOEFL test design specifies that each item be coded for content and statisshytical characteristics Content coding ensures that each examinee will receive test questions that assess a variety of skills (reading comprehending the main idea or understanding inferences) and cover a variety of subject matter without unduly biasing the content toward a subset of test-takers (for example in the listening secshytion involving an academic lecture the content must be universal enough for stushydents from many different academic fields of study) Statistical characteristics including the IRT equivalents of estimates of item ~~ility (IF) and the ability of an item to discriminate (ID) between higher orlower ability levels ate also coded

Items are then designed by a team who select and adapt items solicited from a bank of items that have been deposited by freemiddotlance writers and ErS staff Probes for the reading section for example are usually excerpts from authentic general or academic reading that are edited for linguistic difficulty culture bias or other topic biases Items are designed to test overall comprehension certain specific informashytion and inference

Consider the following sample of a reading selection and ten items based on it from a practice TOEFL (Phillips 2001pp423-424)

For hundreds of years in the early history of America pirates sailed through coastal washyters pillaging and plundering all in their path They stole from other ships andstole from coastal towns not content only to steal they destroyed everything they could not carry avay~ Some of the pirate ships amassed large treasures~ the fates of which are unknown leaving people of today to wonder at their whereabouts and to dream of one day coming across some lost treasure

One notoriously large treasure was on the pirate ship Whidah which sank in the washyters off Cape Cod during a strong storm in 1717 A hundred of the crew members went down with the ship along with its treasure of coins gold silver and jewels The treasure on board had an estimated value on todays market of more than 100 million dollars

The remains of the Whidah were discovered in 1984 by Barry Clifford who had spent years of painstaking research and tireless searching only finally to locate the ship about 500 yards from shore A considerable amount of treasure from the centuries-old ship has been recovered from its watery grave but there is clearly still a lot more out there Just as a reminder of what the waters off the coast have been protecting for hundreds of years occasional pieces of gold or silver or jewels still wash up on the beaches and lucky beach-goers find pieces of the treasure

11 ~ Thepa~e mainly diccus5eS

CH4PTER 4 Standardized Testing 75

(e) what really happened to the Whidahs pirates (D) why people go to the beach

12 It is NOT mentioned in the passage that pirates did which of the following (A) They killed lots of people (B) They robbed other ships (e) They took things from towns (D) They gathered big treasures

13 The word amassed in line 4 is closest in meaning to (A) sold (e) transported (B) hid (D) gathered

14 It is implied in the passage that the Whidahs crew (A) died (B) went diving (e) searched for the treasure (D) escaped with parts of the treasure

15 Which of the following is NOT mentioned as part of the treasure of the Whidah (A) Art objects (B) Coins (e) Gold and si Iver (D) Jewels

16 The word estimated in line 10 is closest in meaning to which of the following (A) Known (C) Approximate (B) Sold (D) Decided

17 The passage indicates that the cargo of theWhidah is worth about (A) $100000 (B) $1000000 (C) $10000000 (D) $100000000

18 The work that Barry Clifford did to locate the Whidah was NOT (A) successfu I (B) effortless (C) detailed (D) lengthy

19 It is mentioned in the passage that the treasure of the Whidah (A) is not very valuable (8) is all in museums (C) has not all been found (D) was taken to share by the pi rates

20 The paragraph following the passage most likely discusses (A) what Barry Clifford is doing today (8) the fate of the Whidahs crew (e) other storms in the area of Cape Cod (D) additional pieces that turn up from the Whidahs treasure

76 CHAPTER 4 Standardized Testing

As you can see items target the assessment of comprehension of the main idea (item 11) stated details (17 19) unstated details (12 15 18) implied details (14 20) and vocabulary in context (13 16) An argument could be made about the cultural schemata implied in a passage about pirate ships and you could engage in an angels on the head of a pin argument about the importance of picking cershytain vocabulary for emphasis but every test item is a sample of a larger domain and each of these fulfills its designated specification

Before any such items are released into a form of the TOEFL (or any validated standardized test) they are piloted and sCientifically selected to meet difficulty specshyifications within each subsection section and the test overall Furthermore those items are also selected to meet a desired discrimination index Both of these indices are important considerations in the design of a computer-adaptive test where pershyformance on one item determines the next one to be presented to the test-taker (See Chapter 3 for a complete treatment of multiple-choice item design)

(B)The selection of items in the ESLPT entailed two-entirel) different processes In the two subsections of -the test that elicit writing performance (summary of reading response to reading) the main hurdles were (a) selecting appropriate passhysages for test-takers to read (b) providing appropriate prompts and (c) processing data from pilot testing Passages have to conform to standards of content validity by being within the genre and the difficulty of the material used in the courses The prompt in each case (the section asking for a summary and the section asking for a response) has to be tailOred to fit the passage but a general template is used

[n the multiple-choice editing test that seeks to test grammar proofreading ability the first and easier task is to choose an appropriate essay within which to embed errors The more complicated task is to embed a specified number of errors from a previously determined taxonomy of error categories Those error categories came directly from student errors as perceived by their teachers (verb tenses verb agreeshyment logical connectors articles etc) The disttactors for each item were selected from actual errors that students make Itemsiti pilot versions were then coded fordifshyficulty and discrinlination indices after which final assembly of items could occur

(C) The GET prompts are designed by a faculty committee of examiners who are speCialists in the field of university academic writing The assumption is made that the topics are universally appealing and capable of yielding the intended product of an essay that requires an organized logical argument and conclusion No pilot testing of prompts is conducted The conditions for administration remain constant two-hour time limit sit-down context paper and pencil closed-book format Consider the following recent prompt

Graduate Essay Test sample prompt

In the Middletown Elementary School District the assistant superintendent has just been made superintendent in another district Her resignation leaves vacant the districts only administrative position ever held by a woman The School Board in response to strong

CHAPTER 4 Standardized Testing 77

arguments from the Teachers Association has urged that a woman be hired to replace her As a member of the hiring committee you must help choose her successor

Only one woman applicant meets the written qualifications for the job the two top male applicants are both more experienced than she

The hiring committee has asked each committee member to prepare a written statement to distribute before meeting together to discuss the issue Write a report that represents your position making it as logical and persuasive as possible

Some facts you may wish to draw on 1 Women make up more than 75 percent of classroom teachers but hold fewer than

10 percent of administrative positions in education Administrators salaries average 30 percent more than teachers salaries

2 The local Teachers Association is 89 percent women mostly under 40 In a heated debate on television a member of the National Organization of Women (NOW) and the chair of the Teachers Association threatened if a man is hired to bring a class-action suit against the district on behalf of all women teachers who cannot expect advancement because of discriminatory hiring practices

3 The local Lions Club which contributes heavily to school sports says hiring the less experienced woman would not be in the best interests of the schoolthe children or the teachers

The finalists for the position

1 Carole Gates Classroom teacher 10 years Teacher of the Year 1985 supervisor ofpractice teachers at Teachers College former president of Teachers Associ ati on Administrative Credential 1984 EdD degree 1986 assistant principal of Hoptown Elementary School 2 years

2 Spud Stonewall Principal of Middletown Elementary 15 years PhD in educational adminis~ration State Board of Education Committee for Improving Elementary School Curriculum 1982-present

3 Jim Henderson School Administrator 22 yearsgradesK-9-supports innovation in education Fair Bargaining Award 1981 former coach for winning collegiate basketball team 10 years

It is clear from such a prompt that the problem the test-takers must address is complex that there is sufficient information here for writing an essay and that testshytakers will be reasonably challenged to write a clear statement of opinion What also emerges from this prompt (and virtually any prompt that one might propose) is the potential cultural effect on the numerous international students who must take the GIIT Is it possible that such students who are not familiar with school systems in the United States with hiring procedures and perhaps with the politics of school board elections might be at a disadvantage in mounting their arguments within a two-hour time frame Some (such as Hosoya 2001) have strongly claimed such a bias

78 CHAPTER 4 Standardized Testing

4 Make appropriate evaluations of different kinds of items

In Chapter 3 the concepts of item facility (IF) item discrimination (ID) and disshytractor analysis were introduced As the discussion there showed such calculations provide useful infornlation for classroom tests but sometimes the time and effort involved in perfornling them may not be practical especially if the classroom-based test is a one-time test Yet for a standardized multiple-choice test that is designed to be marketed commercially andor administered a number of times andor adminisshytered in a different form these indices are a must

For other types of response formats namely production responses different forms of evaluation become importantThe principles of p-mpoundti~ality ~d poundabWty are prominent along with the concept o(JacjJjt Practicality issues in such items include the clarity of directions timing of the test ease of administration and how much time is required to score responses Reliability is a major player in instances where more than one scorer is employed and to a lesser extent when a single scorer has to evaluate tests over long spans of time that could lead to deterioration of stanshydards Facility is also a key to the validity and success of an item type ~irecshytions complex- language obscure topics fuz~Qata and culturally biased

~Jfiformatioifma~alliead to a highei1eVermiddotof diffiCidty than one desires (A) The IF ID and efficiency statistics of the multiple-choice items of current

forms of the TOEFL are not publicly available information For reasons of security and protection of patented copyrighted materials they must remain behind the closed doors of the ETS development staff Those statistics remain of paramount importance in the ongoing production ofTOEFL items and forms and are the founshydation stones for demonstrating the equatability of forms Statistical indices on retired forms of the TOEFL are available on request for research purposes

The essay portion of theTOEFL undergoes scrutiny for its practicality reliability and facility Special attention is given to reliabilIty since two human scorers must read each essay and every time a third reader becomes necessary (when the two readers disagree by more than one point) it costs ETS more money

(B) In the case of the open-ended responses on the two written tasks on the ESLPT a similar set of judgments must be made Some evaluative impressions of the effectiveness of prompts and passages are gained from informal student and scorer feedback In the developmental stage of the newly revised ESLPT both types of feedshyback were formally solicited through questiQnnaires and interviews That informashytion proved to be invaluable in the revisIon of prompts and stimulus reading passages After each administration now the teacher-scorers provide informal feedshyback on their perceptions of the effectiveness of the prompts and readings

The multiple-choice editing passage showed the value of statistical findings in determining the usefulness of items and pointing administrators toward revisions Following is a sample of the format used

CHAPTER 4- Standardized Testing 79

Multiple-choice editing passage

(1)EYer since supermarkets first appeared they have beentake over ~ world ABC 0

(2) Supermarkets have changed peoples life ~ yet and at the same time changes in ABC

peoples life ~ have encouraged the opening of supermarkets o

The task was to locate the error in each sentence Statistical tests on the experishymental version of this section revealed that a number of the 45 items were found to be of zero IF (no difficulty whatsoever) and of inconsequential discrimination power (some IDs of 15 and lower) Many distractors were of no consequence because they lured no one Such information led to a revision of numerous it~ms and their options eventually strengthening the effectiveness of this section

(C)The GET like its written counterparts in the ESLPT is a test ofwritten ability with a single prompt and therefore questions of practicality and J~~illy~are also largely observational No data are collected from students on their perceptions but the scorers have an opportunity to reflect on the validity ofa given topiC After one sitting a topic is retired which eliininates the possibility of improving a specific topiC but future framing of topics might benefit from scorers evaluations Inter-rater reliability is checked periodically and reader training sessions are modified if too many instances of unreliability appear

5 Specify scoring procedures and reporting formats - ---

A systematic assembly of test items in pre-selected arrangements and sequences all of which are validated to confo~ to an e~pected difficulty level should yield a test that can then be scored accurately and reported back to test-takers and institutions efficiently

(A) Of the three tests being exemplifled here the most straightforward scoring procedure comes from the TO~FL the one with the most complex issues of validashytion deSign and assembly Scores are calculated and reported fora) three sections of the TOEFL (the essay ratings are combined with the Structure and Written Expression score) and (b) a total score (range 40 to 300 on the computer-based TOEFL and 310 to 677 on the paper-and-pencil TOEFL) A separate score (c) for the Essay (range 0 to 6) is also provided on the examinees score record (see simulation of a score record on page 80)

80 CHAPTER 4 Standardized Testing

Facsimile of a TOEFLreg score report

TOEFL Scaled Scores Claudia Y Estudiante Peru ___

19 17 17 177 Listening Structure Writing Reading Total Score

Essay rati ng 30

The rating scale for the essay is virtually the same one that is used for the Test of Written English (see Chapter 9 for details) with a zero level added for no response copying the topic only writing completely off topic or not writing in English

(B) The ESLPT reports a score for each of themiddot essay sections but the rating scale differs between them because in one case the objective is to write a summary and in the other to write a response to a reading ~ch essayi~pd lgtY ~o readet~ ifhFfF js a discrepancy of more than one level a third reader1resolves the differenceThe ~ditiilg section is machine-scanned and -scored with a total score and ~th part-scores for each ofthe grammaticaVrhetorlcal sectionS From these data placement administrators have adequate information to make placements and teachers receive some diagnostic inforshymation on each student in their classes Students do not receive their essays back

(C) Each GET is read by two trained readers who give a score between 1 and 4 according to the following scale

Graduate Essay Test Scoring Guide

Please make no marks on the writers work Write your reader number and score on the front cover of each test booklet

4 Superior The opening establishes context purpose and point of view the body of the essay developsmiddot recommendations-logically and coherently The writer demonshystrates awareness of the complexities in the situation and provides analysis of the probJem offers compelling or common-sense reasons for recommendations made makes underlying assumptions explicit

The writer uses fluent and idiomatic English with few mechanical errors Style reshyveals syntactic maturity is dear and direct is not choppy or over-colloquial nor over-formal stuffy or unfocused Occasional spelling or punctuation errors may be easily attributed to hasty transcription under pressure

3 Competent After an opening that establishes context and purpose the paper unfolds with few lapses in coherence but may have somewhat less clear organization of less explicit transitions than a top-score paper It may have somewhat less compelling logic or slightly less-wellreasoned suggestions than a 4 paper though it will provide reasons for the recommendations made

The writer uses dear fluent and generally idiomatic English but may make minor or infrequent ESL errors (preposition errors dropped articles or verb endings etc) or repeat a single error (eg not punctuate possessive nouns) Occasional lapses of style are offSet by demonstrated mastery of syntax

CHAPTE84 Standardized Testing 81

2 Weak The writer makes somewhat simplistic suggestions not fully supported with reashysons fails to cite key facts offers little analysis of the problem or shows a limited grasp of the situation the given information is copied or listed withlittle integration into argument Points may be random or repetitious Writing may be badly focused with careless use of abstract language resulting in predication errors or illogical sentences

ESL andlor careless mechanical errors are frequent enough to be distracting OR sentences may be choppy style over-casual usage occasionally unidiomatic

1 Inadequate The essay may be disjointed incoherent or minimally developed The writer shows little grasp of the complex issues involved is unable to establish conshytext point of view or purpose in opening of paper or has a poor sense of audience Mechanical andor ESL errors or unidiomatic usages are frequent sentences may be ungrammatical OR correct but short and very simple

The two readers scores are added to yield a total possible score of 2 to 8 Test administrators recommend a score of 6 as the threshold for allowing a student to pursue graduate-level courses Anything below that is accompanied by a recomshymendation that the student either repeat the test or take a remedial course in gradshyuate writing offered in one of several different departments Students receive neither their essays nor any feedback other than the fmal score

6 Perform ongoing construct validation studies

From the above discussion it should be clear that no standardized instrument is expected to be used repeatedly without a ramporou~program of ongoing c~-sectmct valiltiatiOll Any standardized test once developed must be accompanied by sysshy~

tematic periodic corroboration of its effectiveness and by steps toward its improveshyment This rigor is especially true of tests that are produced in equated forms that is forms must be reliable across tests such that a score on a subsequent form of a test-has-the~same validityand-interpretability as its original

(A) The TOEFL program in cooperation with other tests produced by ETS has an impressive program of research Over the years dozens of TOEFL-sponsored research studies have appeared in the TOEFL Monograph Series An early example ofsuch a study was the seminal Duran et aI (1985) study TOEFLfrom a Communicative ViewpOint on Language Proficiency which examined the content characteristics of the TOEFL from a communicative perspective based on current research in applied linguistics and lanshyguage proficiency assessment More recent studies (such as Ginther 2001 Leacock amp Chodorow 2001 Powers et aI 2002) demonstrate an impressive array of scrutiny

(B) For approximately 20 years the ESLPT appeared to be placing students relishyably by means of an essay and a multiple-choice grammar and vocabulary test Over the years the security of the latter became s1lspect and the faculty administrators wished to see some content validity achieved in the process In the year 2000 that process began with a group of graduate students (Imao et aI 2000) in consl1ltation with faculty members and continued to fruition in the form of a new ESLPT reported in lmao (2002) The development of the new ESlPT involved a lengthy process of

82 CHAPTER 4 Standardized Testing

both content and construct validation along with facing such practical issues as scoring the written sections and a machine scorable multiple-choice answer sheet

The process of ongoing validation will no doubt continue as new forms of the editing section are created and as new prompts and reading passages are created for the writing section Such a validation process should also include consistent checks on placement accuracy and on face validity

(C) At this time there is little or no research to validate the GET itself For its conshy struct validation its administrators rely on a stockpile of research on university-level academic writing tests such as theTWEThe holistic scoring rubric and the topics and administrative conditions of the GET are to some extent patterned after that of the TWE In recent years some criticism of the GEf has come from international test-takers (Hosoya 2001) who posit that the topics and time limits of the GET among other facshytors work to the disadvantage of writers whose native language is not English These validity issues remain to be fully addressed in a comprehensive research study

I I

STANDARDIZED IANGUAGE PROFICIENCY TESTING

Tests of language profiCiency presuppose a comprehensive definition of the specific competencies that comprise overall language ability The specifications for the TOEFL provided an illustration of an operational definition of ability for assessment purposes This is not the only way to conceptualize the concept Swain (1990) offered a multidimensional view of profiCiency assessment by referring to three linshyguistic traits (grammar discourse and sociolinguistics) that can be assessed by means of oral multiple-choice and written responses (see Table 41) Swains conshyception was not meant to be an exhaustive analysis of ability but rather to serve as an operational framework for constructing proficiency assessments

Another defmition and conceptualization of profiCiency is suggested by the ACTFL association mentioned earlier ACfFL takes a holistic and more unitary view of proficiency in describing four levels superior advanced intermediate and noviceWithin each level descriptions of listening speaking reading and writing are provided as guidelines for assessment For example the ACfFL Guidelines describe the superior level of speaking as follows

ACTFL speaking guidelines summary superior-level

Superior-level speakers are characterized by the ability to

bull participate fully and effectively in conversations in formal and informal settings on topics related to practical needs and areas of professional andor scholarly interests

bull provide a structured argument to explain and defend opinions and develop effective hypotheses within extended discourse

bull discuss topics concretely and abstractly bull deal with a linguistically unfamiliar situation bull maintain a high degree of linguistic accuracy bull satisfy the linguistic demands of professional andor scholarly life

CHAPTER4 Standardized Testing 83

The other three ACfFL levels use the same parameters in describing progressively lower proficiencies across all four skills Such taxonomie~ have the advantage of considering a number of functions of linguistic discourse but the disadvantage at the lower levels of overly emphasizing test-takers deficiencies

Table 41 Traits of second language proficiency (Swain 1990 p 403)

Trait Grammar Discourse Sociolinguistic

focus on grammatical focus on textual focus on social accuracy within cohesion and appropriateness of sentences coherence language use

Method

Oral structured interview story telling and argumentationpersuasion

role-play ofspeech acts requests offers complaints

scored for accuracy of verbal morphology prepositions syntax

detailed rating for identification logical sequence and time orientation and global ratings for coherence

scored for ability to distinguish formal and informal register

Multiple-choice

sentence-level select the correct form exercise

paragraph-level select the coherent sentence exercise

speech act-Ievelselect the appropriate utterance exercise

(45 items) (29 items) (28 items)

involving verb morphology prepositionsan-d-uther items

Written composition

narrative and letter of persuasion

narrative and letter of persuasion

formal request letter and informal note

scored for accuracy of verb morphology prepositions syntax

detailed ratings much as for oral discourse and global rating for coherence

scored for the ability to distinguish formal and inforJ1lil1 register

FOUR STANDARDIZED lANGUAGE PROFICIENCY TESTS

We now tum to some of the better-known standardized tests of overall language ability or profiCiency to examine some of the typical formats used in commercially available tests We will not look at standardized tests of other specific skills here but that should not lead you to think by any means that proficiency is the only kind of test in the field that is standardized Three standardized oral production tests the

84 CHAPTER 4 Standardized Testing

Test of Spoken English (fSE) the Oral Proficiency Inventory (OPI) and PbonePassreg are discussed in Chapter 7 and the Test of Written English (WE) is covered in ChapterS

Four commercially produced standardized tests of English language proficiency are described briefly in this section the TOEFL the Michigan English Language Assessment Battery (MELAB) the International English Language Testing System (lELTS) and the Test of English for International Communication (fOEICreg) In an appendix to this chapter are sample items from each section of each test When you turn to that appendix use the following questions to help you evaluate these four tests and their subsections

1 What item types are included 2 How practical and reliable does each subsection of each test appear to be 3 Do the item types and tasks appropriately represent a conceptualizatio~ of

language proficiency (ability) That is can you evaluate their construct validity

4 Do the tasks achieve face validity 5 Are the tasks authentic 6 Is there some washback potential in the tasks

Test of English as a Foreign Language (TOEFL)

Producer Educational Testing Service (ETS) Objective To test overall proficiency (language ability) Primary market Almost exclusively US universities and colleges for admission

purposes Type Computer-based (CB) (and two sections are-computer-adaptive)

A traditional paper-based (PB) version is also available Response modes Multiple-choice responses essay Specifications See the box on pp 72-73 Time allocation Up to 4 hours (CB) 3 hours (PB) Internet access wwwtoeflorg

Comments In the North American context the TOEFL is the most widely used comshymercially available standardized test of proficiency Each year the TOEFL test is adminisshytered to approximately 800000 candidates in more than 200 countries It is highly respected because of the thorough program of ongoing research and development conshyducted by ETS The TOEFLs primary use is to set proficiency standards for international students seeking admission to English-speaking universities More than 4200 academic institutions government agencies scholarship programs and licensingcertification agenshycies in more than 80 countries use TOEFL scores By 2004 the TOEFL will include a secshytion on oral production

CHAPTER 4 Standardi~ed Testing 85

Michigan English Language Assessment Battery (MELAB)

Producer English language Institute University of Michigan Objective To test overall proficiency (language ability) Primary market Mostly US and Canadian language programs and colleges

some worldwide educational settings as well Type Paper-based Response modes Multiple-choice responses essay Time allocation 25 to 35 hours Internet access wwwlsaumicheduelimelabhtm

Specifications The MElAB consists of three sections Part 1 a 3D-minute impromptu essay is written on an assigned topic Part 2 a 25-minute multiple-choice listening comshyprehension test is delivered via tape recorder Part 3 is a 100-item 75-minute multipleshychoice test containing grammar doze reading vocabulary and reading comprehension An oral interview (speaking test) is optional

Comments The Ell at the University of Michigan has been producing the MELAB and its earlier incarnation (Michigan Test of English language Proficiency) since 1961 like the TOEFL it serves a North American audience but is also used internationally While its use is not as widespread as the TOEFL its validity is widely respected Because it is cheaper than the TOEFL and more easily obtained it is popular among language schools and institutes Many institutions and companies accept MElAB scores in lieu ofTOEFL scores

International English Language Testing System (IELTS)

Producer Jointly managed by The University of Cambridge local Examinations Syndicate (UClES) The British Council and lOP Education Australia

Objective To test overall proficiency (language ability) Primary-market Australian British Canadian and New Zealand academic

institutions and professional organizations American academic institutions are increasingly accepting IELTS for admissions purposes

1)rpe Computer-based (for the Reading and Writing sections) papershybased for the listening and Speaking modules

Response modes Multiple-choice responses essay oral production Time allocation 2 hours 45 minutes Internet access httpwwwieltsorgl

httpwwwudesorguk httpwwwbritishcouncilorg

Specifications Reading candidates choose between academic reading or general training reading (60 minutes) Writing the same option academic writing or general training writing (60 minutes) Listening four sections for all candidates (30 minutes) Speaking five sections for all candidates (1015 minutes)

86 CHAPTER 4 Standardized Testing

Comments The University of Cambridge local Examinations Syndicate (UCLES) has been producing English language tests since 1858 Now with three organizations cooperatshying to form the IELTS more than a million examinations are administered every year In 2002 a computer-based version of the Reading and Writing modules of the IELTS became available at selected centers around the world The other sections are administered locally by an examinet The paper-based IELTS remains an option for candidates The IELTS retains the distinct advantage of requiring all four skills in the test-takers performance

Test of English for International Communication (TOEICreg)

Producer The Chauncey Group International a subsidiary of Educational Testing Service

Objelttive To test overall proficiency (langlJage ability) Primary market Worldwide business commerce and industry contexts

(workplace settings) Type Computer-based and paper-based versions Response modes Multiple-choice responses Time allocation 2 hours Internet access httpwwwtoeiccom

Specifications Listening Comprehension 100 items administered by audiocassette Four types of task statements questions short conversations and short talks (approxishymately 45 minutes) Reading 100 items Three types of task cloze sentences error recogshynition and reading comprehension (75 minutes)

Comments The TOEIC has become a very widely used international test of English proficiency in workplace settings where English is required for job performance The conshytent includes many different employment settings such as conferences presentations sales ordering shipping schedules reservations (etters and memoranda It is approprishyate to use in educational settings where vocational or workplace English courses are being offered

sect sect sect sect sect

The construction of a valid standardized test is no minor accomplishment whether the instrument is large- or small-scale The designing of specifications alone as this chapter illustrates requires a sophisticated process of construct valishydation coupled with considerations of practicality Then the construction of items and scoringinterpretation procedures may require a lengthy period of trial and error with prototypes of the final form of the testWith painstaking attention to all the details of construction the end product can result in a cost-effective timeshysaving accurate instrument Your use of the results of such assessments can provide useful data on learners language abilities But your caution is warranted as well for all the reasons discussed in this chapter The next chapter will elaborate on what lies behind that need for a cautious approach to standardized assessment

CHAPTER4 Standardized Testing 87

EXERCISES

[Note (I) Individual work (G) Group or pair work (C) Whole-class discussion]

1 (C) Tell the class about the worst test experience youve ever had Briefly anamiddot lyze what made the experience so unbearable and try to come up with sugshygestions for improvement of the test andor its administrative conditions

2 (G) In pairs or small groups compile a brief list of pros and cons of standardshyized testing Cite illustrations of as many items in each list as possible Report your lists and examples to the rest o~ the class

3 (I) Select a standardized test that you are quite familiar with (probably a recent experience) Mentally evaluate that test using the five principles of practicality reliability validity authenticity and washback Report yourevaluashytion to the class

4 (G) The appendix to this chapter provides sample items from Jour different tests of language proficiency In groups one test for each group analyze your test for (a) content validity (b) face validity and (c) authenticity

5 (C) Do you think that the sample TOEFL reading passage about pirates (pages 74-75) and the Graduate EssayTest prompt (pages 76-77) about a school board hiring committee have any culture bias Discuss this and other cultural biases you have noticed in tests Is it possible to design a test that is completely free of culture bias

6 (CG) Compare the differences in conceptualization of language proficiency represented by Swains model the TOEFL and the ACfFL philosophy Which one best represents current thinking about communicative language ability What are the strengths and weaknesses of each approach

FORYOlIILEURTHER READING

Gronlund Norman E (1998) Assessment of student achievement Sixth Edition Boston Allyn and Bacon

Gronlunds classic also mentioned in Chapter 3 offers a concise overview of features of standardized tests offering definitions and examples of the statistical considerations in interpreting scores His approach is unbiased cleady written and accessible to those who might fear the mathematics of standardized testing

Phillips Deborah 2001 Long1nan introductory course for the TOEFL test White Plains NY Pearson Education

A careful examination of this or any other reputable preparation course for a standardized language test is well worth a students time Note especially how the book acquaints the user with the specifications of the test and offers a number of useful strategie~ that can be llsed in preparation for the test and during irs adn1inistration

88 CHAPTER 4 Standardized Testing

APPENDIX TO CHAPTER 4

Commercial Proficiency Tests Sample Items and Tasks

Test of English a~ a Foreign Language (TOEFLreg)

Listening r

Part A

In this section you will hear short conversations between two people In some ofthe conversations each person speaks only once In other conversations one or both of the people speak more than once Each conversation is followed by one questionabQlt it Each question in this part has four answer choices You should click on the best answer to each question Answer the questions on the basis of what is stated or implied by the speakers Here is an example On the computerscreen you will see

[man and woman talking]

On the recording you will hear

(woman) Hey wheres your sociology book (man) At home Why carry it around when were just going to be taking

a test (woman) Dont you remember Professor Smith said we could us it during

the test (man) Ohl no Well Ive still got an hour right Im so glad I ran into you

You wiII then see and hear the question before the answer choices appear

What will the man probably do next

o Begin studying for the sociology test o Explain the problem to his professor o Go home to get his textbook o Borrow the womans book

To choose an answer you will click on an oval The oval next to that answer will darken After you click on Next and Confirm Answer the next conversation will be presented

Part B

In this section you will hear several longer conversations and talks Each conversation or talk is followed by several questions The conversations talks and questions will not be repeated The conversations and talks are about a variety of topics You do not need speshycial knowledge of the topics to answer the questions correctly Rather you should answer each question on the basis of what is stated or implied by the speakers in the conversashytions or talks

For most of the questions you will need to click on the best of four possible answers Some questions will have special directions The special directions will appear in a box on the computer screen Here is an exampie ot a conversation and some questions

CHAPTER 4 Standardized Testing 89

Marine Biology (narrator) Listen to part of a discussion in a marine biology class

(professor) A few years ago our local government passed a number of strict environmental laws As a result Sunrise Beach looks nothing Ii ke it did ten years ago The water is cleaner and theres been a tremendous increase in all kinds of marine life which is why were going there on Thursday

(woman) I dont know if I agree that the water quality has improved I mean I was out there last weekend and it looked all brown It didnt seem too clean to me

(professor) Actually the color of the water doesnt always indicate whether its polluted The brown color you mentioned might be a result of pollution or it can mean a kind of brown algae is growing there Its called devils apron and it actually serves as food for whales

(man) So when does the water look blue (professor) Well water thats completely unpolluted is actually colorless But

it often looks bluish-green because the sunlight can penetrate deep down and thats the color thats reflected

(woman) But sometimes it looks really green Whats that about (professor) Ok well its the same principle as with devils apron the

water might be green because of different types of green algae there-gulfweed phytoplankton You all should finish reading about algae and plankton before we go In fact those are the types of living things Im going to ask you to be looking for when were there

Now get ready to answer the questions

What is the discussion mainly about

o The importance of protecting ocean environments o The reasons why ocean water appears to be different colors o The survival of whales in polluted water o The effect that colored ocean water has on algae

To choose an answer click on an oval The oval next to that answer will darken After you click on Next and Confirm Answer the next question will be presented

According to the professor what can make ocean water look browngt

o Pollution o Cloudy Skies o Sand o Algae

Click on 2 answers

To choose your answers you will click on the squares An XII wiii appear in each square

bullbullbullbullbullbullbull

90 CHAPTER 4 Standardized Testing

Structure and Written Expression This section measures the ability to recognize language that is appropriate for standard written English There are two types ofquestions in this section In the first type ofquestion there are incomplete sentences Beneath each sentence there are four words or phrases

Directions CIiSk on the one word or phrase that best completes the sentence

The colum~ine flower __ to nearly all of the United States can be raised from seed in almost any garden

native how native is how native is it is native

Time Help Confirm

After you click on Next and Confirm Answ~ the next question willbe presented

The second type of question has four underlined words or phrases You will choose the one underlined word or phrase that must be changed for the sentence to be correct

Directions Click on the one underlined word or phrase that must be changed for the senshytence to be correct

One of the most difficult problems in understanding sleep is determining what the funcshytions of sleep ~

lime Help Confirm

Clicking on an underlined word or phrase will darken it

Reading This section measures the ability to read and understand short passages similar in topic and style to those that students are likely to encounter in North American universities and colleges This section contains reading passages and questions about the passages There are several different types of questions in this section In the Reading section you will first have the opportunity to read the passage

The temperature of the Sun is over 10000 degrees Fahrenheit at the surface but it rises perhaps more than 270000000 at the center The Sun is so much hotter than the Earth that matter can exist only as a gasl except perhaps at the core In the core of the Sun the pressures are so great that despite the high temperature there may be a small solid core However no one really knows since the center of the Sun can never be directly observed ~ Solar astronomers do know that the Sun is divided into five general layers or zones Starting at the outside and going down into the Sun the zones are the corona chromoshysphere hotosphere convection zone and finally the core The first three zones are reshygarded as the Suns atmosphere But since the Sun has no solid surface it is hard to middottell where the atmosphere ends and the main body of the Sun begins

The Suns outermost layer begins about 10000 miles above the visible surface and goes outward for millions of miles This is the only part of the Sun that can be seen during an eclipse such as the one in February 1979 At any other time the corona can be seen

bullbullbullbullbullbullbull

bull bullbullbullbullbullbull

CHAPTER 4 Standardized Testing 91

only when special instruments are used on cameras and telescopes to block the light from the photosphere

The corona is a brilliant pearly white filmy light about as bright as the full Moon Its beautiful rays are a sensational sight during an eclipse The coronas rays flash out in a brilliant fan that has wispy spikelike rays near the Suns north and south poles The corona is generally thickest at the Suns equator The corona is made up of gases streamshying outward at tremendous speeds that reach a temperature of more than 2 million deshygrees Fahrenheit The gas thins out as it reaches the space around the planets By the time the gas of the corona reaches the Earth it has a relatively low density

When you have finished reading the passage you will use the mouse to click on Proceed Then the questions about the passage will be presented You are to choose the one best anshyswer to each question Answer all questions about the information in a passage on the basis ofwhat is stated or implied in that passage Most ofthe questions will be multiple-choice questions To answer these questions you will click on a choice below the question

With what topic is paragraph 2 mainly concerned

o How the Sun evolved o The structure of the Sun o Why scientists study the Sun o The distaflce of the Sun from the planets

Paragraph 2 is marked with an arrow (~)

You will see the next question after you click on Next

To answer some questions you will click on a word or phrase Here is an example

Look at the word one in the passage Click on the word or phrase in the bold text that one refers to To answer you can click on any part of the word or phrase in the passage Jour choice will darken to show which word you have chosen

The Suns outermost layer begins about 10000 miles above the visible surface and goes outward for millions of miles This is the only part of the Sun that can be seen durshying an eclipse such as the one in February 1979 At any other time the corona can be seen only when special instruments are used on cameras and telescopes to block the Iight from the photosphere

You will see the next question after you click on~ To answer some q~estions you will click on a sentence in the passage Here is an example

~ The corona is a brilliant pearly white filmy light about as bright as the full Moon Its beautiful rays are a sensational sight during an eclipse The coronas rays flash out in a brilliant fan that has wispy spikelike rays near the Suns north and south poles The corona is generally thickest at the Suns equator ~ The corona is made up of gases streaming outward at tremendous speeds that reach a temperature of more than 2 million degrees Fahrenheit The gas thins out as it reaches the space around the planets By the time the gas of the corona reaches the Earth it has a relatively low density

bull bullbullbullbullbullbull

92 CHAPTER 4 Standardized Testing

Click on the sentence in paragraph 4 or 5 in which the author compares the light of the Suns outermost layer to that ofanother astronomical body Paragraphs 4 and 5 are marked with arrows (~)

To answer some questions you will click on a square to add a sentence to the passage Here is an example -The following sentence can be added to paragraph 1

At the center of the Earths solar system lies the Sun

Where would it best fit in paragraph I Click on the square to add the sentence to the paragraph

D The temperature of the Sun is over 10000 degrees Fahrenheit at the surface but it rises to perhaps morethan 27000000deg at the center 0 The Sun is so much hotter than the Earth that matter can exist only as a gasi except p~rHapsatth~ c6relp the c~re of the ii Sun the pressures are so great that despite the high temperature there may be a small solid core D However no one really knows since the center of the Sun can never be directly observed D 0100

When you click on a square the sentence will appear in the passage at the place you have chosen You can read the sentence added to the paragraph to see if this is the best place to add it You can click on another square to change your answer The sentence will be added and shown in a dark box

Writing In this section you will have an opportunity to demonstrate your ability to write in Enshyglish This includes the ability to generate and organize ideas to support those ideas with examples or evidence and to compose in standard written English in response to an asshysigned topic You will have 30 minutes to write your essay on that topic You must write on the topic you are assigned An essay on any other topic will receive a score of 0 Read the topic below and then make any notes that will help you plan your response Begin typing your response in the box at the bottom of the screen or write your answer on the answer sheet provided to you

Following is a sample topic

Do you agree or disagree with the following statemenH

Teachers should make learning enjoyable and fun for their students

Use specific reasons and examples to support your opinion

CHAPTER 4 Standarczed Testing 93

Michigan English Language Assessment Battery (MELAB)

Composition The time limit for the composition is 30 minutes You must write on only one of the top~

ics below If you write about something else your composition paper will not be graded and you cannot be given a final score If you do not understand the topics ask the exam~ iner to explain or to translate them You may be asked to give your opinion ofsomething and explain why you believe this to describe something from your experience or to exshyplain a problem and offer possible solutions You should write at least one page Some sample topics are

1 What do you think is your countrys greatest problem Explain in detail and tell what you think can be done about it

2 What are the characteristics of a good teacher Explain and give examples 3 An optimist is someone who sees the good side of things A pessimist sees the

bad side Are you an optimist or a pessimist Relate a personal experience that shows this

4 In your opinion are the benefits of space exploration really worth the enormous costs Discuss

Most MELAB compositions are one or two pages long (about 200-300 words) If your paper is extremely short (less than 150 words) your composition will be given a lower score Before you begin writing you might want to take 2 or 3 minutes to plan your comshyposition and to make a short outline to organize your thoughts Such outlines will not be graded they are only to help you You should use the last 5 minutes to read through your composition and to make changes or corrections

Your composition will be graded on how clearly you express yourself in English and on the range of English you are able to use and your control in doing so This means your composition should be well organized your arguments should be fully developed and you should show a range ofgrammatical structures and broad vocabulary Compositions that consist only of very short sentences and very simple vocabulary cannot be given the

middothighest scores If errors are not frequent and if they do not confuse your meaning they will not lower your score very much

Listening Now you will hear a short lecture You may take notes during the lecture Following the lecture you will be asked some questions about it

Therell be a two-week exhibit of the paintings of the little-known master Laura Bernhart at the Claire Osmond Galleries starting on the fifteenth of the month and running through the thirtieth Bernharts known for her innovative designs in abstract expressionism Though a true original she declared a spiritual heritage from Salvador Dali the famous Spanish painter Since Bernhart lived a rather solitary life and died while only in her twenties few people are aware of her works This showing at the Osmond Galleries will provide many with an introduction to her works

10 Where is the exhibit a the Art Museum b the Dali Galleries c the Osmond Galleries

94 CHAPTER 4 Standardized Testing

11 What is Bernhart known for a her copies of Dalis paintings b the originality of her designs c her exhibitions

12 What will going to the exhibit allow most people to do a to see Saivador Dalis paintings b to see Bernharts works for the first time c to learn about Spanish art

Grammar

1 What did the teacher just tell you

She reminded our notebooksI a us to bring b that we bring c our bringing d we should bring

2 Is Bill a good dancer

Not really __ he tries very hard a in spite of h despite c even though d while

3 your clothes are all wet1

Yes I didnt come __ the rain soon enough a away to b over to c down with d in from

Cloze In years to come zoos will not only be places where animals are exhibited to the public but repositories where rare species can be saved from extinction (7) captive breeding The most powerful force (8) the future of many animals-and of zoos-is the decline of the wild (9) even zoo directors would argue that (10) are better places for animals than the fields and forest of their native (11) yet zoos may be the last chance for some creatures that would otherwise pass qUietly into oblivion

7 a through c from b of d damage

8 a bringing c to b that d influencing

9 a But c Not b So d Then

10 a where c even b zoos d wilds

11 alands c residence b life d field

CHAPTER 4 Standardized Testing 95

Vocabulary

12 Mark has a flair for writing a need b purpose c talent d dislike

13 Bill Collins launched his restaurant last June a moved b started c sold d bought

14 John will not accept the censure a burden b blame c credit d decision

15 I cant think of the answer Can you give me a __ a hint b token c taste d gaze

16 Because fewer people are taking expensive vacations the tourist industry is in a a choke b grope c grumble d slump

17 I disagree with a few of his opinions but __ we agree a deliberately b conclusively c essentially d immensely

Reading The influenza virus is a single molecule built from many millions of single atoms You must have heard of the viruses which are sometimes called living molecules While bacteria can be considered as a type of plant secreting pOisonous substances into the body of the organism they attack viruses are living organisms themselves We may conshysider them as regular chemical molecules since they have a strictly aefined atomic strucshyture but on the other hand we must also consider them as being alive since they are able to multiply in unlimited quantities

18 According to the passage bacteria are a poisons

b larger than viruses c very small d plants

96 CHAPTER 4 Standardized Testing

19 The writer says that viruses are alive because they a have a complex atomic structure b move c multiply d need warmth and light

20 The atomic structure of viruses a is -tJIariable b is strictly defined c cannot be analyzed chemically d is more complex than that of bacteria

International English Language Testing System (fELTS)

I

listening

The Listening Module has four sections The first two sections are concerned with social needs There is a conversation between two speakers and then a monologue For examshyple a conversation about travel arrangements or decisions on a night out and a speech about student services on a university campus or arrangements for meals during a confershyence The final two sections are concerned with situations related more closely to educashytional or training contexts For example conversation between a tutor and a student about an assignment or between three students planning a research project and a lecture or talk ofgeneral academic interest All the topics are ofgeneral interest and it makes no difference what subjects candidates study Tests and tasks become more difficult as the sections progress A range of English accents and dialects are used in the recording which reflects the international usage of IELTS

Academic Reading [A 7S0-word article on-th-e- topic of Wind Power in the US with a short glossary at the end]

Questions 1-5

Complete the summary below

Choose your answers from the box below the summary and write them in boxes 1-5 on your answer sheet Note There are more words or phrases than you will need to fill the gaps You may use any word or phrase more than once

Example The failure during the late 1970s and early 19805 of an attempt to establish a widespread wind power industry in the United States resulted largely from the (1) bull in oil prices during this period The industry is now experiencing a steady (2) due to improveshyments in technology and an increased awareness of the potential in the power of wind The wind turbines that are now being made based in part on the (3) of wide- ranging research in Europe are easier to manufacture and maintain than their predecesshysors This has led wind-turbine makers to be able to standardise and thus minimize (4) There has been growing (S) of the importance of wind power as an energy source

CHAPTER 4 Standardized Testing 97

criticism stability skepticism success operating costs decisions design costs fall effects production costs growth decline failure recognition results

Questions 6-1 0 Look at the following list of issues (Questions 6-10) and implications (A-C) Match each issue with one implication Write the appropriate letters A-C in boxes 6-10 on your anshyswer sheet

Example The current price of one wind-generated kilowatt Answer

6 The recent installation of systems taking advantage of economies of scale

7 The potential of meeting one fifth of current U5 energy requirements by wind power

8 The level of acceptance of current wind turbine technology

9 A comparison of costs between conventional and wind power sources

10 The view of wind power in the European Union

Implications

A provides evidence against claims that electricity produced from wind power is relatively expensive

B supports claims that wind power js an important source of energy

C opposes the view that wind power technology requires further-development

General Training Reading Read the passage on Daybreak trips by coach and look at the statements below On your answer sheet write

TRUE if the statement is true FALSE jf the statement is false

NOlGIVEN if the information is not given in the leaflet

1 MiIlers Coaches owns Cambridges Cam bus fleet

2 Premier is an older company than Millers

3 Most of the Daybreak coaches are less than 5 years old

4 Daybreak fares are more expensive than most of their competitors

5 Soft drinks and refreshments are served on most longer journeys

6 Smoking is permitted at the rear of the coach on longer journeys

7 Tickets must be bought in advance from an authorised Daybreak agent

6 Tickets and seats can be reserved by phoning the Daybreak Hotline

9 Daybreak passengers must join their coach at Cambridge Drummer Street

10 Daybreak cannot guarantee return times

98 CHAPTER 4 Standardized Testing

FROM CAMBRIDGE AND SURROUNDING AREA

SPRING IS INTHEAIR

Welcome to our Spring Daybreak programme which continues the tradition of offering unbeatable value for money day trips and tours All the excursions in this brochure will be operated by Pr~mier Travel Services Limited or Millers Coaches both companies are part of the CHLGroup owners of Cambridges Cambus fleet

WERE PROUD OF OUR TRADITION

Premier was established in 1936 the Company now offers the highest standards of coaching in todays competitive operating environment Miller has an enviable reputation stretching back over the past 20 years offering coach services at realistic prices Weve traveled a long way since our early days of pre-war seaside trips Now our fleet of 50 modern coaches (few are more than five years old) operate throughout Britain and Europe but were pleased to still maintain the high standards of quality and service the trademark of our founders nearly sixty years ago

EXCLUSIVE FEATURES

Admission-inclusive fares All Daybreak fares (unless specifically otherwise stated) include admission charges to the attractions shows and exhibits we visit Many full-day scenic tours are accompanied by a fully trained English Tourist Board Blue Badge guide or local experienced driverguide Some Daybreaks include lunch or afternoon tea Compare our admission inclusive fares and see how much you save Cheapest is not the best and value for money is guaranteed If you compare our bargain Daybreak fares beware--most of our competishytors do not offer an all-inclusive fare

SEAT RESERVATIONS

We value the freedom of choice so you can choose your seat when you book The seat reservation is guaranteed a-nd remains yours at all times when aboard the coach

NO SMOKING COMFORT

With the comfort of our passengers in mind coaches on all our Daybreaks are no smokshying throughout In the interests of fellow passengers comfort we kindly ask that smokers observe our no smoking policy On scenic tours and longer journeys ample refreshment stops are provided when of course smoking is permitted

YOUR QUESTIONS ANSWERED

Do I need to book Booking in advance is strongly recommended as all Daybreak tours are subject to demand Subject to availability stand-by tickets can be purchased from the driver

What ti me does the coach leave The coach departs from Cambridge Drummer Street (Bay 12 adjacent to public toilets) at the time shown There are many additional joining points indicated by departure codes in the brochure If you are joining at one of our less popular joining points you will be adshyvised of your pick-up time (normally by telephone) not less than 48 hours before deparshyture In this way we can minimize the length of pick-up routes and reduce journey times for the majority of passengers

CHAPTER 4 Standardized Testing 99

What time do we get back An approximate return time is shown for each excursion The tim~s shown serve as a guide but road conditions can sometimes cause delay If your arrival will be later than advertised your driver will try to allow for a telephone call during the return journey

Where can I board the coach All the Daybreaks in the brochure leave from Cambridge Drummer Street (Bay 12 adjashycent to public toilets) at the time shown Many Daybreaks offer additional pick-ups for pre-booked passengers within Cambridge and the surrounding area This facility must be requested at the time of booking

Academic Writing Writing Task 1 You should spend about 20 minutes on this task

The graph below shows the different modes of transport used to travel to and from work in one European city in 1950 1970 and 1990

[graph shown here]

Write a report for a university lecturer describing the information shown below You should write at least 150 words

Writing Task 2 You should spend about 40 minutes on this task

Present a written argument or case to an educated reader with no specialist knowledge of the folowing topic

It is inevitable that as technology develops so traditional cultures must be lost Technolshyogy and tradition are incompatible-you cannot have both together

To what extent do you agree or disagree with this statement Give reasons for your answer You should write at least 250 words You should use your own ideas knowlshyedge and experience and support your arguments with examples and relevant evidence

General Training Writing Writing Task 1 You should spend about 20 minutes on this task You rent a house through an agency The heating system has stopped working You phoned the agency a week ago but it has still not been mended Write a letter to the agency Explain the situation and teil them what you want them to do about it

You should write at least 150 words You do NOT need to write your own address

Begin your letter as follows

Dear - ___-I

Writing Task 2 You should spend about 40 minutes on this task As part ofa class assignment you have to write about the following topic

100 CHAPTER 4 Standardized Testing

Some businesses now say that no one can smoke cigarettes in any of their offices Some governments have banned smoking in all public placesThis is a good idea but it takes away some of our freedom

Do you agree or disagree Give reasons for your answer You should write at least 250 words

Speaking In each ofthe three parts of the speaking module a specific function is fulfilled In Part 1 the candidates answer general questions about themselves their homes or families their jobs or studies their interests and a range ofsimilar familiar topic areas This part lasts between four and five minutes In Part 2 the candidate is given a verbal prompt on a card and is asked to talk on a particular topic The candidate has one minute to prepare before speaking at length for between one and two minutes The examiner then asks one or two wind-down questions In Part 3 the examiner and candidate engage in a discusshysion of more abstract issues and concepts which are thematically linked to the topic prompt in Part 2 The discussion lasts between four and five minutes

All interviews are recorded on audiocassette Here is a sample ofa Part 2 topic

Describe a teacher who has greatly influenced you in your education

You shou Id say

where you met them what subject they taught what was special about them

and explain why this person influenced you so much

You will have to talk about the topic for 1 to 2 minutes You have 1 minute to think about what you are going to say You can make some notes if you wish

Test of English for International Communication (TOEICreg)

listening

Part 1 Photographs Directions For each question you will see a picture in your test book and you will hear four short statements The statements will be spoken just one time They will not be printed in your test book so you must listen carefully to understand what the speaker says When you hear the four statements look at the picture in your test book and choose the statement that best describes what you see in the picture Then on your answer sheet find the number of the question and mark your answer

[photograph of a scientist looking through a microscope]

You will hear Look at the picture marked number 1 in your test book

(A) Shes speaking into a microphone (B) Shes put on her glasses (C) She has both eyes open (D) Shes using a microscope

CHAPTER 4 Standardized Testing 101

Part 2 Question-Response Directions In this part of the test you will hear a question or statement spoken in Enshyglish followed by three responses also spoken in English The question or staten1ent and the responses will be spoken just one time They will not be printed in your test book so you must listen carefully to understand what the speakers say You are to choose the best response to each question or statement

Question 1 You will hear Ms Morikawa has worked here for a long time hasnt she

(A) At three oclock (B) No Ive lost my watch (C) More than ten years

Question 2 You will hear Which of these papers has a wider circulation

(A) The morning edition (B) Get more exercise (C) By messenger

Part 3 Short Conversations Directions In this part of the test you will hear short conversations between two people The conversations will not be printed in your test book You will hear the conversations only once so you must listen carefully to understand what the speakers say In your test book you will read a question about each conversation The question will be followed by four answers You are to choose the best answer to each question and mark it on your answer sheet

Question 1 (Man) We should think about finding another restaurant for lunch (Woman) Why The food and service here are great

(Man) Yes but the prices are going up every week

You will read Why is this man unhappy with the restaurant

(A) It is too noisy (B) It is too expensive (C) It is too crowded (D) It is too difficult to find

Question 2 (Woman A) How was Dr Borgs recent trip to Singapore (Woman B) She enjoyed the tour of the port very much (Woman A) They say its one of the most active in Asia

You will read 2 What did Dr Borg find interesting

(A) The tourist center (B) The airport (C) The musical performance (D) The harbor

Part 4 Short Talks Directions In this part of the test you vill hear several short talks Each will be spoken just one time They will not be printed in your test book so you must listen carefully to understand and remember what is said In your test book you will read two or more questions about each short talk The questions will be followed by four answers You are to choose the best answer to each question and mark it on your answer sheet

102 CHAPTER 4 Standardized Testing

You will hear Questions 1 and 2 refer to the following announcement

Good afternoon and welcome aboard Nordair Flight 857 from Copenhagen to Bangkok with intermediate stops in Dubai and Calcutta We are preparing for departure in a few minutes At this time your seat back should be returned to its full upright position and your seat belt s~ould be fastened OUf anticipated total flying time to Dubai is six hours and twenty-five minutes I hope you enjoy the flight You will hecJr Now read question 1 in your test book and answer it You will read 1 What is the final destination of the flight

(A) Bangkok (B) Copenhagen (C) Dubai (O) Calcutta

You will hear Now read question 2 in your test book and answer it You will read 2 What will happen in a few minutes

(A) The flight will land in Dubai I

(B) The passengers will board the plane (C) The plane will take off (0) The gate number will be announced

Reading In this section of the test you will have the chance to show how well you understand written English There are three parts to this section with special directions for each part

Part 4 Incomplete Sentences Directions This part of the test has incomplete sentences Four words or phrases marked (A) (8) (e) (D) are given beneath each sentence You are to choose the one word or phrase that best completes the sentence Then on your answer sheet find the number of the question and mark your answer

1 Mr Yangs trip will __ him away from the office for ten days (A) withdraw (B) continue (C) retain (0) keep

2 The company that Marie DuBois started now sells __ products throughout the world (A) its (B) it (C) theirs (D) them

3 If your shipment is not delivered __ Tuesday you can request a full refund for the merchandise (A) at (B) by (C) within (D) while

CHAPTER 4 Standardized Testing 103

Part 6 Error Recognition Directions In this part ofthe test each sentence has four words or phrases underlined The four underlined parts of the sentence are marked (A) (B) (C) (D) You are to identify the one underlined word or phrase that should be corrected or rewritten Then on your answer sheet find the number of the question and mark your answer

1 The pamphlet contains some importance information about the current exhibit ABC D

2 No matter how Jong it taking to finish the annual report it must be done properly ABC D

3 The popularity of jogging appears to have decreased since the past couple of years ABC D

Part 7 Reading Comprehension Directions The questions in this part of the test are based on a selection of reading mateshyrials such as notices letters) forms newspaper and magazine articles) and advertisements You are to choose the one best answer (A) (B) (C) or (OJ to each quesshytion Then on your ariswefsheelfindthe number of the qUestion andmcirkyour answer Answer all questions following each reading selection on thebasis of what is stated or implied in that selection

The Museum ofTechnology is a hands-on museum designed for people to experience science at w()rk~ Visitors are encouraged to use test and handle the objects o~ display Special demonstrations are scheduled for the first and second Wednesdays of each month at 1330 Open Tuesday-Friday 1200-1630 Saturday 1000-1730 and Sunday 11 00-1630

1 When during the month can visitors see special demonstrations (A) Every weekend (B) The first two Wednesdays (C) One afternoon a week (D) Every other Wednesday

Questions 2 and 3 refer to the followi ng notice

NOTICE If you are unable to work because of an extended illness or injury that is not workshyrelated you may be entitled to receive weekly benefits from your employer or the firms insurance company To claim benefits you must file a claim form within thirty days of the first day of your disability Before filing the claim you must ask your doctor to fill in the Doctors Statement on the claim form stating the period of disability

3 To whom is this notice addressed (A) Employers (8) Doctors (C) Employees (D) When paying the bill

4 When must the claim form be filed (A) On the first of the month (8) On the thirtieth of the month (C) On the first day ofdisabifity (D) Within 30 days of the start of disability

Page 10: Standardized Testing Chapter 4 Brown

CH4PTER 4 Standardized Testing 75

(e) what really happened to the Whidahs pirates (D) why people go to the beach

12 It is NOT mentioned in the passage that pirates did which of the following (A) They killed lots of people (B) They robbed other ships (e) They took things from towns (D) They gathered big treasures

13 The word amassed in line 4 is closest in meaning to (A) sold (e) transported (B) hid (D) gathered

14 It is implied in the passage that the Whidahs crew (A) died (B) went diving (e) searched for the treasure (D) escaped with parts of the treasure

15 Which of the following is NOT mentioned as part of the treasure of the Whidah (A) Art objects (B) Coins (e) Gold and si Iver (D) Jewels

16 The word estimated in line 10 is closest in meaning to which of the following (A) Known (C) Approximate (B) Sold (D) Decided

17 The passage indicates that the cargo of theWhidah is worth about (A) $100000 (B) $1000000 (C) $10000000 (D) $100000000

18 The work that Barry Clifford did to locate the Whidah was NOT (A) successfu I (B) effortless (C) detailed (D) lengthy

19 It is mentioned in the passage that the treasure of the Whidah (A) is not very valuable (8) is all in museums (C) has not all been found (D) was taken to share by the pi rates

20 The paragraph following the passage most likely discusses (A) what Barry Clifford is doing today (8) the fate of the Whidahs crew (e) other storms in the area of Cape Cod (D) additional pieces that turn up from the Whidahs treasure

76 CHAPTER 4 Standardized Testing

As you can see items target the assessment of comprehension of the main idea (item 11) stated details (17 19) unstated details (12 15 18) implied details (14 20) and vocabulary in context (13 16) An argument could be made about the cultural schemata implied in a passage about pirate ships and you could engage in an angels on the head of a pin argument about the importance of picking cershytain vocabulary for emphasis but every test item is a sample of a larger domain and each of these fulfills its designated specification

Before any such items are released into a form of the TOEFL (or any validated standardized test) they are piloted and sCientifically selected to meet difficulty specshyifications within each subsection section and the test overall Furthermore those items are also selected to meet a desired discrimination index Both of these indices are important considerations in the design of a computer-adaptive test where pershyformance on one item determines the next one to be presented to the test-taker (See Chapter 3 for a complete treatment of multiple-choice item design)

(B)The selection of items in the ESLPT entailed two-entirel) different processes In the two subsections of -the test that elicit writing performance (summary of reading response to reading) the main hurdles were (a) selecting appropriate passhysages for test-takers to read (b) providing appropriate prompts and (c) processing data from pilot testing Passages have to conform to standards of content validity by being within the genre and the difficulty of the material used in the courses The prompt in each case (the section asking for a summary and the section asking for a response) has to be tailOred to fit the passage but a general template is used

[n the multiple-choice editing test that seeks to test grammar proofreading ability the first and easier task is to choose an appropriate essay within which to embed errors The more complicated task is to embed a specified number of errors from a previously determined taxonomy of error categories Those error categories came directly from student errors as perceived by their teachers (verb tenses verb agreeshyment logical connectors articles etc) The disttactors for each item were selected from actual errors that students make Itemsiti pilot versions were then coded fordifshyficulty and discrinlination indices after which final assembly of items could occur

(C) The GET prompts are designed by a faculty committee of examiners who are speCialists in the field of university academic writing The assumption is made that the topics are universally appealing and capable of yielding the intended product of an essay that requires an organized logical argument and conclusion No pilot testing of prompts is conducted The conditions for administration remain constant two-hour time limit sit-down context paper and pencil closed-book format Consider the following recent prompt

Graduate Essay Test sample prompt

In the Middletown Elementary School District the assistant superintendent has just been made superintendent in another district Her resignation leaves vacant the districts only administrative position ever held by a woman The School Board in response to strong

CHAPTER 4 Standardized Testing 77

arguments from the Teachers Association has urged that a woman be hired to replace her As a member of the hiring committee you must help choose her successor

Only one woman applicant meets the written qualifications for the job the two top male applicants are both more experienced than she

The hiring committee has asked each committee member to prepare a written statement to distribute before meeting together to discuss the issue Write a report that represents your position making it as logical and persuasive as possible

Some facts you may wish to draw on 1 Women make up more than 75 percent of classroom teachers but hold fewer than

10 percent of administrative positions in education Administrators salaries average 30 percent more than teachers salaries

2 The local Teachers Association is 89 percent women mostly under 40 In a heated debate on television a member of the National Organization of Women (NOW) and the chair of the Teachers Association threatened if a man is hired to bring a class-action suit against the district on behalf of all women teachers who cannot expect advancement because of discriminatory hiring practices

3 The local Lions Club which contributes heavily to school sports says hiring the less experienced woman would not be in the best interests of the schoolthe children or the teachers

The finalists for the position

1 Carole Gates Classroom teacher 10 years Teacher of the Year 1985 supervisor ofpractice teachers at Teachers College former president of Teachers Associ ati on Administrative Credential 1984 EdD degree 1986 assistant principal of Hoptown Elementary School 2 years

2 Spud Stonewall Principal of Middletown Elementary 15 years PhD in educational adminis~ration State Board of Education Committee for Improving Elementary School Curriculum 1982-present

3 Jim Henderson School Administrator 22 yearsgradesK-9-supports innovation in education Fair Bargaining Award 1981 former coach for winning collegiate basketball team 10 years

It is clear from such a prompt that the problem the test-takers must address is complex that there is sufficient information here for writing an essay and that testshytakers will be reasonably challenged to write a clear statement of opinion What also emerges from this prompt (and virtually any prompt that one might propose) is the potential cultural effect on the numerous international students who must take the GIIT Is it possible that such students who are not familiar with school systems in the United States with hiring procedures and perhaps with the politics of school board elections might be at a disadvantage in mounting their arguments within a two-hour time frame Some (such as Hosoya 2001) have strongly claimed such a bias

78 CHAPTER 4 Standardized Testing

4 Make appropriate evaluations of different kinds of items

In Chapter 3 the concepts of item facility (IF) item discrimination (ID) and disshytractor analysis were introduced As the discussion there showed such calculations provide useful infornlation for classroom tests but sometimes the time and effort involved in perfornling them may not be practical especially if the classroom-based test is a one-time test Yet for a standardized multiple-choice test that is designed to be marketed commercially andor administered a number of times andor adminisshytered in a different form these indices are a must

For other types of response formats namely production responses different forms of evaluation become importantThe principles of p-mpoundti~ality ~d poundabWty are prominent along with the concept o(JacjJjt Practicality issues in such items include the clarity of directions timing of the test ease of administration and how much time is required to score responses Reliability is a major player in instances where more than one scorer is employed and to a lesser extent when a single scorer has to evaluate tests over long spans of time that could lead to deterioration of stanshydards Facility is also a key to the validity and success of an item type ~irecshytions complex- language obscure topics fuz~Qata and culturally biased

~Jfiformatioifma~alliead to a highei1eVermiddotof diffiCidty than one desires (A) The IF ID and efficiency statistics of the multiple-choice items of current

forms of the TOEFL are not publicly available information For reasons of security and protection of patented copyrighted materials they must remain behind the closed doors of the ETS development staff Those statistics remain of paramount importance in the ongoing production ofTOEFL items and forms and are the founshydation stones for demonstrating the equatability of forms Statistical indices on retired forms of the TOEFL are available on request for research purposes

The essay portion of theTOEFL undergoes scrutiny for its practicality reliability and facility Special attention is given to reliabilIty since two human scorers must read each essay and every time a third reader becomes necessary (when the two readers disagree by more than one point) it costs ETS more money

(B) In the case of the open-ended responses on the two written tasks on the ESLPT a similar set of judgments must be made Some evaluative impressions of the effectiveness of prompts and passages are gained from informal student and scorer feedback In the developmental stage of the newly revised ESLPT both types of feedshyback were formally solicited through questiQnnaires and interviews That informashytion proved to be invaluable in the revisIon of prompts and stimulus reading passages After each administration now the teacher-scorers provide informal feedshyback on their perceptions of the effectiveness of the prompts and readings

The multiple-choice editing passage showed the value of statistical findings in determining the usefulness of items and pointing administrators toward revisions Following is a sample of the format used

CHAPTER 4- Standardized Testing 79

Multiple-choice editing passage

(1)EYer since supermarkets first appeared they have beentake over ~ world ABC 0

(2) Supermarkets have changed peoples life ~ yet and at the same time changes in ABC

peoples life ~ have encouraged the opening of supermarkets o

The task was to locate the error in each sentence Statistical tests on the experishymental version of this section revealed that a number of the 45 items were found to be of zero IF (no difficulty whatsoever) and of inconsequential discrimination power (some IDs of 15 and lower) Many distractors were of no consequence because they lured no one Such information led to a revision of numerous it~ms and their options eventually strengthening the effectiveness of this section

(C)The GET like its written counterparts in the ESLPT is a test ofwritten ability with a single prompt and therefore questions of practicality and J~~illy~are also largely observational No data are collected from students on their perceptions but the scorers have an opportunity to reflect on the validity ofa given topiC After one sitting a topic is retired which eliininates the possibility of improving a specific topiC but future framing of topics might benefit from scorers evaluations Inter-rater reliability is checked periodically and reader training sessions are modified if too many instances of unreliability appear

5 Specify scoring procedures and reporting formats - ---

A systematic assembly of test items in pre-selected arrangements and sequences all of which are validated to confo~ to an e~pected difficulty level should yield a test that can then be scored accurately and reported back to test-takers and institutions efficiently

(A) Of the three tests being exemplifled here the most straightforward scoring procedure comes from the TO~FL the one with the most complex issues of validashytion deSign and assembly Scores are calculated and reported fora) three sections of the TOEFL (the essay ratings are combined with the Structure and Written Expression score) and (b) a total score (range 40 to 300 on the computer-based TOEFL and 310 to 677 on the paper-and-pencil TOEFL) A separate score (c) for the Essay (range 0 to 6) is also provided on the examinees score record (see simulation of a score record on page 80)

80 CHAPTER 4 Standardized Testing

Facsimile of a TOEFLreg score report

TOEFL Scaled Scores Claudia Y Estudiante Peru ___

19 17 17 177 Listening Structure Writing Reading Total Score

Essay rati ng 30

The rating scale for the essay is virtually the same one that is used for the Test of Written English (see Chapter 9 for details) with a zero level added for no response copying the topic only writing completely off topic or not writing in English

(B) The ESLPT reports a score for each of themiddot essay sections but the rating scale differs between them because in one case the objective is to write a summary and in the other to write a response to a reading ~ch essayi~pd lgtY ~o readet~ ifhFfF js a discrepancy of more than one level a third reader1resolves the differenceThe ~ditiilg section is machine-scanned and -scored with a total score and ~th part-scores for each ofthe grammaticaVrhetorlcal sectionS From these data placement administrators have adequate information to make placements and teachers receive some diagnostic inforshymation on each student in their classes Students do not receive their essays back

(C) Each GET is read by two trained readers who give a score between 1 and 4 according to the following scale

Graduate Essay Test Scoring Guide

Please make no marks on the writers work Write your reader number and score on the front cover of each test booklet

4 Superior The opening establishes context purpose and point of view the body of the essay developsmiddot recommendations-logically and coherently The writer demonshystrates awareness of the complexities in the situation and provides analysis of the probJem offers compelling or common-sense reasons for recommendations made makes underlying assumptions explicit

The writer uses fluent and idiomatic English with few mechanical errors Style reshyveals syntactic maturity is dear and direct is not choppy or over-colloquial nor over-formal stuffy or unfocused Occasional spelling or punctuation errors may be easily attributed to hasty transcription under pressure

3 Competent After an opening that establishes context and purpose the paper unfolds with few lapses in coherence but may have somewhat less clear organization of less explicit transitions than a top-score paper It may have somewhat less compelling logic or slightly less-wellreasoned suggestions than a 4 paper though it will provide reasons for the recommendations made

The writer uses dear fluent and generally idiomatic English but may make minor or infrequent ESL errors (preposition errors dropped articles or verb endings etc) or repeat a single error (eg not punctuate possessive nouns) Occasional lapses of style are offSet by demonstrated mastery of syntax

CHAPTE84 Standardized Testing 81

2 Weak The writer makes somewhat simplistic suggestions not fully supported with reashysons fails to cite key facts offers little analysis of the problem or shows a limited grasp of the situation the given information is copied or listed withlittle integration into argument Points may be random or repetitious Writing may be badly focused with careless use of abstract language resulting in predication errors or illogical sentences

ESL andlor careless mechanical errors are frequent enough to be distracting OR sentences may be choppy style over-casual usage occasionally unidiomatic

1 Inadequate The essay may be disjointed incoherent or minimally developed The writer shows little grasp of the complex issues involved is unable to establish conshytext point of view or purpose in opening of paper or has a poor sense of audience Mechanical andor ESL errors or unidiomatic usages are frequent sentences may be ungrammatical OR correct but short and very simple

The two readers scores are added to yield a total possible score of 2 to 8 Test administrators recommend a score of 6 as the threshold for allowing a student to pursue graduate-level courses Anything below that is accompanied by a recomshymendation that the student either repeat the test or take a remedial course in gradshyuate writing offered in one of several different departments Students receive neither their essays nor any feedback other than the fmal score

6 Perform ongoing construct validation studies

From the above discussion it should be clear that no standardized instrument is expected to be used repeatedly without a ramporou~program of ongoing c~-sectmct valiltiatiOll Any standardized test once developed must be accompanied by sysshy~

tematic periodic corroboration of its effectiveness and by steps toward its improveshyment This rigor is especially true of tests that are produced in equated forms that is forms must be reliable across tests such that a score on a subsequent form of a test-has-the~same validityand-interpretability as its original

(A) The TOEFL program in cooperation with other tests produced by ETS has an impressive program of research Over the years dozens of TOEFL-sponsored research studies have appeared in the TOEFL Monograph Series An early example ofsuch a study was the seminal Duran et aI (1985) study TOEFLfrom a Communicative ViewpOint on Language Proficiency which examined the content characteristics of the TOEFL from a communicative perspective based on current research in applied linguistics and lanshyguage proficiency assessment More recent studies (such as Ginther 2001 Leacock amp Chodorow 2001 Powers et aI 2002) demonstrate an impressive array of scrutiny

(B) For approximately 20 years the ESLPT appeared to be placing students relishyably by means of an essay and a multiple-choice grammar and vocabulary test Over the years the security of the latter became s1lspect and the faculty administrators wished to see some content validity achieved in the process In the year 2000 that process began with a group of graduate students (Imao et aI 2000) in consl1ltation with faculty members and continued to fruition in the form of a new ESLPT reported in lmao (2002) The development of the new ESlPT involved a lengthy process of

82 CHAPTER 4 Standardized Testing

both content and construct validation along with facing such practical issues as scoring the written sections and a machine scorable multiple-choice answer sheet

The process of ongoing validation will no doubt continue as new forms of the editing section are created and as new prompts and reading passages are created for the writing section Such a validation process should also include consistent checks on placement accuracy and on face validity

(C) At this time there is little or no research to validate the GET itself For its conshy struct validation its administrators rely on a stockpile of research on university-level academic writing tests such as theTWEThe holistic scoring rubric and the topics and administrative conditions of the GET are to some extent patterned after that of the TWE In recent years some criticism of the GEf has come from international test-takers (Hosoya 2001) who posit that the topics and time limits of the GET among other facshytors work to the disadvantage of writers whose native language is not English These validity issues remain to be fully addressed in a comprehensive research study

I I

STANDARDIZED IANGUAGE PROFICIENCY TESTING

Tests of language profiCiency presuppose a comprehensive definition of the specific competencies that comprise overall language ability The specifications for the TOEFL provided an illustration of an operational definition of ability for assessment purposes This is not the only way to conceptualize the concept Swain (1990) offered a multidimensional view of profiCiency assessment by referring to three linshyguistic traits (grammar discourse and sociolinguistics) that can be assessed by means of oral multiple-choice and written responses (see Table 41) Swains conshyception was not meant to be an exhaustive analysis of ability but rather to serve as an operational framework for constructing proficiency assessments

Another defmition and conceptualization of profiCiency is suggested by the ACTFL association mentioned earlier ACfFL takes a holistic and more unitary view of proficiency in describing four levels superior advanced intermediate and noviceWithin each level descriptions of listening speaking reading and writing are provided as guidelines for assessment For example the ACfFL Guidelines describe the superior level of speaking as follows

ACTFL speaking guidelines summary superior-level

Superior-level speakers are characterized by the ability to

bull participate fully and effectively in conversations in formal and informal settings on topics related to practical needs and areas of professional andor scholarly interests

bull provide a structured argument to explain and defend opinions and develop effective hypotheses within extended discourse

bull discuss topics concretely and abstractly bull deal with a linguistically unfamiliar situation bull maintain a high degree of linguistic accuracy bull satisfy the linguistic demands of professional andor scholarly life

CHAPTER4 Standardized Testing 83

The other three ACfFL levels use the same parameters in describing progressively lower proficiencies across all four skills Such taxonomie~ have the advantage of considering a number of functions of linguistic discourse but the disadvantage at the lower levels of overly emphasizing test-takers deficiencies

Table 41 Traits of second language proficiency (Swain 1990 p 403)

Trait Grammar Discourse Sociolinguistic

focus on grammatical focus on textual focus on social accuracy within cohesion and appropriateness of sentences coherence language use

Method

Oral structured interview story telling and argumentationpersuasion

role-play ofspeech acts requests offers complaints

scored for accuracy of verbal morphology prepositions syntax

detailed rating for identification logical sequence and time orientation and global ratings for coherence

scored for ability to distinguish formal and informal register

Multiple-choice

sentence-level select the correct form exercise

paragraph-level select the coherent sentence exercise

speech act-Ievelselect the appropriate utterance exercise

(45 items) (29 items) (28 items)

involving verb morphology prepositionsan-d-uther items

Written composition

narrative and letter of persuasion

narrative and letter of persuasion

formal request letter and informal note

scored for accuracy of verb morphology prepositions syntax

detailed ratings much as for oral discourse and global rating for coherence

scored for the ability to distinguish formal and inforJ1lil1 register

FOUR STANDARDIZED lANGUAGE PROFICIENCY TESTS

We now tum to some of the better-known standardized tests of overall language ability or profiCiency to examine some of the typical formats used in commercially available tests We will not look at standardized tests of other specific skills here but that should not lead you to think by any means that proficiency is the only kind of test in the field that is standardized Three standardized oral production tests the

84 CHAPTER 4 Standardized Testing

Test of Spoken English (fSE) the Oral Proficiency Inventory (OPI) and PbonePassreg are discussed in Chapter 7 and the Test of Written English (WE) is covered in ChapterS

Four commercially produced standardized tests of English language proficiency are described briefly in this section the TOEFL the Michigan English Language Assessment Battery (MELAB) the International English Language Testing System (lELTS) and the Test of English for International Communication (fOEICreg) In an appendix to this chapter are sample items from each section of each test When you turn to that appendix use the following questions to help you evaluate these four tests and their subsections

1 What item types are included 2 How practical and reliable does each subsection of each test appear to be 3 Do the item types and tasks appropriately represent a conceptualizatio~ of

language proficiency (ability) That is can you evaluate their construct validity

4 Do the tasks achieve face validity 5 Are the tasks authentic 6 Is there some washback potential in the tasks

Test of English as a Foreign Language (TOEFL)

Producer Educational Testing Service (ETS) Objective To test overall proficiency (language ability) Primary market Almost exclusively US universities and colleges for admission

purposes Type Computer-based (CB) (and two sections are-computer-adaptive)

A traditional paper-based (PB) version is also available Response modes Multiple-choice responses essay Specifications See the box on pp 72-73 Time allocation Up to 4 hours (CB) 3 hours (PB) Internet access wwwtoeflorg

Comments In the North American context the TOEFL is the most widely used comshymercially available standardized test of proficiency Each year the TOEFL test is adminisshytered to approximately 800000 candidates in more than 200 countries It is highly respected because of the thorough program of ongoing research and development conshyducted by ETS The TOEFLs primary use is to set proficiency standards for international students seeking admission to English-speaking universities More than 4200 academic institutions government agencies scholarship programs and licensingcertification agenshycies in more than 80 countries use TOEFL scores By 2004 the TOEFL will include a secshytion on oral production

CHAPTER 4 Standardi~ed Testing 85

Michigan English Language Assessment Battery (MELAB)

Producer English language Institute University of Michigan Objective To test overall proficiency (language ability) Primary market Mostly US and Canadian language programs and colleges

some worldwide educational settings as well Type Paper-based Response modes Multiple-choice responses essay Time allocation 25 to 35 hours Internet access wwwlsaumicheduelimelabhtm

Specifications The MElAB consists of three sections Part 1 a 3D-minute impromptu essay is written on an assigned topic Part 2 a 25-minute multiple-choice listening comshyprehension test is delivered via tape recorder Part 3 is a 100-item 75-minute multipleshychoice test containing grammar doze reading vocabulary and reading comprehension An oral interview (speaking test) is optional

Comments The Ell at the University of Michigan has been producing the MELAB and its earlier incarnation (Michigan Test of English language Proficiency) since 1961 like the TOEFL it serves a North American audience but is also used internationally While its use is not as widespread as the TOEFL its validity is widely respected Because it is cheaper than the TOEFL and more easily obtained it is popular among language schools and institutes Many institutions and companies accept MElAB scores in lieu ofTOEFL scores

International English Language Testing System (IELTS)

Producer Jointly managed by The University of Cambridge local Examinations Syndicate (UClES) The British Council and lOP Education Australia

Objective To test overall proficiency (language ability) Primary-market Australian British Canadian and New Zealand academic

institutions and professional organizations American academic institutions are increasingly accepting IELTS for admissions purposes

1)rpe Computer-based (for the Reading and Writing sections) papershybased for the listening and Speaking modules

Response modes Multiple-choice responses essay oral production Time allocation 2 hours 45 minutes Internet access httpwwwieltsorgl

httpwwwudesorguk httpwwwbritishcouncilorg

Specifications Reading candidates choose between academic reading or general training reading (60 minutes) Writing the same option academic writing or general training writing (60 minutes) Listening four sections for all candidates (30 minutes) Speaking five sections for all candidates (1015 minutes)

86 CHAPTER 4 Standardized Testing

Comments The University of Cambridge local Examinations Syndicate (UCLES) has been producing English language tests since 1858 Now with three organizations cooperatshying to form the IELTS more than a million examinations are administered every year In 2002 a computer-based version of the Reading and Writing modules of the IELTS became available at selected centers around the world The other sections are administered locally by an examinet The paper-based IELTS remains an option for candidates The IELTS retains the distinct advantage of requiring all four skills in the test-takers performance

Test of English for International Communication (TOEICreg)

Producer The Chauncey Group International a subsidiary of Educational Testing Service

Objelttive To test overall proficiency (langlJage ability) Primary market Worldwide business commerce and industry contexts

(workplace settings) Type Computer-based and paper-based versions Response modes Multiple-choice responses Time allocation 2 hours Internet access httpwwwtoeiccom

Specifications Listening Comprehension 100 items administered by audiocassette Four types of task statements questions short conversations and short talks (approxishymately 45 minutes) Reading 100 items Three types of task cloze sentences error recogshynition and reading comprehension (75 minutes)

Comments The TOEIC has become a very widely used international test of English proficiency in workplace settings where English is required for job performance The conshytent includes many different employment settings such as conferences presentations sales ordering shipping schedules reservations (etters and memoranda It is approprishyate to use in educational settings where vocational or workplace English courses are being offered

sect sect sect sect sect

The construction of a valid standardized test is no minor accomplishment whether the instrument is large- or small-scale The designing of specifications alone as this chapter illustrates requires a sophisticated process of construct valishydation coupled with considerations of practicality Then the construction of items and scoringinterpretation procedures may require a lengthy period of trial and error with prototypes of the final form of the testWith painstaking attention to all the details of construction the end product can result in a cost-effective timeshysaving accurate instrument Your use of the results of such assessments can provide useful data on learners language abilities But your caution is warranted as well for all the reasons discussed in this chapter The next chapter will elaborate on what lies behind that need for a cautious approach to standardized assessment

CHAPTER4 Standardized Testing 87

EXERCISES

[Note (I) Individual work (G) Group or pair work (C) Whole-class discussion]

1 (C) Tell the class about the worst test experience youve ever had Briefly anamiddot lyze what made the experience so unbearable and try to come up with sugshygestions for improvement of the test andor its administrative conditions

2 (G) In pairs or small groups compile a brief list of pros and cons of standardshyized testing Cite illustrations of as many items in each list as possible Report your lists and examples to the rest o~ the class

3 (I) Select a standardized test that you are quite familiar with (probably a recent experience) Mentally evaluate that test using the five principles of practicality reliability validity authenticity and washback Report yourevaluashytion to the class

4 (G) The appendix to this chapter provides sample items from Jour different tests of language proficiency In groups one test for each group analyze your test for (a) content validity (b) face validity and (c) authenticity

5 (C) Do you think that the sample TOEFL reading passage about pirates (pages 74-75) and the Graduate EssayTest prompt (pages 76-77) about a school board hiring committee have any culture bias Discuss this and other cultural biases you have noticed in tests Is it possible to design a test that is completely free of culture bias

6 (CG) Compare the differences in conceptualization of language proficiency represented by Swains model the TOEFL and the ACfFL philosophy Which one best represents current thinking about communicative language ability What are the strengths and weaknesses of each approach

FORYOlIILEURTHER READING

Gronlund Norman E (1998) Assessment of student achievement Sixth Edition Boston Allyn and Bacon

Gronlunds classic also mentioned in Chapter 3 offers a concise overview of features of standardized tests offering definitions and examples of the statistical considerations in interpreting scores His approach is unbiased cleady written and accessible to those who might fear the mathematics of standardized testing

Phillips Deborah 2001 Long1nan introductory course for the TOEFL test White Plains NY Pearson Education

A careful examination of this or any other reputable preparation course for a standardized language test is well worth a students time Note especially how the book acquaints the user with the specifications of the test and offers a number of useful strategie~ that can be llsed in preparation for the test and during irs adn1inistration

88 CHAPTER 4 Standardized Testing

APPENDIX TO CHAPTER 4

Commercial Proficiency Tests Sample Items and Tasks

Test of English a~ a Foreign Language (TOEFLreg)

Listening r

Part A

In this section you will hear short conversations between two people In some ofthe conversations each person speaks only once In other conversations one or both of the people speak more than once Each conversation is followed by one questionabQlt it Each question in this part has four answer choices You should click on the best answer to each question Answer the questions on the basis of what is stated or implied by the speakers Here is an example On the computerscreen you will see

[man and woman talking]

On the recording you will hear

(woman) Hey wheres your sociology book (man) At home Why carry it around when were just going to be taking

a test (woman) Dont you remember Professor Smith said we could us it during

the test (man) Ohl no Well Ive still got an hour right Im so glad I ran into you

You wiII then see and hear the question before the answer choices appear

What will the man probably do next

o Begin studying for the sociology test o Explain the problem to his professor o Go home to get his textbook o Borrow the womans book

To choose an answer you will click on an oval The oval next to that answer will darken After you click on Next and Confirm Answer the next conversation will be presented

Part B

In this section you will hear several longer conversations and talks Each conversation or talk is followed by several questions The conversations talks and questions will not be repeated The conversations and talks are about a variety of topics You do not need speshycial knowledge of the topics to answer the questions correctly Rather you should answer each question on the basis of what is stated or implied by the speakers in the conversashytions or talks

For most of the questions you will need to click on the best of four possible answers Some questions will have special directions The special directions will appear in a box on the computer screen Here is an exampie ot a conversation and some questions

CHAPTER 4 Standardized Testing 89

Marine Biology (narrator) Listen to part of a discussion in a marine biology class

(professor) A few years ago our local government passed a number of strict environmental laws As a result Sunrise Beach looks nothing Ii ke it did ten years ago The water is cleaner and theres been a tremendous increase in all kinds of marine life which is why were going there on Thursday

(woman) I dont know if I agree that the water quality has improved I mean I was out there last weekend and it looked all brown It didnt seem too clean to me

(professor) Actually the color of the water doesnt always indicate whether its polluted The brown color you mentioned might be a result of pollution or it can mean a kind of brown algae is growing there Its called devils apron and it actually serves as food for whales

(man) So when does the water look blue (professor) Well water thats completely unpolluted is actually colorless But

it often looks bluish-green because the sunlight can penetrate deep down and thats the color thats reflected

(woman) But sometimes it looks really green Whats that about (professor) Ok well its the same principle as with devils apron the

water might be green because of different types of green algae there-gulfweed phytoplankton You all should finish reading about algae and plankton before we go In fact those are the types of living things Im going to ask you to be looking for when were there

Now get ready to answer the questions

What is the discussion mainly about

o The importance of protecting ocean environments o The reasons why ocean water appears to be different colors o The survival of whales in polluted water o The effect that colored ocean water has on algae

To choose an answer click on an oval The oval next to that answer will darken After you click on Next and Confirm Answer the next question will be presented

According to the professor what can make ocean water look browngt

o Pollution o Cloudy Skies o Sand o Algae

Click on 2 answers

To choose your answers you will click on the squares An XII wiii appear in each square

bullbullbullbullbullbullbull

90 CHAPTER 4 Standardized Testing

Structure and Written Expression This section measures the ability to recognize language that is appropriate for standard written English There are two types ofquestions in this section In the first type ofquestion there are incomplete sentences Beneath each sentence there are four words or phrases

Directions CIiSk on the one word or phrase that best completes the sentence

The colum~ine flower __ to nearly all of the United States can be raised from seed in almost any garden

native how native is how native is it is native

Time Help Confirm

After you click on Next and Confirm Answ~ the next question willbe presented

The second type of question has four underlined words or phrases You will choose the one underlined word or phrase that must be changed for the sentence to be correct

Directions Click on the one underlined word or phrase that must be changed for the senshytence to be correct

One of the most difficult problems in understanding sleep is determining what the funcshytions of sleep ~

lime Help Confirm

Clicking on an underlined word or phrase will darken it

Reading This section measures the ability to read and understand short passages similar in topic and style to those that students are likely to encounter in North American universities and colleges This section contains reading passages and questions about the passages There are several different types of questions in this section In the Reading section you will first have the opportunity to read the passage

The temperature of the Sun is over 10000 degrees Fahrenheit at the surface but it rises perhaps more than 270000000 at the center The Sun is so much hotter than the Earth that matter can exist only as a gasl except perhaps at the core In the core of the Sun the pressures are so great that despite the high temperature there may be a small solid core However no one really knows since the center of the Sun can never be directly observed ~ Solar astronomers do know that the Sun is divided into five general layers or zones Starting at the outside and going down into the Sun the zones are the corona chromoshysphere hotosphere convection zone and finally the core The first three zones are reshygarded as the Suns atmosphere But since the Sun has no solid surface it is hard to middottell where the atmosphere ends and the main body of the Sun begins

The Suns outermost layer begins about 10000 miles above the visible surface and goes outward for millions of miles This is the only part of the Sun that can be seen during an eclipse such as the one in February 1979 At any other time the corona can be seen

bullbullbullbullbullbullbull

bull bullbullbullbullbullbull

CHAPTER 4 Standardized Testing 91

only when special instruments are used on cameras and telescopes to block the light from the photosphere

The corona is a brilliant pearly white filmy light about as bright as the full Moon Its beautiful rays are a sensational sight during an eclipse The coronas rays flash out in a brilliant fan that has wispy spikelike rays near the Suns north and south poles The corona is generally thickest at the Suns equator The corona is made up of gases streamshying outward at tremendous speeds that reach a temperature of more than 2 million deshygrees Fahrenheit The gas thins out as it reaches the space around the planets By the time the gas of the corona reaches the Earth it has a relatively low density

When you have finished reading the passage you will use the mouse to click on Proceed Then the questions about the passage will be presented You are to choose the one best anshyswer to each question Answer all questions about the information in a passage on the basis ofwhat is stated or implied in that passage Most ofthe questions will be multiple-choice questions To answer these questions you will click on a choice below the question

With what topic is paragraph 2 mainly concerned

o How the Sun evolved o The structure of the Sun o Why scientists study the Sun o The distaflce of the Sun from the planets

Paragraph 2 is marked with an arrow (~)

You will see the next question after you click on Next

To answer some questions you will click on a word or phrase Here is an example

Look at the word one in the passage Click on the word or phrase in the bold text that one refers to To answer you can click on any part of the word or phrase in the passage Jour choice will darken to show which word you have chosen

The Suns outermost layer begins about 10000 miles above the visible surface and goes outward for millions of miles This is the only part of the Sun that can be seen durshying an eclipse such as the one in February 1979 At any other time the corona can be seen only when special instruments are used on cameras and telescopes to block the Iight from the photosphere

You will see the next question after you click on~ To answer some q~estions you will click on a sentence in the passage Here is an example

~ The corona is a brilliant pearly white filmy light about as bright as the full Moon Its beautiful rays are a sensational sight during an eclipse The coronas rays flash out in a brilliant fan that has wispy spikelike rays near the Suns north and south poles The corona is generally thickest at the Suns equator ~ The corona is made up of gases streaming outward at tremendous speeds that reach a temperature of more than 2 million degrees Fahrenheit The gas thins out as it reaches the space around the planets By the time the gas of the corona reaches the Earth it has a relatively low density

bull bullbullbullbullbullbull

92 CHAPTER 4 Standardized Testing

Click on the sentence in paragraph 4 or 5 in which the author compares the light of the Suns outermost layer to that ofanother astronomical body Paragraphs 4 and 5 are marked with arrows (~)

To answer some questions you will click on a square to add a sentence to the passage Here is an example -The following sentence can be added to paragraph 1

At the center of the Earths solar system lies the Sun

Where would it best fit in paragraph I Click on the square to add the sentence to the paragraph

D The temperature of the Sun is over 10000 degrees Fahrenheit at the surface but it rises to perhaps morethan 27000000deg at the center 0 The Sun is so much hotter than the Earth that matter can exist only as a gasi except p~rHapsatth~ c6relp the c~re of the ii Sun the pressures are so great that despite the high temperature there may be a small solid core D However no one really knows since the center of the Sun can never be directly observed D 0100

When you click on a square the sentence will appear in the passage at the place you have chosen You can read the sentence added to the paragraph to see if this is the best place to add it You can click on another square to change your answer The sentence will be added and shown in a dark box

Writing In this section you will have an opportunity to demonstrate your ability to write in Enshyglish This includes the ability to generate and organize ideas to support those ideas with examples or evidence and to compose in standard written English in response to an asshysigned topic You will have 30 minutes to write your essay on that topic You must write on the topic you are assigned An essay on any other topic will receive a score of 0 Read the topic below and then make any notes that will help you plan your response Begin typing your response in the box at the bottom of the screen or write your answer on the answer sheet provided to you

Following is a sample topic

Do you agree or disagree with the following statemenH

Teachers should make learning enjoyable and fun for their students

Use specific reasons and examples to support your opinion

CHAPTER 4 Standarczed Testing 93

Michigan English Language Assessment Battery (MELAB)

Composition The time limit for the composition is 30 minutes You must write on only one of the top~

ics below If you write about something else your composition paper will not be graded and you cannot be given a final score If you do not understand the topics ask the exam~ iner to explain or to translate them You may be asked to give your opinion ofsomething and explain why you believe this to describe something from your experience or to exshyplain a problem and offer possible solutions You should write at least one page Some sample topics are

1 What do you think is your countrys greatest problem Explain in detail and tell what you think can be done about it

2 What are the characteristics of a good teacher Explain and give examples 3 An optimist is someone who sees the good side of things A pessimist sees the

bad side Are you an optimist or a pessimist Relate a personal experience that shows this

4 In your opinion are the benefits of space exploration really worth the enormous costs Discuss

Most MELAB compositions are one or two pages long (about 200-300 words) If your paper is extremely short (less than 150 words) your composition will be given a lower score Before you begin writing you might want to take 2 or 3 minutes to plan your comshyposition and to make a short outline to organize your thoughts Such outlines will not be graded they are only to help you You should use the last 5 minutes to read through your composition and to make changes or corrections

Your composition will be graded on how clearly you express yourself in English and on the range of English you are able to use and your control in doing so This means your composition should be well organized your arguments should be fully developed and you should show a range ofgrammatical structures and broad vocabulary Compositions that consist only of very short sentences and very simple vocabulary cannot be given the

middothighest scores If errors are not frequent and if they do not confuse your meaning they will not lower your score very much

Listening Now you will hear a short lecture You may take notes during the lecture Following the lecture you will be asked some questions about it

Therell be a two-week exhibit of the paintings of the little-known master Laura Bernhart at the Claire Osmond Galleries starting on the fifteenth of the month and running through the thirtieth Bernharts known for her innovative designs in abstract expressionism Though a true original she declared a spiritual heritage from Salvador Dali the famous Spanish painter Since Bernhart lived a rather solitary life and died while only in her twenties few people are aware of her works This showing at the Osmond Galleries will provide many with an introduction to her works

10 Where is the exhibit a the Art Museum b the Dali Galleries c the Osmond Galleries

94 CHAPTER 4 Standardized Testing

11 What is Bernhart known for a her copies of Dalis paintings b the originality of her designs c her exhibitions

12 What will going to the exhibit allow most people to do a to see Saivador Dalis paintings b to see Bernharts works for the first time c to learn about Spanish art

Grammar

1 What did the teacher just tell you

She reminded our notebooksI a us to bring b that we bring c our bringing d we should bring

2 Is Bill a good dancer

Not really __ he tries very hard a in spite of h despite c even though d while

3 your clothes are all wet1

Yes I didnt come __ the rain soon enough a away to b over to c down with d in from

Cloze In years to come zoos will not only be places where animals are exhibited to the public but repositories where rare species can be saved from extinction (7) captive breeding The most powerful force (8) the future of many animals-and of zoos-is the decline of the wild (9) even zoo directors would argue that (10) are better places for animals than the fields and forest of their native (11) yet zoos may be the last chance for some creatures that would otherwise pass qUietly into oblivion

7 a through c from b of d damage

8 a bringing c to b that d influencing

9 a But c Not b So d Then

10 a where c even b zoos d wilds

11 alands c residence b life d field

CHAPTER 4 Standardized Testing 95

Vocabulary

12 Mark has a flair for writing a need b purpose c talent d dislike

13 Bill Collins launched his restaurant last June a moved b started c sold d bought

14 John will not accept the censure a burden b blame c credit d decision

15 I cant think of the answer Can you give me a __ a hint b token c taste d gaze

16 Because fewer people are taking expensive vacations the tourist industry is in a a choke b grope c grumble d slump

17 I disagree with a few of his opinions but __ we agree a deliberately b conclusively c essentially d immensely

Reading The influenza virus is a single molecule built from many millions of single atoms You must have heard of the viruses which are sometimes called living molecules While bacteria can be considered as a type of plant secreting pOisonous substances into the body of the organism they attack viruses are living organisms themselves We may conshysider them as regular chemical molecules since they have a strictly aefined atomic strucshyture but on the other hand we must also consider them as being alive since they are able to multiply in unlimited quantities

18 According to the passage bacteria are a poisons

b larger than viruses c very small d plants

96 CHAPTER 4 Standardized Testing

19 The writer says that viruses are alive because they a have a complex atomic structure b move c multiply d need warmth and light

20 The atomic structure of viruses a is -tJIariable b is strictly defined c cannot be analyzed chemically d is more complex than that of bacteria

International English Language Testing System (fELTS)

I

listening

The Listening Module has four sections The first two sections are concerned with social needs There is a conversation between two speakers and then a monologue For examshyple a conversation about travel arrangements or decisions on a night out and a speech about student services on a university campus or arrangements for meals during a confershyence The final two sections are concerned with situations related more closely to educashytional or training contexts For example conversation between a tutor and a student about an assignment or between three students planning a research project and a lecture or talk ofgeneral academic interest All the topics are ofgeneral interest and it makes no difference what subjects candidates study Tests and tasks become more difficult as the sections progress A range of English accents and dialects are used in the recording which reflects the international usage of IELTS

Academic Reading [A 7S0-word article on-th-e- topic of Wind Power in the US with a short glossary at the end]

Questions 1-5

Complete the summary below

Choose your answers from the box below the summary and write them in boxes 1-5 on your answer sheet Note There are more words or phrases than you will need to fill the gaps You may use any word or phrase more than once

Example The failure during the late 1970s and early 19805 of an attempt to establish a widespread wind power industry in the United States resulted largely from the (1) bull in oil prices during this period The industry is now experiencing a steady (2) due to improveshyments in technology and an increased awareness of the potential in the power of wind The wind turbines that are now being made based in part on the (3) of wide- ranging research in Europe are easier to manufacture and maintain than their predecesshysors This has led wind-turbine makers to be able to standardise and thus minimize (4) There has been growing (S) of the importance of wind power as an energy source

CHAPTER 4 Standardized Testing 97

criticism stability skepticism success operating costs decisions design costs fall effects production costs growth decline failure recognition results

Questions 6-1 0 Look at the following list of issues (Questions 6-10) and implications (A-C) Match each issue with one implication Write the appropriate letters A-C in boxes 6-10 on your anshyswer sheet

Example The current price of one wind-generated kilowatt Answer

6 The recent installation of systems taking advantage of economies of scale

7 The potential of meeting one fifth of current U5 energy requirements by wind power

8 The level of acceptance of current wind turbine technology

9 A comparison of costs between conventional and wind power sources

10 The view of wind power in the European Union

Implications

A provides evidence against claims that electricity produced from wind power is relatively expensive

B supports claims that wind power js an important source of energy

C opposes the view that wind power technology requires further-development

General Training Reading Read the passage on Daybreak trips by coach and look at the statements below On your answer sheet write

TRUE if the statement is true FALSE jf the statement is false

NOlGIVEN if the information is not given in the leaflet

1 MiIlers Coaches owns Cambridges Cam bus fleet

2 Premier is an older company than Millers

3 Most of the Daybreak coaches are less than 5 years old

4 Daybreak fares are more expensive than most of their competitors

5 Soft drinks and refreshments are served on most longer journeys

6 Smoking is permitted at the rear of the coach on longer journeys

7 Tickets must be bought in advance from an authorised Daybreak agent

6 Tickets and seats can be reserved by phoning the Daybreak Hotline

9 Daybreak passengers must join their coach at Cambridge Drummer Street

10 Daybreak cannot guarantee return times

98 CHAPTER 4 Standardized Testing

FROM CAMBRIDGE AND SURROUNDING AREA

SPRING IS INTHEAIR

Welcome to our Spring Daybreak programme which continues the tradition of offering unbeatable value for money day trips and tours All the excursions in this brochure will be operated by Pr~mier Travel Services Limited or Millers Coaches both companies are part of the CHLGroup owners of Cambridges Cambus fleet

WERE PROUD OF OUR TRADITION

Premier was established in 1936 the Company now offers the highest standards of coaching in todays competitive operating environment Miller has an enviable reputation stretching back over the past 20 years offering coach services at realistic prices Weve traveled a long way since our early days of pre-war seaside trips Now our fleet of 50 modern coaches (few are more than five years old) operate throughout Britain and Europe but were pleased to still maintain the high standards of quality and service the trademark of our founders nearly sixty years ago

EXCLUSIVE FEATURES

Admission-inclusive fares All Daybreak fares (unless specifically otherwise stated) include admission charges to the attractions shows and exhibits we visit Many full-day scenic tours are accompanied by a fully trained English Tourist Board Blue Badge guide or local experienced driverguide Some Daybreaks include lunch or afternoon tea Compare our admission inclusive fares and see how much you save Cheapest is not the best and value for money is guaranteed If you compare our bargain Daybreak fares beware--most of our competishytors do not offer an all-inclusive fare

SEAT RESERVATIONS

We value the freedom of choice so you can choose your seat when you book The seat reservation is guaranteed a-nd remains yours at all times when aboard the coach

NO SMOKING COMFORT

With the comfort of our passengers in mind coaches on all our Daybreaks are no smokshying throughout In the interests of fellow passengers comfort we kindly ask that smokers observe our no smoking policy On scenic tours and longer journeys ample refreshment stops are provided when of course smoking is permitted

YOUR QUESTIONS ANSWERED

Do I need to book Booking in advance is strongly recommended as all Daybreak tours are subject to demand Subject to availability stand-by tickets can be purchased from the driver

What ti me does the coach leave The coach departs from Cambridge Drummer Street (Bay 12 adjacent to public toilets) at the time shown There are many additional joining points indicated by departure codes in the brochure If you are joining at one of our less popular joining points you will be adshyvised of your pick-up time (normally by telephone) not less than 48 hours before deparshyture In this way we can minimize the length of pick-up routes and reduce journey times for the majority of passengers

CHAPTER 4 Standardized Testing 99

What time do we get back An approximate return time is shown for each excursion The tim~s shown serve as a guide but road conditions can sometimes cause delay If your arrival will be later than advertised your driver will try to allow for a telephone call during the return journey

Where can I board the coach All the Daybreaks in the brochure leave from Cambridge Drummer Street (Bay 12 adjashycent to public toilets) at the time shown Many Daybreaks offer additional pick-ups for pre-booked passengers within Cambridge and the surrounding area This facility must be requested at the time of booking

Academic Writing Writing Task 1 You should spend about 20 minutes on this task

The graph below shows the different modes of transport used to travel to and from work in one European city in 1950 1970 and 1990

[graph shown here]

Write a report for a university lecturer describing the information shown below You should write at least 150 words

Writing Task 2 You should spend about 40 minutes on this task

Present a written argument or case to an educated reader with no specialist knowledge of the folowing topic

It is inevitable that as technology develops so traditional cultures must be lost Technolshyogy and tradition are incompatible-you cannot have both together

To what extent do you agree or disagree with this statement Give reasons for your answer You should write at least 250 words You should use your own ideas knowlshyedge and experience and support your arguments with examples and relevant evidence

General Training Writing Writing Task 1 You should spend about 20 minutes on this task You rent a house through an agency The heating system has stopped working You phoned the agency a week ago but it has still not been mended Write a letter to the agency Explain the situation and teil them what you want them to do about it

You should write at least 150 words You do NOT need to write your own address

Begin your letter as follows

Dear - ___-I

Writing Task 2 You should spend about 40 minutes on this task As part ofa class assignment you have to write about the following topic

100 CHAPTER 4 Standardized Testing

Some businesses now say that no one can smoke cigarettes in any of their offices Some governments have banned smoking in all public placesThis is a good idea but it takes away some of our freedom

Do you agree or disagree Give reasons for your answer You should write at least 250 words

Speaking In each ofthe three parts of the speaking module a specific function is fulfilled In Part 1 the candidates answer general questions about themselves their homes or families their jobs or studies their interests and a range ofsimilar familiar topic areas This part lasts between four and five minutes In Part 2 the candidate is given a verbal prompt on a card and is asked to talk on a particular topic The candidate has one minute to prepare before speaking at length for between one and two minutes The examiner then asks one or two wind-down questions In Part 3 the examiner and candidate engage in a discusshysion of more abstract issues and concepts which are thematically linked to the topic prompt in Part 2 The discussion lasts between four and five minutes

All interviews are recorded on audiocassette Here is a sample ofa Part 2 topic

Describe a teacher who has greatly influenced you in your education

You shou Id say

where you met them what subject they taught what was special about them

and explain why this person influenced you so much

You will have to talk about the topic for 1 to 2 minutes You have 1 minute to think about what you are going to say You can make some notes if you wish

Test of English for International Communication (TOEICreg)

listening

Part 1 Photographs Directions For each question you will see a picture in your test book and you will hear four short statements The statements will be spoken just one time They will not be printed in your test book so you must listen carefully to understand what the speaker says When you hear the four statements look at the picture in your test book and choose the statement that best describes what you see in the picture Then on your answer sheet find the number of the question and mark your answer

[photograph of a scientist looking through a microscope]

You will hear Look at the picture marked number 1 in your test book

(A) Shes speaking into a microphone (B) Shes put on her glasses (C) She has both eyes open (D) Shes using a microscope

CHAPTER 4 Standardized Testing 101

Part 2 Question-Response Directions In this part of the test you will hear a question or statement spoken in Enshyglish followed by three responses also spoken in English The question or staten1ent and the responses will be spoken just one time They will not be printed in your test book so you must listen carefully to understand what the speakers say You are to choose the best response to each question or statement

Question 1 You will hear Ms Morikawa has worked here for a long time hasnt she

(A) At three oclock (B) No Ive lost my watch (C) More than ten years

Question 2 You will hear Which of these papers has a wider circulation

(A) The morning edition (B) Get more exercise (C) By messenger

Part 3 Short Conversations Directions In this part of the test you will hear short conversations between two people The conversations will not be printed in your test book You will hear the conversations only once so you must listen carefully to understand what the speakers say In your test book you will read a question about each conversation The question will be followed by four answers You are to choose the best answer to each question and mark it on your answer sheet

Question 1 (Man) We should think about finding another restaurant for lunch (Woman) Why The food and service here are great

(Man) Yes but the prices are going up every week

You will read Why is this man unhappy with the restaurant

(A) It is too noisy (B) It is too expensive (C) It is too crowded (D) It is too difficult to find

Question 2 (Woman A) How was Dr Borgs recent trip to Singapore (Woman B) She enjoyed the tour of the port very much (Woman A) They say its one of the most active in Asia

You will read 2 What did Dr Borg find interesting

(A) The tourist center (B) The airport (C) The musical performance (D) The harbor

Part 4 Short Talks Directions In this part of the test you vill hear several short talks Each will be spoken just one time They will not be printed in your test book so you must listen carefully to understand and remember what is said In your test book you will read two or more questions about each short talk The questions will be followed by four answers You are to choose the best answer to each question and mark it on your answer sheet

102 CHAPTER 4 Standardized Testing

You will hear Questions 1 and 2 refer to the following announcement

Good afternoon and welcome aboard Nordair Flight 857 from Copenhagen to Bangkok with intermediate stops in Dubai and Calcutta We are preparing for departure in a few minutes At this time your seat back should be returned to its full upright position and your seat belt s~ould be fastened OUf anticipated total flying time to Dubai is six hours and twenty-five minutes I hope you enjoy the flight You will hecJr Now read question 1 in your test book and answer it You will read 1 What is the final destination of the flight

(A) Bangkok (B) Copenhagen (C) Dubai (O) Calcutta

You will hear Now read question 2 in your test book and answer it You will read 2 What will happen in a few minutes

(A) The flight will land in Dubai I

(B) The passengers will board the plane (C) The plane will take off (0) The gate number will be announced

Reading In this section of the test you will have the chance to show how well you understand written English There are three parts to this section with special directions for each part

Part 4 Incomplete Sentences Directions This part of the test has incomplete sentences Four words or phrases marked (A) (8) (e) (D) are given beneath each sentence You are to choose the one word or phrase that best completes the sentence Then on your answer sheet find the number of the question and mark your answer

1 Mr Yangs trip will __ him away from the office for ten days (A) withdraw (B) continue (C) retain (0) keep

2 The company that Marie DuBois started now sells __ products throughout the world (A) its (B) it (C) theirs (D) them

3 If your shipment is not delivered __ Tuesday you can request a full refund for the merchandise (A) at (B) by (C) within (D) while

CHAPTER 4 Standardized Testing 103

Part 6 Error Recognition Directions In this part ofthe test each sentence has four words or phrases underlined The four underlined parts of the sentence are marked (A) (B) (C) (D) You are to identify the one underlined word or phrase that should be corrected or rewritten Then on your answer sheet find the number of the question and mark your answer

1 The pamphlet contains some importance information about the current exhibit ABC D

2 No matter how Jong it taking to finish the annual report it must be done properly ABC D

3 The popularity of jogging appears to have decreased since the past couple of years ABC D

Part 7 Reading Comprehension Directions The questions in this part of the test are based on a selection of reading mateshyrials such as notices letters) forms newspaper and magazine articles) and advertisements You are to choose the one best answer (A) (B) (C) or (OJ to each quesshytion Then on your ariswefsheelfindthe number of the qUestion andmcirkyour answer Answer all questions following each reading selection on thebasis of what is stated or implied in that selection

The Museum ofTechnology is a hands-on museum designed for people to experience science at w()rk~ Visitors are encouraged to use test and handle the objects o~ display Special demonstrations are scheduled for the first and second Wednesdays of each month at 1330 Open Tuesday-Friday 1200-1630 Saturday 1000-1730 and Sunday 11 00-1630

1 When during the month can visitors see special demonstrations (A) Every weekend (B) The first two Wednesdays (C) One afternoon a week (D) Every other Wednesday

Questions 2 and 3 refer to the followi ng notice

NOTICE If you are unable to work because of an extended illness or injury that is not workshyrelated you may be entitled to receive weekly benefits from your employer or the firms insurance company To claim benefits you must file a claim form within thirty days of the first day of your disability Before filing the claim you must ask your doctor to fill in the Doctors Statement on the claim form stating the period of disability

3 To whom is this notice addressed (A) Employers (8) Doctors (C) Employees (D) When paying the bill

4 When must the claim form be filed (A) On the first of the month (8) On the thirtieth of the month (C) On the first day ofdisabifity (D) Within 30 days of the start of disability

Page 11: Standardized Testing Chapter 4 Brown

76 CHAPTER 4 Standardized Testing

As you can see items target the assessment of comprehension of the main idea (item 11) stated details (17 19) unstated details (12 15 18) implied details (14 20) and vocabulary in context (13 16) An argument could be made about the cultural schemata implied in a passage about pirate ships and you could engage in an angels on the head of a pin argument about the importance of picking cershytain vocabulary for emphasis but every test item is a sample of a larger domain and each of these fulfills its designated specification

Before any such items are released into a form of the TOEFL (or any validated standardized test) they are piloted and sCientifically selected to meet difficulty specshyifications within each subsection section and the test overall Furthermore those items are also selected to meet a desired discrimination index Both of these indices are important considerations in the design of a computer-adaptive test where pershyformance on one item determines the next one to be presented to the test-taker (See Chapter 3 for a complete treatment of multiple-choice item design)

(B)The selection of items in the ESLPT entailed two-entirel) different processes In the two subsections of -the test that elicit writing performance (summary of reading response to reading) the main hurdles were (a) selecting appropriate passhysages for test-takers to read (b) providing appropriate prompts and (c) processing data from pilot testing Passages have to conform to standards of content validity by being within the genre and the difficulty of the material used in the courses The prompt in each case (the section asking for a summary and the section asking for a response) has to be tailOred to fit the passage but a general template is used

[n the multiple-choice editing test that seeks to test grammar proofreading ability the first and easier task is to choose an appropriate essay within which to embed errors The more complicated task is to embed a specified number of errors from a previously determined taxonomy of error categories Those error categories came directly from student errors as perceived by their teachers (verb tenses verb agreeshyment logical connectors articles etc) The disttactors for each item were selected from actual errors that students make Itemsiti pilot versions were then coded fordifshyficulty and discrinlination indices after which final assembly of items could occur

(C) The GET prompts are designed by a faculty committee of examiners who are speCialists in the field of university academic writing The assumption is made that the topics are universally appealing and capable of yielding the intended product of an essay that requires an organized logical argument and conclusion No pilot testing of prompts is conducted The conditions for administration remain constant two-hour time limit sit-down context paper and pencil closed-book format Consider the following recent prompt

Graduate Essay Test sample prompt

In the Middletown Elementary School District the assistant superintendent has just been made superintendent in another district Her resignation leaves vacant the districts only administrative position ever held by a woman The School Board in response to strong

CHAPTER 4 Standardized Testing 77

arguments from the Teachers Association has urged that a woman be hired to replace her As a member of the hiring committee you must help choose her successor

Only one woman applicant meets the written qualifications for the job the two top male applicants are both more experienced than she

The hiring committee has asked each committee member to prepare a written statement to distribute before meeting together to discuss the issue Write a report that represents your position making it as logical and persuasive as possible

Some facts you may wish to draw on 1 Women make up more than 75 percent of classroom teachers but hold fewer than

10 percent of administrative positions in education Administrators salaries average 30 percent more than teachers salaries

2 The local Teachers Association is 89 percent women mostly under 40 In a heated debate on television a member of the National Organization of Women (NOW) and the chair of the Teachers Association threatened if a man is hired to bring a class-action suit against the district on behalf of all women teachers who cannot expect advancement because of discriminatory hiring practices

3 The local Lions Club which contributes heavily to school sports says hiring the less experienced woman would not be in the best interests of the schoolthe children or the teachers

The finalists for the position

1 Carole Gates Classroom teacher 10 years Teacher of the Year 1985 supervisor ofpractice teachers at Teachers College former president of Teachers Associ ati on Administrative Credential 1984 EdD degree 1986 assistant principal of Hoptown Elementary School 2 years

2 Spud Stonewall Principal of Middletown Elementary 15 years PhD in educational adminis~ration State Board of Education Committee for Improving Elementary School Curriculum 1982-present

3 Jim Henderson School Administrator 22 yearsgradesK-9-supports innovation in education Fair Bargaining Award 1981 former coach for winning collegiate basketball team 10 years

It is clear from such a prompt that the problem the test-takers must address is complex that there is sufficient information here for writing an essay and that testshytakers will be reasonably challenged to write a clear statement of opinion What also emerges from this prompt (and virtually any prompt that one might propose) is the potential cultural effect on the numerous international students who must take the GIIT Is it possible that such students who are not familiar with school systems in the United States with hiring procedures and perhaps with the politics of school board elections might be at a disadvantage in mounting their arguments within a two-hour time frame Some (such as Hosoya 2001) have strongly claimed such a bias

78 CHAPTER 4 Standardized Testing

4 Make appropriate evaluations of different kinds of items

In Chapter 3 the concepts of item facility (IF) item discrimination (ID) and disshytractor analysis were introduced As the discussion there showed such calculations provide useful infornlation for classroom tests but sometimes the time and effort involved in perfornling them may not be practical especially if the classroom-based test is a one-time test Yet for a standardized multiple-choice test that is designed to be marketed commercially andor administered a number of times andor adminisshytered in a different form these indices are a must

For other types of response formats namely production responses different forms of evaluation become importantThe principles of p-mpoundti~ality ~d poundabWty are prominent along with the concept o(JacjJjt Practicality issues in such items include the clarity of directions timing of the test ease of administration and how much time is required to score responses Reliability is a major player in instances where more than one scorer is employed and to a lesser extent when a single scorer has to evaluate tests over long spans of time that could lead to deterioration of stanshydards Facility is also a key to the validity and success of an item type ~irecshytions complex- language obscure topics fuz~Qata and culturally biased

~Jfiformatioifma~alliead to a highei1eVermiddotof diffiCidty than one desires (A) The IF ID and efficiency statistics of the multiple-choice items of current

forms of the TOEFL are not publicly available information For reasons of security and protection of patented copyrighted materials they must remain behind the closed doors of the ETS development staff Those statistics remain of paramount importance in the ongoing production ofTOEFL items and forms and are the founshydation stones for demonstrating the equatability of forms Statistical indices on retired forms of the TOEFL are available on request for research purposes

The essay portion of theTOEFL undergoes scrutiny for its practicality reliability and facility Special attention is given to reliabilIty since two human scorers must read each essay and every time a third reader becomes necessary (when the two readers disagree by more than one point) it costs ETS more money

(B) In the case of the open-ended responses on the two written tasks on the ESLPT a similar set of judgments must be made Some evaluative impressions of the effectiveness of prompts and passages are gained from informal student and scorer feedback In the developmental stage of the newly revised ESLPT both types of feedshyback were formally solicited through questiQnnaires and interviews That informashytion proved to be invaluable in the revisIon of prompts and stimulus reading passages After each administration now the teacher-scorers provide informal feedshyback on their perceptions of the effectiveness of the prompts and readings

The multiple-choice editing passage showed the value of statistical findings in determining the usefulness of items and pointing administrators toward revisions Following is a sample of the format used

CHAPTER 4- Standardized Testing 79

Multiple-choice editing passage

(1)EYer since supermarkets first appeared they have beentake over ~ world ABC 0

(2) Supermarkets have changed peoples life ~ yet and at the same time changes in ABC

peoples life ~ have encouraged the opening of supermarkets o

The task was to locate the error in each sentence Statistical tests on the experishymental version of this section revealed that a number of the 45 items were found to be of zero IF (no difficulty whatsoever) and of inconsequential discrimination power (some IDs of 15 and lower) Many distractors were of no consequence because they lured no one Such information led to a revision of numerous it~ms and their options eventually strengthening the effectiveness of this section

(C)The GET like its written counterparts in the ESLPT is a test ofwritten ability with a single prompt and therefore questions of practicality and J~~illy~are also largely observational No data are collected from students on their perceptions but the scorers have an opportunity to reflect on the validity ofa given topiC After one sitting a topic is retired which eliininates the possibility of improving a specific topiC but future framing of topics might benefit from scorers evaluations Inter-rater reliability is checked periodically and reader training sessions are modified if too many instances of unreliability appear

5 Specify scoring procedures and reporting formats - ---

A systematic assembly of test items in pre-selected arrangements and sequences all of which are validated to confo~ to an e~pected difficulty level should yield a test that can then be scored accurately and reported back to test-takers and institutions efficiently

(A) Of the three tests being exemplifled here the most straightforward scoring procedure comes from the TO~FL the one with the most complex issues of validashytion deSign and assembly Scores are calculated and reported fora) three sections of the TOEFL (the essay ratings are combined with the Structure and Written Expression score) and (b) a total score (range 40 to 300 on the computer-based TOEFL and 310 to 677 on the paper-and-pencil TOEFL) A separate score (c) for the Essay (range 0 to 6) is also provided on the examinees score record (see simulation of a score record on page 80)

80 CHAPTER 4 Standardized Testing

Facsimile of a TOEFLreg score report

TOEFL Scaled Scores Claudia Y Estudiante Peru ___

19 17 17 177 Listening Structure Writing Reading Total Score

Essay rati ng 30

The rating scale for the essay is virtually the same one that is used for the Test of Written English (see Chapter 9 for details) with a zero level added for no response copying the topic only writing completely off topic or not writing in English

(B) The ESLPT reports a score for each of themiddot essay sections but the rating scale differs between them because in one case the objective is to write a summary and in the other to write a response to a reading ~ch essayi~pd lgtY ~o readet~ ifhFfF js a discrepancy of more than one level a third reader1resolves the differenceThe ~ditiilg section is machine-scanned and -scored with a total score and ~th part-scores for each ofthe grammaticaVrhetorlcal sectionS From these data placement administrators have adequate information to make placements and teachers receive some diagnostic inforshymation on each student in their classes Students do not receive their essays back

(C) Each GET is read by two trained readers who give a score between 1 and 4 according to the following scale

Graduate Essay Test Scoring Guide

Please make no marks on the writers work Write your reader number and score on the front cover of each test booklet

4 Superior The opening establishes context purpose and point of view the body of the essay developsmiddot recommendations-logically and coherently The writer demonshystrates awareness of the complexities in the situation and provides analysis of the probJem offers compelling or common-sense reasons for recommendations made makes underlying assumptions explicit

The writer uses fluent and idiomatic English with few mechanical errors Style reshyveals syntactic maturity is dear and direct is not choppy or over-colloquial nor over-formal stuffy or unfocused Occasional spelling or punctuation errors may be easily attributed to hasty transcription under pressure

3 Competent After an opening that establishes context and purpose the paper unfolds with few lapses in coherence but may have somewhat less clear organization of less explicit transitions than a top-score paper It may have somewhat less compelling logic or slightly less-wellreasoned suggestions than a 4 paper though it will provide reasons for the recommendations made

The writer uses dear fluent and generally idiomatic English but may make minor or infrequent ESL errors (preposition errors dropped articles or verb endings etc) or repeat a single error (eg not punctuate possessive nouns) Occasional lapses of style are offSet by demonstrated mastery of syntax

CHAPTE84 Standardized Testing 81

2 Weak The writer makes somewhat simplistic suggestions not fully supported with reashysons fails to cite key facts offers little analysis of the problem or shows a limited grasp of the situation the given information is copied or listed withlittle integration into argument Points may be random or repetitious Writing may be badly focused with careless use of abstract language resulting in predication errors or illogical sentences

ESL andlor careless mechanical errors are frequent enough to be distracting OR sentences may be choppy style over-casual usage occasionally unidiomatic

1 Inadequate The essay may be disjointed incoherent or minimally developed The writer shows little grasp of the complex issues involved is unable to establish conshytext point of view or purpose in opening of paper or has a poor sense of audience Mechanical andor ESL errors or unidiomatic usages are frequent sentences may be ungrammatical OR correct but short and very simple

The two readers scores are added to yield a total possible score of 2 to 8 Test administrators recommend a score of 6 as the threshold for allowing a student to pursue graduate-level courses Anything below that is accompanied by a recomshymendation that the student either repeat the test or take a remedial course in gradshyuate writing offered in one of several different departments Students receive neither their essays nor any feedback other than the fmal score

6 Perform ongoing construct validation studies

From the above discussion it should be clear that no standardized instrument is expected to be used repeatedly without a ramporou~program of ongoing c~-sectmct valiltiatiOll Any standardized test once developed must be accompanied by sysshy~

tematic periodic corroboration of its effectiveness and by steps toward its improveshyment This rigor is especially true of tests that are produced in equated forms that is forms must be reliable across tests such that a score on a subsequent form of a test-has-the~same validityand-interpretability as its original

(A) The TOEFL program in cooperation with other tests produced by ETS has an impressive program of research Over the years dozens of TOEFL-sponsored research studies have appeared in the TOEFL Monograph Series An early example ofsuch a study was the seminal Duran et aI (1985) study TOEFLfrom a Communicative ViewpOint on Language Proficiency which examined the content characteristics of the TOEFL from a communicative perspective based on current research in applied linguistics and lanshyguage proficiency assessment More recent studies (such as Ginther 2001 Leacock amp Chodorow 2001 Powers et aI 2002) demonstrate an impressive array of scrutiny

(B) For approximately 20 years the ESLPT appeared to be placing students relishyably by means of an essay and a multiple-choice grammar and vocabulary test Over the years the security of the latter became s1lspect and the faculty administrators wished to see some content validity achieved in the process In the year 2000 that process began with a group of graduate students (Imao et aI 2000) in consl1ltation with faculty members and continued to fruition in the form of a new ESLPT reported in lmao (2002) The development of the new ESlPT involved a lengthy process of

82 CHAPTER 4 Standardized Testing

both content and construct validation along with facing such practical issues as scoring the written sections and a machine scorable multiple-choice answer sheet

The process of ongoing validation will no doubt continue as new forms of the editing section are created and as new prompts and reading passages are created for the writing section Such a validation process should also include consistent checks on placement accuracy and on face validity

(C) At this time there is little or no research to validate the GET itself For its conshy struct validation its administrators rely on a stockpile of research on university-level academic writing tests such as theTWEThe holistic scoring rubric and the topics and administrative conditions of the GET are to some extent patterned after that of the TWE In recent years some criticism of the GEf has come from international test-takers (Hosoya 2001) who posit that the topics and time limits of the GET among other facshytors work to the disadvantage of writers whose native language is not English These validity issues remain to be fully addressed in a comprehensive research study

I I

STANDARDIZED IANGUAGE PROFICIENCY TESTING

Tests of language profiCiency presuppose a comprehensive definition of the specific competencies that comprise overall language ability The specifications for the TOEFL provided an illustration of an operational definition of ability for assessment purposes This is not the only way to conceptualize the concept Swain (1990) offered a multidimensional view of profiCiency assessment by referring to three linshyguistic traits (grammar discourse and sociolinguistics) that can be assessed by means of oral multiple-choice and written responses (see Table 41) Swains conshyception was not meant to be an exhaustive analysis of ability but rather to serve as an operational framework for constructing proficiency assessments

Another defmition and conceptualization of profiCiency is suggested by the ACTFL association mentioned earlier ACfFL takes a holistic and more unitary view of proficiency in describing four levels superior advanced intermediate and noviceWithin each level descriptions of listening speaking reading and writing are provided as guidelines for assessment For example the ACfFL Guidelines describe the superior level of speaking as follows

ACTFL speaking guidelines summary superior-level

Superior-level speakers are characterized by the ability to

bull participate fully and effectively in conversations in formal and informal settings on topics related to practical needs and areas of professional andor scholarly interests

bull provide a structured argument to explain and defend opinions and develop effective hypotheses within extended discourse

bull discuss topics concretely and abstractly bull deal with a linguistically unfamiliar situation bull maintain a high degree of linguistic accuracy bull satisfy the linguistic demands of professional andor scholarly life

CHAPTER4 Standardized Testing 83

The other three ACfFL levels use the same parameters in describing progressively lower proficiencies across all four skills Such taxonomie~ have the advantage of considering a number of functions of linguistic discourse but the disadvantage at the lower levels of overly emphasizing test-takers deficiencies

Table 41 Traits of second language proficiency (Swain 1990 p 403)

Trait Grammar Discourse Sociolinguistic

focus on grammatical focus on textual focus on social accuracy within cohesion and appropriateness of sentences coherence language use

Method

Oral structured interview story telling and argumentationpersuasion

role-play ofspeech acts requests offers complaints

scored for accuracy of verbal morphology prepositions syntax

detailed rating for identification logical sequence and time orientation and global ratings for coherence

scored for ability to distinguish formal and informal register

Multiple-choice

sentence-level select the correct form exercise

paragraph-level select the coherent sentence exercise

speech act-Ievelselect the appropriate utterance exercise

(45 items) (29 items) (28 items)

involving verb morphology prepositionsan-d-uther items

Written composition

narrative and letter of persuasion

narrative and letter of persuasion

formal request letter and informal note

scored for accuracy of verb morphology prepositions syntax

detailed ratings much as for oral discourse and global rating for coherence

scored for the ability to distinguish formal and inforJ1lil1 register

FOUR STANDARDIZED lANGUAGE PROFICIENCY TESTS

We now tum to some of the better-known standardized tests of overall language ability or profiCiency to examine some of the typical formats used in commercially available tests We will not look at standardized tests of other specific skills here but that should not lead you to think by any means that proficiency is the only kind of test in the field that is standardized Three standardized oral production tests the

84 CHAPTER 4 Standardized Testing

Test of Spoken English (fSE) the Oral Proficiency Inventory (OPI) and PbonePassreg are discussed in Chapter 7 and the Test of Written English (WE) is covered in ChapterS

Four commercially produced standardized tests of English language proficiency are described briefly in this section the TOEFL the Michigan English Language Assessment Battery (MELAB) the International English Language Testing System (lELTS) and the Test of English for International Communication (fOEICreg) In an appendix to this chapter are sample items from each section of each test When you turn to that appendix use the following questions to help you evaluate these four tests and their subsections

1 What item types are included 2 How practical and reliable does each subsection of each test appear to be 3 Do the item types and tasks appropriately represent a conceptualizatio~ of

language proficiency (ability) That is can you evaluate their construct validity

4 Do the tasks achieve face validity 5 Are the tasks authentic 6 Is there some washback potential in the tasks

Test of English as a Foreign Language (TOEFL)

Producer Educational Testing Service (ETS) Objective To test overall proficiency (language ability) Primary market Almost exclusively US universities and colleges for admission

purposes Type Computer-based (CB) (and two sections are-computer-adaptive)

A traditional paper-based (PB) version is also available Response modes Multiple-choice responses essay Specifications See the box on pp 72-73 Time allocation Up to 4 hours (CB) 3 hours (PB) Internet access wwwtoeflorg

Comments In the North American context the TOEFL is the most widely used comshymercially available standardized test of proficiency Each year the TOEFL test is adminisshytered to approximately 800000 candidates in more than 200 countries It is highly respected because of the thorough program of ongoing research and development conshyducted by ETS The TOEFLs primary use is to set proficiency standards for international students seeking admission to English-speaking universities More than 4200 academic institutions government agencies scholarship programs and licensingcertification agenshycies in more than 80 countries use TOEFL scores By 2004 the TOEFL will include a secshytion on oral production

CHAPTER 4 Standardi~ed Testing 85

Michigan English Language Assessment Battery (MELAB)

Producer English language Institute University of Michigan Objective To test overall proficiency (language ability) Primary market Mostly US and Canadian language programs and colleges

some worldwide educational settings as well Type Paper-based Response modes Multiple-choice responses essay Time allocation 25 to 35 hours Internet access wwwlsaumicheduelimelabhtm

Specifications The MElAB consists of three sections Part 1 a 3D-minute impromptu essay is written on an assigned topic Part 2 a 25-minute multiple-choice listening comshyprehension test is delivered via tape recorder Part 3 is a 100-item 75-minute multipleshychoice test containing grammar doze reading vocabulary and reading comprehension An oral interview (speaking test) is optional

Comments The Ell at the University of Michigan has been producing the MELAB and its earlier incarnation (Michigan Test of English language Proficiency) since 1961 like the TOEFL it serves a North American audience but is also used internationally While its use is not as widespread as the TOEFL its validity is widely respected Because it is cheaper than the TOEFL and more easily obtained it is popular among language schools and institutes Many institutions and companies accept MElAB scores in lieu ofTOEFL scores

International English Language Testing System (IELTS)

Producer Jointly managed by The University of Cambridge local Examinations Syndicate (UClES) The British Council and lOP Education Australia

Objective To test overall proficiency (language ability) Primary-market Australian British Canadian and New Zealand academic

institutions and professional organizations American academic institutions are increasingly accepting IELTS for admissions purposes

1)rpe Computer-based (for the Reading and Writing sections) papershybased for the listening and Speaking modules

Response modes Multiple-choice responses essay oral production Time allocation 2 hours 45 minutes Internet access httpwwwieltsorgl

httpwwwudesorguk httpwwwbritishcouncilorg

Specifications Reading candidates choose between academic reading or general training reading (60 minutes) Writing the same option academic writing or general training writing (60 minutes) Listening four sections for all candidates (30 minutes) Speaking five sections for all candidates (1015 minutes)

86 CHAPTER 4 Standardized Testing

Comments The University of Cambridge local Examinations Syndicate (UCLES) has been producing English language tests since 1858 Now with three organizations cooperatshying to form the IELTS more than a million examinations are administered every year In 2002 a computer-based version of the Reading and Writing modules of the IELTS became available at selected centers around the world The other sections are administered locally by an examinet The paper-based IELTS remains an option for candidates The IELTS retains the distinct advantage of requiring all four skills in the test-takers performance

Test of English for International Communication (TOEICreg)

Producer The Chauncey Group International a subsidiary of Educational Testing Service

Objelttive To test overall proficiency (langlJage ability) Primary market Worldwide business commerce and industry contexts

(workplace settings) Type Computer-based and paper-based versions Response modes Multiple-choice responses Time allocation 2 hours Internet access httpwwwtoeiccom

Specifications Listening Comprehension 100 items administered by audiocassette Four types of task statements questions short conversations and short talks (approxishymately 45 minutes) Reading 100 items Three types of task cloze sentences error recogshynition and reading comprehension (75 minutes)

Comments The TOEIC has become a very widely used international test of English proficiency in workplace settings where English is required for job performance The conshytent includes many different employment settings such as conferences presentations sales ordering shipping schedules reservations (etters and memoranda It is approprishyate to use in educational settings where vocational or workplace English courses are being offered

sect sect sect sect sect

The construction of a valid standardized test is no minor accomplishment whether the instrument is large- or small-scale The designing of specifications alone as this chapter illustrates requires a sophisticated process of construct valishydation coupled with considerations of practicality Then the construction of items and scoringinterpretation procedures may require a lengthy period of trial and error with prototypes of the final form of the testWith painstaking attention to all the details of construction the end product can result in a cost-effective timeshysaving accurate instrument Your use of the results of such assessments can provide useful data on learners language abilities But your caution is warranted as well for all the reasons discussed in this chapter The next chapter will elaborate on what lies behind that need for a cautious approach to standardized assessment

CHAPTER4 Standardized Testing 87

EXERCISES

[Note (I) Individual work (G) Group or pair work (C) Whole-class discussion]

1 (C) Tell the class about the worst test experience youve ever had Briefly anamiddot lyze what made the experience so unbearable and try to come up with sugshygestions for improvement of the test andor its administrative conditions

2 (G) In pairs or small groups compile a brief list of pros and cons of standardshyized testing Cite illustrations of as many items in each list as possible Report your lists and examples to the rest o~ the class

3 (I) Select a standardized test that you are quite familiar with (probably a recent experience) Mentally evaluate that test using the five principles of practicality reliability validity authenticity and washback Report yourevaluashytion to the class

4 (G) The appendix to this chapter provides sample items from Jour different tests of language proficiency In groups one test for each group analyze your test for (a) content validity (b) face validity and (c) authenticity

5 (C) Do you think that the sample TOEFL reading passage about pirates (pages 74-75) and the Graduate EssayTest prompt (pages 76-77) about a school board hiring committee have any culture bias Discuss this and other cultural biases you have noticed in tests Is it possible to design a test that is completely free of culture bias

6 (CG) Compare the differences in conceptualization of language proficiency represented by Swains model the TOEFL and the ACfFL philosophy Which one best represents current thinking about communicative language ability What are the strengths and weaknesses of each approach

FORYOlIILEURTHER READING

Gronlund Norman E (1998) Assessment of student achievement Sixth Edition Boston Allyn and Bacon

Gronlunds classic also mentioned in Chapter 3 offers a concise overview of features of standardized tests offering definitions and examples of the statistical considerations in interpreting scores His approach is unbiased cleady written and accessible to those who might fear the mathematics of standardized testing

Phillips Deborah 2001 Long1nan introductory course for the TOEFL test White Plains NY Pearson Education

A careful examination of this or any other reputable preparation course for a standardized language test is well worth a students time Note especially how the book acquaints the user with the specifications of the test and offers a number of useful strategie~ that can be llsed in preparation for the test and during irs adn1inistration

88 CHAPTER 4 Standardized Testing

APPENDIX TO CHAPTER 4

Commercial Proficiency Tests Sample Items and Tasks

Test of English a~ a Foreign Language (TOEFLreg)

Listening r

Part A

In this section you will hear short conversations between two people In some ofthe conversations each person speaks only once In other conversations one or both of the people speak more than once Each conversation is followed by one questionabQlt it Each question in this part has four answer choices You should click on the best answer to each question Answer the questions on the basis of what is stated or implied by the speakers Here is an example On the computerscreen you will see

[man and woman talking]

On the recording you will hear

(woman) Hey wheres your sociology book (man) At home Why carry it around when were just going to be taking

a test (woman) Dont you remember Professor Smith said we could us it during

the test (man) Ohl no Well Ive still got an hour right Im so glad I ran into you

You wiII then see and hear the question before the answer choices appear

What will the man probably do next

o Begin studying for the sociology test o Explain the problem to his professor o Go home to get his textbook o Borrow the womans book

To choose an answer you will click on an oval The oval next to that answer will darken After you click on Next and Confirm Answer the next conversation will be presented

Part B

In this section you will hear several longer conversations and talks Each conversation or talk is followed by several questions The conversations talks and questions will not be repeated The conversations and talks are about a variety of topics You do not need speshycial knowledge of the topics to answer the questions correctly Rather you should answer each question on the basis of what is stated or implied by the speakers in the conversashytions or talks

For most of the questions you will need to click on the best of four possible answers Some questions will have special directions The special directions will appear in a box on the computer screen Here is an exampie ot a conversation and some questions

CHAPTER 4 Standardized Testing 89

Marine Biology (narrator) Listen to part of a discussion in a marine biology class

(professor) A few years ago our local government passed a number of strict environmental laws As a result Sunrise Beach looks nothing Ii ke it did ten years ago The water is cleaner and theres been a tremendous increase in all kinds of marine life which is why were going there on Thursday

(woman) I dont know if I agree that the water quality has improved I mean I was out there last weekend and it looked all brown It didnt seem too clean to me

(professor) Actually the color of the water doesnt always indicate whether its polluted The brown color you mentioned might be a result of pollution or it can mean a kind of brown algae is growing there Its called devils apron and it actually serves as food for whales

(man) So when does the water look blue (professor) Well water thats completely unpolluted is actually colorless But

it often looks bluish-green because the sunlight can penetrate deep down and thats the color thats reflected

(woman) But sometimes it looks really green Whats that about (professor) Ok well its the same principle as with devils apron the

water might be green because of different types of green algae there-gulfweed phytoplankton You all should finish reading about algae and plankton before we go In fact those are the types of living things Im going to ask you to be looking for when were there

Now get ready to answer the questions

What is the discussion mainly about

o The importance of protecting ocean environments o The reasons why ocean water appears to be different colors o The survival of whales in polluted water o The effect that colored ocean water has on algae

To choose an answer click on an oval The oval next to that answer will darken After you click on Next and Confirm Answer the next question will be presented

According to the professor what can make ocean water look browngt

o Pollution o Cloudy Skies o Sand o Algae

Click on 2 answers

To choose your answers you will click on the squares An XII wiii appear in each square

bullbullbullbullbullbullbull

90 CHAPTER 4 Standardized Testing

Structure and Written Expression This section measures the ability to recognize language that is appropriate for standard written English There are two types ofquestions in this section In the first type ofquestion there are incomplete sentences Beneath each sentence there are four words or phrases

Directions CIiSk on the one word or phrase that best completes the sentence

The colum~ine flower __ to nearly all of the United States can be raised from seed in almost any garden

native how native is how native is it is native

Time Help Confirm

After you click on Next and Confirm Answ~ the next question willbe presented

The second type of question has four underlined words or phrases You will choose the one underlined word or phrase that must be changed for the sentence to be correct

Directions Click on the one underlined word or phrase that must be changed for the senshytence to be correct

One of the most difficult problems in understanding sleep is determining what the funcshytions of sleep ~

lime Help Confirm

Clicking on an underlined word or phrase will darken it

Reading This section measures the ability to read and understand short passages similar in topic and style to those that students are likely to encounter in North American universities and colleges This section contains reading passages and questions about the passages There are several different types of questions in this section In the Reading section you will first have the opportunity to read the passage

The temperature of the Sun is over 10000 degrees Fahrenheit at the surface but it rises perhaps more than 270000000 at the center The Sun is so much hotter than the Earth that matter can exist only as a gasl except perhaps at the core In the core of the Sun the pressures are so great that despite the high temperature there may be a small solid core However no one really knows since the center of the Sun can never be directly observed ~ Solar astronomers do know that the Sun is divided into five general layers or zones Starting at the outside and going down into the Sun the zones are the corona chromoshysphere hotosphere convection zone and finally the core The first three zones are reshygarded as the Suns atmosphere But since the Sun has no solid surface it is hard to middottell where the atmosphere ends and the main body of the Sun begins

The Suns outermost layer begins about 10000 miles above the visible surface and goes outward for millions of miles This is the only part of the Sun that can be seen during an eclipse such as the one in February 1979 At any other time the corona can be seen

bullbullbullbullbullbullbull

bull bullbullbullbullbullbull

CHAPTER 4 Standardized Testing 91

only when special instruments are used on cameras and telescopes to block the light from the photosphere

The corona is a brilliant pearly white filmy light about as bright as the full Moon Its beautiful rays are a sensational sight during an eclipse The coronas rays flash out in a brilliant fan that has wispy spikelike rays near the Suns north and south poles The corona is generally thickest at the Suns equator The corona is made up of gases streamshying outward at tremendous speeds that reach a temperature of more than 2 million deshygrees Fahrenheit The gas thins out as it reaches the space around the planets By the time the gas of the corona reaches the Earth it has a relatively low density

When you have finished reading the passage you will use the mouse to click on Proceed Then the questions about the passage will be presented You are to choose the one best anshyswer to each question Answer all questions about the information in a passage on the basis ofwhat is stated or implied in that passage Most ofthe questions will be multiple-choice questions To answer these questions you will click on a choice below the question

With what topic is paragraph 2 mainly concerned

o How the Sun evolved o The structure of the Sun o Why scientists study the Sun o The distaflce of the Sun from the planets

Paragraph 2 is marked with an arrow (~)

You will see the next question after you click on Next

To answer some questions you will click on a word or phrase Here is an example

Look at the word one in the passage Click on the word or phrase in the bold text that one refers to To answer you can click on any part of the word or phrase in the passage Jour choice will darken to show which word you have chosen

The Suns outermost layer begins about 10000 miles above the visible surface and goes outward for millions of miles This is the only part of the Sun that can be seen durshying an eclipse such as the one in February 1979 At any other time the corona can be seen only when special instruments are used on cameras and telescopes to block the Iight from the photosphere

You will see the next question after you click on~ To answer some q~estions you will click on a sentence in the passage Here is an example

~ The corona is a brilliant pearly white filmy light about as bright as the full Moon Its beautiful rays are a sensational sight during an eclipse The coronas rays flash out in a brilliant fan that has wispy spikelike rays near the Suns north and south poles The corona is generally thickest at the Suns equator ~ The corona is made up of gases streaming outward at tremendous speeds that reach a temperature of more than 2 million degrees Fahrenheit The gas thins out as it reaches the space around the planets By the time the gas of the corona reaches the Earth it has a relatively low density

bull bullbullbullbullbullbull

92 CHAPTER 4 Standardized Testing

Click on the sentence in paragraph 4 or 5 in which the author compares the light of the Suns outermost layer to that ofanother astronomical body Paragraphs 4 and 5 are marked with arrows (~)

To answer some questions you will click on a square to add a sentence to the passage Here is an example -The following sentence can be added to paragraph 1

At the center of the Earths solar system lies the Sun

Where would it best fit in paragraph I Click on the square to add the sentence to the paragraph

D The temperature of the Sun is over 10000 degrees Fahrenheit at the surface but it rises to perhaps morethan 27000000deg at the center 0 The Sun is so much hotter than the Earth that matter can exist only as a gasi except p~rHapsatth~ c6relp the c~re of the ii Sun the pressures are so great that despite the high temperature there may be a small solid core D However no one really knows since the center of the Sun can never be directly observed D 0100

When you click on a square the sentence will appear in the passage at the place you have chosen You can read the sentence added to the paragraph to see if this is the best place to add it You can click on another square to change your answer The sentence will be added and shown in a dark box

Writing In this section you will have an opportunity to demonstrate your ability to write in Enshyglish This includes the ability to generate and organize ideas to support those ideas with examples or evidence and to compose in standard written English in response to an asshysigned topic You will have 30 minutes to write your essay on that topic You must write on the topic you are assigned An essay on any other topic will receive a score of 0 Read the topic below and then make any notes that will help you plan your response Begin typing your response in the box at the bottom of the screen or write your answer on the answer sheet provided to you

Following is a sample topic

Do you agree or disagree with the following statemenH

Teachers should make learning enjoyable and fun for their students

Use specific reasons and examples to support your opinion

CHAPTER 4 Standarczed Testing 93

Michigan English Language Assessment Battery (MELAB)

Composition The time limit for the composition is 30 minutes You must write on only one of the top~

ics below If you write about something else your composition paper will not be graded and you cannot be given a final score If you do not understand the topics ask the exam~ iner to explain or to translate them You may be asked to give your opinion ofsomething and explain why you believe this to describe something from your experience or to exshyplain a problem and offer possible solutions You should write at least one page Some sample topics are

1 What do you think is your countrys greatest problem Explain in detail and tell what you think can be done about it

2 What are the characteristics of a good teacher Explain and give examples 3 An optimist is someone who sees the good side of things A pessimist sees the

bad side Are you an optimist or a pessimist Relate a personal experience that shows this

4 In your opinion are the benefits of space exploration really worth the enormous costs Discuss

Most MELAB compositions are one or two pages long (about 200-300 words) If your paper is extremely short (less than 150 words) your composition will be given a lower score Before you begin writing you might want to take 2 or 3 minutes to plan your comshyposition and to make a short outline to organize your thoughts Such outlines will not be graded they are only to help you You should use the last 5 minutes to read through your composition and to make changes or corrections

Your composition will be graded on how clearly you express yourself in English and on the range of English you are able to use and your control in doing so This means your composition should be well organized your arguments should be fully developed and you should show a range ofgrammatical structures and broad vocabulary Compositions that consist only of very short sentences and very simple vocabulary cannot be given the

middothighest scores If errors are not frequent and if they do not confuse your meaning they will not lower your score very much

Listening Now you will hear a short lecture You may take notes during the lecture Following the lecture you will be asked some questions about it

Therell be a two-week exhibit of the paintings of the little-known master Laura Bernhart at the Claire Osmond Galleries starting on the fifteenth of the month and running through the thirtieth Bernharts known for her innovative designs in abstract expressionism Though a true original she declared a spiritual heritage from Salvador Dali the famous Spanish painter Since Bernhart lived a rather solitary life and died while only in her twenties few people are aware of her works This showing at the Osmond Galleries will provide many with an introduction to her works

10 Where is the exhibit a the Art Museum b the Dali Galleries c the Osmond Galleries

94 CHAPTER 4 Standardized Testing

11 What is Bernhart known for a her copies of Dalis paintings b the originality of her designs c her exhibitions

12 What will going to the exhibit allow most people to do a to see Saivador Dalis paintings b to see Bernharts works for the first time c to learn about Spanish art

Grammar

1 What did the teacher just tell you

She reminded our notebooksI a us to bring b that we bring c our bringing d we should bring

2 Is Bill a good dancer

Not really __ he tries very hard a in spite of h despite c even though d while

3 your clothes are all wet1

Yes I didnt come __ the rain soon enough a away to b over to c down with d in from

Cloze In years to come zoos will not only be places where animals are exhibited to the public but repositories where rare species can be saved from extinction (7) captive breeding The most powerful force (8) the future of many animals-and of zoos-is the decline of the wild (9) even zoo directors would argue that (10) are better places for animals than the fields and forest of their native (11) yet zoos may be the last chance for some creatures that would otherwise pass qUietly into oblivion

7 a through c from b of d damage

8 a bringing c to b that d influencing

9 a But c Not b So d Then

10 a where c even b zoos d wilds

11 alands c residence b life d field

CHAPTER 4 Standardized Testing 95

Vocabulary

12 Mark has a flair for writing a need b purpose c talent d dislike

13 Bill Collins launched his restaurant last June a moved b started c sold d bought

14 John will not accept the censure a burden b blame c credit d decision

15 I cant think of the answer Can you give me a __ a hint b token c taste d gaze

16 Because fewer people are taking expensive vacations the tourist industry is in a a choke b grope c grumble d slump

17 I disagree with a few of his opinions but __ we agree a deliberately b conclusively c essentially d immensely

Reading The influenza virus is a single molecule built from many millions of single atoms You must have heard of the viruses which are sometimes called living molecules While bacteria can be considered as a type of plant secreting pOisonous substances into the body of the organism they attack viruses are living organisms themselves We may conshysider them as regular chemical molecules since they have a strictly aefined atomic strucshyture but on the other hand we must also consider them as being alive since they are able to multiply in unlimited quantities

18 According to the passage bacteria are a poisons

b larger than viruses c very small d plants

96 CHAPTER 4 Standardized Testing

19 The writer says that viruses are alive because they a have a complex atomic structure b move c multiply d need warmth and light

20 The atomic structure of viruses a is -tJIariable b is strictly defined c cannot be analyzed chemically d is more complex than that of bacteria

International English Language Testing System (fELTS)

I

listening

The Listening Module has four sections The first two sections are concerned with social needs There is a conversation between two speakers and then a monologue For examshyple a conversation about travel arrangements or decisions on a night out and a speech about student services on a university campus or arrangements for meals during a confershyence The final two sections are concerned with situations related more closely to educashytional or training contexts For example conversation between a tutor and a student about an assignment or between three students planning a research project and a lecture or talk ofgeneral academic interest All the topics are ofgeneral interest and it makes no difference what subjects candidates study Tests and tasks become more difficult as the sections progress A range of English accents and dialects are used in the recording which reflects the international usage of IELTS

Academic Reading [A 7S0-word article on-th-e- topic of Wind Power in the US with a short glossary at the end]

Questions 1-5

Complete the summary below

Choose your answers from the box below the summary and write them in boxes 1-5 on your answer sheet Note There are more words or phrases than you will need to fill the gaps You may use any word or phrase more than once

Example The failure during the late 1970s and early 19805 of an attempt to establish a widespread wind power industry in the United States resulted largely from the (1) bull in oil prices during this period The industry is now experiencing a steady (2) due to improveshyments in technology and an increased awareness of the potential in the power of wind The wind turbines that are now being made based in part on the (3) of wide- ranging research in Europe are easier to manufacture and maintain than their predecesshysors This has led wind-turbine makers to be able to standardise and thus minimize (4) There has been growing (S) of the importance of wind power as an energy source

CHAPTER 4 Standardized Testing 97

criticism stability skepticism success operating costs decisions design costs fall effects production costs growth decline failure recognition results

Questions 6-1 0 Look at the following list of issues (Questions 6-10) and implications (A-C) Match each issue with one implication Write the appropriate letters A-C in boxes 6-10 on your anshyswer sheet

Example The current price of one wind-generated kilowatt Answer

6 The recent installation of systems taking advantage of economies of scale

7 The potential of meeting one fifth of current U5 energy requirements by wind power

8 The level of acceptance of current wind turbine technology

9 A comparison of costs between conventional and wind power sources

10 The view of wind power in the European Union

Implications

A provides evidence against claims that electricity produced from wind power is relatively expensive

B supports claims that wind power js an important source of energy

C opposes the view that wind power technology requires further-development

General Training Reading Read the passage on Daybreak trips by coach and look at the statements below On your answer sheet write

TRUE if the statement is true FALSE jf the statement is false

NOlGIVEN if the information is not given in the leaflet

1 MiIlers Coaches owns Cambridges Cam bus fleet

2 Premier is an older company than Millers

3 Most of the Daybreak coaches are less than 5 years old

4 Daybreak fares are more expensive than most of their competitors

5 Soft drinks and refreshments are served on most longer journeys

6 Smoking is permitted at the rear of the coach on longer journeys

7 Tickets must be bought in advance from an authorised Daybreak agent

6 Tickets and seats can be reserved by phoning the Daybreak Hotline

9 Daybreak passengers must join their coach at Cambridge Drummer Street

10 Daybreak cannot guarantee return times

98 CHAPTER 4 Standardized Testing

FROM CAMBRIDGE AND SURROUNDING AREA

SPRING IS INTHEAIR

Welcome to our Spring Daybreak programme which continues the tradition of offering unbeatable value for money day trips and tours All the excursions in this brochure will be operated by Pr~mier Travel Services Limited or Millers Coaches both companies are part of the CHLGroup owners of Cambridges Cambus fleet

WERE PROUD OF OUR TRADITION

Premier was established in 1936 the Company now offers the highest standards of coaching in todays competitive operating environment Miller has an enviable reputation stretching back over the past 20 years offering coach services at realistic prices Weve traveled a long way since our early days of pre-war seaside trips Now our fleet of 50 modern coaches (few are more than five years old) operate throughout Britain and Europe but were pleased to still maintain the high standards of quality and service the trademark of our founders nearly sixty years ago

EXCLUSIVE FEATURES

Admission-inclusive fares All Daybreak fares (unless specifically otherwise stated) include admission charges to the attractions shows and exhibits we visit Many full-day scenic tours are accompanied by a fully trained English Tourist Board Blue Badge guide or local experienced driverguide Some Daybreaks include lunch or afternoon tea Compare our admission inclusive fares and see how much you save Cheapest is not the best and value for money is guaranteed If you compare our bargain Daybreak fares beware--most of our competishytors do not offer an all-inclusive fare

SEAT RESERVATIONS

We value the freedom of choice so you can choose your seat when you book The seat reservation is guaranteed a-nd remains yours at all times when aboard the coach

NO SMOKING COMFORT

With the comfort of our passengers in mind coaches on all our Daybreaks are no smokshying throughout In the interests of fellow passengers comfort we kindly ask that smokers observe our no smoking policy On scenic tours and longer journeys ample refreshment stops are provided when of course smoking is permitted

YOUR QUESTIONS ANSWERED

Do I need to book Booking in advance is strongly recommended as all Daybreak tours are subject to demand Subject to availability stand-by tickets can be purchased from the driver

What ti me does the coach leave The coach departs from Cambridge Drummer Street (Bay 12 adjacent to public toilets) at the time shown There are many additional joining points indicated by departure codes in the brochure If you are joining at one of our less popular joining points you will be adshyvised of your pick-up time (normally by telephone) not less than 48 hours before deparshyture In this way we can minimize the length of pick-up routes and reduce journey times for the majority of passengers

CHAPTER 4 Standardized Testing 99

What time do we get back An approximate return time is shown for each excursion The tim~s shown serve as a guide but road conditions can sometimes cause delay If your arrival will be later than advertised your driver will try to allow for a telephone call during the return journey

Where can I board the coach All the Daybreaks in the brochure leave from Cambridge Drummer Street (Bay 12 adjashycent to public toilets) at the time shown Many Daybreaks offer additional pick-ups for pre-booked passengers within Cambridge and the surrounding area This facility must be requested at the time of booking

Academic Writing Writing Task 1 You should spend about 20 minutes on this task

The graph below shows the different modes of transport used to travel to and from work in one European city in 1950 1970 and 1990

[graph shown here]

Write a report for a university lecturer describing the information shown below You should write at least 150 words

Writing Task 2 You should spend about 40 minutes on this task

Present a written argument or case to an educated reader with no specialist knowledge of the folowing topic

It is inevitable that as technology develops so traditional cultures must be lost Technolshyogy and tradition are incompatible-you cannot have both together

To what extent do you agree or disagree with this statement Give reasons for your answer You should write at least 250 words You should use your own ideas knowlshyedge and experience and support your arguments with examples and relevant evidence

General Training Writing Writing Task 1 You should spend about 20 minutes on this task You rent a house through an agency The heating system has stopped working You phoned the agency a week ago but it has still not been mended Write a letter to the agency Explain the situation and teil them what you want them to do about it

You should write at least 150 words You do NOT need to write your own address

Begin your letter as follows

Dear - ___-I

Writing Task 2 You should spend about 40 minutes on this task As part ofa class assignment you have to write about the following topic

100 CHAPTER 4 Standardized Testing

Some businesses now say that no one can smoke cigarettes in any of their offices Some governments have banned smoking in all public placesThis is a good idea but it takes away some of our freedom

Do you agree or disagree Give reasons for your answer You should write at least 250 words

Speaking In each ofthe three parts of the speaking module a specific function is fulfilled In Part 1 the candidates answer general questions about themselves their homes or families their jobs or studies their interests and a range ofsimilar familiar topic areas This part lasts between four and five minutes In Part 2 the candidate is given a verbal prompt on a card and is asked to talk on a particular topic The candidate has one minute to prepare before speaking at length for between one and two minutes The examiner then asks one or two wind-down questions In Part 3 the examiner and candidate engage in a discusshysion of more abstract issues and concepts which are thematically linked to the topic prompt in Part 2 The discussion lasts between four and five minutes

All interviews are recorded on audiocassette Here is a sample ofa Part 2 topic

Describe a teacher who has greatly influenced you in your education

You shou Id say

where you met them what subject they taught what was special about them

and explain why this person influenced you so much

You will have to talk about the topic for 1 to 2 minutes You have 1 minute to think about what you are going to say You can make some notes if you wish

Test of English for International Communication (TOEICreg)

listening

Part 1 Photographs Directions For each question you will see a picture in your test book and you will hear four short statements The statements will be spoken just one time They will not be printed in your test book so you must listen carefully to understand what the speaker says When you hear the four statements look at the picture in your test book and choose the statement that best describes what you see in the picture Then on your answer sheet find the number of the question and mark your answer

[photograph of a scientist looking through a microscope]

You will hear Look at the picture marked number 1 in your test book

(A) Shes speaking into a microphone (B) Shes put on her glasses (C) She has both eyes open (D) Shes using a microscope

CHAPTER 4 Standardized Testing 101

Part 2 Question-Response Directions In this part of the test you will hear a question or statement spoken in Enshyglish followed by three responses also spoken in English The question or staten1ent and the responses will be spoken just one time They will not be printed in your test book so you must listen carefully to understand what the speakers say You are to choose the best response to each question or statement

Question 1 You will hear Ms Morikawa has worked here for a long time hasnt she

(A) At three oclock (B) No Ive lost my watch (C) More than ten years

Question 2 You will hear Which of these papers has a wider circulation

(A) The morning edition (B) Get more exercise (C) By messenger

Part 3 Short Conversations Directions In this part of the test you will hear short conversations between two people The conversations will not be printed in your test book You will hear the conversations only once so you must listen carefully to understand what the speakers say In your test book you will read a question about each conversation The question will be followed by four answers You are to choose the best answer to each question and mark it on your answer sheet

Question 1 (Man) We should think about finding another restaurant for lunch (Woman) Why The food and service here are great

(Man) Yes but the prices are going up every week

You will read Why is this man unhappy with the restaurant

(A) It is too noisy (B) It is too expensive (C) It is too crowded (D) It is too difficult to find

Question 2 (Woman A) How was Dr Borgs recent trip to Singapore (Woman B) She enjoyed the tour of the port very much (Woman A) They say its one of the most active in Asia

You will read 2 What did Dr Borg find interesting

(A) The tourist center (B) The airport (C) The musical performance (D) The harbor

Part 4 Short Talks Directions In this part of the test you vill hear several short talks Each will be spoken just one time They will not be printed in your test book so you must listen carefully to understand and remember what is said In your test book you will read two or more questions about each short talk The questions will be followed by four answers You are to choose the best answer to each question and mark it on your answer sheet

102 CHAPTER 4 Standardized Testing

You will hear Questions 1 and 2 refer to the following announcement

Good afternoon and welcome aboard Nordair Flight 857 from Copenhagen to Bangkok with intermediate stops in Dubai and Calcutta We are preparing for departure in a few minutes At this time your seat back should be returned to its full upright position and your seat belt s~ould be fastened OUf anticipated total flying time to Dubai is six hours and twenty-five minutes I hope you enjoy the flight You will hecJr Now read question 1 in your test book and answer it You will read 1 What is the final destination of the flight

(A) Bangkok (B) Copenhagen (C) Dubai (O) Calcutta

You will hear Now read question 2 in your test book and answer it You will read 2 What will happen in a few minutes

(A) The flight will land in Dubai I

(B) The passengers will board the plane (C) The plane will take off (0) The gate number will be announced

Reading In this section of the test you will have the chance to show how well you understand written English There are three parts to this section with special directions for each part

Part 4 Incomplete Sentences Directions This part of the test has incomplete sentences Four words or phrases marked (A) (8) (e) (D) are given beneath each sentence You are to choose the one word or phrase that best completes the sentence Then on your answer sheet find the number of the question and mark your answer

1 Mr Yangs trip will __ him away from the office for ten days (A) withdraw (B) continue (C) retain (0) keep

2 The company that Marie DuBois started now sells __ products throughout the world (A) its (B) it (C) theirs (D) them

3 If your shipment is not delivered __ Tuesday you can request a full refund for the merchandise (A) at (B) by (C) within (D) while

CHAPTER 4 Standardized Testing 103

Part 6 Error Recognition Directions In this part ofthe test each sentence has four words or phrases underlined The four underlined parts of the sentence are marked (A) (B) (C) (D) You are to identify the one underlined word or phrase that should be corrected or rewritten Then on your answer sheet find the number of the question and mark your answer

1 The pamphlet contains some importance information about the current exhibit ABC D

2 No matter how Jong it taking to finish the annual report it must be done properly ABC D

3 The popularity of jogging appears to have decreased since the past couple of years ABC D

Part 7 Reading Comprehension Directions The questions in this part of the test are based on a selection of reading mateshyrials such as notices letters) forms newspaper and magazine articles) and advertisements You are to choose the one best answer (A) (B) (C) or (OJ to each quesshytion Then on your ariswefsheelfindthe number of the qUestion andmcirkyour answer Answer all questions following each reading selection on thebasis of what is stated or implied in that selection

The Museum ofTechnology is a hands-on museum designed for people to experience science at w()rk~ Visitors are encouraged to use test and handle the objects o~ display Special demonstrations are scheduled for the first and second Wednesdays of each month at 1330 Open Tuesday-Friday 1200-1630 Saturday 1000-1730 and Sunday 11 00-1630

1 When during the month can visitors see special demonstrations (A) Every weekend (B) The first two Wednesdays (C) One afternoon a week (D) Every other Wednesday

Questions 2 and 3 refer to the followi ng notice

NOTICE If you are unable to work because of an extended illness or injury that is not workshyrelated you may be entitled to receive weekly benefits from your employer or the firms insurance company To claim benefits you must file a claim form within thirty days of the first day of your disability Before filing the claim you must ask your doctor to fill in the Doctors Statement on the claim form stating the period of disability

3 To whom is this notice addressed (A) Employers (8) Doctors (C) Employees (D) When paying the bill

4 When must the claim form be filed (A) On the first of the month (8) On the thirtieth of the month (C) On the first day ofdisabifity (D) Within 30 days of the start of disability

Page 12: Standardized Testing Chapter 4 Brown

CHAPTER 4 Standardized Testing 77

arguments from the Teachers Association has urged that a woman be hired to replace her As a member of the hiring committee you must help choose her successor

Only one woman applicant meets the written qualifications for the job the two top male applicants are both more experienced than she

The hiring committee has asked each committee member to prepare a written statement to distribute before meeting together to discuss the issue Write a report that represents your position making it as logical and persuasive as possible

Some facts you may wish to draw on 1 Women make up more than 75 percent of classroom teachers but hold fewer than

10 percent of administrative positions in education Administrators salaries average 30 percent more than teachers salaries

2 The local Teachers Association is 89 percent women mostly under 40 In a heated debate on television a member of the National Organization of Women (NOW) and the chair of the Teachers Association threatened if a man is hired to bring a class-action suit against the district on behalf of all women teachers who cannot expect advancement because of discriminatory hiring practices

3 The local Lions Club which contributes heavily to school sports says hiring the less experienced woman would not be in the best interests of the schoolthe children or the teachers

The finalists for the position

1 Carole Gates Classroom teacher 10 years Teacher of the Year 1985 supervisor ofpractice teachers at Teachers College former president of Teachers Associ ati on Administrative Credential 1984 EdD degree 1986 assistant principal of Hoptown Elementary School 2 years

2 Spud Stonewall Principal of Middletown Elementary 15 years PhD in educational adminis~ration State Board of Education Committee for Improving Elementary School Curriculum 1982-present

3 Jim Henderson School Administrator 22 yearsgradesK-9-supports innovation in education Fair Bargaining Award 1981 former coach for winning collegiate basketball team 10 years

It is clear from such a prompt that the problem the test-takers must address is complex that there is sufficient information here for writing an essay and that testshytakers will be reasonably challenged to write a clear statement of opinion What also emerges from this prompt (and virtually any prompt that one might propose) is the potential cultural effect on the numerous international students who must take the GIIT Is it possible that such students who are not familiar with school systems in the United States with hiring procedures and perhaps with the politics of school board elections might be at a disadvantage in mounting their arguments within a two-hour time frame Some (such as Hosoya 2001) have strongly claimed such a bias

78 CHAPTER 4 Standardized Testing

4 Make appropriate evaluations of different kinds of items

In Chapter 3 the concepts of item facility (IF) item discrimination (ID) and disshytractor analysis were introduced As the discussion there showed such calculations provide useful infornlation for classroom tests but sometimes the time and effort involved in perfornling them may not be practical especially if the classroom-based test is a one-time test Yet for a standardized multiple-choice test that is designed to be marketed commercially andor administered a number of times andor adminisshytered in a different form these indices are a must

For other types of response formats namely production responses different forms of evaluation become importantThe principles of p-mpoundti~ality ~d poundabWty are prominent along with the concept o(JacjJjt Practicality issues in such items include the clarity of directions timing of the test ease of administration and how much time is required to score responses Reliability is a major player in instances where more than one scorer is employed and to a lesser extent when a single scorer has to evaluate tests over long spans of time that could lead to deterioration of stanshydards Facility is also a key to the validity and success of an item type ~irecshytions complex- language obscure topics fuz~Qata and culturally biased

~Jfiformatioifma~alliead to a highei1eVermiddotof diffiCidty than one desires (A) The IF ID and efficiency statistics of the multiple-choice items of current

forms of the TOEFL are not publicly available information For reasons of security and protection of patented copyrighted materials they must remain behind the closed doors of the ETS development staff Those statistics remain of paramount importance in the ongoing production ofTOEFL items and forms and are the founshydation stones for demonstrating the equatability of forms Statistical indices on retired forms of the TOEFL are available on request for research purposes

The essay portion of theTOEFL undergoes scrutiny for its practicality reliability and facility Special attention is given to reliabilIty since two human scorers must read each essay and every time a third reader becomes necessary (when the two readers disagree by more than one point) it costs ETS more money

(B) In the case of the open-ended responses on the two written tasks on the ESLPT a similar set of judgments must be made Some evaluative impressions of the effectiveness of prompts and passages are gained from informal student and scorer feedback In the developmental stage of the newly revised ESLPT both types of feedshyback were formally solicited through questiQnnaires and interviews That informashytion proved to be invaluable in the revisIon of prompts and stimulus reading passages After each administration now the teacher-scorers provide informal feedshyback on their perceptions of the effectiveness of the prompts and readings

The multiple-choice editing passage showed the value of statistical findings in determining the usefulness of items and pointing administrators toward revisions Following is a sample of the format used

CHAPTER 4- Standardized Testing 79

Multiple-choice editing passage

(1)EYer since supermarkets first appeared they have beentake over ~ world ABC 0

(2) Supermarkets have changed peoples life ~ yet and at the same time changes in ABC

peoples life ~ have encouraged the opening of supermarkets o

The task was to locate the error in each sentence Statistical tests on the experishymental version of this section revealed that a number of the 45 items were found to be of zero IF (no difficulty whatsoever) and of inconsequential discrimination power (some IDs of 15 and lower) Many distractors were of no consequence because they lured no one Such information led to a revision of numerous it~ms and their options eventually strengthening the effectiveness of this section

(C)The GET like its written counterparts in the ESLPT is a test ofwritten ability with a single prompt and therefore questions of practicality and J~~illy~are also largely observational No data are collected from students on their perceptions but the scorers have an opportunity to reflect on the validity ofa given topiC After one sitting a topic is retired which eliininates the possibility of improving a specific topiC but future framing of topics might benefit from scorers evaluations Inter-rater reliability is checked periodically and reader training sessions are modified if too many instances of unreliability appear

5 Specify scoring procedures and reporting formats - ---

A systematic assembly of test items in pre-selected arrangements and sequences all of which are validated to confo~ to an e~pected difficulty level should yield a test that can then be scored accurately and reported back to test-takers and institutions efficiently

(A) Of the three tests being exemplifled here the most straightforward scoring procedure comes from the TO~FL the one with the most complex issues of validashytion deSign and assembly Scores are calculated and reported fora) three sections of the TOEFL (the essay ratings are combined with the Structure and Written Expression score) and (b) a total score (range 40 to 300 on the computer-based TOEFL and 310 to 677 on the paper-and-pencil TOEFL) A separate score (c) for the Essay (range 0 to 6) is also provided on the examinees score record (see simulation of a score record on page 80)

80 CHAPTER 4 Standardized Testing

Facsimile of a TOEFLreg score report

TOEFL Scaled Scores Claudia Y Estudiante Peru ___

19 17 17 177 Listening Structure Writing Reading Total Score

Essay rati ng 30

The rating scale for the essay is virtually the same one that is used for the Test of Written English (see Chapter 9 for details) with a zero level added for no response copying the topic only writing completely off topic or not writing in English

(B) The ESLPT reports a score for each of themiddot essay sections but the rating scale differs between them because in one case the objective is to write a summary and in the other to write a response to a reading ~ch essayi~pd lgtY ~o readet~ ifhFfF js a discrepancy of more than one level a third reader1resolves the differenceThe ~ditiilg section is machine-scanned and -scored with a total score and ~th part-scores for each ofthe grammaticaVrhetorlcal sectionS From these data placement administrators have adequate information to make placements and teachers receive some diagnostic inforshymation on each student in their classes Students do not receive their essays back

(C) Each GET is read by two trained readers who give a score between 1 and 4 according to the following scale

Graduate Essay Test Scoring Guide

Please make no marks on the writers work Write your reader number and score on the front cover of each test booklet

4 Superior The opening establishes context purpose and point of view the body of the essay developsmiddot recommendations-logically and coherently The writer demonshystrates awareness of the complexities in the situation and provides analysis of the probJem offers compelling or common-sense reasons for recommendations made makes underlying assumptions explicit

The writer uses fluent and idiomatic English with few mechanical errors Style reshyveals syntactic maturity is dear and direct is not choppy or over-colloquial nor over-formal stuffy or unfocused Occasional spelling or punctuation errors may be easily attributed to hasty transcription under pressure

3 Competent After an opening that establishes context and purpose the paper unfolds with few lapses in coherence but may have somewhat less clear organization of less explicit transitions than a top-score paper It may have somewhat less compelling logic or slightly less-wellreasoned suggestions than a 4 paper though it will provide reasons for the recommendations made

The writer uses dear fluent and generally idiomatic English but may make minor or infrequent ESL errors (preposition errors dropped articles or verb endings etc) or repeat a single error (eg not punctuate possessive nouns) Occasional lapses of style are offSet by demonstrated mastery of syntax

CHAPTE84 Standardized Testing 81

2 Weak The writer makes somewhat simplistic suggestions not fully supported with reashysons fails to cite key facts offers little analysis of the problem or shows a limited grasp of the situation the given information is copied or listed withlittle integration into argument Points may be random or repetitious Writing may be badly focused with careless use of abstract language resulting in predication errors or illogical sentences

ESL andlor careless mechanical errors are frequent enough to be distracting OR sentences may be choppy style over-casual usage occasionally unidiomatic

1 Inadequate The essay may be disjointed incoherent or minimally developed The writer shows little grasp of the complex issues involved is unable to establish conshytext point of view or purpose in opening of paper or has a poor sense of audience Mechanical andor ESL errors or unidiomatic usages are frequent sentences may be ungrammatical OR correct but short and very simple

The two readers scores are added to yield a total possible score of 2 to 8 Test administrators recommend a score of 6 as the threshold for allowing a student to pursue graduate-level courses Anything below that is accompanied by a recomshymendation that the student either repeat the test or take a remedial course in gradshyuate writing offered in one of several different departments Students receive neither their essays nor any feedback other than the fmal score

6 Perform ongoing construct validation studies

From the above discussion it should be clear that no standardized instrument is expected to be used repeatedly without a ramporou~program of ongoing c~-sectmct valiltiatiOll Any standardized test once developed must be accompanied by sysshy~

tematic periodic corroboration of its effectiveness and by steps toward its improveshyment This rigor is especially true of tests that are produced in equated forms that is forms must be reliable across tests such that a score on a subsequent form of a test-has-the~same validityand-interpretability as its original

(A) The TOEFL program in cooperation with other tests produced by ETS has an impressive program of research Over the years dozens of TOEFL-sponsored research studies have appeared in the TOEFL Monograph Series An early example ofsuch a study was the seminal Duran et aI (1985) study TOEFLfrom a Communicative ViewpOint on Language Proficiency which examined the content characteristics of the TOEFL from a communicative perspective based on current research in applied linguistics and lanshyguage proficiency assessment More recent studies (such as Ginther 2001 Leacock amp Chodorow 2001 Powers et aI 2002) demonstrate an impressive array of scrutiny

(B) For approximately 20 years the ESLPT appeared to be placing students relishyably by means of an essay and a multiple-choice grammar and vocabulary test Over the years the security of the latter became s1lspect and the faculty administrators wished to see some content validity achieved in the process In the year 2000 that process began with a group of graduate students (Imao et aI 2000) in consl1ltation with faculty members and continued to fruition in the form of a new ESLPT reported in lmao (2002) The development of the new ESlPT involved a lengthy process of

82 CHAPTER 4 Standardized Testing

both content and construct validation along with facing such practical issues as scoring the written sections and a machine scorable multiple-choice answer sheet

The process of ongoing validation will no doubt continue as new forms of the editing section are created and as new prompts and reading passages are created for the writing section Such a validation process should also include consistent checks on placement accuracy and on face validity

(C) At this time there is little or no research to validate the GET itself For its conshy struct validation its administrators rely on a stockpile of research on university-level academic writing tests such as theTWEThe holistic scoring rubric and the topics and administrative conditions of the GET are to some extent patterned after that of the TWE In recent years some criticism of the GEf has come from international test-takers (Hosoya 2001) who posit that the topics and time limits of the GET among other facshytors work to the disadvantage of writers whose native language is not English These validity issues remain to be fully addressed in a comprehensive research study

I I

STANDARDIZED IANGUAGE PROFICIENCY TESTING

Tests of language profiCiency presuppose a comprehensive definition of the specific competencies that comprise overall language ability The specifications for the TOEFL provided an illustration of an operational definition of ability for assessment purposes This is not the only way to conceptualize the concept Swain (1990) offered a multidimensional view of profiCiency assessment by referring to three linshyguistic traits (grammar discourse and sociolinguistics) that can be assessed by means of oral multiple-choice and written responses (see Table 41) Swains conshyception was not meant to be an exhaustive analysis of ability but rather to serve as an operational framework for constructing proficiency assessments

Another defmition and conceptualization of profiCiency is suggested by the ACTFL association mentioned earlier ACfFL takes a holistic and more unitary view of proficiency in describing four levels superior advanced intermediate and noviceWithin each level descriptions of listening speaking reading and writing are provided as guidelines for assessment For example the ACfFL Guidelines describe the superior level of speaking as follows

ACTFL speaking guidelines summary superior-level

Superior-level speakers are characterized by the ability to

bull participate fully and effectively in conversations in formal and informal settings on topics related to practical needs and areas of professional andor scholarly interests

bull provide a structured argument to explain and defend opinions and develop effective hypotheses within extended discourse

bull discuss topics concretely and abstractly bull deal with a linguistically unfamiliar situation bull maintain a high degree of linguistic accuracy bull satisfy the linguistic demands of professional andor scholarly life

CHAPTER4 Standardized Testing 83

The other three ACfFL levels use the same parameters in describing progressively lower proficiencies across all four skills Such taxonomie~ have the advantage of considering a number of functions of linguistic discourse but the disadvantage at the lower levels of overly emphasizing test-takers deficiencies

Table 41 Traits of second language proficiency (Swain 1990 p 403)

Trait Grammar Discourse Sociolinguistic

focus on grammatical focus on textual focus on social accuracy within cohesion and appropriateness of sentences coherence language use

Method

Oral structured interview story telling and argumentationpersuasion

role-play ofspeech acts requests offers complaints

scored for accuracy of verbal morphology prepositions syntax

detailed rating for identification logical sequence and time orientation and global ratings for coherence

scored for ability to distinguish formal and informal register

Multiple-choice

sentence-level select the correct form exercise

paragraph-level select the coherent sentence exercise

speech act-Ievelselect the appropriate utterance exercise

(45 items) (29 items) (28 items)

involving verb morphology prepositionsan-d-uther items

Written composition

narrative and letter of persuasion

narrative and letter of persuasion

formal request letter and informal note

scored for accuracy of verb morphology prepositions syntax

detailed ratings much as for oral discourse and global rating for coherence

scored for the ability to distinguish formal and inforJ1lil1 register

FOUR STANDARDIZED lANGUAGE PROFICIENCY TESTS

We now tum to some of the better-known standardized tests of overall language ability or profiCiency to examine some of the typical formats used in commercially available tests We will not look at standardized tests of other specific skills here but that should not lead you to think by any means that proficiency is the only kind of test in the field that is standardized Three standardized oral production tests the

84 CHAPTER 4 Standardized Testing

Test of Spoken English (fSE) the Oral Proficiency Inventory (OPI) and PbonePassreg are discussed in Chapter 7 and the Test of Written English (WE) is covered in ChapterS

Four commercially produced standardized tests of English language proficiency are described briefly in this section the TOEFL the Michigan English Language Assessment Battery (MELAB) the International English Language Testing System (lELTS) and the Test of English for International Communication (fOEICreg) In an appendix to this chapter are sample items from each section of each test When you turn to that appendix use the following questions to help you evaluate these four tests and their subsections

1 What item types are included 2 How practical and reliable does each subsection of each test appear to be 3 Do the item types and tasks appropriately represent a conceptualizatio~ of

language proficiency (ability) That is can you evaluate their construct validity

4 Do the tasks achieve face validity 5 Are the tasks authentic 6 Is there some washback potential in the tasks

Test of English as a Foreign Language (TOEFL)

Producer Educational Testing Service (ETS) Objective To test overall proficiency (language ability) Primary market Almost exclusively US universities and colleges for admission

purposes Type Computer-based (CB) (and two sections are-computer-adaptive)

A traditional paper-based (PB) version is also available Response modes Multiple-choice responses essay Specifications See the box on pp 72-73 Time allocation Up to 4 hours (CB) 3 hours (PB) Internet access wwwtoeflorg

Comments In the North American context the TOEFL is the most widely used comshymercially available standardized test of proficiency Each year the TOEFL test is adminisshytered to approximately 800000 candidates in more than 200 countries It is highly respected because of the thorough program of ongoing research and development conshyducted by ETS The TOEFLs primary use is to set proficiency standards for international students seeking admission to English-speaking universities More than 4200 academic institutions government agencies scholarship programs and licensingcertification agenshycies in more than 80 countries use TOEFL scores By 2004 the TOEFL will include a secshytion on oral production

CHAPTER 4 Standardi~ed Testing 85

Michigan English Language Assessment Battery (MELAB)

Producer English language Institute University of Michigan Objective To test overall proficiency (language ability) Primary market Mostly US and Canadian language programs and colleges

some worldwide educational settings as well Type Paper-based Response modes Multiple-choice responses essay Time allocation 25 to 35 hours Internet access wwwlsaumicheduelimelabhtm

Specifications The MElAB consists of three sections Part 1 a 3D-minute impromptu essay is written on an assigned topic Part 2 a 25-minute multiple-choice listening comshyprehension test is delivered via tape recorder Part 3 is a 100-item 75-minute multipleshychoice test containing grammar doze reading vocabulary and reading comprehension An oral interview (speaking test) is optional

Comments The Ell at the University of Michigan has been producing the MELAB and its earlier incarnation (Michigan Test of English language Proficiency) since 1961 like the TOEFL it serves a North American audience but is also used internationally While its use is not as widespread as the TOEFL its validity is widely respected Because it is cheaper than the TOEFL and more easily obtained it is popular among language schools and institutes Many institutions and companies accept MElAB scores in lieu ofTOEFL scores

International English Language Testing System (IELTS)

Producer Jointly managed by The University of Cambridge local Examinations Syndicate (UClES) The British Council and lOP Education Australia

Objective To test overall proficiency (language ability) Primary-market Australian British Canadian and New Zealand academic

institutions and professional organizations American academic institutions are increasingly accepting IELTS for admissions purposes

1)rpe Computer-based (for the Reading and Writing sections) papershybased for the listening and Speaking modules

Response modes Multiple-choice responses essay oral production Time allocation 2 hours 45 minutes Internet access httpwwwieltsorgl

httpwwwudesorguk httpwwwbritishcouncilorg

Specifications Reading candidates choose between academic reading or general training reading (60 minutes) Writing the same option academic writing or general training writing (60 minutes) Listening four sections for all candidates (30 minutes) Speaking five sections for all candidates (1015 minutes)

86 CHAPTER 4 Standardized Testing

Comments The University of Cambridge local Examinations Syndicate (UCLES) has been producing English language tests since 1858 Now with three organizations cooperatshying to form the IELTS more than a million examinations are administered every year In 2002 a computer-based version of the Reading and Writing modules of the IELTS became available at selected centers around the world The other sections are administered locally by an examinet The paper-based IELTS remains an option for candidates The IELTS retains the distinct advantage of requiring all four skills in the test-takers performance

Test of English for International Communication (TOEICreg)

Producer The Chauncey Group International a subsidiary of Educational Testing Service

Objelttive To test overall proficiency (langlJage ability) Primary market Worldwide business commerce and industry contexts

(workplace settings) Type Computer-based and paper-based versions Response modes Multiple-choice responses Time allocation 2 hours Internet access httpwwwtoeiccom

Specifications Listening Comprehension 100 items administered by audiocassette Four types of task statements questions short conversations and short talks (approxishymately 45 minutes) Reading 100 items Three types of task cloze sentences error recogshynition and reading comprehension (75 minutes)

Comments The TOEIC has become a very widely used international test of English proficiency in workplace settings where English is required for job performance The conshytent includes many different employment settings such as conferences presentations sales ordering shipping schedules reservations (etters and memoranda It is approprishyate to use in educational settings where vocational or workplace English courses are being offered

sect sect sect sect sect

The construction of a valid standardized test is no minor accomplishment whether the instrument is large- or small-scale The designing of specifications alone as this chapter illustrates requires a sophisticated process of construct valishydation coupled with considerations of practicality Then the construction of items and scoringinterpretation procedures may require a lengthy period of trial and error with prototypes of the final form of the testWith painstaking attention to all the details of construction the end product can result in a cost-effective timeshysaving accurate instrument Your use of the results of such assessments can provide useful data on learners language abilities But your caution is warranted as well for all the reasons discussed in this chapter The next chapter will elaborate on what lies behind that need for a cautious approach to standardized assessment

CHAPTER4 Standardized Testing 87

EXERCISES

[Note (I) Individual work (G) Group or pair work (C) Whole-class discussion]

1 (C) Tell the class about the worst test experience youve ever had Briefly anamiddot lyze what made the experience so unbearable and try to come up with sugshygestions for improvement of the test andor its administrative conditions

2 (G) In pairs or small groups compile a brief list of pros and cons of standardshyized testing Cite illustrations of as many items in each list as possible Report your lists and examples to the rest o~ the class

3 (I) Select a standardized test that you are quite familiar with (probably a recent experience) Mentally evaluate that test using the five principles of practicality reliability validity authenticity and washback Report yourevaluashytion to the class

4 (G) The appendix to this chapter provides sample items from Jour different tests of language proficiency In groups one test for each group analyze your test for (a) content validity (b) face validity and (c) authenticity

5 (C) Do you think that the sample TOEFL reading passage about pirates (pages 74-75) and the Graduate EssayTest prompt (pages 76-77) about a school board hiring committee have any culture bias Discuss this and other cultural biases you have noticed in tests Is it possible to design a test that is completely free of culture bias

6 (CG) Compare the differences in conceptualization of language proficiency represented by Swains model the TOEFL and the ACfFL philosophy Which one best represents current thinking about communicative language ability What are the strengths and weaknesses of each approach

FORYOlIILEURTHER READING

Gronlund Norman E (1998) Assessment of student achievement Sixth Edition Boston Allyn and Bacon

Gronlunds classic also mentioned in Chapter 3 offers a concise overview of features of standardized tests offering definitions and examples of the statistical considerations in interpreting scores His approach is unbiased cleady written and accessible to those who might fear the mathematics of standardized testing

Phillips Deborah 2001 Long1nan introductory course for the TOEFL test White Plains NY Pearson Education

A careful examination of this or any other reputable preparation course for a standardized language test is well worth a students time Note especially how the book acquaints the user with the specifications of the test and offers a number of useful strategie~ that can be llsed in preparation for the test and during irs adn1inistration

88 CHAPTER 4 Standardized Testing

APPENDIX TO CHAPTER 4

Commercial Proficiency Tests Sample Items and Tasks

Test of English a~ a Foreign Language (TOEFLreg)

Listening r

Part A

In this section you will hear short conversations between two people In some ofthe conversations each person speaks only once In other conversations one or both of the people speak more than once Each conversation is followed by one questionabQlt it Each question in this part has four answer choices You should click on the best answer to each question Answer the questions on the basis of what is stated or implied by the speakers Here is an example On the computerscreen you will see

[man and woman talking]

On the recording you will hear

(woman) Hey wheres your sociology book (man) At home Why carry it around when were just going to be taking

a test (woman) Dont you remember Professor Smith said we could us it during

the test (man) Ohl no Well Ive still got an hour right Im so glad I ran into you

You wiII then see and hear the question before the answer choices appear

What will the man probably do next

o Begin studying for the sociology test o Explain the problem to his professor o Go home to get his textbook o Borrow the womans book

To choose an answer you will click on an oval The oval next to that answer will darken After you click on Next and Confirm Answer the next conversation will be presented

Part B

In this section you will hear several longer conversations and talks Each conversation or talk is followed by several questions The conversations talks and questions will not be repeated The conversations and talks are about a variety of topics You do not need speshycial knowledge of the topics to answer the questions correctly Rather you should answer each question on the basis of what is stated or implied by the speakers in the conversashytions or talks

For most of the questions you will need to click on the best of four possible answers Some questions will have special directions The special directions will appear in a box on the computer screen Here is an exampie ot a conversation and some questions

CHAPTER 4 Standardized Testing 89

Marine Biology (narrator) Listen to part of a discussion in a marine biology class

(professor) A few years ago our local government passed a number of strict environmental laws As a result Sunrise Beach looks nothing Ii ke it did ten years ago The water is cleaner and theres been a tremendous increase in all kinds of marine life which is why were going there on Thursday

(woman) I dont know if I agree that the water quality has improved I mean I was out there last weekend and it looked all brown It didnt seem too clean to me

(professor) Actually the color of the water doesnt always indicate whether its polluted The brown color you mentioned might be a result of pollution or it can mean a kind of brown algae is growing there Its called devils apron and it actually serves as food for whales

(man) So when does the water look blue (professor) Well water thats completely unpolluted is actually colorless But

it often looks bluish-green because the sunlight can penetrate deep down and thats the color thats reflected

(woman) But sometimes it looks really green Whats that about (professor) Ok well its the same principle as with devils apron the

water might be green because of different types of green algae there-gulfweed phytoplankton You all should finish reading about algae and plankton before we go In fact those are the types of living things Im going to ask you to be looking for when were there

Now get ready to answer the questions

What is the discussion mainly about

o The importance of protecting ocean environments o The reasons why ocean water appears to be different colors o The survival of whales in polluted water o The effect that colored ocean water has on algae

To choose an answer click on an oval The oval next to that answer will darken After you click on Next and Confirm Answer the next question will be presented

According to the professor what can make ocean water look browngt

o Pollution o Cloudy Skies o Sand o Algae

Click on 2 answers

To choose your answers you will click on the squares An XII wiii appear in each square

bullbullbullbullbullbullbull

90 CHAPTER 4 Standardized Testing

Structure and Written Expression This section measures the ability to recognize language that is appropriate for standard written English There are two types ofquestions in this section In the first type ofquestion there are incomplete sentences Beneath each sentence there are four words or phrases

Directions CIiSk on the one word or phrase that best completes the sentence

The colum~ine flower __ to nearly all of the United States can be raised from seed in almost any garden

native how native is how native is it is native

Time Help Confirm

After you click on Next and Confirm Answ~ the next question willbe presented

The second type of question has four underlined words or phrases You will choose the one underlined word or phrase that must be changed for the sentence to be correct

Directions Click on the one underlined word or phrase that must be changed for the senshytence to be correct

One of the most difficult problems in understanding sleep is determining what the funcshytions of sleep ~

lime Help Confirm

Clicking on an underlined word or phrase will darken it

Reading This section measures the ability to read and understand short passages similar in topic and style to those that students are likely to encounter in North American universities and colleges This section contains reading passages and questions about the passages There are several different types of questions in this section In the Reading section you will first have the opportunity to read the passage

The temperature of the Sun is over 10000 degrees Fahrenheit at the surface but it rises perhaps more than 270000000 at the center The Sun is so much hotter than the Earth that matter can exist only as a gasl except perhaps at the core In the core of the Sun the pressures are so great that despite the high temperature there may be a small solid core However no one really knows since the center of the Sun can never be directly observed ~ Solar astronomers do know that the Sun is divided into five general layers or zones Starting at the outside and going down into the Sun the zones are the corona chromoshysphere hotosphere convection zone and finally the core The first three zones are reshygarded as the Suns atmosphere But since the Sun has no solid surface it is hard to middottell where the atmosphere ends and the main body of the Sun begins

The Suns outermost layer begins about 10000 miles above the visible surface and goes outward for millions of miles This is the only part of the Sun that can be seen during an eclipse such as the one in February 1979 At any other time the corona can be seen

bullbullbullbullbullbullbull

bull bullbullbullbullbullbull

CHAPTER 4 Standardized Testing 91

only when special instruments are used on cameras and telescopes to block the light from the photosphere

The corona is a brilliant pearly white filmy light about as bright as the full Moon Its beautiful rays are a sensational sight during an eclipse The coronas rays flash out in a brilliant fan that has wispy spikelike rays near the Suns north and south poles The corona is generally thickest at the Suns equator The corona is made up of gases streamshying outward at tremendous speeds that reach a temperature of more than 2 million deshygrees Fahrenheit The gas thins out as it reaches the space around the planets By the time the gas of the corona reaches the Earth it has a relatively low density

When you have finished reading the passage you will use the mouse to click on Proceed Then the questions about the passage will be presented You are to choose the one best anshyswer to each question Answer all questions about the information in a passage on the basis ofwhat is stated or implied in that passage Most ofthe questions will be multiple-choice questions To answer these questions you will click on a choice below the question

With what topic is paragraph 2 mainly concerned

o How the Sun evolved o The structure of the Sun o Why scientists study the Sun o The distaflce of the Sun from the planets

Paragraph 2 is marked with an arrow (~)

You will see the next question after you click on Next

To answer some questions you will click on a word or phrase Here is an example

Look at the word one in the passage Click on the word or phrase in the bold text that one refers to To answer you can click on any part of the word or phrase in the passage Jour choice will darken to show which word you have chosen

The Suns outermost layer begins about 10000 miles above the visible surface and goes outward for millions of miles This is the only part of the Sun that can be seen durshying an eclipse such as the one in February 1979 At any other time the corona can be seen only when special instruments are used on cameras and telescopes to block the Iight from the photosphere

You will see the next question after you click on~ To answer some q~estions you will click on a sentence in the passage Here is an example

~ The corona is a brilliant pearly white filmy light about as bright as the full Moon Its beautiful rays are a sensational sight during an eclipse The coronas rays flash out in a brilliant fan that has wispy spikelike rays near the Suns north and south poles The corona is generally thickest at the Suns equator ~ The corona is made up of gases streaming outward at tremendous speeds that reach a temperature of more than 2 million degrees Fahrenheit The gas thins out as it reaches the space around the planets By the time the gas of the corona reaches the Earth it has a relatively low density

bull bullbullbullbullbullbull

92 CHAPTER 4 Standardized Testing

Click on the sentence in paragraph 4 or 5 in which the author compares the light of the Suns outermost layer to that ofanother astronomical body Paragraphs 4 and 5 are marked with arrows (~)

To answer some questions you will click on a square to add a sentence to the passage Here is an example -The following sentence can be added to paragraph 1

At the center of the Earths solar system lies the Sun

Where would it best fit in paragraph I Click on the square to add the sentence to the paragraph

D The temperature of the Sun is over 10000 degrees Fahrenheit at the surface but it rises to perhaps morethan 27000000deg at the center 0 The Sun is so much hotter than the Earth that matter can exist only as a gasi except p~rHapsatth~ c6relp the c~re of the ii Sun the pressures are so great that despite the high temperature there may be a small solid core D However no one really knows since the center of the Sun can never be directly observed D 0100

When you click on a square the sentence will appear in the passage at the place you have chosen You can read the sentence added to the paragraph to see if this is the best place to add it You can click on another square to change your answer The sentence will be added and shown in a dark box

Writing In this section you will have an opportunity to demonstrate your ability to write in Enshyglish This includes the ability to generate and organize ideas to support those ideas with examples or evidence and to compose in standard written English in response to an asshysigned topic You will have 30 minutes to write your essay on that topic You must write on the topic you are assigned An essay on any other topic will receive a score of 0 Read the topic below and then make any notes that will help you plan your response Begin typing your response in the box at the bottom of the screen or write your answer on the answer sheet provided to you

Following is a sample topic

Do you agree or disagree with the following statemenH

Teachers should make learning enjoyable and fun for their students

Use specific reasons and examples to support your opinion

CHAPTER 4 Standarczed Testing 93

Michigan English Language Assessment Battery (MELAB)

Composition The time limit for the composition is 30 minutes You must write on only one of the top~

ics below If you write about something else your composition paper will not be graded and you cannot be given a final score If you do not understand the topics ask the exam~ iner to explain or to translate them You may be asked to give your opinion ofsomething and explain why you believe this to describe something from your experience or to exshyplain a problem and offer possible solutions You should write at least one page Some sample topics are

1 What do you think is your countrys greatest problem Explain in detail and tell what you think can be done about it

2 What are the characteristics of a good teacher Explain and give examples 3 An optimist is someone who sees the good side of things A pessimist sees the

bad side Are you an optimist or a pessimist Relate a personal experience that shows this

4 In your opinion are the benefits of space exploration really worth the enormous costs Discuss

Most MELAB compositions are one or two pages long (about 200-300 words) If your paper is extremely short (less than 150 words) your composition will be given a lower score Before you begin writing you might want to take 2 or 3 minutes to plan your comshyposition and to make a short outline to organize your thoughts Such outlines will not be graded they are only to help you You should use the last 5 minutes to read through your composition and to make changes or corrections

Your composition will be graded on how clearly you express yourself in English and on the range of English you are able to use and your control in doing so This means your composition should be well organized your arguments should be fully developed and you should show a range ofgrammatical structures and broad vocabulary Compositions that consist only of very short sentences and very simple vocabulary cannot be given the

middothighest scores If errors are not frequent and if they do not confuse your meaning they will not lower your score very much

Listening Now you will hear a short lecture You may take notes during the lecture Following the lecture you will be asked some questions about it

Therell be a two-week exhibit of the paintings of the little-known master Laura Bernhart at the Claire Osmond Galleries starting on the fifteenth of the month and running through the thirtieth Bernharts known for her innovative designs in abstract expressionism Though a true original she declared a spiritual heritage from Salvador Dali the famous Spanish painter Since Bernhart lived a rather solitary life and died while only in her twenties few people are aware of her works This showing at the Osmond Galleries will provide many with an introduction to her works

10 Where is the exhibit a the Art Museum b the Dali Galleries c the Osmond Galleries

94 CHAPTER 4 Standardized Testing

11 What is Bernhart known for a her copies of Dalis paintings b the originality of her designs c her exhibitions

12 What will going to the exhibit allow most people to do a to see Saivador Dalis paintings b to see Bernharts works for the first time c to learn about Spanish art

Grammar

1 What did the teacher just tell you

She reminded our notebooksI a us to bring b that we bring c our bringing d we should bring

2 Is Bill a good dancer

Not really __ he tries very hard a in spite of h despite c even though d while

3 your clothes are all wet1

Yes I didnt come __ the rain soon enough a away to b over to c down with d in from

Cloze In years to come zoos will not only be places where animals are exhibited to the public but repositories where rare species can be saved from extinction (7) captive breeding The most powerful force (8) the future of many animals-and of zoos-is the decline of the wild (9) even zoo directors would argue that (10) are better places for animals than the fields and forest of their native (11) yet zoos may be the last chance for some creatures that would otherwise pass qUietly into oblivion

7 a through c from b of d damage

8 a bringing c to b that d influencing

9 a But c Not b So d Then

10 a where c even b zoos d wilds

11 alands c residence b life d field

CHAPTER 4 Standardized Testing 95

Vocabulary

12 Mark has a flair for writing a need b purpose c talent d dislike

13 Bill Collins launched his restaurant last June a moved b started c sold d bought

14 John will not accept the censure a burden b blame c credit d decision

15 I cant think of the answer Can you give me a __ a hint b token c taste d gaze

16 Because fewer people are taking expensive vacations the tourist industry is in a a choke b grope c grumble d slump

17 I disagree with a few of his opinions but __ we agree a deliberately b conclusively c essentially d immensely

Reading The influenza virus is a single molecule built from many millions of single atoms You must have heard of the viruses which are sometimes called living molecules While bacteria can be considered as a type of plant secreting pOisonous substances into the body of the organism they attack viruses are living organisms themselves We may conshysider them as regular chemical molecules since they have a strictly aefined atomic strucshyture but on the other hand we must also consider them as being alive since they are able to multiply in unlimited quantities

18 According to the passage bacteria are a poisons

b larger than viruses c very small d plants

96 CHAPTER 4 Standardized Testing

19 The writer says that viruses are alive because they a have a complex atomic structure b move c multiply d need warmth and light

20 The atomic structure of viruses a is -tJIariable b is strictly defined c cannot be analyzed chemically d is more complex than that of bacteria

International English Language Testing System (fELTS)

I

listening

The Listening Module has four sections The first two sections are concerned with social needs There is a conversation between two speakers and then a monologue For examshyple a conversation about travel arrangements or decisions on a night out and a speech about student services on a university campus or arrangements for meals during a confershyence The final two sections are concerned with situations related more closely to educashytional or training contexts For example conversation between a tutor and a student about an assignment or between three students planning a research project and a lecture or talk ofgeneral academic interest All the topics are ofgeneral interest and it makes no difference what subjects candidates study Tests and tasks become more difficult as the sections progress A range of English accents and dialects are used in the recording which reflects the international usage of IELTS

Academic Reading [A 7S0-word article on-th-e- topic of Wind Power in the US with a short glossary at the end]

Questions 1-5

Complete the summary below

Choose your answers from the box below the summary and write them in boxes 1-5 on your answer sheet Note There are more words or phrases than you will need to fill the gaps You may use any word or phrase more than once

Example The failure during the late 1970s and early 19805 of an attempt to establish a widespread wind power industry in the United States resulted largely from the (1) bull in oil prices during this period The industry is now experiencing a steady (2) due to improveshyments in technology and an increased awareness of the potential in the power of wind The wind turbines that are now being made based in part on the (3) of wide- ranging research in Europe are easier to manufacture and maintain than their predecesshysors This has led wind-turbine makers to be able to standardise and thus minimize (4) There has been growing (S) of the importance of wind power as an energy source

CHAPTER 4 Standardized Testing 97

criticism stability skepticism success operating costs decisions design costs fall effects production costs growth decline failure recognition results

Questions 6-1 0 Look at the following list of issues (Questions 6-10) and implications (A-C) Match each issue with one implication Write the appropriate letters A-C in boxes 6-10 on your anshyswer sheet

Example The current price of one wind-generated kilowatt Answer

6 The recent installation of systems taking advantage of economies of scale

7 The potential of meeting one fifth of current U5 energy requirements by wind power

8 The level of acceptance of current wind turbine technology

9 A comparison of costs between conventional and wind power sources

10 The view of wind power in the European Union

Implications

A provides evidence against claims that electricity produced from wind power is relatively expensive

B supports claims that wind power js an important source of energy

C opposes the view that wind power technology requires further-development

General Training Reading Read the passage on Daybreak trips by coach and look at the statements below On your answer sheet write

TRUE if the statement is true FALSE jf the statement is false

NOlGIVEN if the information is not given in the leaflet

1 MiIlers Coaches owns Cambridges Cam bus fleet

2 Premier is an older company than Millers

3 Most of the Daybreak coaches are less than 5 years old

4 Daybreak fares are more expensive than most of their competitors

5 Soft drinks and refreshments are served on most longer journeys

6 Smoking is permitted at the rear of the coach on longer journeys

7 Tickets must be bought in advance from an authorised Daybreak agent

6 Tickets and seats can be reserved by phoning the Daybreak Hotline

9 Daybreak passengers must join their coach at Cambridge Drummer Street

10 Daybreak cannot guarantee return times

98 CHAPTER 4 Standardized Testing

FROM CAMBRIDGE AND SURROUNDING AREA

SPRING IS INTHEAIR

Welcome to our Spring Daybreak programme which continues the tradition of offering unbeatable value for money day trips and tours All the excursions in this brochure will be operated by Pr~mier Travel Services Limited or Millers Coaches both companies are part of the CHLGroup owners of Cambridges Cambus fleet

WERE PROUD OF OUR TRADITION

Premier was established in 1936 the Company now offers the highest standards of coaching in todays competitive operating environment Miller has an enviable reputation stretching back over the past 20 years offering coach services at realistic prices Weve traveled a long way since our early days of pre-war seaside trips Now our fleet of 50 modern coaches (few are more than five years old) operate throughout Britain and Europe but were pleased to still maintain the high standards of quality and service the trademark of our founders nearly sixty years ago

EXCLUSIVE FEATURES

Admission-inclusive fares All Daybreak fares (unless specifically otherwise stated) include admission charges to the attractions shows and exhibits we visit Many full-day scenic tours are accompanied by a fully trained English Tourist Board Blue Badge guide or local experienced driverguide Some Daybreaks include lunch or afternoon tea Compare our admission inclusive fares and see how much you save Cheapest is not the best and value for money is guaranteed If you compare our bargain Daybreak fares beware--most of our competishytors do not offer an all-inclusive fare

SEAT RESERVATIONS

We value the freedom of choice so you can choose your seat when you book The seat reservation is guaranteed a-nd remains yours at all times when aboard the coach

NO SMOKING COMFORT

With the comfort of our passengers in mind coaches on all our Daybreaks are no smokshying throughout In the interests of fellow passengers comfort we kindly ask that smokers observe our no smoking policy On scenic tours and longer journeys ample refreshment stops are provided when of course smoking is permitted

YOUR QUESTIONS ANSWERED

Do I need to book Booking in advance is strongly recommended as all Daybreak tours are subject to demand Subject to availability stand-by tickets can be purchased from the driver

What ti me does the coach leave The coach departs from Cambridge Drummer Street (Bay 12 adjacent to public toilets) at the time shown There are many additional joining points indicated by departure codes in the brochure If you are joining at one of our less popular joining points you will be adshyvised of your pick-up time (normally by telephone) not less than 48 hours before deparshyture In this way we can minimize the length of pick-up routes and reduce journey times for the majority of passengers

CHAPTER 4 Standardized Testing 99

What time do we get back An approximate return time is shown for each excursion The tim~s shown serve as a guide but road conditions can sometimes cause delay If your arrival will be later than advertised your driver will try to allow for a telephone call during the return journey

Where can I board the coach All the Daybreaks in the brochure leave from Cambridge Drummer Street (Bay 12 adjashycent to public toilets) at the time shown Many Daybreaks offer additional pick-ups for pre-booked passengers within Cambridge and the surrounding area This facility must be requested at the time of booking

Academic Writing Writing Task 1 You should spend about 20 minutes on this task

The graph below shows the different modes of transport used to travel to and from work in one European city in 1950 1970 and 1990

[graph shown here]

Write a report for a university lecturer describing the information shown below You should write at least 150 words

Writing Task 2 You should spend about 40 minutes on this task

Present a written argument or case to an educated reader with no specialist knowledge of the folowing topic

It is inevitable that as technology develops so traditional cultures must be lost Technolshyogy and tradition are incompatible-you cannot have both together

To what extent do you agree or disagree with this statement Give reasons for your answer You should write at least 250 words You should use your own ideas knowlshyedge and experience and support your arguments with examples and relevant evidence

General Training Writing Writing Task 1 You should spend about 20 minutes on this task You rent a house through an agency The heating system has stopped working You phoned the agency a week ago but it has still not been mended Write a letter to the agency Explain the situation and teil them what you want them to do about it

You should write at least 150 words You do NOT need to write your own address

Begin your letter as follows

Dear - ___-I

Writing Task 2 You should spend about 40 minutes on this task As part ofa class assignment you have to write about the following topic

100 CHAPTER 4 Standardized Testing

Some businesses now say that no one can smoke cigarettes in any of their offices Some governments have banned smoking in all public placesThis is a good idea but it takes away some of our freedom

Do you agree or disagree Give reasons for your answer You should write at least 250 words

Speaking In each ofthe three parts of the speaking module a specific function is fulfilled In Part 1 the candidates answer general questions about themselves their homes or families their jobs or studies their interests and a range ofsimilar familiar topic areas This part lasts between four and five minutes In Part 2 the candidate is given a verbal prompt on a card and is asked to talk on a particular topic The candidate has one minute to prepare before speaking at length for between one and two minutes The examiner then asks one or two wind-down questions In Part 3 the examiner and candidate engage in a discusshysion of more abstract issues and concepts which are thematically linked to the topic prompt in Part 2 The discussion lasts between four and five minutes

All interviews are recorded on audiocassette Here is a sample ofa Part 2 topic

Describe a teacher who has greatly influenced you in your education

You shou Id say

where you met them what subject they taught what was special about them

and explain why this person influenced you so much

You will have to talk about the topic for 1 to 2 minutes You have 1 minute to think about what you are going to say You can make some notes if you wish

Test of English for International Communication (TOEICreg)

listening

Part 1 Photographs Directions For each question you will see a picture in your test book and you will hear four short statements The statements will be spoken just one time They will not be printed in your test book so you must listen carefully to understand what the speaker says When you hear the four statements look at the picture in your test book and choose the statement that best describes what you see in the picture Then on your answer sheet find the number of the question and mark your answer

[photograph of a scientist looking through a microscope]

You will hear Look at the picture marked number 1 in your test book

(A) Shes speaking into a microphone (B) Shes put on her glasses (C) She has both eyes open (D) Shes using a microscope

CHAPTER 4 Standardized Testing 101

Part 2 Question-Response Directions In this part of the test you will hear a question or statement spoken in Enshyglish followed by three responses also spoken in English The question or staten1ent and the responses will be spoken just one time They will not be printed in your test book so you must listen carefully to understand what the speakers say You are to choose the best response to each question or statement

Question 1 You will hear Ms Morikawa has worked here for a long time hasnt she

(A) At three oclock (B) No Ive lost my watch (C) More than ten years

Question 2 You will hear Which of these papers has a wider circulation

(A) The morning edition (B) Get more exercise (C) By messenger

Part 3 Short Conversations Directions In this part of the test you will hear short conversations between two people The conversations will not be printed in your test book You will hear the conversations only once so you must listen carefully to understand what the speakers say In your test book you will read a question about each conversation The question will be followed by four answers You are to choose the best answer to each question and mark it on your answer sheet

Question 1 (Man) We should think about finding another restaurant for lunch (Woman) Why The food and service here are great

(Man) Yes but the prices are going up every week

You will read Why is this man unhappy with the restaurant

(A) It is too noisy (B) It is too expensive (C) It is too crowded (D) It is too difficult to find

Question 2 (Woman A) How was Dr Borgs recent trip to Singapore (Woman B) She enjoyed the tour of the port very much (Woman A) They say its one of the most active in Asia

You will read 2 What did Dr Borg find interesting

(A) The tourist center (B) The airport (C) The musical performance (D) The harbor

Part 4 Short Talks Directions In this part of the test you vill hear several short talks Each will be spoken just one time They will not be printed in your test book so you must listen carefully to understand and remember what is said In your test book you will read two or more questions about each short talk The questions will be followed by four answers You are to choose the best answer to each question and mark it on your answer sheet

102 CHAPTER 4 Standardized Testing

You will hear Questions 1 and 2 refer to the following announcement

Good afternoon and welcome aboard Nordair Flight 857 from Copenhagen to Bangkok with intermediate stops in Dubai and Calcutta We are preparing for departure in a few minutes At this time your seat back should be returned to its full upright position and your seat belt s~ould be fastened OUf anticipated total flying time to Dubai is six hours and twenty-five minutes I hope you enjoy the flight You will hecJr Now read question 1 in your test book and answer it You will read 1 What is the final destination of the flight

(A) Bangkok (B) Copenhagen (C) Dubai (O) Calcutta

You will hear Now read question 2 in your test book and answer it You will read 2 What will happen in a few minutes

(A) The flight will land in Dubai I

(B) The passengers will board the plane (C) The plane will take off (0) The gate number will be announced

Reading In this section of the test you will have the chance to show how well you understand written English There are three parts to this section with special directions for each part

Part 4 Incomplete Sentences Directions This part of the test has incomplete sentences Four words or phrases marked (A) (8) (e) (D) are given beneath each sentence You are to choose the one word or phrase that best completes the sentence Then on your answer sheet find the number of the question and mark your answer

1 Mr Yangs trip will __ him away from the office for ten days (A) withdraw (B) continue (C) retain (0) keep

2 The company that Marie DuBois started now sells __ products throughout the world (A) its (B) it (C) theirs (D) them

3 If your shipment is not delivered __ Tuesday you can request a full refund for the merchandise (A) at (B) by (C) within (D) while

CHAPTER 4 Standardized Testing 103

Part 6 Error Recognition Directions In this part ofthe test each sentence has four words or phrases underlined The four underlined parts of the sentence are marked (A) (B) (C) (D) You are to identify the one underlined word or phrase that should be corrected or rewritten Then on your answer sheet find the number of the question and mark your answer

1 The pamphlet contains some importance information about the current exhibit ABC D

2 No matter how Jong it taking to finish the annual report it must be done properly ABC D

3 The popularity of jogging appears to have decreased since the past couple of years ABC D

Part 7 Reading Comprehension Directions The questions in this part of the test are based on a selection of reading mateshyrials such as notices letters) forms newspaper and magazine articles) and advertisements You are to choose the one best answer (A) (B) (C) or (OJ to each quesshytion Then on your ariswefsheelfindthe number of the qUestion andmcirkyour answer Answer all questions following each reading selection on thebasis of what is stated or implied in that selection

The Museum ofTechnology is a hands-on museum designed for people to experience science at w()rk~ Visitors are encouraged to use test and handle the objects o~ display Special demonstrations are scheduled for the first and second Wednesdays of each month at 1330 Open Tuesday-Friday 1200-1630 Saturday 1000-1730 and Sunday 11 00-1630

1 When during the month can visitors see special demonstrations (A) Every weekend (B) The first two Wednesdays (C) One afternoon a week (D) Every other Wednesday

Questions 2 and 3 refer to the followi ng notice

NOTICE If you are unable to work because of an extended illness or injury that is not workshyrelated you may be entitled to receive weekly benefits from your employer or the firms insurance company To claim benefits you must file a claim form within thirty days of the first day of your disability Before filing the claim you must ask your doctor to fill in the Doctors Statement on the claim form stating the period of disability

3 To whom is this notice addressed (A) Employers (8) Doctors (C) Employees (D) When paying the bill

4 When must the claim form be filed (A) On the first of the month (8) On the thirtieth of the month (C) On the first day ofdisabifity (D) Within 30 days of the start of disability

Page 13: Standardized Testing Chapter 4 Brown

78 CHAPTER 4 Standardized Testing

4 Make appropriate evaluations of different kinds of items

In Chapter 3 the concepts of item facility (IF) item discrimination (ID) and disshytractor analysis were introduced As the discussion there showed such calculations provide useful infornlation for classroom tests but sometimes the time and effort involved in perfornling them may not be practical especially if the classroom-based test is a one-time test Yet for a standardized multiple-choice test that is designed to be marketed commercially andor administered a number of times andor adminisshytered in a different form these indices are a must

For other types of response formats namely production responses different forms of evaluation become importantThe principles of p-mpoundti~ality ~d poundabWty are prominent along with the concept o(JacjJjt Practicality issues in such items include the clarity of directions timing of the test ease of administration and how much time is required to score responses Reliability is a major player in instances where more than one scorer is employed and to a lesser extent when a single scorer has to evaluate tests over long spans of time that could lead to deterioration of stanshydards Facility is also a key to the validity and success of an item type ~irecshytions complex- language obscure topics fuz~Qata and culturally biased

~Jfiformatioifma~alliead to a highei1eVermiddotof diffiCidty than one desires (A) The IF ID and efficiency statistics of the multiple-choice items of current

forms of the TOEFL are not publicly available information For reasons of security and protection of patented copyrighted materials they must remain behind the closed doors of the ETS development staff Those statistics remain of paramount importance in the ongoing production ofTOEFL items and forms and are the founshydation stones for demonstrating the equatability of forms Statistical indices on retired forms of the TOEFL are available on request for research purposes

The essay portion of theTOEFL undergoes scrutiny for its practicality reliability and facility Special attention is given to reliabilIty since two human scorers must read each essay and every time a third reader becomes necessary (when the two readers disagree by more than one point) it costs ETS more money

(B) In the case of the open-ended responses on the two written tasks on the ESLPT a similar set of judgments must be made Some evaluative impressions of the effectiveness of prompts and passages are gained from informal student and scorer feedback In the developmental stage of the newly revised ESLPT both types of feedshyback were formally solicited through questiQnnaires and interviews That informashytion proved to be invaluable in the revisIon of prompts and stimulus reading passages After each administration now the teacher-scorers provide informal feedshyback on their perceptions of the effectiveness of the prompts and readings

The multiple-choice editing passage showed the value of statistical findings in determining the usefulness of items and pointing administrators toward revisions Following is a sample of the format used

CHAPTER 4- Standardized Testing 79

Multiple-choice editing passage

(1)EYer since supermarkets first appeared they have beentake over ~ world ABC 0

(2) Supermarkets have changed peoples life ~ yet and at the same time changes in ABC

peoples life ~ have encouraged the opening of supermarkets o

The task was to locate the error in each sentence Statistical tests on the experishymental version of this section revealed that a number of the 45 items were found to be of zero IF (no difficulty whatsoever) and of inconsequential discrimination power (some IDs of 15 and lower) Many distractors were of no consequence because they lured no one Such information led to a revision of numerous it~ms and their options eventually strengthening the effectiveness of this section

(C)The GET like its written counterparts in the ESLPT is a test ofwritten ability with a single prompt and therefore questions of practicality and J~~illy~are also largely observational No data are collected from students on their perceptions but the scorers have an opportunity to reflect on the validity ofa given topiC After one sitting a topic is retired which eliininates the possibility of improving a specific topiC but future framing of topics might benefit from scorers evaluations Inter-rater reliability is checked periodically and reader training sessions are modified if too many instances of unreliability appear

5 Specify scoring procedures and reporting formats - ---

A systematic assembly of test items in pre-selected arrangements and sequences all of which are validated to confo~ to an e~pected difficulty level should yield a test that can then be scored accurately and reported back to test-takers and institutions efficiently

(A) Of the three tests being exemplifled here the most straightforward scoring procedure comes from the TO~FL the one with the most complex issues of validashytion deSign and assembly Scores are calculated and reported fora) three sections of the TOEFL (the essay ratings are combined with the Structure and Written Expression score) and (b) a total score (range 40 to 300 on the computer-based TOEFL and 310 to 677 on the paper-and-pencil TOEFL) A separate score (c) for the Essay (range 0 to 6) is also provided on the examinees score record (see simulation of a score record on page 80)

80 CHAPTER 4 Standardized Testing

Facsimile of a TOEFLreg score report

TOEFL Scaled Scores Claudia Y Estudiante Peru ___

19 17 17 177 Listening Structure Writing Reading Total Score

Essay rati ng 30

The rating scale for the essay is virtually the same one that is used for the Test of Written English (see Chapter 9 for details) with a zero level added for no response copying the topic only writing completely off topic or not writing in English

(B) The ESLPT reports a score for each of themiddot essay sections but the rating scale differs between them because in one case the objective is to write a summary and in the other to write a response to a reading ~ch essayi~pd lgtY ~o readet~ ifhFfF js a discrepancy of more than one level a third reader1resolves the differenceThe ~ditiilg section is machine-scanned and -scored with a total score and ~th part-scores for each ofthe grammaticaVrhetorlcal sectionS From these data placement administrators have adequate information to make placements and teachers receive some diagnostic inforshymation on each student in their classes Students do not receive their essays back

(C) Each GET is read by two trained readers who give a score between 1 and 4 according to the following scale

Graduate Essay Test Scoring Guide

Please make no marks on the writers work Write your reader number and score on the front cover of each test booklet

4 Superior The opening establishes context purpose and point of view the body of the essay developsmiddot recommendations-logically and coherently The writer demonshystrates awareness of the complexities in the situation and provides analysis of the probJem offers compelling or common-sense reasons for recommendations made makes underlying assumptions explicit

The writer uses fluent and idiomatic English with few mechanical errors Style reshyveals syntactic maturity is dear and direct is not choppy or over-colloquial nor over-formal stuffy or unfocused Occasional spelling or punctuation errors may be easily attributed to hasty transcription under pressure

3 Competent After an opening that establishes context and purpose the paper unfolds with few lapses in coherence but may have somewhat less clear organization of less explicit transitions than a top-score paper It may have somewhat less compelling logic or slightly less-wellreasoned suggestions than a 4 paper though it will provide reasons for the recommendations made

The writer uses dear fluent and generally idiomatic English but may make minor or infrequent ESL errors (preposition errors dropped articles or verb endings etc) or repeat a single error (eg not punctuate possessive nouns) Occasional lapses of style are offSet by demonstrated mastery of syntax

CHAPTE84 Standardized Testing 81

2 Weak The writer makes somewhat simplistic suggestions not fully supported with reashysons fails to cite key facts offers little analysis of the problem or shows a limited grasp of the situation the given information is copied or listed withlittle integration into argument Points may be random or repetitious Writing may be badly focused with careless use of abstract language resulting in predication errors or illogical sentences

ESL andlor careless mechanical errors are frequent enough to be distracting OR sentences may be choppy style over-casual usage occasionally unidiomatic

1 Inadequate The essay may be disjointed incoherent or minimally developed The writer shows little grasp of the complex issues involved is unable to establish conshytext point of view or purpose in opening of paper or has a poor sense of audience Mechanical andor ESL errors or unidiomatic usages are frequent sentences may be ungrammatical OR correct but short and very simple

The two readers scores are added to yield a total possible score of 2 to 8 Test administrators recommend a score of 6 as the threshold for allowing a student to pursue graduate-level courses Anything below that is accompanied by a recomshymendation that the student either repeat the test or take a remedial course in gradshyuate writing offered in one of several different departments Students receive neither their essays nor any feedback other than the fmal score

6 Perform ongoing construct validation studies

From the above discussion it should be clear that no standardized instrument is expected to be used repeatedly without a ramporou~program of ongoing c~-sectmct valiltiatiOll Any standardized test once developed must be accompanied by sysshy~

tematic periodic corroboration of its effectiveness and by steps toward its improveshyment This rigor is especially true of tests that are produced in equated forms that is forms must be reliable across tests such that a score on a subsequent form of a test-has-the~same validityand-interpretability as its original

(A) The TOEFL program in cooperation with other tests produced by ETS has an impressive program of research Over the years dozens of TOEFL-sponsored research studies have appeared in the TOEFL Monograph Series An early example ofsuch a study was the seminal Duran et aI (1985) study TOEFLfrom a Communicative ViewpOint on Language Proficiency which examined the content characteristics of the TOEFL from a communicative perspective based on current research in applied linguistics and lanshyguage proficiency assessment More recent studies (such as Ginther 2001 Leacock amp Chodorow 2001 Powers et aI 2002) demonstrate an impressive array of scrutiny

(B) For approximately 20 years the ESLPT appeared to be placing students relishyably by means of an essay and a multiple-choice grammar and vocabulary test Over the years the security of the latter became s1lspect and the faculty administrators wished to see some content validity achieved in the process In the year 2000 that process began with a group of graduate students (Imao et aI 2000) in consl1ltation with faculty members and continued to fruition in the form of a new ESLPT reported in lmao (2002) The development of the new ESlPT involved a lengthy process of

82 CHAPTER 4 Standardized Testing

both content and construct validation along with facing such practical issues as scoring the written sections and a machine scorable multiple-choice answer sheet

The process of ongoing validation will no doubt continue as new forms of the editing section are created and as new prompts and reading passages are created for the writing section Such a validation process should also include consistent checks on placement accuracy and on face validity

(C) At this time there is little or no research to validate the GET itself For its conshy struct validation its administrators rely on a stockpile of research on university-level academic writing tests such as theTWEThe holistic scoring rubric and the topics and administrative conditions of the GET are to some extent patterned after that of the TWE In recent years some criticism of the GEf has come from international test-takers (Hosoya 2001) who posit that the topics and time limits of the GET among other facshytors work to the disadvantage of writers whose native language is not English These validity issues remain to be fully addressed in a comprehensive research study

I I

STANDARDIZED IANGUAGE PROFICIENCY TESTING

Tests of language profiCiency presuppose a comprehensive definition of the specific competencies that comprise overall language ability The specifications for the TOEFL provided an illustration of an operational definition of ability for assessment purposes This is not the only way to conceptualize the concept Swain (1990) offered a multidimensional view of profiCiency assessment by referring to three linshyguistic traits (grammar discourse and sociolinguistics) that can be assessed by means of oral multiple-choice and written responses (see Table 41) Swains conshyception was not meant to be an exhaustive analysis of ability but rather to serve as an operational framework for constructing proficiency assessments

Another defmition and conceptualization of profiCiency is suggested by the ACTFL association mentioned earlier ACfFL takes a holistic and more unitary view of proficiency in describing four levels superior advanced intermediate and noviceWithin each level descriptions of listening speaking reading and writing are provided as guidelines for assessment For example the ACfFL Guidelines describe the superior level of speaking as follows

ACTFL speaking guidelines summary superior-level

Superior-level speakers are characterized by the ability to

bull participate fully and effectively in conversations in formal and informal settings on topics related to practical needs and areas of professional andor scholarly interests

bull provide a structured argument to explain and defend opinions and develop effective hypotheses within extended discourse

bull discuss topics concretely and abstractly bull deal with a linguistically unfamiliar situation bull maintain a high degree of linguistic accuracy bull satisfy the linguistic demands of professional andor scholarly life

CHAPTER4 Standardized Testing 83

The other three ACfFL levels use the same parameters in describing progressively lower proficiencies across all four skills Such taxonomie~ have the advantage of considering a number of functions of linguistic discourse but the disadvantage at the lower levels of overly emphasizing test-takers deficiencies

Table 41 Traits of second language proficiency (Swain 1990 p 403)

Trait Grammar Discourse Sociolinguistic

focus on grammatical focus on textual focus on social accuracy within cohesion and appropriateness of sentences coherence language use

Method

Oral structured interview story telling and argumentationpersuasion

role-play ofspeech acts requests offers complaints

scored for accuracy of verbal morphology prepositions syntax

detailed rating for identification logical sequence and time orientation and global ratings for coherence

scored for ability to distinguish formal and informal register

Multiple-choice

sentence-level select the correct form exercise

paragraph-level select the coherent sentence exercise

speech act-Ievelselect the appropriate utterance exercise

(45 items) (29 items) (28 items)

involving verb morphology prepositionsan-d-uther items

Written composition

narrative and letter of persuasion

narrative and letter of persuasion

formal request letter and informal note

scored for accuracy of verb morphology prepositions syntax

detailed ratings much as for oral discourse and global rating for coherence

scored for the ability to distinguish formal and inforJ1lil1 register

FOUR STANDARDIZED lANGUAGE PROFICIENCY TESTS

We now tum to some of the better-known standardized tests of overall language ability or profiCiency to examine some of the typical formats used in commercially available tests We will not look at standardized tests of other specific skills here but that should not lead you to think by any means that proficiency is the only kind of test in the field that is standardized Three standardized oral production tests the

84 CHAPTER 4 Standardized Testing

Test of Spoken English (fSE) the Oral Proficiency Inventory (OPI) and PbonePassreg are discussed in Chapter 7 and the Test of Written English (WE) is covered in ChapterS

Four commercially produced standardized tests of English language proficiency are described briefly in this section the TOEFL the Michigan English Language Assessment Battery (MELAB) the International English Language Testing System (lELTS) and the Test of English for International Communication (fOEICreg) In an appendix to this chapter are sample items from each section of each test When you turn to that appendix use the following questions to help you evaluate these four tests and their subsections

1 What item types are included 2 How practical and reliable does each subsection of each test appear to be 3 Do the item types and tasks appropriately represent a conceptualizatio~ of

language proficiency (ability) That is can you evaluate their construct validity

4 Do the tasks achieve face validity 5 Are the tasks authentic 6 Is there some washback potential in the tasks

Test of English as a Foreign Language (TOEFL)

Producer Educational Testing Service (ETS) Objective To test overall proficiency (language ability) Primary market Almost exclusively US universities and colleges for admission

purposes Type Computer-based (CB) (and two sections are-computer-adaptive)

A traditional paper-based (PB) version is also available Response modes Multiple-choice responses essay Specifications See the box on pp 72-73 Time allocation Up to 4 hours (CB) 3 hours (PB) Internet access wwwtoeflorg

Comments In the North American context the TOEFL is the most widely used comshymercially available standardized test of proficiency Each year the TOEFL test is adminisshytered to approximately 800000 candidates in more than 200 countries It is highly respected because of the thorough program of ongoing research and development conshyducted by ETS The TOEFLs primary use is to set proficiency standards for international students seeking admission to English-speaking universities More than 4200 academic institutions government agencies scholarship programs and licensingcertification agenshycies in more than 80 countries use TOEFL scores By 2004 the TOEFL will include a secshytion on oral production

CHAPTER 4 Standardi~ed Testing 85

Michigan English Language Assessment Battery (MELAB)

Producer English language Institute University of Michigan Objective To test overall proficiency (language ability) Primary market Mostly US and Canadian language programs and colleges

some worldwide educational settings as well Type Paper-based Response modes Multiple-choice responses essay Time allocation 25 to 35 hours Internet access wwwlsaumicheduelimelabhtm

Specifications The MElAB consists of three sections Part 1 a 3D-minute impromptu essay is written on an assigned topic Part 2 a 25-minute multiple-choice listening comshyprehension test is delivered via tape recorder Part 3 is a 100-item 75-minute multipleshychoice test containing grammar doze reading vocabulary and reading comprehension An oral interview (speaking test) is optional

Comments The Ell at the University of Michigan has been producing the MELAB and its earlier incarnation (Michigan Test of English language Proficiency) since 1961 like the TOEFL it serves a North American audience but is also used internationally While its use is not as widespread as the TOEFL its validity is widely respected Because it is cheaper than the TOEFL and more easily obtained it is popular among language schools and institutes Many institutions and companies accept MElAB scores in lieu ofTOEFL scores

International English Language Testing System (IELTS)

Producer Jointly managed by The University of Cambridge local Examinations Syndicate (UClES) The British Council and lOP Education Australia

Objective To test overall proficiency (language ability) Primary-market Australian British Canadian and New Zealand academic

institutions and professional organizations American academic institutions are increasingly accepting IELTS for admissions purposes

1)rpe Computer-based (for the Reading and Writing sections) papershybased for the listening and Speaking modules

Response modes Multiple-choice responses essay oral production Time allocation 2 hours 45 minutes Internet access httpwwwieltsorgl

httpwwwudesorguk httpwwwbritishcouncilorg

Specifications Reading candidates choose between academic reading or general training reading (60 minutes) Writing the same option academic writing or general training writing (60 minutes) Listening four sections for all candidates (30 minutes) Speaking five sections for all candidates (1015 minutes)

86 CHAPTER 4 Standardized Testing

Comments The University of Cambridge local Examinations Syndicate (UCLES) has been producing English language tests since 1858 Now with three organizations cooperatshying to form the IELTS more than a million examinations are administered every year In 2002 a computer-based version of the Reading and Writing modules of the IELTS became available at selected centers around the world The other sections are administered locally by an examinet The paper-based IELTS remains an option for candidates The IELTS retains the distinct advantage of requiring all four skills in the test-takers performance

Test of English for International Communication (TOEICreg)

Producer The Chauncey Group International a subsidiary of Educational Testing Service

Objelttive To test overall proficiency (langlJage ability) Primary market Worldwide business commerce and industry contexts

(workplace settings) Type Computer-based and paper-based versions Response modes Multiple-choice responses Time allocation 2 hours Internet access httpwwwtoeiccom

Specifications Listening Comprehension 100 items administered by audiocassette Four types of task statements questions short conversations and short talks (approxishymately 45 minutes) Reading 100 items Three types of task cloze sentences error recogshynition and reading comprehension (75 minutes)

Comments The TOEIC has become a very widely used international test of English proficiency in workplace settings where English is required for job performance The conshytent includes many different employment settings such as conferences presentations sales ordering shipping schedules reservations (etters and memoranda It is approprishyate to use in educational settings where vocational or workplace English courses are being offered

sect sect sect sect sect

The construction of a valid standardized test is no minor accomplishment whether the instrument is large- or small-scale The designing of specifications alone as this chapter illustrates requires a sophisticated process of construct valishydation coupled with considerations of practicality Then the construction of items and scoringinterpretation procedures may require a lengthy period of trial and error with prototypes of the final form of the testWith painstaking attention to all the details of construction the end product can result in a cost-effective timeshysaving accurate instrument Your use of the results of such assessments can provide useful data on learners language abilities But your caution is warranted as well for all the reasons discussed in this chapter The next chapter will elaborate on what lies behind that need for a cautious approach to standardized assessment

CHAPTER4 Standardized Testing 87

EXERCISES

[Note (I) Individual work (G) Group or pair work (C) Whole-class discussion]

1 (C) Tell the class about the worst test experience youve ever had Briefly anamiddot lyze what made the experience so unbearable and try to come up with sugshygestions for improvement of the test andor its administrative conditions

2 (G) In pairs or small groups compile a brief list of pros and cons of standardshyized testing Cite illustrations of as many items in each list as possible Report your lists and examples to the rest o~ the class

3 (I) Select a standardized test that you are quite familiar with (probably a recent experience) Mentally evaluate that test using the five principles of practicality reliability validity authenticity and washback Report yourevaluashytion to the class

4 (G) The appendix to this chapter provides sample items from Jour different tests of language proficiency In groups one test for each group analyze your test for (a) content validity (b) face validity and (c) authenticity

5 (C) Do you think that the sample TOEFL reading passage about pirates (pages 74-75) and the Graduate EssayTest prompt (pages 76-77) about a school board hiring committee have any culture bias Discuss this and other cultural biases you have noticed in tests Is it possible to design a test that is completely free of culture bias

6 (CG) Compare the differences in conceptualization of language proficiency represented by Swains model the TOEFL and the ACfFL philosophy Which one best represents current thinking about communicative language ability What are the strengths and weaknesses of each approach

FORYOlIILEURTHER READING

Gronlund Norman E (1998) Assessment of student achievement Sixth Edition Boston Allyn and Bacon

Gronlunds classic also mentioned in Chapter 3 offers a concise overview of features of standardized tests offering definitions and examples of the statistical considerations in interpreting scores His approach is unbiased cleady written and accessible to those who might fear the mathematics of standardized testing

Phillips Deborah 2001 Long1nan introductory course for the TOEFL test White Plains NY Pearson Education

A careful examination of this or any other reputable preparation course for a standardized language test is well worth a students time Note especially how the book acquaints the user with the specifications of the test and offers a number of useful strategie~ that can be llsed in preparation for the test and during irs adn1inistration

88 CHAPTER 4 Standardized Testing

APPENDIX TO CHAPTER 4

Commercial Proficiency Tests Sample Items and Tasks

Test of English a~ a Foreign Language (TOEFLreg)

Listening r

Part A

In this section you will hear short conversations between two people In some ofthe conversations each person speaks only once In other conversations one or both of the people speak more than once Each conversation is followed by one questionabQlt it Each question in this part has four answer choices You should click on the best answer to each question Answer the questions on the basis of what is stated or implied by the speakers Here is an example On the computerscreen you will see

[man and woman talking]

On the recording you will hear

(woman) Hey wheres your sociology book (man) At home Why carry it around when were just going to be taking

a test (woman) Dont you remember Professor Smith said we could us it during

the test (man) Ohl no Well Ive still got an hour right Im so glad I ran into you

You wiII then see and hear the question before the answer choices appear

What will the man probably do next

o Begin studying for the sociology test o Explain the problem to his professor o Go home to get his textbook o Borrow the womans book

To choose an answer you will click on an oval The oval next to that answer will darken After you click on Next and Confirm Answer the next conversation will be presented

Part B

In this section you will hear several longer conversations and talks Each conversation or talk is followed by several questions The conversations talks and questions will not be repeated The conversations and talks are about a variety of topics You do not need speshycial knowledge of the topics to answer the questions correctly Rather you should answer each question on the basis of what is stated or implied by the speakers in the conversashytions or talks

For most of the questions you will need to click on the best of four possible answers Some questions will have special directions The special directions will appear in a box on the computer screen Here is an exampie ot a conversation and some questions

CHAPTER 4 Standardized Testing 89

Marine Biology (narrator) Listen to part of a discussion in a marine biology class

(professor) A few years ago our local government passed a number of strict environmental laws As a result Sunrise Beach looks nothing Ii ke it did ten years ago The water is cleaner and theres been a tremendous increase in all kinds of marine life which is why were going there on Thursday

(woman) I dont know if I agree that the water quality has improved I mean I was out there last weekend and it looked all brown It didnt seem too clean to me

(professor) Actually the color of the water doesnt always indicate whether its polluted The brown color you mentioned might be a result of pollution or it can mean a kind of brown algae is growing there Its called devils apron and it actually serves as food for whales

(man) So when does the water look blue (professor) Well water thats completely unpolluted is actually colorless But

it often looks bluish-green because the sunlight can penetrate deep down and thats the color thats reflected

(woman) But sometimes it looks really green Whats that about (professor) Ok well its the same principle as with devils apron the

water might be green because of different types of green algae there-gulfweed phytoplankton You all should finish reading about algae and plankton before we go In fact those are the types of living things Im going to ask you to be looking for when were there

Now get ready to answer the questions

What is the discussion mainly about

o The importance of protecting ocean environments o The reasons why ocean water appears to be different colors o The survival of whales in polluted water o The effect that colored ocean water has on algae

To choose an answer click on an oval The oval next to that answer will darken After you click on Next and Confirm Answer the next question will be presented

According to the professor what can make ocean water look browngt

o Pollution o Cloudy Skies o Sand o Algae

Click on 2 answers

To choose your answers you will click on the squares An XII wiii appear in each square

bullbullbullbullbullbullbull

90 CHAPTER 4 Standardized Testing

Structure and Written Expression This section measures the ability to recognize language that is appropriate for standard written English There are two types ofquestions in this section In the first type ofquestion there are incomplete sentences Beneath each sentence there are four words or phrases

Directions CIiSk on the one word or phrase that best completes the sentence

The colum~ine flower __ to nearly all of the United States can be raised from seed in almost any garden

native how native is how native is it is native

Time Help Confirm

After you click on Next and Confirm Answ~ the next question willbe presented

The second type of question has four underlined words or phrases You will choose the one underlined word or phrase that must be changed for the sentence to be correct

Directions Click on the one underlined word or phrase that must be changed for the senshytence to be correct

One of the most difficult problems in understanding sleep is determining what the funcshytions of sleep ~

lime Help Confirm

Clicking on an underlined word or phrase will darken it

Reading This section measures the ability to read and understand short passages similar in topic and style to those that students are likely to encounter in North American universities and colleges This section contains reading passages and questions about the passages There are several different types of questions in this section In the Reading section you will first have the opportunity to read the passage

The temperature of the Sun is over 10000 degrees Fahrenheit at the surface but it rises perhaps more than 270000000 at the center The Sun is so much hotter than the Earth that matter can exist only as a gasl except perhaps at the core In the core of the Sun the pressures are so great that despite the high temperature there may be a small solid core However no one really knows since the center of the Sun can never be directly observed ~ Solar astronomers do know that the Sun is divided into five general layers or zones Starting at the outside and going down into the Sun the zones are the corona chromoshysphere hotosphere convection zone and finally the core The first three zones are reshygarded as the Suns atmosphere But since the Sun has no solid surface it is hard to middottell where the atmosphere ends and the main body of the Sun begins

The Suns outermost layer begins about 10000 miles above the visible surface and goes outward for millions of miles This is the only part of the Sun that can be seen during an eclipse such as the one in February 1979 At any other time the corona can be seen

bullbullbullbullbullbullbull

bull bullbullbullbullbullbull

CHAPTER 4 Standardized Testing 91

only when special instruments are used on cameras and telescopes to block the light from the photosphere

The corona is a brilliant pearly white filmy light about as bright as the full Moon Its beautiful rays are a sensational sight during an eclipse The coronas rays flash out in a brilliant fan that has wispy spikelike rays near the Suns north and south poles The corona is generally thickest at the Suns equator The corona is made up of gases streamshying outward at tremendous speeds that reach a temperature of more than 2 million deshygrees Fahrenheit The gas thins out as it reaches the space around the planets By the time the gas of the corona reaches the Earth it has a relatively low density

When you have finished reading the passage you will use the mouse to click on Proceed Then the questions about the passage will be presented You are to choose the one best anshyswer to each question Answer all questions about the information in a passage on the basis ofwhat is stated or implied in that passage Most ofthe questions will be multiple-choice questions To answer these questions you will click on a choice below the question

With what topic is paragraph 2 mainly concerned

o How the Sun evolved o The structure of the Sun o Why scientists study the Sun o The distaflce of the Sun from the planets

Paragraph 2 is marked with an arrow (~)

You will see the next question after you click on Next

To answer some questions you will click on a word or phrase Here is an example

Look at the word one in the passage Click on the word or phrase in the bold text that one refers to To answer you can click on any part of the word or phrase in the passage Jour choice will darken to show which word you have chosen

The Suns outermost layer begins about 10000 miles above the visible surface and goes outward for millions of miles This is the only part of the Sun that can be seen durshying an eclipse such as the one in February 1979 At any other time the corona can be seen only when special instruments are used on cameras and telescopes to block the Iight from the photosphere

You will see the next question after you click on~ To answer some q~estions you will click on a sentence in the passage Here is an example

~ The corona is a brilliant pearly white filmy light about as bright as the full Moon Its beautiful rays are a sensational sight during an eclipse The coronas rays flash out in a brilliant fan that has wispy spikelike rays near the Suns north and south poles The corona is generally thickest at the Suns equator ~ The corona is made up of gases streaming outward at tremendous speeds that reach a temperature of more than 2 million degrees Fahrenheit The gas thins out as it reaches the space around the planets By the time the gas of the corona reaches the Earth it has a relatively low density

bull bullbullbullbullbullbull

92 CHAPTER 4 Standardized Testing

Click on the sentence in paragraph 4 or 5 in which the author compares the light of the Suns outermost layer to that ofanother astronomical body Paragraphs 4 and 5 are marked with arrows (~)

To answer some questions you will click on a square to add a sentence to the passage Here is an example -The following sentence can be added to paragraph 1

At the center of the Earths solar system lies the Sun

Where would it best fit in paragraph I Click on the square to add the sentence to the paragraph

D The temperature of the Sun is over 10000 degrees Fahrenheit at the surface but it rises to perhaps morethan 27000000deg at the center 0 The Sun is so much hotter than the Earth that matter can exist only as a gasi except p~rHapsatth~ c6relp the c~re of the ii Sun the pressures are so great that despite the high temperature there may be a small solid core D However no one really knows since the center of the Sun can never be directly observed D 0100

When you click on a square the sentence will appear in the passage at the place you have chosen You can read the sentence added to the paragraph to see if this is the best place to add it You can click on another square to change your answer The sentence will be added and shown in a dark box

Writing In this section you will have an opportunity to demonstrate your ability to write in Enshyglish This includes the ability to generate and organize ideas to support those ideas with examples or evidence and to compose in standard written English in response to an asshysigned topic You will have 30 minutes to write your essay on that topic You must write on the topic you are assigned An essay on any other topic will receive a score of 0 Read the topic below and then make any notes that will help you plan your response Begin typing your response in the box at the bottom of the screen or write your answer on the answer sheet provided to you

Following is a sample topic

Do you agree or disagree with the following statemenH

Teachers should make learning enjoyable and fun for their students

Use specific reasons and examples to support your opinion

CHAPTER 4 Standarczed Testing 93

Michigan English Language Assessment Battery (MELAB)

Composition The time limit for the composition is 30 minutes You must write on only one of the top~

ics below If you write about something else your composition paper will not be graded and you cannot be given a final score If you do not understand the topics ask the exam~ iner to explain or to translate them You may be asked to give your opinion ofsomething and explain why you believe this to describe something from your experience or to exshyplain a problem and offer possible solutions You should write at least one page Some sample topics are

1 What do you think is your countrys greatest problem Explain in detail and tell what you think can be done about it

2 What are the characteristics of a good teacher Explain and give examples 3 An optimist is someone who sees the good side of things A pessimist sees the

bad side Are you an optimist or a pessimist Relate a personal experience that shows this

4 In your opinion are the benefits of space exploration really worth the enormous costs Discuss

Most MELAB compositions are one or two pages long (about 200-300 words) If your paper is extremely short (less than 150 words) your composition will be given a lower score Before you begin writing you might want to take 2 or 3 minutes to plan your comshyposition and to make a short outline to organize your thoughts Such outlines will not be graded they are only to help you You should use the last 5 minutes to read through your composition and to make changes or corrections

Your composition will be graded on how clearly you express yourself in English and on the range of English you are able to use and your control in doing so This means your composition should be well organized your arguments should be fully developed and you should show a range ofgrammatical structures and broad vocabulary Compositions that consist only of very short sentences and very simple vocabulary cannot be given the

middothighest scores If errors are not frequent and if they do not confuse your meaning they will not lower your score very much

Listening Now you will hear a short lecture You may take notes during the lecture Following the lecture you will be asked some questions about it

Therell be a two-week exhibit of the paintings of the little-known master Laura Bernhart at the Claire Osmond Galleries starting on the fifteenth of the month and running through the thirtieth Bernharts known for her innovative designs in abstract expressionism Though a true original she declared a spiritual heritage from Salvador Dali the famous Spanish painter Since Bernhart lived a rather solitary life and died while only in her twenties few people are aware of her works This showing at the Osmond Galleries will provide many with an introduction to her works

10 Where is the exhibit a the Art Museum b the Dali Galleries c the Osmond Galleries

94 CHAPTER 4 Standardized Testing

11 What is Bernhart known for a her copies of Dalis paintings b the originality of her designs c her exhibitions

12 What will going to the exhibit allow most people to do a to see Saivador Dalis paintings b to see Bernharts works for the first time c to learn about Spanish art

Grammar

1 What did the teacher just tell you

She reminded our notebooksI a us to bring b that we bring c our bringing d we should bring

2 Is Bill a good dancer

Not really __ he tries very hard a in spite of h despite c even though d while

3 your clothes are all wet1

Yes I didnt come __ the rain soon enough a away to b over to c down with d in from

Cloze In years to come zoos will not only be places where animals are exhibited to the public but repositories where rare species can be saved from extinction (7) captive breeding The most powerful force (8) the future of many animals-and of zoos-is the decline of the wild (9) even zoo directors would argue that (10) are better places for animals than the fields and forest of their native (11) yet zoos may be the last chance for some creatures that would otherwise pass qUietly into oblivion

7 a through c from b of d damage

8 a bringing c to b that d influencing

9 a But c Not b So d Then

10 a where c even b zoos d wilds

11 alands c residence b life d field

CHAPTER 4 Standardized Testing 95

Vocabulary

12 Mark has a flair for writing a need b purpose c talent d dislike

13 Bill Collins launched his restaurant last June a moved b started c sold d bought

14 John will not accept the censure a burden b blame c credit d decision

15 I cant think of the answer Can you give me a __ a hint b token c taste d gaze

16 Because fewer people are taking expensive vacations the tourist industry is in a a choke b grope c grumble d slump

17 I disagree with a few of his opinions but __ we agree a deliberately b conclusively c essentially d immensely

Reading The influenza virus is a single molecule built from many millions of single atoms You must have heard of the viruses which are sometimes called living molecules While bacteria can be considered as a type of plant secreting pOisonous substances into the body of the organism they attack viruses are living organisms themselves We may conshysider them as regular chemical molecules since they have a strictly aefined atomic strucshyture but on the other hand we must also consider them as being alive since they are able to multiply in unlimited quantities

18 According to the passage bacteria are a poisons

b larger than viruses c very small d plants

96 CHAPTER 4 Standardized Testing

19 The writer says that viruses are alive because they a have a complex atomic structure b move c multiply d need warmth and light

20 The atomic structure of viruses a is -tJIariable b is strictly defined c cannot be analyzed chemically d is more complex than that of bacteria

International English Language Testing System (fELTS)

I

listening

The Listening Module has four sections The first two sections are concerned with social needs There is a conversation between two speakers and then a monologue For examshyple a conversation about travel arrangements or decisions on a night out and a speech about student services on a university campus or arrangements for meals during a confershyence The final two sections are concerned with situations related more closely to educashytional or training contexts For example conversation between a tutor and a student about an assignment or between three students planning a research project and a lecture or talk ofgeneral academic interest All the topics are ofgeneral interest and it makes no difference what subjects candidates study Tests and tasks become more difficult as the sections progress A range of English accents and dialects are used in the recording which reflects the international usage of IELTS

Academic Reading [A 7S0-word article on-th-e- topic of Wind Power in the US with a short glossary at the end]

Questions 1-5

Complete the summary below

Choose your answers from the box below the summary and write them in boxes 1-5 on your answer sheet Note There are more words or phrases than you will need to fill the gaps You may use any word or phrase more than once

Example The failure during the late 1970s and early 19805 of an attempt to establish a widespread wind power industry in the United States resulted largely from the (1) bull in oil prices during this period The industry is now experiencing a steady (2) due to improveshyments in technology and an increased awareness of the potential in the power of wind The wind turbines that are now being made based in part on the (3) of wide- ranging research in Europe are easier to manufacture and maintain than their predecesshysors This has led wind-turbine makers to be able to standardise and thus minimize (4) There has been growing (S) of the importance of wind power as an energy source

CHAPTER 4 Standardized Testing 97

criticism stability skepticism success operating costs decisions design costs fall effects production costs growth decline failure recognition results

Questions 6-1 0 Look at the following list of issues (Questions 6-10) and implications (A-C) Match each issue with one implication Write the appropriate letters A-C in boxes 6-10 on your anshyswer sheet

Example The current price of one wind-generated kilowatt Answer

6 The recent installation of systems taking advantage of economies of scale

7 The potential of meeting one fifth of current U5 energy requirements by wind power

8 The level of acceptance of current wind turbine technology

9 A comparison of costs between conventional and wind power sources

10 The view of wind power in the European Union

Implications

A provides evidence against claims that electricity produced from wind power is relatively expensive

B supports claims that wind power js an important source of energy

C opposes the view that wind power technology requires further-development

General Training Reading Read the passage on Daybreak trips by coach and look at the statements below On your answer sheet write

TRUE if the statement is true FALSE jf the statement is false

NOlGIVEN if the information is not given in the leaflet

1 MiIlers Coaches owns Cambridges Cam bus fleet

2 Premier is an older company than Millers

3 Most of the Daybreak coaches are less than 5 years old

4 Daybreak fares are more expensive than most of their competitors

5 Soft drinks and refreshments are served on most longer journeys

6 Smoking is permitted at the rear of the coach on longer journeys

7 Tickets must be bought in advance from an authorised Daybreak agent

6 Tickets and seats can be reserved by phoning the Daybreak Hotline

9 Daybreak passengers must join their coach at Cambridge Drummer Street

10 Daybreak cannot guarantee return times

98 CHAPTER 4 Standardized Testing

FROM CAMBRIDGE AND SURROUNDING AREA

SPRING IS INTHEAIR

Welcome to our Spring Daybreak programme which continues the tradition of offering unbeatable value for money day trips and tours All the excursions in this brochure will be operated by Pr~mier Travel Services Limited or Millers Coaches both companies are part of the CHLGroup owners of Cambridges Cambus fleet

WERE PROUD OF OUR TRADITION

Premier was established in 1936 the Company now offers the highest standards of coaching in todays competitive operating environment Miller has an enviable reputation stretching back over the past 20 years offering coach services at realistic prices Weve traveled a long way since our early days of pre-war seaside trips Now our fleet of 50 modern coaches (few are more than five years old) operate throughout Britain and Europe but were pleased to still maintain the high standards of quality and service the trademark of our founders nearly sixty years ago

EXCLUSIVE FEATURES

Admission-inclusive fares All Daybreak fares (unless specifically otherwise stated) include admission charges to the attractions shows and exhibits we visit Many full-day scenic tours are accompanied by a fully trained English Tourist Board Blue Badge guide or local experienced driverguide Some Daybreaks include lunch or afternoon tea Compare our admission inclusive fares and see how much you save Cheapest is not the best and value for money is guaranteed If you compare our bargain Daybreak fares beware--most of our competishytors do not offer an all-inclusive fare

SEAT RESERVATIONS

We value the freedom of choice so you can choose your seat when you book The seat reservation is guaranteed a-nd remains yours at all times when aboard the coach

NO SMOKING COMFORT

With the comfort of our passengers in mind coaches on all our Daybreaks are no smokshying throughout In the interests of fellow passengers comfort we kindly ask that smokers observe our no smoking policy On scenic tours and longer journeys ample refreshment stops are provided when of course smoking is permitted

YOUR QUESTIONS ANSWERED

Do I need to book Booking in advance is strongly recommended as all Daybreak tours are subject to demand Subject to availability stand-by tickets can be purchased from the driver

What ti me does the coach leave The coach departs from Cambridge Drummer Street (Bay 12 adjacent to public toilets) at the time shown There are many additional joining points indicated by departure codes in the brochure If you are joining at one of our less popular joining points you will be adshyvised of your pick-up time (normally by telephone) not less than 48 hours before deparshyture In this way we can minimize the length of pick-up routes and reduce journey times for the majority of passengers

CHAPTER 4 Standardized Testing 99

What time do we get back An approximate return time is shown for each excursion The tim~s shown serve as a guide but road conditions can sometimes cause delay If your arrival will be later than advertised your driver will try to allow for a telephone call during the return journey

Where can I board the coach All the Daybreaks in the brochure leave from Cambridge Drummer Street (Bay 12 adjashycent to public toilets) at the time shown Many Daybreaks offer additional pick-ups for pre-booked passengers within Cambridge and the surrounding area This facility must be requested at the time of booking

Academic Writing Writing Task 1 You should spend about 20 minutes on this task

The graph below shows the different modes of transport used to travel to and from work in one European city in 1950 1970 and 1990

[graph shown here]

Write a report for a university lecturer describing the information shown below You should write at least 150 words

Writing Task 2 You should spend about 40 minutes on this task

Present a written argument or case to an educated reader with no specialist knowledge of the folowing topic

It is inevitable that as technology develops so traditional cultures must be lost Technolshyogy and tradition are incompatible-you cannot have both together

To what extent do you agree or disagree with this statement Give reasons for your answer You should write at least 250 words You should use your own ideas knowlshyedge and experience and support your arguments with examples and relevant evidence

General Training Writing Writing Task 1 You should spend about 20 minutes on this task You rent a house through an agency The heating system has stopped working You phoned the agency a week ago but it has still not been mended Write a letter to the agency Explain the situation and teil them what you want them to do about it

You should write at least 150 words You do NOT need to write your own address

Begin your letter as follows

Dear - ___-I

Writing Task 2 You should spend about 40 minutes on this task As part ofa class assignment you have to write about the following topic

100 CHAPTER 4 Standardized Testing

Some businesses now say that no one can smoke cigarettes in any of their offices Some governments have banned smoking in all public placesThis is a good idea but it takes away some of our freedom

Do you agree or disagree Give reasons for your answer You should write at least 250 words

Speaking In each ofthe three parts of the speaking module a specific function is fulfilled In Part 1 the candidates answer general questions about themselves their homes or families their jobs or studies their interests and a range ofsimilar familiar topic areas This part lasts between four and five minutes In Part 2 the candidate is given a verbal prompt on a card and is asked to talk on a particular topic The candidate has one minute to prepare before speaking at length for between one and two minutes The examiner then asks one or two wind-down questions In Part 3 the examiner and candidate engage in a discusshysion of more abstract issues and concepts which are thematically linked to the topic prompt in Part 2 The discussion lasts between four and five minutes

All interviews are recorded on audiocassette Here is a sample ofa Part 2 topic

Describe a teacher who has greatly influenced you in your education

You shou Id say

where you met them what subject they taught what was special about them

and explain why this person influenced you so much

You will have to talk about the topic for 1 to 2 minutes You have 1 minute to think about what you are going to say You can make some notes if you wish

Test of English for International Communication (TOEICreg)

listening

Part 1 Photographs Directions For each question you will see a picture in your test book and you will hear four short statements The statements will be spoken just one time They will not be printed in your test book so you must listen carefully to understand what the speaker says When you hear the four statements look at the picture in your test book and choose the statement that best describes what you see in the picture Then on your answer sheet find the number of the question and mark your answer

[photograph of a scientist looking through a microscope]

You will hear Look at the picture marked number 1 in your test book

(A) Shes speaking into a microphone (B) Shes put on her glasses (C) She has both eyes open (D) Shes using a microscope

CHAPTER 4 Standardized Testing 101

Part 2 Question-Response Directions In this part of the test you will hear a question or statement spoken in Enshyglish followed by three responses also spoken in English The question or staten1ent and the responses will be spoken just one time They will not be printed in your test book so you must listen carefully to understand what the speakers say You are to choose the best response to each question or statement

Question 1 You will hear Ms Morikawa has worked here for a long time hasnt she

(A) At three oclock (B) No Ive lost my watch (C) More than ten years

Question 2 You will hear Which of these papers has a wider circulation

(A) The morning edition (B) Get more exercise (C) By messenger

Part 3 Short Conversations Directions In this part of the test you will hear short conversations between two people The conversations will not be printed in your test book You will hear the conversations only once so you must listen carefully to understand what the speakers say In your test book you will read a question about each conversation The question will be followed by four answers You are to choose the best answer to each question and mark it on your answer sheet

Question 1 (Man) We should think about finding another restaurant for lunch (Woman) Why The food and service here are great

(Man) Yes but the prices are going up every week

You will read Why is this man unhappy with the restaurant

(A) It is too noisy (B) It is too expensive (C) It is too crowded (D) It is too difficult to find

Question 2 (Woman A) How was Dr Borgs recent trip to Singapore (Woman B) She enjoyed the tour of the port very much (Woman A) They say its one of the most active in Asia

You will read 2 What did Dr Borg find interesting

(A) The tourist center (B) The airport (C) The musical performance (D) The harbor

Part 4 Short Talks Directions In this part of the test you vill hear several short talks Each will be spoken just one time They will not be printed in your test book so you must listen carefully to understand and remember what is said In your test book you will read two or more questions about each short talk The questions will be followed by four answers You are to choose the best answer to each question and mark it on your answer sheet

102 CHAPTER 4 Standardized Testing

You will hear Questions 1 and 2 refer to the following announcement

Good afternoon and welcome aboard Nordair Flight 857 from Copenhagen to Bangkok with intermediate stops in Dubai and Calcutta We are preparing for departure in a few minutes At this time your seat back should be returned to its full upright position and your seat belt s~ould be fastened OUf anticipated total flying time to Dubai is six hours and twenty-five minutes I hope you enjoy the flight You will hecJr Now read question 1 in your test book and answer it You will read 1 What is the final destination of the flight

(A) Bangkok (B) Copenhagen (C) Dubai (O) Calcutta

You will hear Now read question 2 in your test book and answer it You will read 2 What will happen in a few minutes

(A) The flight will land in Dubai I

(B) The passengers will board the plane (C) The plane will take off (0) The gate number will be announced

Reading In this section of the test you will have the chance to show how well you understand written English There are three parts to this section with special directions for each part

Part 4 Incomplete Sentences Directions This part of the test has incomplete sentences Four words or phrases marked (A) (8) (e) (D) are given beneath each sentence You are to choose the one word or phrase that best completes the sentence Then on your answer sheet find the number of the question and mark your answer

1 Mr Yangs trip will __ him away from the office for ten days (A) withdraw (B) continue (C) retain (0) keep

2 The company that Marie DuBois started now sells __ products throughout the world (A) its (B) it (C) theirs (D) them

3 If your shipment is not delivered __ Tuesday you can request a full refund for the merchandise (A) at (B) by (C) within (D) while

CHAPTER 4 Standardized Testing 103

Part 6 Error Recognition Directions In this part ofthe test each sentence has four words or phrases underlined The four underlined parts of the sentence are marked (A) (B) (C) (D) You are to identify the one underlined word or phrase that should be corrected or rewritten Then on your answer sheet find the number of the question and mark your answer

1 The pamphlet contains some importance information about the current exhibit ABC D

2 No matter how Jong it taking to finish the annual report it must be done properly ABC D

3 The popularity of jogging appears to have decreased since the past couple of years ABC D

Part 7 Reading Comprehension Directions The questions in this part of the test are based on a selection of reading mateshyrials such as notices letters) forms newspaper and magazine articles) and advertisements You are to choose the one best answer (A) (B) (C) or (OJ to each quesshytion Then on your ariswefsheelfindthe number of the qUestion andmcirkyour answer Answer all questions following each reading selection on thebasis of what is stated or implied in that selection

The Museum ofTechnology is a hands-on museum designed for people to experience science at w()rk~ Visitors are encouraged to use test and handle the objects o~ display Special demonstrations are scheduled for the first and second Wednesdays of each month at 1330 Open Tuesday-Friday 1200-1630 Saturday 1000-1730 and Sunday 11 00-1630

1 When during the month can visitors see special demonstrations (A) Every weekend (B) The first two Wednesdays (C) One afternoon a week (D) Every other Wednesday

Questions 2 and 3 refer to the followi ng notice

NOTICE If you are unable to work because of an extended illness or injury that is not workshyrelated you may be entitled to receive weekly benefits from your employer or the firms insurance company To claim benefits you must file a claim form within thirty days of the first day of your disability Before filing the claim you must ask your doctor to fill in the Doctors Statement on the claim form stating the period of disability

3 To whom is this notice addressed (A) Employers (8) Doctors (C) Employees (D) When paying the bill

4 When must the claim form be filed (A) On the first of the month (8) On the thirtieth of the month (C) On the first day ofdisabifity (D) Within 30 days of the start of disability

Page 14: Standardized Testing Chapter 4 Brown

CHAPTER 4- Standardized Testing 79

Multiple-choice editing passage

(1)EYer since supermarkets first appeared they have beentake over ~ world ABC 0

(2) Supermarkets have changed peoples life ~ yet and at the same time changes in ABC

peoples life ~ have encouraged the opening of supermarkets o

The task was to locate the error in each sentence Statistical tests on the experishymental version of this section revealed that a number of the 45 items were found to be of zero IF (no difficulty whatsoever) and of inconsequential discrimination power (some IDs of 15 and lower) Many distractors were of no consequence because they lured no one Such information led to a revision of numerous it~ms and their options eventually strengthening the effectiveness of this section

(C)The GET like its written counterparts in the ESLPT is a test ofwritten ability with a single prompt and therefore questions of practicality and J~~illy~are also largely observational No data are collected from students on their perceptions but the scorers have an opportunity to reflect on the validity ofa given topiC After one sitting a topic is retired which eliininates the possibility of improving a specific topiC but future framing of topics might benefit from scorers evaluations Inter-rater reliability is checked periodically and reader training sessions are modified if too many instances of unreliability appear

5 Specify scoring procedures and reporting formats - ---

A systematic assembly of test items in pre-selected arrangements and sequences all of which are validated to confo~ to an e~pected difficulty level should yield a test that can then be scored accurately and reported back to test-takers and institutions efficiently

(A) Of the three tests being exemplifled here the most straightforward scoring procedure comes from the TO~FL the one with the most complex issues of validashytion deSign and assembly Scores are calculated and reported fora) three sections of the TOEFL (the essay ratings are combined with the Structure and Written Expression score) and (b) a total score (range 40 to 300 on the computer-based TOEFL and 310 to 677 on the paper-and-pencil TOEFL) A separate score (c) for the Essay (range 0 to 6) is also provided on the examinees score record (see simulation of a score record on page 80)

80 CHAPTER 4 Standardized Testing

Facsimile of a TOEFLreg score report

TOEFL Scaled Scores Claudia Y Estudiante Peru ___

19 17 17 177 Listening Structure Writing Reading Total Score

Essay rati ng 30

The rating scale for the essay is virtually the same one that is used for the Test of Written English (see Chapter 9 for details) with a zero level added for no response copying the topic only writing completely off topic or not writing in English

(B) The ESLPT reports a score for each of themiddot essay sections but the rating scale differs between them because in one case the objective is to write a summary and in the other to write a response to a reading ~ch essayi~pd lgtY ~o readet~ ifhFfF js a discrepancy of more than one level a third reader1resolves the differenceThe ~ditiilg section is machine-scanned and -scored with a total score and ~th part-scores for each ofthe grammaticaVrhetorlcal sectionS From these data placement administrators have adequate information to make placements and teachers receive some diagnostic inforshymation on each student in their classes Students do not receive their essays back

(C) Each GET is read by two trained readers who give a score between 1 and 4 according to the following scale

Graduate Essay Test Scoring Guide

Please make no marks on the writers work Write your reader number and score on the front cover of each test booklet

4 Superior The opening establishes context purpose and point of view the body of the essay developsmiddot recommendations-logically and coherently The writer demonshystrates awareness of the complexities in the situation and provides analysis of the probJem offers compelling or common-sense reasons for recommendations made makes underlying assumptions explicit

The writer uses fluent and idiomatic English with few mechanical errors Style reshyveals syntactic maturity is dear and direct is not choppy or over-colloquial nor over-formal stuffy or unfocused Occasional spelling or punctuation errors may be easily attributed to hasty transcription under pressure

3 Competent After an opening that establishes context and purpose the paper unfolds with few lapses in coherence but may have somewhat less clear organization of less explicit transitions than a top-score paper It may have somewhat less compelling logic or slightly less-wellreasoned suggestions than a 4 paper though it will provide reasons for the recommendations made

The writer uses dear fluent and generally idiomatic English but may make minor or infrequent ESL errors (preposition errors dropped articles or verb endings etc) or repeat a single error (eg not punctuate possessive nouns) Occasional lapses of style are offSet by demonstrated mastery of syntax

CHAPTE84 Standardized Testing 81

2 Weak The writer makes somewhat simplistic suggestions not fully supported with reashysons fails to cite key facts offers little analysis of the problem or shows a limited grasp of the situation the given information is copied or listed withlittle integration into argument Points may be random or repetitious Writing may be badly focused with careless use of abstract language resulting in predication errors or illogical sentences

ESL andlor careless mechanical errors are frequent enough to be distracting OR sentences may be choppy style over-casual usage occasionally unidiomatic

1 Inadequate The essay may be disjointed incoherent or minimally developed The writer shows little grasp of the complex issues involved is unable to establish conshytext point of view or purpose in opening of paper or has a poor sense of audience Mechanical andor ESL errors or unidiomatic usages are frequent sentences may be ungrammatical OR correct but short and very simple

The two readers scores are added to yield a total possible score of 2 to 8 Test administrators recommend a score of 6 as the threshold for allowing a student to pursue graduate-level courses Anything below that is accompanied by a recomshymendation that the student either repeat the test or take a remedial course in gradshyuate writing offered in one of several different departments Students receive neither their essays nor any feedback other than the fmal score

6 Perform ongoing construct validation studies

From the above discussion it should be clear that no standardized instrument is expected to be used repeatedly without a ramporou~program of ongoing c~-sectmct valiltiatiOll Any standardized test once developed must be accompanied by sysshy~

tematic periodic corroboration of its effectiveness and by steps toward its improveshyment This rigor is especially true of tests that are produced in equated forms that is forms must be reliable across tests such that a score on a subsequent form of a test-has-the~same validityand-interpretability as its original

(A) The TOEFL program in cooperation with other tests produced by ETS has an impressive program of research Over the years dozens of TOEFL-sponsored research studies have appeared in the TOEFL Monograph Series An early example ofsuch a study was the seminal Duran et aI (1985) study TOEFLfrom a Communicative ViewpOint on Language Proficiency which examined the content characteristics of the TOEFL from a communicative perspective based on current research in applied linguistics and lanshyguage proficiency assessment More recent studies (such as Ginther 2001 Leacock amp Chodorow 2001 Powers et aI 2002) demonstrate an impressive array of scrutiny

(B) For approximately 20 years the ESLPT appeared to be placing students relishyably by means of an essay and a multiple-choice grammar and vocabulary test Over the years the security of the latter became s1lspect and the faculty administrators wished to see some content validity achieved in the process In the year 2000 that process began with a group of graduate students (Imao et aI 2000) in consl1ltation with faculty members and continued to fruition in the form of a new ESLPT reported in lmao (2002) The development of the new ESlPT involved a lengthy process of

82 CHAPTER 4 Standardized Testing

both content and construct validation along with facing such practical issues as scoring the written sections and a machine scorable multiple-choice answer sheet

The process of ongoing validation will no doubt continue as new forms of the editing section are created and as new prompts and reading passages are created for the writing section Such a validation process should also include consistent checks on placement accuracy and on face validity

(C) At this time there is little or no research to validate the GET itself For its conshy struct validation its administrators rely on a stockpile of research on university-level academic writing tests such as theTWEThe holistic scoring rubric and the topics and administrative conditions of the GET are to some extent patterned after that of the TWE In recent years some criticism of the GEf has come from international test-takers (Hosoya 2001) who posit that the topics and time limits of the GET among other facshytors work to the disadvantage of writers whose native language is not English These validity issues remain to be fully addressed in a comprehensive research study

I I

STANDARDIZED IANGUAGE PROFICIENCY TESTING

Tests of language profiCiency presuppose a comprehensive definition of the specific competencies that comprise overall language ability The specifications for the TOEFL provided an illustration of an operational definition of ability for assessment purposes This is not the only way to conceptualize the concept Swain (1990) offered a multidimensional view of profiCiency assessment by referring to three linshyguistic traits (grammar discourse and sociolinguistics) that can be assessed by means of oral multiple-choice and written responses (see Table 41) Swains conshyception was not meant to be an exhaustive analysis of ability but rather to serve as an operational framework for constructing proficiency assessments

Another defmition and conceptualization of profiCiency is suggested by the ACTFL association mentioned earlier ACfFL takes a holistic and more unitary view of proficiency in describing four levels superior advanced intermediate and noviceWithin each level descriptions of listening speaking reading and writing are provided as guidelines for assessment For example the ACfFL Guidelines describe the superior level of speaking as follows

ACTFL speaking guidelines summary superior-level

Superior-level speakers are characterized by the ability to

bull participate fully and effectively in conversations in formal and informal settings on topics related to practical needs and areas of professional andor scholarly interests

bull provide a structured argument to explain and defend opinions and develop effective hypotheses within extended discourse

bull discuss topics concretely and abstractly bull deal with a linguistically unfamiliar situation bull maintain a high degree of linguistic accuracy bull satisfy the linguistic demands of professional andor scholarly life

CHAPTER4 Standardized Testing 83

The other three ACfFL levels use the same parameters in describing progressively lower proficiencies across all four skills Such taxonomie~ have the advantage of considering a number of functions of linguistic discourse but the disadvantage at the lower levels of overly emphasizing test-takers deficiencies

Table 41 Traits of second language proficiency (Swain 1990 p 403)

Trait Grammar Discourse Sociolinguistic

focus on grammatical focus on textual focus on social accuracy within cohesion and appropriateness of sentences coherence language use

Method

Oral structured interview story telling and argumentationpersuasion

role-play ofspeech acts requests offers complaints

scored for accuracy of verbal morphology prepositions syntax

detailed rating for identification logical sequence and time orientation and global ratings for coherence

scored for ability to distinguish formal and informal register

Multiple-choice

sentence-level select the correct form exercise

paragraph-level select the coherent sentence exercise

speech act-Ievelselect the appropriate utterance exercise

(45 items) (29 items) (28 items)

involving verb morphology prepositionsan-d-uther items

Written composition

narrative and letter of persuasion

narrative and letter of persuasion

formal request letter and informal note

scored for accuracy of verb morphology prepositions syntax

detailed ratings much as for oral discourse and global rating for coherence

scored for the ability to distinguish formal and inforJ1lil1 register

FOUR STANDARDIZED lANGUAGE PROFICIENCY TESTS

We now tum to some of the better-known standardized tests of overall language ability or profiCiency to examine some of the typical formats used in commercially available tests We will not look at standardized tests of other specific skills here but that should not lead you to think by any means that proficiency is the only kind of test in the field that is standardized Three standardized oral production tests the

84 CHAPTER 4 Standardized Testing

Test of Spoken English (fSE) the Oral Proficiency Inventory (OPI) and PbonePassreg are discussed in Chapter 7 and the Test of Written English (WE) is covered in ChapterS

Four commercially produced standardized tests of English language proficiency are described briefly in this section the TOEFL the Michigan English Language Assessment Battery (MELAB) the International English Language Testing System (lELTS) and the Test of English for International Communication (fOEICreg) In an appendix to this chapter are sample items from each section of each test When you turn to that appendix use the following questions to help you evaluate these four tests and their subsections

1 What item types are included 2 How practical and reliable does each subsection of each test appear to be 3 Do the item types and tasks appropriately represent a conceptualizatio~ of

language proficiency (ability) That is can you evaluate their construct validity

4 Do the tasks achieve face validity 5 Are the tasks authentic 6 Is there some washback potential in the tasks

Test of English as a Foreign Language (TOEFL)

Producer Educational Testing Service (ETS) Objective To test overall proficiency (language ability) Primary market Almost exclusively US universities and colleges for admission

purposes Type Computer-based (CB) (and two sections are-computer-adaptive)

A traditional paper-based (PB) version is also available Response modes Multiple-choice responses essay Specifications See the box on pp 72-73 Time allocation Up to 4 hours (CB) 3 hours (PB) Internet access wwwtoeflorg

Comments In the North American context the TOEFL is the most widely used comshymercially available standardized test of proficiency Each year the TOEFL test is adminisshytered to approximately 800000 candidates in more than 200 countries It is highly respected because of the thorough program of ongoing research and development conshyducted by ETS The TOEFLs primary use is to set proficiency standards for international students seeking admission to English-speaking universities More than 4200 academic institutions government agencies scholarship programs and licensingcertification agenshycies in more than 80 countries use TOEFL scores By 2004 the TOEFL will include a secshytion on oral production

CHAPTER 4 Standardi~ed Testing 85

Michigan English Language Assessment Battery (MELAB)

Producer English language Institute University of Michigan Objective To test overall proficiency (language ability) Primary market Mostly US and Canadian language programs and colleges

some worldwide educational settings as well Type Paper-based Response modes Multiple-choice responses essay Time allocation 25 to 35 hours Internet access wwwlsaumicheduelimelabhtm

Specifications The MElAB consists of three sections Part 1 a 3D-minute impromptu essay is written on an assigned topic Part 2 a 25-minute multiple-choice listening comshyprehension test is delivered via tape recorder Part 3 is a 100-item 75-minute multipleshychoice test containing grammar doze reading vocabulary and reading comprehension An oral interview (speaking test) is optional

Comments The Ell at the University of Michigan has been producing the MELAB and its earlier incarnation (Michigan Test of English language Proficiency) since 1961 like the TOEFL it serves a North American audience but is also used internationally While its use is not as widespread as the TOEFL its validity is widely respected Because it is cheaper than the TOEFL and more easily obtained it is popular among language schools and institutes Many institutions and companies accept MElAB scores in lieu ofTOEFL scores

International English Language Testing System (IELTS)

Producer Jointly managed by The University of Cambridge local Examinations Syndicate (UClES) The British Council and lOP Education Australia

Objective To test overall proficiency (language ability) Primary-market Australian British Canadian and New Zealand academic

institutions and professional organizations American academic institutions are increasingly accepting IELTS for admissions purposes

1)rpe Computer-based (for the Reading and Writing sections) papershybased for the listening and Speaking modules

Response modes Multiple-choice responses essay oral production Time allocation 2 hours 45 minutes Internet access httpwwwieltsorgl

httpwwwudesorguk httpwwwbritishcouncilorg

Specifications Reading candidates choose between academic reading or general training reading (60 minutes) Writing the same option academic writing or general training writing (60 minutes) Listening four sections for all candidates (30 minutes) Speaking five sections for all candidates (1015 minutes)

86 CHAPTER 4 Standardized Testing

Comments The University of Cambridge local Examinations Syndicate (UCLES) has been producing English language tests since 1858 Now with three organizations cooperatshying to form the IELTS more than a million examinations are administered every year In 2002 a computer-based version of the Reading and Writing modules of the IELTS became available at selected centers around the world The other sections are administered locally by an examinet The paper-based IELTS remains an option for candidates The IELTS retains the distinct advantage of requiring all four skills in the test-takers performance

Test of English for International Communication (TOEICreg)

Producer The Chauncey Group International a subsidiary of Educational Testing Service

Objelttive To test overall proficiency (langlJage ability) Primary market Worldwide business commerce and industry contexts

(workplace settings) Type Computer-based and paper-based versions Response modes Multiple-choice responses Time allocation 2 hours Internet access httpwwwtoeiccom

Specifications Listening Comprehension 100 items administered by audiocassette Four types of task statements questions short conversations and short talks (approxishymately 45 minutes) Reading 100 items Three types of task cloze sentences error recogshynition and reading comprehension (75 minutes)

Comments The TOEIC has become a very widely used international test of English proficiency in workplace settings where English is required for job performance The conshytent includes many different employment settings such as conferences presentations sales ordering shipping schedules reservations (etters and memoranda It is approprishyate to use in educational settings where vocational or workplace English courses are being offered

sect sect sect sect sect

The construction of a valid standardized test is no minor accomplishment whether the instrument is large- or small-scale The designing of specifications alone as this chapter illustrates requires a sophisticated process of construct valishydation coupled with considerations of practicality Then the construction of items and scoringinterpretation procedures may require a lengthy period of trial and error with prototypes of the final form of the testWith painstaking attention to all the details of construction the end product can result in a cost-effective timeshysaving accurate instrument Your use of the results of such assessments can provide useful data on learners language abilities But your caution is warranted as well for all the reasons discussed in this chapter The next chapter will elaborate on what lies behind that need for a cautious approach to standardized assessment

CHAPTER4 Standardized Testing 87

EXERCISES

[Note (I) Individual work (G) Group or pair work (C) Whole-class discussion]

1 (C) Tell the class about the worst test experience youve ever had Briefly anamiddot lyze what made the experience so unbearable and try to come up with sugshygestions for improvement of the test andor its administrative conditions

2 (G) In pairs or small groups compile a brief list of pros and cons of standardshyized testing Cite illustrations of as many items in each list as possible Report your lists and examples to the rest o~ the class

3 (I) Select a standardized test that you are quite familiar with (probably a recent experience) Mentally evaluate that test using the five principles of practicality reliability validity authenticity and washback Report yourevaluashytion to the class

4 (G) The appendix to this chapter provides sample items from Jour different tests of language proficiency In groups one test for each group analyze your test for (a) content validity (b) face validity and (c) authenticity

5 (C) Do you think that the sample TOEFL reading passage about pirates (pages 74-75) and the Graduate EssayTest prompt (pages 76-77) about a school board hiring committee have any culture bias Discuss this and other cultural biases you have noticed in tests Is it possible to design a test that is completely free of culture bias

6 (CG) Compare the differences in conceptualization of language proficiency represented by Swains model the TOEFL and the ACfFL philosophy Which one best represents current thinking about communicative language ability What are the strengths and weaknesses of each approach

FORYOlIILEURTHER READING

Gronlund Norman E (1998) Assessment of student achievement Sixth Edition Boston Allyn and Bacon

Gronlunds classic also mentioned in Chapter 3 offers a concise overview of features of standardized tests offering definitions and examples of the statistical considerations in interpreting scores His approach is unbiased cleady written and accessible to those who might fear the mathematics of standardized testing

Phillips Deborah 2001 Long1nan introductory course for the TOEFL test White Plains NY Pearson Education

A careful examination of this or any other reputable preparation course for a standardized language test is well worth a students time Note especially how the book acquaints the user with the specifications of the test and offers a number of useful strategie~ that can be llsed in preparation for the test and during irs adn1inistration

88 CHAPTER 4 Standardized Testing

APPENDIX TO CHAPTER 4

Commercial Proficiency Tests Sample Items and Tasks

Test of English a~ a Foreign Language (TOEFLreg)

Listening r

Part A

In this section you will hear short conversations between two people In some ofthe conversations each person speaks only once In other conversations one or both of the people speak more than once Each conversation is followed by one questionabQlt it Each question in this part has four answer choices You should click on the best answer to each question Answer the questions on the basis of what is stated or implied by the speakers Here is an example On the computerscreen you will see

[man and woman talking]

On the recording you will hear

(woman) Hey wheres your sociology book (man) At home Why carry it around when were just going to be taking

a test (woman) Dont you remember Professor Smith said we could us it during

the test (man) Ohl no Well Ive still got an hour right Im so glad I ran into you

You wiII then see and hear the question before the answer choices appear

What will the man probably do next

o Begin studying for the sociology test o Explain the problem to his professor o Go home to get his textbook o Borrow the womans book

To choose an answer you will click on an oval The oval next to that answer will darken After you click on Next and Confirm Answer the next conversation will be presented

Part B

In this section you will hear several longer conversations and talks Each conversation or talk is followed by several questions The conversations talks and questions will not be repeated The conversations and talks are about a variety of topics You do not need speshycial knowledge of the topics to answer the questions correctly Rather you should answer each question on the basis of what is stated or implied by the speakers in the conversashytions or talks

For most of the questions you will need to click on the best of four possible answers Some questions will have special directions The special directions will appear in a box on the computer screen Here is an exampie ot a conversation and some questions

CHAPTER 4 Standardized Testing 89

Marine Biology (narrator) Listen to part of a discussion in a marine biology class

(professor) A few years ago our local government passed a number of strict environmental laws As a result Sunrise Beach looks nothing Ii ke it did ten years ago The water is cleaner and theres been a tremendous increase in all kinds of marine life which is why were going there on Thursday

(woman) I dont know if I agree that the water quality has improved I mean I was out there last weekend and it looked all brown It didnt seem too clean to me

(professor) Actually the color of the water doesnt always indicate whether its polluted The brown color you mentioned might be a result of pollution or it can mean a kind of brown algae is growing there Its called devils apron and it actually serves as food for whales

(man) So when does the water look blue (professor) Well water thats completely unpolluted is actually colorless But

it often looks bluish-green because the sunlight can penetrate deep down and thats the color thats reflected

(woman) But sometimes it looks really green Whats that about (professor) Ok well its the same principle as with devils apron the

water might be green because of different types of green algae there-gulfweed phytoplankton You all should finish reading about algae and plankton before we go In fact those are the types of living things Im going to ask you to be looking for when were there

Now get ready to answer the questions

What is the discussion mainly about

o The importance of protecting ocean environments o The reasons why ocean water appears to be different colors o The survival of whales in polluted water o The effect that colored ocean water has on algae

To choose an answer click on an oval The oval next to that answer will darken After you click on Next and Confirm Answer the next question will be presented

According to the professor what can make ocean water look browngt

o Pollution o Cloudy Skies o Sand o Algae

Click on 2 answers

To choose your answers you will click on the squares An XII wiii appear in each square

bullbullbullbullbullbullbull

90 CHAPTER 4 Standardized Testing

Structure and Written Expression This section measures the ability to recognize language that is appropriate for standard written English There are two types ofquestions in this section In the first type ofquestion there are incomplete sentences Beneath each sentence there are four words or phrases

Directions CIiSk on the one word or phrase that best completes the sentence

The colum~ine flower __ to nearly all of the United States can be raised from seed in almost any garden

native how native is how native is it is native

Time Help Confirm

After you click on Next and Confirm Answ~ the next question willbe presented

The second type of question has four underlined words or phrases You will choose the one underlined word or phrase that must be changed for the sentence to be correct

Directions Click on the one underlined word or phrase that must be changed for the senshytence to be correct

One of the most difficult problems in understanding sleep is determining what the funcshytions of sleep ~

lime Help Confirm

Clicking on an underlined word or phrase will darken it

Reading This section measures the ability to read and understand short passages similar in topic and style to those that students are likely to encounter in North American universities and colleges This section contains reading passages and questions about the passages There are several different types of questions in this section In the Reading section you will first have the opportunity to read the passage

The temperature of the Sun is over 10000 degrees Fahrenheit at the surface but it rises perhaps more than 270000000 at the center The Sun is so much hotter than the Earth that matter can exist only as a gasl except perhaps at the core In the core of the Sun the pressures are so great that despite the high temperature there may be a small solid core However no one really knows since the center of the Sun can never be directly observed ~ Solar astronomers do know that the Sun is divided into five general layers or zones Starting at the outside and going down into the Sun the zones are the corona chromoshysphere hotosphere convection zone and finally the core The first three zones are reshygarded as the Suns atmosphere But since the Sun has no solid surface it is hard to middottell where the atmosphere ends and the main body of the Sun begins

The Suns outermost layer begins about 10000 miles above the visible surface and goes outward for millions of miles This is the only part of the Sun that can be seen during an eclipse such as the one in February 1979 At any other time the corona can be seen

bullbullbullbullbullbullbull

bull bullbullbullbullbullbull

CHAPTER 4 Standardized Testing 91

only when special instruments are used on cameras and telescopes to block the light from the photosphere

The corona is a brilliant pearly white filmy light about as bright as the full Moon Its beautiful rays are a sensational sight during an eclipse The coronas rays flash out in a brilliant fan that has wispy spikelike rays near the Suns north and south poles The corona is generally thickest at the Suns equator The corona is made up of gases streamshying outward at tremendous speeds that reach a temperature of more than 2 million deshygrees Fahrenheit The gas thins out as it reaches the space around the planets By the time the gas of the corona reaches the Earth it has a relatively low density

When you have finished reading the passage you will use the mouse to click on Proceed Then the questions about the passage will be presented You are to choose the one best anshyswer to each question Answer all questions about the information in a passage on the basis ofwhat is stated or implied in that passage Most ofthe questions will be multiple-choice questions To answer these questions you will click on a choice below the question

With what topic is paragraph 2 mainly concerned

o How the Sun evolved o The structure of the Sun o Why scientists study the Sun o The distaflce of the Sun from the planets

Paragraph 2 is marked with an arrow (~)

You will see the next question after you click on Next

To answer some questions you will click on a word or phrase Here is an example

Look at the word one in the passage Click on the word or phrase in the bold text that one refers to To answer you can click on any part of the word or phrase in the passage Jour choice will darken to show which word you have chosen

The Suns outermost layer begins about 10000 miles above the visible surface and goes outward for millions of miles This is the only part of the Sun that can be seen durshying an eclipse such as the one in February 1979 At any other time the corona can be seen only when special instruments are used on cameras and telescopes to block the Iight from the photosphere

You will see the next question after you click on~ To answer some q~estions you will click on a sentence in the passage Here is an example

~ The corona is a brilliant pearly white filmy light about as bright as the full Moon Its beautiful rays are a sensational sight during an eclipse The coronas rays flash out in a brilliant fan that has wispy spikelike rays near the Suns north and south poles The corona is generally thickest at the Suns equator ~ The corona is made up of gases streaming outward at tremendous speeds that reach a temperature of more than 2 million degrees Fahrenheit The gas thins out as it reaches the space around the planets By the time the gas of the corona reaches the Earth it has a relatively low density

bull bullbullbullbullbullbull

92 CHAPTER 4 Standardized Testing

Click on the sentence in paragraph 4 or 5 in which the author compares the light of the Suns outermost layer to that ofanother astronomical body Paragraphs 4 and 5 are marked with arrows (~)

To answer some questions you will click on a square to add a sentence to the passage Here is an example -The following sentence can be added to paragraph 1

At the center of the Earths solar system lies the Sun

Where would it best fit in paragraph I Click on the square to add the sentence to the paragraph

D The temperature of the Sun is over 10000 degrees Fahrenheit at the surface but it rises to perhaps morethan 27000000deg at the center 0 The Sun is so much hotter than the Earth that matter can exist only as a gasi except p~rHapsatth~ c6relp the c~re of the ii Sun the pressures are so great that despite the high temperature there may be a small solid core D However no one really knows since the center of the Sun can never be directly observed D 0100

When you click on a square the sentence will appear in the passage at the place you have chosen You can read the sentence added to the paragraph to see if this is the best place to add it You can click on another square to change your answer The sentence will be added and shown in a dark box

Writing In this section you will have an opportunity to demonstrate your ability to write in Enshyglish This includes the ability to generate and organize ideas to support those ideas with examples or evidence and to compose in standard written English in response to an asshysigned topic You will have 30 minutes to write your essay on that topic You must write on the topic you are assigned An essay on any other topic will receive a score of 0 Read the topic below and then make any notes that will help you plan your response Begin typing your response in the box at the bottom of the screen or write your answer on the answer sheet provided to you

Following is a sample topic

Do you agree or disagree with the following statemenH

Teachers should make learning enjoyable and fun for their students

Use specific reasons and examples to support your opinion

CHAPTER 4 Standarczed Testing 93

Michigan English Language Assessment Battery (MELAB)

Composition The time limit for the composition is 30 minutes You must write on only one of the top~

ics below If you write about something else your composition paper will not be graded and you cannot be given a final score If you do not understand the topics ask the exam~ iner to explain or to translate them You may be asked to give your opinion ofsomething and explain why you believe this to describe something from your experience or to exshyplain a problem and offer possible solutions You should write at least one page Some sample topics are

1 What do you think is your countrys greatest problem Explain in detail and tell what you think can be done about it

2 What are the characteristics of a good teacher Explain and give examples 3 An optimist is someone who sees the good side of things A pessimist sees the

bad side Are you an optimist or a pessimist Relate a personal experience that shows this

4 In your opinion are the benefits of space exploration really worth the enormous costs Discuss

Most MELAB compositions are one or two pages long (about 200-300 words) If your paper is extremely short (less than 150 words) your composition will be given a lower score Before you begin writing you might want to take 2 or 3 minutes to plan your comshyposition and to make a short outline to organize your thoughts Such outlines will not be graded they are only to help you You should use the last 5 minutes to read through your composition and to make changes or corrections

Your composition will be graded on how clearly you express yourself in English and on the range of English you are able to use and your control in doing so This means your composition should be well organized your arguments should be fully developed and you should show a range ofgrammatical structures and broad vocabulary Compositions that consist only of very short sentences and very simple vocabulary cannot be given the

middothighest scores If errors are not frequent and if they do not confuse your meaning they will not lower your score very much

Listening Now you will hear a short lecture You may take notes during the lecture Following the lecture you will be asked some questions about it

Therell be a two-week exhibit of the paintings of the little-known master Laura Bernhart at the Claire Osmond Galleries starting on the fifteenth of the month and running through the thirtieth Bernharts known for her innovative designs in abstract expressionism Though a true original she declared a spiritual heritage from Salvador Dali the famous Spanish painter Since Bernhart lived a rather solitary life and died while only in her twenties few people are aware of her works This showing at the Osmond Galleries will provide many with an introduction to her works

10 Where is the exhibit a the Art Museum b the Dali Galleries c the Osmond Galleries

94 CHAPTER 4 Standardized Testing

11 What is Bernhart known for a her copies of Dalis paintings b the originality of her designs c her exhibitions

12 What will going to the exhibit allow most people to do a to see Saivador Dalis paintings b to see Bernharts works for the first time c to learn about Spanish art

Grammar

1 What did the teacher just tell you

She reminded our notebooksI a us to bring b that we bring c our bringing d we should bring

2 Is Bill a good dancer

Not really __ he tries very hard a in spite of h despite c even though d while

3 your clothes are all wet1

Yes I didnt come __ the rain soon enough a away to b over to c down with d in from

Cloze In years to come zoos will not only be places where animals are exhibited to the public but repositories where rare species can be saved from extinction (7) captive breeding The most powerful force (8) the future of many animals-and of zoos-is the decline of the wild (9) even zoo directors would argue that (10) are better places for animals than the fields and forest of their native (11) yet zoos may be the last chance for some creatures that would otherwise pass qUietly into oblivion

7 a through c from b of d damage

8 a bringing c to b that d influencing

9 a But c Not b So d Then

10 a where c even b zoos d wilds

11 alands c residence b life d field

CHAPTER 4 Standardized Testing 95

Vocabulary

12 Mark has a flair for writing a need b purpose c talent d dislike

13 Bill Collins launched his restaurant last June a moved b started c sold d bought

14 John will not accept the censure a burden b blame c credit d decision

15 I cant think of the answer Can you give me a __ a hint b token c taste d gaze

16 Because fewer people are taking expensive vacations the tourist industry is in a a choke b grope c grumble d slump

17 I disagree with a few of his opinions but __ we agree a deliberately b conclusively c essentially d immensely

Reading The influenza virus is a single molecule built from many millions of single atoms You must have heard of the viruses which are sometimes called living molecules While bacteria can be considered as a type of plant secreting pOisonous substances into the body of the organism they attack viruses are living organisms themselves We may conshysider them as regular chemical molecules since they have a strictly aefined atomic strucshyture but on the other hand we must also consider them as being alive since they are able to multiply in unlimited quantities

18 According to the passage bacteria are a poisons

b larger than viruses c very small d plants

96 CHAPTER 4 Standardized Testing

19 The writer says that viruses are alive because they a have a complex atomic structure b move c multiply d need warmth and light

20 The atomic structure of viruses a is -tJIariable b is strictly defined c cannot be analyzed chemically d is more complex than that of bacteria

International English Language Testing System (fELTS)

I

listening

The Listening Module has four sections The first two sections are concerned with social needs There is a conversation between two speakers and then a monologue For examshyple a conversation about travel arrangements or decisions on a night out and a speech about student services on a university campus or arrangements for meals during a confershyence The final two sections are concerned with situations related more closely to educashytional or training contexts For example conversation between a tutor and a student about an assignment or between three students planning a research project and a lecture or talk ofgeneral academic interest All the topics are ofgeneral interest and it makes no difference what subjects candidates study Tests and tasks become more difficult as the sections progress A range of English accents and dialects are used in the recording which reflects the international usage of IELTS

Academic Reading [A 7S0-word article on-th-e- topic of Wind Power in the US with a short glossary at the end]

Questions 1-5

Complete the summary below

Choose your answers from the box below the summary and write them in boxes 1-5 on your answer sheet Note There are more words or phrases than you will need to fill the gaps You may use any word or phrase more than once

Example The failure during the late 1970s and early 19805 of an attempt to establish a widespread wind power industry in the United States resulted largely from the (1) bull in oil prices during this period The industry is now experiencing a steady (2) due to improveshyments in technology and an increased awareness of the potential in the power of wind The wind turbines that are now being made based in part on the (3) of wide- ranging research in Europe are easier to manufacture and maintain than their predecesshysors This has led wind-turbine makers to be able to standardise and thus minimize (4) There has been growing (S) of the importance of wind power as an energy source

CHAPTER 4 Standardized Testing 97

criticism stability skepticism success operating costs decisions design costs fall effects production costs growth decline failure recognition results

Questions 6-1 0 Look at the following list of issues (Questions 6-10) and implications (A-C) Match each issue with one implication Write the appropriate letters A-C in boxes 6-10 on your anshyswer sheet

Example The current price of one wind-generated kilowatt Answer

6 The recent installation of systems taking advantage of economies of scale

7 The potential of meeting one fifth of current U5 energy requirements by wind power

8 The level of acceptance of current wind turbine technology

9 A comparison of costs between conventional and wind power sources

10 The view of wind power in the European Union

Implications

A provides evidence against claims that electricity produced from wind power is relatively expensive

B supports claims that wind power js an important source of energy

C opposes the view that wind power technology requires further-development

General Training Reading Read the passage on Daybreak trips by coach and look at the statements below On your answer sheet write

TRUE if the statement is true FALSE jf the statement is false

NOlGIVEN if the information is not given in the leaflet

1 MiIlers Coaches owns Cambridges Cam bus fleet

2 Premier is an older company than Millers

3 Most of the Daybreak coaches are less than 5 years old

4 Daybreak fares are more expensive than most of their competitors

5 Soft drinks and refreshments are served on most longer journeys

6 Smoking is permitted at the rear of the coach on longer journeys

7 Tickets must be bought in advance from an authorised Daybreak agent

6 Tickets and seats can be reserved by phoning the Daybreak Hotline

9 Daybreak passengers must join their coach at Cambridge Drummer Street

10 Daybreak cannot guarantee return times

98 CHAPTER 4 Standardized Testing

FROM CAMBRIDGE AND SURROUNDING AREA

SPRING IS INTHEAIR

Welcome to our Spring Daybreak programme which continues the tradition of offering unbeatable value for money day trips and tours All the excursions in this brochure will be operated by Pr~mier Travel Services Limited or Millers Coaches both companies are part of the CHLGroup owners of Cambridges Cambus fleet

WERE PROUD OF OUR TRADITION

Premier was established in 1936 the Company now offers the highest standards of coaching in todays competitive operating environment Miller has an enviable reputation stretching back over the past 20 years offering coach services at realistic prices Weve traveled a long way since our early days of pre-war seaside trips Now our fleet of 50 modern coaches (few are more than five years old) operate throughout Britain and Europe but were pleased to still maintain the high standards of quality and service the trademark of our founders nearly sixty years ago

EXCLUSIVE FEATURES

Admission-inclusive fares All Daybreak fares (unless specifically otherwise stated) include admission charges to the attractions shows and exhibits we visit Many full-day scenic tours are accompanied by a fully trained English Tourist Board Blue Badge guide or local experienced driverguide Some Daybreaks include lunch or afternoon tea Compare our admission inclusive fares and see how much you save Cheapest is not the best and value for money is guaranteed If you compare our bargain Daybreak fares beware--most of our competishytors do not offer an all-inclusive fare

SEAT RESERVATIONS

We value the freedom of choice so you can choose your seat when you book The seat reservation is guaranteed a-nd remains yours at all times when aboard the coach

NO SMOKING COMFORT

With the comfort of our passengers in mind coaches on all our Daybreaks are no smokshying throughout In the interests of fellow passengers comfort we kindly ask that smokers observe our no smoking policy On scenic tours and longer journeys ample refreshment stops are provided when of course smoking is permitted

YOUR QUESTIONS ANSWERED

Do I need to book Booking in advance is strongly recommended as all Daybreak tours are subject to demand Subject to availability stand-by tickets can be purchased from the driver

What ti me does the coach leave The coach departs from Cambridge Drummer Street (Bay 12 adjacent to public toilets) at the time shown There are many additional joining points indicated by departure codes in the brochure If you are joining at one of our less popular joining points you will be adshyvised of your pick-up time (normally by telephone) not less than 48 hours before deparshyture In this way we can minimize the length of pick-up routes and reduce journey times for the majority of passengers

CHAPTER 4 Standardized Testing 99

What time do we get back An approximate return time is shown for each excursion The tim~s shown serve as a guide but road conditions can sometimes cause delay If your arrival will be later than advertised your driver will try to allow for a telephone call during the return journey

Where can I board the coach All the Daybreaks in the brochure leave from Cambridge Drummer Street (Bay 12 adjashycent to public toilets) at the time shown Many Daybreaks offer additional pick-ups for pre-booked passengers within Cambridge and the surrounding area This facility must be requested at the time of booking

Academic Writing Writing Task 1 You should spend about 20 minutes on this task

The graph below shows the different modes of transport used to travel to and from work in one European city in 1950 1970 and 1990

[graph shown here]

Write a report for a university lecturer describing the information shown below You should write at least 150 words

Writing Task 2 You should spend about 40 minutes on this task

Present a written argument or case to an educated reader with no specialist knowledge of the folowing topic

It is inevitable that as technology develops so traditional cultures must be lost Technolshyogy and tradition are incompatible-you cannot have both together

To what extent do you agree or disagree with this statement Give reasons for your answer You should write at least 250 words You should use your own ideas knowlshyedge and experience and support your arguments with examples and relevant evidence

General Training Writing Writing Task 1 You should spend about 20 minutes on this task You rent a house through an agency The heating system has stopped working You phoned the agency a week ago but it has still not been mended Write a letter to the agency Explain the situation and teil them what you want them to do about it

You should write at least 150 words You do NOT need to write your own address

Begin your letter as follows

Dear - ___-I

Writing Task 2 You should spend about 40 minutes on this task As part ofa class assignment you have to write about the following topic

100 CHAPTER 4 Standardized Testing

Some businesses now say that no one can smoke cigarettes in any of their offices Some governments have banned smoking in all public placesThis is a good idea but it takes away some of our freedom

Do you agree or disagree Give reasons for your answer You should write at least 250 words

Speaking In each ofthe three parts of the speaking module a specific function is fulfilled In Part 1 the candidates answer general questions about themselves their homes or families their jobs or studies their interests and a range ofsimilar familiar topic areas This part lasts between four and five minutes In Part 2 the candidate is given a verbal prompt on a card and is asked to talk on a particular topic The candidate has one minute to prepare before speaking at length for between one and two minutes The examiner then asks one or two wind-down questions In Part 3 the examiner and candidate engage in a discusshysion of more abstract issues and concepts which are thematically linked to the topic prompt in Part 2 The discussion lasts between four and five minutes

All interviews are recorded on audiocassette Here is a sample ofa Part 2 topic

Describe a teacher who has greatly influenced you in your education

You shou Id say

where you met them what subject they taught what was special about them

and explain why this person influenced you so much

You will have to talk about the topic for 1 to 2 minutes You have 1 minute to think about what you are going to say You can make some notes if you wish

Test of English for International Communication (TOEICreg)

listening

Part 1 Photographs Directions For each question you will see a picture in your test book and you will hear four short statements The statements will be spoken just one time They will not be printed in your test book so you must listen carefully to understand what the speaker says When you hear the four statements look at the picture in your test book and choose the statement that best describes what you see in the picture Then on your answer sheet find the number of the question and mark your answer

[photograph of a scientist looking through a microscope]

You will hear Look at the picture marked number 1 in your test book

(A) Shes speaking into a microphone (B) Shes put on her glasses (C) She has both eyes open (D) Shes using a microscope

CHAPTER 4 Standardized Testing 101

Part 2 Question-Response Directions In this part of the test you will hear a question or statement spoken in Enshyglish followed by three responses also spoken in English The question or staten1ent and the responses will be spoken just one time They will not be printed in your test book so you must listen carefully to understand what the speakers say You are to choose the best response to each question or statement

Question 1 You will hear Ms Morikawa has worked here for a long time hasnt she

(A) At three oclock (B) No Ive lost my watch (C) More than ten years

Question 2 You will hear Which of these papers has a wider circulation

(A) The morning edition (B) Get more exercise (C) By messenger

Part 3 Short Conversations Directions In this part of the test you will hear short conversations between two people The conversations will not be printed in your test book You will hear the conversations only once so you must listen carefully to understand what the speakers say In your test book you will read a question about each conversation The question will be followed by four answers You are to choose the best answer to each question and mark it on your answer sheet

Question 1 (Man) We should think about finding another restaurant for lunch (Woman) Why The food and service here are great

(Man) Yes but the prices are going up every week

You will read Why is this man unhappy with the restaurant

(A) It is too noisy (B) It is too expensive (C) It is too crowded (D) It is too difficult to find

Question 2 (Woman A) How was Dr Borgs recent trip to Singapore (Woman B) She enjoyed the tour of the port very much (Woman A) They say its one of the most active in Asia

You will read 2 What did Dr Borg find interesting

(A) The tourist center (B) The airport (C) The musical performance (D) The harbor

Part 4 Short Talks Directions In this part of the test you vill hear several short talks Each will be spoken just one time They will not be printed in your test book so you must listen carefully to understand and remember what is said In your test book you will read two or more questions about each short talk The questions will be followed by four answers You are to choose the best answer to each question and mark it on your answer sheet

102 CHAPTER 4 Standardized Testing

You will hear Questions 1 and 2 refer to the following announcement

Good afternoon and welcome aboard Nordair Flight 857 from Copenhagen to Bangkok with intermediate stops in Dubai and Calcutta We are preparing for departure in a few minutes At this time your seat back should be returned to its full upright position and your seat belt s~ould be fastened OUf anticipated total flying time to Dubai is six hours and twenty-five minutes I hope you enjoy the flight You will hecJr Now read question 1 in your test book and answer it You will read 1 What is the final destination of the flight

(A) Bangkok (B) Copenhagen (C) Dubai (O) Calcutta

You will hear Now read question 2 in your test book and answer it You will read 2 What will happen in a few minutes

(A) The flight will land in Dubai I

(B) The passengers will board the plane (C) The plane will take off (0) The gate number will be announced

Reading In this section of the test you will have the chance to show how well you understand written English There are three parts to this section with special directions for each part

Part 4 Incomplete Sentences Directions This part of the test has incomplete sentences Four words or phrases marked (A) (8) (e) (D) are given beneath each sentence You are to choose the one word or phrase that best completes the sentence Then on your answer sheet find the number of the question and mark your answer

1 Mr Yangs trip will __ him away from the office for ten days (A) withdraw (B) continue (C) retain (0) keep

2 The company that Marie DuBois started now sells __ products throughout the world (A) its (B) it (C) theirs (D) them

3 If your shipment is not delivered __ Tuesday you can request a full refund for the merchandise (A) at (B) by (C) within (D) while

CHAPTER 4 Standardized Testing 103

Part 6 Error Recognition Directions In this part ofthe test each sentence has four words or phrases underlined The four underlined parts of the sentence are marked (A) (B) (C) (D) You are to identify the one underlined word or phrase that should be corrected or rewritten Then on your answer sheet find the number of the question and mark your answer

1 The pamphlet contains some importance information about the current exhibit ABC D

2 No matter how Jong it taking to finish the annual report it must be done properly ABC D

3 The popularity of jogging appears to have decreased since the past couple of years ABC D

Part 7 Reading Comprehension Directions The questions in this part of the test are based on a selection of reading mateshyrials such as notices letters) forms newspaper and magazine articles) and advertisements You are to choose the one best answer (A) (B) (C) or (OJ to each quesshytion Then on your ariswefsheelfindthe number of the qUestion andmcirkyour answer Answer all questions following each reading selection on thebasis of what is stated or implied in that selection

The Museum ofTechnology is a hands-on museum designed for people to experience science at w()rk~ Visitors are encouraged to use test and handle the objects o~ display Special demonstrations are scheduled for the first and second Wednesdays of each month at 1330 Open Tuesday-Friday 1200-1630 Saturday 1000-1730 and Sunday 11 00-1630

1 When during the month can visitors see special demonstrations (A) Every weekend (B) The first two Wednesdays (C) One afternoon a week (D) Every other Wednesday

Questions 2 and 3 refer to the followi ng notice

NOTICE If you are unable to work because of an extended illness or injury that is not workshyrelated you may be entitled to receive weekly benefits from your employer or the firms insurance company To claim benefits you must file a claim form within thirty days of the first day of your disability Before filing the claim you must ask your doctor to fill in the Doctors Statement on the claim form stating the period of disability

3 To whom is this notice addressed (A) Employers (8) Doctors (C) Employees (D) When paying the bill

4 When must the claim form be filed (A) On the first of the month (8) On the thirtieth of the month (C) On the first day ofdisabifity (D) Within 30 days of the start of disability

Page 15: Standardized Testing Chapter 4 Brown

80 CHAPTER 4 Standardized Testing

Facsimile of a TOEFLreg score report

TOEFL Scaled Scores Claudia Y Estudiante Peru ___

19 17 17 177 Listening Structure Writing Reading Total Score

Essay rati ng 30

The rating scale for the essay is virtually the same one that is used for the Test of Written English (see Chapter 9 for details) with a zero level added for no response copying the topic only writing completely off topic or not writing in English

(B) The ESLPT reports a score for each of themiddot essay sections but the rating scale differs between them because in one case the objective is to write a summary and in the other to write a response to a reading ~ch essayi~pd lgtY ~o readet~ ifhFfF js a discrepancy of more than one level a third reader1resolves the differenceThe ~ditiilg section is machine-scanned and -scored with a total score and ~th part-scores for each ofthe grammaticaVrhetorlcal sectionS From these data placement administrators have adequate information to make placements and teachers receive some diagnostic inforshymation on each student in their classes Students do not receive their essays back

(C) Each GET is read by two trained readers who give a score between 1 and 4 according to the following scale

Graduate Essay Test Scoring Guide

Please make no marks on the writers work Write your reader number and score on the front cover of each test booklet

4 Superior The opening establishes context purpose and point of view the body of the essay developsmiddot recommendations-logically and coherently The writer demonshystrates awareness of the complexities in the situation and provides analysis of the probJem offers compelling or common-sense reasons for recommendations made makes underlying assumptions explicit

The writer uses fluent and idiomatic English with few mechanical errors Style reshyveals syntactic maturity is dear and direct is not choppy or over-colloquial nor over-formal stuffy or unfocused Occasional spelling or punctuation errors may be easily attributed to hasty transcription under pressure

3 Competent After an opening that establishes context and purpose the paper unfolds with few lapses in coherence but may have somewhat less clear organization of less explicit transitions than a top-score paper It may have somewhat less compelling logic or slightly less-wellreasoned suggestions than a 4 paper though it will provide reasons for the recommendations made

The writer uses dear fluent and generally idiomatic English but may make minor or infrequent ESL errors (preposition errors dropped articles or verb endings etc) or repeat a single error (eg not punctuate possessive nouns) Occasional lapses of style are offSet by demonstrated mastery of syntax

CHAPTE84 Standardized Testing 81

2 Weak The writer makes somewhat simplistic suggestions not fully supported with reashysons fails to cite key facts offers little analysis of the problem or shows a limited grasp of the situation the given information is copied or listed withlittle integration into argument Points may be random or repetitious Writing may be badly focused with careless use of abstract language resulting in predication errors or illogical sentences

ESL andlor careless mechanical errors are frequent enough to be distracting OR sentences may be choppy style over-casual usage occasionally unidiomatic

1 Inadequate The essay may be disjointed incoherent or minimally developed The writer shows little grasp of the complex issues involved is unable to establish conshytext point of view or purpose in opening of paper or has a poor sense of audience Mechanical andor ESL errors or unidiomatic usages are frequent sentences may be ungrammatical OR correct but short and very simple

The two readers scores are added to yield a total possible score of 2 to 8 Test administrators recommend a score of 6 as the threshold for allowing a student to pursue graduate-level courses Anything below that is accompanied by a recomshymendation that the student either repeat the test or take a remedial course in gradshyuate writing offered in one of several different departments Students receive neither their essays nor any feedback other than the fmal score

6 Perform ongoing construct validation studies

From the above discussion it should be clear that no standardized instrument is expected to be used repeatedly without a ramporou~program of ongoing c~-sectmct valiltiatiOll Any standardized test once developed must be accompanied by sysshy~

tematic periodic corroboration of its effectiveness and by steps toward its improveshyment This rigor is especially true of tests that are produced in equated forms that is forms must be reliable across tests such that a score on a subsequent form of a test-has-the~same validityand-interpretability as its original

(A) The TOEFL program in cooperation with other tests produced by ETS has an impressive program of research Over the years dozens of TOEFL-sponsored research studies have appeared in the TOEFL Monograph Series An early example ofsuch a study was the seminal Duran et aI (1985) study TOEFLfrom a Communicative ViewpOint on Language Proficiency which examined the content characteristics of the TOEFL from a communicative perspective based on current research in applied linguistics and lanshyguage proficiency assessment More recent studies (such as Ginther 2001 Leacock amp Chodorow 2001 Powers et aI 2002) demonstrate an impressive array of scrutiny

(B) For approximately 20 years the ESLPT appeared to be placing students relishyably by means of an essay and a multiple-choice grammar and vocabulary test Over the years the security of the latter became s1lspect and the faculty administrators wished to see some content validity achieved in the process In the year 2000 that process began with a group of graduate students (Imao et aI 2000) in consl1ltation with faculty members and continued to fruition in the form of a new ESLPT reported in lmao (2002) The development of the new ESlPT involved a lengthy process of

82 CHAPTER 4 Standardized Testing

both content and construct validation along with facing such practical issues as scoring the written sections and a machine scorable multiple-choice answer sheet

The process of ongoing validation will no doubt continue as new forms of the editing section are created and as new prompts and reading passages are created for the writing section Such a validation process should also include consistent checks on placement accuracy and on face validity

(C) At this time there is little or no research to validate the GET itself For its conshy struct validation its administrators rely on a stockpile of research on university-level academic writing tests such as theTWEThe holistic scoring rubric and the topics and administrative conditions of the GET are to some extent patterned after that of the TWE In recent years some criticism of the GEf has come from international test-takers (Hosoya 2001) who posit that the topics and time limits of the GET among other facshytors work to the disadvantage of writers whose native language is not English These validity issues remain to be fully addressed in a comprehensive research study

I I

STANDARDIZED IANGUAGE PROFICIENCY TESTING

Tests of language profiCiency presuppose a comprehensive definition of the specific competencies that comprise overall language ability The specifications for the TOEFL provided an illustration of an operational definition of ability for assessment purposes This is not the only way to conceptualize the concept Swain (1990) offered a multidimensional view of profiCiency assessment by referring to three linshyguistic traits (grammar discourse and sociolinguistics) that can be assessed by means of oral multiple-choice and written responses (see Table 41) Swains conshyception was not meant to be an exhaustive analysis of ability but rather to serve as an operational framework for constructing proficiency assessments

Another defmition and conceptualization of profiCiency is suggested by the ACTFL association mentioned earlier ACfFL takes a holistic and more unitary view of proficiency in describing four levels superior advanced intermediate and noviceWithin each level descriptions of listening speaking reading and writing are provided as guidelines for assessment For example the ACfFL Guidelines describe the superior level of speaking as follows

ACTFL speaking guidelines summary superior-level

Superior-level speakers are characterized by the ability to

bull participate fully and effectively in conversations in formal and informal settings on topics related to practical needs and areas of professional andor scholarly interests

bull provide a structured argument to explain and defend opinions and develop effective hypotheses within extended discourse

bull discuss topics concretely and abstractly bull deal with a linguistically unfamiliar situation bull maintain a high degree of linguistic accuracy bull satisfy the linguistic demands of professional andor scholarly life

CHAPTER4 Standardized Testing 83

The other three ACfFL levels use the same parameters in describing progressively lower proficiencies across all four skills Such taxonomie~ have the advantage of considering a number of functions of linguistic discourse but the disadvantage at the lower levels of overly emphasizing test-takers deficiencies

Table 41 Traits of second language proficiency (Swain 1990 p 403)

Trait Grammar Discourse Sociolinguistic

focus on grammatical focus on textual focus on social accuracy within cohesion and appropriateness of sentences coherence language use

Method

Oral structured interview story telling and argumentationpersuasion

role-play ofspeech acts requests offers complaints

scored for accuracy of verbal morphology prepositions syntax

detailed rating for identification logical sequence and time orientation and global ratings for coherence

scored for ability to distinguish formal and informal register

Multiple-choice

sentence-level select the correct form exercise

paragraph-level select the coherent sentence exercise

speech act-Ievelselect the appropriate utterance exercise

(45 items) (29 items) (28 items)

involving verb morphology prepositionsan-d-uther items

Written composition

narrative and letter of persuasion

narrative and letter of persuasion

formal request letter and informal note

scored for accuracy of verb morphology prepositions syntax

detailed ratings much as for oral discourse and global rating for coherence

scored for the ability to distinguish formal and inforJ1lil1 register

FOUR STANDARDIZED lANGUAGE PROFICIENCY TESTS

We now tum to some of the better-known standardized tests of overall language ability or profiCiency to examine some of the typical formats used in commercially available tests We will not look at standardized tests of other specific skills here but that should not lead you to think by any means that proficiency is the only kind of test in the field that is standardized Three standardized oral production tests the

84 CHAPTER 4 Standardized Testing

Test of Spoken English (fSE) the Oral Proficiency Inventory (OPI) and PbonePassreg are discussed in Chapter 7 and the Test of Written English (WE) is covered in ChapterS

Four commercially produced standardized tests of English language proficiency are described briefly in this section the TOEFL the Michigan English Language Assessment Battery (MELAB) the International English Language Testing System (lELTS) and the Test of English for International Communication (fOEICreg) In an appendix to this chapter are sample items from each section of each test When you turn to that appendix use the following questions to help you evaluate these four tests and their subsections

1 What item types are included 2 How practical and reliable does each subsection of each test appear to be 3 Do the item types and tasks appropriately represent a conceptualizatio~ of

language proficiency (ability) That is can you evaluate their construct validity

4 Do the tasks achieve face validity 5 Are the tasks authentic 6 Is there some washback potential in the tasks

Test of English as a Foreign Language (TOEFL)

Producer Educational Testing Service (ETS) Objective To test overall proficiency (language ability) Primary market Almost exclusively US universities and colleges for admission

purposes Type Computer-based (CB) (and two sections are-computer-adaptive)

A traditional paper-based (PB) version is also available Response modes Multiple-choice responses essay Specifications See the box on pp 72-73 Time allocation Up to 4 hours (CB) 3 hours (PB) Internet access wwwtoeflorg

Comments In the North American context the TOEFL is the most widely used comshymercially available standardized test of proficiency Each year the TOEFL test is adminisshytered to approximately 800000 candidates in more than 200 countries It is highly respected because of the thorough program of ongoing research and development conshyducted by ETS The TOEFLs primary use is to set proficiency standards for international students seeking admission to English-speaking universities More than 4200 academic institutions government agencies scholarship programs and licensingcertification agenshycies in more than 80 countries use TOEFL scores By 2004 the TOEFL will include a secshytion on oral production

CHAPTER 4 Standardi~ed Testing 85

Michigan English Language Assessment Battery (MELAB)

Producer English language Institute University of Michigan Objective To test overall proficiency (language ability) Primary market Mostly US and Canadian language programs and colleges

some worldwide educational settings as well Type Paper-based Response modes Multiple-choice responses essay Time allocation 25 to 35 hours Internet access wwwlsaumicheduelimelabhtm

Specifications The MElAB consists of three sections Part 1 a 3D-minute impromptu essay is written on an assigned topic Part 2 a 25-minute multiple-choice listening comshyprehension test is delivered via tape recorder Part 3 is a 100-item 75-minute multipleshychoice test containing grammar doze reading vocabulary and reading comprehension An oral interview (speaking test) is optional

Comments The Ell at the University of Michigan has been producing the MELAB and its earlier incarnation (Michigan Test of English language Proficiency) since 1961 like the TOEFL it serves a North American audience but is also used internationally While its use is not as widespread as the TOEFL its validity is widely respected Because it is cheaper than the TOEFL and more easily obtained it is popular among language schools and institutes Many institutions and companies accept MElAB scores in lieu ofTOEFL scores

International English Language Testing System (IELTS)

Producer Jointly managed by The University of Cambridge local Examinations Syndicate (UClES) The British Council and lOP Education Australia

Objective To test overall proficiency (language ability) Primary-market Australian British Canadian and New Zealand academic

institutions and professional organizations American academic institutions are increasingly accepting IELTS for admissions purposes

1)rpe Computer-based (for the Reading and Writing sections) papershybased for the listening and Speaking modules

Response modes Multiple-choice responses essay oral production Time allocation 2 hours 45 minutes Internet access httpwwwieltsorgl

httpwwwudesorguk httpwwwbritishcouncilorg

Specifications Reading candidates choose between academic reading or general training reading (60 minutes) Writing the same option academic writing or general training writing (60 minutes) Listening four sections for all candidates (30 minutes) Speaking five sections for all candidates (1015 minutes)

86 CHAPTER 4 Standardized Testing

Comments The University of Cambridge local Examinations Syndicate (UCLES) has been producing English language tests since 1858 Now with three organizations cooperatshying to form the IELTS more than a million examinations are administered every year In 2002 a computer-based version of the Reading and Writing modules of the IELTS became available at selected centers around the world The other sections are administered locally by an examinet The paper-based IELTS remains an option for candidates The IELTS retains the distinct advantage of requiring all four skills in the test-takers performance

Test of English for International Communication (TOEICreg)

Producer The Chauncey Group International a subsidiary of Educational Testing Service

Objelttive To test overall proficiency (langlJage ability) Primary market Worldwide business commerce and industry contexts

(workplace settings) Type Computer-based and paper-based versions Response modes Multiple-choice responses Time allocation 2 hours Internet access httpwwwtoeiccom

Specifications Listening Comprehension 100 items administered by audiocassette Four types of task statements questions short conversations and short talks (approxishymately 45 minutes) Reading 100 items Three types of task cloze sentences error recogshynition and reading comprehension (75 minutes)

Comments The TOEIC has become a very widely used international test of English proficiency in workplace settings where English is required for job performance The conshytent includes many different employment settings such as conferences presentations sales ordering shipping schedules reservations (etters and memoranda It is approprishyate to use in educational settings where vocational or workplace English courses are being offered

sect sect sect sect sect

The construction of a valid standardized test is no minor accomplishment whether the instrument is large- or small-scale The designing of specifications alone as this chapter illustrates requires a sophisticated process of construct valishydation coupled with considerations of practicality Then the construction of items and scoringinterpretation procedures may require a lengthy period of trial and error with prototypes of the final form of the testWith painstaking attention to all the details of construction the end product can result in a cost-effective timeshysaving accurate instrument Your use of the results of such assessments can provide useful data on learners language abilities But your caution is warranted as well for all the reasons discussed in this chapter The next chapter will elaborate on what lies behind that need for a cautious approach to standardized assessment

CHAPTER4 Standardized Testing 87

EXERCISES

[Note (I) Individual work (G) Group or pair work (C) Whole-class discussion]

1 (C) Tell the class about the worst test experience youve ever had Briefly anamiddot lyze what made the experience so unbearable and try to come up with sugshygestions for improvement of the test andor its administrative conditions

2 (G) In pairs or small groups compile a brief list of pros and cons of standardshyized testing Cite illustrations of as many items in each list as possible Report your lists and examples to the rest o~ the class

3 (I) Select a standardized test that you are quite familiar with (probably a recent experience) Mentally evaluate that test using the five principles of practicality reliability validity authenticity and washback Report yourevaluashytion to the class

4 (G) The appendix to this chapter provides sample items from Jour different tests of language proficiency In groups one test for each group analyze your test for (a) content validity (b) face validity and (c) authenticity

5 (C) Do you think that the sample TOEFL reading passage about pirates (pages 74-75) and the Graduate EssayTest prompt (pages 76-77) about a school board hiring committee have any culture bias Discuss this and other cultural biases you have noticed in tests Is it possible to design a test that is completely free of culture bias

6 (CG) Compare the differences in conceptualization of language proficiency represented by Swains model the TOEFL and the ACfFL philosophy Which one best represents current thinking about communicative language ability What are the strengths and weaknesses of each approach

FORYOlIILEURTHER READING

Gronlund Norman E (1998) Assessment of student achievement Sixth Edition Boston Allyn and Bacon

Gronlunds classic also mentioned in Chapter 3 offers a concise overview of features of standardized tests offering definitions and examples of the statistical considerations in interpreting scores His approach is unbiased cleady written and accessible to those who might fear the mathematics of standardized testing

Phillips Deborah 2001 Long1nan introductory course for the TOEFL test White Plains NY Pearson Education

A careful examination of this or any other reputable preparation course for a standardized language test is well worth a students time Note especially how the book acquaints the user with the specifications of the test and offers a number of useful strategie~ that can be llsed in preparation for the test and during irs adn1inistration

88 CHAPTER 4 Standardized Testing

APPENDIX TO CHAPTER 4

Commercial Proficiency Tests Sample Items and Tasks

Test of English a~ a Foreign Language (TOEFLreg)

Listening r

Part A

In this section you will hear short conversations between two people In some ofthe conversations each person speaks only once In other conversations one or both of the people speak more than once Each conversation is followed by one questionabQlt it Each question in this part has four answer choices You should click on the best answer to each question Answer the questions on the basis of what is stated or implied by the speakers Here is an example On the computerscreen you will see

[man and woman talking]

On the recording you will hear

(woman) Hey wheres your sociology book (man) At home Why carry it around when were just going to be taking

a test (woman) Dont you remember Professor Smith said we could us it during

the test (man) Ohl no Well Ive still got an hour right Im so glad I ran into you

You wiII then see and hear the question before the answer choices appear

What will the man probably do next

o Begin studying for the sociology test o Explain the problem to his professor o Go home to get his textbook o Borrow the womans book

To choose an answer you will click on an oval The oval next to that answer will darken After you click on Next and Confirm Answer the next conversation will be presented

Part B

In this section you will hear several longer conversations and talks Each conversation or talk is followed by several questions The conversations talks and questions will not be repeated The conversations and talks are about a variety of topics You do not need speshycial knowledge of the topics to answer the questions correctly Rather you should answer each question on the basis of what is stated or implied by the speakers in the conversashytions or talks

For most of the questions you will need to click on the best of four possible answers Some questions will have special directions The special directions will appear in a box on the computer screen Here is an exampie ot a conversation and some questions

CHAPTER 4 Standardized Testing 89

Marine Biology (narrator) Listen to part of a discussion in a marine biology class

(professor) A few years ago our local government passed a number of strict environmental laws As a result Sunrise Beach looks nothing Ii ke it did ten years ago The water is cleaner and theres been a tremendous increase in all kinds of marine life which is why were going there on Thursday

(woman) I dont know if I agree that the water quality has improved I mean I was out there last weekend and it looked all brown It didnt seem too clean to me

(professor) Actually the color of the water doesnt always indicate whether its polluted The brown color you mentioned might be a result of pollution or it can mean a kind of brown algae is growing there Its called devils apron and it actually serves as food for whales

(man) So when does the water look blue (professor) Well water thats completely unpolluted is actually colorless But

it often looks bluish-green because the sunlight can penetrate deep down and thats the color thats reflected

(woman) But sometimes it looks really green Whats that about (professor) Ok well its the same principle as with devils apron the

water might be green because of different types of green algae there-gulfweed phytoplankton You all should finish reading about algae and plankton before we go In fact those are the types of living things Im going to ask you to be looking for when were there

Now get ready to answer the questions

What is the discussion mainly about

o The importance of protecting ocean environments o The reasons why ocean water appears to be different colors o The survival of whales in polluted water o The effect that colored ocean water has on algae

To choose an answer click on an oval The oval next to that answer will darken After you click on Next and Confirm Answer the next question will be presented

According to the professor what can make ocean water look browngt

o Pollution o Cloudy Skies o Sand o Algae

Click on 2 answers

To choose your answers you will click on the squares An XII wiii appear in each square

bullbullbullbullbullbullbull

90 CHAPTER 4 Standardized Testing

Structure and Written Expression This section measures the ability to recognize language that is appropriate for standard written English There are two types ofquestions in this section In the first type ofquestion there are incomplete sentences Beneath each sentence there are four words or phrases

Directions CIiSk on the one word or phrase that best completes the sentence

The colum~ine flower __ to nearly all of the United States can be raised from seed in almost any garden

native how native is how native is it is native

Time Help Confirm

After you click on Next and Confirm Answ~ the next question willbe presented

The second type of question has four underlined words or phrases You will choose the one underlined word or phrase that must be changed for the sentence to be correct

Directions Click on the one underlined word or phrase that must be changed for the senshytence to be correct

One of the most difficult problems in understanding sleep is determining what the funcshytions of sleep ~

lime Help Confirm

Clicking on an underlined word or phrase will darken it

Reading This section measures the ability to read and understand short passages similar in topic and style to those that students are likely to encounter in North American universities and colleges This section contains reading passages and questions about the passages There are several different types of questions in this section In the Reading section you will first have the opportunity to read the passage

The temperature of the Sun is over 10000 degrees Fahrenheit at the surface but it rises perhaps more than 270000000 at the center The Sun is so much hotter than the Earth that matter can exist only as a gasl except perhaps at the core In the core of the Sun the pressures are so great that despite the high temperature there may be a small solid core However no one really knows since the center of the Sun can never be directly observed ~ Solar astronomers do know that the Sun is divided into five general layers or zones Starting at the outside and going down into the Sun the zones are the corona chromoshysphere hotosphere convection zone and finally the core The first three zones are reshygarded as the Suns atmosphere But since the Sun has no solid surface it is hard to middottell where the atmosphere ends and the main body of the Sun begins

The Suns outermost layer begins about 10000 miles above the visible surface and goes outward for millions of miles This is the only part of the Sun that can be seen during an eclipse such as the one in February 1979 At any other time the corona can be seen

bullbullbullbullbullbullbull

bull bullbullbullbullbullbull

CHAPTER 4 Standardized Testing 91

only when special instruments are used on cameras and telescopes to block the light from the photosphere

The corona is a brilliant pearly white filmy light about as bright as the full Moon Its beautiful rays are a sensational sight during an eclipse The coronas rays flash out in a brilliant fan that has wispy spikelike rays near the Suns north and south poles The corona is generally thickest at the Suns equator The corona is made up of gases streamshying outward at tremendous speeds that reach a temperature of more than 2 million deshygrees Fahrenheit The gas thins out as it reaches the space around the planets By the time the gas of the corona reaches the Earth it has a relatively low density

When you have finished reading the passage you will use the mouse to click on Proceed Then the questions about the passage will be presented You are to choose the one best anshyswer to each question Answer all questions about the information in a passage on the basis ofwhat is stated or implied in that passage Most ofthe questions will be multiple-choice questions To answer these questions you will click on a choice below the question

With what topic is paragraph 2 mainly concerned

o How the Sun evolved o The structure of the Sun o Why scientists study the Sun o The distaflce of the Sun from the planets

Paragraph 2 is marked with an arrow (~)

You will see the next question after you click on Next

To answer some questions you will click on a word or phrase Here is an example

Look at the word one in the passage Click on the word or phrase in the bold text that one refers to To answer you can click on any part of the word or phrase in the passage Jour choice will darken to show which word you have chosen

The Suns outermost layer begins about 10000 miles above the visible surface and goes outward for millions of miles This is the only part of the Sun that can be seen durshying an eclipse such as the one in February 1979 At any other time the corona can be seen only when special instruments are used on cameras and telescopes to block the Iight from the photosphere

You will see the next question after you click on~ To answer some q~estions you will click on a sentence in the passage Here is an example

~ The corona is a brilliant pearly white filmy light about as bright as the full Moon Its beautiful rays are a sensational sight during an eclipse The coronas rays flash out in a brilliant fan that has wispy spikelike rays near the Suns north and south poles The corona is generally thickest at the Suns equator ~ The corona is made up of gases streaming outward at tremendous speeds that reach a temperature of more than 2 million degrees Fahrenheit The gas thins out as it reaches the space around the planets By the time the gas of the corona reaches the Earth it has a relatively low density

bull bullbullbullbullbullbull

92 CHAPTER 4 Standardized Testing

Click on the sentence in paragraph 4 or 5 in which the author compares the light of the Suns outermost layer to that ofanother astronomical body Paragraphs 4 and 5 are marked with arrows (~)

To answer some questions you will click on a square to add a sentence to the passage Here is an example -The following sentence can be added to paragraph 1

At the center of the Earths solar system lies the Sun

Where would it best fit in paragraph I Click on the square to add the sentence to the paragraph

D The temperature of the Sun is over 10000 degrees Fahrenheit at the surface but it rises to perhaps morethan 27000000deg at the center 0 The Sun is so much hotter than the Earth that matter can exist only as a gasi except p~rHapsatth~ c6relp the c~re of the ii Sun the pressures are so great that despite the high temperature there may be a small solid core D However no one really knows since the center of the Sun can never be directly observed D 0100

When you click on a square the sentence will appear in the passage at the place you have chosen You can read the sentence added to the paragraph to see if this is the best place to add it You can click on another square to change your answer The sentence will be added and shown in a dark box

Writing In this section you will have an opportunity to demonstrate your ability to write in Enshyglish This includes the ability to generate and organize ideas to support those ideas with examples or evidence and to compose in standard written English in response to an asshysigned topic You will have 30 minutes to write your essay on that topic You must write on the topic you are assigned An essay on any other topic will receive a score of 0 Read the topic below and then make any notes that will help you plan your response Begin typing your response in the box at the bottom of the screen or write your answer on the answer sheet provided to you

Following is a sample topic

Do you agree or disagree with the following statemenH

Teachers should make learning enjoyable and fun for their students

Use specific reasons and examples to support your opinion

CHAPTER 4 Standarczed Testing 93

Michigan English Language Assessment Battery (MELAB)

Composition The time limit for the composition is 30 minutes You must write on only one of the top~

ics below If you write about something else your composition paper will not be graded and you cannot be given a final score If you do not understand the topics ask the exam~ iner to explain or to translate them You may be asked to give your opinion ofsomething and explain why you believe this to describe something from your experience or to exshyplain a problem and offer possible solutions You should write at least one page Some sample topics are

1 What do you think is your countrys greatest problem Explain in detail and tell what you think can be done about it

2 What are the characteristics of a good teacher Explain and give examples 3 An optimist is someone who sees the good side of things A pessimist sees the

bad side Are you an optimist or a pessimist Relate a personal experience that shows this

4 In your opinion are the benefits of space exploration really worth the enormous costs Discuss

Most MELAB compositions are one or two pages long (about 200-300 words) If your paper is extremely short (less than 150 words) your composition will be given a lower score Before you begin writing you might want to take 2 or 3 minutes to plan your comshyposition and to make a short outline to organize your thoughts Such outlines will not be graded they are only to help you You should use the last 5 minutes to read through your composition and to make changes or corrections

Your composition will be graded on how clearly you express yourself in English and on the range of English you are able to use and your control in doing so This means your composition should be well organized your arguments should be fully developed and you should show a range ofgrammatical structures and broad vocabulary Compositions that consist only of very short sentences and very simple vocabulary cannot be given the

middothighest scores If errors are not frequent and if they do not confuse your meaning they will not lower your score very much

Listening Now you will hear a short lecture You may take notes during the lecture Following the lecture you will be asked some questions about it

Therell be a two-week exhibit of the paintings of the little-known master Laura Bernhart at the Claire Osmond Galleries starting on the fifteenth of the month and running through the thirtieth Bernharts known for her innovative designs in abstract expressionism Though a true original she declared a spiritual heritage from Salvador Dali the famous Spanish painter Since Bernhart lived a rather solitary life and died while only in her twenties few people are aware of her works This showing at the Osmond Galleries will provide many with an introduction to her works

10 Where is the exhibit a the Art Museum b the Dali Galleries c the Osmond Galleries

94 CHAPTER 4 Standardized Testing

11 What is Bernhart known for a her copies of Dalis paintings b the originality of her designs c her exhibitions

12 What will going to the exhibit allow most people to do a to see Saivador Dalis paintings b to see Bernharts works for the first time c to learn about Spanish art

Grammar

1 What did the teacher just tell you

She reminded our notebooksI a us to bring b that we bring c our bringing d we should bring

2 Is Bill a good dancer

Not really __ he tries very hard a in spite of h despite c even though d while

3 your clothes are all wet1

Yes I didnt come __ the rain soon enough a away to b over to c down with d in from

Cloze In years to come zoos will not only be places where animals are exhibited to the public but repositories where rare species can be saved from extinction (7) captive breeding The most powerful force (8) the future of many animals-and of zoos-is the decline of the wild (9) even zoo directors would argue that (10) are better places for animals than the fields and forest of their native (11) yet zoos may be the last chance for some creatures that would otherwise pass qUietly into oblivion

7 a through c from b of d damage

8 a bringing c to b that d influencing

9 a But c Not b So d Then

10 a where c even b zoos d wilds

11 alands c residence b life d field

CHAPTER 4 Standardized Testing 95

Vocabulary

12 Mark has a flair for writing a need b purpose c talent d dislike

13 Bill Collins launched his restaurant last June a moved b started c sold d bought

14 John will not accept the censure a burden b blame c credit d decision

15 I cant think of the answer Can you give me a __ a hint b token c taste d gaze

16 Because fewer people are taking expensive vacations the tourist industry is in a a choke b grope c grumble d slump

17 I disagree with a few of his opinions but __ we agree a deliberately b conclusively c essentially d immensely

Reading The influenza virus is a single molecule built from many millions of single atoms You must have heard of the viruses which are sometimes called living molecules While bacteria can be considered as a type of plant secreting pOisonous substances into the body of the organism they attack viruses are living organisms themselves We may conshysider them as regular chemical molecules since they have a strictly aefined atomic strucshyture but on the other hand we must also consider them as being alive since they are able to multiply in unlimited quantities

18 According to the passage bacteria are a poisons

b larger than viruses c very small d plants

96 CHAPTER 4 Standardized Testing

19 The writer says that viruses are alive because they a have a complex atomic structure b move c multiply d need warmth and light

20 The atomic structure of viruses a is -tJIariable b is strictly defined c cannot be analyzed chemically d is more complex than that of bacteria

International English Language Testing System (fELTS)

I

listening

The Listening Module has four sections The first two sections are concerned with social needs There is a conversation between two speakers and then a monologue For examshyple a conversation about travel arrangements or decisions on a night out and a speech about student services on a university campus or arrangements for meals during a confershyence The final two sections are concerned with situations related more closely to educashytional or training contexts For example conversation between a tutor and a student about an assignment or between three students planning a research project and a lecture or talk ofgeneral academic interest All the topics are ofgeneral interest and it makes no difference what subjects candidates study Tests and tasks become more difficult as the sections progress A range of English accents and dialects are used in the recording which reflects the international usage of IELTS

Academic Reading [A 7S0-word article on-th-e- topic of Wind Power in the US with a short glossary at the end]

Questions 1-5

Complete the summary below

Choose your answers from the box below the summary and write them in boxes 1-5 on your answer sheet Note There are more words or phrases than you will need to fill the gaps You may use any word or phrase more than once

Example The failure during the late 1970s and early 19805 of an attempt to establish a widespread wind power industry in the United States resulted largely from the (1) bull in oil prices during this period The industry is now experiencing a steady (2) due to improveshyments in technology and an increased awareness of the potential in the power of wind The wind turbines that are now being made based in part on the (3) of wide- ranging research in Europe are easier to manufacture and maintain than their predecesshysors This has led wind-turbine makers to be able to standardise and thus minimize (4) There has been growing (S) of the importance of wind power as an energy source

CHAPTER 4 Standardized Testing 97

criticism stability skepticism success operating costs decisions design costs fall effects production costs growth decline failure recognition results

Questions 6-1 0 Look at the following list of issues (Questions 6-10) and implications (A-C) Match each issue with one implication Write the appropriate letters A-C in boxes 6-10 on your anshyswer sheet

Example The current price of one wind-generated kilowatt Answer

6 The recent installation of systems taking advantage of economies of scale

7 The potential of meeting one fifth of current U5 energy requirements by wind power

8 The level of acceptance of current wind turbine technology

9 A comparison of costs between conventional and wind power sources

10 The view of wind power in the European Union

Implications

A provides evidence against claims that electricity produced from wind power is relatively expensive

B supports claims that wind power js an important source of energy

C opposes the view that wind power technology requires further-development

General Training Reading Read the passage on Daybreak trips by coach and look at the statements below On your answer sheet write

TRUE if the statement is true FALSE jf the statement is false

NOlGIVEN if the information is not given in the leaflet

1 MiIlers Coaches owns Cambridges Cam bus fleet

2 Premier is an older company than Millers

3 Most of the Daybreak coaches are less than 5 years old

4 Daybreak fares are more expensive than most of their competitors

5 Soft drinks and refreshments are served on most longer journeys

6 Smoking is permitted at the rear of the coach on longer journeys

7 Tickets must be bought in advance from an authorised Daybreak agent

6 Tickets and seats can be reserved by phoning the Daybreak Hotline

9 Daybreak passengers must join their coach at Cambridge Drummer Street

10 Daybreak cannot guarantee return times

98 CHAPTER 4 Standardized Testing

FROM CAMBRIDGE AND SURROUNDING AREA

SPRING IS INTHEAIR

Welcome to our Spring Daybreak programme which continues the tradition of offering unbeatable value for money day trips and tours All the excursions in this brochure will be operated by Pr~mier Travel Services Limited or Millers Coaches both companies are part of the CHLGroup owners of Cambridges Cambus fleet

WERE PROUD OF OUR TRADITION

Premier was established in 1936 the Company now offers the highest standards of coaching in todays competitive operating environment Miller has an enviable reputation stretching back over the past 20 years offering coach services at realistic prices Weve traveled a long way since our early days of pre-war seaside trips Now our fleet of 50 modern coaches (few are more than five years old) operate throughout Britain and Europe but were pleased to still maintain the high standards of quality and service the trademark of our founders nearly sixty years ago

EXCLUSIVE FEATURES

Admission-inclusive fares All Daybreak fares (unless specifically otherwise stated) include admission charges to the attractions shows and exhibits we visit Many full-day scenic tours are accompanied by a fully trained English Tourist Board Blue Badge guide or local experienced driverguide Some Daybreaks include lunch or afternoon tea Compare our admission inclusive fares and see how much you save Cheapest is not the best and value for money is guaranteed If you compare our bargain Daybreak fares beware--most of our competishytors do not offer an all-inclusive fare

SEAT RESERVATIONS

We value the freedom of choice so you can choose your seat when you book The seat reservation is guaranteed a-nd remains yours at all times when aboard the coach

NO SMOKING COMFORT

With the comfort of our passengers in mind coaches on all our Daybreaks are no smokshying throughout In the interests of fellow passengers comfort we kindly ask that smokers observe our no smoking policy On scenic tours and longer journeys ample refreshment stops are provided when of course smoking is permitted

YOUR QUESTIONS ANSWERED

Do I need to book Booking in advance is strongly recommended as all Daybreak tours are subject to demand Subject to availability stand-by tickets can be purchased from the driver

What ti me does the coach leave The coach departs from Cambridge Drummer Street (Bay 12 adjacent to public toilets) at the time shown There are many additional joining points indicated by departure codes in the brochure If you are joining at one of our less popular joining points you will be adshyvised of your pick-up time (normally by telephone) not less than 48 hours before deparshyture In this way we can minimize the length of pick-up routes and reduce journey times for the majority of passengers

CHAPTER 4 Standardized Testing 99

What time do we get back An approximate return time is shown for each excursion The tim~s shown serve as a guide but road conditions can sometimes cause delay If your arrival will be later than advertised your driver will try to allow for a telephone call during the return journey

Where can I board the coach All the Daybreaks in the brochure leave from Cambridge Drummer Street (Bay 12 adjashycent to public toilets) at the time shown Many Daybreaks offer additional pick-ups for pre-booked passengers within Cambridge and the surrounding area This facility must be requested at the time of booking

Academic Writing Writing Task 1 You should spend about 20 minutes on this task

The graph below shows the different modes of transport used to travel to and from work in one European city in 1950 1970 and 1990

[graph shown here]

Write a report for a university lecturer describing the information shown below You should write at least 150 words

Writing Task 2 You should spend about 40 minutes on this task

Present a written argument or case to an educated reader with no specialist knowledge of the folowing topic

It is inevitable that as technology develops so traditional cultures must be lost Technolshyogy and tradition are incompatible-you cannot have both together

To what extent do you agree or disagree with this statement Give reasons for your answer You should write at least 250 words You should use your own ideas knowlshyedge and experience and support your arguments with examples and relevant evidence

General Training Writing Writing Task 1 You should spend about 20 minutes on this task You rent a house through an agency The heating system has stopped working You phoned the agency a week ago but it has still not been mended Write a letter to the agency Explain the situation and teil them what you want them to do about it

You should write at least 150 words You do NOT need to write your own address

Begin your letter as follows

Dear - ___-I

Writing Task 2 You should spend about 40 minutes on this task As part ofa class assignment you have to write about the following topic

100 CHAPTER 4 Standardized Testing

Some businesses now say that no one can smoke cigarettes in any of their offices Some governments have banned smoking in all public placesThis is a good idea but it takes away some of our freedom

Do you agree or disagree Give reasons for your answer You should write at least 250 words

Speaking In each ofthe three parts of the speaking module a specific function is fulfilled In Part 1 the candidates answer general questions about themselves their homes or families their jobs or studies their interests and a range ofsimilar familiar topic areas This part lasts between four and five minutes In Part 2 the candidate is given a verbal prompt on a card and is asked to talk on a particular topic The candidate has one minute to prepare before speaking at length for between one and two minutes The examiner then asks one or two wind-down questions In Part 3 the examiner and candidate engage in a discusshysion of more abstract issues and concepts which are thematically linked to the topic prompt in Part 2 The discussion lasts between four and five minutes

All interviews are recorded on audiocassette Here is a sample ofa Part 2 topic

Describe a teacher who has greatly influenced you in your education

You shou Id say

where you met them what subject they taught what was special about them

and explain why this person influenced you so much

You will have to talk about the topic for 1 to 2 minutes You have 1 minute to think about what you are going to say You can make some notes if you wish

Test of English for International Communication (TOEICreg)

listening

Part 1 Photographs Directions For each question you will see a picture in your test book and you will hear four short statements The statements will be spoken just one time They will not be printed in your test book so you must listen carefully to understand what the speaker says When you hear the four statements look at the picture in your test book and choose the statement that best describes what you see in the picture Then on your answer sheet find the number of the question and mark your answer

[photograph of a scientist looking through a microscope]

You will hear Look at the picture marked number 1 in your test book

(A) Shes speaking into a microphone (B) Shes put on her glasses (C) She has both eyes open (D) Shes using a microscope

CHAPTER 4 Standardized Testing 101

Part 2 Question-Response Directions In this part of the test you will hear a question or statement spoken in Enshyglish followed by three responses also spoken in English The question or staten1ent and the responses will be spoken just one time They will not be printed in your test book so you must listen carefully to understand what the speakers say You are to choose the best response to each question or statement

Question 1 You will hear Ms Morikawa has worked here for a long time hasnt she

(A) At three oclock (B) No Ive lost my watch (C) More than ten years

Question 2 You will hear Which of these papers has a wider circulation

(A) The morning edition (B) Get more exercise (C) By messenger

Part 3 Short Conversations Directions In this part of the test you will hear short conversations between two people The conversations will not be printed in your test book You will hear the conversations only once so you must listen carefully to understand what the speakers say In your test book you will read a question about each conversation The question will be followed by four answers You are to choose the best answer to each question and mark it on your answer sheet

Question 1 (Man) We should think about finding another restaurant for lunch (Woman) Why The food and service here are great

(Man) Yes but the prices are going up every week

You will read Why is this man unhappy with the restaurant

(A) It is too noisy (B) It is too expensive (C) It is too crowded (D) It is too difficult to find

Question 2 (Woman A) How was Dr Borgs recent trip to Singapore (Woman B) She enjoyed the tour of the port very much (Woman A) They say its one of the most active in Asia

You will read 2 What did Dr Borg find interesting

(A) The tourist center (B) The airport (C) The musical performance (D) The harbor

Part 4 Short Talks Directions In this part of the test you vill hear several short talks Each will be spoken just one time They will not be printed in your test book so you must listen carefully to understand and remember what is said In your test book you will read two or more questions about each short talk The questions will be followed by four answers You are to choose the best answer to each question and mark it on your answer sheet

102 CHAPTER 4 Standardized Testing

You will hear Questions 1 and 2 refer to the following announcement

Good afternoon and welcome aboard Nordair Flight 857 from Copenhagen to Bangkok with intermediate stops in Dubai and Calcutta We are preparing for departure in a few minutes At this time your seat back should be returned to its full upright position and your seat belt s~ould be fastened OUf anticipated total flying time to Dubai is six hours and twenty-five minutes I hope you enjoy the flight You will hecJr Now read question 1 in your test book and answer it You will read 1 What is the final destination of the flight

(A) Bangkok (B) Copenhagen (C) Dubai (O) Calcutta

You will hear Now read question 2 in your test book and answer it You will read 2 What will happen in a few minutes

(A) The flight will land in Dubai I

(B) The passengers will board the plane (C) The plane will take off (0) The gate number will be announced

Reading In this section of the test you will have the chance to show how well you understand written English There are three parts to this section with special directions for each part

Part 4 Incomplete Sentences Directions This part of the test has incomplete sentences Four words or phrases marked (A) (8) (e) (D) are given beneath each sentence You are to choose the one word or phrase that best completes the sentence Then on your answer sheet find the number of the question and mark your answer

1 Mr Yangs trip will __ him away from the office for ten days (A) withdraw (B) continue (C) retain (0) keep

2 The company that Marie DuBois started now sells __ products throughout the world (A) its (B) it (C) theirs (D) them

3 If your shipment is not delivered __ Tuesday you can request a full refund for the merchandise (A) at (B) by (C) within (D) while

CHAPTER 4 Standardized Testing 103

Part 6 Error Recognition Directions In this part ofthe test each sentence has four words or phrases underlined The four underlined parts of the sentence are marked (A) (B) (C) (D) You are to identify the one underlined word or phrase that should be corrected or rewritten Then on your answer sheet find the number of the question and mark your answer

1 The pamphlet contains some importance information about the current exhibit ABC D

2 No matter how Jong it taking to finish the annual report it must be done properly ABC D

3 The popularity of jogging appears to have decreased since the past couple of years ABC D

Part 7 Reading Comprehension Directions The questions in this part of the test are based on a selection of reading mateshyrials such as notices letters) forms newspaper and magazine articles) and advertisements You are to choose the one best answer (A) (B) (C) or (OJ to each quesshytion Then on your ariswefsheelfindthe number of the qUestion andmcirkyour answer Answer all questions following each reading selection on thebasis of what is stated or implied in that selection

The Museum ofTechnology is a hands-on museum designed for people to experience science at w()rk~ Visitors are encouraged to use test and handle the objects o~ display Special demonstrations are scheduled for the first and second Wednesdays of each month at 1330 Open Tuesday-Friday 1200-1630 Saturday 1000-1730 and Sunday 11 00-1630

1 When during the month can visitors see special demonstrations (A) Every weekend (B) The first two Wednesdays (C) One afternoon a week (D) Every other Wednesday

Questions 2 and 3 refer to the followi ng notice

NOTICE If you are unable to work because of an extended illness or injury that is not workshyrelated you may be entitled to receive weekly benefits from your employer or the firms insurance company To claim benefits you must file a claim form within thirty days of the first day of your disability Before filing the claim you must ask your doctor to fill in the Doctors Statement on the claim form stating the period of disability

3 To whom is this notice addressed (A) Employers (8) Doctors (C) Employees (D) When paying the bill

4 When must the claim form be filed (A) On the first of the month (8) On the thirtieth of the month (C) On the first day ofdisabifity (D) Within 30 days of the start of disability

Page 16: Standardized Testing Chapter 4 Brown

CHAPTE84 Standardized Testing 81

2 Weak The writer makes somewhat simplistic suggestions not fully supported with reashysons fails to cite key facts offers little analysis of the problem or shows a limited grasp of the situation the given information is copied or listed withlittle integration into argument Points may be random or repetitious Writing may be badly focused with careless use of abstract language resulting in predication errors or illogical sentences

ESL andlor careless mechanical errors are frequent enough to be distracting OR sentences may be choppy style over-casual usage occasionally unidiomatic

1 Inadequate The essay may be disjointed incoherent or minimally developed The writer shows little grasp of the complex issues involved is unable to establish conshytext point of view or purpose in opening of paper or has a poor sense of audience Mechanical andor ESL errors or unidiomatic usages are frequent sentences may be ungrammatical OR correct but short and very simple

The two readers scores are added to yield a total possible score of 2 to 8 Test administrators recommend a score of 6 as the threshold for allowing a student to pursue graduate-level courses Anything below that is accompanied by a recomshymendation that the student either repeat the test or take a remedial course in gradshyuate writing offered in one of several different departments Students receive neither their essays nor any feedback other than the fmal score

6 Perform ongoing construct validation studies

From the above discussion it should be clear that no standardized instrument is expected to be used repeatedly without a ramporou~program of ongoing c~-sectmct valiltiatiOll Any standardized test once developed must be accompanied by sysshy~

tematic periodic corroboration of its effectiveness and by steps toward its improveshyment This rigor is especially true of tests that are produced in equated forms that is forms must be reliable across tests such that a score on a subsequent form of a test-has-the~same validityand-interpretability as its original

(A) The TOEFL program in cooperation with other tests produced by ETS has an impressive program of research Over the years dozens of TOEFL-sponsored research studies have appeared in the TOEFL Monograph Series An early example ofsuch a study was the seminal Duran et aI (1985) study TOEFLfrom a Communicative ViewpOint on Language Proficiency which examined the content characteristics of the TOEFL from a communicative perspective based on current research in applied linguistics and lanshyguage proficiency assessment More recent studies (such as Ginther 2001 Leacock amp Chodorow 2001 Powers et aI 2002) demonstrate an impressive array of scrutiny

(B) For approximately 20 years the ESLPT appeared to be placing students relishyably by means of an essay and a multiple-choice grammar and vocabulary test Over the years the security of the latter became s1lspect and the faculty administrators wished to see some content validity achieved in the process In the year 2000 that process began with a group of graduate students (Imao et aI 2000) in consl1ltation with faculty members and continued to fruition in the form of a new ESLPT reported in lmao (2002) The development of the new ESlPT involved a lengthy process of

82 CHAPTER 4 Standardized Testing

both content and construct validation along with facing such practical issues as scoring the written sections and a machine scorable multiple-choice answer sheet

The process of ongoing validation will no doubt continue as new forms of the editing section are created and as new prompts and reading passages are created for the writing section Such a validation process should also include consistent checks on placement accuracy and on face validity

(C) At this time there is little or no research to validate the GET itself For its conshy struct validation its administrators rely on a stockpile of research on university-level academic writing tests such as theTWEThe holistic scoring rubric and the topics and administrative conditions of the GET are to some extent patterned after that of the TWE In recent years some criticism of the GEf has come from international test-takers (Hosoya 2001) who posit that the topics and time limits of the GET among other facshytors work to the disadvantage of writers whose native language is not English These validity issues remain to be fully addressed in a comprehensive research study

I I

STANDARDIZED IANGUAGE PROFICIENCY TESTING

Tests of language profiCiency presuppose a comprehensive definition of the specific competencies that comprise overall language ability The specifications for the TOEFL provided an illustration of an operational definition of ability for assessment purposes This is not the only way to conceptualize the concept Swain (1990) offered a multidimensional view of profiCiency assessment by referring to three linshyguistic traits (grammar discourse and sociolinguistics) that can be assessed by means of oral multiple-choice and written responses (see Table 41) Swains conshyception was not meant to be an exhaustive analysis of ability but rather to serve as an operational framework for constructing proficiency assessments

Another defmition and conceptualization of profiCiency is suggested by the ACTFL association mentioned earlier ACfFL takes a holistic and more unitary view of proficiency in describing four levels superior advanced intermediate and noviceWithin each level descriptions of listening speaking reading and writing are provided as guidelines for assessment For example the ACfFL Guidelines describe the superior level of speaking as follows

ACTFL speaking guidelines summary superior-level

Superior-level speakers are characterized by the ability to

bull participate fully and effectively in conversations in formal and informal settings on topics related to practical needs and areas of professional andor scholarly interests

bull provide a structured argument to explain and defend opinions and develop effective hypotheses within extended discourse

bull discuss topics concretely and abstractly bull deal with a linguistically unfamiliar situation bull maintain a high degree of linguistic accuracy bull satisfy the linguistic demands of professional andor scholarly life

CHAPTER4 Standardized Testing 83

The other three ACfFL levels use the same parameters in describing progressively lower proficiencies across all four skills Such taxonomie~ have the advantage of considering a number of functions of linguistic discourse but the disadvantage at the lower levels of overly emphasizing test-takers deficiencies

Table 41 Traits of second language proficiency (Swain 1990 p 403)

Trait Grammar Discourse Sociolinguistic

focus on grammatical focus on textual focus on social accuracy within cohesion and appropriateness of sentences coherence language use

Method

Oral structured interview story telling and argumentationpersuasion

role-play ofspeech acts requests offers complaints

scored for accuracy of verbal morphology prepositions syntax

detailed rating for identification logical sequence and time orientation and global ratings for coherence

scored for ability to distinguish formal and informal register

Multiple-choice

sentence-level select the correct form exercise

paragraph-level select the coherent sentence exercise

speech act-Ievelselect the appropriate utterance exercise

(45 items) (29 items) (28 items)

involving verb morphology prepositionsan-d-uther items

Written composition

narrative and letter of persuasion

narrative and letter of persuasion

formal request letter and informal note

scored for accuracy of verb morphology prepositions syntax

detailed ratings much as for oral discourse and global rating for coherence

scored for the ability to distinguish formal and inforJ1lil1 register

FOUR STANDARDIZED lANGUAGE PROFICIENCY TESTS

We now tum to some of the better-known standardized tests of overall language ability or profiCiency to examine some of the typical formats used in commercially available tests We will not look at standardized tests of other specific skills here but that should not lead you to think by any means that proficiency is the only kind of test in the field that is standardized Three standardized oral production tests the

84 CHAPTER 4 Standardized Testing

Test of Spoken English (fSE) the Oral Proficiency Inventory (OPI) and PbonePassreg are discussed in Chapter 7 and the Test of Written English (WE) is covered in ChapterS

Four commercially produced standardized tests of English language proficiency are described briefly in this section the TOEFL the Michigan English Language Assessment Battery (MELAB) the International English Language Testing System (lELTS) and the Test of English for International Communication (fOEICreg) In an appendix to this chapter are sample items from each section of each test When you turn to that appendix use the following questions to help you evaluate these four tests and their subsections

1 What item types are included 2 How practical and reliable does each subsection of each test appear to be 3 Do the item types and tasks appropriately represent a conceptualizatio~ of

language proficiency (ability) That is can you evaluate their construct validity

4 Do the tasks achieve face validity 5 Are the tasks authentic 6 Is there some washback potential in the tasks

Test of English as a Foreign Language (TOEFL)

Producer Educational Testing Service (ETS) Objective To test overall proficiency (language ability) Primary market Almost exclusively US universities and colleges for admission

purposes Type Computer-based (CB) (and two sections are-computer-adaptive)

A traditional paper-based (PB) version is also available Response modes Multiple-choice responses essay Specifications See the box on pp 72-73 Time allocation Up to 4 hours (CB) 3 hours (PB) Internet access wwwtoeflorg

Comments In the North American context the TOEFL is the most widely used comshymercially available standardized test of proficiency Each year the TOEFL test is adminisshytered to approximately 800000 candidates in more than 200 countries It is highly respected because of the thorough program of ongoing research and development conshyducted by ETS The TOEFLs primary use is to set proficiency standards for international students seeking admission to English-speaking universities More than 4200 academic institutions government agencies scholarship programs and licensingcertification agenshycies in more than 80 countries use TOEFL scores By 2004 the TOEFL will include a secshytion on oral production

CHAPTER 4 Standardi~ed Testing 85

Michigan English Language Assessment Battery (MELAB)

Producer English language Institute University of Michigan Objective To test overall proficiency (language ability) Primary market Mostly US and Canadian language programs and colleges

some worldwide educational settings as well Type Paper-based Response modes Multiple-choice responses essay Time allocation 25 to 35 hours Internet access wwwlsaumicheduelimelabhtm

Specifications The MElAB consists of three sections Part 1 a 3D-minute impromptu essay is written on an assigned topic Part 2 a 25-minute multiple-choice listening comshyprehension test is delivered via tape recorder Part 3 is a 100-item 75-minute multipleshychoice test containing grammar doze reading vocabulary and reading comprehension An oral interview (speaking test) is optional

Comments The Ell at the University of Michigan has been producing the MELAB and its earlier incarnation (Michigan Test of English language Proficiency) since 1961 like the TOEFL it serves a North American audience but is also used internationally While its use is not as widespread as the TOEFL its validity is widely respected Because it is cheaper than the TOEFL and more easily obtained it is popular among language schools and institutes Many institutions and companies accept MElAB scores in lieu ofTOEFL scores

International English Language Testing System (IELTS)

Producer Jointly managed by The University of Cambridge local Examinations Syndicate (UClES) The British Council and lOP Education Australia

Objective To test overall proficiency (language ability) Primary-market Australian British Canadian and New Zealand academic

institutions and professional organizations American academic institutions are increasingly accepting IELTS for admissions purposes

1)rpe Computer-based (for the Reading and Writing sections) papershybased for the listening and Speaking modules

Response modes Multiple-choice responses essay oral production Time allocation 2 hours 45 minutes Internet access httpwwwieltsorgl

httpwwwudesorguk httpwwwbritishcouncilorg

Specifications Reading candidates choose between academic reading or general training reading (60 minutes) Writing the same option academic writing or general training writing (60 minutes) Listening four sections for all candidates (30 minutes) Speaking five sections for all candidates (1015 minutes)

86 CHAPTER 4 Standardized Testing

Comments The University of Cambridge local Examinations Syndicate (UCLES) has been producing English language tests since 1858 Now with three organizations cooperatshying to form the IELTS more than a million examinations are administered every year In 2002 a computer-based version of the Reading and Writing modules of the IELTS became available at selected centers around the world The other sections are administered locally by an examinet The paper-based IELTS remains an option for candidates The IELTS retains the distinct advantage of requiring all four skills in the test-takers performance

Test of English for International Communication (TOEICreg)

Producer The Chauncey Group International a subsidiary of Educational Testing Service

Objelttive To test overall proficiency (langlJage ability) Primary market Worldwide business commerce and industry contexts

(workplace settings) Type Computer-based and paper-based versions Response modes Multiple-choice responses Time allocation 2 hours Internet access httpwwwtoeiccom

Specifications Listening Comprehension 100 items administered by audiocassette Four types of task statements questions short conversations and short talks (approxishymately 45 minutes) Reading 100 items Three types of task cloze sentences error recogshynition and reading comprehension (75 minutes)

Comments The TOEIC has become a very widely used international test of English proficiency in workplace settings where English is required for job performance The conshytent includes many different employment settings such as conferences presentations sales ordering shipping schedules reservations (etters and memoranda It is approprishyate to use in educational settings where vocational or workplace English courses are being offered

sect sect sect sect sect

The construction of a valid standardized test is no minor accomplishment whether the instrument is large- or small-scale The designing of specifications alone as this chapter illustrates requires a sophisticated process of construct valishydation coupled with considerations of practicality Then the construction of items and scoringinterpretation procedures may require a lengthy period of trial and error with prototypes of the final form of the testWith painstaking attention to all the details of construction the end product can result in a cost-effective timeshysaving accurate instrument Your use of the results of such assessments can provide useful data on learners language abilities But your caution is warranted as well for all the reasons discussed in this chapter The next chapter will elaborate on what lies behind that need for a cautious approach to standardized assessment

CHAPTER4 Standardized Testing 87

EXERCISES

[Note (I) Individual work (G) Group or pair work (C) Whole-class discussion]

1 (C) Tell the class about the worst test experience youve ever had Briefly anamiddot lyze what made the experience so unbearable and try to come up with sugshygestions for improvement of the test andor its administrative conditions

2 (G) In pairs or small groups compile a brief list of pros and cons of standardshyized testing Cite illustrations of as many items in each list as possible Report your lists and examples to the rest o~ the class

3 (I) Select a standardized test that you are quite familiar with (probably a recent experience) Mentally evaluate that test using the five principles of practicality reliability validity authenticity and washback Report yourevaluashytion to the class

4 (G) The appendix to this chapter provides sample items from Jour different tests of language proficiency In groups one test for each group analyze your test for (a) content validity (b) face validity and (c) authenticity

5 (C) Do you think that the sample TOEFL reading passage about pirates (pages 74-75) and the Graduate EssayTest prompt (pages 76-77) about a school board hiring committee have any culture bias Discuss this and other cultural biases you have noticed in tests Is it possible to design a test that is completely free of culture bias

6 (CG) Compare the differences in conceptualization of language proficiency represented by Swains model the TOEFL and the ACfFL philosophy Which one best represents current thinking about communicative language ability What are the strengths and weaknesses of each approach

FORYOlIILEURTHER READING

Gronlund Norman E (1998) Assessment of student achievement Sixth Edition Boston Allyn and Bacon

Gronlunds classic also mentioned in Chapter 3 offers a concise overview of features of standardized tests offering definitions and examples of the statistical considerations in interpreting scores His approach is unbiased cleady written and accessible to those who might fear the mathematics of standardized testing

Phillips Deborah 2001 Long1nan introductory course for the TOEFL test White Plains NY Pearson Education

A careful examination of this or any other reputable preparation course for a standardized language test is well worth a students time Note especially how the book acquaints the user with the specifications of the test and offers a number of useful strategie~ that can be llsed in preparation for the test and during irs adn1inistration

88 CHAPTER 4 Standardized Testing

APPENDIX TO CHAPTER 4

Commercial Proficiency Tests Sample Items and Tasks

Test of English a~ a Foreign Language (TOEFLreg)

Listening r

Part A

In this section you will hear short conversations between two people In some ofthe conversations each person speaks only once In other conversations one or both of the people speak more than once Each conversation is followed by one questionabQlt it Each question in this part has four answer choices You should click on the best answer to each question Answer the questions on the basis of what is stated or implied by the speakers Here is an example On the computerscreen you will see

[man and woman talking]

On the recording you will hear

(woman) Hey wheres your sociology book (man) At home Why carry it around when were just going to be taking

a test (woman) Dont you remember Professor Smith said we could us it during

the test (man) Ohl no Well Ive still got an hour right Im so glad I ran into you

You wiII then see and hear the question before the answer choices appear

What will the man probably do next

o Begin studying for the sociology test o Explain the problem to his professor o Go home to get his textbook o Borrow the womans book

To choose an answer you will click on an oval The oval next to that answer will darken After you click on Next and Confirm Answer the next conversation will be presented

Part B

In this section you will hear several longer conversations and talks Each conversation or talk is followed by several questions The conversations talks and questions will not be repeated The conversations and talks are about a variety of topics You do not need speshycial knowledge of the topics to answer the questions correctly Rather you should answer each question on the basis of what is stated or implied by the speakers in the conversashytions or talks

For most of the questions you will need to click on the best of four possible answers Some questions will have special directions The special directions will appear in a box on the computer screen Here is an exampie ot a conversation and some questions

CHAPTER 4 Standardized Testing 89

Marine Biology (narrator) Listen to part of a discussion in a marine biology class

(professor) A few years ago our local government passed a number of strict environmental laws As a result Sunrise Beach looks nothing Ii ke it did ten years ago The water is cleaner and theres been a tremendous increase in all kinds of marine life which is why were going there on Thursday

(woman) I dont know if I agree that the water quality has improved I mean I was out there last weekend and it looked all brown It didnt seem too clean to me

(professor) Actually the color of the water doesnt always indicate whether its polluted The brown color you mentioned might be a result of pollution or it can mean a kind of brown algae is growing there Its called devils apron and it actually serves as food for whales

(man) So when does the water look blue (professor) Well water thats completely unpolluted is actually colorless But

it often looks bluish-green because the sunlight can penetrate deep down and thats the color thats reflected

(woman) But sometimes it looks really green Whats that about (professor) Ok well its the same principle as with devils apron the

water might be green because of different types of green algae there-gulfweed phytoplankton You all should finish reading about algae and plankton before we go In fact those are the types of living things Im going to ask you to be looking for when were there

Now get ready to answer the questions

What is the discussion mainly about

o The importance of protecting ocean environments o The reasons why ocean water appears to be different colors o The survival of whales in polluted water o The effect that colored ocean water has on algae

To choose an answer click on an oval The oval next to that answer will darken After you click on Next and Confirm Answer the next question will be presented

According to the professor what can make ocean water look browngt

o Pollution o Cloudy Skies o Sand o Algae

Click on 2 answers

To choose your answers you will click on the squares An XII wiii appear in each square

bullbullbullbullbullbullbull

90 CHAPTER 4 Standardized Testing

Structure and Written Expression This section measures the ability to recognize language that is appropriate for standard written English There are two types ofquestions in this section In the first type ofquestion there are incomplete sentences Beneath each sentence there are four words or phrases

Directions CIiSk on the one word or phrase that best completes the sentence

The colum~ine flower __ to nearly all of the United States can be raised from seed in almost any garden

native how native is how native is it is native

Time Help Confirm

After you click on Next and Confirm Answ~ the next question willbe presented

The second type of question has four underlined words or phrases You will choose the one underlined word or phrase that must be changed for the sentence to be correct

Directions Click on the one underlined word or phrase that must be changed for the senshytence to be correct

One of the most difficult problems in understanding sleep is determining what the funcshytions of sleep ~

lime Help Confirm

Clicking on an underlined word or phrase will darken it

Reading This section measures the ability to read and understand short passages similar in topic and style to those that students are likely to encounter in North American universities and colleges This section contains reading passages and questions about the passages There are several different types of questions in this section In the Reading section you will first have the opportunity to read the passage

The temperature of the Sun is over 10000 degrees Fahrenheit at the surface but it rises perhaps more than 270000000 at the center The Sun is so much hotter than the Earth that matter can exist only as a gasl except perhaps at the core In the core of the Sun the pressures are so great that despite the high temperature there may be a small solid core However no one really knows since the center of the Sun can never be directly observed ~ Solar astronomers do know that the Sun is divided into five general layers or zones Starting at the outside and going down into the Sun the zones are the corona chromoshysphere hotosphere convection zone and finally the core The first three zones are reshygarded as the Suns atmosphere But since the Sun has no solid surface it is hard to middottell where the atmosphere ends and the main body of the Sun begins

The Suns outermost layer begins about 10000 miles above the visible surface and goes outward for millions of miles This is the only part of the Sun that can be seen during an eclipse such as the one in February 1979 At any other time the corona can be seen

bullbullbullbullbullbullbull

bull bullbullbullbullbullbull

CHAPTER 4 Standardized Testing 91

only when special instruments are used on cameras and telescopes to block the light from the photosphere

The corona is a brilliant pearly white filmy light about as bright as the full Moon Its beautiful rays are a sensational sight during an eclipse The coronas rays flash out in a brilliant fan that has wispy spikelike rays near the Suns north and south poles The corona is generally thickest at the Suns equator The corona is made up of gases streamshying outward at tremendous speeds that reach a temperature of more than 2 million deshygrees Fahrenheit The gas thins out as it reaches the space around the planets By the time the gas of the corona reaches the Earth it has a relatively low density

When you have finished reading the passage you will use the mouse to click on Proceed Then the questions about the passage will be presented You are to choose the one best anshyswer to each question Answer all questions about the information in a passage on the basis ofwhat is stated or implied in that passage Most ofthe questions will be multiple-choice questions To answer these questions you will click on a choice below the question

With what topic is paragraph 2 mainly concerned

o How the Sun evolved o The structure of the Sun o Why scientists study the Sun o The distaflce of the Sun from the planets

Paragraph 2 is marked with an arrow (~)

You will see the next question after you click on Next

To answer some questions you will click on a word or phrase Here is an example

Look at the word one in the passage Click on the word or phrase in the bold text that one refers to To answer you can click on any part of the word or phrase in the passage Jour choice will darken to show which word you have chosen

The Suns outermost layer begins about 10000 miles above the visible surface and goes outward for millions of miles This is the only part of the Sun that can be seen durshying an eclipse such as the one in February 1979 At any other time the corona can be seen only when special instruments are used on cameras and telescopes to block the Iight from the photosphere

You will see the next question after you click on~ To answer some q~estions you will click on a sentence in the passage Here is an example

~ The corona is a brilliant pearly white filmy light about as bright as the full Moon Its beautiful rays are a sensational sight during an eclipse The coronas rays flash out in a brilliant fan that has wispy spikelike rays near the Suns north and south poles The corona is generally thickest at the Suns equator ~ The corona is made up of gases streaming outward at tremendous speeds that reach a temperature of more than 2 million degrees Fahrenheit The gas thins out as it reaches the space around the planets By the time the gas of the corona reaches the Earth it has a relatively low density

bull bullbullbullbullbullbull

92 CHAPTER 4 Standardized Testing

Click on the sentence in paragraph 4 or 5 in which the author compares the light of the Suns outermost layer to that ofanother astronomical body Paragraphs 4 and 5 are marked with arrows (~)

To answer some questions you will click on a square to add a sentence to the passage Here is an example -The following sentence can be added to paragraph 1

At the center of the Earths solar system lies the Sun

Where would it best fit in paragraph I Click on the square to add the sentence to the paragraph

D The temperature of the Sun is over 10000 degrees Fahrenheit at the surface but it rises to perhaps morethan 27000000deg at the center 0 The Sun is so much hotter than the Earth that matter can exist only as a gasi except p~rHapsatth~ c6relp the c~re of the ii Sun the pressures are so great that despite the high temperature there may be a small solid core D However no one really knows since the center of the Sun can never be directly observed D 0100

When you click on a square the sentence will appear in the passage at the place you have chosen You can read the sentence added to the paragraph to see if this is the best place to add it You can click on another square to change your answer The sentence will be added and shown in a dark box

Writing In this section you will have an opportunity to demonstrate your ability to write in Enshyglish This includes the ability to generate and organize ideas to support those ideas with examples or evidence and to compose in standard written English in response to an asshysigned topic You will have 30 minutes to write your essay on that topic You must write on the topic you are assigned An essay on any other topic will receive a score of 0 Read the topic below and then make any notes that will help you plan your response Begin typing your response in the box at the bottom of the screen or write your answer on the answer sheet provided to you

Following is a sample topic

Do you agree or disagree with the following statemenH

Teachers should make learning enjoyable and fun for their students

Use specific reasons and examples to support your opinion

CHAPTER 4 Standarczed Testing 93

Michigan English Language Assessment Battery (MELAB)

Composition The time limit for the composition is 30 minutes You must write on only one of the top~

ics below If you write about something else your composition paper will not be graded and you cannot be given a final score If you do not understand the topics ask the exam~ iner to explain or to translate them You may be asked to give your opinion ofsomething and explain why you believe this to describe something from your experience or to exshyplain a problem and offer possible solutions You should write at least one page Some sample topics are

1 What do you think is your countrys greatest problem Explain in detail and tell what you think can be done about it

2 What are the characteristics of a good teacher Explain and give examples 3 An optimist is someone who sees the good side of things A pessimist sees the

bad side Are you an optimist or a pessimist Relate a personal experience that shows this

4 In your opinion are the benefits of space exploration really worth the enormous costs Discuss

Most MELAB compositions are one or two pages long (about 200-300 words) If your paper is extremely short (less than 150 words) your composition will be given a lower score Before you begin writing you might want to take 2 or 3 minutes to plan your comshyposition and to make a short outline to organize your thoughts Such outlines will not be graded they are only to help you You should use the last 5 minutes to read through your composition and to make changes or corrections

Your composition will be graded on how clearly you express yourself in English and on the range of English you are able to use and your control in doing so This means your composition should be well organized your arguments should be fully developed and you should show a range ofgrammatical structures and broad vocabulary Compositions that consist only of very short sentences and very simple vocabulary cannot be given the

middothighest scores If errors are not frequent and if they do not confuse your meaning they will not lower your score very much

Listening Now you will hear a short lecture You may take notes during the lecture Following the lecture you will be asked some questions about it

Therell be a two-week exhibit of the paintings of the little-known master Laura Bernhart at the Claire Osmond Galleries starting on the fifteenth of the month and running through the thirtieth Bernharts known for her innovative designs in abstract expressionism Though a true original she declared a spiritual heritage from Salvador Dali the famous Spanish painter Since Bernhart lived a rather solitary life and died while only in her twenties few people are aware of her works This showing at the Osmond Galleries will provide many with an introduction to her works

10 Where is the exhibit a the Art Museum b the Dali Galleries c the Osmond Galleries

94 CHAPTER 4 Standardized Testing

11 What is Bernhart known for a her copies of Dalis paintings b the originality of her designs c her exhibitions

12 What will going to the exhibit allow most people to do a to see Saivador Dalis paintings b to see Bernharts works for the first time c to learn about Spanish art

Grammar

1 What did the teacher just tell you

She reminded our notebooksI a us to bring b that we bring c our bringing d we should bring

2 Is Bill a good dancer

Not really __ he tries very hard a in spite of h despite c even though d while

3 your clothes are all wet1

Yes I didnt come __ the rain soon enough a away to b over to c down with d in from

Cloze In years to come zoos will not only be places where animals are exhibited to the public but repositories where rare species can be saved from extinction (7) captive breeding The most powerful force (8) the future of many animals-and of zoos-is the decline of the wild (9) even zoo directors would argue that (10) are better places for animals than the fields and forest of their native (11) yet zoos may be the last chance for some creatures that would otherwise pass qUietly into oblivion

7 a through c from b of d damage

8 a bringing c to b that d influencing

9 a But c Not b So d Then

10 a where c even b zoos d wilds

11 alands c residence b life d field

CHAPTER 4 Standardized Testing 95

Vocabulary

12 Mark has a flair for writing a need b purpose c talent d dislike

13 Bill Collins launched his restaurant last June a moved b started c sold d bought

14 John will not accept the censure a burden b blame c credit d decision

15 I cant think of the answer Can you give me a __ a hint b token c taste d gaze

16 Because fewer people are taking expensive vacations the tourist industry is in a a choke b grope c grumble d slump

17 I disagree with a few of his opinions but __ we agree a deliberately b conclusively c essentially d immensely

Reading The influenza virus is a single molecule built from many millions of single atoms You must have heard of the viruses which are sometimes called living molecules While bacteria can be considered as a type of plant secreting pOisonous substances into the body of the organism they attack viruses are living organisms themselves We may conshysider them as regular chemical molecules since they have a strictly aefined atomic strucshyture but on the other hand we must also consider them as being alive since they are able to multiply in unlimited quantities

18 According to the passage bacteria are a poisons

b larger than viruses c very small d plants

96 CHAPTER 4 Standardized Testing

19 The writer says that viruses are alive because they a have a complex atomic structure b move c multiply d need warmth and light

20 The atomic structure of viruses a is -tJIariable b is strictly defined c cannot be analyzed chemically d is more complex than that of bacteria

International English Language Testing System (fELTS)

I

listening

The Listening Module has four sections The first two sections are concerned with social needs There is a conversation between two speakers and then a monologue For examshyple a conversation about travel arrangements or decisions on a night out and a speech about student services on a university campus or arrangements for meals during a confershyence The final two sections are concerned with situations related more closely to educashytional or training contexts For example conversation between a tutor and a student about an assignment or between three students planning a research project and a lecture or talk ofgeneral academic interest All the topics are ofgeneral interest and it makes no difference what subjects candidates study Tests and tasks become more difficult as the sections progress A range of English accents and dialects are used in the recording which reflects the international usage of IELTS

Academic Reading [A 7S0-word article on-th-e- topic of Wind Power in the US with a short glossary at the end]

Questions 1-5

Complete the summary below

Choose your answers from the box below the summary and write them in boxes 1-5 on your answer sheet Note There are more words or phrases than you will need to fill the gaps You may use any word or phrase more than once

Example The failure during the late 1970s and early 19805 of an attempt to establish a widespread wind power industry in the United States resulted largely from the (1) bull in oil prices during this period The industry is now experiencing a steady (2) due to improveshyments in technology and an increased awareness of the potential in the power of wind The wind turbines that are now being made based in part on the (3) of wide- ranging research in Europe are easier to manufacture and maintain than their predecesshysors This has led wind-turbine makers to be able to standardise and thus minimize (4) There has been growing (S) of the importance of wind power as an energy source

CHAPTER 4 Standardized Testing 97

criticism stability skepticism success operating costs decisions design costs fall effects production costs growth decline failure recognition results

Questions 6-1 0 Look at the following list of issues (Questions 6-10) and implications (A-C) Match each issue with one implication Write the appropriate letters A-C in boxes 6-10 on your anshyswer sheet

Example The current price of one wind-generated kilowatt Answer

6 The recent installation of systems taking advantage of economies of scale

7 The potential of meeting one fifth of current U5 energy requirements by wind power

8 The level of acceptance of current wind turbine technology

9 A comparison of costs between conventional and wind power sources

10 The view of wind power in the European Union

Implications

A provides evidence against claims that electricity produced from wind power is relatively expensive

B supports claims that wind power js an important source of energy

C opposes the view that wind power technology requires further-development

General Training Reading Read the passage on Daybreak trips by coach and look at the statements below On your answer sheet write

TRUE if the statement is true FALSE jf the statement is false

NOlGIVEN if the information is not given in the leaflet

1 MiIlers Coaches owns Cambridges Cam bus fleet

2 Premier is an older company than Millers

3 Most of the Daybreak coaches are less than 5 years old

4 Daybreak fares are more expensive than most of their competitors

5 Soft drinks and refreshments are served on most longer journeys

6 Smoking is permitted at the rear of the coach on longer journeys

7 Tickets must be bought in advance from an authorised Daybreak agent

6 Tickets and seats can be reserved by phoning the Daybreak Hotline

9 Daybreak passengers must join their coach at Cambridge Drummer Street

10 Daybreak cannot guarantee return times

98 CHAPTER 4 Standardized Testing

FROM CAMBRIDGE AND SURROUNDING AREA

SPRING IS INTHEAIR

Welcome to our Spring Daybreak programme which continues the tradition of offering unbeatable value for money day trips and tours All the excursions in this brochure will be operated by Pr~mier Travel Services Limited or Millers Coaches both companies are part of the CHLGroup owners of Cambridges Cambus fleet

WERE PROUD OF OUR TRADITION

Premier was established in 1936 the Company now offers the highest standards of coaching in todays competitive operating environment Miller has an enviable reputation stretching back over the past 20 years offering coach services at realistic prices Weve traveled a long way since our early days of pre-war seaside trips Now our fleet of 50 modern coaches (few are more than five years old) operate throughout Britain and Europe but were pleased to still maintain the high standards of quality and service the trademark of our founders nearly sixty years ago

EXCLUSIVE FEATURES

Admission-inclusive fares All Daybreak fares (unless specifically otherwise stated) include admission charges to the attractions shows and exhibits we visit Many full-day scenic tours are accompanied by a fully trained English Tourist Board Blue Badge guide or local experienced driverguide Some Daybreaks include lunch or afternoon tea Compare our admission inclusive fares and see how much you save Cheapest is not the best and value for money is guaranteed If you compare our bargain Daybreak fares beware--most of our competishytors do not offer an all-inclusive fare

SEAT RESERVATIONS

We value the freedom of choice so you can choose your seat when you book The seat reservation is guaranteed a-nd remains yours at all times when aboard the coach

NO SMOKING COMFORT

With the comfort of our passengers in mind coaches on all our Daybreaks are no smokshying throughout In the interests of fellow passengers comfort we kindly ask that smokers observe our no smoking policy On scenic tours and longer journeys ample refreshment stops are provided when of course smoking is permitted

YOUR QUESTIONS ANSWERED

Do I need to book Booking in advance is strongly recommended as all Daybreak tours are subject to demand Subject to availability stand-by tickets can be purchased from the driver

What ti me does the coach leave The coach departs from Cambridge Drummer Street (Bay 12 adjacent to public toilets) at the time shown There are many additional joining points indicated by departure codes in the brochure If you are joining at one of our less popular joining points you will be adshyvised of your pick-up time (normally by telephone) not less than 48 hours before deparshyture In this way we can minimize the length of pick-up routes and reduce journey times for the majority of passengers

CHAPTER 4 Standardized Testing 99

What time do we get back An approximate return time is shown for each excursion The tim~s shown serve as a guide but road conditions can sometimes cause delay If your arrival will be later than advertised your driver will try to allow for a telephone call during the return journey

Where can I board the coach All the Daybreaks in the brochure leave from Cambridge Drummer Street (Bay 12 adjashycent to public toilets) at the time shown Many Daybreaks offer additional pick-ups for pre-booked passengers within Cambridge and the surrounding area This facility must be requested at the time of booking

Academic Writing Writing Task 1 You should spend about 20 minutes on this task

The graph below shows the different modes of transport used to travel to and from work in one European city in 1950 1970 and 1990

[graph shown here]

Write a report for a university lecturer describing the information shown below You should write at least 150 words

Writing Task 2 You should spend about 40 minutes on this task

Present a written argument or case to an educated reader with no specialist knowledge of the folowing topic

It is inevitable that as technology develops so traditional cultures must be lost Technolshyogy and tradition are incompatible-you cannot have both together

To what extent do you agree or disagree with this statement Give reasons for your answer You should write at least 250 words You should use your own ideas knowlshyedge and experience and support your arguments with examples and relevant evidence

General Training Writing Writing Task 1 You should spend about 20 minutes on this task You rent a house through an agency The heating system has stopped working You phoned the agency a week ago but it has still not been mended Write a letter to the agency Explain the situation and teil them what you want them to do about it

You should write at least 150 words You do NOT need to write your own address

Begin your letter as follows

Dear - ___-I

Writing Task 2 You should spend about 40 minutes on this task As part ofa class assignment you have to write about the following topic

100 CHAPTER 4 Standardized Testing

Some businesses now say that no one can smoke cigarettes in any of their offices Some governments have banned smoking in all public placesThis is a good idea but it takes away some of our freedom

Do you agree or disagree Give reasons for your answer You should write at least 250 words

Speaking In each ofthe three parts of the speaking module a specific function is fulfilled In Part 1 the candidates answer general questions about themselves their homes or families their jobs or studies their interests and a range ofsimilar familiar topic areas This part lasts between four and five minutes In Part 2 the candidate is given a verbal prompt on a card and is asked to talk on a particular topic The candidate has one minute to prepare before speaking at length for between one and two minutes The examiner then asks one or two wind-down questions In Part 3 the examiner and candidate engage in a discusshysion of more abstract issues and concepts which are thematically linked to the topic prompt in Part 2 The discussion lasts between four and five minutes

All interviews are recorded on audiocassette Here is a sample ofa Part 2 topic

Describe a teacher who has greatly influenced you in your education

You shou Id say

where you met them what subject they taught what was special about them

and explain why this person influenced you so much

You will have to talk about the topic for 1 to 2 minutes You have 1 minute to think about what you are going to say You can make some notes if you wish

Test of English for International Communication (TOEICreg)

listening

Part 1 Photographs Directions For each question you will see a picture in your test book and you will hear four short statements The statements will be spoken just one time They will not be printed in your test book so you must listen carefully to understand what the speaker says When you hear the four statements look at the picture in your test book and choose the statement that best describes what you see in the picture Then on your answer sheet find the number of the question and mark your answer

[photograph of a scientist looking through a microscope]

You will hear Look at the picture marked number 1 in your test book

(A) Shes speaking into a microphone (B) Shes put on her glasses (C) She has both eyes open (D) Shes using a microscope

CHAPTER 4 Standardized Testing 101

Part 2 Question-Response Directions In this part of the test you will hear a question or statement spoken in Enshyglish followed by three responses also spoken in English The question or staten1ent and the responses will be spoken just one time They will not be printed in your test book so you must listen carefully to understand what the speakers say You are to choose the best response to each question or statement

Question 1 You will hear Ms Morikawa has worked here for a long time hasnt she

(A) At three oclock (B) No Ive lost my watch (C) More than ten years

Question 2 You will hear Which of these papers has a wider circulation

(A) The morning edition (B) Get more exercise (C) By messenger

Part 3 Short Conversations Directions In this part of the test you will hear short conversations between two people The conversations will not be printed in your test book You will hear the conversations only once so you must listen carefully to understand what the speakers say In your test book you will read a question about each conversation The question will be followed by four answers You are to choose the best answer to each question and mark it on your answer sheet

Question 1 (Man) We should think about finding another restaurant for lunch (Woman) Why The food and service here are great

(Man) Yes but the prices are going up every week

You will read Why is this man unhappy with the restaurant

(A) It is too noisy (B) It is too expensive (C) It is too crowded (D) It is too difficult to find

Question 2 (Woman A) How was Dr Borgs recent trip to Singapore (Woman B) She enjoyed the tour of the port very much (Woman A) They say its one of the most active in Asia

You will read 2 What did Dr Borg find interesting

(A) The tourist center (B) The airport (C) The musical performance (D) The harbor

Part 4 Short Talks Directions In this part of the test you vill hear several short talks Each will be spoken just one time They will not be printed in your test book so you must listen carefully to understand and remember what is said In your test book you will read two or more questions about each short talk The questions will be followed by four answers You are to choose the best answer to each question and mark it on your answer sheet

102 CHAPTER 4 Standardized Testing

You will hear Questions 1 and 2 refer to the following announcement

Good afternoon and welcome aboard Nordair Flight 857 from Copenhagen to Bangkok with intermediate stops in Dubai and Calcutta We are preparing for departure in a few minutes At this time your seat back should be returned to its full upright position and your seat belt s~ould be fastened OUf anticipated total flying time to Dubai is six hours and twenty-five minutes I hope you enjoy the flight You will hecJr Now read question 1 in your test book and answer it You will read 1 What is the final destination of the flight

(A) Bangkok (B) Copenhagen (C) Dubai (O) Calcutta

You will hear Now read question 2 in your test book and answer it You will read 2 What will happen in a few minutes

(A) The flight will land in Dubai I

(B) The passengers will board the plane (C) The plane will take off (0) The gate number will be announced

Reading In this section of the test you will have the chance to show how well you understand written English There are three parts to this section with special directions for each part

Part 4 Incomplete Sentences Directions This part of the test has incomplete sentences Four words or phrases marked (A) (8) (e) (D) are given beneath each sentence You are to choose the one word or phrase that best completes the sentence Then on your answer sheet find the number of the question and mark your answer

1 Mr Yangs trip will __ him away from the office for ten days (A) withdraw (B) continue (C) retain (0) keep

2 The company that Marie DuBois started now sells __ products throughout the world (A) its (B) it (C) theirs (D) them

3 If your shipment is not delivered __ Tuesday you can request a full refund for the merchandise (A) at (B) by (C) within (D) while

CHAPTER 4 Standardized Testing 103

Part 6 Error Recognition Directions In this part ofthe test each sentence has four words or phrases underlined The four underlined parts of the sentence are marked (A) (B) (C) (D) You are to identify the one underlined word or phrase that should be corrected or rewritten Then on your answer sheet find the number of the question and mark your answer

1 The pamphlet contains some importance information about the current exhibit ABC D

2 No matter how Jong it taking to finish the annual report it must be done properly ABC D

3 The popularity of jogging appears to have decreased since the past couple of years ABC D

Part 7 Reading Comprehension Directions The questions in this part of the test are based on a selection of reading mateshyrials such as notices letters) forms newspaper and magazine articles) and advertisements You are to choose the one best answer (A) (B) (C) or (OJ to each quesshytion Then on your ariswefsheelfindthe number of the qUestion andmcirkyour answer Answer all questions following each reading selection on thebasis of what is stated or implied in that selection

The Museum ofTechnology is a hands-on museum designed for people to experience science at w()rk~ Visitors are encouraged to use test and handle the objects o~ display Special demonstrations are scheduled for the first and second Wednesdays of each month at 1330 Open Tuesday-Friday 1200-1630 Saturday 1000-1730 and Sunday 11 00-1630

1 When during the month can visitors see special demonstrations (A) Every weekend (B) The first two Wednesdays (C) One afternoon a week (D) Every other Wednesday

Questions 2 and 3 refer to the followi ng notice

NOTICE If you are unable to work because of an extended illness or injury that is not workshyrelated you may be entitled to receive weekly benefits from your employer or the firms insurance company To claim benefits you must file a claim form within thirty days of the first day of your disability Before filing the claim you must ask your doctor to fill in the Doctors Statement on the claim form stating the period of disability

3 To whom is this notice addressed (A) Employers (8) Doctors (C) Employees (D) When paying the bill

4 When must the claim form be filed (A) On the first of the month (8) On the thirtieth of the month (C) On the first day ofdisabifity (D) Within 30 days of the start of disability

Page 17: Standardized Testing Chapter 4 Brown

82 CHAPTER 4 Standardized Testing

both content and construct validation along with facing such practical issues as scoring the written sections and a machine scorable multiple-choice answer sheet

The process of ongoing validation will no doubt continue as new forms of the editing section are created and as new prompts and reading passages are created for the writing section Such a validation process should also include consistent checks on placement accuracy and on face validity

(C) At this time there is little or no research to validate the GET itself For its conshy struct validation its administrators rely on a stockpile of research on university-level academic writing tests such as theTWEThe holistic scoring rubric and the topics and administrative conditions of the GET are to some extent patterned after that of the TWE In recent years some criticism of the GEf has come from international test-takers (Hosoya 2001) who posit that the topics and time limits of the GET among other facshytors work to the disadvantage of writers whose native language is not English These validity issues remain to be fully addressed in a comprehensive research study

I I

STANDARDIZED IANGUAGE PROFICIENCY TESTING

Tests of language profiCiency presuppose a comprehensive definition of the specific competencies that comprise overall language ability The specifications for the TOEFL provided an illustration of an operational definition of ability for assessment purposes This is not the only way to conceptualize the concept Swain (1990) offered a multidimensional view of profiCiency assessment by referring to three linshyguistic traits (grammar discourse and sociolinguistics) that can be assessed by means of oral multiple-choice and written responses (see Table 41) Swains conshyception was not meant to be an exhaustive analysis of ability but rather to serve as an operational framework for constructing proficiency assessments

Another defmition and conceptualization of profiCiency is suggested by the ACTFL association mentioned earlier ACfFL takes a holistic and more unitary view of proficiency in describing four levels superior advanced intermediate and noviceWithin each level descriptions of listening speaking reading and writing are provided as guidelines for assessment For example the ACfFL Guidelines describe the superior level of speaking as follows

ACTFL speaking guidelines summary superior-level

Superior-level speakers are characterized by the ability to

bull participate fully and effectively in conversations in formal and informal settings on topics related to practical needs and areas of professional andor scholarly interests

bull provide a structured argument to explain and defend opinions and develop effective hypotheses within extended discourse

bull discuss topics concretely and abstractly bull deal with a linguistically unfamiliar situation bull maintain a high degree of linguistic accuracy bull satisfy the linguistic demands of professional andor scholarly life

CHAPTER4 Standardized Testing 83

The other three ACfFL levels use the same parameters in describing progressively lower proficiencies across all four skills Such taxonomie~ have the advantage of considering a number of functions of linguistic discourse but the disadvantage at the lower levels of overly emphasizing test-takers deficiencies

Table 41 Traits of second language proficiency (Swain 1990 p 403)

Trait Grammar Discourse Sociolinguistic

focus on grammatical focus on textual focus on social accuracy within cohesion and appropriateness of sentences coherence language use

Method

Oral structured interview story telling and argumentationpersuasion

role-play ofspeech acts requests offers complaints

scored for accuracy of verbal morphology prepositions syntax

detailed rating for identification logical sequence and time orientation and global ratings for coherence

scored for ability to distinguish formal and informal register

Multiple-choice

sentence-level select the correct form exercise

paragraph-level select the coherent sentence exercise

speech act-Ievelselect the appropriate utterance exercise

(45 items) (29 items) (28 items)

involving verb morphology prepositionsan-d-uther items

Written composition

narrative and letter of persuasion

narrative and letter of persuasion

formal request letter and informal note

scored for accuracy of verb morphology prepositions syntax

detailed ratings much as for oral discourse and global rating for coherence

scored for the ability to distinguish formal and inforJ1lil1 register

FOUR STANDARDIZED lANGUAGE PROFICIENCY TESTS

We now tum to some of the better-known standardized tests of overall language ability or profiCiency to examine some of the typical formats used in commercially available tests We will not look at standardized tests of other specific skills here but that should not lead you to think by any means that proficiency is the only kind of test in the field that is standardized Three standardized oral production tests the

84 CHAPTER 4 Standardized Testing

Test of Spoken English (fSE) the Oral Proficiency Inventory (OPI) and PbonePassreg are discussed in Chapter 7 and the Test of Written English (WE) is covered in ChapterS

Four commercially produced standardized tests of English language proficiency are described briefly in this section the TOEFL the Michigan English Language Assessment Battery (MELAB) the International English Language Testing System (lELTS) and the Test of English for International Communication (fOEICreg) In an appendix to this chapter are sample items from each section of each test When you turn to that appendix use the following questions to help you evaluate these four tests and their subsections

1 What item types are included 2 How practical and reliable does each subsection of each test appear to be 3 Do the item types and tasks appropriately represent a conceptualizatio~ of

language proficiency (ability) That is can you evaluate their construct validity

4 Do the tasks achieve face validity 5 Are the tasks authentic 6 Is there some washback potential in the tasks

Test of English as a Foreign Language (TOEFL)

Producer Educational Testing Service (ETS) Objective To test overall proficiency (language ability) Primary market Almost exclusively US universities and colleges for admission

purposes Type Computer-based (CB) (and two sections are-computer-adaptive)

A traditional paper-based (PB) version is also available Response modes Multiple-choice responses essay Specifications See the box on pp 72-73 Time allocation Up to 4 hours (CB) 3 hours (PB) Internet access wwwtoeflorg

Comments In the North American context the TOEFL is the most widely used comshymercially available standardized test of proficiency Each year the TOEFL test is adminisshytered to approximately 800000 candidates in more than 200 countries It is highly respected because of the thorough program of ongoing research and development conshyducted by ETS The TOEFLs primary use is to set proficiency standards for international students seeking admission to English-speaking universities More than 4200 academic institutions government agencies scholarship programs and licensingcertification agenshycies in more than 80 countries use TOEFL scores By 2004 the TOEFL will include a secshytion on oral production

CHAPTER 4 Standardi~ed Testing 85

Michigan English Language Assessment Battery (MELAB)

Producer English language Institute University of Michigan Objective To test overall proficiency (language ability) Primary market Mostly US and Canadian language programs and colleges

some worldwide educational settings as well Type Paper-based Response modes Multiple-choice responses essay Time allocation 25 to 35 hours Internet access wwwlsaumicheduelimelabhtm

Specifications The MElAB consists of three sections Part 1 a 3D-minute impromptu essay is written on an assigned topic Part 2 a 25-minute multiple-choice listening comshyprehension test is delivered via tape recorder Part 3 is a 100-item 75-minute multipleshychoice test containing grammar doze reading vocabulary and reading comprehension An oral interview (speaking test) is optional

Comments The Ell at the University of Michigan has been producing the MELAB and its earlier incarnation (Michigan Test of English language Proficiency) since 1961 like the TOEFL it serves a North American audience but is also used internationally While its use is not as widespread as the TOEFL its validity is widely respected Because it is cheaper than the TOEFL and more easily obtained it is popular among language schools and institutes Many institutions and companies accept MElAB scores in lieu ofTOEFL scores

International English Language Testing System (IELTS)

Producer Jointly managed by The University of Cambridge local Examinations Syndicate (UClES) The British Council and lOP Education Australia

Objective To test overall proficiency (language ability) Primary-market Australian British Canadian and New Zealand academic

institutions and professional organizations American academic institutions are increasingly accepting IELTS for admissions purposes

1)rpe Computer-based (for the Reading and Writing sections) papershybased for the listening and Speaking modules

Response modes Multiple-choice responses essay oral production Time allocation 2 hours 45 minutes Internet access httpwwwieltsorgl

httpwwwudesorguk httpwwwbritishcouncilorg

Specifications Reading candidates choose between academic reading or general training reading (60 minutes) Writing the same option academic writing or general training writing (60 minutes) Listening four sections for all candidates (30 minutes) Speaking five sections for all candidates (1015 minutes)

86 CHAPTER 4 Standardized Testing

Comments The University of Cambridge local Examinations Syndicate (UCLES) has been producing English language tests since 1858 Now with three organizations cooperatshying to form the IELTS more than a million examinations are administered every year In 2002 a computer-based version of the Reading and Writing modules of the IELTS became available at selected centers around the world The other sections are administered locally by an examinet The paper-based IELTS remains an option for candidates The IELTS retains the distinct advantage of requiring all four skills in the test-takers performance

Test of English for International Communication (TOEICreg)

Producer The Chauncey Group International a subsidiary of Educational Testing Service

Objelttive To test overall proficiency (langlJage ability) Primary market Worldwide business commerce and industry contexts

(workplace settings) Type Computer-based and paper-based versions Response modes Multiple-choice responses Time allocation 2 hours Internet access httpwwwtoeiccom

Specifications Listening Comprehension 100 items administered by audiocassette Four types of task statements questions short conversations and short talks (approxishymately 45 minutes) Reading 100 items Three types of task cloze sentences error recogshynition and reading comprehension (75 minutes)

Comments The TOEIC has become a very widely used international test of English proficiency in workplace settings where English is required for job performance The conshytent includes many different employment settings such as conferences presentations sales ordering shipping schedules reservations (etters and memoranda It is approprishyate to use in educational settings where vocational or workplace English courses are being offered

sect sect sect sect sect

The construction of a valid standardized test is no minor accomplishment whether the instrument is large- or small-scale The designing of specifications alone as this chapter illustrates requires a sophisticated process of construct valishydation coupled with considerations of practicality Then the construction of items and scoringinterpretation procedures may require a lengthy period of trial and error with prototypes of the final form of the testWith painstaking attention to all the details of construction the end product can result in a cost-effective timeshysaving accurate instrument Your use of the results of such assessments can provide useful data on learners language abilities But your caution is warranted as well for all the reasons discussed in this chapter The next chapter will elaborate on what lies behind that need for a cautious approach to standardized assessment

CHAPTER4 Standardized Testing 87

EXERCISES

[Note (I) Individual work (G) Group or pair work (C) Whole-class discussion]

1 (C) Tell the class about the worst test experience youve ever had Briefly anamiddot lyze what made the experience so unbearable and try to come up with sugshygestions for improvement of the test andor its administrative conditions

2 (G) In pairs or small groups compile a brief list of pros and cons of standardshyized testing Cite illustrations of as many items in each list as possible Report your lists and examples to the rest o~ the class

3 (I) Select a standardized test that you are quite familiar with (probably a recent experience) Mentally evaluate that test using the five principles of practicality reliability validity authenticity and washback Report yourevaluashytion to the class

4 (G) The appendix to this chapter provides sample items from Jour different tests of language proficiency In groups one test for each group analyze your test for (a) content validity (b) face validity and (c) authenticity

5 (C) Do you think that the sample TOEFL reading passage about pirates (pages 74-75) and the Graduate EssayTest prompt (pages 76-77) about a school board hiring committee have any culture bias Discuss this and other cultural biases you have noticed in tests Is it possible to design a test that is completely free of culture bias

6 (CG) Compare the differences in conceptualization of language proficiency represented by Swains model the TOEFL and the ACfFL philosophy Which one best represents current thinking about communicative language ability What are the strengths and weaknesses of each approach

FORYOlIILEURTHER READING

Gronlund Norman E (1998) Assessment of student achievement Sixth Edition Boston Allyn and Bacon

Gronlunds classic also mentioned in Chapter 3 offers a concise overview of features of standardized tests offering definitions and examples of the statistical considerations in interpreting scores His approach is unbiased cleady written and accessible to those who might fear the mathematics of standardized testing

Phillips Deborah 2001 Long1nan introductory course for the TOEFL test White Plains NY Pearson Education

A careful examination of this or any other reputable preparation course for a standardized language test is well worth a students time Note especially how the book acquaints the user with the specifications of the test and offers a number of useful strategie~ that can be llsed in preparation for the test and during irs adn1inistration

88 CHAPTER 4 Standardized Testing

APPENDIX TO CHAPTER 4

Commercial Proficiency Tests Sample Items and Tasks

Test of English a~ a Foreign Language (TOEFLreg)

Listening r

Part A

In this section you will hear short conversations between two people In some ofthe conversations each person speaks only once In other conversations one or both of the people speak more than once Each conversation is followed by one questionabQlt it Each question in this part has four answer choices You should click on the best answer to each question Answer the questions on the basis of what is stated or implied by the speakers Here is an example On the computerscreen you will see

[man and woman talking]

On the recording you will hear

(woman) Hey wheres your sociology book (man) At home Why carry it around when were just going to be taking

a test (woman) Dont you remember Professor Smith said we could us it during

the test (man) Ohl no Well Ive still got an hour right Im so glad I ran into you

You wiII then see and hear the question before the answer choices appear

What will the man probably do next

o Begin studying for the sociology test o Explain the problem to his professor o Go home to get his textbook o Borrow the womans book

To choose an answer you will click on an oval The oval next to that answer will darken After you click on Next and Confirm Answer the next conversation will be presented

Part B

In this section you will hear several longer conversations and talks Each conversation or talk is followed by several questions The conversations talks and questions will not be repeated The conversations and talks are about a variety of topics You do not need speshycial knowledge of the topics to answer the questions correctly Rather you should answer each question on the basis of what is stated or implied by the speakers in the conversashytions or talks

For most of the questions you will need to click on the best of four possible answers Some questions will have special directions The special directions will appear in a box on the computer screen Here is an exampie ot a conversation and some questions

CHAPTER 4 Standardized Testing 89

Marine Biology (narrator) Listen to part of a discussion in a marine biology class

(professor) A few years ago our local government passed a number of strict environmental laws As a result Sunrise Beach looks nothing Ii ke it did ten years ago The water is cleaner and theres been a tremendous increase in all kinds of marine life which is why were going there on Thursday

(woman) I dont know if I agree that the water quality has improved I mean I was out there last weekend and it looked all brown It didnt seem too clean to me

(professor) Actually the color of the water doesnt always indicate whether its polluted The brown color you mentioned might be a result of pollution or it can mean a kind of brown algae is growing there Its called devils apron and it actually serves as food for whales

(man) So when does the water look blue (professor) Well water thats completely unpolluted is actually colorless But

it often looks bluish-green because the sunlight can penetrate deep down and thats the color thats reflected

(woman) But sometimes it looks really green Whats that about (professor) Ok well its the same principle as with devils apron the

water might be green because of different types of green algae there-gulfweed phytoplankton You all should finish reading about algae and plankton before we go In fact those are the types of living things Im going to ask you to be looking for when were there

Now get ready to answer the questions

What is the discussion mainly about

o The importance of protecting ocean environments o The reasons why ocean water appears to be different colors o The survival of whales in polluted water o The effect that colored ocean water has on algae

To choose an answer click on an oval The oval next to that answer will darken After you click on Next and Confirm Answer the next question will be presented

According to the professor what can make ocean water look browngt

o Pollution o Cloudy Skies o Sand o Algae

Click on 2 answers

To choose your answers you will click on the squares An XII wiii appear in each square

bullbullbullbullbullbullbull

90 CHAPTER 4 Standardized Testing

Structure and Written Expression This section measures the ability to recognize language that is appropriate for standard written English There are two types ofquestions in this section In the first type ofquestion there are incomplete sentences Beneath each sentence there are four words or phrases

Directions CIiSk on the one word or phrase that best completes the sentence

The colum~ine flower __ to nearly all of the United States can be raised from seed in almost any garden

native how native is how native is it is native

Time Help Confirm

After you click on Next and Confirm Answ~ the next question willbe presented

The second type of question has four underlined words or phrases You will choose the one underlined word or phrase that must be changed for the sentence to be correct

Directions Click on the one underlined word or phrase that must be changed for the senshytence to be correct

One of the most difficult problems in understanding sleep is determining what the funcshytions of sleep ~

lime Help Confirm

Clicking on an underlined word or phrase will darken it

Reading This section measures the ability to read and understand short passages similar in topic and style to those that students are likely to encounter in North American universities and colleges This section contains reading passages and questions about the passages There are several different types of questions in this section In the Reading section you will first have the opportunity to read the passage

The temperature of the Sun is over 10000 degrees Fahrenheit at the surface but it rises perhaps more than 270000000 at the center The Sun is so much hotter than the Earth that matter can exist only as a gasl except perhaps at the core In the core of the Sun the pressures are so great that despite the high temperature there may be a small solid core However no one really knows since the center of the Sun can never be directly observed ~ Solar astronomers do know that the Sun is divided into five general layers or zones Starting at the outside and going down into the Sun the zones are the corona chromoshysphere hotosphere convection zone and finally the core The first three zones are reshygarded as the Suns atmosphere But since the Sun has no solid surface it is hard to middottell where the atmosphere ends and the main body of the Sun begins

The Suns outermost layer begins about 10000 miles above the visible surface and goes outward for millions of miles This is the only part of the Sun that can be seen during an eclipse such as the one in February 1979 At any other time the corona can be seen

bullbullbullbullbullbullbull

bull bullbullbullbullbullbull

CHAPTER 4 Standardized Testing 91

only when special instruments are used on cameras and telescopes to block the light from the photosphere

The corona is a brilliant pearly white filmy light about as bright as the full Moon Its beautiful rays are a sensational sight during an eclipse The coronas rays flash out in a brilliant fan that has wispy spikelike rays near the Suns north and south poles The corona is generally thickest at the Suns equator The corona is made up of gases streamshying outward at tremendous speeds that reach a temperature of more than 2 million deshygrees Fahrenheit The gas thins out as it reaches the space around the planets By the time the gas of the corona reaches the Earth it has a relatively low density

When you have finished reading the passage you will use the mouse to click on Proceed Then the questions about the passage will be presented You are to choose the one best anshyswer to each question Answer all questions about the information in a passage on the basis ofwhat is stated or implied in that passage Most ofthe questions will be multiple-choice questions To answer these questions you will click on a choice below the question

With what topic is paragraph 2 mainly concerned

o How the Sun evolved o The structure of the Sun o Why scientists study the Sun o The distaflce of the Sun from the planets

Paragraph 2 is marked with an arrow (~)

You will see the next question after you click on Next

To answer some questions you will click on a word or phrase Here is an example

Look at the word one in the passage Click on the word or phrase in the bold text that one refers to To answer you can click on any part of the word or phrase in the passage Jour choice will darken to show which word you have chosen

The Suns outermost layer begins about 10000 miles above the visible surface and goes outward for millions of miles This is the only part of the Sun that can be seen durshying an eclipse such as the one in February 1979 At any other time the corona can be seen only when special instruments are used on cameras and telescopes to block the Iight from the photosphere

You will see the next question after you click on~ To answer some q~estions you will click on a sentence in the passage Here is an example

~ The corona is a brilliant pearly white filmy light about as bright as the full Moon Its beautiful rays are a sensational sight during an eclipse The coronas rays flash out in a brilliant fan that has wispy spikelike rays near the Suns north and south poles The corona is generally thickest at the Suns equator ~ The corona is made up of gases streaming outward at tremendous speeds that reach a temperature of more than 2 million degrees Fahrenheit The gas thins out as it reaches the space around the planets By the time the gas of the corona reaches the Earth it has a relatively low density

bull bullbullbullbullbullbull

92 CHAPTER 4 Standardized Testing

Click on the sentence in paragraph 4 or 5 in which the author compares the light of the Suns outermost layer to that ofanother astronomical body Paragraphs 4 and 5 are marked with arrows (~)

To answer some questions you will click on a square to add a sentence to the passage Here is an example -The following sentence can be added to paragraph 1

At the center of the Earths solar system lies the Sun

Where would it best fit in paragraph I Click on the square to add the sentence to the paragraph

D The temperature of the Sun is over 10000 degrees Fahrenheit at the surface but it rises to perhaps morethan 27000000deg at the center 0 The Sun is so much hotter than the Earth that matter can exist only as a gasi except p~rHapsatth~ c6relp the c~re of the ii Sun the pressures are so great that despite the high temperature there may be a small solid core D However no one really knows since the center of the Sun can never be directly observed D 0100

When you click on a square the sentence will appear in the passage at the place you have chosen You can read the sentence added to the paragraph to see if this is the best place to add it You can click on another square to change your answer The sentence will be added and shown in a dark box

Writing In this section you will have an opportunity to demonstrate your ability to write in Enshyglish This includes the ability to generate and organize ideas to support those ideas with examples or evidence and to compose in standard written English in response to an asshysigned topic You will have 30 minutes to write your essay on that topic You must write on the topic you are assigned An essay on any other topic will receive a score of 0 Read the topic below and then make any notes that will help you plan your response Begin typing your response in the box at the bottom of the screen or write your answer on the answer sheet provided to you

Following is a sample topic

Do you agree or disagree with the following statemenH

Teachers should make learning enjoyable and fun for their students

Use specific reasons and examples to support your opinion

CHAPTER 4 Standarczed Testing 93

Michigan English Language Assessment Battery (MELAB)

Composition The time limit for the composition is 30 minutes You must write on only one of the top~

ics below If you write about something else your composition paper will not be graded and you cannot be given a final score If you do not understand the topics ask the exam~ iner to explain or to translate them You may be asked to give your opinion ofsomething and explain why you believe this to describe something from your experience or to exshyplain a problem and offer possible solutions You should write at least one page Some sample topics are

1 What do you think is your countrys greatest problem Explain in detail and tell what you think can be done about it

2 What are the characteristics of a good teacher Explain and give examples 3 An optimist is someone who sees the good side of things A pessimist sees the

bad side Are you an optimist or a pessimist Relate a personal experience that shows this

4 In your opinion are the benefits of space exploration really worth the enormous costs Discuss

Most MELAB compositions are one or two pages long (about 200-300 words) If your paper is extremely short (less than 150 words) your composition will be given a lower score Before you begin writing you might want to take 2 or 3 minutes to plan your comshyposition and to make a short outline to organize your thoughts Such outlines will not be graded they are only to help you You should use the last 5 minutes to read through your composition and to make changes or corrections

Your composition will be graded on how clearly you express yourself in English and on the range of English you are able to use and your control in doing so This means your composition should be well organized your arguments should be fully developed and you should show a range ofgrammatical structures and broad vocabulary Compositions that consist only of very short sentences and very simple vocabulary cannot be given the

middothighest scores If errors are not frequent and if they do not confuse your meaning they will not lower your score very much

Listening Now you will hear a short lecture You may take notes during the lecture Following the lecture you will be asked some questions about it

Therell be a two-week exhibit of the paintings of the little-known master Laura Bernhart at the Claire Osmond Galleries starting on the fifteenth of the month and running through the thirtieth Bernharts known for her innovative designs in abstract expressionism Though a true original she declared a spiritual heritage from Salvador Dali the famous Spanish painter Since Bernhart lived a rather solitary life and died while only in her twenties few people are aware of her works This showing at the Osmond Galleries will provide many with an introduction to her works

10 Where is the exhibit a the Art Museum b the Dali Galleries c the Osmond Galleries

94 CHAPTER 4 Standardized Testing

11 What is Bernhart known for a her copies of Dalis paintings b the originality of her designs c her exhibitions

12 What will going to the exhibit allow most people to do a to see Saivador Dalis paintings b to see Bernharts works for the first time c to learn about Spanish art

Grammar

1 What did the teacher just tell you

She reminded our notebooksI a us to bring b that we bring c our bringing d we should bring

2 Is Bill a good dancer

Not really __ he tries very hard a in spite of h despite c even though d while

3 your clothes are all wet1

Yes I didnt come __ the rain soon enough a away to b over to c down with d in from

Cloze In years to come zoos will not only be places where animals are exhibited to the public but repositories where rare species can be saved from extinction (7) captive breeding The most powerful force (8) the future of many animals-and of zoos-is the decline of the wild (9) even zoo directors would argue that (10) are better places for animals than the fields and forest of their native (11) yet zoos may be the last chance for some creatures that would otherwise pass qUietly into oblivion

7 a through c from b of d damage

8 a bringing c to b that d influencing

9 a But c Not b So d Then

10 a where c even b zoos d wilds

11 alands c residence b life d field

CHAPTER 4 Standardized Testing 95

Vocabulary

12 Mark has a flair for writing a need b purpose c talent d dislike

13 Bill Collins launched his restaurant last June a moved b started c sold d bought

14 John will not accept the censure a burden b blame c credit d decision

15 I cant think of the answer Can you give me a __ a hint b token c taste d gaze

16 Because fewer people are taking expensive vacations the tourist industry is in a a choke b grope c grumble d slump

17 I disagree with a few of his opinions but __ we agree a deliberately b conclusively c essentially d immensely

Reading The influenza virus is a single molecule built from many millions of single atoms You must have heard of the viruses which are sometimes called living molecules While bacteria can be considered as a type of plant secreting pOisonous substances into the body of the organism they attack viruses are living organisms themselves We may conshysider them as regular chemical molecules since they have a strictly aefined atomic strucshyture but on the other hand we must also consider them as being alive since they are able to multiply in unlimited quantities

18 According to the passage bacteria are a poisons

b larger than viruses c very small d plants

96 CHAPTER 4 Standardized Testing

19 The writer says that viruses are alive because they a have a complex atomic structure b move c multiply d need warmth and light

20 The atomic structure of viruses a is -tJIariable b is strictly defined c cannot be analyzed chemically d is more complex than that of bacteria

International English Language Testing System (fELTS)

I

listening

The Listening Module has four sections The first two sections are concerned with social needs There is a conversation between two speakers and then a monologue For examshyple a conversation about travel arrangements or decisions on a night out and a speech about student services on a university campus or arrangements for meals during a confershyence The final two sections are concerned with situations related more closely to educashytional or training contexts For example conversation between a tutor and a student about an assignment or between three students planning a research project and a lecture or talk ofgeneral academic interest All the topics are ofgeneral interest and it makes no difference what subjects candidates study Tests and tasks become more difficult as the sections progress A range of English accents and dialects are used in the recording which reflects the international usage of IELTS

Academic Reading [A 7S0-word article on-th-e- topic of Wind Power in the US with a short glossary at the end]

Questions 1-5

Complete the summary below

Choose your answers from the box below the summary and write them in boxes 1-5 on your answer sheet Note There are more words or phrases than you will need to fill the gaps You may use any word or phrase more than once

Example The failure during the late 1970s and early 19805 of an attempt to establish a widespread wind power industry in the United States resulted largely from the (1) bull in oil prices during this period The industry is now experiencing a steady (2) due to improveshyments in technology and an increased awareness of the potential in the power of wind The wind turbines that are now being made based in part on the (3) of wide- ranging research in Europe are easier to manufacture and maintain than their predecesshysors This has led wind-turbine makers to be able to standardise and thus minimize (4) There has been growing (S) of the importance of wind power as an energy source

CHAPTER 4 Standardized Testing 97

criticism stability skepticism success operating costs decisions design costs fall effects production costs growth decline failure recognition results

Questions 6-1 0 Look at the following list of issues (Questions 6-10) and implications (A-C) Match each issue with one implication Write the appropriate letters A-C in boxes 6-10 on your anshyswer sheet

Example The current price of one wind-generated kilowatt Answer

6 The recent installation of systems taking advantage of economies of scale

7 The potential of meeting one fifth of current U5 energy requirements by wind power

8 The level of acceptance of current wind turbine technology

9 A comparison of costs between conventional and wind power sources

10 The view of wind power in the European Union

Implications

A provides evidence against claims that electricity produced from wind power is relatively expensive

B supports claims that wind power js an important source of energy

C opposes the view that wind power technology requires further-development

General Training Reading Read the passage on Daybreak trips by coach and look at the statements below On your answer sheet write

TRUE if the statement is true FALSE jf the statement is false

NOlGIVEN if the information is not given in the leaflet

1 MiIlers Coaches owns Cambridges Cam bus fleet

2 Premier is an older company than Millers

3 Most of the Daybreak coaches are less than 5 years old

4 Daybreak fares are more expensive than most of their competitors

5 Soft drinks and refreshments are served on most longer journeys

6 Smoking is permitted at the rear of the coach on longer journeys

7 Tickets must be bought in advance from an authorised Daybreak agent

6 Tickets and seats can be reserved by phoning the Daybreak Hotline

9 Daybreak passengers must join their coach at Cambridge Drummer Street

10 Daybreak cannot guarantee return times

98 CHAPTER 4 Standardized Testing

FROM CAMBRIDGE AND SURROUNDING AREA

SPRING IS INTHEAIR

Welcome to our Spring Daybreak programme which continues the tradition of offering unbeatable value for money day trips and tours All the excursions in this brochure will be operated by Pr~mier Travel Services Limited or Millers Coaches both companies are part of the CHLGroup owners of Cambridges Cambus fleet

WERE PROUD OF OUR TRADITION

Premier was established in 1936 the Company now offers the highest standards of coaching in todays competitive operating environment Miller has an enviable reputation stretching back over the past 20 years offering coach services at realistic prices Weve traveled a long way since our early days of pre-war seaside trips Now our fleet of 50 modern coaches (few are more than five years old) operate throughout Britain and Europe but were pleased to still maintain the high standards of quality and service the trademark of our founders nearly sixty years ago

EXCLUSIVE FEATURES

Admission-inclusive fares All Daybreak fares (unless specifically otherwise stated) include admission charges to the attractions shows and exhibits we visit Many full-day scenic tours are accompanied by a fully trained English Tourist Board Blue Badge guide or local experienced driverguide Some Daybreaks include lunch or afternoon tea Compare our admission inclusive fares and see how much you save Cheapest is not the best and value for money is guaranteed If you compare our bargain Daybreak fares beware--most of our competishytors do not offer an all-inclusive fare

SEAT RESERVATIONS

We value the freedom of choice so you can choose your seat when you book The seat reservation is guaranteed a-nd remains yours at all times when aboard the coach

NO SMOKING COMFORT

With the comfort of our passengers in mind coaches on all our Daybreaks are no smokshying throughout In the interests of fellow passengers comfort we kindly ask that smokers observe our no smoking policy On scenic tours and longer journeys ample refreshment stops are provided when of course smoking is permitted

YOUR QUESTIONS ANSWERED

Do I need to book Booking in advance is strongly recommended as all Daybreak tours are subject to demand Subject to availability stand-by tickets can be purchased from the driver

What ti me does the coach leave The coach departs from Cambridge Drummer Street (Bay 12 adjacent to public toilets) at the time shown There are many additional joining points indicated by departure codes in the brochure If you are joining at one of our less popular joining points you will be adshyvised of your pick-up time (normally by telephone) not less than 48 hours before deparshyture In this way we can minimize the length of pick-up routes and reduce journey times for the majority of passengers

CHAPTER 4 Standardized Testing 99

What time do we get back An approximate return time is shown for each excursion The tim~s shown serve as a guide but road conditions can sometimes cause delay If your arrival will be later than advertised your driver will try to allow for a telephone call during the return journey

Where can I board the coach All the Daybreaks in the brochure leave from Cambridge Drummer Street (Bay 12 adjashycent to public toilets) at the time shown Many Daybreaks offer additional pick-ups for pre-booked passengers within Cambridge and the surrounding area This facility must be requested at the time of booking

Academic Writing Writing Task 1 You should spend about 20 minutes on this task

The graph below shows the different modes of transport used to travel to and from work in one European city in 1950 1970 and 1990

[graph shown here]

Write a report for a university lecturer describing the information shown below You should write at least 150 words

Writing Task 2 You should spend about 40 minutes on this task

Present a written argument or case to an educated reader with no specialist knowledge of the folowing topic

It is inevitable that as technology develops so traditional cultures must be lost Technolshyogy and tradition are incompatible-you cannot have both together

To what extent do you agree or disagree with this statement Give reasons for your answer You should write at least 250 words You should use your own ideas knowlshyedge and experience and support your arguments with examples and relevant evidence

General Training Writing Writing Task 1 You should spend about 20 minutes on this task You rent a house through an agency The heating system has stopped working You phoned the agency a week ago but it has still not been mended Write a letter to the agency Explain the situation and teil them what you want them to do about it

You should write at least 150 words You do NOT need to write your own address

Begin your letter as follows

Dear - ___-I

Writing Task 2 You should spend about 40 minutes on this task As part ofa class assignment you have to write about the following topic

100 CHAPTER 4 Standardized Testing

Some businesses now say that no one can smoke cigarettes in any of their offices Some governments have banned smoking in all public placesThis is a good idea but it takes away some of our freedom

Do you agree or disagree Give reasons for your answer You should write at least 250 words

Speaking In each ofthe three parts of the speaking module a specific function is fulfilled In Part 1 the candidates answer general questions about themselves their homes or families their jobs or studies their interests and a range ofsimilar familiar topic areas This part lasts between four and five minutes In Part 2 the candidate is given a verbal prompt on a card and is asked to talk on a particular topic The candidate has one minute to prepare before speaking at length for between one and two minutes The examiner then asks one or two wind-down questions In Part 3 the examiner and candidate engage in a discusshysion of more abstract issues and concepts which are thematically linked to the topic prompt in Part 2 The discussion lasts between four and five minutes

All interviews are recorded on audiocassette Here is a sample ofa Part 2 topic

Describe a teacher who has greatly influenced you in your education

You shou Id say

where you met them what subject they taught what was special about them

and explain why this person influenced you so much

You will have to talk about the topic for 1 to 2 minutes You have 1 minute to think about what you are going to say You can make some notes if you wish

Test of English for International Communication (TOEICreg)

listening

Part 1 Photographs Directions For each question you will see a picture in your test book and you will hear four short statements The statements will be spoken just one time They will not be printed in your test book so you must listen carefully to understand what the speaker says When you hear the four statements look at the picture in your test book and choose the statement that best describes what you see in the picture Then on your answer sheet find the number of the question and mark your answer

[photograph of a scientist looking through a microscope]

You will hear Look at the picture marked number 1 in your test book

(A) Shes speaking into a microphone (B) Shes put on her glasses (C) She has both eyes open (D) Shes using a microscope

CHAPTER 4 Standardized Testing 101

Part 2 Question-Response Directions In this part of the test you will hear a question or statement spoken in Enshyglish followed by three responses also spoken in English The question or staten1ent and the responses will be spoken just one time They will not be printed in your test book so you must listen carefully to understand what the speakers say You are to choose the best response to each question or statement

Question 1 You will hear Ms Morikawa has worked here for a long time hasnt she

(A) At three oclock (B) No Ive lost my watch (C) More than ten years

Question 2 You will hear Which of these papers has a wider circulation

(A) The morning edition (B) Get more exercise (C) By messenger

Part 3 Short Conversations Directions In this part of the test you will hear short conversations between two people The conversations will not be printed in your test book You will hear the conversations only once so you must listen carefully to understand what the speakers say In your test book you will read a question about each conversation The question will be followed by four answers You are to choose the best answer to each question and mark it on your answer sheet

Question 1 (Man) We should think about finding another restaurant for lunch (Woman) Why The food and service here are great

(Man) Yes but the prices are going up every week

You will read Why is this man unhappy with the restaurant

(A) It is too noisy (B) It is too expensive (C) It is too crowded (D) It is too difficult to find

Question 2 (Woman A) How was Dr Borgs recent trip to Singapore (Woman B) She enjoyed the tour of the port very much (Woman A) They say its one of the most active in Asia

You will read 2 What did Dr Borg find interesting

(A) The tourist center (B) The airport (C) The musical performance (D) The harbor

Part 4 Short Talks Directions In this part of the test you vill hear several short talks Each will be spoken just one time They will not be printed in your test book so you must listen carefully to understand and remember what is said In your test book you will read two or more questions about each short talk The questions will be followed by four answers You are to choose the best answer to each question and mark it on your answer sheet

102 CHAPTER 4 Standardized Testing

You will hear Questions 1 and 2 refer to the following announcement

Good afternoon and welcome aboard Nordair Flight 857 from Copenhagen to Bangkok with intermediate stops in Dubai and Calcutta We are preparing for departure in a few minutes At this time your seat back should be returned to its full upright position and your seat belt s~ould be fastened OUf anticipated total flying time to Dubai is six hours and twenty-five minutes I hope you enjoy the flight You will hecJr Now read question 1 in your test book and answer it You will read 1 What is the final destination of the flight

(A) Bangkok (B) Copenhagen (C) Dubai (O) Calcutta

You will hear Now read question 2 in your test book and answer it You will read 2 What will happen in a few minutes

(A) The flight will land in Dubai I

(B) The passengers will board the plane (C) The plane will take off (0) The gate number will be announced

Reading In this section of the test you will have the chance to show how well you understand written English There are three parts to this section with special directions for each part

Part 4 Incomplete Sentences Directions This part of the test has incomplete sentences Four words or phrases marked (A) (8) (e) (D) are given beneath each sentence You are to choose the one word or phrase that best completes the sentence Then on your answer sheet find the number of the question and mark your answer

1 Mr Yangs trip will __ him away from the office for ten days (A) withdraw (B) continue (C) retain (0) keep

2 The company that Marie DuBois started now sells __ products throughout the world (A) its (B) it (C) theirs (D) them

3 If your shipment is not delivered __ Tuesday you can request a full refund for the merchandise (A) at (B) by (C) within (D) while

CHAPTER 4 Standardized Testing 103

Part 6 Error Recognition Directions In this part ofthe test each sentence has four words or phrases underlined The four underlined parts of the sentence are marked (A) (B) (C) (D) You are to identify the one underlined word or phrase that should be corrected or rewritten Then on your answer sheet find the number of the question and mark your answer

1 The pamphlet contains some importance information about the current exhibit ABC D

2 No matter how Jong it taking to finish the annual report it must be done properly ABC D

3 The popularity of jogging appears to have decreased since the past couple of years ABC D

Part 7 Reading Comprehension Directions The questions in this part of the test are based on a selection of reading mateshyrials such as notices letters) forms newspaper and magazine articles) and advertisements You are to choose the one best answer (A) (B) (C) or (OJ to each quesshytion Then on your ariswefsheelfindthe number of the qUestion andmcirkyour answer Answer all questions following each reading selection on thebasis of what is stated or implied in that selection

The Museum ofTechnology is a hands-on museum designed for people to experience science at w()rk~ Visitors are encouraged to use test and handle the objects o~ display Special demonstrations are scheduled for the first and second Wednesdays of each month at 1330 Open Tuesday-Friday 1200-1630 Saturday 1000-1730 and Sunday 11 00-1630

1 When during the month can visitors see special demonstrations (A) Every weekend (B) The first two Wednesdays (C) One afternoon a week (D) Every other Wednesday

Questions 2 and 3 refer to the followi ng notice

NOTICE If you are unable to work because of an extended illness or injury that is not workshyrelated you may be entitled to receive weekly benefits from your employer or the firms insurance company To claim benefits you must file a claim form within thirty days of the first day of your disability Before filing the claim you must ask your doctor to fill in the Doctors Statement on the claim form stating the period of disability

3 To whom is this notice addressed (A) Employers (8) Doctors (C) Employees (D) When paying the bill

4 When must the claim form be filed (A) On the first of the month (8) On the thirtieth of the month (C) On the first day ofdisabifity (D) Within 30 days of the start of disability

Page 18: Standardized Testing Chapter 4 Brown

CHAPTER4 Standardized Testing 83

The other three ACfFL levels use the same parameters in describing progressively lower proficiencies across all four skills Such taxonomie~ have the advantage of considering a number of functions of linguistic discourse but the disadvantage at the lower levels of overly emphasizing test-takers deficiencies

Table 41 Traits of second language proficiency (Swain 1990 p 403)

Trait Grammar Discourse Sociolinguistic

focus on grammatical focus on textual focus on social accuracy within cohesion and appropriateness of sentences coherence language use

Method

Oral structured interview story telling and argumentationpersuasion

role-play ofspeech acts requests offers complaints

scored for accuracy of verbal morphology prepositions syntax

detailed rating for identification logical sequence and time orientation and global ratings for coherence

scored for ability to distinguish formal and informal register

Multiple-choice

sentence-level select the correct form exercise

paragraph-level select the coherent sentence exercise

speech act-Ievelselect the appropriate utterance exercise

(45 items) (29 items) (28 items)

involving verb morphology prepositionsan-d-uther items

Written composition

narrative and letter of persuasion

narrative and letter of persuasion

formal request letter and informal note

scored for accuracy of verb morphology prepositions syntax

detailed ratings much as for oral discourse and global rating for coherence

scored for the ability to distinguish formal and inforJ1lil1 register

FOUR STANDARDIZED lANGUAGE PROFICIENCY TESTS

We now tum to some of the better-known standardized tests of overall language ability or profiCiency to examine some of the typical formats used in commercially available tests We will not look at standardized tests of other specific skills here but that should not lead you to think by any means that proficiency is the only kind of test in the field that is standardized Three standardized oral production tests the

84 CHAPTER 4 Standardized Testing

Test of Spoken English (fSE) the Oral Proficiency Inventory (OPI) and PbonePassreg are discussed in Chapter 7 and the Test of Written English (WE) is covered in ChapterS

Four commercially produced standardized tests of English language proficiency are described briefly in this section the TOEFL the Michigan English Language Assessment Battery (MELAB) the International English Language Testing System (lELTS) and the Test of English for International Communication (fOEICreg) In an appendix to this chapter are sample items from each section of each test When you turn to that appendix use the following questions to help you evaluate these four tests and their subsections

1 What item types are included 2 How practical and reliable does each subsection of each test appear to be 3 Do the item types and tasks appropriately represent a conceptualizatio~ of

language proficiency (ability) That is can you evaluate their construct validity

4 Do the tasks achieve face validity 5 Are the tasks authentic 6 Is there some washback potential in the tasks

Test of English as a Foreign Language (TOEFL)

Producer Educational Testing Service (ETS) Objective To test overall proficiency (language ability) Primary market Almost exclusively US universities and colleges for admission

purposes Type Computer-based (CB) (and two sections are-computer-adaptive)

A traditional paper-based (PB) version is also available Response modes Multiple-choice responses essay Specifications See the box on pp 72-73 Time allocation Up to 4 hours (CB) 3 hours (PB) Internet access wwwtoeflorg

Comments In the North American context the TOEFL is the most widely used comshymercially available standardized test of proficiency Each year the TOEFL test is adminisshytered to approximately 800000 candidates in more than 200 countries It is highly respected because of the thorough program of ongoing research and development conshyducted by ETS The TOEFLs primary use is to set proficiency standards for international students seeking admission to English-speaking universities More than 4200 academic institutions government agencies scholarship programs and licensingcertification agenshycies in more than 80 countries use TOEFL scores By 2004 the TOEFL will include a secshytion on oral production

CHAPTER 4 Standardi~ed Testing 85

Michigan English Language Assessment Battery (MELAB)

Producer English language Institute University of Michigan Objective To test overall proficiency (language ability) Primary market Mostly US and Canadian language programs and colleges

some worldwide educational settings as well Type Paper-based Response modes Multiple-choice responses essay Time allocation 25 to 35 hours Internet access wwwlsaumicheduelimelabhtm

Specifications The MElAB consists of three sections Part 1 a 3D-minute impromptu essay is written on an assigned topic Part 2 a 25-minute multiple-choice listening comshyprehension test is delivered via tape recorder Part 3 is a 100-item 75-minute multipleshychoice test containing grammar doze reading vocabulary and reading comprehension An oral interview (speaking test) is optional

Comments The Ell at the University of Michigan has been producing the MELAB and its earlier incarnation (Michigan Test of English language Proficiency) since 1961 like the TOEFL it serves a North American audience but is also used internationally While its use is not as widespread as the TOEFL its validity is widely respected Because it is cheaper than the TOEFL and more easily obtained it is popular among language schools and institutes Many institutions and companies accept MElAB scores in lieu ofTOEFL scores

International English Language Testing System (IELTS)

Producer Jointly managed by The University of Cambridge local Examinations Syndicate (UClES) The British Council and lOP Education Australia

Objective To test overall proficiency (language ability) Primary-market Australian British Canadian and New Zealand academic

institutions and professional organizations American academic institutions are increasingly accepting IELTS for admissions purposes

1)rpe Computer-based (for the Reading and Writing sections) papershybased for the listening and Speaking modules

Response modes Multiple-choice responses essay oral production Time allocation 2 hours 45 minutes Internet access httpwwwieltsorgl

httpwwwudesorguk httpwwwbritishcouncilorg

Specifications Reading candidates choose between academic reading or general training reading (60 minutes) Writing the same option academic writing or general training writing (60 minutes) Listening four sections for all candidates (30 minutes) Speaking five sections for all candidates (1015 minutes)

86 CHAPTER 4 Standardized Testing

Comments The University of Cambridge local Examinations Syndicate (UCLES) has been producing English language tests since 1858 Now with three organizations cooperatshying to form the IELTS more than a million examinations are administered every year In 2002 a computer-based version of the Reading and Writing modules of the IELTS became available at selected centers around the world The other sections are administered locally by an examinet The paper-based IELTS remains an option for candidates The IELTS retains the distinct advantage of requiring all four skills in the test-takers performance

Test of English for International Communication (TOEICreg)

Producer The Chauncey Group International a subsidiary of Educational Testing Service

Objelttive To test overall proficiency (langlJage ability) Primary market Worldwide business commerce and industry contexts

(workplace settings) Type Computer-based and paper-based versions Response modes Multiple-choice responses Time allocation 2 hours Internet access httpwwwtoeiccom

Specifications Listening Comprehension 100 items administered by audiocassette Four types of task statements questions short conversations and short talks (approxishymately 45 minutes) Reading 100 items Three types of task cloze sentences error recogshynition and reading comprehension (75 minutes)

Comments The TOEIC has become a very widely used international test of English proficiency in workplace settings where English is required for job performance The conshytent includes many different employment settings such as conferences presentations sales ordering shipping schedules reservations (etters and memoranda It is approprishyate to use in educational settings where vocational or workplace English courses are being offered

sect sect sect sect sect

The construction of a valid standardized test is no minor accomplishment whether the instrument is large- or small-scale The designing of specifications alone as this chapter illustrates requires a sophisticated process of construct valishydation coupled with considerations of practicality Then the construction of items and scoringinterpretation procedures may require a lengthy period of trial and error with prototypes of the final form of the testWith painstaking attention to all the details of construction the end product can result in a cost-effective timeshysaving accurate instrument Your use of the results of such assessments can provide useful data on learners language abilities But your caution is warranted as well for all the reasons discussed in this chapter The next chapter will elaborate on what lies behind that need for a cautious approach to standardized assessment

CHAPTER4 Standardized Testing 87

EXERCISES

[Note (I) Individual work (G) Group or pair work (C) Whole-class discussion]

1 (C) Tell the class about the worst test experience youve ever had Briefly anamiddot lyze what made the experience so unbearable and try to come up with sugshygestions for improvement of the test andor its administrative conditions

2 (G) In pairs or small groups compile a brief list of pros and cons of standardshyized testing Cite illustrations of as many items in each list as possible Report your lists and examples to the rest o~ the class

3 (I) Select a standardized test that you are quite familiar with (probably a recent experience) Mentally evaluate that test using the five principles of practicality reliability validity authenticity and washback Report yourevaluashytion to the class

4 (G) The appendix to this chapter provides sample items from Jour different tests of language proficiency In groups one test for each group analyze your test for (a) content validity (b) face validity and (c) authenticity

5 (C) Do you think that the sample TOEFL reading passage about pirates (pages 74-75) and the Graduate EssayTest prompt (pages 76-77) about a school board hiring committee have any culture bias Discuss this and other cultural biases you have noticed in tests Is it possible to design a test that is completely free of culture bias

6 (CG) Compare the differences in conceptualization of language proficiency represented by Swains model the TOEFL and the ACfFL philosophy Which one best represents current thinking about communicative language ability What are the strengths and weaknesses of each approach

FORYOlIILEURTHER READING

Gronlund Norman E (1998) Assessment of student achievement Sixth Edition Boston Allyn and Bacon

Gronlunds classic also mentioned in Chapter 3 offers a concise overview of features of standardized tests offering definitions and examples of the statistical considerations in interpreting scores His approach is unbiased cleady written and accessible to those who might fear the mathematics of standardized testing

Phillips Deborah 2001 Long1nan introductory course for the TOEFL test White Plains NY Pearson Education

A careful examination of this or any other reputable preparation course for a standardized language test is well worth a students time Note especially how the book acquaints the user with the specifications of the test and offers a number of useful strategie~ that can be llsed in preparation for the test and during irs adn1inistration

88 CHAPTER 4 Standardized Testing

APPENDIX TO CHAPTER 4

Commercial Proficiency Tests Sample Items and Tasks

Test of English a~ a Foreign Language (TOEFLreg)

Listening r

Part A

In this section you will hear short conversations between two people In some ofthe conversations each person speaks only once In other conversations one or both of the people speak more than once Each conversation is followed by one questionabQlt it Each question in this part has four answer choices You should click on the best answer to each question Answer the questions on the basis of what is stated or implied by the speakers Here is an example On the computerscreen you will see

[man and woman talking]

On the recording you will hear

(woman) Hey wheres your sociology book (man) At home Why carry it around when were just going to be taking

a test (woman) Dont you remember Professor Smith said we could us it during

the test (man) Ohl no Well Ive still got an hour right Im so glad I ran into you

You wiII then see and hear the question before the answer choices appear

What will the man probably do next

o Begin studying for the sociology test o Explain the problem to his professor o Go home to get his textbook o Borrow the womans book

To choose an answer you will click on an oval The oval next to that answer will darken After you click on Next and Confirm Answer the next conversation will be presented

Part B

In this section you will hear several longer conversations and talks Each conversation or talk is followed by several questions The conversations talks and questions will not be repeated The conversations and talks are about a variety of topics You do not need speshycial knowledge of the topics to answer the questions correctly Rather you should answer each question on the basis of what is stated or implied by the speakers in the conversashytions or talks

For most of the questions you will need to click on the best of four possible answers Some questions will have special directions The special directions will appear in a box on the computer screen Here is an exampie ot a conversation and some questions

CHAPTER 4 Standardized Testing 89

Marine Biology (narrator) Listen to part of a discussion in a marine biology class

(professor) A few years ago our local government passed a number of strict environmental laws As a result Sunrise Beach looks nothing Ii ke it did ten years ago The water is cleaner and theres been a tremendous increase in all kinds of marine life which is why were going there on Thursday

(woman) I dont know if I agree that the water quality has improved I mean I was out there last weekend and it looked all brown It didnt seem too clean to me

(professor) Actually the color of the water doesnt always indicate whether its polluted The brown color you mentioned might be a result of pollution or it can mean a kind of brown algae is growing there Its called devils apron and it actually serves as food for whales

(man) So when does the water look blue (professor) Well water thats completely unpolluted is actually colorless But

it often looks bluish-green because the sunlight can penetrate deep down and thats the color thats reflected

(woman) But sometimes it looks really green Whats that about (professor) Ok well its the same principle as with devils apron the

water might be green because of different types of green algae there-gulfweed phytoplankton You all should finish reading about algae and plankton before we go In fact those are the types of living things Im going to ask you to be looking for when were there

Now get ready to answer the questions

What is the discussion mainly about

o The importance of protecting ocean environments o The reasons why ocean water appears to be different colors o The survival of whales in polluted water o The effect that colored ocean water has on algae

To choose an answer click on an oval The oval next to that answer will darken After you click on Next and Confirm Answer the next question will be presented

According to the professor what can make ocean water look browngt

o Pollution o Cloudy Skies o Sand o Algae

Click on 2 answers

To choose your answers you will click on the squares An XII wiii appear in each square

bullbullbullbullbullbullbull

90 CHAPTER 4 Standardized Testing

Structure and Written Expression This section measures the ability to recognize language that is appropriate for standard written English There are two types ofquestions in this section In the first type ofquestion there are incomplete sentences Beneath each sentence there are four words or phrases

Directions CIiSk on the one word or phrase that best completes the sentence

The colum~ine flower __ to nearly all of the United States can be raised from seed in almost any garden

native how native is how native is it is native

Time Help Confirm

After you click on Next and Confirm Answ~ the next question willbe presented

The second type of question has four underlined words or phrases You will choose the one underlined word or phrase that must be changed for the sentence to be correct

Directions Click on the one underlined word or phrase that must be changed for the senshytence to be correct

One of the most difficult problems in understanding sleep is determining what the funcshytions of sleep ~

lime Help Confirm

Clicking on an underlined word or phrase will darken it

Reading This section measures the ability to read and understand short passages similar in topic and style to those that students are likely to encounter in North American universities and colleges This section contains reading passages and questions about the passages There are several different types of questions in this section In the Reading section you will first have the opportunity to read the passage

The temperature of the Sun is over 10000 degrees Fahrenheit at the surface but it rises perhaps more than 270000000 at the center The Sun is so much hotter than the Earth that matter can exist only as a gasl except perhaps at the core In the core of the Sun the pressures are so great that despite the high temperature there may be a small solid core However no one really knows since the center of the Sun can never be directly observed ~ Solar astronomers do know that the Sun is divided into five general layers or zones Starting at the outside and going down into the Sun the zones are the corona chromoshysphere hotosphere convection zone and finally the core The first three zones are reshygarded as the Suns atmosphere But since the Sun has no solid surface it is hard to middottell where the atmosphere ends and the main body of the Sun begins

The Suns outermost layer begins about 10000 miles above the visible surface and goes outward for millions of miles This is the only part of the Sun that can be seen during an eclipse such as the one in February 1979 At any other time the corona can be seen

bullbullbullbullbullbullbull

bull bullbullbullbullbullbull

CHAPTER 4 Standardized Testing 91

only when special instruments are used on cameras and telescopes to block the light from the photosphere

The corona is a brilliant pearly white filmy light about as bright as the full Moon Its beautiful rays are a sensational sight during an eclipse The coronas rays flash out in a brilliant fan that has wispy spikelike rays near the Suns north and south poles The corona is generally thickest at the Suns equator The corona is made up of gases streamshying outward at tremendous speeds that reach a temperature of more than 2 million deshygrees Fahrenheit The gas thins out as it reaches the space around the planets By the time the gas of the corona reaches the Earth it has a relatively low density

When you have finished reading the passage you will use the mouse to click on Proceed Then the questions about the passage will be presented You are to choose the one best anshyswer to each question Answer all questions about the information in a passage on the basis ofwhat is stated or implied in that passage Most ofthe questions will be multiple-choice questions To answer these questions you will click on a choice below the question

With what topic is paragraph 2 mainly concerned

o How the Sun evolved o The structure of the Sun o Why scientists study the Sun o The distaflce of the Sun from the planets

Paragraph 2 is marked with an arrow (~)

You will see the next question after you click on Next

To answer some questions you will click on a word or phrase Here is an example

Look at the word one in the passage Click on the word or phrase in the bold text that one refers to To answer you can click on any part of the word or phrase in the passage Jour choice will darken to show which word you have chosen

The Suns outermost layer begins about 10000 miles above the visible surface and goes outward for millions of miles This is the only part of the Sun that can be seen durshying an eclipse such as the one in February 1979 At any other time the corona can be seen only when special instruments are used on cameras and telescopes to block the Iight from the photosphere

You will see the next question after you click on~ To answer some q~estions you will click on a sentence in the passage Here is an example

~ The corona is a brilliant pearly white filmy light about as bright as the full Moon Its beautiful rays are a sensational sight during an eclipse The coronas rays flash out in a brilliant fan that has wispy spikelike rays near the Suns north and south poles The corona is generally thickest at the Suns equator ~ The corona is made up of gases streaming outward at tremendous speeds that reach a temperature of more than 2 million degrees Fahrenheit The gas thins out as it reaches the space around the planets By the time the gas of the corona reaches the Earth it has a relatively low density

bull bullbullbullbullbullbull

92 CHAPTER 4 Standardized Testing

Click on the sentence in paragraph 4 or 5 in which the author compares the light of the Suns outermost layer to that ofanother astronomical body Paragraphs 4 and 5 are marked with arrows (~)

To answer some questions you will click on a square to add a sentence to the passage Here is an example -The following sentence can be added to paragraph 1

At the center of the Earths solar system lies the Sun

Where would it best fit in paragraph I Click on the square to add the sentence to the paragraph

D The temperature of the Sun is over 10000 degrees Fahrenheit at the surface but it rises to perhaps morethan 27000000deg at the center 0 The Sun is so much hotter than the Earth that matter can exist only as a gasi except p~rHapsatth~ c6relp the c~re of the ii Sun the pressures are so great that despite the high temperature there may be a small solid core D However no one really knows since the center of the Sun can never be directly observed D 0100

When you click on a square the sentence will appear in the passage at the place you have chosen You can read the sentence added to the paragraph to see if this is the best place to add it You can click on another square to change your answer The sentence will be added and shown in a dark box

Writing In this section you will have an opportunity to demonstrate your ability to write in Enshyglish This includes the ability to generate and organize ideas to support those ideas with examples or evidence and to compose in standard written English in response to an asshysigned topic You will have 30 minutes to write your essay on that topic You must write on the topic you are assigned An essay on any other topic will receive a score of 0 Read the topic below and then make any notes that will help you plan your response Begin typing your response in the box at the bottom of the screen or write your answer on the answer sheet provided to you

Following is a sample topic

Do you agree or disagree with the following statemenH

Teachers should make learning enjoyable and fun for their students

Use specific reasons and examples to support your opinion

CHAPTER 4 Standarczed Testing 93

Michigan English Language Assessment Battery (MELAB)

Composition The time limit for the composition is 30 minutes You must write on only one of the top~

ics below If you write about something else your composition paper will not be graded and you cannot be given a final score If you do not understand the topics ask the exam~ iner to explain or to translate them You may be asked to give your opinion ofsomething and explain why you believe this to describe something from your experience or to exshyplain a problem and offer possible solutions You should write at least one page Some sample topics are

1 What do you think is your countrys greatest problem Explain in detail and tell what you think can be done about it

2 What are the characteristics of a good teacher Explain and give examples 3 An optimist is someone who sees the good side of things A pessimist sees the

bad side Are you an optimist or a pessimist Relate a personal experience that shows this

4 In your opinion are the benefits of space exploration really worth the enormous costs Discuss

Most MELAB compositions are one or two pages long (about 200-300 words) If your paper is extremely short (less than 150 words) your composition will be given a lower score Before you begin writing you might want to take 2 or 3 minutes to plan your comshyposition and to make a short outline to organize your thoughts Such outlines will not be graded they are only to help you You should use the last 5 minutes to read through your composition and to make changes or corrections

Your composition will be graded on how clearly you express yourself in English and on the range of English you are able to use and your control in doing so This means your composition should be well organized your arguments should be fully developed and you should show a range ofgrammatical structures and broad vocabulary Compositions that consist only of very short sentences and very simple vocabulary cannot be given the

middothighest scores If errors are not frequent and if they do not confuse your meaning they will not lower your score very much

Listening Now you will hear a short lecture You may take notes during the lecture Following the lecture you will be asked some questions about it

Therell be a two-week exhibit of the paintings of the little-known master Laura Bernhart at the Claire Osmond Galleries starting on the fifteenth of the month and running through the thirtieth Bernharts known for her innovative designs in abstract expressionism Though a true original she declared a spiritual heritage from Salvador Dali the famous Spanish painter Since Bernhart lived a rather solitary life and died while only in her twenties few people are aware of her works This showing at the Osmond Galleries will provide many with an introduction to her works

10 Where is the exhibit a the Art Museum b the Dali Galleries c the Osmond Galleries

94 CHAPTER 4 Standardized Testing

11 What is Bernhart known for a her copies of Dalis paintings b the originality of her designs c her exhibitions

12 What will going to the exhibit allow most people to do a to see Saivador Dalis paintings b to see Bernharts works for the first time c to learn about Spanish art

Grammar

1 What did the teacher just tell you

She reminded our notebooksI a us to bring b that we bring c our bringing d we should bring

2 Is Bill a good dancer

Not really __ he tries very hard a in spite of h despite c even though d while

3 your clothes are all wet1

Yes I didnt come __ the rain soon enough a away to b over to c down with d in from

Cloze In years to come zoos will not only be places where animals are exhibited to the public but repositories where rare species can be saved from extinction (7) captive breeding The most powerful force (8) the future of many animals-and of zoos-is the decline of the wild (9) even zoo directors would argue that (10) are better places for animals than the fields and forest of their native (11) yet zoos may be the last chance for some creatures that would otherwise pass qUietly into oblivion

7 a through c from b of d damage

8 a bringing c to b that d influencing

9 a But c Not b So d Then

10 a where c even b zoos d wilds

11 alands c residence b life d field

CHAPTER 4 Standardized Testing 95

Vocabulary

12 Mark has a flair for writing a need b purpose c talent d dislike

13 Bill Collins launched his restaurant last June a moved b started c sold d bought

14 John will not accept the censure a burden b blame c credit d decision

15 I cant think of the answer Can you give me a __ a hint b token c taste d gaze

16 Because fewer people are taking expensive vacations the tourist industry is in a a choke b grope c grumble d slump

17 I disagree with a few of his opinions but __ we agree a deliberately b conclusively c essentially d immensely

Reading The influenza virus is a single molecule built from many millions of single atoms You must have heard of the viruses which are sometimes called living molecules While bacteria can be considered as a type of plant secreting pOisonous substances into the body of the organism they attack viruses are living organisms themselves We may conshysider them as regular chemical molecules since they have a strictly aefined atomic strucshyture but on the other hand we must also consider them as being alive since they are able to multiply in unlimited quantities

18 According to the passage bacteria are a poisons

b larger than viruses c very small d plants

96 CHAPTER 4 Standardized Testing

19 The writer says that viruses are alive because they a have a complex atomic structure b move c multiply d need warmth and light

20 The atomic structure of viruses a is -tJIariable b is strictly defined c cannot be analyzed chemically d is more complex than that of bacteria

International English Language Testing System (fELTS)

I

listening

The Listening Module has four sections The first two sections are concerned with social needs There is a conversation between two speakers and then a monologue For examshyple a conversation about travel arrangements or decisions on a night out and a speech about student services on a university campus or arrangements for meals during a confershyence The final two sections are concerned with situations related more closely to educashytional or training contexts For example conversation between a tutor and a student about an assignment or between three students planning a research project and a lecture or talk ofgeneral academic interest All the topics are ofgeneral interest and it makes no difference what subjects candidates study Tests and tasks become more difficult as the sections progress A range of English accents and dialects are used in the recording which reflects the international usage of IELTS

Academic Reading [A 7S0-word article on-th-e- topic of Wind Power in the US with a short glossary at the end]

Questions 1-5

Complete the summary below

Choose your answers from the box below the summary and write them in boxes 1-5 on your answer sheet Note There are more words or phrases than you will need to fill the gaps You may use any word or phrase more than once

Example The failure during the late 1970s and early 19805 of an attempt to establish a widespread wind power industry in the United States resulted largely from the (1) bull in oil prices during this period The industry is now experiencing a steady (2) due to improveshyments in technology and an increased awareness of the potential in the power of wind The wind turbines that are now being made based in part on the (3) of wide- ranging research in Europe are easier to manufacture and maintain than their predecesshysors This has led wind-turbine makers to be able to standardise and thus minimize (4) There has been growing (S) of the importance of wind power as an energy source

CHAPTER 4 Standardized Testing 97

criticism stability skepticism success operating costs decisions design costs fall effects production costs growth decline failure recognition results

Questions 6-1 0 Look at the following list of issues (Questions 6-10) and implications (A-C) Match each issue with one implication Write the appropriate letters A-C in boxes 6-10 on your anshyswer sheet

Example The current price of one wind-generated kilowatt Answer

6 The recent installation of systems taking advantage of economies of scale

7 The potential of meeting one fifth of current U5 energy requirements by wind power

8 The level of acceptance of current wind turbine technology

9 A comparison of costs between conventional and wind power sources

10 The view of wind power in the European Union

Implications

A provides evidence against claims that electricity produced from wind power is relatively expensive

B supports claims that wind power js an important source of energy

C opposes the view that wind power technology requires further-development

General Training Reading Read the passage on Daybreak trips by coach and look at the statements below On your answer sheet write

TRUE if the statement is true FALSE jf the statement is false

NOlGIVEN if the information is not given in the leaflet

1 MiIlers Coaches owns Cambridges Cam bus fleet

2 Premier is an older company than Millers

3 Most of the Daybreak coaches are less than 5 years old

4 Daybreak fares are more expensive than most of their competitors

5 Soft drinks and refreshments are served on most longer journeys

6 Smoking is permitted at the rear of the coach on longer journeys

7 Tickets must be bought in advance from an authorised Daybreak agent

6 Tickets and seats can be reserved by phoning the Daybreak Hotline

9 Daybreak passengers must join their coach at Cambridge Drummer Street

10 Daybreak cannot guarantee return times

98 CHAPTER 4 Standardized Testing

FROM CAMBRIDGE AND SURROUNDING AREA

SPRING IS INTHEAIR

Welcome to our Spring Daybreak programme which continues the tradition of offering unbeatable value for money day trips and tours All the excursions in this brochure will be operated by Pr~mier Travel Services Limited or Millers Coaches both companies are part of the CHLGroup owners of Cambridges Cambus fleet

WERE PROUD OF OUR TRADITION

Premier was established in 1936 the Company now offers the highest standards of coaching in todays competitive operating environment Miller has an enviable reputation stretching back over the past 20 years offering coach services at realistic prices Weve traveled a long way since our early days of pre-war seaside trips Now our fleet of 50 modern coaches (few are more than five years old) operate throughout Britain and Europe but were pleased to still maintain the high standards of quality and service the trademark of our founders nearly sixty years ago

EXCLUSIVE FEATURES

Admission-inclusive fares All Daybreak fares (unless specifically otherwise stated) include admission charges to the attractions shows and exhibits we visit Many full-day scenic tours are accompanied by a fully trained English Tourist Board Blue Badge guide or local experienced driverguide Some Daybreaks include lunch or afternoon tea Compare our admission inclusive fares and see how much you save Cheapest is not the best and value for money is guaranteed If you compare our bargain Daybreak fares beware--most of our competishytors do not offer an all-inclusive fare

SEAT RESERVATIONS

We value the freedom of choice so you can choose your seat when you book The seat reservation is guaranteed a-nd remains yours at all times when aboard the coach

NO SMOKING COMFORT

With the comfort of our passengers in mind coaches on all our Daybreaks are no smokshying throughout In the interests of fellow passengers comfort we kindly ask that smokers observe our no smoking policy On scenic tours and longer journeys ample refreshment stops are provided when of course smoking is permitted

YOUR QUESTIONS ANSWERED

Do I need to book Booking in advance is strongly recommended as all Daybreak tours are subject to demand Subject to availability stand-by tickets can be purchased from the driver

What ti me does the coach leave The coach departs from Cambridge Drummer Street (Bay 12 adjacent to public toilets) at the time shown There are many additional joining points indicated by departure codes in the brochure If you are joining at one of our less popular joining points you will be adshyvised of your pick-up time (normally by telephone) not less than 48 hours before deparshyture In this way we can minimize the length of pick-up routes and reduce journey times for the majority of passengers

CHAPTER 4 Standardized Testing 99

What time do we get back An approximate return time is shown for each excursion The tim~s shown serve as a guide but road conditions can sometimes cause delay If your arrival will be later than advertised your driver will try to allow for a telephone call during the return journey

Where can I board the coach All the Daybreaks in the brochure leave from Cambridge Drummer Street (Bay 12 adjashycent to public toilets) at the time shown Many Daybreaks offer additional pick-ups for pre-booked passengers within Cambridge and the surrounding area This facility must be requested at the time of booking

Academic Writing Writing Task 1 You should spend about 20 minutes on this task

The graph below shows the different modes of transport used to travel to and from work in one European city in 1950 1970 and 1990

[graph shown here]

Write a report for a university lecturer describing the information shown below You should write at least 150 words

Writing Task 2 You should spend about 40 minutes on this task

Present a written argument or case to an educated reader with no specialist knowledge of the folowing topic

It is inevitable that as technology develops so traditional cultures must be lost Technolshyogy and tradition are incompatible-you cannot have both together

To what extent do you agree or disagree with this statement Give reasons for your answer You should write at least 250 words You should use your own ideas knowlshyedge and experience and support your arguments with examples and relevant evidence

General Training Writing Writing Task 1 You should spend about 20 minutes on this task You rent a house through an agency The heating system has stopped working You phoned the agency a week ago but it has still not been mended Write a letter to the agency Explain the situation and teil them what you want them to do about it

You should write at least 150 words You do NOT need to write your own address

Begin your letter as follows

Dear - ___-I

Writing Task 2 You should spend about 40 minutes on this task As part ofa class assignment you have to write about the following topic

100 CHAPTER 4 Standardized Testing

Some businesses now say that no one can smoke cigarettes in any of their offices Some governments have banned smoking in all public placesThis is a good idea but it takes away some of our freedom

Do you agree or disagree Give reasons for your answer You should write at least 250 words

Speaking In each ofthe three parts of the speaking module a specific function is fulfilled In Part 1 the candidates answer general questions about themselves their homes or families their jobs or studies their interests and a range ofsimilar familiar topic areas This part lasts between four and five minutes In Part 2 the candidate is given a verbal prompt on a card and is asked to talk on a particular topic The candidate has one minute to prepare before speaking at length for between one and two minutes The examiner then asks one or two wind-down questions In Part 3 the examiner and candidate engage in a discusshysion of more abstract issues and concepts which are thematically linked to the topic prompt in Part 2 The discussion lasts between four and five minutes

All interviews are recorded on audiocassette Here is a sample ofa Part 2 topic

Describe a teacher who has greatly influenced you in your education

You shou Id say

where you met them what subject they taught what was special about them

and explain why this person influenced you so much

You will have to talk about the topic for 1 to 2 minutes You have 1 minute to think about what you are going to say You can make some notes if you wish

Test of English for International Communication (TOEICreg)

listening

Part 1 Photographs Directions For each question you will see a picture in your test book and you will hear four short statements The statements will be spoken just one time They will not be printed in your test book so you must listen carefully to understand what the speaker says When you hear the four statements look at the picture in your test book and choose the statement that best describes what you see in the picture Then on your answer sheet find the number of the question and mark your answer

[photograph of a scientist looking through a microscope]

You will hear Look at the picture marked number 1 in your test book

(A) Shes speaking into a microphone (B) Shes put on her glasses (C) She has both eyes open (D) Shes using a microscope

CHAPTER 4 Standardized Testing 101

Part 2 Question-Response Directions In this part of the test you will hear a question or statement spoken in Enshyglish followed by three responses also spoken in English The question or staten1ent and the responses will be spoken just one time They will not be printed in your test book so you must listen carefully to understand what the speakers say You are to choose the best response to each question or statement

Question 1 You will hear Ms Morikawa has worked here for a long time hasnt she

(A) At three oclock (B) No Ive lost my watch (C) More than ten years

Question 2 You will hear Which of these papers has a wider circulation

(A) The morning edition (B) Get more exercise (C) By messenger

Part 3 Short Conversations Directions In this part of the test you will hear short conversations between two people The conversations will not be printed in your test book You will hear the conversations only once so you must listen carefully to understand what the speakers say In your test book you will read a question about each conversation The question will be followed by four answers You are to choose the best answer to each question and mark it on your answer sheet

Question 1 (Man) We should think about finding another restaurant for lunch (Woman) Why The food and service here are great

(Man) Yes but the prices are going up every week

You will read Why is this man unhappy with the restaurant

(A) It is too noisy (B) It is too expensive (C) It is too crowded (D) It is too difficult to find

Question 2 (Woman A) How was Dr Borgs recent trip to Singapore (Woman B) She enjoyed the tour of the port very much (Woman A) They say its one of the most active in Asia

You will read 2 What did Dr Borg find interesting

(A) The tourist center (B) The airport (C) The musical performance (D) The harbor

Part 4 Short Talks Directions In this part of the test you vill hear several short talks Each will be spoken just one time They will not be printed in your test book so you must listen carefully to understand and remember what is said In your test book you will read two or more questions about each short talk The questions will be followed by four answers You are to choose the best answer to each question and mark it on your answer sheet

102 CHAPTER 4 Standardized Testing

You will hear Questions 1 and 2 refer to the following announcement

Good afternoon and welcome aboard Nordair Flight 857 from Copenhagen to Bangkok with intermediate stops in Dubai and Calcutta We are preparing for departure in a few minutes At this time your seat back should be returned to its full upright position and your seat belt s~ould be fastened OUf anticipated total flying time to Dubai is six hours and twenty-five minutes I hope you enjoy the flight You will hecJr Now read question 1 in your test book and answer it You will read 1 What is the final destination of the flight

(A) Bangkok (B) Copenhagen (C) Dubai (O) Calcutta

You will hear Now read question 2 in your test book and answer it You will read 2 What will happen in a few minutes

(A) The flight will land in Dubai I

(B) The passengers will board the plane (C) The plane will take off (0) The gate number will be announced

Reading In this section of the test you will have the chance to show how well you understand written English There are three parts to this section with special directions for each part

Part 4 Incomplete Sentences Directions This part of the test has incomplete sentences Four words or phrases marked (A) (8) (e) (D) are given beneath each sentence You are to choose the one word or phrase that best completes the sentence Then on your answer sheet find the number of the question and mark your answer

1 Mr Yangs trip will __ him away from the office for ten days (A) withdraw (B) continue (C) retain (0) keep

2 The company that Marie DuBois started now sells __ products throughout the world (A) its (B) it (C) theirs (D) them

3 If your shipment is not delivered __ Tuesday you can request a full refund for the merchandise (A) at (B) by (C) within (D) while

CHAPTER 4 Standardized Testing 103

Part 6 Error Recognition Directions In this part ofthe test each sentence has four words or phrases underlined The four underlined parts of the sentence are marked (A) (B) (C) (D) You are to identify the one underlined word or phrase that should be corrected or rewritten Then on your answer sheet find the number of the question and mark your answer

1 The pamphlet contains some importance information about the current exhibit ABC D

2 No matter how Jong it taking to finish the annual report it must be done properly ABC D

3 The popularity of jogging appears to have decreased since the past couple of years ABC D

Part 7 Reading Comprehension Directions The questions in this part of the test are based on a selection of reading mateshyrials such as notices letters) forms newspaper and magazine articles) and advertisements You are to choose the one best answer (A) (B) (C) or (OJ to each quesshytion Then on your ariswefsheelfindthe number of the qUestion andmcirkyour answer Answer all questions following each reading selection on thebasis of what is stated or implied in that selection

The Museum ofTechnology is a hands-on museum designed for people to experience science at w()rk~ Visitors are encouraged to use test and handle the objects o~ display Special demonstrations are scheduled for the first and second Wednesdays of each month at 1330 Open Tuesday-Friday 1200-1630 Saturday 1000-1730 and Sunday 11 00-1630

1 When during the month can visitors see special demonstrations (A) Every weekend (B) The first two Wednesdays (C) One afternoon a week (D) Every other Wednesday

Questions 2 and 3 refer to the followi ng notice

NOTICE If you are unable to work because of an extended illness or injury that is not workshyrelated you may be entitled to receive weekly benefits from your employer or the firms insurance company To claim benefits you must file a claim form within thirty days of the first day of your disability Before filing the claim you must ask your doctor to fill in the Doctors Statement on the claim form stating the period of disability

3 To whom is this notice addressed (A) Employers (8) Doctors (C) Employees (D) When paying the bill

4 When must the claim form be filed (A) On the first of the month (8) On the thirtieth of the month (C) On the first day ofdisabifity (D) Within 30 days of the start of disability

Page 19: Standardized Testing Chapter 4 Brown

84 CHAPTER 4 Standardized Testing

Test of Spoken English (fSE) the Oral Proficiency Inventory (OPI) and PbonePassreg are discussed in Chapter 7 and the Test of Written English (WE) is covered in ChapterS

Four commercially produced standardized tests of English language proficiency are described briefly in this section the TOEFL the Michigan English Language Assessment Battery (MELAB) the International English Language Testing System (lELTS) and the Test of English for International Communication (fOEICreg) In an appendix to this chapter are sample items from each section of each test When you turn to that appendix use the following questions to help you evaluate these four tests and their subsections

1 What item types are included 2 How practical and reliable does each subsection of each test appear to be 3 Do the item types and tasks appropriately represent a conceptualizatio~ of

language proficiency (ability) That is can you evaluate their construct validity

4 Do the tasks achieve face validity 5 Are the tasks authentic 6 Is there some washback potential in the tasks

Test of English as a Foreign Language (TOEFL)

Producer Educational Testing Service (ETS) Objective To test overall proficiency (language ability) Primary market Almost exclusively US universities and colleges for admission

purposes Type Computer-based (CB) (and two sections are-computer-adaptive)

A traditional paper-based (PB) version is also available Response modes Multiple-choice responses essay Specifications See the box on pp 72-73 Time allocation Up to 4 hours (CB) 3 hours (PB) Internet access wwwtoeflorg

Comments In the North American context the TOEFL is the most widely used comshymercially available standardized test of proficiency Each year the TOEFL test is adminisshytered to approximately 800000 candidates in more than 200 countries It is highly respected because of the thorough program of ongoing research and development conshyducted by ETS The TOEFLs primary use is to set proficiency standards for international students seeking admission to English-speaking universities More than 4200 academic institutions government agencies scholarship programs and licensingcertification agenshycies in more than 80 countries use TOEFL scores By 2004 the TOEFL will include a secshytion on oral production

CHAPTER 4 Standardi~ed Testing 85

Michigan English Language Assessment Battery (MELAB)

Producer English language Institute University of Michigan Objective To test overall proficiency (language ability) Primary market Mostly US and Canadian language programs and colleges

some worldwide educational settings as well Type Paper-based Response modes Multiple-choice responses essay Time allocation 25 to 35 hours Internet access wwwlsaumicheduelimelabhtm

Specifications The MElAB consists of three sections Part 1 a 3D-minute impromptu essay is written on an assigned topic Part 2 a 25-minute multiple-choice listening comshyprehension test is delivered via tape recorder Part 3 is a 100-item 75-minute multipleshychoice test containing grammar doze reading vocabulary and reading comprehension An oral interview (speaking test) is optional

Comments The Ell at the University of Michigan has been producing the MELAB and its earlier incarnation (Michigan Test of English language Proficiency) since 1961 like the TOEFL it serves a North American audience but is also used internationally While its use is not as widespread as the TOEFL its validity is widely respected Because it is cheaper than the TOEFL and more easily obtained it is popular among language schools and institutes Many institutions and companies accept MElAB scores in lieu ofTOEFL scores

International English Language Testing System (IELTS)

Producer Jointly managed by The University of Cambridge local Examinations Syndicate (UClES) The British Council and lOP Education Australia

Objective To test overall proficiency (language ability) Primary-market Australian British Canadian and New Zealand academic

institutions and professional organizations American academic institutions are increasingly accepting IELTS for admissions purposes

1)rpe Computer-based (for the Reading and Writing sections) papershybased for the listening and Speaking modules

Response modes Multiple-choice responses essay oral production Time allocation 2 hours 45 minutes Internet access httpwwwieltsorgl

httpwwwudesorguk httpwwwbritishcouncilorg

Specifications Reading candidates choose between academic reading or general training reading (60 minutes) Writing the same option academic writing or general training writing (60 minutes) Listening four sections for all candidates (30 minutes) Speaking five sections for all candidates (1015 minutes)

86 CHAPTER 4 Standardized Testing

Comments The University of Cambridge local Examinations Syndicate (UCLES) has been producing English language tests since 1858 Now with three organizations cooperatshying to form the IELTS more than a million examinations are administered every year In 2002 a computer-based version of the Reading and Writing modules of the IELTS became available at selected centers around the world The other sections are administered locally by an examinet The paper-based IELTS remains an option for candidates The IELTS retains the distinct advantage of requiring all four skills in the test-takers performance

Test of English for International Communication (TOEICreg)

Producer The Chauncey Group International a subsidiary of Educational Testing Service

Objelttive To test overall proficiency (langlJage ability) Primary market Worldwide business commerce and industry contexts

(workplace settings) Type Computer-based and paper-based versions Response modes Multiple-choice responses Time allocation 2 hours Internet access httpwwwtoeiccom

Specifications Listening Comprehension 100 items administered by audiocassette Four types of task statements questions short conversations and short talks (approxishymately 45 minutes) Reading 100 items Three types of task cloze sentences error recogshynition and reading comprehension (75 minutes)

Comments The TOEIC has become a very widely used international test of English proficiency in workplace settings where English is required for job performance The conshytent includes many different employment settings such as conferences presentations sales ordering shipping schedules reservations (etters and memoranda It is approprishyate to use in educational settings where vocational or workplace English courses are being offered

sect sect sect sect sect

The construction of a valid standardized test is no minor accomplishment whether the instrument is large- or small-scale The designing of specifications alone as this chapter illustrates requires a sophisticated process of construct valishydation coupled with considerations of practicality Then the construction of items and scoringinterpretation procedures may require a lengthy period of trial and error with prototypes of the final form of the testWith painstaking attention to all the details of construction the end product can result in a cost-effective timeshysaving accurate instrument Your use of the results of such assessments can provide useful data on learners language abilities But your caution is warranted as well for all the reasons discussed in this chapter The next chapter will elaborate on what lies behind that need for a cautious approach to standardized assessment

CHAPTER4 Standardized Testing 87

EXERCISES

[Note (I) Individual work (G) Group or pair work (C) Whole-class discussion]

1 (C) Tell the class about the worst test experience youve ever had Briefly anamiddot lyze what made the experience so unbearable and try to come up with sugshygestions for improvement of the test andor its administrative conditions

2 (G) In pairs or small groups compile a brief list of pros and cons of standardshyized testing Cite illustrations of as many items in each list as possible Report your lists and examples to the rest o~ the class

3 (I) Select a standardized test that you are quite familiar with (probably a recent experience) Mentally evaluate that test using the five principles of practicality reliability validity authenticity and washback Report yourevaluashytion to the class

4 (G) The appendix to this chapter provides sample items from Jour different tests of language proficiency In groups one test for each group analyze your test for (a) content validity (b) face validity and (c) authenticity

5 (C) Do you think that the sample TOEFL reading passage about pirates (pages 74-75) and the Graduate EssayTest prompt (pages 76-77) about a school board hiring committee have any culture bias Discuss this and other cultural biases you have noticed in tests Is it possible to design a test that is completely free of culture bias

6 (CG) Compare the differences in conceptualization of language proficiency represented by Swains model the TOEFL and the ACfFL philosophy Which one best represents current thinking about communicative language ability What are the strengths and weaknesses of each approach

FORYOlIILEURTHER READING

Gronlund Norman E (1998) Assessment of student achievement Sixth Edition Boston Allyn and Bacon

Gronlunds classic also mentioned in Chapter 3 offers a concise overview of features of standardized tests offering definitions and examples of the statistical considerations in interpreting scores His approach is unbiased cleady written and accessible to those who might fear the mathematics of standardized testing

Phillips Deborah 2001 Long1nan introductory course for the TOEFL test White Plains NY Pearson Education

A careful examination of this or any other reputable preparation course for a standardized language test is well worth a students time Note especially how the book acquaints the user with the specifications of the test and offers a number of useful strategie~ that can be llsed in preparation for the test and during irs adn1inistration

88 CHAPTER 4 Standardized Testing

APPENDIX TO CHAPTER 4

Commercial Proficiency Tests Sample Items and Tasks

Test of English a~ a Foreign Language (TOEFLreg)

Listening r

Part A

In this section you will hear short conversations between two people In some ofthe conversations each person speaks only once In other conversations one or both of the people speak more than once Each conversation is followed by one questionabQlt it Each question in this part has four answer choices You should click on the best answer to each question Answer the questions on the basis of what is stated or implied by the speakers Here is an example On the computerscreen you will see

[man and woman talking]

On the recording you will hear

(woman) Hey wheres your sociology book (man) At home Why carry it around when were just going to be taking

a test (woman) Dont you remember Professor Smith said we could us it during

the test (man) Ohl no Well Ive still got an hour right Im so glad I ran into you

You wiII then see and hear the question before the answer choices appear

What will the man probably do next

o Begin studying for the sociology test o Explain the problem to his professor o Go home to get his textbook o Borrow the womans book

To choose an answer you will click on an oval The oval next to that answer will darken After you click on Next and Confirm Answer the next conversation will be presented

Part B

In this section you will hear several longer conversations and talks Each conversation or talk is followed by several questions The conversations talks and questions will not be repeated The conversations and talks are about a variety of topics You do not need speshycial knowledge of the topics to answer the questions correctly Rather you should answer each question on the basis of what is stated or implied by the speakers in the conversashytions or talks

For most of the questions you will need to click on the best of four possible answers Some questions will have special directions The special directions will appear in a box on the computer screen Here is an exampie ot a conversation and some questions

CHAPTER 4 Standardized Testing 89

Marine Biology (narrator) Listen to part of a discussion in a marine biology class

(professor) A few years ago our local government passed a number of strict environmental laws As a result Sunrise Beach looks nothing Ii ke it did ten years ago The water is cleaner and theres been a tremendous increase in all kinds of marine life which is why were going there on Thursday

(woman) I dont know if I agree that the water quality has improved I mean I was out there last weekend and it looked all brown It didnt seem too clean to me

(professor) Actually the color of the water doesnt always indicate whether its polluted The brown color you mentioned might be a result of pollution or it can mean a kind of brown algae is growing there Its called devils apron and it actually serves as food for whales

(man) So when does the water look blue (professor) Well water thats completely unpolluted is actually colorless But

it often looks bluish-green because the sunlight can penetrate deep down and thats the color thats reflected

(woman) But sometimes it looks really green Whats that about (professor) Ok well its the same principle as with devils apron the

water might be green because of different types of green algae there-gulfweed phytoplankton You all should finish reading about algae and plankton before we go In fact those are the types of living things Im going to ask you to be looking for when were there

Now get ready to answer the questions

What is the discussion mainly about

o The importance of protecting ocean environments o The reasons why ocean water appears to be different colors o The survival of whales in polluted water o The effect that colored ocean water has on algae

To choose an answer click on an oval The oval next to that answer will darken After you click on Next and Confirm Answer the next question will be presented

According to the professor what can make ocean water look browngt

o Pollution o Cloudy Skies o Sand o Algae

Click on 2 answers

To choose your answers you will click on the squares An XII wiii appear in each square

bullbullbullbullbullbullbull

90 CHAPTER 4 Standardized Testing

Structure and Written Expression This section measures the ability to recognize language that is appropriate for standard written English There are two types ofquestions in this section In the first type ofquestion there are incomplete sentences Beneath each sentence there are four words or phrases

Directions CIiSk on the one word or phrase that best completes the sentence

The colum~ine flower __ to nearly all of the United States can be raised from seed in almost any garden

native how native is how native is it is native

Time Help Confirm

After you click on Next and Confirm Answ~ the next question willbe presented

The second type of question has four underlined words or phrases You will choose the one underlined word or phrase that must be changed for the sentence to be correct

Directions Click on the one underlined word or phrase that must be changed for the senshytence to be correct

One of the most difficult problems in understanding sleep is determining what the funcshytions of sleep ~

lime Help Confirm

Clicking on an underlined word or phrase will darken it

Reading This section measures the ability to read and understand short passages similar in topic and style to those that students are likely to encounter in North American universities and colleges This section contains reading passages and questions about the passages There are several different types of questions in this section In the Reading section you will first have the opportunity to read the passage

The temperature of the Sun is over 10000 degrees Fahrenheit at the surface but it rises perhaps more than 270000000 at the center The Sun is so much hotter than the Earth that matter can exist only as a gasl except perhaps at the core In the core of the Sun the pressures are so great that despite the high temperature there may be a small solid core However no one really knows since the center of the Sun can never be directly observed ~ Solar astronomers do know that the Sun is divided into five general layers or zones Starting at the outside and going down into the Sun the zones are the corona chromoshysphere hotosphere convection zone and finally the core The first three zones are reshygarded as the Suns atmosphere But since the Sun has no solid surface it is hard to middottell where the atmosphere ends and the main body of the Sun begins

The Suns outermost layer begins about 10000 miles above the visible surface and goes outward for millions of miles This is the only part of the Sun that can be seen during an eclipse such as the one in February 1979 At any other time the corona can be seen

bullbullbullbullbullbullbull

bull bullbullbullbullbullbull

CHAPTER 4 Standardized Testing 91

only when special instruments are used on cameras and telescopes to block the light from the photosphere

The corona is a brilliant pearly white filmy light about as bright as the full Moon Its beautiful rays are a sensational sight during an eclipse The coronas rays flash out in a brilliant fan that has wispy spikelike rays near the Suns north and south poles The corona is generally thickest at the Suns equator The corona is made up of gases streamshying outward at tremendous speeds that reach a temperature of more than 2 million deshygrees Fahrenheit The gas thins out as it reaches the space around the planets By the time the gas of the corona reaches the Earth it has a relatively low density

When you have finished reading the passage you will use the mouse to click on Proceed Then the questions about the passage will be presented You are to choose the one best anshyswer to each question Answer all questions about the information in a passage on the basis ofwhat is stated or implied in that passage Most ofthe questions will be multiple-choice questions To answer these questions you will click on a choice below the question

With what topic is paragraph 2 mainly concerned

o How the Sun evolved o The structure of the Sun o Why scientists study the Sun o The distaflce of the Sun from the planets

Paragraph 2 is marked with an arrow (~)

You will see the next question after you click on Next

To answer some questions you will click on a word or phrase Here is an example

Look at the word one in the passage Click on the word or phrase in the bold text that one refers to To answer you can click on any part of the word or phrase in the passage Jour choice will darken to show which word you have chosen

The Suns outermost layer begins about 10000 miles above the visible surface and goes outward for millions of miles This is the only part of the Sun that can be seen durshying an eclipse such as the one in February 1979 At any other time the corona can be seen only when special instruments are used on cameras and telescopes to block the Iight from the photosphere

You will see the next question after you click on~ To answer some q~estions you will click on a sentence in the passage Here is an example

~ The corona is a brilliant pearly white filmy light about as bright as the full Moon Its beautiful rays are a sensational sight during an eclipse The coronas rays flash out in a brilliant fan that has wispy spikelike rays near the Suns north and south poles The corona is generally thickest at the Suns equator ~ The corona is made up of gases streaming outward at tremendous speeds that reach a temperature of more than 2 million degrees Fahrenheit The gas thins out as it reaches the space around the planets By the time the gas of the corona reaches the Earth it has a relatively low density

bull bullbullbullbullbullbull

92 CHAPTER 4 Standardized Testing

Click on the sentence in paragraph 4 or 5 in which the author compares the light of the Suns outermost layer to that ofanother astronomical body Paragraphs 4 and 5 are marked with arrows (~)

To answer some questions you will click on a square to add a sentence to the passage Here is an example -The following sentence can be added to paragraph 1

At the center of the Earths solar system lies the Sun

Where would it best fit in paragraph I Click on the square to add the sentence to the paragraph

D The temperature of the Sun is over 10000 degrees Fahrenheit at the surface but it rises to perhaps morethan 27000000deg at the center 0 The Sun is so much hotter than the Earth that matter can exist only as a gasi except p~rHapsatth~ c6relp the c~re of the ii Sun the pressures are so great that despite the high temperature there may be a small solid core D However no one really knows since the center of the Sun can never be directly observed D 0100

When you click on a square the sentence will appear in the passage at the place you have chosen You can read the sentence added to the paragraph to see if this is the best place to add it You can click on another square to change your answer The sentence will be added and shown in a dark box

Writing In this section you will have an opportunity to demonstrate your ability to write in Enshyglish This includes the ability to generate and organize ideas to support those ideas with examples or evidence and to compose in standard written English in response to an asshysigned topic You will have 30 minutes to write your essay on that topic You must write on the topic you are assigned An essay on any other topic will receive a score of 0 Read the topic below and then make any notes that will help you plan your response Begin typing your response in the box at the bottom of the screen or write your answer on the answer sheet provided to you

Following is a sample topic

Do you agree or disagree with the following statemenH

Teachers should make learning enjoyable and fun for their students

Use specific reasons and examples to support your opinion

CHAPTER 4 Standarczed Testing 93

Michigan English Language Assessment Battery (MELAB)

Composition The time limit for the composition is 30 minutes You must write on only one of the top~

ics below If you write about something else your composition paper will not be graded and you cannot be given a final score If you do not understand the topics ask the exam~ iner to explain or to translate them You may be asked to give your opinion ofsomething and explain why you believe this to describe something from your experience or to exshyplain a problem and offer possible solutions You should write at least one page Some sample topics are

1 What do you think is your countrys greatest problem Explain in detail and tell what you think can be done about it

2 What are the characteristics of a good teacher Explain and give examples 3 An optimist is someone who sees the good side of things A pessimist sees the

bad side Are you an optimist or a pessimist Relate a personal experience that shows this

4 In your opinion are the benefits of space exploration really worth the enormous costs Discuss

Most MELAB compositions are one or two pages long (about 200-300 words) If your paper is extremely short (less than 150 words) your composition will be given a lower score Before you begin writing you might want to take 2 or 3 minutes to plan your comshyposition and to make a short outline to organize your thoughts Such outlines will not be graded they are only to help you You should use the last 5 minutes to read through your composition and to make changes or corrections

Your composition will be graded on how clearly you express yourself in English and on the range of English you are able to use and your control in doing so This means your composition should be well organized your arguments should be fully developed and you should show a range ofgrammatical structures and broad vocabulary Compositions that consist only of very short sentences and very simple vocabulary cannot be given the

middothighest scores If errors are not frequent and if they do not confuse your meaning they will not lower your score very much

Listening Now you will hear a short lecture You may take notes during the lecture Following the lecture you will be asked some questions about it

Therell be a two-week exhibit of the paintings of the little-known master Laura Bernhart at the Claire Osmond Galleries starting on the fifteenth of the month and running through the thirtieth Bernharts known for her innovative designs in abstract expressionism Though a true original she declared a spiritual heritage from Salvador Dali the famous Spanish painter Since Bernhart lived a rather solitary life and died while only in her twenties few people are aware of her works This showing at the Osmond Galleries will provide many with an introduction to her works

10 Where is the exhibit a the Art Museum b the Dali Galleries c the Osmond Galleries

94 CHAPTER 4 Standardized Testing

11 What is Bernhart known for a her copies of Dalis paintings b the originality of her designs c her exhibitions

12 What will going to the exhibit allow most people to do a to see Saivador Dalis paintings b to see Bernharts works for the first time c to learn about Spanish art

Grammar

1 What did the teacher just tell you

She reminded our notebooksI a us to bring b that we bring c our bringing d we should bring

2 Is Bill a good dancer

Not really __ he tries very hard a in spite of h despite c even though d while

3 your clothes are all wet1

Yes I didnt come __ the rain soon enough a away to b over to c down with d in from

Cloze In years to come zoos will not only be places where animals are exhibited to the public but repositories where rare species can be saved from extinction (7) captive breeding The most powerful force (8) the future of many animals-and of zoos-is the decline of the wild (9) even zoo directors would argue that (10) are better places for animals than the fields and forest of their native (11) yet zoos may be the last chance for some creatures that would otherwise pass qUietly into oblivion

7 a through c from b of d damage

8 a bringing c to b that d influencing

9 a But c Not b So d Then

10 a where c even b zoos d wilds

11 alands c residence b life d field

CHAPTER 4 Standardized Testing 95

Vocabulary

12 Mark has a flair for writing a need b purpose c talent d dislike

13 Bill Collins launched his restaurant last June a moved b started c sold d bought

14 John will not accept the censure a burden b blame c credit d decision

15 I cant think of the answer Can you give me a __ a hint b token c taste d gaze

16 Because fewer people are taking expensive vacations the tourist industry is in a a choke b grope c grumble d slump

17 I disagree with a few of his opinions but __ we agree a deliberately b conclusively c essentially d immensely

Reading The influenza virus is a single molecule built from many millions of single atoms You must have heard of the viruses which are sometimes called living molecules While bacteria can be considered as a type of plant secreting pOisonous substances into the body of the organism they attack viruses are living organisms themselves We may conshysider them as regular chemical molecules since they have a strictly aefined atomic strucshyture but on the other hand we must also consider them as being alive since they are able to multiply in unlimited quantities

18 According to the passage bacteria are a poisons

b larger than viruses c very small d plants

96 CHAPTER 4 Standardized Testing

19 The writer says that viruses are alive because they a have a complex atomic structure b move c multiply d need warmth and light

20 The atomic structure of viruses a is -tJIariable b is strictly defined c cannot be analyzed chemically d is more complex than that of bacteria

International English Language Testing System (fELTS)

I

listening

The Listening Module has four sections The first two sections are concerned with social needs There is a conversation between two speakers and then a monologue For examshyple a conversation about travel arrangements or decisions on a night out and a speech about student services on a university campus or arrangements for meals during a confershyence The final two sections are concerned with situations related more closely to educashytional or training contexts For example conversation between a tutor and a student about an assignment or between three students planning a research project and a lecture or talk ofgeneral academic interest All the topics are ofgeneral interest and it makes no difference what subjects candidates study Tests and tasks become more difficult as the sections progress A range of English accents and dialects are used in the recording which reflects the international usage of IELTS

Academic Reading [A 7S0-word article on-th-e- topic of Wind Power in the US with a short glossary at the end]

Questions 1-5

Complete the summary below

Choose your answers from the box below the summary and write them in boxes 1-5 on your answer sheet Note There are more words or phrases than you will need to fill the gaps You may use any word or phrase more than once

Example The failure during the late 1970s and early 19805 of an attempt to establish a widespread wind power industry in the United States resulted largely from the (1) bull in oil prices during this period The industry is now experiencing a steady (2) due to improveshyments in technology and an increased awareness of the potential in the power of wind The wind turbines that are now being made based in part on the (3) of wide- ranging research in Europe are easier to manufacture and maintain than their predecesshysors This has led wind-turbine makers to be able to standardise and thus minimize (4) There has been growing (S) of the importance of wind power as an energy source

CHAPTER 4 Standardized Testing 97

criticism stability skepticism success operating costs decisions design costs fall effects production costs growth decline failure recognition results

Questions 6-1 0 Look at the following list of issues (Questions 6-10) and implications (A-C) Match each issue with one implication Write the appropriate letters A-C in boxes 6-10 on your anshyswer sheet

Example The current price of one wind-generated kilowatt Answer

6 The recent installation of systems taking advantage of economies of scale

7 The potential of meeting one fifth of current U5 energy requirements by wind power

8 The level of acceptance of current wind turbine technology

9 A comparison of costs between conventional and wind power sources

10 The view of wind power in the European Union

Implications

A provides evidence against claims that electricity produced from wind power is relatively expensive

B supports claims that wind power js an important source of energy

C opposes the view that wind power technology requires further-development

General Training Reading Read the passage on Daybreak trips by coach and look at the statements below On your answer sheet write

TRUE if the statement is true FALSE jf the statement is false

NOlGIVEN if the information is not given in the leaflet

1 MiIlers Coaches owns Cambridges Cam bus fleet

2 Premier is an older company than Millers

3 Most of the Daybreak coaches are less than 5 years old

4 Daybreak fares are more expensive than most of their competitors

5 Soft drinks and refreshments are served on most longer journeys

6 Smoking is permitted at the rear of the coach on longer journeys

7 Tickets must be bought in advance from an authorised Daybreak agent

6 Tickets and seats can be reserved by phoning the Daybreak Hotline

9 Daybreak passengers must join their coach at Cambridge Drummer Street

10 Daybreak cannot guarantee return times

98 CHAPTER 4 Standardized Testing

FROM CAMBRIDGE AND SURROUNDING AREA

SPRING IS INTHEAIR

Welcome to our Spring Daybreak programme which continues the tradition of offering unbeatable value for money day trips and tours All the excursions in this brochure will be operated by Pr~mier Travel Services Limited or Millers Coaches both companies are part of the CHLGroup owners of Cambridges Cambus fleet

WERE PROUD OF OUR TRADITION

Premier was established in 1936 the Company now offers the highest standards of coaching in todays competitive operating environment Miller has an enviable reputation stretching back over the past 20 years offering coach services at realistic prices Weve traveled a long way since our early days of pre-war seaside trips Now our fleet of 50 modern coaches (few are more than five years old) operate throughout Britain and Europe but were pleased to still maintain the high standards of quality and service the trademark of our founders nearly sixty years ago

EXCLUSIVE FEATURES

Admission-inclusive fares All Daybreak fares (unless specifically otherwise stated) include admission charges to the attractions shows and exhibits we visit Many full-day scenic tours are accompanied by a fully trained English Tourist Board Blue Badge guide or local experienced driverguide Some Daybreaks include lunch or afternoon tea Compare our admission inclusive fares and see how much you save Cheapest is not the best and value for money is guaranteed If you compare our bargain Daybreak fares beware--most of our competishytors do not offer an all-inclusive fare

SEAT RESERVATIONS

We value the freedom of choice so you can choose your seat when you book The seat reservation is guaranteed a-nd remains yours at all times when aboard the coach

NO SMOKING COMFORT

With the comfort of our passengers in mind coaches on all our Daybreaks are no smokshying throughout In the interests of fellow passengers comfort we kindly ask that smokers observe our no smoking policy On scenic tours and longer journeys ample refreshment stops are provided when of course smoking is permitted

YOUR QUESTIONS ANSWERED

Do I need to book Booking in advance is strongly recommended as all Daybreak tours are subject to demand Subject to availability stand-by tickets can be purchased from the driver

What ti me does the coach leave The coach departs from Cambridge Drummer Street (Bay 12 adjacent to public toilets) at the time shown There are many additional joining points indicated by departure codes in the brochure If you are joining at one of our less popular joining points you will be adshyvised of your pick-up time (normally by telephone) not less than 48 hours before deparshyture In this way we can minimize the length of pick-up routes and reduce journey times for the majority of passengers

CHAPTER 4 Standardized Testing 99

What time do we get back An approximate return time is shown for each excursion The tim~s shown serve as a guide but road conditions can sometimes cause delay If your arrival will be later than advertised your driver will try to allow for a telephone call during the return journey

Where can I board the coach All the Daybreaks in the brochure leave from Cambridge Drummer Street (Bay 12 adjashycent to public toilets) at the time shown Many Daybreaks offer additional pick-ups for pre-booked passengers within Cambridge and the surrounding area This facility must be requested at the time of booking

Academic Writing Writing Task 1 You should spend about 20 minutes on this task

The graph below shows the different modes of transport used to travel to and from work in one European city in 1950 1970 and 1990

[graph shown here]

Write a report for a university lecturer describing the information shown below You should write at least 150 words

Writing Task 2 You should spend about 40 minutes on this task

Present a written argument or case to an educated reader with no specialist knowledge of the folowing topic

It is inevitable that as technology develops so traditional cultures must be lost Technolshyogy and tradition are incompatible-you cannot have both together

To what extent do you agree or disagree with this statement Give reasons for your answer You should write at least 250 words You should use your own ideas knowlshyedge and experience and support your arguments with examples and relevant evidence

General Training Writing Writing Task 1 You should spend about 20 minutes on this task You rent a house through an agency The heating system has stopped working You phoned the agency a week ago but it has still not been mended Write a letter to the agency Explain the situation and teil them what you want them to do about it

You should write at least 150 words You do NOT need to write your own address

Begin your letter as follows

Dear - ___-I

Writing Task 2 You should spend about 40 minutes on this task As part ofa class assignment you have to write about the following topic

100 CHAPTER 4 Standardized Testing

Some businesses now say that no one can smoke cigarettes in any of their offices Some governments have banned smoking in all public placesThis is a good idea but it takes away some of our freedom

Do you agree or disagree Give reasons for your answer You should write at least 250 words

Speaking In each ofthe three parts of the speaking module a specific function is fulfilled In Part 1 the candidates answer general questions about themselves their homes or families their jobs or studies their interests and a range ofsimilar familiar topic areas This part lasts between four and five minutes In Part 2 the candidate is given a verbal prompt on a card and is asked to talk on a particular topic The candidate has one minute to prepare before speaking at length for between one and two minutes The examiner then asks one or two wind-down questions In Part 3 the examiner and candidate engage in a discusshysion of more abstract issues and concepts which are thematically linked to the topic prompt in Part 2 The discussion lasts between four and five minutes

All interviews are recorded on audiocassette Here is a sample ofa Part 2 topic

Describe a teacher who has greatly influenced you in your education

You shou Id say

where you met them what subject they taught what was special about them

and explain why this person influenced you so much

You will have to talk about the topic for 1 to 2 minutes You have 1 minute to think about what you are going to say You can make some notes if you wish

Test of English for International Communication (TOEICreg)

listening

Part 1 Photographs Directions For each question you will see a picture in your test book and you will hear four short statements The statements will be spoken just one time They will not be printed in your test book so you must listen carefully to understand what the speaker says When you hear the four statements look at the picture in your test book and choose the statement that best describes what you see in the picture Then on your answer sheet find the number of the question and mark your answer

[photograph of a scientist looking through a microscope]

You will hear Look at the picture marked number 1 in your test book

(A) Shes speaking into a microphone (B) Shes put on her glasses (C) She has both eyes open (D) Shes using a microscope

CHAPTER 4 Standardized Testing 101

Part 2 Question-Response Directions In this part of the test you will hear a question or statement spoken in Enshyglish followed by three responses also spoken in English The question or staten1ent and the responses will be spoken just one time They will not be printed in your test book so you must listen carefully to understand what the speakers say You are to choose the best response to each question or statement

Question 1 You will hear Ms Morikawa has worked here for a long time hasnt she

(A) At three oclock (B) No Ive lost my watch (C) More than ten years

Question 2 You will hear Which of these papers has a wider circulation

(A) The morning edition (B) Get more exercise (C) By messenger

Part 3 Short Conversations Directions In this part of the test you will hear short conversations between two people The conversations will not be printed in your test book You will hear the conversations only once so you must listen carefully to understand what the speakers say In your test book you will read a question about each conversation The question will be followed by four answers You are to choose the best answer to each question and mark it on your answer sheet

Question 1 (Man) We should think about finding another restaurant for lunch (Woman) Why The food and service here are great

(Man) Yes but the prices are going up every week

You will read Why is this man unhappy with the restaurant

(A) It is too noisy (B) It is too expensive (C) It is too crowded (D) It is too difficult to find

Question 2 (Woman A) How was Dr Borgs recent trip to Singapore (Woman B) She enjoyed the tour of the port very much (Woman A) They say its one of the most active in Asia

You will read 2 What did Dr Borg find interesting

(A) The tourist center (B) The airport (C) The musical performance (D) The harbor

Part 4 Short Talks Directions In this part of the test you vill hear several short talks Each will be spoken just one time They will not be printed in your test book so you must listen carefully to understand and remember what is said In your test book you will read two or more questions about each short talk The questions will be followed by four answers You are to choose the best answer to each question and mark it on your answer sheet

102 CHAPTER 4 Standardized Testing

You will hear Questions 1 and 2 refer to the following announcement

Good afternoon and welcome aboard Nordair Flight 857 from Copenhagen to Bangkok with intermediate stops in Dubai and Calcutta We are preparing for departure in a few minutes At this time your seat back should be returned to its full upright position and your seat belt s~ould be fastened OUf anticipated total flying time to Dubai is six hours and twenty-five minutes I hope you enjoy the flight You will hecJr Now read question 1 in your test book and answer it You will read 1 What is the final destination of the flight

(A) Bangkok (B) Copenhagen (C) Dubai (O) Calcutta

You will hear Now read question 2 in your test book and answer it You will read 2 What will happen in a few minutes

(A) The flight will land in Dubai I

(B) The passengers will board the plane (C) The plane will take off (0) The gate number will be announced

Reading In this section of the test you will have the chance to show how well you understand written English There are three parts to this section with special directions for each part

Part 4 Incomplete Sentences Directions This part of the test has incomplete sentences Four words or phrases marked (A) (8) (e) (D) are given beneath each sentence You are to choose the one word or phrase that best completes the sentence Then on your answer sheet find the number of the question and mark your answer

1 Mr Yangs trip will __ him away from the office for ten days (A) withdraw (B) continue (C) retain (0) keep

2 The company that Marie DuBois started now sells __ products throughout the world (A) its (B) it (C) theirs (D) them

3 If your shipment is not delivered __ Tuesday you can request a full refund for the merchandise (A) at (B) by (C) within (D) while

CHAPTER 4 Standardized Testing 103

Part 6 Error Recognition Directions In this part ofthe test each sentence has four words or phrases underlined The four underlined parts of the sentence are marked (A) (B) (C) (D) You are to identify the one underlined word or phrase that should be corrected or rewritten Then on your answer sheet find the number of the question and mark your answer

1 The pamphlet contains some importance information about the current exhibit ABC D

2 No matter how Jong it taking to finish the annual report it must be done properly ABC D

3 The popularity of jogging appears to have decreased since the past couple of years ABC D

Part 7 Reading Comprehension Directions The questions in this part of the test are based on a selection of reading mateshyrials such as notices letters) forms newspaper and magazine articles) and advertisements You are to choose the one best answer (A) (B) (C) or (OJ to each quesshytion Then on your ariswefsheelfindthe number of the qUestion andmcirkyour answer Answer all questions following each reading selection on thebasis of what is stated or implied in that selection

The Museum ofTechnology is a hands-on museum designed for people to experience science at w()rk~ Visitors are encouraged to use test and handle the objects o~ display Special demonstrations are scheduled for the first and second Wednesdays of each month at 1330 Open Tuesday-Friday 1200-1630 Saturday 1000-1730 and Sunday 11 00-1630

1 When during the month can visitors see special demonstrations (A) Every weekend (B) The first two Wednesdays (C) One afternoon a week (D) Every other Wednesday

Questions 2 and 3 refer to the followi ng notice

NOTICE If you are unable to work because of an extended illness or injury that is not workshyrelated you may be entitled to receive weekly benefits from your employer or the firms insurance company To claim benefits you must file a claim form within thirty days of the first day of your disability Before filing the claim you must ask your doctor to fill in the Doctors Statement on the claim form stating the period of disability

3 To whom is this notice addressed (A) Employers (8) Doctors (C) Employees (D) When paying the bill

4 When must the claim form be filed (A) On the first of the month (8) On the thirtieth of the month (C) On the first day ofdisabifity (D) Within 30 days of the start of disability

Page 20: Standardized Testing Chapter 4 Brown

CHAPTER 4 Standardi~ed Testing 85

Michigan English Language Assessment Battery (MELAB)

Producer English language Institute University of Michigan Objective To test overall proficiency (language ability) Primary market Mostly US and Canadian language programs and colleges

some worldwide educational settings as well Type Paper-based Response modes Multiple-choice responses essay Time allocation 25 to 35 hours Internet access wwwlsaumicheduelimelabhtm

Specifications The MElAB consists of three sections Part 1 a 3D-minute impromptu essay is written on an assigned topic Part 2 a 25-minute multiple-choice listening comshyprehension test is delivered via tape recorder Part 3 is a 100-item 75-minute multipleshychoice test containing grammar doze reading vocabulary and reading comprehension An oral interview (speaking test) is optional

Comments The Ell at the University of Michigan has been producing the MELAB and its earlier incarnation (Michigan Test of English language Proficiency) since 1961 like the TOEFL it serves a North American audience but is also used internationally While its use is not as widespread as the TOEFL its validity is widely respected Because it is cheaper than the TOEFL and more easily obtained it is popular among language schools and institutes Many institutions and companies accept MElAB scores in lieu ofTOEFL scores

International English Language Testing System (IELTS)

Producer Jointly managed by The University of Cambridge local Examinations Syndicate (UClES) The British Council and lOP Education Australia

Objective To test overall proficiency (language ability) Primary-market Australian British Canadian and New Zealand academic

institutions and professional organizations American academic institutions are increasingly accepting IELTS for admissions purposes

1)rpe Computer-based (for the Reading and Writing sections) papershybased for the listening and Speaking modules

Response modes Multiple-choice responses essay oral production Time allocation 2 hours 45 minutes Internet access httpwwwieltsorgl

httpwwwudesorguk httpwwwbritishcouncilorg

Specifications Reading candidates choose between academic reading or general training reading (60 minutes) Writing the same option academic writing or general training writing (60 minutes) Listening four sections for all candidates (30 minutes) Speaking five sections for all candidates (1015 minutes)

86 CHAPTER 4 Standardized Testing

Comments The University of Cambridge local Examinations Syndicate (UCLES) has been producing English language tests since 1858 Now with three organizations cooperatshying to form the IELTS more than a million examinations are administered every year In 2002 a computer-based version of the Reading and Writing modules of the IELTS became available at selected centers around the world The other sections are administered locally by an examinet The paper-based IELTS remains an option for candidates The IELTS retains the distinct advantage of requiring all four skills in the test-takers performance

Test of English for International Communication (TOEICreg)

Producer The Chauncey Group International a subsidiary of Educational Testing Service

Objelttive To test overall proficiency (langlJage ability) Primary market Worldwide business commerce and industry contexts

(workplace settings) Type Computer-based and paper-based versions Response modes Multiple-choice responses Time allocation 2 hours Internet access httpwwwtoeiccom

Specifications Listening Comprehension 100 items administered by audiocassette Four types of task statements questions short conversations and short talks (approxishymately 45 minutes) Reading 100 items Three types of task cloze sentences error recogshynition and reading comprehension (75 minutes)

Comments The TOEIC has become a very widely used international test of English proficiency in workplace settings where English is required for job performance The conshytent includes many different employment settings such as conferences presentations sales ordering shipping schedules reservations (etters and memoranda It is approprishyate to use in educational settings where vocational or workplace English courses are being offered

sect sect sect sect sect

The construction of a valid standardized test is no minor accomplishment whether the instrument is large- or small-scale The designing of specifications alone as this chapter illustrates requires a sophisticated process of construct valishydation coupled with considerations of practicality Then the construction of items and scoringinterpretation procedures may require a lengthy period of trial and error with prototypes of the final form of the testWith painstaking attention to all the details of construction the end product can result in a cost-effective timeshysaving accurate instrument Your use of the results of such assessments can provide useful data on learners language abilities But your caution is warranted as well for all the reasons discussed in this chapter The next chapter will elaborate on what lies behind that need for a cautious approach to standardized assessment

CHAPTER4 Standardized Testing 87

EXERCISES

[Note (I) Individual work (G) Group or pair work (C) Whole-class discussion]

1 (C) Tell the class about the worst test experience youve ever had Briefly anamiddot lyze what made the experience so unbearable and try to come up with sugshygestions for improvement of the test andor its administrative conditions

2 (G) In pairs or small groups compile a brief list of pros and cons of standardshyized testing Cite illustrations of as many items in each list as possible Report your lists and examples to the rest o~ the class

3 (I) Select a standardized test that you are quite familiar with (probably a recent experience) Mentally evaluate that test using the five principles of practicality reliability validity authenticity and washback Report yourevaluashytion to the class

4 (G) The appendix to this chapter provides sample items from Jour different tests of language proficiency In groups one test for each group analyze your test for (a) content validity (b) face validity and (c) authenticity

5 (C) Do you think that the sample TOEFL reading passage about pirates (pages 74-75) and the Graduate EssayTest prompt (pages 76-77) about a school board hiring committee have any culture bias Discuss this and other cultural biases you have noticed in tests Is it possible to design a test that is completely free of culture bias

6 (CG) Compare the differences in conceptualization of language proficiency represented by Swains model the TOEFL and the ACfFL philosophy Which one best represents current thinking about communicative language ability What are the strengths and weaknesses of each approach

FORYOlIILEURTHER READING

Gronlund Norman E (1998) Assessment of student achievement Sixth Edition Boston Allyn and Bacon

Gronlunds classic also mentioned in Chapter 3 offers a concise overview of features of standardized tests offering definitions and examples of the statistical considerations in interpreting scores His approach is unbiased cleady written and accessible to those who might fear the mathematics of standardized testing

Phillips Deborah 2001 Long1nan introductory course for the TOEFL test White Plains NY Pearson Education

A careful examination of this or any other reputable preparation course for a standardized language test is well worth a students time Note especially how the book acquaints the user with the specifications of the test and offers a number of useful strategie~ that can be llsed in preparation for the test and during irs adn1inistration

88 CHAPTER 4 Standardized Testing

APPENDIX TO CHAPTER 4

Commercial Proficiency Tests Sample Items and Tasks

Test of English a~ a Foreign Language (TOEFLreg)

Listening r

Part A

In this section you will hear short conversations between two people In some ofthe conversations each person speaks only once In other conversations one or both of the people speak more than once Each conversation is followed by one questionabQlt it Each question in this part has four answer choices You should click on the best answer to each question Answer the questions on the basis of what is stated or implied by the speakers Here is an example On the computerscreen you will see

[man and woman talking]

On the recording you will hear

(woman) Hey wheres your sociology book (man) At home Why carry it around when were just going to be taking

a test (woman) Dont you remember Professor Smith said we could us it during

the test (man) Ohl no Well Ive still got an hour right Im so glad I ran into you

You wiII then see and hear the question before the answer choices appear

What will the man probably do next

o Begin studying for the sociology test o Explain the problem to his professor o Go home to get his textbook o Borrow the womans book

To choose an answer you will click on an oval The oval next to that answer will darken After you click on Next and Confirm Answer the next conversation will be presented

Part B

In this section you will hear several longer conversations and talks Each conversation or talk is followed by several questions The conversations talks and questions will not be repeated The conversations and talks are about a variety of topics You do not need speshycial knowledge of the topics to answer the questions correctly Rather you should answer each question on the basis of what is stated or implied by the speakers in the conversashytions or talks

For most of the questions you will need to click on the best of four possible answers Some questions will have special directions The special directions will appear in a box on the computer screen Here is an exampie ot a conversation and some questions

CHAPTER 4 Standardized Testing 89

Marine Biology (narrator) Listen to part of a discussion in a marine biology class

(professor) A few years ago our local government passed a number of strict environmental laws As a result Sunrise Beach looks nothing Ii ke it did ten years ago The water is cleaner and theres been a tremendous increase in all kinds of marine life which is why were going there on Thursday

(woman) I dont know if I agree that the water quality has improved I mean I was out there last weekend and it looked all brown It didnt seem too clean to me

(professor) Actually the color of the water doesnt always indicate whether its polluted The brown color you mentioned might be a result of pollution or it can mean a kind of brown algae is growing there Its called devils apron and it actually serves as food for whales

(man) So when does the water look blue (professor) Well water thats completely unpolluted is actually colorless But

it often looks bluish-green because the sunlight can penetrate deep down and thats the color thats reflected

(woman) But sometimes it looks really green Whats that about (professor) Ok well its the same principle as with devils apron the

water might be green because of different types of green algae there-gulfweed phytoplankton You all should finish reading about algae and plankton before we go In fact those are the types of living things Im going to ask you to be looking for when were there

Now get ready to answer the questions

What is the discussion mainly about

o The importance of protecting ocean environments o The reasons why ocean water appears to be different colors o The survival of whales in polluted water o The effect that colored ocean water has on algae

To choose an answer click on an oval The oval next to that answer will darken After you click on Next and Confirm Answer the next question will be presented

According to the professor what can make ocean water look browngt

o Pollution o Cloudy Skies o Sand o Algae

Click on 2 answers

To choose your answers you will click on the squares An XII wiii appear in each square

bullbullbullbullbullbullbull

90 CHAPTER 4 Standardized Testing

Structure and Written Expression This section measures the ability to recognize language that is appropriate for standard written English There are two types ofquestions in this section In the first type ofquestion there are incomplete sentences Beneath each sentence there are four words or phrases

Directions CIiSk on the one word or phrase that best completes the sentence

The colum~ine flower __ to nearly all of the United States can be raised from seed in almost any garden

native how native is how native is it is native

Time Help Confirm

After you click on Next and Confirm Answ~ the next question willbe presented

The second type of question has four underlined words or phrases You will choose the one underlined word or phrase that must be changed for the sentence to be correct

Directions Click on the one underlined word or phrase that must be changed for the senshytence to be correct

One of the most difficult problems in understanding sleep is determining what the funcshytions of sleep ~

lime Help Confirm

Clicking on an underlined word or phrase will darken it

Reading This section measures the ability to read and understand short passages similar in topic and style to those that students are likely to encounter in North American universities and colleges This section contains reading passages and questions about the passages There are several different types of questions in this section In the Reading section you will first have the opportunity to read the passage

The temperature of the Sun is over 10000 degrees Fahrenheit at the surface but it rises perhaps more than 270000000 at the center The Sun is so much hotter than the Earth that matter can exist only as a gasl except perhaps at the core In the core of the Sun the pressures are so great that despite the high temperature there may be a small solid core However no one really knows since the center of the Sun can never be directly observed ~ Solar astronomers do know that the Sun is divided into five general layers or zones Starting at the outside and going down into the Sun the zones are the corona chromoshysphere hotosphere convection zone and finally the core The first three zones are reshygarded as the Suns atmosphere But since the Sun has no solid surface it is hard to middottell where the atmosphere ends and the main body of the Sun begins

The Suns outermost layer begins about 10000 miles above the visible surface and goes outward for millions of miles This is the only part of the Sun that can be seen during an eclipse such as the one in February 1979 At any other time the corona can be seen

bullbullbullbullbullbullbull

bull bullbullbullbullbullbull

CHAPTER 4 Standardized Testing 91

only when special instruments are used on cameras and telescopes to block the light from the photosphere

The corona is a brilliant pearly white filmy light about as bright as the full Moon Its beautiful rays are a sensational sight during an eclipse The coronas rays flash out in a brilliant fan that has wispy spikelike rays near the Suns north and south poles The corona is generally thickest at the Suns equator The corona is made up of gases streamshying outward at tremendous speeds that reach a temperature of more than 2 million deshygrees Fahrenheit The gas thins out as it reaches the space around the planets By the time the gas of the corona reaches the Earth it has a relatively low density

When you have finished reading the passage you will use the mouse to click on Proceed Then the questions about the passage will be presented You are to choose the one best anshyswer to each question Answer all questions about the information in a passage on the basis ofwhat is stated or implied in that passage Most ofthe questions will be multiple-choice questions To answer these questions you will click on a choice below the question

With what topic is paragraph 2 mainly concerned

o How the Sun evolved o The structure of the Sun o Why scientists study the Sun o The distaflce of the Sun from the planets

Paragraph 2 is marked with an arrow (~)

You will see the next question after you click on Next

To answer some questions you will click on a word or phrase Here is an example

Look at the word one in the passage Click on the word or phrase in the bold text that one refers to To answer you can click on any part of the word or phrase in the passage Jour choice will darken to show which word you have chosen

The Suns outermost layer begins about 10000 miles above the visible surface and goes outward for millions of miles This is the only part of the Sun that can be seen durshying an eclipse such as the one in February 1979 At any other time the corona can be seen only when special instruments are used on cameras and telescopes to block the Iight from the photosphere

You will see the next question after you click on~ To answer some q~estions you will click on a sentence in the passage Here is an example

~ The corona is a brilliant pearly white filmy light about as bright as the full Moon Its beautiful rays are a sensational sight during an eclipse The coronas rays flash out in a brilliant fan that has wispy spikelike rays near the Suns north and south poles The corona is generally thickest at the Suns equator ~ The corona is made up of gases streaming outward at tremendous speeds that reach a temperature of more than 2 million degrees Fahrenheit The gas thins out as it reaches the space around the planets By the time the gas of the corona reaches the Earth it has a relatively low density

bull bullbullbullbullbullbull

92 CHAPTER 4 Standardized Testing

Click on the sentence in paragraph 4 or 5 in which the author compares the light of the Suns outermost layer to that ofanother astronomical body Paragraphs 4 and 5 are marked with arrows (~)

To answer some questions you will click on a square to add a sentence to the passage Here is an example -The following sentence can be added to paragraph 1

At the center of the Earths solar system lies the Sun

Where would it best fit in paragraph I Click on the square to add the sentence to the paragraph

D The temperature of the Sun is over 10000 degrees Fahrenheit at the surface but it rises to perhaps morethan 27000000deg at the center 0 The Sun is so much hotter than the Earth that matter can exist only as a gasi except p~rHapsatth~ c6relp the c~re of the ii Sun the pressures are so great that despite the high temperature there may be a small solid core D However no one really knows since the center of the Sun can never be directly observed D 0100

When you click on a square the sentence will appear in the passage at the place you have chosen You can read the sentence added to the paragraph to see if this is the best place to add it You can click on another square to change your answer The sentence will be added and shown in a dark box

Writing In this section you will have an opportunity to demonstrate your ability to write in Enshyglish This includes the ability to generate and organize ideas to support those ideas with examples or evidence and to compose in standard written English in response to an asshysigned topic You will have 30 minutes to write your essay on that topic You must write on the topic you are assigned An essay on any other topic will receive a score of 0 Read the topic below and then make any notes that will help you plan your response Begin typing your response in the box at the bottom of the screen or write your answer on the answer sheet provided to you

Following is a sample topic

Do you agree or disagree with the following statemenH

Teachers should make learning enjoyable and fun for their students

Use specific reasons and examples to support your opinion

CHAPTER 4 Standarczed Testing 93

Michigan English Language Assessment Battery (MELAB)

Composition The time limit for the composition is 30 minutes You must write on only one of the top~

ics below If you write about something else your composition paper will not be graded and you cannot be given a final score If you do not understand the topics ask the exam~ iner to explain or to translate them You may be asked to give your opinion ofsomething and explain why you believe this to describe something from your experience or to exshyplain a problem and offer possible solutions You should write at least one page Some sample topics are

1 What do you think is your countrys greatest problem Explain in detail and tell what you think can be done about it

2 What are the characteristics of a good teacher Explain and give examples 3 An optimist is someone who sees the good side of things A pessimist sees the

bad side Are you an optimist or a pessimist Relate a personal experience that shows this

4 In your opinion are the benefits of space exploration really worth the enormous costs Discuss

Most MELAB compositions are one or two pages long (about 200-300 words) If your paper is extremely short (less than 150 words) your composition will be given a lower score Before you begin writing you might want to take 2 or 3 minutes to plan your comshyposition and to make a short outline to organize your thoughts Such outlines will not be graded they are only to help you You should use the last 5 minutes to read through your composition and to make changes or corrections

Your composition will be graded on how clearly you express yourself in English and on the range of English you are able to use and your control in doing so This means your composition should be well organized your arguments should be fully developed and you should show a range ofgrammatical structures and broad vocabulary Compositions that consist only of very short sentences and very simple vocabulary cannot be given the

middothighest scores If errors are not frequent and if they do not confuse your meaning they will not lower your score very much

Listening Now you will hear a short lecture You may take notes during the lecture Following the lecture you will be asked some questions about it

Therell be a two-week exhibit of the paintings of the little-known master Laura Bernhart at the Claire Osmond Galleries starting on the fifteenth of the month and running through the thirtieth Bernharts known for her innovative designs in abstract expressionism Though a true original she declared a spiritual heritage from Salvador Dali the famous Spanish painter Since Bernhart lived a rather solitary life and died while only in her twenties few people are aware of her works This showing at the Osmond Galleries will provide many with an introduction to her works

10 Where is the exhibit a the Art Museum b the Dali Galleries c the Osmond Galleries

94 CHAPTER 4 Standardized Testing

11 What is Bernhart known for a her copies of Dalis paintings b the originality of her designs c her exhibitions

12 What will going to the exhibit allow most people to do a to see Saivador Dalis paintings b to see Bernharts works for the first time c to learn about Spanish art

Grammar

1 What did the teacher just tell you

She reminded our notebooksI a us to bring b that we bring c our bringing d we should bring

2 Is Bill a good dancer

Not really __ he tries very hard a in spite of h despite c even though d while

3 your clothes are all wet1

Yes I didnt come __ the rain soon enough a away to b over to c down with d in from

Cloze In years to come zoos will not only be places where animals are exhibited to the public but repositories where rare species can be saved from extinction (7) captive breeding The most powerful force (8) the future of many animals-and of zoos-is the decline of the wild (9) even zoo directors would argue that (10) are better places for animals than the fields and forest of their native (11) yet zoos may be the last chance for some creatures that would otherwise pass qUietly into oblivion

7 a through c from b of d damage

8 a bringing c to b that d influencing

9 a But c Not b So d Then

10 a where c even b zoos d wilds

11 alands c residence b life d field

CHAPTER 4 Standardized Testing 95

Vocabulary

12 Mark has a flair for writing a need b purpose c talent d dislike

13 Bill Collins launched his restaurant last June a moved b started c sold d bought

14 John will not accept the censure a burden b blame c credit d decision

15 I cant think of the answer Can you give me a __ a hint b token c taste d gaze

16 Because fewer people are taking expensive vacations the tourist industry is in a a choke b grope c grumble d slump

17 I disagree with a few of his opinions but __ we agree a deliberately b conclusively c essentially d immensely

Reading The influenza virus is a single molecule built from many millions of single atoms You must have heard of the viruses which are sometimes called living molecules While bacteria can be considered as a type of plant secreting pOisonous substances into the body of the organism they attack viruses are living organisms themselves We may conshysider them as regular chemical molecules since they have a strictly aefined atomic strucshyture but on the other hand we must also consider them as being alive since they are able to multiply in unlimited quantities

18 According to the passage bacteria are a poisons

b larger than viruses c very small d plants

96 CHAPTER 4 Standardized Testing

19 The writer says that viruses are alive because they a have a complex atomic structure b move c multiply d need warmth and light

20 The atomic structure of viruses a is -tJIariable b is strictly defined c cannot be analyzed chemically d is more complex than that of bacteria

International English Language Testing System (fELTS)

I

listening

The Listening Module has four sections The first two sections are concerned with social needs There is a conversation between two speakers and then a monologue For examshyple a conversation about travel arrangements or decisions on a night out and a speech about student services on a university campus or arrangements for meals during a confershyence The final two sections are concerned with situations related more closely to educashytional or training contexts For example conversation between a tutor and a student about an assignment or between three students planning a research project and a lecture or talk ofgeneral academic interest All the topics are ofgeneral interest and it makes no difference what subjects candidates study Tests and tasks become more difficult as the sections progress A range of English accents and dialects are used in the recording which reflects the international usage of IELTS

Academic Reading [A 7S0-word article on-th-e- topic of Wind Power in the US with a short glossary at the end]

Questions 1-5

Complete the summary below

Choose your answers from the box below the summary and write them in boxes 1-5 on your answer sheet Note There are more words or phrases than you will need to fill the gaps You may use any word or phrase more than once

Example The failure during the late 1970s and early 19805 of an attempt to establish a widespread wind power industry in the United States resulted largely from the (1) bull in oil prices during this period The industry is now experiencing a steady (2) due to improveshyments in technology and an increased awareness of the potential in the power of wind The wind turbines that are now being made based in part on the (3) of wide- ranging research in Europe are easier to manufacture and maintain than their predecesshysors This has led wind-turbine makers to be able to standardise and thus minimize (4) There has been growing (S) of the importance of wind power as an energy source

CHAPTER 4 Standardized Testing 97

criticism stability skepticism success operating costs decisions design costs fall effects production costs growth decline failure recognition results

Questions 6-1 0 Look at the following list of issues (Questions 6-10) and implications (A-C) Match each issue with one implication Write the appropriate letters A-C in boxes 6-10 on your anshyswer sheet

Example The current price of one wind-generated kilowatt Answer

6 The recent installation of systems taking advantage of economies of scale

7 The potential of meeting one fifth of current U5 energy requirements by wind power

8 The level of acceptance of current wind turbine technology

9 A comparison of costs between conventional and wind power sources

10 The view of wind power in the European Union

Implications

A provides evidence against claims that electricity produced from wind power is relatively expensive

B supports claims that wind power js an important source of energy

C opposes the view that wind power technology requires further-development

General Training Reading Read the passage on Daybreak trips by coach and look at the statements below On your answer sheet write

TRUE if the statement is true FALSE jf the statement is false

NOlGIVEN if the information is not given in the leaflet

1 MiIlers Coaches owns Cambridges Cam bus fleet

2 Premier is an older company than Millers

3 Most of the Daybreak coaches are less than 5 years old

4 Daybreak fares are more expensive than most of their competitors

5 Soft drinks and refreshments are served on most longer journeys

6 Smoking is permitted at the rear of the coach on longer journeys

7 Tickets must be bought in advance from an authorised Daybreak agent

6 Tickets and seats can be reserved by phoning the Daybreak Hotline

9 Daybreak passengers must join their coach at Cambridge Drummer Street

10 Daybreak cannot guarantee return times

98 CHAPTER 4 Standardized Testing

FROM CAMBRIDGE AND SURROUNDING AREA

SPRING IS INTHEAIR

Welcome to our Spring Daybreak programme which continues the tradition of offering unbeatable value for money day trips and tours All the excursions in this brochure will be operated by Pr~mier Travel Services Limited or Millers Coaches both companies are part of the CHLGroup owners of Cambridges Cambus fleet

WERE PROUD OF OUR TRADITION

Premier was established in 1936 the Company now offers the highest standards of coaching in todays competitive operating environment Miller has an enviable reputation stretching back over the past 20 years offering coach services at realistic prices Weve traveled a long way since our early days of pre-war seaside trips Now our fleet of 50 modern coaches (few are more than five years old) operate throughout Britain and Europe but were pleased to still maintain the high standards of quality and service the trademark of our founders nearly sixty years ago

EXCLUSIVE FEATURES

Admission-inclusive fares All Daybreak fares (unless specifically otherwise stated) include admission charges to the attractions shows and exhibits we visit Many full-day scenic tours are accompanied by a fully trained English Tourist Board Blue Badge guide or local experienced driverguide Some Daybreaks include lunch or afternoon tea Compare our admission inclusive fares and see how much you save Cheapest is not the best and value for money is guaranteed If you compare our bargain Daybreak fares beware--most of our competishytors do not offer an all-inclusive fare

SEAT RESERVATIONS

We value the freedom of choice so you can choose your seat when you book The seat reservation is guaranteed a-nd remains yours at all times when aboard the coach

NO SMOKING COMFORT

With the comfort of our passengers in mind coaches on all our Daybreaks are no smokshying throughout In the interests of fellow passengers comfort we kindly ask that smokers observe our no smoking policy On scenic tours and longer journeys ample refreshment stops are provided when of course smoking is permitted

YOUR QUESTIONS ANSWERED

Do I need to book Booking in advance is strongly recommended as all Daybreak tours are subject to demand Subject to availability stand-by tickets can be purchased from the driver

What ti me does the coach leave The coach departs from Cambridge Drummer Street (Bay 12 adjacent to public toilets) at the time shown There are many additional joining points indicated by departure codes in the brochure If you are joining at one of our less popular joining points you will be adshyvised of your pick-up time (normally by telephone) not less than 48 hours before deparshyture In this way we can minimize the length of pick-up routes and reduce journey times for the majority of passengers

CHAPTER 4 Standardized Testing 99

What time do we get back An approximate return time is shown for each excursion The tim~s shown serve as a guide but road conditions can sometimes cause delay If your arrival will be later than advertised your driver will try to allow for a telephone call during the return journey

Where can I board the coach All the Daybreaks in the brochure leave from Cambridge Drummer Street (Bay 12 adjashycent to public toilets) at the time shown Many Daybreaks offer additional pick-ups for pre-booked passengers within Cambridge and the surrounding area This facility must be requested at the time of booking

Academic Writing Writing Task 1 You should spend about 20 minutes on this task

The graph below shows the different modes of transport used to travel to and from work in one European city in 1950 1970 and 1990

[graph shown here]

Write a report for a university lecturer describing the information shown below You should write at least 150 words

Writing Task 2 You should spend about 40 minutes on this task

Present a written argument or case to an educated reader with no specialist knowledge of the folowing topic

It is inevitable that as technology develops so traditional cultures must be lost Technolshyogy and tradition are incompatible-you cannot have both together

To what extent do you agree or disagree with this statement Give reasons for your answer You should write at least 250 words You should use your own ideas knowlshyedge and experience and support your arguments with examples and relevant evidence

General Training Writing Writing Task 1 You should spend about 20 minutes on this task You rent a house through an agency The heating system has stopped working You phoned the agency a week ago but it has still not been mended Write a letter to the agency Explain the situation and teil them what you want them to do about it

You should write at least 150 words You do NOT need to write your own address

Begin your letter as follows

Dear - ___-I

Writing Task 2 You should spend about 40 minutes on this task As part ofa class assignment you have to write about the following topic

100 CHAPTER 4 Standardized Testing

Some businesses now say that no one can smoke cigarettes in any of their offices Some governments have banned smoking in all public placesThis is a good idea but it takes away some of our freedom

Do you agree or disagree Give reasons for your answer You should write at least 250 words

Speaking In each ofthe three parts of the speaking module a specific function is fulfilled In Part 1 the candidates answer general questions about themselves their homes or families their jobs or studies their interests and a range ofsimilar familiar topic areas This part lasts between four and five minutes In Part 2 the candidate is given a verbal prompt on a card and is asked to talk on a particular topic The candidate has one minute to prepare before speaking at length for between one and two minutes The examiner then asks one or two wind-down questions In Part 3 the examiner and candidate engage in a discusshysion of more abstract issues and concepts which are thematically linked to the topic prompt in Part 2 The discussion lasts between four and five minutes

All interviews are recorded on audiocassette Here is a sample ofa Part 2 topic

Describe a teacher who has greatly influenced you in your education

You shou Id say

where you met them what subject they taught what was special about them

and explain why this person influenced you so much

You will have to talk about the topic for 1 to 2 minutes You have 1 minute to think about what you are going to say You can make some notes if you wish

Test of English for International Communication (TOEICreg)

listening

Part 1 Photographs Directions For each question you will see a picture in your test book and you will hear four short statements The statements will be spoken just one time They will not be printed in your test book so you must listen carefully to understand what the speaker says When you hear the four statements look at the picture in your test book and choose the statement that best describes what you see in the picture Then on your answer sheet find the number of the question and mark your answer

[photograph of a scientist looking through a microscope]

You will hear Look at the picture marked number 1 in your test book

(A) Shes speaking into a microphone (B) Shes put on her glasses (C) She has both eyes open (D) Shes using a microscope

CHAPTER 4 Standardized Testing 101

Part 2 Question-Response Directions In this part of the test you will hear a question or statement spoken in Enshyglish followed by three responses also spoken in English The question or staten1ent and the responses will be spoken just one time They will not be printed in your test book so you must listen carefully to understand what the speakers say You are to choose the best response to each question or statement

Question 1 You will hear Ms Morikawa has worked here for a long time hasnt she

(A) At three oclock (B) No Ive lost my watch (C) More than ten years

Question 2 You will hear Which of these papers has a wider circulation

(A) The morning edition (B) Get more exercise (C) By messenger

Part 3 Short Conversations Directions In this part of the test you will hear short conversations between two people The conversations will not be printed in your test book You will hear the conversations only once so you must listen carefully to understand what the speakers say In your test book you will read a question about each conversation The question will be followed by four answers You are to choose the best answer to each question and mark it on your answer sheet

Question 1 (Man) We should think about finding another restaurant for lunch (Woman) Why The food and service here are great

(Man) Yes but the prices are going up every week

You will read Why is this man unhappy with the restaurant

(A) It is too noisy (B) It is too expensive (C) It is too crowded (D) It is too difficult to find

Question 2 (Woman A) How was Dr Borgs recent trip to Singapore (Woman B) She enjoyed the tour of the port very much (Woman A) They say its one of the most active in Asia

You will read 2 What did Dr Borg find interesting

(A) The tourist center (B) The airport (C) The musical performance (D) The harbor

Part 4 Short Talks Directions In this part of the test you vill hear several short talks Each will be spoken just one time They will not be printed in your test book so you must listen carefully to understand and remember what is said In your test book you will read two or more questions about each short talk The questions will be followed by four answers You are to choose the best answer to each question and mark it on your answer sheet

102 CHAPTER 4 Standardized Testing

You will hear Questions 1 and 2 refer to the following announcement

Good afternoon and welcome aboard Nordair Flight 857 from Copenhagen to Bangkok with intermediate stops in Dubai and Calcutta We are preparing for departure in a few minutes At this time your seat back should be returned to its full upright position and your seat belt s~ould be fastened OUf anticipated total flying time to Dubai is six hours and twenty-five minutes I hope you enjoy the flight You will hecJr Now read question 1 in your test book and answer it You will read 1 What is the final destination of the flight

(A) Bangkok (B) Copenhagen (C) Dubai (O) Calcutta

You will hear Now read question 2 in your test book and answer it You will read 2 What will happen in a few minutes

(A) The flight will land in Dubai I

(B) The passengers will board the plane (C) The plane will take off (0) The gate number will be announced

Reading In this section of the test you will have the chance to show how well you understand written English There are three parts to this section with special directions for each part

Part 4 Incomplete Sentences Directions This part of the test has incomplete sentences Four words or phrases marked (A) (8) (e) (D) are given beneath each sentence You are to choose the one word or phrase that best completes the sentence Then on your answer sheet find the number of the question and mark your answer

1 Mr Yangs trip will __ him away from the office for ten days (A) withdraw (B) continue (C) retain (0) keep

2 The company that Marie DuBois started now sells __ products throughout the world (A) its (B) it (C) theirs (D) them

3 If your shipment is not delivered __ Tuesday you can request a full refund for the merchandise (A) at (B) by (C) within (D) while

CHAPTER 4 Standardized Testing 103

Part 6 Error Recognition Directions In this part ofthe test each sentence has four words or phrases underlined The four underlined parts of the sentence are marked (A) (B) (C) (D) You are to identify the one underlined word or phrase that should be corrected or rewritten Then on your answer sheet find the number of the question and mark your answer

1 The pamphlet contains some importance information about the current exhibit ABC D

2 No matter how Jong it taking to finish the annual report it must be done properly ABC D

3 The popularity of jogging appears to have decreased since the past couple of years ABC D

Part 7 Reading Comprehension Directions The questions in this part of the test are based on a selection of reading mateshyrials such as notices letters) forms newspaper and magazine articles) and advertisements You are to choose the one best answer (A) (B) (C) or (OJ to each quesshytion Then on your ariswefsheelfindthe number of the qUestion andmcirkyour answer Answer all questions following each reading selection on thebasis of what is stated or implied in that selection

The Museum ofTechnology is a hands-on museum designed for people to experience science at w()rk~ Visitors are encouraged to use test and handle the objects o~ display Special demonstrations are scheduled for the first and second Wednesdays of each month at 1330 Open Tuesday-Friday 1200-1630 Saturday 1000-1730 and Sunday 11 00-1630

1 When during the month can visitors see special demonstrations (A) Every weekend (B) The first two Wednesdays (C) One afternoon a week (D) Every other Wednesday

Questions 2 and 3 refer to the followi ng notice

NOTICE If you are unable to work because of an extended illness or injury that is not workshyrelated you may be entitled to receive weekly benefits from your employer or the firms insurance company To claim benefits you must file a claim form within thirty days of the first day of your disability Before filing the claim you must ask your doctor to fill in the Doctors Statement on the claim form stating the period of disability

3 To whom is this notice addressed (A) Employers (8) Doctors (C) Employees (D) When paying the bill

4 When must the claim form be filed (A) On the first of the month (8) On the thirtieth of the month (C) On the first day ofdisabifity (D) Within 30 days of the start of disability

Page 21: Standardized Testing Chapter 4 Brown

86 CHAPTER 4 Standardized Testing

Comments The University of Cambridge local Examinations Syndicate (UCLES) has been producing English language tests since 1858 Now with three organizations cooperatshying to form the IELTS more than a million examinations are administered every year In 2002 a computer-based version of the Reading and Writing modules of the IELTS became available at selected centers around the world The other sections are administered locally by an examinet The paper-based IELTS remains an option for candidates The IELTS retains the distinct advantage of requiring all four skills in the test-takers performance

Test of English for International Communication (TOEICreg)

Producer The Chauncey Group International a subsidiary of Educational Testing Service

Objelttive To test overall proficiency (langlJage ability) Primary market Worldwide business commerce and industry contexts

(workplace settings) Type Computer-based and paper-based versions Response modes Multiple-choice responses Time allocation 2 hours Internet access httpwwwtoeiccom

Specifications Listening Comprehension 100 items administered by audiocassette Four types of task statements questions short conversations and short talks (approxishymately 45 minutes) Reading 100 items Three types of task cloze sentences error recogshynition and reading comprehension (75 minutes)

Comments The TOEIC has become a very widely used international test of English proficiency in workplace settings where English is required for job performance The conshytent includes many different employment settings such as conferences presentations sales ordering shipping schedules reservations (etters and memoranda It is approprishyate to use in educational settings where vocational or workplace English courses are being offered

sect sect sect sect sect

The construction of a valid standardized test is no minor accomplishment whether the instrument is large- or small-scale The designing of specifications alone as this chapter illustrates requires a sophisticated process of construct valishydation coupled with considerations of practicality Then the construction of items and scoringinterpretation procedures may require a lengthy period of trial and error with prototypes of the final form of the testWith painstaking attention to all the details of construction the end product can result in a cost-effective timeshysaving accurate instrument Your use of the results of such assessments can provide useful data on learners language abilities But your caution is warranted as well for all the reasons discussed in this chapter The next chapter will elaborate on what lies behind that need for a cautious approach to standardized assessment

CHAPTER4 Standardized Testing 87

EXERCISES

[Note (I) Individual work (G) Group or pair work (C) Whole-class discussion]

1 (C) Tell the class about the worst test experience youve ever had Briefly anamiddot lyze what made the experience so unbearable and try to come up with sugshygestions for improvement of the test andor its administrative conditions

2 (G) In pairs or small groups compile a brief list of pros and cons of standardshyized testing Cite illustrations of as many items in each list as possible Report your lists and examples to the rest o~ the class

3 (I) Select a standardized test that you are quite familiar with (probably a recent experience) Mentally evaluate that test using the five principles of practicality reliability validity authenticity and washback Report yourevaluashytion to the class

4 (G) The appendix to this chapter provides sample items from Jour different tests of language proficiency In groups one test for each group analyze your test for (a) content validity (b) face validity and (c) authenticity

5 (C) Do you think that the sample TOEFL reading passage about pirates (pages 74-75) and the Graduate EssayTest prompt (pages 76-77) about a school board hiring committee have any culture bias Discuss this and other cultural biases you have noticed in tests Is it possible to design a test that is completely free of culture bias

6 (CG) Compare the differences in conceptualization of language proficiency represented by Swains model the TOEFL and the ACfFL philosophy Which one best represents current thinking about communicative language ability What are the strengths and weaknesses of each approach

FORYOlIILEURTHER READING

Gronlund Norman E (1998) Assessment of student achievement Sixth Edition Boston Allyn and Bacon

Gronlunds classic also mentioned in Chapter 3 offers a concise overview of features of standardized tests offering definitions and examples of the statistical considerations in interpreting scores His approach is unbiased cleady written and accessible to those who might fear the mathematics of standardized testing

Phillips Deborah 2001 Long1nan introductory course for the TOEFL test White Plains NY Pearson Education

A careful examination of this or any other reputable preparation course for a standardized language test is well worth a students time Note especially how the book acquaints the user with the specifications of the test and offers a number of useful strategie~ that can be llsed in preparation for the test and during irs adn1inistration

88 CHAPTER 4 Standardized Testing

APPENDIX TO CHAPTER 4

Commercial Proficiency Tests Sample Items and Tasks

Test of English a~ a Foreign Language (TOEFLreg)

Listening r

Part A

In this section you will hear short conversations between two people In some ofthe conversations each person speaks only once In other conversations one or both of the people speak more than once Each conversation is followed by one questionabQlt it Each question in this part has four answer choices You should click on the best answer to each question Answer the questions on the basis of what is stated or implied by the speakers Here is an example On the computerscreen you will see

[man and woman talking]

On the recording you will hear

(woman) Hey wheres your sociology book (man) At home Why carry it around when were just going to be taking

a test (woman) Dont you remember Professor Smith said we could us it during

the test (man) Ohl no Well Ive still got an hour right Im so glad I ran into you

You wiII then see and hear the question before the answer choices appear

What will the man probably do next

o Begin studying for the sociology test o Explain the problem to his professor o Go home to get his textbook o Borrow the womans book

To choose an answer you will click on an oval The oval next to that answer will darken After you click on Next and Confirm Answer the next conversation will be presented

Part B

In this section you will hear several longer conversations and talks Each conversation or talk is followed by several questions The conversations talks and questions will not be repeated The conversations and talks are about a variety of topics You do not need speshycial knowledge of the topics to answer the questions correctly Rather you should answer each question on the basis of what is stated or implied by the speakers in the conversashytions or talks

For most of the questions you will need to click on the best of four possible answers Some questions will have special directions The special directions will appear in a box on the computer screen Here is an exampie ot a conversation and some questions

CHAPTER 4 Standardized Testing 89

Marine Biology (narrator) Listen to part of a discussion in a marine biology class

(professor) A few years ago our local government passed a number of strict environmental laws As a result Sunrise Beach looks nothing Ii ke it did ten years ago The water is cleaner and theres been a tremendous increase in all kinds of marine life which is why were going there on Thursday

(woman) I dont know if I agree that the water quality has improved I mean I was out there last weekend and it looked all brown It didnt seem too clean to me

(professor) Actually the color of the water doesnt always indicate whether its polluted The brown color you mentioned might be a result of pollution or it can mean a kind of brown algae is growing there Its called devils apron and it actually serves as food for whales

(man) So when does the water look blue (professor) Well water thats completely unpolluted is actually colorless But

it often looks bluish-green because the sunlight can penetrate deep down and thats the color thats reflected

(woman) But sometimes it looks really green Whats that about (professor) Ok well its the same principle as with devils apron the

water might be green because of different types of green algae there-gulfweed phytoplankton You all should finish reading about algae and plankton before we go In fact those are the types of living things Im going to ask you to be looking for when were there

Now get ready to answer the questions

What is the discussion mainly about

o The importance of protecting ocean environments o The reasons why ocean water appears to be different colors o The survival of whales in polluted water o The effect that colored ocean water has on algae

To choose an answer click on an oval The oval next to that answer will darken After you click on Next and Confirm Answer the next question will be presented

According to the professor what can make ocean water look browngt

o Pollution o Cloudy Skies o Sand o Algae

Click on 2 answers

To choose your answers you will click on the squares An XII wiii appear in each square

bullbullbullbullbullbullbull

90 CHAPTER 4 Standardized Testing

Structure and Written Expression This section measures the ability to recognize language that is appropriate for standard written English There are two types ofquestions in this section In the first type ofquestion there are incomplete sentences Beneath each sentence there are four words or phrases

Directions CIiSk on the one word or phrase that best completes the sentence

The colum~ine flower __ to nearly all of the United States can be raised from seed in almost any garden

native how native is how native is it is native

Time Help Confirm

After you click on Next and Confirm Answ~ the next question willbe presented

The second type of question has four underlined words or phrases You will choose the one underlined word or phrase that must be changed for the sentence to be correct

Directions Click on the one underlined word or phrase that must be changed for the senshytence to be correct

One of the most difficult problems in understanding sleep is determining what the funcshytions of sleep ~

lime Help Confirm

Clicking on an underlined word or phrase will darken it

Reading This section measures the ability to read and understand short passages similar in topic and style to those that students are likely to encounter in North American universities and colleges This section contains reading passages and questions about the passages There are several different types of questions in this section In the Reading section you will first have the opportunity to read the passage

The temperature of the Sun is over 10000 degrees Fahrenheit at the surface but it rises perhaps more than 270000000 at the center The Sun is so much hotter than the Earth that matter can exist only as a gasl except perhaps at the core In the core of the Sun the pressures are so great that despite the high temperature there may be a small solid core However no one really knows since the center of the Sun can never be directly observed ~ Solar astronomers do know that the Sun is divided into five general layers or zones Starting at the outside and going down into the Sun the zones are the corona chromoshysphere hotosphere convection zone and finally the core The first three zones are reshygarded as the Suns atmosphere But since the Sun has no solid surface it is hard to middottell where the atmosphere ends and the main body of the Sun begins

The Suns outermost layer begins about 10000 miles above the visible surface and goes outward for millions of miles This is the only part of the Sun that can be seen during an eclipse such as the one in February 1979 At any other time the corona can be seen

bullbullbullbullbullbullbull

bull bullbullbullbullbullbull

CHAPTER 4 Standardized Testing 91

only when special instruments are used on cameras and telescopes to block the light from the photosphere

The corona is a brilliant pearly white filmy light about as bright as the full Moon Its beautiful rays are a sensational sight during an eclipse The coronas rays flash out in a brilliant fan that has wispy spikelike rays near the Suns north and south poles The corona is generally thickest at the Suns equator The corona is made up of gases streamshying outward at tremendous speeds that reach a temperature of more than 2 million deshygrees Fahrenheit The gas thins out as it reaches the space around the planets By the time the gas of the corona reaches the Earth it has a relatively low density

When you have finished reading the passage you will use the mouse to click on Proceed Then the questions about the passage will be presented You are to choose the one best anshyswer to each question Answer all questions about the information in a passage on the basis ofwhat is stated or implied in that passage Most ofthe questions will be multiple-choice questions To answer these questions you will click on a choice below the question

With what topic is paragraph 2 mainly concerned

o How the Sun evolved o The structure of the Sun o Why scientists study the Sun o The distaflce of the Sun from the planets

Paragraph 2 is marked with an arrow (~)

You will see the next question after you click on Next

To answer some questions you will click on a word or phrase Here is an example

Look at the word one in the passage Click on the word or phrase in the bold text that one refers to To answer you can click on any part of the word or phrase in the passage Jour choice will darken to show which word you have chosen

The Suns outermost layer begins about 10000 miles above the visible surface and goes outward for millions of miles This is the only part of the Sun that can be seen durshying an eclipse such as the one in February 1979 At any other time the corona can be seen only when special instruments are used on cameras and telescopes to block the Iight from the photosphere

You will see the next question after you click on~ To answer some q~estions you will click on a sentence in the passage Here is an example

~ The corona is a brilliant pearly white filmy light about as bright as the full Moon Its beautiful rays are a sensational sight during an eclipse The coronas rays flash out in a brilliant fan that has wispy spikelike rays near the Suns north and south poles The corona is generally thickest at the Suns equator ~ The corona is made up of gases streaming outward at tremendous speeds that reach a temperature of more than 2 million degrees Fahrenheit The gas thins out as it reaches the space around the planets By the time the gas of the corona reaches the Earth it has a relatively low density

bull bullbullbullbullbullbull

92 CHAPTER 4 Standardized Testing

Click on the sentence in paragraph 4 or 5 in which the author compares the light of the Suns outermost layer to that ofanother astronomical body Paragraphs 4 and 5 are marked with arrows (~)

To answer some questions you will click on a square to add a sentence to the passage Here is an example -The following sentence can be added to paragraph 1

At the center of the Earths solar system lies the Sun

Where would it best fit in paragraph I Click on the square to add the sentence to the paragraph

D The temperature of the Sun is over 10000 degrees Fahrenheit at the surface but it rises to perhaps morethan 27000000deg at the center 0 The Sun is so much hotter than the Earth that matter can exist only as a gasi except p~rHapsatth~ c6relp the c~re of the ii Sun the pressures are so great that despite the high temperature there may be a small solid core D However no one really knows since the center of the Sun can never be directly observed D 0100

When you click on a square the sentence will appear in the passage at the place you have chosen You can read the sentence added to the paragraph to see if this is the best place to add it You can click on another square to change your answer The sentence will be added and shown in a dark box

Writing In this section you will have an opportunity to demonstrate your ability to write in Enshyglish This includes the ability to generate and organize ideas to support those ideas with examples or evidence and to compose in standard written English in response to an asshysigned topic You will have 30 minutes to write your essay on that topic You must write on the topic you are assigned An essay on any other topic will receive a score of 0 Read the topic below and then make any notes that will help you plan your response Begin typing your response in the box at the bottom of the screen or write your answer on the answer sheet provided to you

Following is a sample topic

Do you agree or disagree with the following statemenH

Teachers should make learning enjoyable and fun for their students

Use specific reasons and examples to support your opinion

CHAPTER 4 Standarczed Testing 93

Michigan English Language Assessment Battery (MELAB)

Composition The time limit for the composition is 30 minutes You must write on only one of the top~

ics below If you write about something else your composition paper will not be graded and you cannot be given a final score If you do not understand the topics ask the exam~ iner to explain or to translate them You may be asked to give your opinion ofsomething and explain why you believe this to describe something from your experience or to exshyplain a problem and offer possible solutions You should write at least one page Some sample topics are

1 What do you think is your countrys greatest problem Explain in detail and tell what you think can be done about it

2 What are the characteristics of a good teacher Explain and give examples 3 An optimist is someone who sees the good side of things A pessimist sees the

bad side Are you an optimist or a pessimist Relate a personal experience that shows this

4 In your opinion are the benefits of space exploration really worth the enormous costs Discuss

Most MELAB compositions are one or two pages long (about 200-300 words) If your paper is extremely short (less than 150 words) your composition will be given a lower score Before you begin writing you might want to take 2 or 3 minutes to plan your comshyposition and to make a short outline to organize your thoughts Such outlines will not be graded they are only to help you You should use the last 5 minutes to read through your composition and to make changes or corrections

Your composition will be graded on how clearly you express yourself in English and on the range of English you are able to use and your control in doing so This means your composition should be well organized your arguments should be fully developed and you should show a range ofgrammatical structures and broad vocabulary Compositions that consist only of very short sentences and very simple vocabulary cannot be given the

middothighest scores If errors are not frequent and if they do not confuse your meaning they will not lower your score very much

Listening Now you will hear a short lecture You may take notes during the lecture Following the lecture you will be asked some questions about it

Therell be a two-week exhibit of the paintings of the little-known master Laura Bernhart at the Claire Osmond Galleries starting on the fifteenth of the month and running through the thirtieth Bernharts known for her innovative designs in abstract expressionism Though a true original she declared a spiritual heritage from Salvador Dali the famous Spanish painter Since Bernhart lived a rather solitary life and died while only in her twenties few people are aware of her works This showing at the Osmond Galleries will provide many with an introduction to her works

10 Where is the exhibit a the Art Museum b the Dali Galleries c the Osmond Galleries

94 CHAPTER 4 Standardized Testing

11 What is Bernhart known for a her copies of Dalis paintings b the originality of her designs c her exhibitions

12 What will going to the exhibit allow most people to do a to see Saivador Dalis paintings b to see Bernharts works for the first time c to learn about Spanish art

Grammar

1 What did the teacher just tell you

She reminded our notebooksI a us to bring b that we bring c our bringing d we should bring

2 Is Bill a good dancer

Not really __ he tries very hard a in spite of h despite c even though d while

3 your clothes are all wet1

Yes I didnt come __ the rain soon enough a away to b over to c down with d in from

Cloze In years to come zoos will not only be places where animals are exhibited to the public but repositories where rare species can be saved from extinction (7) captive breeding The most powerful force (8) the future of many animals-and of zoos-is the decline of the wild (9) even zoo directors would argue that (10) are better places for animals than the fields and forest of their native (11) yet zoos may be the last chance for some creatures that would otherwise pass qUietly into oblivion

7 a through c from b of d damage

8 a bringing c to b that d influencing

9 a But c Not b So d Then

10 a where c even b zoos d wilds

11 alands c residence b life d field

CHAPTER 4 Standardized Testing 95

Vocabulary

12 Mark has a flair for writing a need b purpose c talent d dislike

13 Bill Collins launched his restaurant last June a moved b started c sold d bought

14 John will not accept the censure a burden b blame c credit d decision

15 I cant think of the answer Can you give me a __ a hint b token c taste d gaze

16 Because fewer people are taking expensive vacations the tourist industry is in a a choke b grope c grumble d slump

17 I disagree with a few of his opinions but __ we agree a deliberately b conclusively c essentially d immensely

Reading The influenza virus is a single molecule built from many millions of single atoms You must have heard of the viruses which are sometimes called living molecules While bacteria can be considered as a type of plant secreting pOisonous substances into the body of the organism they attack viruses are living organisms themselves We may conshysider them as regular chemical molecules since they have a strictly aefined atomic strucshyture but on the other hand we must also consider them as being alive since they are able to multiply in unlimited quantities

18 According to the passage bacteria are a poisons

b larger than viruses c very small d plants

96 CHAPTER 4 Standardized Testing

19 The writer says that viruses are alive because they a have a complex atomic structure b move c multiply d need warmth and light

20 The atomic structure of viruses a is -tJIariable b is strictly defined c cannot be analyzed chemically d is more complex than that of bacteria

International English Language Testing System (fELTS)

I

listening

The Listening Module has four sections The first two sections are concerned with social needs There is a conversation between two speakers and then a monologue For examshyple a conversation about travel arrangements or decisions on a night out and a speech about student services on a university campus or arrangements for meals during a confershyence The final two sections are concerned with situations related more closely to educashytional or training contexts For example conversation between a tutor and a student about an assignment or between three students planning a research project and a lecture or talk ofgeneral academic interest All the topics are ofgeneral interest and it makes no difference what subjects candidates study Tests and tasks become more difficult as the sections progress A range of English accents and dialects are used in the recording which reflects the international usage of IELTS

Academic Reading [A 7S0-word article on-th-e- topic of Wind Power in the US with a short glossary at the end]

Questions 1-5

Complete the summary below

Choose your answers from the box below the summary and write them in boxes 1-5 on your answer sheet Note There are more words or phrases than you will need to fill the gaps You may use any word or phrase more than once

Example The failure during the late 1970s and early 19805 of an attempt to establish a widespread wind power industry in the United States resulted largely from the (1) bull in oil prices during this period The industry is now experiencing a steady (2) due to improveshyments in technology and an increased awareness of the potential in the power of wind The wind turbines that are now being made based in part on the (3) of wide- ranging research in Europe are easier to manufacture and maintain than their predecesshysors This has led wind-turbine makers to be able to standardise and thus minimize (4) There has been growing (S) of the importance of wind power as an energy source

CHAPTER 4 Standardized Testing 97

criticism stability skepticism success operating costs decisions design costs fall effects production costs growth decline failure recognition results

Questions 6-1 0 Look at the following list of issues (Questions 6-10) and implications (A-C) Match each issue with one implication Write the appropriate letters A-C in boxes 6-10 on your anshyswer sheet

Example The current price of one wind-generated kilowatt Answer

6 The recent installation of systems taking advantage of economies of scale

7 The potential of meeting one fifth of current U5 energy requirements by wind power

8 The level of acceptance of current wind turbine technology

9 A comparison of costs between conventional and wind power sources

10 The view of wind power in the European Union

Implications

A provides evidence against claims that electricity produced from wind power is relatively expensive

B supports claims that wind power js an important source of energy

C opposes the view that wind power technology requires further-development

General Training Reading Read the passage on Daybreak trips by coach and look at the statements below On your answer sheet write

TRUE if the statement is true FALSE jf the statement is false

NOlGIVEN if the information is not given in the leaflet

1 MiIlers Coaches owns Cambridges Cam bus fleet

2 Premier is an older company than Millers

3 Most of the Daybreak coaches are less than 5 years old

4 Daybreak fares are more expensive than most of their competitors

5 Soft drinks and refreshments are served on most longer journeys

6 Smoking is permitted at the rear of the coach on longer journeys

7 Tickets must be bought in advance from an authorised Daybreak agent

6 Tickets and seats can be reserved by phoning the Daybreak Hotline

9 Daybreak passengers must join their coach at Cambridge Drummer Street

10 Daybreak cannot guarantee return times

98 CHAPTER 4 Standardized Testing

FROM CAMBRIDGE AND SURROUNDING AREA

SPRING IS INTHEAIR

Welcome to our Spring Daybreak programme which continues the tradition of offering unbeatable value for money day trips and tours All the excursions in this brochure will be operated by Pr~mier Travel Services Limited or Millers Coaches both companies are part of the CHLGroup owners of Cambridges Cambus fleet

WERE PROUD OF OUR TRADITION

Premier was established in 1936 the Company now offers the highest standards of coaching in todays competitive operating environment Miller has an enviable reputation stretching back over the past 20 years offering coach services at realistic prices Weve traveled a long way since our early days of pre-war seaside trips Now our fleet of 50 modern coaches (few are more than five years old) operate throughout Britain and Europe but were pleased to still maintain the high standards of quality and service the trademark of our founders nearly sixty years ago

EXCLUSIVE FEATURES

Admission-inclusive fares All Daybreak fares (unless specifically otherwise stated) include admission charges to the attractions shows and exhibits we visit Many full-day scenic tours are accompanied by a fully trained English Tourist Board Blue Badge guide or local experienced driverguide Some Daybreaks include lunch or afternoon tea Compare our admission inclusive fares and see how much you save Cheapest is not the best and value for money is guaranteed If you compare our bargain Daybreak fares beware--most of our competishytors do not offer an all-inclusive fare

SEAT RESERVATIONS

We value the freedom of choice so you can choose your seat when you book The seat reservation is guaranteed a-nd remains yours at all times when aboard the coach

NO SMOKING COMFORT

With the comfort of our passengers in mind coaches on all our Daybreaks are no smokshying throughout In the interests of fellow passengers comfort we kindly ask that smokers observe our no smoking policy On scenic tours and longer journeys ample refreshment stops are provided when of course smoking is permitted

YOUR QUESTIONS ANSWERED

Do I need to book Booking in advance is strongly recommended as all Daybreak tours are subject to demand Subject to availability stand-by tickets can be purchased from the driver

What ti me does the coach leave The coach departs from Cambridge Drummer Street (Bay 12 adjacent to public toilets) at the time shown There are many additional joining points indicated by departure codes in the brochure If you are joining at one of our less popular joining points you will be adshyvised of your pick-up time (normally by telephone) not less than 48 hours before deparshyture In this way we can minimize the length of pick-up routes and reduce journey times for the majority of passengers

CHAPTER 4 Standardized Testing 99

What time do we get back An approximate return time is shown for each excursion The tim~s shown serve as a guide but road conditions can sometimes cause delay If your arrival will be later than advertised your driver will try to allow for a telephone call during the return journey

Where can I board the coach All the Daybreaks in the brochure leave from Cambridge Drummer Street (Bay 12 adjashycent to public toilets) at the time shown Many Daybreaks offer additional pick-ups for pre-booked passengers within Cambridge and the surrounding area This facility must be requested at the time of booking

Academic Writing Writing Task 1 You should spend about 20 minutes on this task

The graph below shows the different modes of transport used to travel to and from work in one European city in 1950 1970 and 1990

[graph shown here]

Write a report for a university lecturer describing the information shown below You should write at least 150 words

Writing Task 2 You should spend about 40 minutes on this task

Present a written argument or case to an educated reader with no specialist knowledge of the folowing topic

It is inevitable that as technology develops so traditional cultures must be lost Technolshyogy and tradition are incompatible-you cannot have both together

To what extent do you agree or disagree with this statement Give reasons for your answer You should write at least 250 words You should use your own ideas knowlshyedge and experience and support your arguments with examples and relevant evidence

General Training Writing Writing Task 1 You should spend about 20 minutes on this task You rent a house through an agency The heating system has stopped working You phoned the agency a week ago but it has still not been mended Write a letter to the agency Explain the situation and teil them what you want them to do about it

You should write at least 150 words You do NOT need to write your own address

Begin your letter as follows

Dear - ___-I

Writing Task 2 You should spend about 40 minutes on this task As part ofa class assignment you have to write about the following topic

100 CHAPTER 4 Standardized Testing

Some businesses now say that no one can smoke cigarettes in any of their offices Some governments have banned smoking in all public placesThis is a good idea but it takes away some of our freedom

Do you agree or disagree Give reasons for your answer You should write at least 250 words

Speaking In each ofthe three parts of the speaking module a specific function is fulfilled In Part 1 the candidates answer general questions about themselves their homes or families their jobs or studies their interests and a range ofsimilar familiar topic areas This part lasts between four and five minutes In Part 2 the candidate is given a verbal prompt on a card and is asked to talk on a particular topic The candidate has one minute to prepare before speaking at length for between one and two minutes The examiner then asks one or two wind-down questions In Part 3 the examiner and candidate engage in a discusshysion of more abstract issues and concepts which are thematically linked to the topic prompt in Part 2 The discussion lasts between four and five minutes

All interviews are recorded on audiocassette Here is a sample ofa Part 2 topic

Describe a teacher who has greatly influenced you in your education

You shou Id say

where you met them what subject they taught what was special about them

and explain why this person influenced you so much

You will have to talk about the topic for 1 to 2 minutes You have 1 minute to think about what you are going to say You can make some notes if you wish

Test of English for International Communication (TOEICreg)

listening

Part 1 Photographs Directions For each question you will see a picture in your test book and you will hear four short statements The statements will be spoken just one time They will not be printed in your test book so you must listen carefully to understand what the speaker says When you hear the four statements look at the picture in your test book and choose the statement that best describes what you see in the picture Then on your answer sheet find the number of the question and mark your answer

[photograph of a scientist looking through a microscope]

You will hear Look at the picture marked number 1 in your test book

(A) Shes speaking into a microphone (B) Shes put on her glasses (C) She has both eyes open (D) Shes using a microscope

CHAPTER 4 Standardized Testing 101

Part 2 Question-Response Directions In this part of the test you will hear a question or statement spoken in Enshyglish followed by three responses also spoken in English The question or staten1ent and the responses will be spoken just one time They will not be printed in your test book so you must listen carefully to understand what the speakers say You are to choose the best response to each question or statement

Question 1 You will hear Ms Morikawa has worked here for a long time hasnt she

(A) At three oclock (B) No Ive lost my watch (C) More than ten years

Question 2 You will hear Which of these papers has a wider circulation

(A) The morning edition (B) Get more exercise (C) By messenger

Part 3 Short Conversations Directions In this part of the test you will hear short conversations between two people The conversations will not be printed in your test book You will hear the conversations only once so you must listen carefully to understand what the speakers say In your test book you will read a question about each conversation The question will be followed by four answers You are to choose the best answer to each question and mark it on your answer sheet

Question 1 (Man) We should think about finding another restaurant for lunch (Woman) Why The food and service here are great

(Man) Yes but the prices are going up every week

You will read Why is this man unhappy with the restaurant

(A) It is too noisy (B) It is too expensive (C) It is too crowded (D) It is too difficult to find

Question 2 (Woman A) How was Dr Borgs recent trip to Singapore (Woman B) She enjoyed the tour of the port very much (Woman A) They say its one of the most active in Asia

You will read 2 What did Dr Borg find interesting

(A) The tourist center (B) The airport (C) The musical performance (D) The harbor

Part 4 Short Talks Directions In this part of the test you vill hear several short talks Each will be spoken just one time They will not be printed in your test book so you must listen carefully to understand and remember what is said In your test book you will read two or more questions about each short talk The questions will be followed by four answers You are to choose the best answer to each question and mark it on your answer sheet

102 CHAPTER 4 Standardized Testing

You will hear Questions 1 and 2 refer to the following announcement

Good afternoon and welcome aboard Nordair Flight 857 from Copenhagen to Bangkok with intermediate stops in Dubai and Calcutta We are preparing for departure in a few minutes At this time your seat back should be returned to its full upright position and your seat belt s~ould be fastened OUf anticipated total flying time to Dubai is six hours and twenty-five minutes I hope you enjoy the flight You will hecJr Now read question 1 in your test book and answer it You will read 1 What is the final destination of the flight

(A) Bangkok (B) Copenhagen (C) Dubai (O) Calcutta

You will hear Now read question 2 in your test book and answer it You will read 2 What will happen in a few minutes

(A) The flight will land in Dubai I

(B) The passengers will board the plane (C) The plane will take off (0) The gate number will be announced

Reading In this section of the test you will have the chance to show how well you understand written English There are three parts to this section with special directions for each part

Part 4 Incomplete Sentences Directions This part of the test has incomplete sentences Four words or phrases marked (A) (8) (e) (D) are given beneath each sentence You are to choose the one word or phrase that best completes the sentence Then on your answer sheet find the number of the question and mark your answer

1 Mr Yangs trip will __ him away from the office for ten days (A) withdraw (B) continue (C) retain (0) keep

2 The company that Marie DuBois started now sells __ products throughout the world (A) its (B) it (C) theirs (D) them

3 If your shipment is not delivered __ Tuesday you can request a full refund for the merchandise (A) at (B) by (C) within (D) while

CHAPTER 4 Standardized Testing 103

Part 6 Error Recognition Directions In this part ofthe test each sentence has four words or phrases underlined The four underlined parts of the sentence are marked (A) (B) (C) (D) You are to identify the one underlined word or phrase that should be corrected or rewritten Then on your answer sheet find the number of the question and mark your answer

1 The pamphlet contains some importance information about the current exhibit ABC D

2 No matter how Jong it taking to finish the annual report it must be done properly ABC D

3 The popularity of jogging appears to have decreased since the past couple of years ABC D

Part 7 Reading Comprehension Directions The questions in this part of the test are based on a selection of reading mateshyrials such as notices letters) forms newspaper and magazine articles) and advertisements You are to choose the one best answer (A) (B) (C) or (OJ to each quesshytion Then on your ariswefsheelfindthe number of the qUestion andmcirkyour answer Answer all questions following each reading selection on thebasis of what is stated or implied in that selection

The Museum ofTechnology is a hands-on museum designed for people to experience science at w()rk~ Visitors are encouraged to use test and handle the objects o~ display Special demonstrations are scheduled for the first and second Wednesdays of each month at 1330 Open Tuesday-Friday 1200-1630 Saturday 1000-1730 and Sunday 11 00-1630

1 When during the month can visitors see special demonstrations (A) Every weekend (B) The first two Wednesdays (C) One afternoon a week (D) Every other Wednesday

Questions 2 and 3 refer to the followi ng notice

NOTICE If you are unable to work because of an extended illness or injury that is not workshyrelated you may be entitled to receive weekly benefits from your employer or the firms insurance company To claim benefits you must file a claim form within thirty days of the first day of your disability Before filing the claim you must ask your doctor to fill in the Doctors Statement on the claim form stating the period of disability

3 To whom is this notice addressed (A) Employers (8) Doctors (C) Employees (D) When paying the bill

4 When must the claim form be filed (A) On the first of the month (8) On the thirtieth of the month (C) On the first day ofdisabifity (D) Within 30 days of the start of disability

Page 22: Standardized Testing Chapter 4 Brown

CHAPTER4 Standardized Testing 87

EXERCISES

[Note (I) Individual work (G) Group or pair work (C) Whole-class discussion]

1 (C) Tell the class about the worst test experience youve ever had Briefly anamiddot lyze what made the experience so unbearable and try to come up with sugshygestions for improvement of the test andor its administrative conditions

2 (G) In pairs or small groups compile a brief list of pros and cons of standardshyized testing Cite illustrations of as many items in each list as possible Report your lists and examples to the rest o~ the class

3 (I) Select a standardized test that you are quite familiar with (probably a recent experience) Mentally evaluate that test using the five principles of practicality reliability validity authenticity and washback Report yourevaluashytion to the class

4 (G) The appendix to this chapter provides sample items from Jour different tests of language proficiency In groups one test for each group analyze your test for (a) content validity (b) face validity and (c) authenticity

5 (C) Do you think that the sample TOEFL reading passage about pirates (pages 74-75) and the Graduate EssayTest prompt (pages 76-77) about a school board hiring committee have any culture bias Discuss this and other cultural biases you have noticed in tests Is it possible to design a test that is completely free of culture bias

6 (CG) Compare the differences in conceptualization of language proficiency represented by Swains model the TOEFL and the ACfFL philosophy Which one best represents current thinking about communicative language ability What are the strengths and weaknesses of each approach

FORYOlIILEURTHER READING

Gronlund Norman E (1998) Assessment of student achievement Sixth Edition Boston Allyn and Bacon

Gronlunds classic also mentioned in Chapter 3 offers a concise overview of features of standardized tests offering definitions and examples of the statistical considerations in interpreting scores His approach is unbiased cleady written and accessible to those who might fear the mathematics of standardized testing

Phillips Deborah 2001 Long1nan introductory course for the TOEFL test White Plains NY Pearson Education

A careful examination of this or any other reputable preparation course for a standardized language test is well worth a students time Note especially how the book acquaints the user with the specifications of the test and offers a number of useful strategie~ that can be llsed in preparation for the test and during irs adn1inistration

88 CHAPTER 4 Standardized Testing

APPENDIX TO CHAPTER 4

Commercial Proficiency Tests Sample Items and Tasks

Test of English a~ a Foreign Language (TOEFLreg)

Listening r

Part A

In this section you will hear short conversations between two people In some ofthe conversations each person speaks only once In other conversations one or both of the people speak more than once Each conversation is followed by one questionabQlt it Each question in this part has four answer choices You should click on the best answer to each question Answer the questions on the basis of what is stated or implied by the speakers Here is an example On the computerscreen you will see

[man and woman talking]

On the recording you will hear

(woman) Hey wheres your sociology book (man) At home Why carry it around when were just going to be taking

a test (woman) Dont you remember Professor Smith said we could us it during

the test (man) Ohl no Well Ive still got an hour right Im so glad I ran into you

You wiII then see and hear the question before the answer choices appear

What will the man probably do next

o Begin studying for the sociology test o Explain the problem to his professor o Go home to get his textbook o Borrow the womans book

To choose an answer you will click on an oval The oval next to that answer will darken After you click on Next and Confirm Answer the next conversation will be presented

Part B

In this section you will hear several longer conversations and talks Each conversation or talk is followed by several questions The conversations talks and questions will not be repeated The conversations and talks are about a variety of topics You do not need speshycial knowledge of the topics to answer the questions correctly Rather you should answer each question on the basis of what is stated or implied by the speakers in the conversashytions or talks

For most of the questions you will need to click on the best of four possible answers Some questions will have special directions The special directions will appear in a box on the computer screen Here is an exampie ot a conversation and some questions

CHAPTER 4 Standardized Testing 89

Marine Biology (narrator) Listen to part of a discussion in a marine biology class

(professor) A few years ago our local government passed a number of strict environmental laws As a result Sunrise Beach looks nothing Ii ke it did ten years ago The water is cleaner and theres been a tremendous increase in all kinds of marine life which is why were going there on Thursday

(woman) I dont know if I agree that the water quality has improved I mean I was out there last weekend and it looked all brown It didnt seem too clean to me

(professor) Actually the color of the water doesnt always indicate whether its polluted The brown color you mentioned might be a result of pollution or it can mean a kind of brown algae is growing there Its called devils apron and it actually serves as food for whales

(man) So when does the water look blue (professor) Well water thats completely unpolluted is actually colorless But

it often looks bluish-green because the sunlight can penetrate deep down and thats the color thats reflected

(woman) But sometimes it looks really green Whats that about (professor) Ok well its the same principle as with devils apron the

water might be green because of different types of green algae there-gulfweed phytoplankton You all should finish reading about algae and plankton before we go In fact those are the types of living things Im going to ask you to be looking for when were there

Now get ready to answer the questions

What is the discussion mainly about

o The importance of protecting ocean environments o The reasons why ocean water appears to be different colors o The survival of whales in polluted water o The effect that colored ocean water has on algae

To choose an answer click on an oval The oval next to that answer will darken After you click on Next and Confirm Answer the next question will be presented

According to the professor what can make ocean water look browngt

o Pollution o Cloudy Skies o Sand o Algae

Click on 2 answers

To choose your answers you will click on the squares An XII wiii appear in each square

bullbullbullbullbullbullbull

90 CHAPTER 4 Standardized Testing

Structure and Written Expression This section measures the ability to recognize language that is appropriate for standard written English There are two types ofquestions in this section In the first type ofquestion there are incomplete sentences Beneath each sentence there are four words or phrases

Directions CIiSk on the one word or phrase that best completes the sentence

The colum~ine flower __ to nearly all of the United States can be raised from seed in almost any garden

native how native is how native is it is native

Time Help Confirm

After you click on Next and Confirm Answ~ the next question willbe presented

The second type of question has four underlined words or phrases You will choose the one underlined word or phrase that must be changed for the sentence to be correct

Directions Click on the one underlined word or phrase that must be changed for the senshytence to be correct

One of the most difficult problems in understanding sleep is determining what the funcshytions of sleep ~

lime Help Confirm

Clicking on an underlined word or phrase will darken it

Reading This section measures the ability to read and understand short passages similar in topic and style to those that students are likely to encounter in North American universities and colleges This section contains reading passages and questions about the passages There are several different types of questions in this section In the Reading section you will first have the opportunity to read the passage

The temperature of the Sun is over 10000 degrees Fahrenheit at the surface but it rises perhaps more than 270000000 at the center The Sun is so much hotter than the Earth that matter can exist only as a gasl except perhaps at the core In the core of the Sun the pressures are so great that despite the high temperature there may be a small solid core However no one really knows since the center of the Sun can never be directly observed ~ Solar astronomers do know that the Sun is divided into five general layers or zones Starting at the outside and going down into the Sun the zones are the corona chromoshysphere hotosphere convection zone and finally the core The first three zones are reshygarded as the Suns atmosphere But since the Sun has no solid surface it is hard to middottell where the atmosphere ends and the main body of the Sun begins

The Suns outermost layer begins about 10000 miles above the visible surface and goes outward for millions of miles This is the only part of the Sun that can be seen during an eclipse such as the one in February 1979 At any other time the corona can be seen

bullbullbullbullbullbullbull

bull bullbullbullbullbullbull

CHAPTER 4 Standardized Testing 91

only when special instruments are used on cameras and telescopes to block the light from the photosphere

The corona is a brilliant pearly white filmy light about as bright as the full Moon Its beautiful rays are a sensational sight during an eclipse The coronas rays flash out in a brilliant fan that has wispy spikelike rays near the Suns north and south poles The corona is generally thickest at the Suns equator The corona is made up of gases streamshying outward at tremendous speeds that reach a temperature of more than 2 million deshygrees Fahrenheit The gas thins out as it reaches the space around the planets By the time the gas of the corona reaches the Earth it has a relatively low density

When you have finished reading the passage you will use the mouse to click on Proceed Then the questions about the passage will be presented You are to choose the one best anshyswer to each question Answer all questions about the information in a passage on the basis ofwhat is stated or implied in that passage Most ofthe questions will be multiple-choice questions To answer these questions you will click on a choice below the question

With what topic is paragraph 2 mainly concerned

o How the Sun evolved o The structure of the Sun o Why scientists study the Sun o The distaflce of the Sun from the planets

Paragraph 2 is marked with an arrow (~)

You will see the next question after you click on Next

To answer some questions you will click on a word or phrase Here is an example

Look at the word one in the passage Click on the word or phrase in the bold text that one refers to To answer you can click on any part of the word or phrase in the passage Jour choice will darken to show which word you have chosen

The Suns outermost layer begins about 10000 miles above the visible surface and goes outward for millions of miles This is the only part of the Sun that can be seen durshying an eclipse such as the one in February 1979 At any other time the corona can be seen only when special instruments are used on cameras and telescopes to block the Iight from the photosphere

You will see the next question after you click on~ To answer some q~estions you will click on a sentence in the passage Here is an example

~ The corona is a brilliant pearly white filmy light about as bright as the full Moon Its beautiful rays are a sensational sight during an eclipse The coronas rays flash out in a brilliant fan that has wispy spikelike rays near the Suns north and south poles The corona is generally thickest at the Suns equator ~ The corona is made up of gases streaming outward at tremendous speeds that reach a temperature of more than 2 million degrees Fahrenheit The gas thins out as it reaches the space around the planets By the time the gas of the corona reaches the Earth it has a relatively low density

bull bullbullbullbullbullbull

92 CHAPTER 4 Standardized Testing

Click on the sentence in paragraph 4 or 5 in which the author compares the light of the Suns outermost layer to that ofanother astronomical body Paragraphs 4 and 5 are marked with arrows (~)

To answer some questions you will click on a square to add a sentence to the passage Here is an example -The following sentence can be added to paragraph 1

At the center of the Earths solar system lies the Sun

Where would it best fit in paragraph I Click on the square to add the sentence to the paragraph

D The temperature of the Sun is over 10000 degrees Fahrenheit at the surface but it rises to perhaps morethan 27000000deg at the center 0 The Sun is so much hotter than the Earth that matter can exist only as a gasi except p~rHapsatth~ c6relp the c~re of the ii Sun the pressures are so great that despite the high temperature there may be a small solid core D However no one really knows since the center of the Sun can never be directly observed D 0100

When you click on a square the sentence will appear in the passage at the place you have chosen You can read the sentence added to the paragraph to see if this is the best place to add it You can click on another square to change your answer The sentence will be added and shown in a dark box

Writing In this section you will have an opportunity to demonstrate your ability to write in Enshyglish This includes the ability to generate and organize ideas to support those ideas with examples or evidence and to compose in standard written English in response to an asshysigned topic You will have 30 minutes to write your essay on that topic You must write on the topic you are assigned An essay on any other topic will receive a score of 0 Read the topic below and then make any notes that will help you plan your response Begin typing your response in the box at the bottom of the screen or write your answer on the answer sheet provided to you

Following is a sample topic

Do you agree or disagree with the following statemenH

Teachers should make learning enjoyable and fun for their students

Use specific reasons and examples to support your opinion

CHAPTER 4 Standarczed Testing 93

Michigan English Language Assessment Battery (MELAB)

Composition The time limit for the composition is 30 minutes You must write on only one of the top~

ics below If you write about something else your composition paper will not be graded and you cannot be given a final score If you do not understand the topics ask the exam~ iner to explain or to translate them You may be asked to give your opinion ofsomething and explain why you believe this to describe something from your experience or to exshyplain a problem and offer possible solutions You should write at least one page Some sample topics are

1 What do you think is your countrys greatest problem Explain in detail and tell what you think can be done about it

2 What are the characteristics of a good teacher Explain and give examples 3 An optimist is someone who sees the good side of things A pessimist sees the

bad side Are you an optimist or a pessimist Relate a personal experience that shows this

4 In your opinion are the benefits of space exploration really worth the enormous costs Discuss

Most MELAB compositions are one or two pages long (about 200-300 words) If your paper is extremely short (less than 150 words) your composition will be given a lower score Before you begin writing you might want to take 2 or 3 minutes to plan your comshyposition and to make a short outline to organize your thoughts Such outlines will not be graded they are only to help you You should use the last 5 minutes to read through your composition and to make changes or corrections

Your composition will be graded on how clearly you express yourself in English and on the range of English you are able to use and your control in doing so This means your composition should be well organized your arguments should be fully developed and you should show a range ofgrammatical structures and broad vocabulary Compositions that consist only of very short sentences and very simple vocabulary cannot be given the

middothighest scores If errors are not frequent and if they do not confuse your meaning they will not lower your score very much

Listening Now you will hear a short lecture You may take notes during the lecture Following the lecture you will be asked some questions about it

Therell be a two-week exhibit of the paintings of the little-known master Laura Bernhart at the Claire Osmond Galleries starting on the fifteenth of the month and running through the thirtieth Bernharts known for her innovative designs in abstract expressionism Though a true original she declared a spiritual heritage from Salvador Dali the famous Spanish painter Since Bernhart lived a rather solitary life and died while only in her twenties few people are aware of her works This showing at the Osmond Galleries will provide many with an introduction to her works

10 Where is the exhibit a the Art Museum b the Dali Galleries c the Osmond Galleries

94 CHAPTER 4 Standardized Testing

11 What is Bernhart known for a her copies of Dalis paintings b the originality of her designs c her exhibitions

12 What will going to the exhibit allow most people to do a to see Saivador Dalis paintings b to see Bernharts works for the first time c to learn about Spanish art

Grammar

1 What did the teacher just tell you

She reminded our notebooksI a us to bring b that we bring c our bringing d we should bring

2 Is Bill a good dancer

Not really __ he tries very hard a in spite of h despite c even though d while

3 your clothes are all wet1

Yes I didnt come __ the rain soon enough a away to b over to c down with d in from

Cloze In years to come zoos will not only be places where animals are exhibited to the public but repositories where rare species can be saved from extinction (7) captive breeding The most powerful force (8) the future of many animals-and of zoos-is the decline of the wild (9) even zoo directors would argue that (10) are better places for animals than the fields and forest of their native (11) yet zoos may be the last chance for some creatures that would otherwise pass qUietly into oblivion

7 a through c from b of d damage

8 a bringing c to b that d influencing

9 a But c Not b So d Then

10 a where c even b zoos d wilds

11 alands c residence b life d field

CHAPTER 4 Standardized Testing 95

Vocabulary

12 Mark has a flair for writing a need b purpose c talent d dislike

13 Bill Collins launched his restaurant last June a moved b started c sold d bought

14 John will not accept the censure a burden b blame c credit d decision

15 I cant think of the answer Can you give me a __ a hint b token c taste d gaze

16 Because fewer people are taking expensive vacations the tourist industry is in a a choke b grope c grumble d slump

17 I disagree with a few of his opinions but __ we agree a deliberately b conclusively c essentially d immensely

Reading The influenza virus is a single molecule built from many millions of single atoms You must have heard of the viruses which are sometimes called living molecules While bacteria can be considered as a type of plant secreting pOisonous substances into the body of the organism they attack viruses are living organisms themselves We may conshysider them as regular chemical molecules since they have a strictly aefined atomic strucshyture but on the other hand we must also consider them as being alive since they are able to multiply in unlimited quantities

18 According to the passage bacteria are a poisons

b larger than viruses c very small d plants

96 CHAPTER 4 Standardized Testing

19 The writer says that viruses are alive because they a have a complex atomic structure b move c multiply d need warmth and light

20 The atomic structure of viruses a is -tJIariable b is strictly defined c cannot be analyzed chemically d is more complex than that of bacteria

International English Language Testing System (fELTS)

I

listening

The Listening Module has four sections The first two sections are concerned with social needs There is a conversation between two speakers and then a monologue For examshyple a conversation about travel arrangements or decisions on a night out and a speech about student services on a university campus or arrangements for meals during a confershyence The final two sections are concerned with situations related more closely to educashytional or training contexts For example conversation between a tutor and a student about an assignment or between three students planning a research project and a lecture or talk ofgeneral academic interest All the topics are ofgeneral interest and it makes no difference what subjects candidates study Tests and tasks become more difficult as the sections progress A range of English accents and dialects are used in the recording which reflects the international usage of IELTS

Academic Reading [A 7S0-word article on-th-e- topic of Wind Power in the US with a short glossary at the end]

Questions 1-5

Complete the summary below

Choose your answers from the box below the summary and write them in boxes 1-5 on your answer sheet Note There are more words or phrases than you will need to fill the gaps You may use any word or phrase more than once

Example The failure during the late 1970s and early 19805 of an attempt to establish a widespread wind power industry in the United States resulted largely from the (1) bull in oil prices during this period The industry is now experiencing a steady (2) due to improveshyments in technology and an increased awareness of the potential in the power of wind The wind turbines that are now being made based in part on the (3) of wide- ranging research in Europe are easier to manufacture and maintain than their predecesshysors This has led wind-turbine makers to be able to standardise and thus minimize (4) There has been growing (S) of the importance of wind power as an energy source

CHAPTER 4 Standardized Testing 97

criticism stability skepticism success operating costs decisions design costs fall effects production costs growth decline failure recognition results

Questions 6-1 0 Look at the following list of issues (Questions 6-10) and implications (A-C) Match each issue with one implication Write the appropriate letters A-C in boxes 6-10 on your anshyswer sheet

Example The current price of one wind-generated kilowatt Answer

6 The recent installation of systems taking advantage of economies of scale

7 The potential of meeting one fifth of current U5 energy requirements by wind power

8 The level of acceptance of current wind turbine technology

9 A comparison of costs between conventional and wind power sources

10 The view of wind power in the European Union

Implications

A provides evidence against claims that electricity produced from wind power is relatively expensive

B supports claims that wind power js an important source of energy

C opposes the view that wind power technology requires further-development

General Training Reading Read the passage on Daybreak trips by coach and look at the statements below On your answer sheet write

TRUE if the statement is true FALSE jf the statement is false

NOlGIVEN if the information is not given in the leaflet

1 MiIlers Coaches owns Cambridges Cam bus fleet

2 Premier is an older company than Millers

3 Most of the Daybreak coaches are less than 5 years old

4 Daybreak fares are more expensive than most of their competitors

5 Soft drinks and refreshments are served on most longer journeys

6 Smoking is permitted at the rear of the coach on longer journeys

7 Tickets must be bought in advance from an authorised Daybreak agent

6 Tickets and seats can be reserved by phoning the Daybreak Hotline

9 Daybreak passengers must join their coach at Cambridge Drummer Street

10 Daybreak cannot guarantee return times

98 CHAPTER 4 Standardized Testing

FROM CAMBRIDGE AND SURROUNDING AREA

SPRING IS INTHEAIR

Welcome to our Spring Daybreak programme which continues the tradition of offering unbeatable value for money day trips and tours All the excursions in this brochure will be operated by Pr~mier Travel Services Limited or Millers Coaches both companies are part of the CHLGroup owners of Cambridges Cambus fleet

WERE PROUD OF OUR TRADITION

Premier was established in 1936 the Company now offers the highest standards of coaching in todays competitive operating environment Miller has an enviable reputation stretching back over the past 20 years offering coach services at realistic prices Weve traveled a long way since our early days of pre-war seaside trips Now our fleet of 50 modern coaches (few are more than five years old) operate throughout Britain and Europe but were pleased to still maintain the high standards of quality and service the trademark of our founders nearly sixty years ago

EXCLUSIVE FEATURES

Admission-inclusive fares All Daybreak fares (unless specifically otherwise stated) include admission charges to the attractions shows and exhibits we visit Many full-day scenic tours are accompanied by a fully trained English Tourist Board Blue Badge guide or local experienced driverguide Some Daybreaks include lunch or afternoon tea Compare our admission inclusive fares and see how much you save Cheapest is not the best and value for money is guaranteed If you compare our bargain Daybreak fares beware--most of our competishytors do not offer an all-inclusive fare

SEAT RESERVATIONS

We value the freedom of choice so you can choose your seat when you book The seat reservation is guaranteed a-nd remains yours at all times when aboard the coach

NO SMOKING COMFORT

With the comfort of our passengers in mind coaches on all our Daybreaks are no smokshying throughout In the interests of fellow passengers comfort we kindly ask that smokers observe our no smoking policy On scenic tours and longer journeys ample refreshment stops are provided when of course smoking is permitted

YOUR QUESTIONS ANSWERED

Do I need to book Booking in advance is strongly recommended as all Daybreak tours are subject to demand Subject to availability stand-by tickets can be purchased from the driver

What ti me does the coach leave The coach departs from Cambridge Drummer Street (Bay 12 adjacent to public toilets) at the time shown There are many additional joining points indicated by departure codes in the brochure If you are joining at one of our less popular joining points you will be adshyvised of your pick-up time (normally by telephone) not less than 48 hours before deparshyture In this way we can minimize the length of pick-up routes and reduce journey times for the majority of passengers

CHAPTER 4 Standardized Testing 99

What time do we get back An approximate return time is shown for each excursion The tim~s shown serve as a guide but road conditions can sometimes cause delay If your arrival will be later than advertised your driver will try to allow for a telephone call during the return journey

Where can I board the coach All the Daybreaks in the brochure leave from Cambridge Drummer Street (Bay 12 adjashycent to public toilets) at the time shown Many Daybreaks offer additional pick-ups for pre-booked passengers within Cambridge and the surrounding area This facility must be requested at the time of booking

Academic Writing Writing Task 1 You should spend about 20 minutes on this task

The graph below shows the different modes of transport used to travel to and from work in one European city in 1950 1970 and 1990

[graph shown here]

Write a report for a university lecturer describing the information shown below You should write at least 150 words

Writing Task 2 You should spend about 40 minutes on this task

Present a written argument or case to an educated reader with no specialist knowledge of the folowing topic

It is inevitable that as technology develops so traditional cultures must be lost Technolshyogy and tradition are incompatible-you cannot have both together

To what extent do you agree or disagree with this statement Give reasons for your answer You should write at least 250 words You should use your own ideas knowlshyedge and experience and support your arguments with examples and relevant evidence

General Training Writing Writing Task 1 You should spend about 20 minutes on this task You rent a house through an agency The heating system has stopped working You phoned the agency a week ago but it has still not been mended Write a letter to the agency Explain the situation and teil them what you want them to do about it

You should write at least 150 words You do NOT need to write your own address

Begin your letter as follows

Dear - ___-I

Writing Task 2 You should spend about 40 minutes on this task As part ofa class assignment you have to write about the following topic

100 CHAPTER 4 Standardized Testing

Some businesses now say that no one can smoke cigarettes in any of their offices Some governments have banned smoking in all public placesThis is a good idea but it takes away some of our freedom

Do you agree or disagree Give reasons for your answer You should write at least 250 words

Speaking In each ofthe three parts of the speaking module a specific function is fulfilled In Part 1 the candidates answer general questions about themselves their homes or families their jobs or studies their interests and a range ofsimilar familiar topic areas This part lasts between four and five minutes In Part 2 the candidate is given a verbal prompt on a card and is asked to talk on a particular topic The candidate has one minute to prepare before speaking at length for between one and two minutes The examiner then asks one or two wind-down questions In Part 3 the examiner and candidate engage in a discusshysion of more abstract issues and concepts which are thematically linked to the topic prompt in Part 2 The discussion lasts between four and five minutes

All interviews are recorded on audiocassette Here is a sample ofa Part 2 topic

Describe a teacher who has greatly influenced you in your education

You shou Id say

where you met them what subject they taught what was special about them

and explain why this person influenced you so much

You will have to talk about the topic for 1 to 2 minutes You have 1 minute to think about what you are going to say You can make some notes if you wish

Test of English for International Communication (TOEICreg)

listening

Part 1 Photographs Directions For each question you will see a picture in your test book and you will hear four short statements The statements will be spoken just one time They will not be printed in your test book so you must listen carefully to understand what the speaker says When you hear the four statements look at the picture in your test book and choose the statement that best describes what you see in the picture Then on your answer sheet find the number of the question and mark your answer

[photograph of a scientist looking through a microscope]

You will hear Look at the picture marked number 1 in your test book

(A) Shes speaking into a microphone (B) Shes put on her glasses (C) She has both eyes open (D) Shes using a microscope

CHAPTER 4 Standardized Testing 101

Part 2 Question-Response Directions In this part of the test you will hear a question or statement spoken in Enshyglish followed by three responses also spoken in English The question or staten1ent and the responses will be spoken just one time They will not be printed in your test book so you must listen carefully to understand what the speakers say You are to choose the best response to each question or statement

Question 1 You will hear Ms Morikawa has worked here for a long time hasnt she

(A) At three oclock (B) No Ive lost my watch (C) More than ten years

Question 2 You will hear Which of these papers has a wider circulation

(A) The morning edition (B) Get more exercise (C) By messenger

Part 3 Short Conversations Directions In this part of the test you will hear short conversations between two people The conversations will not be printed in your test book You will hear the conversations only once so you must listen carefully to understand what the speakers say In your test book you will read a question about each conversation The question will be followed by four answers You are to choose the best answer to each question and mark it on your answer sheet

Question 1 (Man) We should think about finding another restaurant for lunch (Woman) Why The food and service here are great

(Man) Yes but the prices are going up every week

You will read Why is this man unhappy with the restaurant

(A) It is too noisy (B) It is too expensive (C) It is too crowded (D) It is too difficult to find

Question 2 (Woman A) How was Dr Borgs recent trip to Singapore (Woman B) She enjoyed the tour of the port very much (Woman A) They say its one of the most active in Asia

You will read 2 What did Dr Borg find interesting

(A) The tourist center (B) The airport (C) The musical performance (D) The harbor

Part 4 Short Talks Directions In this part of the test you vill hear several short talks Each will be spoken just one time They will not be printed in your test book so you must listen carefully to understand and remember what is said In your test book you will read two or more questions about each short talk The questions will be followed by four answers You are to choose the best answer to each question and mark it on your answer sheet

102 CHAPTER 4 Standardized Testing

You will hear Questions 1 and 2 refer to the following announcement

Good afternoon and welcome aboard Nordair Flight 857 from Copenhagen to Bangkok with intermediate stops in Dubai and Calcutta We are preparing for departure in a few minutes At this time your seat back should be returned to its full upright position and your seat belt s~ould be fastened OUf anticipated total flying time to Dubai is six hours and twenty-five minutes I hope you enjoy the flight You will hecJr Now read question 1 in your test book and answer it You will read 1 What is the final destination of the flight

(A) Bangkok (B) Copenhagen (C) Dubai (O) Calcutta

You will hear Now read question 2 in your test book and answer it You will read 2 What will happen in a few minutes

(A) The flight will land in Dubai I

(B) The passengers will board the plane (C) The plane will take off (0) The gate number will be announced

Reading In this section of the test you will have the chance to show how well you understand written English There are three parts to this section with special directions for each part

Part 4 Incomplete Sentences Directions This part of the test has incomplete sentences Four words or phrases marked (A) (8) (e) (D) are given beneath each sentence You are to choose the one word or phrase that best completes the sentence Then on your answer sheet find the number of the question and mark your answer

1 Mr Yangs trip will __ him away from the office for ten days (A) withdraw (B) continue (C) retain (0) keep

2 The company that Marie DuBois started now sells __ products throughout the world (A) its (B) it (C) theirs (D) them

3 If your shipment is not delivered __ Tuesday you can request a full refund for the merchandise (A) at (B) by (C) within (D) while

CHAPTER 4 Standardized Testing 103

Part 6 Error Recognition Directions In this part ofthe test each sentence has four words or phrases underlined The four underlined parts of the sentence are marked (A) (B) (C) (D) You are to identify the one underlined word or phrase that should be corrected or rewritten Then on your answer sheet find the number of the question and mark your answer

1 The pamphlet contains some importance information about the current exhibit ABC D

2 No matter how Jong it taking to finish the annual report it must be done properly ABC D

3 The popularity of jogging appears to have decreased since the past couple of years ABC D

Part 7 Reading Comprehension Directions The questions in this part of the test are based on a selection of reading mateshyrials such as notices letters) forms newspaper and magazine articles) and advertisements You are to choose the one best answer (A) (B) (C) or (OJ to each quesshytion Then on your ariswefsheelfindthe number of the qUestion andmcirkyour answer Answer all questions following each reading selection on thebasis of what is stated or implied in that selection

The Museum ofTechnology is a hands-on museum designed for people to experience science at w()rk~ Visitors are encouraged to use test and handle the objects o~ display Special demonstrations are scheduled for the first and second Wednesdays of each month at 1330 Open Tuesday-Friday 1200-1630 Saturday 1000-1730 and Sunday 11 00-1630

1 When during the month can visitors see special demonstrations (A) Every weekend (B) The first two Wednesdays (C) One afternoon a week (D) Every other Wednesday

Questions 2 and 3 refer to the followi ng notice

NOTICE If you are unable to work because of an extended illness or injury that is not workshyrelated you may be entitled to receive weekly benefits from your employer or the firms insurance company To claim benefits you must file a claim form within thirty days of the first day of your disability Before filing the claim you must ask your doctor to fill in the Doctors Statement on the claim form stating the period of disability

3 To whom is this notice addressed (A) Employers (8) Doctors (C) Employees (D) When paying the bill

4 When must the claim form be filed (A) On the first of the month (8) On the thirtieth of the month (C) On the first day ofdisabifity (D) Within 30 days of the start of disability

Page 23: Standardized Testing Chapter 4 Brown

88 CHAPTER 4 Standardized Testing

APPENDIX TO CHAPTER 4

Commercial Proficiency Tests Sample Items and Tasks

Test of English a~ a Foreign Language (TOEFLreg)

Listening r

Part A

In this section you will hear short conversations between two people In some ofthe conversations each person speaks only once In other conversations one or both of the people speak more than once Each conversation is followed by one questionabQlt it Each question in this part has four answer choices You should click on the best answer to each question Answer the questions on the basis of what is stated or implied by the speakers Here is an example On the computerscreen you will see

[man and woman talking]

On the recording you will hear

(woman) Hey wheres your sociology book (man) At home Why carry it around when were just going to be taking

a test (woman) Dont you remember Professor Smith said we could us it during

the test (man) Ohl no Well Ive still got an hour right Im so glad I ran into you

You wiII then see and hear the question before the answer choices appear

What will the man probably do next

o Begin studying for the sociology test o Explain the problem to his professor o Go home to get his textbook o Borrow the womans book

To choose an answer you will click on an oval The oval next to that answer will darken After you click on Next and Confirm Answer the next conversation will be presented

Part B

In this section you will hear several longer conversations and talks Each conversation or talk is followed by several questions The conversations talks and questions will not be repeated The conversations and talks are about a variety of topics You do not need speshycial knowledge of the topics to answer the questions correctly Rather you should answer each question on the basis of what is stated or implied by the speakers in the conversashytions or talks

For most of the questions you will need to click on the best of four possible answers Some questions will have special directions The special directions will appear in a box on the computer screen Here is an exampie ot a conversation and some questions

CHAPTER 4 Standardized Testing 89

Marine Biology (narrator) Listen to part of a discussion in a marine biology class

(professor) A few years ago our local government passed a number of strict environmental laws As a result Sunrise Beach looks nothing Ii ke it did ten years ago The water is cleaner and theres been a tremendous increase in all kinds of marine life which is why were going there on Thursday

(woman) I dont know if I agree that the water quality has improved I mean I was out there last weekend and it looked all brown It didnt seem too clean to me

(professor) Actually the color of the water doesnt always indicate whether its polluted The brown color you mentioned might be a result of pollution or it can mean a kind of brown algae is growing there Its called devils apron and it actually serves as food for whales

(man) So when does the water look blue (professor) Well water thats completely unpolluted is actually colorless But

it often looks bluish-green because the sunlight can penetrate deep down and thats the color thats reflected

(woman) But sometimes it looks really green Whats that about (professor) Ok well its the same principle as with devils apron the

water might be green because of different types of green algae there-gulfweed phytoplankton You all should finish reading about algae and plankton before we go In fact those are the types of living things Im going to ask you to be looking for when were there

Now get ready to answer the questions

What is the discussion mainly about

o The importance of protecting ocean environments o The reasons why ocean water appears to be different colors o The survival of whales in polluted water o The effect that colored ocean water has on algae

To choose an answer click on an oval The oval next to that answer will darken After you click on Next and Confirm Answer the next question will be presented

According to the professor what can make ocean water look browngt

o Pollution o Cloudy Skies o Sand o Algae

Click on 2 answers

To choose your answers you will click on the squares An XII wiii appear in each square

bullbullbullbullbullbullbull

90 CHAPTER 4 Standardized Testing

Structure and Written Expression This section measures the ability to recognize language that is appropriate for standard written English There are two types ofquestions in this section In the first type ofquestion there are incomplete sentences Beneath each sentence there are four words or phrases

Directions CIiSk on the one word or phrase that best completes the sentence

The colum~ine flower __ to nearly all of the United States can be raised from seed in almost any garden

native how native is how native is it is native

Time Help Confirm

After you click on Next and Confirm Answ~ the next question willbe presented

The second type of question has four underlined words or phrases You will choose the one underlined word or phrase that must be changed for the sentence to be correct

Directions Click on the one underlined word or phrase that must be changed for the senshytence to be correct

One of the most difficult problems in understanding sleep is determining what the funcshytions of sleep ~

lime Help Confirm

Clicking on an underlined word or phrase will darken it

Reading This section measures the ability to read and understand short passages similar in topic and style to those that students are likely to encounter in North American universities and colleges This section contains reading passages and questions about the passages There are several different types of questions in this section In the Reading section you will first have the opportunity to read the passage

The temperature of the Sun is over 10000 degrees Fahrenheit at the surface but it rises perhaps more than 270000000 at the center The Sun is so much hotter than the Earth that matter can exist only as a gasl except perhaps at the core In the core of the Sun the pressures are so great that despite the high temperature there may be a small solid core However no one really knows since the center of the Sun can never be directly observed ~ Solar astronomers do know that the Sun is divided into five general layers or zones Starting at the outside and going down into the Sun the zones are the corona chromoshysphere hotosphere convection zone and finally the core The first three zones are reshygarded as the Suns atmosphere But since the Sun has no solid surface it is hard to middottell where the atmosphere ends and the main body of the Sun begins

The Suns outermost layer begins about 10000 miles above the visible surface and goes outward for millions of miles This is the only part of the Sun that can be seen during an eclipse such as the one in February 1979 At any other time the corona can be seen

bullbullbullbullbullbullbull

bull bullbullbullbullbullbull

CHAPTER 4 Standardized Testing 91

only when special instruments are used on cameras and telescopes to block the light from the photosphere

The corona is a brilliant pearly white filmy light about as bright as the full Moon Its beautiful rays are a sensational sight during an eclipse The coronas rays flash out in a brilliant fan that has wispy spikelike rays near the Suns north and south poles The corona is generally thickest at the Suns equator The corona is made up of gases streamshying outward at tremendous speeds that reach a temperature of more than 2 million deshygrees Fahrenheit The gas thins out as it reaches the space around the planets By the time the gas of the corona reaches the Earth it has a relatively low density

When you have finished reading the passage you will use the mouse to click on Proceed Then the questions about the passage will be presented You are to choose the one best anshyswer to each question Answer all questions about the information in a passage on the basis ofwhat is stated or implied in that passage Most ofthe questions will be multiple-choice questions To answer these questions you will click on a choice below the question

With what topic is paragraph 2 mainly concerned

o How the Sun evolved o The structure of the Sun o Why scientists study the Sun o The distaflce of the Sun from the planets

Paragraph 2 is marked with an arrow (~)

You will see the next question after you click on Next

To answer some questions you will click on a word or phrase Here is an example

Look at the word one in the passage Click on the word or phrase in the bold text that one refers to To answer you can click on any part of the word or phrase in the passage Jour choice will darken to show which word you have chosen

The Suns outermost layer begins about 10000 miles above the visible surface and goes outward for millions of miles This is the only part of the Sun that can be seen durshying an eclipse such as the one in February 1979 At any other time the corona can be seen only when special instruments are used on cameras and telescopes to block the Iight from the photosphere

You will see the next question after you click on~ To answer some q~estions you will click on a sentence in the passage Here is an example

~ The corona is a brilliant pearly white filmy light about as bright as the full Moon Its beautiful rays are a sensational sight during an eclipse The coronas rays flash out in a brilliant fan that has wispy spikelike rays near the Suns north and south poles The corona is generally thickest at the Suns equator ~ The corona is made up of gases streaming outward at tremendous speeds that reach a temperature of more than 2 million degrees Fahrenheit The gas thins out as it reaches the space around the planets By the time the gas of the corona reaches the Earth it has a relatively low density

bull bullbullbullbullbullbull

92 CHAPTER 4 Standardized Testing

Click on the sentence in paragraph 4 or 5 in which the author compares the light of the Suns outermost layer to that ofanother astronomical body Paragraphs 4 and 5 are marked with arrows (~)

To answer some questions you will click on a square to add a sentence to the passage Here is an example -The following sentence can be added to paragraph 1

At the center of the Earths solar system lies the Sun

Where would it best fit in paragraph I Click on the square to add the sentence to the paragraph

D The temperature of the Sun is over 10000 degrees Fahrenheit at the surface but it rises to perhaps morethan 27000000deg at the center 0 The Sun is so much hotter than the Earth that matter can exist only as a gasi except p~rHapsatth~ c6relp the c~re of the ii Sun the pressures are so great that despite the high temperature there may be a small solid core D However no one really knows since the center of the Sun can never be directly observed D 0100

When you click on a square the sentence will appear in the passage at the place you have chosen You can read the sentence added to the paragraph to see if this is the best place to add it You can click on another square to change your answer The sentence will be added and shown in a dark box

Writing In this section you will have an opportunity to demonstrate your ability to write in Enshyglish This includes the ability to generate and organize ideas to support those ideas with examples or evidence and to compose in standard written English in response to an asshysigned topic You will have 30 minutes to write your essay on that topic You must write on the topic you are assigned An essay on any other topic will receive a score of 0 Read the topic below and then make any notes that will help you plan your response Begin typing your response in the box at the bottom of the screen or write your answer on the answer sheet provided to you

Following is a sample topic

Do you agree or disagree with the following statemenH

Teachers should make learning enjoyable and fun for their students

Use specific reasons and examples to support your opinion

CHAPTER 4 Standarczed Testing 93

Michigan English Language Assessment Battery (MELAB)

Composition The time limit for the composition is 30 minutes You must write on only one of the top~

ics below If you write about something else your composition paper will not be graded and you cannot be given a final score If you do not understand the topics ask the exam~ iner to explain or to translate them You may be asked to give your opinion ofsomething and explain why you believe this to describe something from your experience or to exshyplain a problem and offer possible solutions You should write at least one page Some sample topics are

1 What do you think is your countrys greatest problem Explain in detail and tell what you think can be done about it

2 What are the characteristics of a good teacher Explain and give examples 3 An optimist is someone who sees the good side of things A pessimist sees the

bad side Are you an optimist or a pessimist Relate a personal experience that shows this

4 In your opinion are the benefits of space exploration really worth the enormous costs Discuss

Most MELAB compositions are one or two pages long (about 200-300 words) If your paper is extremely short (less than 150 words) your composition will be given a lower score Before you begin writing you might want to take 2 or 3 minutes to plan your comshyposition and to make a short outline to organize your thoughts Such outlines will not be graded they are only to help you You should use the last 5 minutes to read through your composition and to make changes or corrections

Your composition will be graded on how clearly you express yourself in English and on the range of English you are able to use and your control in doing so This means your composition should be well organized your arguments should be fully developed and you should show a range ofgrammatical structures and broad vocabulary Compositions that consist only of very short sentences and very simple vocabulary cannot be given the

middothighest scores If errors are not frequent and if they do not confuse your meaning they will not lower your score very much

Listening Now you will hear a short lecture You may take notes during the lecture Following the lecture you will be asked some questions about it

Therell be a two-week exhibit of the paintings of the little-known master Laura Bernhart at the Claire Osmond Galleries starting on the fifteenth of the month and running through the thirtieth Bernharts known for her innovative designs in abstract expressionism Though a true original she declared a spiritual heritage from Salvador Dali the famous Spanish painter Since Bernhart lived a rather solitary life and died while only in her twenties few people are aware of her works This showing at the Osmond Galleries will provide many with an introduction to her works

10 Where is the exhibit a the Art Museum b the Dali Galleries c the Osmond Galleries

94 CHAPTER 4 Standardized Testing

11 What is Bernhart known for a her copies of Dalis paintings b the originality of her designs c her exhibitions

12 What will going to the exhibit allow most people to do a to see Saivador Dalis paintings b to see Bernharts works for the first time c to learn about Spanish art

Grammar

1 What did the teacher just tell you

She reminded our notebooksI a us to bring b that we bring c our bringing d we should bring

2 Is Bill a good dancer

Not really __ he tries very hard a in spite of h despite c even though d while

3 your clothes are all wet1

Yes I didnt come __ the rain soon enough a away to b over to c down with d in from

Cloze In years to come zoos will not only be places where animals are exhibited to the public but repositories where rare species can be saved from extinction (7) captive breeding The most powerful force (8) the future of many animals-and of zoos-is the decline of the wild (9) even zoo directors would argue that (10) are better places for animals than the fields and forest of their native (11) yet zoos may be the last chance for some creatures that would otherwise pass qUietly into oblivion

7 a through c from b of d damage

8 a bringing c to b that d influencing

9 a But c Not b So d Then

10 a where c even b zoos d wilds

11 alands c residence b life d field

CHAPTER 4 Standardized Testing 95

Vocabulary

12 Mark has a flair for writing a need b purpose c talent d dislike

13 Bill Collins launched his restaurant last June a moved b started c sold d bought

14 John will not accept the censure a burden b blame c credit d decision

15 I cant think of the answer Can you give me a __ a hint b token c taste d gaze

16 Because fewer people are taking expensive vacations the tourist industry is in a a choke b grope c grumble d slump

17 I disagree with a few of his opinions but __ we agree a deliberately b conclusively c essentially d immensely

Reading The influenza virus is a single molecule built from many millions of single atoms You must have heard of the viruses which are sometimes called living molecules While bacteria can be considered as a type of plant secreting pOisonous substances into the body of the organism they attack viruses are living organisms themselves We may conshysider them as regular chemical molecules since they have a strictly aefined atomic strucshyture but on the other hand we must also consider them as being alive since they are able to multiply in unlimited quantities

18 According to the passage bacteria are a poisons

b larger than viruses c very small d plants

96 CHAPTER 4 Standardized Testing

19 The writer says that viruses are alive because they a have a complex atomic structure b move c multiply d need warmth and light

20 The atomic structure of viruses a is -tJIariable b is strictly defined c cannot be analyzed chemically d is more complex than that of bacteria

International English Language Testing System (fELTS)

I

listening

The Listening Module has four sections The first two sections are concerned with social needs There is a conversation between two speakers and then a monologue For examshyple a conversation about travel arrangements or decisions on a night out and a speech about student services on a university campus or arrangements for meals during a confershyence The final two sections are concerned with situations related more closely to educashytional or training contexts For example conversation between a tutor and a student about an assignment or between three students planning a research project and a lecture or talk ofgeneral academic interest All the topics are ofgeneral interest and it makes no difference what subjects candidates study Tests and tasks become more difficult as the sections progress A range of English accents and dialects are used in the recording which reflects the international usage of IELTS

Academic Reading [A 7S0-word article on-th-e- topic of Wind Power in the US with a short glossary at the end]

Questions 1-5

Complete the summary below

Choose your answers from the box below the summary and write them in boxes 1-5 on your answer sheet Note There are more words or phrases than you will need to fill the gaps You may use any word or phrase more than once

Example The failure during the late 1970s and early 19805 of an attempt to establish a widespread wind power industry in the United States resulted largely from the (1) bull in oil prices during this period The industry is now experiencing a steady (2) due to improveshyments in technology and an increased awareness of the potential in the power of wind The wind turbines that are now being made based in part on the (3) of wide- ranging research in Europe are easier to manufacture and maintain than their predecesshysors This has led wind-turbine makers to be able to standardise and thus minimize (4) There has been growing (S) of the importance of wind power as an energy source

CHAPTER 4 Standardized Testing 97

criticism stability skepticism success operating costs decisions design costs fall effects production costs growth decline failure recognition results

Questions 6-1 0 Look at the following list of issues (Questions 6-10) and implications (A-C) Match each issue with one implication Write the appropriate letters A-C in boxes 6-10 on your anshyswer sheet

Example The current price of one wind-generated kilowatt Answer

6 The recent installation of systems taking advantage of economies of scale

7 The potential of meeting one fifth of current U5 energy requirements by wind power

8 The level of acceptance of current wind turbine technology

9 A comparison of costs between conventional and wind power sources

10 The view of wind power in the European Union

Implications

A provides evidence against claims that electricity produced from wind power is relatively expensive

B supports claims that wind power js an important source of energy

C opposes the view that wind power technology requires further-development

General Training Reading Read the passage on Daybreak trips by coach and look at the statements below On your answer sheet write

TRUE if the statement is true FALSE jf the statement is false

NOlGIVEN if the information is not given in the leaflet

1 MiIlers Coaches owns Cambridges Cam bus fleet

2 Premier is an older company than Millers

3 Most of the Daybreak coaches are less than 5 years old

4 Daybreak fares are more expensive than most of their competitors

5 Soft drinks and refreshments are served on most longer journeys

6 Smoking is permitted at the rear of the coach on longer journeys

7 Tickets must be bought in advance from an authorised Daybreak agent

6 Tickets and seats can be reserved by phoning the Daybreak Hotline

9 Daybreak passengers must join their coach at Cambridge Drummer Street

10 Daybreak cannot guarantee return times

98 CHAPTER 4 Standardized Testing

FROM CAMBRIDGE AND SURROUNDING AREA

SPRING IS INTHEAIR

Welcome to our Spring Daybreak programme which continues the tradition of offering unbeatable value for money day trips and tours All the excursions in this brochure will be operated by Pr~mier Travel Services Limited or Millers Coaches both companies are part of the CHLGroup owners of Cambridges Cambus fleet

WERE PROUD OF OUR TRADITION

Premier was established in 1936 the Company now offers the highest standards of coaching in todays competitive operating environment Miller has an enviable reputation stretching back over the past 20 years offering coach services at realistic prices Weve traveled a long way since our early days of pre-war seaside trips Now our fleet of 50 modern coaches (few are more than five years old) operate throughout Britain and Europe but were pleased to still maintain the high standards of quality and service the trademark of our founders nearly sixty years ago

EXCLUSIVE FEATURES

Admission-inclusive fares All Daybreak fares (unless specifically otherwise stated) include admission charges to the attractions shows and exhibits we visit Many full-day scenic tours are accompanied by a fully trained English Tourist Board Blue Badge guide or local experienced driverguide Some Daybreaks include lunch or afternoon tea Compare our admission inclusive fares and see how much you save Cheapest is not the best and value for money is guaranteed If you compare our bargain Daybreak fares beware--most of our competishytors do not offer an all-inclusive fare

SEAT RESERVATIONS

We value the freedom of choice so you can choose your seat when you book The seat reservation is guaranteed a-nd remains yours at all times when aboard the coach

NO SMOKING COMFORT

With the comfort of our passengers in mind coaches on all our Daybreaks are no smokshying throughout In the interests of fellow passengers comfort we kindly ask that smokers observe our no smoking policy On scenic tours and longer journeys ample refreshment stops are provided when of course smoking is permitted

YOUR QUESTIONS ANSWERED

Do I need to book Booking in advance is strongly recommended as all Daybreak tours are subject to demand Subject to availability stand-by tickets can be purchased from the driver

What ti me does the coach leave The coach departs from Cambridge Drummer Street (Bay 12 adjacent to public toilets) at the time shown There are many additional joining points indicated by departure codes in the brochure If you are joining at one of our less popular joining points you will be adshyvised of your pick-up time (normally by telephone) not less than 48 hours before deparshyture In this way we can minimize the length of pick-up routes and reduce journey times for the majority of passengers

CHAPTER 4 Standardized Testing 99

What time do we get back An approximate return time is shown for each excursion The tim~s shown serve as a guide but road conditions can sometimes cause delay If your arrival will be later than advertised your driver will try to allow for a telephone call during the return journey

Where can I board the coach All the Daybreaks in the brochure leave from Cambridge Drummer Street (Bay 12 adjashycent to public toilets) at the time shown Many Daybreaks offer additional pick-ups for pre-booked passengers within Cambridge and the surrounding area This facility must be requested at the time of booking

Academic Writing Writing Task 1 You should spend about 20 minutes on this task

The graph below shows the different modes of transport used to travel to and from work in one European city in 1950 1970 and 1990

[graph shown here]

Write a report for a university lecturer describing the information shown below You should write at least 150 words

Writing Task 2 You should spend about 40 minutes on this task

Present a written argument or case to an educated reader with no specialist knowledge of the folowing topic

It is inevitable that as technology develops so traditional cultures must be lost Technolshyogy and tradition are incompatible-you cannot have both together

To what extent do you agree or disagree with this statement Give reasons for your answer You should write at least 250 words You should use your own ideas knowlshyedge and experience and support your arguments with examples and relevant evidence

General Training Writing Writing Task 1 You should spend about 20 minutes on this task You rent a house through an agency The heating system has stopped working You phoned the agency a week ago but it has still not been mended Write a letter to the agency Explain the situation and teil them what you want them to do about it

You should write at least 150 words You do NOT need to write your own address

Begin your letter as follows

Dear - ___-I

Writing Task 2 You should spend about 40 minutes on this task As part ofa class assignment you have to write about the following topic

100 CHAPTER 4 Standardized Testing

Some businesses now say that no one can smoke cigarettes in any of their offices Some governments have banned smoking in all public placesThis is a good idea but it takes away some of our freedom

Do you agree or disagree Give reasons for your answer You should write at least 250 words

Speaking In each ofthe three parts of the speaking module a specific function is fulfilled In Part 1 the candidates answer general questions about themselves their homes or families their jobs or studies their interests and a range ofsimilar familiar topic areas This part lasts between four and five minutes In Part 2 the candidate is given a verbal prompt on a card and is asked to talk on a particular topic The candidate has one minute to prepare before speaking at length for between one and two minutes The examiner then asks one or two wind-down questions In Part 3 the examiner and candidate engage in a discusshysion of more abstract issues and concepts which are thematically linked to the topic prompt in Part 2 The discussion lasts between four and five minutes

All interviews are recorded on audiocassette Here is a sample ofa Part 2 topic

Describe a teacher who has greatly influenced you in your education

You shou Id say

where you met them what subject they taught what was special about them

and explain why this person influenced you so much

You will have to talk about the topic for 1 to 2 minutes You have 1 minute to think about what you are going to say You can make some notes if you wish

Test of English for International Communication (TOEICreg)

listening

Part 1 Photographs Directions For each question you will see a picture in your test book and you will hear four short statements The statements will be spoken just one time They will not be printed in your test book so you must listen carefully to understand what the speaker says When you hear the four statements look at the picture in your test book and choose the statement that best describes what you see in the picture Then on your answer sheet find the number of the question and mark your answer

[photograph of a scientist looking through a microscope]

You will hear Look at the picture marked number 1 in your test book

(A) Shes speaking into a microphone (B) Shes put on her glasses (C) She has both eyes open (D) Shes using a microscope

CHAPTER 4 Standardized Testing 101

Part 2 Question-Response Directions In this part of the test you will hear a question or statement spoken in Enshyglish followed by three responses also spoken in English The question or staten1ent and the responses will be spoken just one time They will not be printed in your test book so you must listen carefully to understand what the speakers say You are to choose the best response to each question or statement

Question 1 You will hear Ms Morikawa has worked here for a long time hasnt she

(A) At three oclock (B) No Ive lost my watch (C) More than ten years

Question 2 You will hear Which of these papers has a wider circulation

(A) The morning edition (B) Get more exercise (C) By messenger

Part 3 Short Conversations Directions In this part of the test you will hear short conversations between two people The conversations will not be printed in your test book You will hear the conversations only once so you must listen carefully to understand what the speakers say In your test book you will read a question about each conversation The question will be followed by four answers You are to choose the best answer to each question and mark it on your answer sheet

Question 1 (Man) We should think about finding another restaurant for lunch (Woman) Why The food and service here are great

(Man) Yes but the prices are going up every week

You will read Why is this man unhappy with the restaurant

(A) It is too noisy (B) It is too expensive (C) It is too crowded (D) It is too difficult to find

Question 2 (Woman A) How was Dr Borgs recent trip to Singapore (Woman B) She enjoyed the tour of the port very much (Woman A) They say its one of the most active in Asia

You will read 2 What did Dr Borg find interesting

(A) The tourist center (B) The airport (C) The musical performance (D) The harbor

Part 4 Short Talks Directions In this part of the test you vill hear several short talks Each will be spoken just one time They will not be printed in your test book so you must listen carefully to understand and remember what is said In your test book you will read two or more questions about each short talk The questions will be followed by four answers You are to choose the best answer to each question and mark it on your answer sheet

102 CHAPTER 4 Standardized Testing

You will hear Questions 1 and 2 refer to the following announcement

Good afternoon and welcome aboard Nordair Flight 857 from Copenhagen to Bangkok with intermediate stops in Dubai and Calcutta We are preparing for departure in a few minutes At this time your seat back should be returned to its full upright position and your seat belt s~ould be fastened OUf anticipated total flying time to Dubai is six hours and twenty-five minutes I hope you enjoy the flight You will hecJr Now read question 1 in your test book and answer it You will read 1 What is the final destination of the flight

(A) Bangkok (B) Copenhagen (C) Dubai (O) Calcutta

You will hear Now read question 2 in your test book and answer it You will read 2 What will happen in a few minutes

(A) The flight will land in Dubai I

(B) The passengers will board the plane (C) The plane will take off (0) The gate number will be announced

Reading In this section of the test you will have the chance to show how well you understand written English There are three parts to this section with special directions for each part

Part 4 Incomplete Sentences Directions This part of the test has incomplete sentences Four words or phrases marked (A) (8) (e) (D) are given beneath each sentence You are to choose the one word or phrase that best completes the sentence Then on your answer sheet find the number of the question and mark your answer

1 Mr Yangs trip will __ him away from the office for ten days (A) withdraw (B) continue (C) retain (0) keep

2 The company that Marie DuBois started now sells __ products throughout the world (A) its (B) it (C) theirs (D) them

3 If your shipment is not delivered __ Tuesday you can request a full refund for the merchandise (A) at (B) by (C) within (D) while

CHAPTER 4 Standardized Testing 103

Part 6 Error Recognition Directions In this part ofthe test each sentence has four words or phrases underlined The four underlined parts of the sentence are marked (A) (B) (C) (D) You are to identify the one underlined word or phrase that should be corrected or rewritten Then on your answer sheet find the number of the question and mark your answer

1 The pamphlet contains some importance information about the current exhibit ABC D

2 No matter how Jong it taking to finish the annual report it must be done properly ABC D

3 The popularity of jogging appears to have decreased since the past couple of years ABC D

Part 7 Reading Comprehension Directions The questions in this part of the test are based on a selection of reading mateshyrials such as notices letters) forms newspaper and magazine articles) and advertisements You are to choose the one best answer (A) (B) (C) or (OJ to each quesshytion Then on your ariswefsheelfindthe number of the qUestion andmcirkyour answer Answer all questions following each reading selection on thebasis of what is stated or implied in that selection

The Museum ofTechnology is a hands-on museum designed for people to experience science at w()rk~ Visitors are encouraged to use test and handle the objects o~ display Special demonstrations are scheduled for the first and second Wednesdays of each month at 1330 Open Tuesday-Friday 1200-1630 Saturday 1000-1730 and Sunday 11 00-1630

1 When during the month can visitors see special demonstrations (A) Every weekend (B) The first two Wednesdays (C) One afternoon a week (D) Every other Wednesday

Questions 2 and 3 refer to the followi ng notice

NOTICE If you are unable to work because of an extended illness or injury that is not workshyrelated you may be entitled to receive weekly benefits from your employer or the firms insurance company To claim benefits you must file a claim form within thirty days of the first day of your disability Before filing the claim you must ask your doctor to fill in the Doctors Statement on the claim form stating the period of disability

3 To whom is this notice addressed (A) Employers (8) Doctors (C) Employees (D) When paying the bill

4 When must the claim form be filed (A) On the first of the month (8) On the thirtieth of the month (C) On the first day ofdisabifity (D) Within 30 days of the start of disability

Page 24: Standardized Testing Chapter 4 Brown

CHAPTER 4 Standardized Testing 89

Marine Biology (narrator) Listen to part of a discussion in a marine biology class

(professor) A few years ago our local government passed a number of strict environmental laws As a result Sunrise Beach looks nothing Ii ke it did ten years ago The water is cleaner and theres been a tremendous increase in all kinds of marine life which is why were going there on Thursday

(woman) I dont know if I agree that the water quality has improved I mean I was out there last weekend and it looked all brown It didnt seem too clean to me

(professor) Actually the color of the water doesnt always indicate whether its polluted The brown color you mentioned might be a result of pollution or it can mean a kind of brown algae is growing there Its called devils apron and it actually serves as food for whales

(man) So when does the water look blue (professor) Well water thats completely unpolluted is actually colorless But

it often looks bluish-green because the sunlight can penetrate deep down and thats the color thats reflected

(woman) But sometimes it looks really green Whats that about (professor) Ok well its the same principle as with devils apron the

water might be green because of different types of green algae there-gulfweed phytoplankton You all should finish reading about algae and plankton before we go In fact those are the types of living things Im going to ask you to be looking for when were there

Now get ready to answer the questions

What is the discussion mainly about

o The importance of protecting ocean environments o The reasons why ocean water appears to be different colors o The survival of whales in polluted water o The effect that colored ocean water has on algae

To choose an answer click on an oval The oval next to that answer will darken After you click on Next and Confirm Answer the next question will be presented

According to the professor what can make ocean water look browngt

o Pollution o Cloudy Skies o Sand o Algae

Click on 2 answers

To choose your answers you will click on the squares An XII wiii appear in each square

bullbullbullbullbullbullbull

90 CHAPTER 4 Standardized Testing

Structure and Written Expression This section measures the ability to recognize language that is appropriate for standard written English There are two types ofquestions in this section In the first type ofquestion there are incomplete sentences Beneath each sentence there are four words or phrases

Directions CIiSk on the one word or phrase that best completes the sentence

The colum~ine flower __ to nearly all of the United States can be raised from seed in almost any garden

native how native is how native is it is native

Time Help Confirm

After you click on Next and Confirm Answ~ the next question willbe presented

The second type of question has four underlined words or phrases You will choose the one underlined word or phrase that must be changed for the sentence to be correct

Directions Click on the one underlined word or phrase that must be changed for the senshytence to be correct

One of the most difficult problems in understanding sleep is determining what the funcshytions of sleep ~

lime Help Confirm

Clicking on an underlined word or phrase will darken it

Reading This section measures the ability to read and understand short passages similar in topic and style to those that students are likely to encounter in North American universities and colleges This section contains reading passages and questions about the passages There are several different types of questions in this section In the Reading section you will first have the opportunity to read the passage

The temperature of the Sun is over 10000 degrees Fahrenheit at the surface but it rises perhaps more than 270000000 at the center The Sun is so much hotter than the Earth that matter can exist only as a gasl except perhaps at the core In the core of the Sun the pressures are so great that despite the high temperature there may be a small solid core However no one really knows since the center of the Sun can never be directly observed ~ Solar astronomers do know that the Sun is divided into five general layers or zones Starting at the outside and going down into the Sun the zones are the corona chromoshysphere hotosphere convection zone and finally the core The first three zones are reshygarded as the Suns atmosphere But since the Sun has no solid surface it is hard to middottell where the atmosphere ends and the main body of the Sun begins

The Suns outermost layer begins about 10000 miles above the visible surface and goes outward for millions of miles This is the only part of the Sun that can be seen during an eclipse such as the one in February 1979 At any other time the corona can be seen

bullbullbullbullbullbullbull

bull bullbullbullbullbullbull

CHAPTER 4 Standardized Testing 91

only when special instruments are used on cameras and telescopes to block the light from the photosphere

The corona is a brilliant pearly white filmy light about as bright as the full Moon Its beautiful rays are a sensational sight during an eclipse The coronas rays flash out in a brilliant fan that has wispy spikelike rays near the Suns north and south poles The corona is generally thickest at the Suns equator The corona is made up of gases streamshying outward at tremendous speeds that reach a temperature of more than 2 million deshygrees Fahrenheit The gas thins out as it reaches the space around the planets By the time the gas of the corona reaches the Earth it has a relatively low density

When you have finished reading the passage you will use the mouse to click on Proceed Then the questions about the passage will be presented You are to choose the one best anshyswer to each question Answer all questions about the information in a passage on the basis ofwhat is stated or implied in that passage Most ofthe questions will be multiple-choice questions To answer these questions you will click on a choice below the question

With what topic is paragraph 2 mainly concerned

o How the Sun evolved o The structure of the Sun o Why scientists study the Sun o The distaflce of the Sun from the planets

Paragraph 2 is marked with an arrow (~)

You will see the next question after you click on Next

To answer some questions you will click on a word or phrase Here is an example

Look at the word one in the passage Click on the word or phrase in the bold text that one refers to To answer you can click on any part of the word or phrase in the passage Jour choice will darken to show which word you have chosen

The Suns outermost layer begins about 10000 miles above the visible surface and goes outward for millions of miles This is the only part of the Sun that can be seen durshying an eclipse such as the one in February 1979 At any other time the corona can be seen only when special instruments are used on cameras and telescopes to block the Iight from the photosphere

You will see the next question after you click on~ To answer some q~estions you will click on a sentence in the passage Here is an example

~ The corona is a brilliant pearly white filmy light about as bright as the full Moon Its beautiful rays are a sensational sight during an eclipse The coronas rays flash out in a brilliant fan that has wispy spikelike rays near the Suns north and south poles The corona is generally thickest at the Suns equator ~ The corona is made up of gases streaming outward at tremendous speeds that reach a temperature of more than 2 million degrees Fahrenheit The gas thins out as it reaches the space around the planets By the time the gas of the corona reaches the Earth it has a relatively low density

bull bullbullbullbullbullbull

92 CHAPTER 4 Standardized Testing

Click on the sentence in paragraph 4 or 5 in which the author compares the light of the Suns outermost layer to that ofanother astronomical body Paragraphs 4 and 5 are marked with arrows (~)

To answer some questions you will click on a square to add a sentence to the passage Here is an example -The following sentence can be added to paragraph 1

At the center of the Earths solar system lies the Sun

Where would it best fit in paragraph I Click on the square to add the sentence to the paragraph

D The temperature of the Sun is over 10000 degrees Fahrenheit at the surface but it rises to perhaps morethan 27000000deg at the center 0 The Sun is so much hotter than the Earth that matter can exist only as a gasi except p~rHapsatth~ c6relp the c~re of the ii Sun the pressures are so great that despite the high temperature there may be a small solid core D However no one really knows since the center of the Sun can never be directly observed D 0100

When you click on a square the sentence will appear in the passage at the place you have chosen You can read the sentence added to the paragraph to see if this is the best place to add it You can click on another square to change your answer The sentence will be added and shown in a dark box

Writing In this section you will have an opportunity to demonstrate your ability to write in Enshyglish This includes the ability to generate and organize ideas to support those ideas with examples or evidence and to compose in standard written English in response to an asshysigned topic You will have 30 minutes to write your essay on that topic You must write on the topic you are assigned An essay on any other topic will receive a score of 0 Read the topic below and then make any notes that will help you plan your response Begin typing your response in the box at the bottom of the screen or write your answer on the answer sheet provided to you

Following is a sample topic

Do you agree or disagree with the following statemenH

Teachers should make learning enjoyable and fun for their students

Use specific reasons and examples to support your opinion

CHAPTER 4 Standarczed Testing 93

Michigan English Language Assessment Battery (MELAB)

Composition The time limit for the composition is 30 minutes You must write on only one of the top~

ics below If you write about something else your composition paper will not be graded and you cannot be given a final score If you do not understand the topics ask the exam~ iner to explain or to translate them You may be asked to give your opinion ofsomething and explain why you believe this to describe something from your experience or to exshyplain a problem and offer possible solutions You should write at least one page Some sample topics are

1 What do you think is your countrys greatest problem Explain in detail and tell what you think can be done about it

2 What are the characteristics of a good teacher Explain and give examples 3 An optimist is someone who sees the good side of things A pessimist sees the

bad side Are you an optimist or a pessimist Relate a personal experience that shows this

4 In your opinion are the benefits of space exploration really worth the enormous costs Discuss

Most MELAB compositions are one or two pages long (about 200-300 words) If your paper is extremely short (less than 150 words) your composition will be given a lower score Before you begin writing you might want to take 2 or 3 minutes to plan your comshyposition and to make a short outline to organize your thoughts Such outlines will not be graded they are only to help you You should use the last 5 minutes to read through your composition and to make changes or corrections

Your composition will be graded on how clearly you express yourself in English and on the range of English you are able to use and your control in doing so This means your composition should be well organized your arguments should be fully developed and you should show a range ofgrammatical structures and broad vocabulary Compositions that consist only of very short sentences and very simple vocabulary cannot be given the

middothighest scores If errors are not frequent and if they do not confuse your meaning they will not lower your score very much

Listening Now you will hear a short lecture You may take notes during the lecture Following the lecture you will be asked some questions about it

Therell be a two-week exhibit of the paintings of the little-known master Laura Bernhart at the Claire Osmond Galleries starting on the fifteenth of the month and running through the thirtieth Bernharts known for her innovative designs in abstract expressionism Though a true original she declared a spiritual heritage from Salvador Dali the famous Spanish painter Since Bernhart lived a rather solitary life and died while only in her twenties few people are aware of her works This showing at the Osmond Galleries will provide many with an introduction to her works

10 Where is the exhibit a the Art Museum b the Dali Galleries c the Osmond Galleries

94 CHAPTER 4 Standardized Testing

11 What is Bernhart known for a her copies of Dalis paintings b the originality of her designs c her exhibitions

12 What will going to the exhibit allow most people to do a to see Saivador Dalis paintings b to see Bernharts works for the first time c to learn about Spanish art

Grammar

1 What did the teacher just tell you

She reminded our notebooksI a us to bring b that we bring c our bringing d we should bring

2 Is Bill a good dancer

Not really __ he tries very hard a in spite of h despite c even though d while

3 your clothes are all wet1

Yes I didnt come __ the rain soon enough a away to b over to c down with d in from

Cloze In years to come zoos will not only be places where animals are exhibited to the public but repositories where rare species can be saved from extinction (7) captive breeding The most powerful force (8) the future of many animals-and of zoos-is the decline of the wild (9) even zoo directors would argue that (10) are better places for animals than the fields and forest of their native (11) yet zoos may be the last chance for some creatures that would otherwise pass qUietly into oblivion

7 a through c from b of d damage

8 a bringing c to b that d influencing

9 a But c Not b So d Then

10 a where c even b zoos d wilds

11 alands c residence b life d field

CHAPTER 4 Standardized Testing 95

Vocabulary

12 Mark has a flair for writing a need b purpose c talent d dislike

13 Bill Collins launched his restaurant last June a moved b started c sold d bought

14 John will not accept the censure a burden b blame c credit d decision

15 I cant think of the answer Can you give me a __ a hint b token c taste d gaze

16 Because fewer people are taking expensive vacations the tourist industry is in a a choke b grope c grumble d slump

17 I disagree with a few of his opinions but __ we agree a deliberately b conclusively c essentially d immensely

Reading The influenza virus is a single molecule built from many millions of single atoms You must have heard of the viruses which are sometimes called living molecules While bacteria can be considered as a type of plant secreting pOisonous substances into the body of the organism they attack viruses are living organisms themselves We may conshysider them as regular chemical molecules since they have a strictly aefined atomic strucshyture but on the other hand we must also consider them as being alive since they are able to multiply in unlimited quantities

18 According to the passage bacteria are a poisons

b larger than viruses c very small d plants

96 CHAPTER 4 Standardized Testing

19 The writer says that viruses are alive because they a have a complex atomic structure b move c multiply d need warmth and light

20 The atomic structure of viruses a is -tJIariable b is strictly defined c cannot be analyzed chemically d is more complex than that of bacteria

International English Language Testing System (fELTS)

I

listening

The Listening Module has four sections The first two sections are concerned with social needs There is a conversation between two speakers and then a monologue For examshyple a conversation about travel arrangements or decisions on a night out and a speech about student services on a university campus or arrangements for meals during a confershyence The final two sections are concerned with situations related more closely to educashytional or training contexts For example conversation between a tutor and a student about an assignment or between three students planning a research project and a lecture or talk ofgeneral academic interest All the topics are ofgeneral interest and it makes no difference what subjects candidates study Tests and tasks become more difficult as the sections progress A range of English accents and dialects are used in the recording which reflects the international usage of IELTS

Academic Reading [A 7S0-word article on-th-e- topic of Wind Power in the US with a short glossary at the end]

Questions 1-5

Complete the summary below

Choose your answers from the box below the summary and write them in boxes 1-5 on your answer sheet Note There are more words or phrases than you will need to fill the gaps You may use any word or phrase more than once

Example The failure during the late 1970s and early 19805 of an attempt to establish a widespread wind power industry in the United States resulted largely from the (1) bull in oil prices during this period The industry is now experiencing a steady (2) due to improveshyments in technology and an increased awareness of the potential in the power of wind The wind turbines that are now being made based in part on the (3) of wide- ranging research in Europe are easier to manufacture and maintain than their predecesshysors This has led wind-turbine makers to be able to standardise and thus minimize (4) There has been growing (S) of the importance of wind power as an energy source

CHAPTER 4 Standardized Testing 97

criticism stability skepticism success operating costs decisions design costs fall effects production costs growth decline failure recognition results

Questions 6-1 0 Look at the following list of issues (Questions 6-10) and implications (A-C) Match each issue with one implication Write the appropriate letters A-C in boxes 6-10 on your anshyswer sheet

Example The current price of one wind-generated kilowatt Answer

6 The recent installation of systems taking advantage of economies of scale

7 The potential of meeting one fifth of current U5 energy requirements by wind power

8 The level of acceptance of current wind turbine technology

9 A comparison of costs between conventional and wind power sources

10 The view of wind power in the European Union

Implications

A provides evidence against claims that electricity produced from wind power is relatively expensive

B supports claims that wind power js an important source of energy

C opposes the view that wind power technology requires further-development

General Training Reading Read the passage on Daybreak trips by coach and look at the statements below On your answer sheet write

TRUE if the statement is true FALSE jf the statement is false

NOlGIVEN if the information is not given in the leaflet

1 MiIlers Coaches owns Cambridges Cam bus fleet

2 Premier is an older company than Millers

3 Most of the Daybreak coaches are less than 5 years old

4 Daybreak fares are more expensive than most of their competitors

5 Soft drinks and refreshments are served on most longer journeys

6 Smoking is permitted at the rear of the coach on longer journeys

7 Tickets must be bought in advance from an authorised Daybreak agent

6 Tickets and seats can be reserved by phoning the Daybreak Hotline

9 Daybreak passengers must join their coach at Cambridge Drummer Street

10 Daybreak cannot guarantee return times

98 CHAPTER 4 Standardized Testing

FROM CAMBRIDGE AND SURROUNDING AREA

SPRING IS INTHEAIR

Welcome to our Spring Daybreak programme which continues the tradition of offering unbeatable value for money day trips and tours All the excursions in this brochure will be operated by Pr~mier Travel Services Limited or Millers Coaches both companies are part of the CHLGroup owners of Cambridges Cambus fleet

WERE PROUD OF OUR TRADITION

Premier was established in 1936 the Company now offers the highest standards of coaching in todays competitive operating environment Miller has an enviable reputation stretching back over the past 20 years offering coach services at realistic prices Weve traveled a long way since our early days of pre-war seaside trips Now our fleet of 50 modern coaches (few are more than five years old) operate throughout Britain and Europe but were pleased to still maintain the high standards of quality and service the trademark of our founders nearly sixty years ago

EXCLUSIVE FEATURES

Admission-inclusive fares All Daybreak fares (unless specifically otherwise stated) include admission charges to the attractions shows and exhibits we visit Many full-day scenic tours are accompanied by a fully trained English Tourist Board Blue Badge guide or local experienced driverguide Some Daybreaks include lunch or afternoon tea Compare our admission inclusive fares and see how much you save Cheapest is not the best and value for money is guaranteed If you compare our bargain Daybreak fares beware--most of our competishytors do not offer an all-inclusive fare

SEAT RESERVATIONS

We value the freedom of choice so you can choose your seat when you book The seat reservation is guaranteed a-nd remains yours at all times when aboard the coach

NO SMOKING COMFORT

With the comfort of our passengers in mind coaches on all our Daybreaks are no smokshying throughout In the interests of fellow passengers comfort we kindly ask that smokers observe our no smoking policy On scenic tours and longer journeys ample refreshment stops are provided when of course smoking is permitted

YOUR QUESTIONS ANSWERED

Do I need to book Booking in advance is strongly recommended as all Daybreak tours are subject to demand Subject to availability stand-by tickets can be purchased from the driver

What ti me does the coach leave The coach departs from Cambridge Drummer Street (Bay 12 adjacent to public toilets) at the time shown There are many additional joining points indicated by departure codes in the brochure If you are joining at one of our less popular joining points you will be adshyvised of your pick-up time (normally by telephone) not less than 48 hours before deparshyture In this way we can minimize the length of pick-up routes and reduce journey times for the majority of passengers

CHAPTER 4 Standardized Testing 99

What time do we get back An approximate return time is shown for each excursion The tim~s shown serve as a guide but road conditions can sometimes cause delay If your arrival will be later than advertised your driver will try to allow for a telephone call during the return journey

Where can I board the coach All the Daybreaks in the brochure leave from Cambridge Drummer Street (Bay 12 adjashycent to public toilets) at the time shown Many Daybreaks offer additional pick-ups for pre-booked passengers within Cambridge and the surrounding area This facility must be requested at the time of booking

Academic Writing Writing Task 1 You should spend about 20 minutes on this task

The graph below shows the different modes of transport used to travel to and from work in one European city in 1950 1970 and 1990

[graph shown here]

Write a report for a university lecturer describing the information shown below You should write at least 150 words

Writing Task 2 You should spend about 40 minutes on this task

Present a written argument or case to an educated reader with no specialist knowledge of the folowing topic

It is inevitable that as technology develops so traditional cultures must be lost Technolshyogy and tradition are incompatible-you cannot have both together

To what extent do you agree or disagree with this statement Give reasons for your answer You should write at least 250 words You should use your own ideas knowlshyedge and experience and support your arguments with examples and relevant evidence

General Training Writing Writing Task 1 You should spend about 20 minutes on this task You rent a house through an agency The heating system has stopped working You phoned the agency a week ago but it has still not been mended Write a letter to the agency Explain the situation and teil them what you want them to do about it

You should write at least 150 words You do NOT need to write your own address

Begin your letter as follows

Dear - ___-I

Writing Task 2 You should spend about 40 minutes on this task As part ofa class assignment you have to write about the following topic

100 CHAPTER 4 Standardized Testing

Some businesses now say that no one can smoke cigarettes in any of their offices Some governments have banned smoking in all public placesThis is a good idea but it takes away some of our freedom

Do you agree or disagree Give reasons for your answer You should write at least 250 words

Speaking In each ofthe three parts of the speaking module a specific function is fulfilled In Part 1 the candidates answer general questions about themselves their homes or families their jobs or studies their interests and a range ofsimilar familiar topic areas This part lasts between four and five minutes In Part 2 the candidate is given a verbal prompt on a card and is asked to talk on a particular topic The candidate has one minute to prepare before speaking at length for between one and two minutes The examiner then asks one or two wind-down questions In Part 3 the examiner and candidate engage in a discusshysion of more abstract issues and concepts which are thematically linked to the topic prompt in Part 2 The discussion lasts between four and five minutes

All interviews are recorded on audiocassette Here is a sample ofa Part 2 topic

Describe a teacher who has greatly influenced you in your education

You shou Id say

where you met them what subject they taught what was special about them

and explain why this person influenced you so much

You will have to talk about the topic for 1 to 2 minutes You have 1 minute to think about what you are going to say You can make some notes if you wish

Test of English for International Communication (TOEICreg)

listening

Part 1 Photographs Directions For each question you will see a picture in your test book and you will hear four short statements The statements will be spoken just one time They will not be printed in your test book so you must listen carefully to understand what the speaker says When you hear the four statements look at the picture in your test book and choose the statement that best describes what you see in the picture Then on your answer sheet find the number of the question and mark your answer

[photograph of a scientist looking through a microscope]

You will hear Look at the picture marked number 1 in your test book

(A) Shes speaking into a microphone (B) Shes put on her glasses (C) She has both eyes open (D) Shes using a microscope

CHAPTER 4 Standardized Testing 101

Part 2 Question-Response Directions In this part of the test you will hear a question or statement spoken in Enshyglish followed by three responses also spoken in English The question or staten1ent and the responses will be spoken just one time They will not be printed in your test book so you must listen carefully to understand what the speakers say You are to choose the best response to each question or statement

Question 1 You will hear Ms Morikawa has worked here for a long time hasnt she

(A) At three oclock (B) No Ive lost my watch (C) More than ten years

Question 2 You will hear Which of these papers has a wider circulation

(A) The morning edition (B) Get more exercise (C) By messenger

Part 3 Short Conversations Directions In this part of the test you will hear short conversations between two people The conversations will not be printed in your test book You will hear the conversations only once so you must listen carefully to understand what the speakers say In your test book you will read a question about each conversation The question will be followed by four answers You are to choose the best answer to each question and mark it on your answer sheet

Question 1 (Man) We should think about finding another restaurant for lunch (Woman) Why The food and service here are great

(Man) Yes but the prices are going up every week

You will read Why is this man unhappy with the restaurant

(A) It is too noisy (B) It is too expensive (C) It is too crowded (D) It is too difficult to find

Question 2 (Woman A) How was Dr Borgs recent trip to Singapore (Woman B) She enjoyed the tour of the port very much (Woman A) They say its one of the most active in Asia

You will read 2 What did Dr Borg find interesting

(A) The tourist center (B) The airport (C) The musical performance (D) The harbor

Part 4 Short Talks Directions In this part of the test you vill hear several short talks Each will be spoken just one time They will not be printed in your test book so you must listen carefully to understand and remember what is said In your test book you will read two or more questions about each short talk The questions will be followed by four answers You are to choose the best answer to each question and mark it on your answer sheet

102 CHAPTER 4 Standardized Testing

You will hear Questions 1 and 2 refer to the following announcement

Good afternoon and welcome aboard Nordair Flight 857 from Copenhagen to Bangkok with intermediate stops in Dubai and Calcutta We are preparing for departure in a few minutes At this time your seat back should be returned to its full upright position and your seat belt s~ould be fastened OUf anticipated total flying time to Dubai is six hours and twenty-five minutes I hope you enjoy the flight You will hecJr Now read question 1 in your test book and answer it You will read 1 What is the final destination of the flight

(A) Bangkok (B) Copenhagen (C) Dubai (O) Calcutta

You will hear Now read question 2 in your test book and answer it You will read 2 What will happen in a few minutes

(A) The flight will land in Dubai I

(B) The passengers will board the plane (C) The plane will take off (0) The gate number will be announced

Reading In this section of the test you will have the chance to show how well you understand written English There are three parts to this section with special directions for each part

Part 4 Incomplete Sentences Directions This part of the test has incomplete sentences Four words or phrases marked (A) (8) (e) (D) are given beneath each sentence You are to choose the one word or phrase that best completes the sentence Then on your answer sheet find the number of the question and mark your answer

1 Mr Yangs trip will __ him away from the office for ten days (A) withdraw (B) continue (C) retain (0) keep

2 The company that Marie DuBois started now sells __ products throughout the world (A) its (B) it (C) theirs (D) them

3 If your shipment is not delivered __ Tuesday you can request a full refund for the merchandise (A) at (B) by (C) within (D) while

CHAPTER 4 Standardized Testing 103

Part 6 Error Recognition Directions In this part ofthe test each sentence has four words or phrases underlined The four underlined parts of the sentence are marked (A) (B) (C) (D) You are to identify the one underlined word or phrase that should be corrected or rewritten Then on your answer sheet find the number of the question and mark your answer

1 The pamphlet contains some importance information about the current exhibit ABC D

2 No matter how Jong it taking to finish the annual report it must be done properly ABC D

3 The popularity of jogging appears to have decreased since the past couple of years ABC D

Part 7 Reading Comprehension Directions The questions in this part of the test are based on a selection of reading mateshyrials such as notices letters) forms newspaper and magazine articles) and advertisements You are to choose the one best answer (A) (B) (C) or (OJ to each quesshytion Then on your ariswefsheelfindthe number of the qUestion andmcirkyour answer Answer all questions following each reading selection on thebasis of what is stated or implied in that selection

The Museum ofTechnology is a hands-on museum designed for people to experience science at w()rk~ Visitors are encouraged to use test and handle the objects o~ display Special demonstrations are scheduled for the first and second Wednesdays of each month at 1330 Open Tuesday-Friday 1200-1630 Saturday 1000-1730 and Sunday 11 00-1630

1 When during the month can visitors see special demonstrations (A) Every weekend (B) The first two Wednesdays (C) One afternoon a week (D) Every other Wednesday

Questions 2 and 3 refer to the followi ng notice

NOTICE If you are unable to work because of an extended illness or injury that is not workshyrelated you may be entitled to receive weekly benefits from your employer or the firms insurance company To claim benefits you must file a claim form within thirty days of the first day of your disability Before filing the claim you must ask your doctor to fill in the Doctors Statement on the claim form stating the period of disability

3 To whom is this notice addressed (A) Employers (8) Doctors (C) Employees (D) When paying the bill

4 When must the claim form be filed (A) On the first of the month (8) On the thirtieth of the month (C) On the first day ofdisabifity (D) Within 30 days of the start of disability

Page 25: Standardized Testing Chapter 4 Brown

bullbullbullbullbullbullbull

90 CHAPTER 4 Standardized Testing

Structure and Written Expression This section measures the ability to recognize language that is appropriate for standard written English There are two types ofquestions in this section In the first type ofquestion there are incomplete sentences Beneath each sentence there are four words or phrases

Directions CIiSk on the one word or phrase that best completes the sentence

The colum~ine flower __ to nearly all of the United States can be raised from seed in almost any garden

native how native is how native is it is native

Time Help Confirm

After you click on Next and Confirm Answ~ the next question willbe presented

The second type of question has four underlined words or phrases You will choose the one underlined word or phrase that must be changed for the sentence to be correct

Directions Click on the one underlined word or phrase that must be changed for the senshytence to be correct

One of the most difficult problems in understanding sleep is determining what the funcshytions of sleep ~

lime Help Confirm

Clicking on an underlined word or phrase will darken it

Reading This section measures the ability to read and understand short passages similar in topic and style to those that students are likely to encounter in North American universities and colleges This section contains reading passages and questions about the passages There are several different types of questions in this section In the Reading section you will first have the opportunity to read the passage

The temperature of the Sun is over 10000 degrees Fahrenheit at the surface but it rises perhaps more than 270000000 at the center The Sun is so much hotter than the Earth that matter can exist only as a gasl except perhaps at the core In the core of the Sun the pressures are so great that despite the high temperature there may be a small solid core However no one really knows since the center of the Sun can never be directly observed ~ Solar astronomers do know that the Sun is divided into five general layers or zones Starting at the outside and going down into the Sun the zones are the corona chromoshysphere hotosphere convection zone and finally the core The first three zones are reshygarded as the Suns atmosphere But since the Sun has no solid surface it is hard to middottell where the atmosphere ends and the main body of the Sun begins

The Suns outermost layer begins about 10000 miles above the visible surface and goes outward for millions of miles This is the only part of the Sun that can be seen during an eclipse such as the one in February 1979 At any other time the corona can be seen

bullbullbullbullbullbullbull

bull bullbullbullbullbullbull

CHAPTER 4 Standardized Testing 91

only when special instruments are used on cameras and telescopes to block the light from the photosphere

The corona is a brilliant pearly white filmy light about as bright as the full Moon Its beautiful rays are a sensational sight during an eclipse The coronas rays flash out in a brilliant fan that has wispy spikelike rays near the Suns north and south poles The corona is generally thickest at the Suns equator The corona is made up of gases streamshying outward at tremendous speeds that reach a temperature of more than 2 million deshygrees Fahrenheit The gas thins out as it reaches the space around the planets By the time the gas of the corona reaches the Earth it has a relatively low density

When you have finished reading the passage you will use the mouse to click on Proceed Then the questions about the passage will be presented You are to choose the one best anshyswer to each question Answer all questions about the information in a passage on the basis ofwhat is stated or implied in that passage Most ofthe questions will be multiple-choice questions To answer these questions you will click on a choice below the question

With what topic is paragraph 2 mainly concerned

o How the Sun evolved o The structure of the Sun o Why scientists study the Sun o The distaflce of the Sun from the planets

Paragraph 2 is marked with an arrow (~)

You will see the next question after you click on Next

To answer some questions you will click on a word or phrase Here is an example

Look at the word one in the passage Click on the word or phrase in the bold text that one refers to To answer you can click on any part of the word or phrase in the passage Jour choice will darken to show which word you have chosen

The Suns outermost layer begins about 10000 miles above the visible surface and goes outward for millions of miles This is the only part of the Sun that can be seen durshying an eclipse such as the one in February 1979 At any other time the corona can be seen only when special instruments are used on cameras and telescopes to block the Iight from the photosphere

You will see the next question after you click on~ To answer some q~estions you will click on a sentence in the passage Here is an example

~ The corona is a brilliant pearly white filmy light about as bright as the full Moon Its beautiful rays are a sensational sight during an eclipse The coronas rays flash out in a brilliant fan that has wispy spikelike rays near the Suns north and south poles The corona is generally thickest at the Suns equator ~ The corona is made up of gases streaming outward at tremendous speeds that reach a temperature of more than 2 million degrees Fahrenheit The gas thins out as it reaches the space around the planets By the time the gas of the corona reaches the Earth it has a relatively low density

bull bullbullbullbullbullbull

92 CHAPTER 4 Standardized Testing

Click on the sentence in paragraph 4 or 5 in which the author compares the light of the Suns outermost layer to that ofanother astronomical body Paragraphs 4 and 5 are marked with arrows (~)

To answer some questions you will click on a square to add a sentence to the passage Here is an example -The following sentence can be added to paragraph 1

At the center of the Earths solar system lies the Sun

Where would it best fit in paragraph I Click on the square to add the sentence to the paragraph

D The temperature of the Sun is over 10000 degrees Fahrenheit at the surface but it rises to perhaps morethan 27000000deg at the center 0 The Sun is so much hotter than the Earth that matter can exist only as a gasi except p~rHapsatth~ c6relp the c~re of the ii Sun the pressures are so great that despite the high temperature there may be a small solid core D However no one really knows since the center of the Sun can never be directly observed D 0100

When you click on a square the sentence will appear in the passage at the place you have chosen You can read the sentence added to the paragraph to see if this is the best place to add it You can click on another square to change your answer The sentence will be added and shown in a dark box

Writing In this section you will have an opportunity to demonstrate your ability to write in Enshyglish This includes the ability to generate and organize ideas to support those ideas with examples or evidence and to compose in standard written English in response to an asshysigned topic You will have 30 minutes to write your essay on that topic You must write on the topic you are assigned An essay on any other topic will receive a score of 0 Read the topic below and then make any notes that will help you plan your response Begin typing your response in the box at the bottom of the screen or write your answer on the answer sheet provided to you

Following is a sample topic

Do you agree or disagree with the following statemenH

Teachers should make learning enjoyable and fun for their students

Use specific reasons and examples to support your opinion

CHAPTER 4 Standarczed Testing 93

Michigan English Language Assessment Battery (MELAB)

Composition The time limit for the composition is 30 minutes You must write on only one of the top~

ics below If you write about something else your composition paper will not be graded and you cannot be given a final score If you do not understand the topics ask the exam~ iner to explain or to translate them You may be asked to give your opinion ofsomething and explain why you believe this to describe something from your experience or to exshyplain a problem and offer possible solutions You should write at least one page Some sample topics are

1 What do you think is your countrys greatest problem Explain in detail and tell what you think can be done about it

2 What are the characteristics of a good teacher Explain and give examples 3 An optimist is someone who sees the good side of things A pessimist sees the

bad side Are you an optimist or a pessimist Relate a personal experience that shows this

4 In your opinion are the benefits of space exploration really worth the enormous costs Discuss

Most MELAB compositions are one or two pages long (about 200-300 words) If your paper is extremely short (less than 150 words) your composition will be given a lower score Before you begin writing you might want to take 2 or 3 minutes to plan your comshyposition and to make a short outline to organize your thoughts Such outlines will not be graded they are only to help you You should use the last 5 minutes to read through your composition and to make changes or corrections

Your composition will be graded on how clearly you express yourself in English and on the range of English you are able to use and your control in doing so This means your composition should be well organized your arguments should be fully developed and you should show a range ofgrammatical structures and broad vocabulary Compositions that consist only of very short sentences and very simple vocabulary cannot be given the

middothighest scores If errors are not frequent and if they do not confuse your meaning they will not lower your score very much

Listening Now you will hear a short lecture You may take notes during the lecture Following the lecture you will be asked some questions about it

Therell be a two-week exhibit of the paintings of the little-known master Laura Bernhart at the Claire Osmond Galleries starting on the fifteenth of the month and running through the thirtieth Bernharts known for her innovative designs in abstract expressionism Though a true original she declared a spiritual heritage from Salvador Dali the famous Spanish painter Since Bernhart lived a rather solitary life and died while only in her twenties few people are aware of her works This showing at the Osmond Galleries will provide many with an introduction to her works

10 Where is the exhibit a the Art Museum b the Dali Galleries c the Osmond Galleries

94 CHAPTER 4 Standardized Testing

11 What is Bernhart known for a her copies of Dalis paintings b the originality of her designs c her exhibitions

12 What will going to the exhibit allow most people to do a to see Saivador Dalis paintings b to see Bernharts works for the first time c to learn about Spanish art

Grammar

1 What did the teacher just tell you

She reminded our notebooksI a us to bring b that we bring c our bringing d we should bring

2 Is Bill a good dancer

Not really __ he tries very hard a in spite of h despite c even though d while

3 your clothes are all wet1

Yes I didnt come __ the rain soon enough a away to b over to c down with d in from

Cloze In years to come zoos will not only be places where animals are exhibited to the public but repositories where rare species can be saved from extinction (7) captive breeding The most powerful force (8) the future of many animals-and of zoos-is the decline of the wild (9) even zoo directors would argue that (10) are better places for animals than the fields and forest of their native (11) yet zoos may be the last chance for some creatures that would otherwise pass qUietly into oblivion

7 a through c from b of d damage

8 a bringing c to b that d influencing

9 a But c Not b So d Then

10 a where c even b zoos d wilds

11 alands c residence b life d field

CHAPTER 4 Standardized Testing 95

Vocabulary

12 Mark has a flair for writing a need b purpose c talent d dislike

13 Bill Collins launched his restaurant last June a moved b started c sold d bought

14 John will not accept the censure a burden b blame c credit d decision

15 I cant think of the answer Can you give me a __ a hint b token c taste d gaze

16 Because fewer people are taking expensive vacations the tourist industry is in a a choke b grope c grumble d slump

17 I disagree with a few of his opinions but __ we agree a deliberately b conclusively c essentially d immensely

Reading The influenza virus is a single molecule built from many millions of single atoms You must have heard of the viruses which are sometimes called living molecules While bacteria can be considered as a type of plant secreting pOisonous substances into the body of the organism they attack viruses are living organisms themselves We may conshysider them as regular chemical molecules since they have a strictly aefined atomic strucshyture but on the other hand we must also consider them as being alive since they are able to multiply in unlimited quantities

18 According to the passage bacteria are a poisons

b larger than viruses c very small d plants

96 CHAPTER 4 Standardized Testing

19 The writer says that viruses are alive because they a have a complex atomic structure b move c multiply d need warmth and light

20 The atomic structure of viruses a is -tJIariable b is strictly defined c cannot be analyzed chemically d is more complex than that of bacteria

International English Language Testing System (fELTS)

I

listening

The Listening Module has four sections The first two sections are concerned with social needs There is a conversation between two speakers and then a monologue For examshyple a conversation about travel arrangements or decisions on a night out and a speech about student services on a university campus or arrangements for meals during a confershyence The final two sections are concerned with situations related more closely to educashytional or training contexts For example conversation between a tutor and a student about an assignment or between three students planning a research project and a lecture or talk ofgeneral academic interest All the topics are ofgeneral interest and it makes no difference what subjects candidates study Tests and tasks become more difficult as the sections progress A range of English accents and dialects are used in the recording which reflects the international usage of IELTS

Academic Reading [A 7S0-word article on-th-e- topic of Wind Power in the US with a short glossary at the end]

Questions 1-5

Complete the summary below

Choose your answers from the box below the summary and write them in boxes 1-5 on your answer sheet Note There are more words or phrases than you will need to fill the gaps You may use any word or phrase more than once

Example The failure during the late 1970s and early 19805 of an attempt to establish a widespread wind power industry in the United States resulted largely from the (1) bull in oil prices during this period The industry is now experiencing a steady (2) due to improveshyments in technology and an increased awareness of the potential in the power of wind The wind turbines that are now being made based in part on the (3) of wide- ranging research in Europe are easier to manufacture and maintain than their predecesshysors This has led wind-turbine makers to be able to standardise and thus minimize (4) There has been growing (S) of the importance of wind power as an energy source

CHAPTER 4 Standardized Testing 97

criticism stability skepticism success operating costs decisions design costs fall effects production costs growth decline failure recognition results

Questions 6-1 0 Look at the following list of issues (Questions 6-10) and implications (A-C) Match each issue with one implication Write the appropriate letters A-C in boxes 6-10 on your anshyswer sheet

Example The current price of one wind-generated kilowatt Answer

6 The recent installation of systems taking advantage of economies of scale

7 The potential of meeting one fifth of current U5 energy requirements by wind power

8 The level of acceptance of current wind turbine technology

9 A comparison of costs between conventional and wind power sources

10 The view of wind power in the European Union

Implications

A provides evidence against claims that electricity produced from wind power is relatively expensive

B supports claims that wind power js an important source of energy

C opposes the view that wind power technology requires further-development

General Training Reading Read the passage on Daybreak trips by coach and look at the statements below On your answer sheet write

TRUE if the statement is true FALSE jf the statement is false

NOlGIVEN if the information is not given in the leaflet

1 MiIlers Coaches owns Cambridges Cam bus fleet

2 Premier is an older company than Millers

3 Most of the Daybreak coaches are less than 5 years old

4 Daybreak fares are more expensive than most of their competitors

5 Soft drinks and refreshments are served on most longer journeys

6 Smoking is permitted at the rear of the coach on longer journeys

7 Tickets must be bought in advance from an authorised Daybreak agent

6 Tickets and seats can be reserved by phoning the Daybreak Hotline

9 Daybreak passengers must join their coach at Cambridge Drummer Street

10 Daybreak cannot guarantee return times

98 CHAPTER 4 Standardized Testing

FROM CAMBRIDGE AND SURROUNDING AREA

SPRING IS INTHEAIR

Welcome to our Spring Daybreak programme which continues the tradition of offering unbeatable value for money day trips and tours All the excursions in this brochure will be operated by Pr~mier Travel Services Limited or Millers Coaches both companies are part of the CHLGroup owners of Cambridges Cambus fleet

WERE PROUD OF OUR TRADITION

Premier was established in 1936 the Company now offers the highest standards of coaching in todays competitive operating environment Miller has an enviable reputation stretching back over the past 20 years offering coach services at realistic prices Weve traveled a long way since our early days of pre-war seaside trips Now our fleet of 50 modern coaches (few are more than five years old) operate throughout Britain and Europe but were pleased to still maintain the high standards of quality and service the trademark of our founders nearly sixty years ago

EXCLUSIVE FEATURES

Admission-inclusive fares All Daybreak fares (unless specifically otherwise stated) include admission charges to the attractions shows and exhibits we visit Many full-day scenic tours are accompanied by a fully trained English Tourist Board Blue Badge guide or local experienced driverguide Some Daybreaks include lunch or afternoon tea Compare our admission inclusive fares and see how much you save Cheapest is not the best and value for money is guaranteed If you compare our bargain Daybreak fares beware--most of our competishytors do not offer an all-inclusive fare

SEAT RESERVATIONS

We value the freedom of choice so you can choose your seat when you book The seat reservation is guaranteed a-nd remains yours at all times when aboard the coach

NO SMOKING COMFORT

With the comfort of our passengers in mind coaches on all our Daybreaks are no smokshying throughout In the interests of fellow passengers comfort we kindly ask that smokers observe our no smoking policy On scenic tours and longer journeys ample refreshment stops are provided when of course smoking is permitted

YOUR QUESTIONS ANSWERED

Do I need to book Booking in advance is strongly recommended as all Daybreak tours are subject to demand Subject to availability stand-by tickets can be purchased from the driver

What ti me does the coach leave The coach departs from Cambridge Drummer Street (Bay 12 adjacent to public toilets) at the time shown There are many additional joining points indicated by departure codes in the brochure If you are joining at one of our less popular joining points you will be adshyvised of your pick-up time (normally by telephone) not less than 48 hours before deparshyture In this way we can minimize the length of pick-up routes and reduce journey times for the majority of passengers

CHAPTER 4 Standardized Testing 99

What time do we get back An approximate return time is shown for each excursion The tim~s shown serve as a guide but road conditions can sometimes cause delay If your arrival will be later than advertised your driver will try to allow for a telephone call during the return journey

Where can I board the coach All the Daybreaks in the brochure leave from Cambridge Drummer Street (Bay 12 adjashycent to public toilets) at the time shown Many Daybreaks offer additional pick-ups for pre-booked passengers within Cambridge and the surrounding area This facility must be requested at the time of booking

Academic Writing Writing Task 1 You should spend about 20 minutes on this task

The graph below shows the different modes of transport used to travel to and from work in one European city in 1950 1970 and 1990

[graph shown here]

Write a report for a university lecturer describing the information shown below You should write at least 150 words

Writing Task 2 You should spend about 40 minutes on this task

Present a written argument or case to an educated reader with no specialist knowledge of the folowing topic

It is inevitable that as technology develops so traditional cultures must be lost Technolshyogy and tradition are incompatible-you cannot have both together

To what extent do you agree or disagree with this statement Give reasons for your answer You should write at least 250 words You should use your own ideas knowlshyedge and experience and support your arguments with examples and relevant evidence

General Training Writing Writing Task 1 You should spend about 20 minutes on this task You rent a house through an agency The heating system has stopped working You phoned the agency a week ago but it has still not been mended Write a letter to the agency Explain the situation and teil them what you want them to do about it

You should write at least 150 words You do NOT need to write your own address

Begin your letter as follows

Dear - ___-I

Writing Task 2 You should spend about 40 minutes on this task As part ofa class assignment you have to write about the following topic

100 CHAPTER 4 Standardized Testing

Some businesses now say that no one can smoke cigarettes in any of their offices Some governments have banned smoking in all public placesThis is a good idea but it takes away some of our freedom

Do you agree or disagree Give reasons for your answer You should write at least 250 words

Speaking In each ofthe three parts of the speaking module a specific function is fulfilled In Part 1 the candidates answer general questions about themselves their homes or families their jobs or studies their interests and a range ofsimilar familiar topic areas This part lasts between four and five minutes In Part 2 the candidate is given a verbal prompt on a card and is asked to talk on a particular topic The candidate has one minute to prepare before speaking at length for between one and two minutes The examiner then asks one or two wind-down questions In Part 3 the examiner and candidate engage in a discusshysion of more abstract issues and concepts which are thematically linked to the topic prompt in Part 2 The discussion lasts between four and five minutes

All interviews are recorded on audiocassette Here is a sample ofa Part 2 topic

Describe a teacher who has greatly influenced you in your education

You shou Id say

where you met them what subject they taught what was special about them

and explain why this person influenced you so much

You will have to talk about the topic for 1 to 2 minutes You have 1 minute to think about what you are going to say You can make some notes if you wish

Test of English for International Communication (TOEICreg)

listening

Part 1 Photographs Directions For each question you will see a picture in your test book and you will hear four short statements The statements will be spoken just one time They will not be printed in your test book so you must listen carefully to understand what the speaker says When you hear the four statements look at the picture in your test book and choose the statement that best describes what you see in the picture Then on your answer sheet find the number of the question and mark your answer

[photograph of a scientist looking through a microscope]

You will hear Look at the picture marked number 1 in your test book

(A) Shes speaking into a microphone (B) Shes put on her glasses (C) She has both eyes open (D) Shes using a microscope

CHAPTER 4 Standardized Testing 101

Part 2 Question-Response Directions In this part of the test you will hear a question or statement spoken in Enshyglish followed by three responses also spoken in English The question or staten1ent and the responses will be spoken just one time They will not be printed in your test book so you must listen carefully to understand what the speakers say You are to choose the best response to each question or statement

Question 1 You will hear Ms Morikawa has worked here for a long time hasnt she

(A) At three oclock (B) No Ive lost my watch (C) More than ten years

Question 2 You will hear Which of these papers has a wider circulation

(A) The morning edition (B) Get more exercise (C) By messenger

Part 3 Short Conversations Directions In this part of the test you will hear short conversations between two people The conversations will not be printed in your test book You will hear the conversations only once so you must listen carefully to understand what the speakers say In your test book you will read a question about each conversation The question will be followed by four answers You are to choose the best answer to each question and mark it on your answer sheet

Question 1 (Man) We should think about finding another restaurant for lunch (Woman) Why The food and service here are great

(Man) Yes but the prices are going up every week

You will read Why is this man unhappy with the restaurant

(A) It is too noisy (B) It is too expensive (C) It is too crowded (D) It is too difficult to find

Question 2 (Woman A) How was Dr Borgs recent trip to Singapore (Woman B) She enjoyed the tour of the port very much (Woman A) They say its one of the most active in Asia

You will read 2 What did Dr Borg find interesting

(A) The tourist center (B) The airport (C) The musical performance (D) The harbor

Part 4 Short Talks Directions In this part of the test you vill hear several short talks Each will be spoken just one time They will not be printed in your test book so you must listen carefully to understand and remember what is said In your test book you will read two or more questions about each short talk The questions will be followed by four answers You are to choose the best answer to each question and mark it on your answer sheet

102 CHAPTER 4 Standardized Testing

You will hear Questions 1 and 2 refer to the following announcement

Good afternoon and welcome aboard Nordair Flight 857 from Copenhagen to Bangkok with intermediate stops in Dubai and Calcutta We are preparing for departure in a few minutes At this time your seat back should be returned to its full upright position and your seat belt s~ould be fastened OUf anticipated total flying time to Dubai is six hours and twenty-five minutes I hope you enjoy the flight You will hecJr Now read question 1 in your test book and answer it You will read 1 What is the final destination of the flight

(A) Bangkok (B) Copenhagen (C) Dubai (O) Calcutta

You will hear Now read question 2 in your test book and answer it You will read 2 What will happen in a few minutes

(A) The flight will land in Dubai I

(B) The passengers will board the plane (C) The plane will take off (0) The gate number will be announced

Reading In this section of the test you will have the chance to show how well you understand written English There are three parts to this section with special directions for each part

Part 4 Incomplete Sentences Directions This part of the test has incomplete sentences Four words or phrases marked (A) (8) (e) (D) are given beneath each sentence You are to choose the one word or phrase that best completes the sentence Then on your answer sheet find the number of the question and mark your answer

1 Mr Yangs trip will __ him away from the office for ten days (A) withdraw (B) continue (C) retain (0) keep

2 The company that Marie DuBois started now sells __ products throughout the world (A) its (B) it (C) theirs (D) them

3 If your shipment is not delivered __ Tuesday you can request a full refund for the merchandise (A) at (B) by (C) within (D) while

CHAPTER 4 Standardized Testing 103

Part 6 Error Recognition Directions In this part ofthe test each sentence has four words or phrases underlined The four underlined parts of the sentence are marked (A) (B) (C) (D) You are to identify the one underlined word or phrase that should be corrected or rewritten Then on your answer sheet find the number of the question and mark your answer

1 The pamphlet contains some importance information about the current exhibit ABC D

2 No matter how Jong it taking to finish the annual report it must be done properly ABC D

3 The popularity of jogging appears to have decreased since the past couple of years ABC D

Part 7 Reading Comprehension Directions The questions in this part of the test are based on a selection of reading mateshyrials such as notices letters) forms newspaper and magazine articles) and advertisements You are to choose the one best answer (A) (B) (C) or (OJ to each quesshytion Then on your ariswefsheelfindthe number of the qUestion andmcirkyour answer Answer all questions following each reading selection on thebasis of what is stated or implied in that selection

The Museum ofTechnology is a hands-on museum designed for people to experience science at w()rk~ Visitors are encouraged to use test and handle the objects o~ display Special demonstrations are scheduled for the first and second Wednesdays of each month at 1330 Open Tuesday-Friday 1200-1630 Saturday 1000-1730 and Sunday 11 00-1630

1 When during the month can visitors see special demonstrations (A) Every weekend (B) The first two Wednesdays (C) One afternoon a week (D) Every other Wednesday

Questions 2 and 3 refer to the followi ng notice

NOTICE If you are unable to work because of an extended illness or injury that is not workshyrelated you may be entitled to receive weekly benefits from your employer or the firms insurance company To claim benefits you must file a claim form within thirty days of the first day of your disability Before filing the claim you must ask your doctor to fill in the Doctors Statement on the claim form stating the period of disability

3 To whom is this notice addressed (A) Employers (8) Doctors (C) Employees (D) When paying the bill

4 When must the claim form be filed (A) On the first of the month (8) On the thirtieth of the month (C) On the first day ofdisabifity (D) Within 30 days of the start of disability

Page 26: Standardized Testing Chapter 4 Brown

bullbullbullbullbullbullbull

bull bullbullbullbullbullbull

CHAPTER 4 Standardized Testing 91

only when special instruments are used on cameras and telescopes to block the light from the photosphere

The corona is a brilliant pearly white filmy light about as bright as the full Moon Its beautiful rays are a sensational sight during an eclipse The coronas rays flash out in a brilliant fan that has wispy spikelike rays near the Suns north and south poles The corona is generally thickest at the Suns equator The corona is made up of gases streamshying outward at tremendous speeds that reach a temperature of more than 2 million deshygrees Fahrenheit The gas thins out as it reaches the space around the planets By the time the gas of the corona reaches the Earth it has a relatively low density

When you have finished reading the passage you will use the mouse to click on Proceed Then the questions about the passage will be presented You are to choose the one best anshyswer to each question Answer all questions about the information in a passage on the basis ofwhat is stated or implied in that passage Most ofthe questions will be multiple-choice questions To answer these questions you will click on a choice below the question

With what topic is paragraph 2 mainly concerned

o How the Sun evolved o The structure of the Sun o Why scientists study the Sun o The distaflce of the Sun from the planets

Paragraph 2 is marked with an arrow (~)

You will see the next question after you click on Next

To answer some questions you will click on a word or phrase Here is an example

Look at the word one in the passage Click on the word or phrase in the bold text that one refers to To answer you can click on any part of the word or phrase in the passage Jour choice will darken to show which word you have chosen

The Suns outermost layer begins about 10000 miles above the visible surface and goes outward for millions of miles This is the only part of the Sun that can be seen durshying an eclipse such as the one in February 1979 At any other time the corona can be seen only when special instruments are used on cameras and telescopes to block the Iight from the photosphere

You will see the next question after you click on~ To answer some q~estions you will click on a sentence in the passage Here is an example

~ The corona is a brilliant pearly white filmy light about as bright as the full Moon Its beautiful rays are a sensational sight during an eclipse The coronas rays flash out in a brilliant fan that has wispy spikelike rays near the Suns north and south poles The corona is generally thickest at the Suns equator ~ The corona is made up of gases streaming outward at tremendous speeds that reach a temperature of more than 2 million degrees Fahrenheit The gas thins out as it reaches the space around the planets By the time the gas of the corona reaches the Earth it has a relatively low density

bull bullbullbullbullbullbull

92 CHAPTER 4 Standardized Testing

Click on the sentence in paragraph 4 or 5 in which the author compares the light of the Suns outermost layer to that ofanother astronomical body Paragraphs 4 and 5 are marked with arrows (~)

To answer some questions you will click on a square to add a sentence to the passage Here is an example -The following sentence can be added to paragraph 1

At the center of the Earths solar system lies the Sun

Where would it best fit in paragraph I Click on the square to add the sentence to the paragraph

D The temperature of the Sun is over 10000 degrees Fahrenheit at the surface but it rises to perhaps morethan 27000000deg at the center 0 The Sun is so much hotter than the Earth that matter can exist only as a gasi except p~rHapsatth~ c6relp the c~re of the ii Sun the pressures are so great that despite the high temperature there may be a small solid core D However no one really knows since the center of the Sun can never be directly observed D 0100

When you click on a square the sentence will appear in the passage at the place you have chosen You can read the sentence added to the paragraph to see if this is the best place to add it You can click on another square to change your answer The sentence will be added and shown in a dark box

Writing In this section you will have an opportunity to demonstrate your ability to write in Enshyglish This includes the ability to generate and organize ideas to support those ideas with examples or evidence and to compose in standard written English in response to an asshysigned topic You will have 30 minutes to write your essay on that topic You must write on the topic you are assigned An essay on any other topic will receive a score of 0 Read the topic below and then make any notes that will help you plan your response Begin typing your response in the box at the bottom of the screen or write your answer on the answer sheet provided to you

Following is a sample topic

Do you agree or disagree with the following statemenH

Teachers should make learning enjoyable and fun for their students

Use specific reasons and examples to support your opinion

CHAPTER 4 Standarczed Testing 93

Michigan English Language Assessment Battery (MELAB)

Composition The time limit for the composition is 30 minutes You must write on only one of the top~

ics below If you write about something else your composition paper will not be graded and you cannot be given a final score If you do not understand the topics ask the exam~ iner to explain or to translate them You may be asked to give your opinion ofsomething and explain why you believe this to describe something from your experience or to exshyplain a problem and offer possible solutions You should write at least one page Some sample topics are

1 What do you think is your countrys greatest problem Explain in detail and tell what you think can be done about it

2 What are the characteristics of a good teacher Explain and give examples 3 An optimist is someone who sees the good side of things A pessimist sees the

bad side Are you an optimist or a pessimist Relate a personal experience that shows this

4 In your opinion are the benefits of space exploration really worth the enormous costs Discuss

Most MELAB compositions are one or two pages long (about 200-300 words) If your paper is extremely short (less than 150 words) your composition will be given a lower score Before you begin writing you might want to take 2 or 3 minutes to plan your comshyposition and to make a short outline to organize your thoughts Such outlines will not be graded they are only to help you You should use the last 5 minutes to read through your composition and to make changes or corrections

Your composition will be graded on how clearly you express yourself in English and on the range of English you are able to use and your control in doing so This means your composition should be well organized your arguments should be fully developed and you should show a range ofgrammatical structures and broad vocabulary Compositions that consist only of very short sentences and very simple vocabulary cannot be given the

middothighest scores If errors are not frequent and if they do not confuse your meaning they will not lower your score very much

Listening Now you will hear a short lecture You may take notes during the lecture Following the lecture you will be asked some questions about it

Therell be a two-week exhibit of the paintings of the little-known master Laura Bernhart at the Claire Osmond Galleries starting on the fifteenth of the month and running through the thirtieth Bernharts known for her innovative designs in abstract expressionism Though a true original she declared a spiritual heritage from Salvador Dali the famous Spanish painter Since Bernhart lived a rather solitary life and died while only in her twenties few people are aware of her works This showing at the Osmond Galleries will provide many with an introduction to her works

10 Where is the exhibit a the Art Museum b the Dali Galleries c the Osmond Galleries

94 CHAPTER 4 Standardized Testing

11 What is Bernhart known for a her copies of Dalis paintings b the originality of her designs c her exhibitions

12 What will going to the exhibit allow most people to do a to see Saivador Dalis paintings b to see Bernharts works for the first time c to learn about Spanish art

Grammar

1 What did the teacher just tell you

She reminded our notebooksI a us to bring b that we bring c our bringing d we should bring

2 Is Bill a good dancer

Not really __ he tries very hard a in spite of h despite c even though d while

3 your clothes are all wet1

Yes I didnt come __ the rain soon enough a away to b over to c down with d in from

Cloze In years to come zoos will not only be places where animals are exhibited to the public but repositories where rare species can be saved from extinction (7) captive breeding The most powerful force (8) the future of many animals-and of zoos-is the decline of the wild (9) even zoo directors would argue that (10) are better places for animals than the fields and forest of their native (11) yet zoos may be the last chance for some creatures that would otherwise pass qUietly into oblivion

7 a through c from b of d damage

8 a bringing c to b that d influencing

9 a But c Not b So d Then

10 a where c even b zoos d wilds

11 alands c residence b life d field

CHAPTER 4 Standardized Testing 95

Vocabulary

12 Mark has a flair for writing a need b purpose c talent d dislike

13 Bill Collins launched his restaurant last June a moved b started c sold d bought

14 John will not accept the censure a burden b blame c credit d decision

15 I cant think of the answer Can you give me a __ a hint b token c taste d gaze

16 Because fewer people are taking expensive vacations the tourist industry is in a a choke b grope c grumble d slump

17 I disagree with a few of his opinions but __ we agree a deliberately b conclusively c essentially d immensely

Reading The influenza virus is a single molecule built from many millions of single atoms You must have heard of the viruses which are sometimes called living molecules While bacteria can be considered as a type of plant secreting pOisonous substances into the body of the organism they attack viruses are living organisms themselves We may conshysider them as regular chemical molecules since they have a strictly aefined atomic strucshyture but on the other hand we must also consider them as being alive since they are able to multiply in unlimited quantities

18 According to the passage bacteria are a poisons

b larger than viruses c very small d plants

96 CHAPTER 4 Standardized Testing

19 The writer says that viruses are alive because they a have a complex atomic structure b move c multiply d need warmth and light

20 The atomic structure of viruses a is -tJIariable b is strictly defined c cannot be analyzed chemically d is more complex than that of bacteria

International English Language Testing System (fELTS)

I

listening

The Listening Module has four sections The first two sections are concerned with social needs There is a conversation between two speakers and then a monologue For examshyple a conversation about travel arrangements or decisions on a night out and a speech about student services on a university campus or arrangements for meals during a confershyence The final two sections are concerned with situations related more closely to educashytional or training contexts For example conversation between a tutor and a student about an assignment or between three students planning a research project and a lecture or talk ofgeneral academic interest All the topics are ofgeneral interest and it makes no difference what subjects candidates study Tests and tasks become more difficult as the sections progress A range of English accents and dialects are used in the recording which reflects the international usage of IELTS

Academic Reading [A 7S0-word article on-th-e- topic of Wind Power in the US with a short glossary at the end]

Questions 1-5

Complete the summary below

Choose your answers from the box below the summary and write them in boxes 1-5 on your answer sheet Note There are more words or phrases than you will need to fill the gaps You may use any word or phrase more than once

Example The failure during the late 1970s and early 19805 of an attempt to establish a widespread wind power industry in the United States resulted largely from the (1) bull in oil prices during this period The industry is now experiencing a steady (2) due to improveshyments in technology and an increased awareness of the potential in the power of wind The wind turbines that are now being made based in part on the (3) of wide- ranging research in Europe are easier to manufacture and maintain than their predecesshysors This has led wind-turbine makers to be able to standardise and thus minimize (4) There has been growing (S) of the importance of wind power as an energy source

CHAPTER 4 Standardized Testing 97

criticism stability skepticism success operating costs decisions design costs fall effects production costs growth decline failure recognition results

Questions 6-1 0 Look at the following list of issues (Questions 6-10) and implications (A-C) Match each issue with one implication Write the appropriate letters A-C in boxes 6-10 on your anshyswer sheet

Example The current price of one wind-generated kilowatt Answer

6 The recent installation of systems taking advantage of economies of scale

7 The potential of meeting one fifth of current U5 energy requirements by wind power

8 The level of acceptance of current wind turbine technology

9 A comparison of costs between conventional and wind power sources

10 The view of wind power in the European Union

Implications

A provides evidence against claims that electricity produced from wind power is relatively expensive

B supports claims that wind power js an important source of energy

C opposes the view that wind power technology requires further-development

General Training Reading Read the passage on Daybreak trips by coach and look at the statements below On your answer sheet write

TRUE if the statement is true FALSE jf the statement is false

NOlGIVEN if the information is not given in the leaflet

1 MiIlers Coaches owns Cambridges Cam bus fleet

2 Premier is an older company than Millers

3 Most of the Daybreak coaches are less than 5 years old

4 Daybreak fares are more expensive than most of their competitors

5 Soft drinks and refreshments are served on most longer journeys

6 Smoking is permitted at the rear of the coach on longer journeys

7 Tickets must be bought in advance from an authorised Daybreak agent

6 Tickets and seats can be reserved by phoning the Daybreak Hotline

9 Daybreak passengers must join their coach at Cambridge Drummer Street

10 Daybreak cannot guarantee return times

98 CHAPTER 4 Standardized Testing

FROM CAMBRIDGE AND SURROUNDING AREA

SPRING IS INTHEAIR

Welcome to our Spring Daybreak programme which continues the tradition of offering unbeatable value for money day trips and tours All the excursions in this brochure will be operated by Pr~mier Travel Services Limited or Millers Coaches both companies are part of the CHLGroup owners of Cambridges Cambus fleet

WERE PROUD OF OUR TRADITION

Premier was established in 1936 the Company now offers the highest standards of coaching in todays competitive operating environment Miller has an enviable reputation stretching back over the past 20 years offering coach services at realistic prices Weve traveled a long way since our early days of pre-war seaside trips Now our fleet of 50 modern coaches (few are more than five years old) operate throughout Britain and Europe but were pleased to still maintain the high standards of quality and service the trademark of our founders nearly sixty years ago

EXCLUSIVE FEATURES

Admission-inclusive fares All Daybreak fares (unless specifically otherwise stated) include admission charges to the attractions shows and exhibits we visit Many full-day scenic tours are accompanied by a fully trained English Tourist Board Blue Badge guide or local experienced driverguide Some Daybreaks include lunch or afternoon tea Compare our admission inclusive fares and see how much you save Cheapest is not the best and value for money is guaranteed If you compare our bargain Daybreak fares beware--most of our competishytors do not offer an all-inclusive fare

SEAT RESERVATIONS

We value the freedom of choice so you can choose your seat when you book The seat reservation is guaranteed a-nd remains yours at all times when aboard the coach

NO SMOKING COMFORT

With the comfort of our passengers in mind coaches on all our Daybreaks are no smokshying throughout In the interests of fellow passengers comfort we kindly ask that smokers observe our no smoking policy On scenic tours and longer journeys ample refreshment stops are provided when of course smoking is permitted

YOUR QUESTIONS ANSWERED

Do I need to book Booking in advance is strongly recommended as all Daybreak tours are subject to demand Subject to availability stand-by tickets can be purchased from the driver

What ti me does the coach leave The coach departs from Cambridge Drummer Street (Bay 12 adjacent to public toilets) at the time shown There are many additional joining points indicated by departure codes in the brochure If you are joining at one of our less popular joining points you will be adshyvised of your pick-up time (normally by telephone) not less than 48 hours before deparshyture In this way we can minimize the length of pick-up routes and reduce journey times for the majority of passengers

CHAPTER 4 Standardized Testing 99

What time do we get back An approximate return time is shown for each excursion The tim~s shown serve as a guide but road conditions can sometimes cause delay If your arrival will be later than advertised your driver will try to allow for a telephone call during the return journey

Where can I board the coach All the Daybreaks in the brochure leave from Cambridge Drummer Street (Bay 12 adjashycent to public toilets) at the time shown Many Daybreaks offer additional pick-ups for pre-booked passengers within Cambridge and the surrounding area This facility must be requested at the time of booking

Academic Writing Writing Task 1 You should spend about 20 minutes on this task

The graph below shows the different modes of transport used to travel to and from work in one European city in 1950 1970 and 1990

[graph shown here]

Write a report for a university lecturer describing the information shown below You should write at least 150 words

Writing Task 2 You should spend about 40 minutes on this task

Present a written argument or case to an educated reader with no specialist knowledge of the folowing topic

It is inevitable that as technology develops so traditional cultures must be lost Technolshyogy and tradition are incompatible-you cannot have both together

To what extent do you agree or disagree with this statement Give reasons for your answer You should write at least 250 words You should use your own ideas knowlshyedge and experience and support your arguments with examples and relevant evidence

General Training Writing Writing Task 1 You should spend about 20 minutes on this task You rent a house through an agency The heating system has stopped working You phoned the agency a week ago but it has still not been mended Write a letter to the agency Explain the situation and teil them what you want them to do about it

You should write at least 150 words You do NOT need to write your own address

Begin your letter as follows

Dear - ___-I

Writing Task 2 You should spend about 40 minutes on this task As part ofa class assignment you have to write about the following topic

100 CHAPTER 4 Standardized Testing

Some businesses now say that no one can smoke cigarettes in any of their offices Some governments have banned smoking in all public placesThis is a good idea but it takes away some of our freedom

Do you agree or disagree Give reasons for your answer You should write at least 250 words

Speaking In each ofthe three parts of the speaking module a specific function is fulfilled In Part 1 the candidates answer general questions about themselves their homes or families their jobs or studies their interests and a range ofsimilar familiar topic areas This part lasts between four and five minutes In Part 2 the candidate is given a verbal prompt on a card and is asked to talk on a particular topic The candidate has one minute to prepare before speaking at length for between one and two minutes The examiner then asks one or two wind-down questions In Part 3 the examiner and candidate engage in a discusshysion of more abstract issues and concepts which are thematically linked to the topic prompt in Part 2 The discussion lasts between four and five minutes

All interviews are recorded on audiocassette Here is a sample ofa Part 2 topic

Describe a teacher who has greatly influenced you in your education

You shou Id say

where you met them what subject they taught what was special about them

and explain why this person influenced you so much

You will have to talk about the topic for 1 to 2 minutes You have 1 minute to think about what you are going to say You can make some notes if you wish

Test of English for International Communication (TOEICreg)

listening

Part 1 Photographs Directions For each question you will see a picture in your test book and you will hear four short statements The statements will be spoken just one time They will not be printed in your test book so you must listen carefully to understand what the speaker says When you hear the four statements look at the picture in your test book and choose the statement that best describes what you see in the picture Then on your answer sheet find the number of the question and mark your answer

[photograph of a scientist looking through a microscope]

You will hear Look at the picture marked number 1 in your test book

(A) Shes speaking into a microphone (B) Shes put on her glasses (C) She has both eyes open (D) Shes using a microscope

CHAPTER 4 Standardized Testing 101

Part 2 Question-Response Directions In this part of the test you will hear a question or statement spoken in Enshyglish followed by three responses also spoken in English The question or staten1ent and the responses will be spoken just one time They will not be printed in your test book so you must listen carefully to understand what the speakers say You are to choose the best response to each question or statement

Question 1 You will hear Ms Morikawa has worked here for a long time hasnt she

(A) At three oclock (B) No Ive lost my watch (C) More than ten years

Question 2 You will hear Which of these papers has a wider circulation

(A) The morning edition (B) Get more exercise (C) By messenger

Part 3 Short Conversations Directions In this part of the test you will hear short conversations between two people The conversations will not be printed in your test book You will hear the conversations only once so you must listen carefully to understand what the speakers say In your test book you will read a question about each conversation The question will be followed by four answers You are to choose the best answer to each question and mark it on your answer sheet

Question 1 (Man) We should think about finding another restaurant for lunch (Woman) Why The food and service here are great

(Man) Yes but the prices are going up every week

You will read Why is this man unhappy with the restaurant

(A) It is too noisy (B) It is too expensive (C) It is too crowded (D) It is too difficult to find

Question 2 (Woman A) How was Dr Borgs recent trip to Singapore (Woman B) She enjoyed the tour of the port very much (Woman A) They say its one of the most active in Asia

You will read 2 What did Dr Borg find interesting

(A) The tourist center (B) The airport (C) The musical performance (D) The harbor

Part 4 Short Talks Directions In this part of the test you vill hear several short talks Each will be spoken just one time They will not be printed in your test book so you must listen carefully to understand and remember what is said In your test book you will read two or more questions about each short talk The questions will be followed by four answers You are to choose the best answer to each question and mark it on your answer sheet

102 CHAPTER 4 Standardized Testing

You will hear Questions 1 and 2 refer to the following announcement

Good afternoon and welcome aboard Nordair Flight 857 from Copenhagen to Bangkok with intermediate stops in Dubai and Calcutta We are preparing for departure in a few minutes At this time your seat back should be returned to its full upright position and your seat belt s~ould be fastened OUf anticipated total flying time to Dubai is six hours and twenty-five minutes I hope you enjoy the flight You will hecJr Now read question 1 in your test book and answer it You will read 1 What is the final destination of the flight

(A) Bangkok (B) Copenhagen (C) Dubai (O) Calcutta

You will hear Now read question 2 in your test book and answer it You will read 2 What will happen in a few minutes

(A) The flight will land in Dubai I

(B) The passengers will board the plane (C) The plane will take off (0) The gate number will be announced

Reading In this section of the test you will have the chance to show how well you understand written English There are three parts to this section with special directions for each part

Part 4 Incomplete Sentences Directions This part of the test has incomplete sentences Four words or phrases marked (A) (8) (e) (D) are given beneath each sentence You are to choose the one word or phrase that best completes the sentence Then on your answer sheet find the number of the question and mark your answer

1 Mr Yangs trip will __ him away from the office for ten days (A) withdraw (B) continue (C) retain (0) keep

2 The company that Marie DuBois started now sells __ products throughout the world (A) its (B) it (C) theirs (D) them

3 If your shipment is not delivered __ Tuesday you can request a full refund for the merchandise (A) at (B) by (C) within (D) while

CHAPTER 4 Standardized Testing 103

Part 6 Error Recognition Directions In this part ofthe test each sentence has four words or phrases underlined The four underlined parts of the sentence are marked (A) (B) (C) (D) You are to identify the one underlined word or phrase that should be corrected or rewritten Then on your answer sheet find the number of the question and mark your answer

1 The pamphlet contains some importance information about the current exhibit ABC D

2 No matter how Jong it taking to finish the annual report it must be done properly ABC D

3 The popularity of jogging appears to have decreased since the past couple of years ABC D

Part 7 Reading Comprehension Directions The questions in this part of the test are based on a selection of reading mateshyrials such as notices letters) forms newspaper and magazine articles) and advertisements You are to choose the one best answer (A) (B) (C) or (OJ to each quesshytion Then on your ariswefsheelfindthe number of the qUestion andmcirkyour answer Answer all questions following each reading selection on thebasis of what is stated or implied in that selection

The Museum ofTechnology is a hands-on museum designed for people to experience science at w()rk~ Visitors are encouraged to use test and handle the objects o~ display Special demonstrations are scheduled for the first and second Wednesdays of each month at 1330 Open Tuesday-Friday 1200-1630 Saturday 1000-1730 and Sunday 11 00-1630

1 When during the month can visitors see special demonstrations (A) Every weekend (B) The first two Wednesdays (C) One afternoon a week (D) Every other Wednesday

Questions 2 and 3 refer to the followi ng notice

NOTICE If you are unable to work because of an extended illness or injury that is not workshyrelated you may be entitled to receive weekly benefits from your employer or the firms insurance company To claim benefits you must file a claim form within thirty days of the first day of your disability Before filing the claim you must ask your doctor to fill in the Doctors Statement on the claim form stating the period of disability

3 To whom is this notice addressed (A) Employers (8) Doctors (C) Employees (D) When paying the bill

4 When must the claim form be filed (A) On the first of the month (8) On the thirtieth of the month (C) On the first day ofdisabifity (D) Within 30 days of the start of disability

Page 27: Standardized Testing Chapter 4 Brown

bull bullbullbullbullbullbull

92 CHAPTER 4 Standardized Testing

Click on the sentence in paragraph 4 or 5 in which the author compares the light of the Suns outermost layer to that ofanother astronomical body Paragraphs 4 and 5 are marked with arrows (~)

To answer some questions you will click on a square to add a sentence to the passage Here is an example -The following sentence can be added to paragraph 1

At the center of the Earths solar system lies the Sun

Where would it best fit in paragraph I Click on the square to add the sentence to the paragraph

D The temperature of the Sun is over 10000 degrees Fahrenheit at the surface but it rises to perhaps morethan 27000000deg at the center 0 The Sun is so much hotter than the Earth that matter can exist only as a gasi except p~rHapsatth~ c6relp the c~re of the ii Sun the pressures are so great that despite the high temperature there may be a small solid core D However no one really knows since the center of the Sun can never be directly observed D 0100

When you click on a square the sentence will appear in the passage at the place you have chosen You can read the sentence added to the paragraph to see if this is the best place to add it You can click on another square to change your answer The sentence will be added and shown in a dark box

Writing In this section you will have an opportunity to demonstrate your ability to write in Enshyglish This includes the ability to generate and organize ideas to support those ideas with examples or evidence and to compose in standard written English in response to an asshysigned topic You will have 30 minutes to write your essay on that topic You must write on the topic you are assigned An essay on any other topic will receive a score of 0 Read the topic below and then make any notes that will help you plan your response Begin typing your response in the box at the bottom of the screen or write your answer on the answer sheet provided to you

Following is a sample topic

Do you agree or disagree with the following statemenH

Teachers should make learning enjoyable and fun for their students

Use specific reasons and examples to support your opinion

CHAPTER 4 Standarczed Testing 93

Michigan English Language Assessment Battery (MELAB)

Composition The time limit for the composition is 30 minutes You must write on only one of the top~

ics below If you write about something else your composition paper will not be graded and you cannot be given a final score If you do not understand the topics ask the exam~ iner to explain or to translate them You may be asked to give your opinion ofsomething and explain why you believe this to describe something from your experience or to exshyplain a problem and offer possible solutions You should write at least one page Some sample topics are

1 What do you think is your countrys greatest problem Explain in detail and tell what you think can be done about it

2 What are the characteristics of a good teacher Explain and give examples 3 An optimist is someone who sees the good side of things A pessimist sees the

bad side Are you an optimist or a pessimist Relate a personal experience that shows this

4 In your opinion are the benefits of space exploration really worth the enormous costs Discuss

Most MELAB compositions are one or two pages long (about 200-300 words) If your paper is extremely short (less than 150 words) your composition will be given a lower score Before you begin writing you might want to take 2 or 3 minutes to plan your comshyposition and to make a short outline to organize your thoughts Such outlines will not be graded they are only to help you You should use the last 5 minutes to read through your composition and to make changes or corrections

Your composition will be graded on how clearly you express yourself in English and on the range of English you are able to use and your control in doing so This means your composition should be well organized your arguments should be fully developed and you should show a range ofgrammatical structures and broad vocabulary Compositions that consist only of very short sentences and very simple vocabulary cannot be given the

middothighest scores If errors are not frequent and if they do not confuse your meaning they will not lower your score very much

Listening Now you will hear a short lecture You may take notes during the lecture Following the lecture you will be asked some questions about it

Therell be a two-week exhibit of the paintings of the little-known master Laura Bernhart at the Claire Osmond Galleries starting on the fifteenth of the month and running through the thirtieth Bernharts known for her innovative designs in abstract expressionism Though a true original she declared a spiritual heritage from Salvador Dali the famous Spanish painter Since Bernhart lived a rather solitary life and died while only in her twenties few people are aware of her works This showing at the Osmond Galleries will provide many with an introduction to her works

10 Where is the exhibit a the Art Museum b the Dali Galleries c the Osmond Galleries

94 CHAPTER 4 Standardized Testing

11 What is Bernhart known for a her copies of Dalis paintings b the originality of her designs c her exhibitions

12 What will going to the exhibit allow most people to do a to see Saivador Dalis paintings b to see Bernharts works for the first time c to learn about Spanish art

Grammar

1 What did the teacher just tell you

She reminded our notebooksI a us to bring b that we bring c our bringing d we should bring

2 Is Bill a good dancer

Not really __ he tries very hard a in spite of h despite c even though d while

3 your clothes are all wet1

Yes I didnt come __ the rain soon enough a away to b over to c down with d in from

Cloze In years to come zoos will not only be places where animals are exhibited to the public but repositories where rare species can be saved from extinction (7) captive breeding The most powerful force (8) the future of many animals-and of zoos-is the decline of the wild (9) even zoo directors would argue that (10) are better places for animals than the fields and forest of their native (11) yet zoos may be the last chance for some creatures that would otherwise pass qUietly into oblivion

7 a through c from b of d damage

8 a bringing c to b that d influencing

9 a But c Not b So d Then

10 a where c even b zoos d wilds

11 alands c residence b life d field

CHAPTER 4 Standardized Testing 95

Vocabulary

12 Mark has a flair for writing a need b purpose c talent d dislike

13 Bill Collins launched his restaurant last June a moved b started c sold d bought

14 John will not accept the censure a burden b blame c credit d decision

15 I cant think of the answer Can you give me a __ a hint b token c taste d gaze

16 Because fewer people are taking expensive vacations the tourist industry is in a a choke b grope c grumble d slump

17 I disagree with a few of his opinions but __ we agree a deliberately b conclusively c essentially d immensely

Reading The influenza virus is a single molecule built from many millions of single atoms You must have heard of the viruses which are sometimes called living molecules While bacteria can be considered as a type of plant secreting pOisonous substances into the body of the organism they attack viruses are living organisms themselves We may conshysider them as regular chemical molecules since they have a strictly aefined atomic strucshyture but on the other hand we must also consider them as being alive since they are able to multiply in unlimited quantities

18 According to the passage bacteria are a poisons

b larger than viruses c very small d plants

96 CHAPTER 4 Standardized Testing

19 The writer says that viruses are alive because they a have a complex atomic structure b move c multiply d need warmth and light

20 The atomic structure of viruses a is -tJIariable b is strictly defined c cannot be analyzed chemically d is more complex than that of bacteria

International English Language Testing System (fELTS)

I

listening

The Listening Module has four sections The first two sections are concerned with social needs There is a conversation between two speakers and then a monologue For examshyple a conversation about travel arrangements or decisions on a night out and a speech about student services on a university campus or arrangements for meals during a confershyence The final two sections are concerned with situations related more closely to educashytional or training contexts For example conversation between a tutor and a student about an assignment or between three students planning a research project and a lecture or talk ofgeneral academic interest All the topics are ofgeneral interest and it makes no difference what subjects candidates study Tests and tasks become more difficult as the sections progress A range of English accents and dialects are used in the recording which reflects the international usage of IELTS

Academic Reading [A 7S0-word article on-th-e- topic of Wind Power in the US with a short glossary at the end]

Questions 1-5

Complete the summary below

Choose your answers from the box below the summary and write them in boxes 1-5 on your answer sheet Note There are more words or phrases than you will need to fill the gaps You may use any word or phrase more than once

Example The failure during the late 1970s and early 19805 of an attempt to establish a widespread wind power industry in the United States resulted largely from the (1) bull in oil prices during this period The industry is now experiencing a steady (2) due to improveshyments in technology and an increased awareness of the potential in the power of wind The wind turbines that are now being made based in part on the (3) of wide- ranging research in Europe are easier to manufacture and maintain than their predecesshysors This has led wind-turbine makers to be able to standardise and thus minimize (4) There has been growing (S) of the importance of wind power as an energy source

CHAPTER 4 Standardized Testing 97

criticism stability skepticism success operating costs decisions design costs fall effects production costs growth decline failure recognition results

Questions 6-1 0 Look at the following list of issues (Questions 6-10) and implications (A-C) Match each issue with one implication Write the appropriate letters A-C in boxes 6-10 on your anshyswer sheet

Example The current price of one wind-generated kilowatt Answer

6 The recent installation of systems taking advantage of economies of scale

7 The potential of meeting one fifth of current U5 energy requirements by wind power

8 The level of acceptance of current wind turbine technology

9 A comparison of costs between conventional and wind power sources

10 The view of wind power in the European Union

Implications

A provides evidence against claims that electricity produced from wind power is relatively expensive

B supports claims that wind power js an important source of energy

C opposes the view that wind power technology requires further-development

General Training Reading Read the passage on Daybreak trips by coach and look at the statements below On your answer sheet write

TRUE if the statement is true FALSE jf the statement is false

NOlGIVEN if the information is not given in the leaflet

1 MiIlers Coaches owns Cambridges Cam bus fleet

2 Premier is an older company than Millers

3 Most of the Daybreak coaches are less than 5 years old

4 Daybreak fares are more expensive than most of their competitors

5 Soft drinks and refreshments are served on most longer journeys

6 Smoking is permitted at the rear of the coach on longer journeys

7 Tickets must be bought in advance from an authorised Daybreak agent

6 Tickets and seats can be reserved by phoning the Daybreak Hotline

9 Daybreak passengers must join their coach at Cambridge Drummer Street

10 Daybreak cannot guarantee return times

98 CHAPTER 4 Standardized Testing

FROM CAMBRIDGE AND SURROUNDING AREA

SPRING IS INTHEAIR

Welcome to our Spring Daybreak programme which continues the tradition of offering unbeatable value for money day trips and tours All the excursions in this brochure will be operated by Pr~mier Travel Services Limited or Millers Coaches both companies are part of the CHLGroup owners of Cambridges Cambus fleet

WERE PROUD OF OUR TRADITION

Premier was established in 1936 the Company now offers the highest standards of coaching in todays competitive operating environment Miller has an enviable reputation stretching back over the past 20 years offering coach services at realistic prices Weve traveled a long way since our early days of pre-war seaside trips Now our fleet of 50 modern coaches (few are more than five years old) operate throughout Britain and Europe but were pleased to still maintain the high standards of quality and service the trademark of our founders nearly sixty years ago

EXCLUSIVE FEATURES

Admission-inclusive fares All Daybreak fares (unless specifically otherwise stated) include admission charges to the attractions shows and exhibits we visit Many full-day scenic tours are accompanied by a fully trained English Tourist Board Blue Badge guide or local experienced driverguide Some Daybreaks include lunch or afternoon tea Compare our admission inclusive fares and see how much you save Cheapest is not the best and value for money is guaranteed If you compare our bargain Daybreak fares beware--most of our competishytors do not offer an all-inclusive fare

SEAT RESERVATIONS

We value the freedom of choice so you can choose your seat when you book The seat reservation is guaranteed a-nd remains yours at all times when aboard the coach

NO SMOKING COMFORT

With the comfort of our passengers in mind coaches on all our Daybreaks are no smokshying throughout In the interests of fellow passengers comfort we kindly ask that smokers observe our no smoking policy On scenic tours and longer journeys ample refreshment stops are provided when of course smoking is permitted

YOUR QUESTIONS ANSWERED

Do I need to book Booking in advance is strongly recommended as all Daybreak tours are subject to demand Subject to availability stand-by tickets can be purchased from the driver

What ti me does the coach leave The coach departs from Cambridge Drummer Street (Bay 12 adjacent to public toilets) at the time shown There are many additional joining points indicated by departure codes in the brochure If you are joining at one of our less popular joining points you will be adshyvised of your pick-up time (normally by telephone) not less than 48 hours before deparshyture In this way we can minimize the length of pick-up routes and reduce journey times for the majority of passengers

CHAPTER 4 Standardized Testing 99

What time do we get back An approximate return time is shown for each excursion The tim~s shown serve as a guide but road conditions can sometimes cause delay If your arrival will be later than advertised your driver will try to allow for a telephone call during the return journey

Where can I board the coach All the Daybreaks in the brochure leave from Cambridge Drummer Street (Bay 12 adjashycent to public toilets) at the time shown Many Daybreaks offer additional pick-ups for pre-booked passengers within Cambridge and the surrounding area This facility must be requested at the time of booking

Academic Writing Writing Task 1 You should spend about 20 minutes on this task

The graph below shows the different modes of transport used to travel to and from work in one European city in 1950 1970 and 1990

[graph shown here]

Write a report for a university lecturer describing the information shown below You should write at least 150 words

Writing Task 2 You should spend about 40 minutes on this task

Present a written argument or case to an educated reader with no specialist knowledge of the folowing topic

It is inevitable that as technology develops so traditional cultures must be lost Technolshyogy and tradition are incompatible-you cannot have both together

To what extent do you agree or disagree with this statement Give reasons for your answer You should write at least 250 words You should use your own ideas knowlshyedge and experience and support your arguments with examples and relevant evidence

General Training Writing Writing Task 1 You should spend about 20 minutes on this task You rent a house through an agency The heating system has stopped working You phoned the agency a week ago but it has still not been mended Write a letter to the agency Explain the situation and teil them what you want them to do about it

You should write at least 150 words You do NOT need to write your own address

Begin your letter as follows

Dear - ___-I

Writing Task 2 You should spend about 40 minutes on this task As part ofa class assignment you have to write about the following topic

100 CHAPTER 4 Standardized Testing

Some businesses now say that no one can smoke cigarettes in any of their offices Some governments have banned smoking in all public placesThis is a good idea but it takes away some of our freedom

Do you agree or disagree Give reasons for your answer You should write at least 250 words

Speaking In each ofthe three parts of the speaking module a specific function is fulfilled In Part 1 the candidates answer general questions about themselves their homes or families their jobs or studies their interests and a range ofsimilar familiar topic areas This part lasts between four and five minutes In Part 2 the candidate is given a verbal prompt on a card and is asked to talk on a particular topic The candidate has one minute to prepare before speaking at length for between one and two minutes The examiner then asks one or two wind-down questions In Part 3 the examiner and candidate engage in a discusshysion of more abstract issues and concepts which are thematically linked to the topic prompt in Part 2 The discussion lasts between four and five minutes

All interviews are recorded on audiocassette Here is a sample ofa Part 2 topic

Describe a teacher who has greatly influenced you in your education

You shou Id say

where you met them what subject they taught what was special about them

and explain why this person influenced you so much

You will have to talk about the topic for 1 to 2 minutes You have 1 minute to think about what you are going to say You can make some notes if you wish

Test of English for International Communication (TOEICreg)

listening

Part 1 Photographs Directions For each question you will see a picture in your test book and you will hear four short statements The statements will be spoken just one time They will not be printed in your test book so you must listen carefully to understand what the speaker says When you hear the four statements look at the picture in your test book and choose the statement that best describes what you see in the picture Then on your answer sheet find the number of the question and mark your answer

[photograph of a scientist looking through a microscope]

You will hear Look at the picture marked number 1 in your test book

(A) Shes speaking into a microphone (B) Shes put on her glasses (C) She has both eyes open (D) Shes using a microscope

CHAPTER 4 Standardized Testing 101

Part 2 Question-Response Directions In this part of the test you will hear a question or statement spoken in Enshyglish followed by three responses also spoken in English The question or staten1ent and the responses will be spoken just one time They will not be printed in your test book so you must listen carefully to understand what the speakers say You are to choose the best response to each question or statement

Question 1 You will hear Ms Morikawa has worked here for a long time hasnt she

(A) At three oclock (B) No Ive lost my watch (C) More than ten years

Question 2 You will hear Which of these papers has a wider circulation

(A) The morning edition (B) Get more exercise (C) By messenger

Part 3 Short Conversations Directions In this part of the test you will hear short conversations between two people The conversations will not be printed in your test book You will hear the conversations only once so you must listen carefully to understand what the speakers say In your test book you will read a question about each conversation The question will be followed by four answers You are to choose the best answer to each question and mark it on your answer sheet

Question 1 (Man) We should think about finding another restaurant for lunch (Woman) Why The food and service here are great

(Man) Yes but the prices are going up every week

You will read Why is this man unhappy with the restaurant

(A) It is too noisy (B) It is too expensive (C) It is too crowded (D) It is too difficult to find

Question 2 (Woman A) How was Dr Borgs recent trip to Singapore (Woman B) She enjoyed the tour of the port very much (Woman A) They say its one of the most active in Asia

You will read 2 What did Dr Borg find interesting

(A) The tourist center (B) The airport (C) The musical performance (D) The harbor

Part 4 Short Talks Directions In this part of the test you vill hear several short talks Each will be spoken just one time They will not be printed in your test book so you must listen carefully to understand and remember what is said In your test book you will read two or more questions about each short talk The questions will be followed by four answers You are to choose the best answer to each question and mark it on your answer sheet

102 CHAPTER 4 Standardized Testing

You will hear Questions 1 and 2 refer to the following announcement

Good afternoon and welcome aboard Nordair Flight 857 from Copenhagen to Bangkok with intermediate stops in Dubai and Calcutta We are preparing for departure in a few minutes At this time your seat back should be returned to its full upright position and your seat belt s~ould be fastened OUf anticipated total flying time to Dubai is six hours and twenty-five minutes I hope you enjoy the flight You will hecJr Now read question 1 in your test book and answer it You will read 1 What is the final destination of the flight

(A) Bangkok (B) Copenhagen (C) Dubai (O) Calcutta

You will hear Now read question 2 in your test book and answer it You will read 2 What will happen in a few minutes

(A) The flight will land in Dubai I

(B) The passengers will board the plane (C) The plane will take off (0) The gate number will be announced

Reading In this section of the test you will have the chance to show how well you understand written English There are three parts to this section with special directions for each part

Part 4 Incomplete Sentences Directions This part of the test has incomplete sentences Four words or phrases marked (A) (8) (e) (D) are given beneath each sentence You are to choose the one word or phrase that best completes the sentence Then on your answer sheet find the number of the question and mark your answer

1 Mr Yangs trip will __ him away from the office for ten days (A) withdraw (B) continue (C) retain (0) keep

2 The company that Marie DuBois started now sells __ products throughout the world (A) its (B) it (C) theirs (D) them

3 If your shipment is not delivered __ Tuesday you can request a full refund for the merchandise (A) at (B) by (C) within (D) while

CHAPTER 4 Standardized Testing 103

Part 6 Error Recognition Directions In this part ofthe test each sentence has four words or phrases underlined The four underlined parts of the sentence are marked (A) (B) (C) (D) You are to identify the one underlined word or phrase that should be corrected or rewritten Then on your answer sheet find the number of the question and mark your answer

1 The pamphlet contains some importance information about the current exhibit ABC D

2 No matter how Jong it taking to finish the annual report it must be done properly ABC D

3 The popularity of jogging appears to have decreased since the past couple of years ABC D

Part 7 Reading Comprehension Directions The questions in this part of the test are based on a selection of reading mateshyrials such as notices letters) forms newspaper and magazine articles) and advertisements You are to choose the one best answer (A) (B) (C) or (OJ to each quesshytion Then on your ariswefsheelfindthe number of the qUestion andmcirkyour answer Answer all questions following each reading selection on thebasis of what is stated or implied in that selection

The Museum ofTechnology is a hands-on museum designed for people to experience science at w()rk~ Visitors are encouraged to use test and handle the objects o~ display Special demonstrations are scheduled for the first and second Wednesdays of each month at 1330 Open Tuesday-Friday 1200-1630 Saturday 1000-1730 and Sunday 11 00-1630

1 When during the month can visitors see special demonstrations (A) Every weekend (B) The first two Wednesdays (C) One afternoon a week (D) Every other Wednesday

Questions 2 and 3 refer to the followi ng notice

NOTICE If you are unable to work because of an extended illness or injury that is not workshyrelated you may be entitled to receive weekly benefits from your employer or the firms insurance company To claim benefits you must file a claim form within thirty days of the first day of your disability Before filing the claim you must ask your doctor to fill in the Doctors Statement on the claim form stating the period of disability

3 To whom is this notice addressed (A) Employers (8) Doctors (C) Employees (D) When paying the bill

4 When must the claim form be filed (A) On the first of the month (8) On the thirtieth of the month (C) On the first day ofdisabifity (D) Within 30 days of the start of disability

Page 28: Standardized Testing Chapter 4 Brown

CHAPTER 4 Standarczed Testing 93

Michigan English Language Assessment Battery (MELAB)

Composition The time limit for the composition is 30 minutes You must write on only one of the top~

ics below If you write about something else your composition paper will not be graded and you cannot be given a final score If you do not understand the topics ask the exam~ iner to explain or to translate them You may be asked to give your opinion ofsomething and explain why you believe this to describe something from your experience or to exshyplain a problem and offer possible solutions You should write at least one page Some sample topics are

1 What do you think is your countrys greatest problem Explain in detail and tell what you think can be done about it

2 What are the characteristics of a good teacher Explain and give examples 3 An optimist is someone who sees the good side of things A pessimist sees the

bad side Are you an optimist or a pessimist Relate a personal experience that shows this

4 In your opinion are the benefits of space exploration really worth the enormous costs Discuss

Most MELAB compositions are one or two pages long (about 200-300 words) If your paper is extremely short (less than 150 words) your composition will be given a lower score Before you begin writing you might want to take 2 or 3 minutes to plan your comshyposition and to make a short outline to organize your thoughts Such outlines will not be graded they are only to help you You should use the last 5 minutes to read through your composition and to make changes or corrections

Your composition will be graded on how clearly you express yourself in English and on the range of English you are able to use and your control in doing so This means your composition should be well organized your arguments should be fully developed and you should show a range ofgrammatical structures and broad vocabulary Compositions that consist only of very short sentences and very simple vocabulary cannot be given the

middothighest scores If errors are not frequent and if they do not confuse your meaning they will not lower your score very much

Listening Now you will hear a short lecture You may take notes during the lecture Following the lecture you will be asked some questions about it

Therell be a two-week exhibit of the paintings of the little-known master Laura Bernhart at the Claire Osmond Galleries starting on the fifteenth of the month and running through the thirtieth Bernharts known for her innovative designs in abstract expressionism Though a true original she declared a spiritual heritage from Salvador Dali the famous Spanish painter Since Bernhart lived a rather solitary life and died while only in her twenties few people are aware of her works This showing at the Osmond Galleries will provide many with an introduction to her works

10 Where is the exhibit a the Art Museum b the Dali Galleries c the Osmond Galleries

94 CHAPTER 4 Standardized Testing

11 What is Bernhart known for a her copies of Dalis paintings b the originality of her designs c her exhibitions

12 What will going to the exhibit allow most people to do a to see Saivador Dalis paintings b to see Bernharts works for the first time c to learn about Spanish art

Grammar

1 What did the teacher just tell you

She reminded our notebooksI a us to bring b that we bring c our bringing d we should bring

2 Is Bill a good dancer

Not really __ he tries very hard a in spite of h despite c even though d while

3 your clothes are all wet1

Yes I didnt come __ the rain soon enough a away to b over to c down with d in from

Cloze In years to come zoos will not only be places where animals are exhibited to the public but repositories where rare species can be saved from extinction (7) captive breeding The most powerful force (8) the future of many animals-and of zoos-is the decline of the wild (9) even zoo directors would argue that (10) are better places for animals than the fields and forest of their native (11) yet zoos may be the last chance for some creatures that would otherwise pass qUietly into oblivion

7 a through c from b of d damage

8 a bringing c to b that d influencing

9 a But c Not b So d Then

10 a where c even b zoos d wilds

11 alands c residence b life d field

CHAPTER 4 Standardized Testing 95

Vocabulary

12 Mark has a flair for writing a need b purpose c talent d dislike

13 Bill Collins launched his restaurant last June a moved b started c sold d bought

14 John will not accept the censure a burden b blame c credit d decision

15 I cant think of the answer Can you give me a __ a hint b token c taste d gaze

16 Because fewer people are taking expensive vacations the tourist industry is in a a choke b grope c grumble d slump

17 I disagree with a few of his opinions but __ we agree a deliberately b conclusively c essentially d immensely

Reading The influenza virus is a single molecule built from many millions of single atoms You must have heard of the viruses which are sometimes called living molecules While bacteria can be considered as a type of plant secreting pOisonous substances into the body of the organism they attack viruses are living organisms themselves We may conshysider them as regular chemical molecules since they have a strictly aefined atomic strucshyture but on the other hand we must also consider them as being alive since they are able to multiply in unlimited quantities

18 According to the passage bacteria are a poisons

b larger than viruses c very small d plants

96 CHAPTER 4 Standardized Testing

19 The writer says that viruses are alive because they a have a complex atomic structure b move c multiply d need warmth and light

20 The atomic structure of viruses a is -tJIariable b is strictly defined c cannot be analyzed chemically d is more complex than that of bacteria

International English Language Testing System (fELTS)

I

listening

The Listening Module has four sections The first two sections are concerned with social needs There is a conversation between two speakers and then a monologue For examshyple a conversation about travel arrangements or decisions on a night out and a speech about student services on a university campus or arrangements for meals during a confershyence The final two sections are concerned with situations related more closely to educashytional or training contexts For example conversation between a tutor and a student about an assignment or between three students planning a research project and a lecture or talk ofgeneral academic interest All the topics are ofgeneral interest and it makes no difference what subjects candidates study Tests and tasks become more difficult as the sections progress A range of English accents and dialects are used in the recording which reflects the international usage of IELTS

Academic Reading [A 7S0-word article on-th-e- topic of Wind Power in the US with a short glossary at the end]

Questions 1-5

Complete the summary below

Choose your answers from the box below the summary and write them in boxes 1-5 on your answer sheet Note There are more words or phrases than you will need to fill the gaps You may use any word or phrase more than once

Example The failure during the late 1970s and early 19805 of an attempt to establish a widespread wind power industry in the United States resulted largely from the (1) bull in oil prices during this period The industry is now experiencing a steady (2) due to improveshyments in technology and an increased awareness of the potential in the power of wind The wind turbines that are now being made based in part on the (3) of wide- ranging research in Europe are easier to manufacture and maintain than their predecesshysors This has led wind-turbine makers to be able to standardise and thus minimize (4) There has been growing (S) of the importance of wind power as an energy source

CHAPTER 4 Standardized Testing 97

criticism stability skepticism success operating costs decisions design costs fall effects production costs growth decline failure recognition results

Questions 6-1 0 Look at the following list of issues (Questions 6-10) and implications (A-C) Match each issue with one implication Write the appropriate letters A-C in boxes 6-10 on your anshyswer sheet

Example The current price of one wind-generated kilowatt Answer

6 The recent installation of systems taking advantage of economies of scale

7 The potential of meeting one fifth of current U5 energy requirements by wind power

8 The level of acceptance of current wind turbine technology

9 A comparison of costs between conventional and wind power sources

10 The view of wind power in the European Union

Implications

A provides evidence against claims that electricity produced from wind power is relatively expensive

B supports claims that wind power js an important source of energy

C opposes the view that wind power technology requires further-development

General Training Reading Read the passage on Daybreak trips by coach and look at the statements below On your answer sheet write

TRUE if the statement is true FALSE jf the statement is false

NOlGIVEN if the information is not given in the leaflet

1 MiIlers Coaches owns Cambridges Cam bus fleet

2 Premier is an older company than Millers

3 Most of the Daybreak coaches are less than 5 years old

4 Daybreak fares are more expensive than most of their competitors

5 Soft drinks and refreshments are served on most longer journeys

6 Smoking is permitted at the rear of the coach on longer journeys

7 Tickets must be bought in advance from an authorised Daybreak agent

6 Tickets and seats can be reserved by phoning the Daybreak Hotline

9 Daybreak passengers must join their coach at Cambridge Drummer Street

10 Daybreak cannot guarantee return times

98 CHAPTER 4 Standardized Testing

FROM CAMBRIDGE AND SURROUNDING AREA

SPRING IS INTHEAIR

Welcome to our Spring Daybreak programme which continues the tradition of offering unbeatable value for money day trips and tours All the excursions in this brochure will be operated by Pr~mier Travel Services Limited or Millers Coaches both companies are part of the CHLGroup owners of Cambridges Cambus fleet

WERE PROUD OF OUR TRADITION

Premier was established in 1936 the Company now offers the highest standards of coaching in todays competitive operating environment Miller has an enviable reputation stretching back over the past 20 years offering coach services at realistic prices Weve traveled a long way since our early days of pre-war seaside trips Now our fleet of 50 modern coaches (few are more than five years old) operate throughout Britain and Europe but were pleased to still maintain the high standards of quality and service the trademark of our founders nearly sixty years ago

EXCLUSIVE FEATURES

Admission-inclusive fares All Daybreak fares (unless specifically otherwise stated) include admission charges to the attractions shows and exhibits we visit Many full-day scenic tours are accompanied by a fully trained English Tourist Board Blue Badge guide or local experienced driverguide Some Daybreaks include lunch or afternoon tea Compare our admission inclusive fares and see how much you save Cheapest is not the best and value for money is guaranteed If you compare our bargain Daybreak fares beware--most of our competishytors do not offer an all-inclusive fare

SEAT RESERVATIONS

We value the freedom of choice so you can choose your seat when you book The seat reservation is guaranteed a-nd remains yours at all times when aboard the coach

NO SMOKING COMFORT

With the comfort of our passengers in mind coaches on all our Daybreaks are no smokshying throughout In the interests of fellow passengers comfort we kindly ask that smokers observe our no smoking policy On scenic tours and longer journeys ample refreshment stops are provided when of course smoking is permitted

YOUR QUESTIONS ANSWERED

Do I need to book Booking in advance is strongly recommended as all Daybreak tours are subject to demand Subject to availability stand-by tickets can be purchased from the driver

What ti me does the coach leave The coach departs from Cambridge Drummer Street (Bay 12 adjacent to public toilets) at the time shown There are many additional joining points indicated by departure codes in the brochure If you are joining at one of our less popular joining points you will be adshyvised of your pick-up time (normally by telephone) not less than 48 hours before deparshyture In this way we can minimize the length of pick-up routes and reduce journey times for the majority of passengers

CHAPTER 4 Standardized Testing 99

What time do we get back An approximate return time is shown for each excursion The tim~s shown serve as a guide but road conditions can sometimes cause delay If your arrival will be later than advertised your driver will try to allow for a telephone call during the return journey

Where can I board the coach All the Daybreaks in the brochure leave from Cambridge Drummer Street (Bay 12 adjashycent to public toilets) at the time shown Many Daybreaks offer additional pick-ups for pre-booked passengers within Cambridge and the surrounding area This facility must be requested at the time of booking

Academic Writing Writing Task 1 You should spend about 20 minutes on this task

The graph below shows the different modes of transport used to travel to and from work in one European city in 1950 1970 and 1990

[graph shown here]

Write a report for a university lecturer describing the information shown below You should write at least 150 words

Writing Task 2 You should spend about 40 minutes on this task

Present a written argument or case to an educated reader with no specialist knowledge of the folowing topic

It is inevitable that as technology develops so traditional cultures must be lost Technolshyogy and tradition are incompatible-you cannot have both together

To what extent do you agree or disagree with this statement Give reasons for your answer You should write at least 250 words You should use your own ideas knowlshyedge and experience and support your arguments with examples and relevant evidence

General Training Writing Writing Task 1 You should spend about 20 minutes on this task You rent a house through an agency The heating system has stopped working You phoned the agency a week ago but it has still not been mended Write a letter to the agency Explain the situation and teil them what you want them to do about it

You should write at least 150 words You do NOT need to write your own address

Begin your letter as follows

Dear - ___-I

Writing Task 2 You should spend about 40 minutes on this task As part ofa class assignment you have to write about the following topic

100 CHAPTER 4 Standardized Testing

Some businesses now say that no one can smoke cigarettes in any of their offices Some governments have banned smoking in all public placesThis is a good idea but it takes away some of our freedom

Do you agree or disagree Give reasons for your answer You should write at least 250 words

Speaking In each ofthe three parts of the speaking module a specific function is fulfilled In Part 1 the candidates answer general questions about themselves their homes or families their jobs or studies their interests and a range ofsimilar familiar topic areas This part lasts between four and five minutes In Part 2 the candidate is given a verbal prompt on a card and is asked to talk on a particular topic The candidate has one minute to prepare before speaking at length for between one and two minutes The examiner then asks one or two wind-down questions In Part 3 the examiner and candidate engage in a discusshysion of more abstract issues and concepts which are thematically linked to the topic prompt in Part 2 The discussion lasts between four and five minutes

All interviews are recorded on audiocassette Here is a sample ofa Part 2 topic

Describe a teacher who has greatly influenced you in your education

You shou Id say

where you met them what subject they taught what was special about them

and explain why this person influenced you so much

You will have to talk about the topic for 1 to 2 minutes You have 1 minute to think about what you are going to say You can make some notes if you wish

Test of English for International Communication (TOEICreg)

listening

Part 1 Photographs Directions For each question you will see a picture in your test book and you will hear four short statements The statements will be spoken just one time They will not be printed in your test book so you must listen carefully to understand what the speaker says When you hear the four statements look at the picture in your test book and choose the statement that best describes what you see in the picture Then on your answer sheet find the number of the question and mark your answer

[photograph of a scientist looking through a microscope]

You will hear Look at the picture marked number 1 in your test book

(A) Shes speaking into a microphone (B) Shes put on her glasses (C) She has both eyes open (D) Shes using a microscope

CHAPTER 4 Standardized Testing 101

Part 2 Question-Response Directions In this part of the test you will hear a question or statement spoken in Enshyglish followed by three responses also spoken in English The question or staten1ent and the responses will be spoken just one time They will not be printed in your test book so you must listen carefully to understand what the speakers say You are to choose the best response to each question or statement

Question 1 You will hear Ms Morikawa has worked here for a long time hasnt she

(A) At three oclock (B) No Ive lost my watch (C) More than ten years

Question 2 You will hear Which of these papers has a wider circulation

(A) The morning edition (B) Get more exercise (C) By messenger

Part 3 Short Conversations Directions In this part of the test you will hear short conversations between two people The conversations will not be printed in your test book You will hear the conversations only once so you must listen carefully to understand what the speakers say In your test book you will read a question about each conversation The question will be followed by four answers You are to choose the best answer to each question and mark it on your answer sheet

Question 1 (Man) We should think about finding another restaurant for lunch (Woman) Why The food and service here are great

(Man) Yes but the prices are going up every week

You will read Why is this man unhappy with the restaurant

(A) It is too noisy (B) It is too expensive (C) It is too crowded (D) It is too difficult to find

Question 2 (Woman A) How was Dr Borgs recent trip to Singapore (Woman B) She enjoyed the tour of the port very much (Woman A) They say its one of the most active in Asia

You will read 2 What did Dr Borg find interesting

(A) The tourist center (B) The airport (C) The musical performance (D) The harbor

Part 4 Short Talks Directions In this part of the test you vill hear several short talks Each will be spoken just one time They will not be printed in your test book so you must listen carefully to understand and remember what is said In your test book you will read two or more questions about each short talk The questions will be followed by four answers You are to choose the best answer to each question and mark it on your answer sheet

102 CHAPTER 4 Standardized Testing

You will hear Questions 1 and 2 refer to the following announcement

Good afternoon and welcome aboard Nordair Flight 857 from Copenhagen to Bangkok with intermediate stops in Dubai and Calcutta We are preparing for departure in a few minutes At this time your seat back should be returned to its full upright position and your seat belt s~ould be fastened OUf anticipated total flying time to Dubai is six hours and twenty-five minutes I hope you enjoy the flight You will hecJr Now read question 1 in your test book and answer it You will read 1 What is the final destination of the flight

(A) Bangkok (B) Copenhagen (C) Dubai (O) Calcutta

You will hear Now read question 2 in your test book and answer it You will read 2 What will happen in a few minutes

(A) The flight will land in Dubai I

(B) The passengers will board the plane (C) The plane will take off (0) The gate number will be announced

Reading In this section of the test you will have the chance to show how well you understand written English There are three parts to this section with special directions for each part

Part 4 Incomplete Sentences Directions This part of the test has incomplete sentences Four words or phrases marked (A) (8) (e) (D) are given beneath each sentence You are to choose the one word or phrase that best completes the sentence Then on your answer sheet find the number of the question and mark your answer

1 Mr Yangs trip will __ him away from the office for ten days (A) withdraw (B) continue (C) retain (0) keep

2 The company that Marie DuBois started now sells __ products throughout the world (A) its (B) it (C) theirs (D) them

3 If your shipment is not delivered __ Tuesday you can request a full refund for the merchandise (A) at (B) by (C) within (D) while

CHAPTER 4 Standardized Testing 103

Part 6 Error Recognition Directions In this part ofthe test each sentence has four words or phrases underlined The four underlined parts of the sentence are marked (A) (B) (C) (D) You are to identify the one underlined word or phrase that should be corrected or rewritten Then on your answer sheet find the number of the question and mark your answer

1 The pamphlet contains some importance information about the current exhibit ABC D

2 No matter how Jong it taking to finish the annual report it must be done properly ABC D

3 The popularity of jogging appears to have decreased since the past couple of years ABC D

Part 7 Reading Comprehension Directions The questions in this part of the test are based on a selection of reading mateshyrials such as notices letters) forms newspaper and magazine articles) and advertisements You are to choose the one best answer (A) (B) (C) or (OJ to each quesshytion Then on your ariswefsheelfindthe number of the qUestion andmcirkyour answer Answer all questions following each reading selection on thebasis of what is stated or implied in that selection

The Museum ofTechnology is a hands-on museum designed for people to experience science at w()rk~ Visitors are encouraged to use test and handle the objects o~ display Special demonstrations are scheduled for the first and second Wednesdays of each month at 1330 Open Tuesday-Friday 1200-1630 Saturday 1000-1730 and Sunday 11 00-1630

1 When during the month can visitors see special demonstrations (A) Every weekend (B) The first two Wednesdays (C) One afternoon a week (D) Every other Wednesday

Questions 2 and 3 refer to the followi ng notice

NOTICE If you are unable to work because of an extended illness or injury that is not workshyrelated you may be entitled to receive weekly benefits from your employer or the firms insurance company To claim benefits you must file a claim form within thirty days of the first day of your disability Before filing the claim you must ask your doctor to fill in the Doctors Statement on the claim form stating the period of disability

3 To whom is this notice addressed (A) Employers (8) Doctors (C) Employees (D) When paying the bill

4 When must the claim form be filed (A) On the first of the month (8) On the thirtieth of the month (C) On the first day ofdisabifity (D) Within 30 days of the start of disability

Page 29: Standardized Testing Chapter 4 Brown

94 CHAPTER 4 Standardized Testing

11 What is Bernhart known for a her copies of Dalis paintings b the originality of her designs c her exhibitions

12 What will going to the exhibit allow most people to do a to see Saivador Dalis paintings b to see Bernharts works for the first time c to learn about Spanish art

Grammar

1 What did the teacher just tell you

She reminded our notebooksI a us to bring b that we bring c our bringing d we should bring

2 Is Bill a good dancer

Not really __ he tries very hard a in spite of h despite c even though d while

3 your clothes are all wet1

Yes I didnt come __ the rain soon enough a away to b over to c down with d in from

Cloze In years to come zoos will not only be places where animals are exhibited to the public but repositories where rare species can be saved from extinction (7) captive breeding The most powerful force (8) the future of many animals-and of zoos-is the decline of the wild (9) even zoo directors would argue that (10) are better places for animals than the fields and forest of their native (11) yet zoos may be the last chance for some creatures that would otherwise pass qUietly into oblivion

7 a through c from b of d damage

8 a bringing c to b that d influencing

9 a But c Not b So d Then

10 a where c even b zoos d wilds

11 alands c residence b life d field

CHAPTER 4 Standardized Testing 95

Vocabulary

12 Mark has a flair for writing a need b purpose c talent d dislike

13 Bill Collins launched his restaurant last June a moved b started c sold d bought

14 John will not accept the censure a burden b blame c credit d decision

15 I cant think of the answer Can you give me a __ a hint b token c taste d gaze

16 Because fewer people are taking expensive vacations the tourist industry is in a a choke b grope c grumble d slump

17 I disagree with a few of his opinions but __ we agree a deliberately b conclusively c essentially d immensely

Reading The influenza virus is a single molecule built from many millions of single atoms You must have heard of the viruses which are sometimes called living molecules While bacteria can be considered as a type of plant secreting pOisonous substances into the body of the organism they attack viruses are living organisms themselves We may conshysider them as regular chemical molecules since they have a strictly aefined atomic strucshyture but on the other hand we must also consider them as being alive since they are able to multiply in unlimited quantities

18 According to the passage bacteria are a poisons

b larger than viruses c very small d plants

96 CHAPTER 4 Standardized Testing

19 The writer says that viruses are alive because they a have a complex atomic structure b move c multiply d need warmth and light

20 The atomic structure of viruses a is -tJIariable b is strictly defined c cannot be analyzed chemically d is more complex than that of bacteria

International English Language Testing System (fELTS)

I

listening

The Listening Module has four sections The first two sections are concerned with social needs There is a conversation between two speakers and then a monologue For examshyple a conversation about travel arrangements or decisions on a night out and a speech about student services on a university campus or arrangements for meals during a confershyence The final two sections are concerned with situations related more closely to educashytional or training contexts For example conversation between a tutor and a student about an assignment or between three students planning a research project and a lecture or talk ofgeneral academic interest All the topics are ofgeneral interest and it makes no difference what subjects candidates study Tests and tasks become more difficult as the sections progress A range of English accents and dialects are used in the recording which reflects the international usage of IELTS

Academic Reading [A 7S0-word article on-th-e- topic of Wind Power in the US with a short glossary at the end]

Questions 1-5

Complete the summary below

Choose your answers from the box below the summary and write them in boxes 1-5 on your answer sheet Note There are more words or phrases than you will need to fill the gaps You may use any word or phrase more than once

Example The failure during the late 1970s and early 19805 of an attempt to establish a widespread wind power industry in the United States resulted largely from the (1) bull in oil prices during this period The industry is now experiencing a steady (2) due to improveshyments in technology and an increased awareness of the potential in the power of wind The wind turbines that are now being made based in part on the (3) of wide- ranging research in Europe are easier to manufacture and maintain than their predecesshysors This has led wind-turbine makers to be able to standardise and thus minimize (4) There has been growing (S) of the importance of wind power as an energy source

CHAPTER 4 Standardized Testing 97

criticism stability skepticism success operating costs decisions design costs fall effects production costs growth decline failure recognition results

Questions 6-1 0 Look at the following list of issues (Questions 6-10) and implications (A-C) Match each issue with one implication Write the appropriate letters A-C in boxes 6-10 on your anshyswer sheet

Example The current price of one wind-generated kilowatt Answer

6 The recent installation of systems taking advantage of economies of scale

7 The potential of meeting one fifth of current U5 energy requirements by wind power

8 The level of acceptance of current wind turbine technology

9 A comparison of costs between conventional and wind power sources

10 The view of wind power in the European Union

Implications

A provides evidence against claims that electricity produced from wind power is relatively expensive

B supports claims that wind power js an important source of energy

C opposes the view that wind power technology requires further-development

General Training Reading Read the passage on Daybreak trips by coach and look at the statements below On your answer sheet write

TRUE if the statement is true FALSE jf the statement is false

NOlGIVEN if the information is not given in the leaflet

1 MiIlers Coaches owns Cambridges Cam bus fleet

2 Premier is an older company than Millers

3 Most of the Daybreak coaches are less than 5 years old

4 Daybreak fares are more expensive than most of their competitors

5 Soft drinks and refreshments are served on most longer journeys

6 Smoking is permitted at the rear of the coach on longer journeys

7 Tickets must be bought in advance from an authorised Daybreak agent

6 Tickets and seats can be reserved by phoning the Daybreak Hotline

9 Daybreak passengers must join their coach at Cambridge Drummer Street

10 Daybreak cannot guarantee return times

98 CHAPTER 4 Standardized Testing

FROM CAMBRIDGE AND SURROUNDING AREA

SPRING IS INTHEAIR

Welcome to our Spring Daybreak programme which continues the tradition of offering unbeatable value for money day trips and tours All the excursions in this brochure will be operated by Pr~mier Travel Services Limited or Millers Coaches both companies are part of the CHLGroup owners of Cambridges Cambus fleet

WERE PROUD OF OUR TRADITION

Premier was established in 1936 the Company now offers the highest standards of coaching in todays competitive operating environment Miller has an enviable reputation stretching back over the past 20 years offering coach services at realistic prices Weve traveled a long way since our early days of pre-war seaside trips Now our fleet of 50 modern coaches (few are more than five years old) operate throughout Britain and Europe but were pleased to still maintain the high standards of quality and service the trademark of our founders nearly sixty years ago

EXCLUSIVE FEATURES

Admission-inclusive fares All Daybreak fares (unless specifically otherwise stated) include admission charges to the attractions shows and exhibits we visit Many full-day scenic tours are accompanied by a fully trained English Tourist Board Blue Badge guide or local experienced driverguide Some Daybreaks include lunch or afternoon tea Compare our admission inclusive fares and see how much you save Cheapest is not the best and value for money is guaranteed If you compare our bargain Daybreak fares beware--most of our competishytors do not offer an all-inclusive fare

SEAT RESERVATIONS

We value the freedom of choice so you can choose your seat when you book The seat reservation is guaranteed a-nd remains yours at all times when aboard the coach

NO SMOKING COMFORT

With the comfort of our passengers in mind coaches on all our Daybreaks are no smokshying throughout In the interests of fellow passengers comfort we kindly ask that smokers observe our no smoking policy On scenic tours and longer journeys ample refreshment stops are provided when of course smoking is permitted

YOUR QUESTIONS ANSWERED

Do I need to book Booking in advance is strongly recommended as all Daybreak tours are subject to demand Subject to availability stand-by tickets can be purchased from the driver

What ti me does the coach leave The coach departs from Cambridge Drummer Street (Bay 12 adjacent to public toilets) at the time shown There are many additional joining points indicated by departure codes in the brochure If you are joining at one of our less popular joining points you will be adshyvised of your pick-up time (normally by telephone) not less than 48 hours before deparshyture In this way we can minimize the length of pick-up routes and reduce journey times for the majority of passengers

CHAPTER 4 Standardized Testing 99

What time do we get back An approximate return time is shown for each excursion The tim~s shown serve as a guide but road conditions can sometimes cause delay If your arrival will be later than advertised your driver will try to allow for a telephone call during the return journey

Where can I board the coach All the Daybreaks in the brochure leave from Cambridge Drummer Street (Bay 12 adjashycent to public toilets) at the time shown Many Daybreaks offer additional pick-ups for pre-booked passengers within Cambridge and the surrounding area This facility must be requested at the time of booking

Academic Writing Writing Task 1 You should spend about 20 minutes on this task

The graph below shows the different modes of transport used to travel to and from work in one European city in 1950 1970 and 1990

[graph shown here]

Write a report for a university lecturer describing the information shown below You should write at least 150 words

Writing Task 2 You should spend about 40 minutes on this task

Present a written argument or case to an educated reader with no specialist knowledge of the folowing topic

It is inevitable that as technology develops so traditional cultures must be lost Technolshyogy and tradition are incompatible-you cannot have both together

To what extent do you agree or disagree with this statement Give reasons for your answer You should write at least 250 words You should use your own ideas knowlshyedge and experience and support your arguments with examples and relevant evidence

General Training Writing Writing Task 1 You should spend about 20 minutes on this task You rent a house through an agency The heating system has stopped working You phoned the agency a week ago but it has still not been mended Write a letter to the agency Explain the situation and teil them what you want them to do about it

You should write at least 150 words You do NOT need to write your own address

Begin your letter as follows

Dear - ___-I

Writing Task 2 You should spend about 40 minutes on this task As part ofa class assignment you have to write about the following topic

100 CHAPTER 4 Standardized Testing

Some businesses now say that no one can smoke cigarettes in any of their offices Some governments have banned smoking in all public placesThis is a good idea but it takes away some of our freedom

Do you agree or disagree Give reasons for your answer You should write at least 250 words

Speaking In each ofthe three parts of the speaking module a specific function is fulfilled In Part 1 the candidates answer general questions about themselves their homes or families their jobs or studies their interests and a range ofsimilar familiar topic areas This part lasts between four and five minutes In Part 2 the candidate is given a verbal prompt on a card and is asked to talk on a particular topic The candidate has one minute to prepare before speaking at length for between one and two minutes The examiner then asks one or two wind-down questions In Part 3 the examiner and candidate engage in a discusshysion of more abstract issues and concepts which are thematically linked to the topic prompt in Part 2 The discussion lasts between four and five minutes

All interviews are recorded on audiocassette Here is a sample ofa Part 2 topic

Describe a teacher who has greatly influenced you in your education

You shou Id say

where you met them what subject they taught what was special about them

and explain why this person influenced you so much

You will have to talk about the topic for 1 to 2 minutes You have 1 minute to think about what you are going to say You can make some notes if you wish

Test of English for International Communication (TOEICreg)

listening

Part 1 Photographs Directions For each question you will see a picture in your test book and you will hear four short statements The statements will be spoken just one time They will not be printed in your test book so you must listen carefully to understand what the speaker says When you hear the four statements look at the picture in your test book and choose the statement that best describes what you see in the picture Then on your answer sheet find the number of the question and mark your answer

[photograph of a scientist looking through a microscope]

You will hear Look at the picture marked number 1 in your test book

(A) Shes speaking into a microphone (B) Shes put on her glasses (C) She has both eyes open (D) Shes using a microscope

CHAPTER 4 Standardized Testing 101

Part 2 Question-Response Directions In this part of the test you will hear a question or statement spoken in Enshyglish followed by three responses also spoken in English The question or staten1ent and the responses will be spoken just one time They will not be printed in your test book so you must listen carefully to understand what the speakers say You are to choose the best response to each question or statement

Question 1 You will hear Ms Morikawa has worked here for a long time hasnt she

(A) At three oclock (B) No Ive lost my watch (C) More than ten years

Question 2 You will hear Which of these papers has a wider circulation

(A) The morning edition (B) Get more exercise (C) By messenger

Part 3 Short Conversations Directions In this part of the test you will hear short conversations between two people The conversations will not be printed in your test book You will hear the conversations only once so you must listen carefully to understand what the speakers say In your test book you will read a question about each conversation The question will be followed by four answers You are to choose the best answer to each question and mark it on your answer sheet

Question 1 (Man) We should think about finding another restaurant for lunch (Woman) Why The food and service here are great

(Man) Yes but the prices are going up every week

You will read Why is this man unhappy with the restaurant

(A) It is too noisy (B) It is too expensive (C) It is too crowded (D) It is too difficult to find

Question 2 (Woman A) How was Dr Borgs recent trip to Singapore (Woman B) She enjoyed the tour of the port very much (Woman A) They say its one of the most active in Asia

You will read 2 What did Dr Borg find interesting

(A) The tourist center (B) The airport (C) The musical performance (D) The harbor

Part 4 Short Talks Directions In this part of the test you vill hear several short talks Each will be spoken just one time They will not be printed in your test book so you must listen carefully to understand and remember what is said In your test book you will read two or more questions about each short talk The questions will be followed by four answers You are to choose the best answer to each question and mark it on your answer sheet

102 CHAPTER 4 Standardized Testing

You will hear Questions 1 and 2 refer to the following announcement

Good afternoon and welcome aboard Nordair Flight 857 from Copenhagen to Bangkok with intermediate stops in Dubai and Calcutta We are preparing for departure in a few minutes At this time your seat back should be returned to its full upright position and your seat belt s~ould be fastened OUf anticipated total flying time to Dubai is six hours and twenty-five minutes I hope you enjoy the flight You will hecJr Now read question 1 in your test book and answer it You will read 1 What is the final destination of the flight

(A) Bangkok (B) Copenhagen (C) Dubai (O) Calcutta

You will hear Now read question 2 in your test book and answer it You will read 2 What will happen in a few minutes

(A) The flight will land in Dubai I

(B) The passengers will board the plane (C) The plane will take off (0) The gate number will be announced

Reading In this section of the test you will have the chance to show how well you understand written English There are three parts to this section with special directions for each part

Part 4 Incomplete Sentences Directions This part of the test has incomplete sentences Four words or phrases marked (A) (8) (e) (D) are given beneath each sentence You are to choose the one word or phrase that best completes the sentence Then on your answer sheet find the number of the question and mark your answer

1 Mr Yangs trip will __ him away from the office for ten days (A) withdraw (B) continue (C) retain (0) keep

2 The company that Marie DuBois started now sells __ products throughout the world (A) its (B) it (C) theirs (D) them

3 If your shipment is not delivered __ Tuesday you can request a full refund for the merchandise (A) at (B) by (C) within (D) while

CHAPTER 4 Standardized Testing 103

Part 6 Error Recognition Directions In this part ofthe test each sentence has four words or phrases underlined The four underlined parts of the sentence are marked (A) (B) (C) (D) You are to identify the one underlined word or phrase that should be corrected or rewritten Then on your answer sheet find the number of the question and mark your answer

1 The pamphlet contains some importance information about the current exhibit ABC D

2 No matter how Jong it taking to finish the annual report it must be done properly ABC D

3 The popularity of jogging appears to have decreased since the past couple of years ABC D

Part 7 Reading Comprehension Directions The questions in this part of the test are based on a selection of reading mateshyrials such as notices letters) forms newspaper and magazine articles) and advertisements You are to choose the one best answer (A) (B) (C) or (OJ to each quesshytion Then on your ariswefsheelfindthe number of the qUestion andmcirkyour answer Answer all questions following each reading selection on thebasis of what is stated or implied in that selection

The Museum ofTechnology is a hands-on museum designed for people to experience science at w()rk~ Visitors are encouraged to use test and handle the objects o~ display Special demonstrations are scheduled for the first and second Wednesdays of each month at 1330 Open Tuesday-Friday 1200-1630 Saturday 1000-1730 and Sunday 11 00-1630

1 When during the month can visitors see special demonstrations (A) Every weekend (B) The first two Wednesdays (C) One afternoon a week (D) Every other Wednesday

Questions 2 and 3 refer to the followi ng notice

NOTICE If you are unable to work because of an extended illness or injury that is not workshyrelated you may be entitled to receive weekly benefits from your employer or the firms insurance company To claim benefits you must file a claim form within thirty days of the first day of your disability Before filing the claim you must ask your doctor to fill in the Doctors Statement on the claim form stating the period of disability

3 To whom is this notice addressed (A) Employers (8) Doctors (C) Employees (D) When paying the bill

4 When must the claim form be filed (A) On the first of the month (8) On the thirtieth of the month (C) On the first day ofdisabifity (D) Within 30 days of the start of disability

Page 30: Standardized Testing Chapter 4 Brown

CHAPTER 4 Standardized Testing 95

Vocabulary

12 Mark has a flair for writing a need b purpose c talent d dislike

13 Bill Collins launched his restaurant last June a moved b started c sold d bought

14 John will not accept the censure a burden b blame c credit d decision

15 I cant think of the answer Can you give me a __ a hint b token c taste d gaze

16 Because fewer people are taking expensive vacations the tourist industry is in a a choke b grope c grumble d slump

17 I disagree with a few of his opinions but __ we agree a deliberately b conclusively c essentially d immensely

Reading The influenza virus is a single molecule built from many millions of single atoms You must have heard of the viruses which are sometimes called living molecules While bacteria can be considered as a type of plant secreting pOisonous substances into the body of the organism they attack viruses are living organisms themselves We may conshysider them as regular chemical molecules since they have a strictly aefined atomic strucshyture but on the other hand we must also consider them as being alive since they are able to multiply in unlimited quantities

18 According to the passage bacteria are a poisons

b larger than viruses c very small d plants

96 CHAPTER 4 Standardized Testing

19 The writer says that viruses are alive because they a have a complex atomic structure b move c multiply d need warmth and light

20 The atomic structure of viruses a is -tJIariable b is strictly defined c cannot be analyzed chemically d is more complex than that of bacteria

International English Language Testing System (fELTS)

I

listening

The Listening Module has four sections The first two sections are concerned with social needs There is a conversation between two speakers and then a monologue For examshyple a conversation about travel arrangements or decisions on a night out and a speech about student services on a university campus or arrangements for meals during a confershyence The final two sections are concerned with situations related more closely to educashytional or training contexts For example conversation between a tutor and a student about an assignment or between three students planning a research project and a lecture or talk ofgeneral academic interest All the topics are ofgeneral interest and it makes no difference what subjects candidates study Tests and tasks become more difficult as the sections progress A range of English accents and dialects are used in the recording which reflects the international usage of IELTS

Academic Reading [A 7S0-word article on-th-e- topic of Wind Power in the US with a short glossary at the end]

Questions 1-5

Complete the summary below

Choose your answers from the box below the summary and write them in boxes 1-5 on your answer sheet Note There are more words or phrases than you will need to fill the gaps You may use any word or phrase more than once

Example The failure during the late 1970s and early 19805 of an attempt to establish a widespread wind power industry in the United States resulted largely from the (1) bull in oil prices during this period The industry is now experiencing a steady (2) due to improveshyments in technology and an increased awareness of the potential in the power of wind The wind turbines that are now being made based in part on the (3) of wide- ranging research in Europe are easier to manufacture and maintain than their predecesshysors This has led wind-turbine makers to be able to standardise and thus minimize (4) There has been growing (S) of the importance of wind power as an energy source

CHAPTER 4 Standardized Testing 97

criticism stability skepticism success operating costs decisions design costs fall effects production costs growth decline failure recognition results

Questions 6-1 0 Look at the following list of issues (Questions 6-10) and implications (A-C) Match each issue with one implication Write the appropriate letters A-C in boxes 6-10 on your anshyswer sheet

Example The current price of one wind-generated kilowatt Answer

6 The recent installation of systems taking advantage of economies of scale

7 The potential of meeting one fifth of current U5 energy requirements by wind power

8 The level of acceptance of current wind turbine technology

9 A comparison of costs between conventional and wind power sources

10 The view of wind power in the European Union

Implications

A provides evidence against claims that electricity produced from wind power is relatively expensive

B supports claims that wind power js an important source of energy

C opposes the view that wind power technology requires further-development

General Training Reading Read the passage on Daybreak trips by coach and look at the statements below On your answer sheet write

TRUE if the statement is true FALSE jf the statement is false

NOlGIVEN if the information is not given in the leaflet

1 MiIlers Coaches owns Cambridges Cam bus fleet

2 Premier is an older company than Millers

3 Most of the Daybreak coaches are less than 5 years old

4 Daybreak fares are more expensive than most of their competitors

5 Soft drinks and refreshments are served on most longer journeys

6 Smoking is permitted at the rear of the coach on longer journeys

7 Tickets must be bought in advance from an authorised Daybreak agent

6 Tickets and seats can be reserved by phoning the Daybreak Hotline

9 Daybreak passengers must join their coach at Cambridge Drummer Street

10 Daybreak cannot guarantee return times

98 CHAPTER 4 Standardized Testing

FROM CAMBRIDGE AND SURROUNDING AREA

SPRING IS INTHEAIR

Welcome to our Spring Daybreak programme which continues the tradition of offering unbeatable value for money day trips and tours All the excursions in this brochure will be operated by Pr~mier Travel Services Limited or Millers Coaches both companies are part of the CHLGroup owners of Cambridges Cambus fleet

WERE PROUD OF OUR TRADITION

Premier was established in 1936 the Company now offers the highest standards of coaching in todays competitive operating environment Miller has an enviable reputation stretching back over the past 20 years offering coach services at realistic prices Weve traveled a long way since our early days of pre-war seaside trips Now our fleet of 50 modern coaches (few are more than five years old) operate throughout Britain and Europe but were pleased to still maintain the high standards of quality and service the trademark of our founders nearly sixty years ago

EXCLUSIVE FEATURES

Admission-inclusive fares All Daybreak fares (unless specifically otherwise stated) include admission charges to the attractions shows and exhibits we visit Many full-day scenic tours are accompanied by a fully trained English Tourist Board Blue Badge guide or local experienced driverguide Some Daybreaks include lunch or afternoon tea Compare our admission inclusive fares and see how much you save Cheapest is not the best and value for money is guaranteed If you compare our bargain Daybreak fares beware--most of our competishytors do not offer an all-inclusive fare

SEAT RESERVATIONS

We value the freedom of choice so you can choose your seat when you book The seat reservation is guaranteed a-nd remains yours at all times when aboard the coach

NO SMOKING COMFORT

With the comfort of our passengers in mind coaches on all our Daybreaks are no smokshying throughout In the interests of fellow passengers comfort we kindly ask that smokers observe our no smoking policy On scenic tours and longer journeys ample refreshment stops are provided when of course smoking is permitted

YOUR QUESTIONS ANSWERED

Do I need to book Booking in advance is strongly recommended as all Daybreak tours are subject to demand Subject to availability stand-by tickets can be purchased from the driver

What ti me does the coach leave The coach departs from Cambridge Drummer Street (Bay 12 adjacent to public toilets) at the time shown There are many additional joining points indicated by departure codes in the brochure If you are joining at one of our less popular joining points you will be adshyvised of your pick-up time (normally by telephone) not less than 48 hours before deparshyture In this way we can minimize the length of pick-up routes and reduce journey times for the majority of passengers

CHAPTER 4 Standardized Testing 99

What time do we get back An approximate return time is shown for each excursion The tim~s shown serve as a guide but road conditions can sometimes cause delay If your arrival will be later than advertised your driver will try to allow for a telephone call during the return journey

Where can I board the coach All the Daybreaks in the brochure leave from Cambridge Drummer Street (Bay 12 adjashycent to public toilets) at the time shown Many Daybreaks offer additional pick-ups for pre-booked passengers within Cambridge and the surrounding area This facility must be requested at the time of booking

Academic Writing Writing Task 1 You should spend about 20 minutes on this task

The graph below shows the different modes of transport used to travel to and from work in one European city in 1950 1970 and 1990

[graph shown here]

Write a report for a university lecturer describing the information shown below You should write at least 150 words

Writing Task 2 You should spend about 40 minutes on this task

Present a written argument or case to an educated reader with no specialist knowledge of the folowing topic

It is inevitable that as technology develops so traditional cultures must be lost Technolshyogy and tradition are incompatible-you cannot have both together

To what extent do you agree or disagree with this statement Give reasons for your answer You should write at least 250 words You should use your own ideas knowlshyedge and experience and support your arguments with examples and relevant evidence

General Training Writing Writing Task 1 You should spend about 20 minutes on this task You rent a house through an agency The heating system has stopped working You phoned the agency a week ago but it has still not been mended Write a letter to the agency Explain the situation and teil them what you want them to do about it

You should write at least 150 words You do NOT need to write your own address

Begin your letter as follows

Dear - ___-I

Writing Task 2 You should spend about 40 minutes on this task As part ofa class assignment you have to write about the following topic

100 CHAPTER 4 Standardized Testing

Some businesses now say that no one can smoke cigarettes in any of their offices Some governments have banned smoking in all public placesThis is a good idea but it takes away some of our freedom

Do you agree or disagree Give reasons for your answer You should write at least 250 words

Speaking In each ofthe three parts of the speaking module a specific function is fulfilled In Part 1 the candidates answer general questions about themselves their homes or families their jobs or studies their interests and a range ofsimilar familiar topic areas This part lasts between four and five minutes In Part 2 the candidate is given a verbal prompt on a card and is asked to talk on a particular topic The candidate has one minute to prepare before speaking at length for between one and two minutes The examiner then asks one or two wind-down questions In Part 3 the examiner and candidate engage in a discusshysion of more abstract issues and concepts which are thematically linked to the topic prompt in Part 2 The discussion lasts between four and five minutes

All interviews are recorded on audiocassette Here is a sample ofa Part 2 topic

Describe a teacher who has greatly influenced you in your education

You shou Id say

where you met them what subject they taught what was special about them

and explain why this person influenced you so much

You will have to talk about the topic for 1 to 2 minutes You have 1 minute to think about what you are going to say You can make some notes if you wish

Test of English for International Communication (TOEICreg)

listening

Part 1 Photographs Directions For each question you will see a picture in your test book and you will hear four short statements The statements will be spoken just one time They will not be printed in your test book so you must listen carefully to understand what the speaker says When you hear the four statements look at the picture in your test book and choose the statement that best describes what you see in the picture Then on your answer sheet find the number of the question and mark your answer

[photograph of a scientist looking through a microscope]

You will hear Look at the picture marked number 1 in your test book

(A) Shes speaking into a microphone (B) Shes put on her glasses (C) She has both eyes open (D) Shes using a microscope

CHAPTER 4 Standardized Testing 101

Part 2 Question-Response Directions In this part of the test you will hear a question or statement spoken in Enshyglish followed by three responses also spoken in English The question or staten1ent and the responses will be spoken just one time They will not be printed in your test book so you must listen carefully to understand what the speakers say You are to choose the best response to each question or statement

Question 1 You will hear Ms Morikawa has worked here for a long time hasnt she

(A) At three oclock (B) No Ive lost my watch (C) More than ten years

Question 2 You will hear Which of these papers has a wider circulation

(A) The morning edition (B) Get more exercise (C) By messenger

Part 3 Short Conversations Directions In this part of the test you will hear short conversations between two people The conversations will not be printed in your test book You will hear the conversations only once so you must listen carefully to understand what the speakers say In your test book you will read a question about each conversation The question will be followed by four answers You are to choose the best answer to each question and mark it on your answer sheet

Question 1 (Man) We should think about finding another restaurant for lunch (Woman) Why The food and service here are great

(Man) Yes but the prices are going up every week

You will read Why is this man unhappy with the restaurant

(A) It is too noisy (B) It is too expensive (C) It is too crowded (D) It is too difficult to find

Question 2 (Woman A) How was Dr Borgs recent trip to Singapore (Woman B) She enjoyed the tour of the port very much (Woman A) They say its one of the most active in Asia

You will read 2 What did Dr Borg find interesting

(A) The tourist center (B) The airport (C) The musical performance (D) The harbor

Part 4 Short Talks Directions In this part of the test you vill hear several short talks Each will be spoken just one time They will not be printed in your test book so you must listen carefully to understand and remember what is said In your test book you will read two or more questions about each short talk The questions will be followed by four answers You are to choose the best answer to each question and mark it on your answer sheet

102 CHAPTER 4 Standardized Testing

You will hear Questions 1 and 2 refer to the following announcement

Good afternoon and welcome aboard Nordair Flight 857 from Copenhagen to Bangkok with intermediate stops in Dubai and Calcutta We are preparing for departure in a few minutes At this time your seat back should be returned to its full upright position and your seat belt s~ould be fastened OUf anticipated total flying time to Dubai is six hours and twenty-five minutes I hope you enjoy the flight You will hecJr Now read question 1 in your test book and answer it You will read 1 What is the final destination of the flight

(A) Bangkok (B) Copenhagen (C) Dubai (O) Calcutta

You will hear Now read question 2 in your test book and answer it You will read 2 What will happen in a few minutes

(A) The flight will land in Dubai I

(B) The passengers will board the plane (C) The plane will take off (0) The gate number will be announced

Reading In this section of the test you will have the chance to show how well you understand written English There are three parts to this section with special directions for each part

Part 4 Incomplete Sentences Directions This part of the test has incomplete sentences Four words or phrases marked (A) (8) (e) (D) are given beneath each sentence You are to choose the one word or phrase that best completes the sentence Then on your answer sheet find the number of the question and mark your answer

1 Mr Yangs trip will __ him away from the office for ten days (A) withdraw (B) continue (C) retain (0) keep

2 The company that Marie DuBois started now sells __ products throughout the world (A) its (B) it (C) theirs (D) them

3 If your shipment is not delivered __ Tuesday you can request a full refund for the merchandise (A) at (B) by (C) within (D) while

CHAPTER 4 Standardized Testing 103

Part 6 Error Recognition Directions In this part ofthe test each sentence has four words or phrases underlined The four underlined parts of the sentence are marked (A) (B) (C) (D) You are to identify the one underlined word or phrase that should be corrected or rewritten Then on your answer sheet find the number of the question and mark your answer

1 The pamphlet contains some importance information about the current exhibit ABC D

2 No matter how Jong it taking to finish the annual report it must be done properly ABC D

3 The popularity of jogging appears to have decreased since the past couple of years ABC D

Part 7 Reading Comprehension Directions The questions in this part of the test are based on a selection of reading mateshyrials such as notices letters) forms newspaper and magazine articles) and advertisements You are to choose the one best answer (A) (B) (C) or (OJ to each quesshytion Then on your ariswefsheelfindthe number of the qUestion andmcirkyour answer Answer all questions following each reading selection on thebasis of what is stated or implied in that selection

The Museum ofTechnology is a hands-on museum designed for people to experience science at w()rk~ Visitors are encouraged to use test and handle the objects o~ display Special demonstrations are scheduled for the first and second Wednesdays of each month at 1330 Open Tuesday-Friday 1200-1630 Saturday 1000-1730 and Sunday 11 00-1630

1 When during the month can visitors see special demonstrations (A) Every weekend (B) The first two Wednesdays (C) One afternoon a week (D) Every other Wednesday

Questions 2 and 3 refer to the followi ng notice

NOTICE If you are unable to work because of an extended illness or injury that is not workshyrelated you may be entitled to receive weekly benefits from your employer or the firms insurance company To claim benefits you must file a claim form within thirty days of the first day of your disability Before filing the claim you must ask your doctor to fill in the Doctors Statement on the claim form stating the period of disability

3 To whom is this notice addressed (A) Employers (8) Doctors (C) Employees (D) When paying the bill

4 When must the claim form be filed (A) On the first of the month (8) On the thirtieth of the month (C) On the first day ofdisabifity (D) Within 30 days of the start of disability

Page 31: Standardized Testing Chapter 4 Brown

96 CHAPTER 4 Standardized Testing

19 The writer says that viruses are alive because they a have a complex atomic structure b move c multiply d need warmth and light

20 The atomic structure of viruses a is -tJIariable b is strictly defined c cannot be analyzed chemically d is more complex than that of bacteria

International English Language Testing System (fELTS)

I

listening

The Listening Module has four sections The first two sections are concerned with social needs There is a conversation between two speakers and then a monologue For examshyple a conversation about travel arrangements or decisions on a night out and a speech about student services on a university campus or arrangements for meals during a confershyence The final two sections are concerned with situations related more closely to educashytional or training contexts For example conversation between a tutor and a student about an assignment or between three students planning a research project and a lecture or talk ofgeneral academic interest All the topics are ofgeneral interest and it makes no difference what subjects candidates study Tests and tasks become more difficult as the sections progress A range of English accents and dialects are used in the recording which reflects the international usage of IELTS

Academic Reading [A 7S0-word article on-th-e- topic of Wind Power in the US with a short glossary at the end]

Questions 1-5

Complete the summary below

Choose your answers from the box below the summary and write them in boxes 1-5 on your answer sheet Note There are more words or phrases than you will need to fill the gaps You may use any word or phrase more than once

Example The failure during the late 1970s and early 19805 of an attempt to establish a widespread wind power industry in the United States resulted largely from the (1) bull in oil prices during this period The industry is now experiencing a steady (2) due to improveshyments in technology and an increased awareness of the potential in the power of wind The wind turbines that are now being made based in part on the (3) of wide- ranging research in Europe are easier to manufacture and maintain than their predecesshysors This has led wind-turbine makers to be able to standardise and thus minimize (4) There has been growing (S) of the importance of wind power as an energy source

CHAPTER 4 Standardized Testing 97

criticism stability skepticism success operating costs decisions design costs fall effects production costs growth decline failure recognition results

Questions 6-1 0 Look at the following list of issues (Questions 6-10) and implications (A-C) Match each issue with one implication Write the appropriate letters A-C in boxes 6-10 on your anshyswer sheet

Example The current price of one wind-generated kilowatt Answer

6 The recent installation of systems taking advantage of economies of scale

7 The potential of meeting one fifth of current U5 energy requirements by wind power

8 The level of acceptance of current wind turbine technology

9 A comparison of costs between conventional and wind power sources

10 The view of wind power in the European Union

Implications

A provides evidence against claims that electricity produced from wind power is relatively expensive

B supports claims that wind power js an important source of energy

C opposes the view that wind power technology requires further-development

General Training Reading Read the passage on Daybreak trips by coach and look at the statements below On your answer sheet write

TRUE if the statement is true FALSE jf the statement is false

NOlGIVEN if the information is not given in the leaflet

1 MiIlers Coaches owns Cambridges Cam bus fleet

2 Premier is an older company than Millers

3 Most of the Daybreak coaches are less than 5 years old

4 Daybreak fares are more expensive than most of their competitors

5 Soft drinks and refreshments are served on most longer journeys

6 Smoking is permitted at the rear of the coach on longer journeys

7 Tickets must be bought in advance from an authorised Daybreak agent

6 Tickets and seats can be reserved by phoning the Daybreak Hotline

9 Daybreak passengers must join their coach at Cambridge Drummer Street

10 Daybreak cannot guarantee return times

98 CHAPTER 4 Standardized Testing

FROM CAMBRIDGE AND SURROUNDING AREA

SPRING IS INTHEAIR

Welcome to our Spring Daybreak programme which continues the tradition of offering unbeatable value for money day trips and tours All the excursions in this brochure will be operated by Pr~mier Travel Services Limited or Millers Coaches both companies are part of the CHLGroup owners of Cambridges Cambus fleet

WERE PROUD OF OUR TRADITION

Premier was established in 1936 the Company now offers the highest standards of coaching in todays competitive operating environment Miller has an enviable reputation stretching back over the past 20 years offering coach services at realistic prices Weve traveled a long way since our early days of pre-war seaside trips Now our fleet of 50 modern coaches (few are more than five years old) operate throughout Britain and Europe but were pleased to still maintain the high standards of quality and service the trademark of our founders nearly sixty years ago

EXCLUSIVE FEATURES

Admission-inclusive fares All Daybreak fares (unless specifically otherwise stated) include admission charges to the attractions shows and exhibits we visit Many full-day scenic tours are accompanied by a fully trained English Tourist Board Blue Badge guide or local experienced driverguide Some Daybreaks include lunch or afternoon tea Compare our admission inclusive fares and see how much you save Cheapest is not the best and value for money is guaranteed If you compare our bargain Daybreak fares beware--most of our competishytors do not offer an all-inclusive fare

SEAT RESERVATIONS

We value the freedom of choice so you can choose your seat when you book The seat reservation is guaranteed a-nd remains yours at all times when aboard the coach

NO SMOKING COMFORT

With the comfort of our passengers in mind coaches on all our Daybreaks are no smokshying throughout In the interests of fellow passengers comfort we kindly ask that smokers observe our no smoking policy On scenic tours and longer journeys ample refreshment stops are provided when of course smoking is permitted

YOUR QUESTIONS ANSWERED

Do I need to book Booking in advance is strongly recommended as all Daybreak tours are subject to demand Subject to availability stand-by tickets can be purchased from the driver

What ti me does the coach leave The coach departs from Cambridge Drummer Street (Bay 12 adjacent to public toilets) at the time shown There are many additional joining points indicated by departure codes in the brochure If you are joining at one of our less popular joining points you will be adshyvised of your pick-up time (normally by telephone) not less than 48 hours before deparshyture In this way we can minimize the length of pick-up routes and reduce journey times for the majority of passengers

CHAPTER 4 Standardized Testing 99

What time do we get back An approximate return time is shown for each excursion The tim~s shown serve as a guide but road conditions can sometimes cause delay If your arrival will be later than advertised your driver will try to allow for a telephone call during the return journey

Where can I board the coach All the Daybreaks in the brochure leave from Cambridge Drummer Street (Bay 12 adjashycent to public toilets) at the time shown Many Daybreaks offer additional pick-ups for pre-booked passengers within Cambridge and the surrounding area This facility must be requested at the time of booking

Academic Writing Writing Task 1 You should spend about 20 minutes on this task

The graph below shows the different modes of transport used to travel to and from work in one European city in 1950 1970 and 1990

[graph shown here]

Write a report for a university lecturer describing the information shown below You should write at least 150 words

Writing Task 2 You should spend about 40 minutes on this task

Present a written argument or case to an educated reader with no specialist knowledge of the folowing topic

It is inevitable that as technology develops so traditional cultures must be lost Technolshyogy and tradition are incompatible-you cannot have both together

To what extent do you agree or disagree with this statement Give reasons for your answer You should write at least 250 words You should use your own ideas knowlshyedge and experience and support your arguments with examples and relevant evidence

General Training Writing Writing Task 1 You should spend about 20 minutes on this task You rent a house through an agency The heating system has stopped working You phoned the agency a week ago but it has still not been mended Write a letter to the agency Explain the situation and teil them what you want them to do about it

You should write at least 150 words You do NOT need to write your own address

Begin your letter as follows

Dear - ___-I

Writing Task 2 You should spend about 40 minutes on this task As part ofa class assignment you have to write about the following topic

100 CHAPTER 4 Standardized Testing

Some businesses now say that no one can smoke cigarettes in any of their offices Some governments have banned smoking in all public placesThis is a good idea but it takes away some of our freedom

Do you agree or disagree Give reasons for your answer You should write at least 250 words

Speaking In each ofthe three parts of the speaking module a specific function is fulfilled In Part 1 the candidates answer general questions about themselves their homes or families their jobs or studies their interests and a range ofsimilar familiar topic areas This part lasts between four and five minutes In Part 2 the candidate is given a verbal prompt on a card and is asked to talk on a particular topic The candidate has one minute to prepare before speaking at length for between one and two minutes The examiner then asks one or two wind-down questions In Part 3 the examiner and candidate engage in a discusshysion of more abstract issues and concepts which are thematically linked to the topic prompt in Part 2 The discussion lasts between four and five minutes

All interviews are recorded on audiocassette Here is a sample ofa Part 2 topic

Describe a teacher who has greatly influenced you in your education

You shou Id say

where you met them what subject they taught what was special about them

and explain why this person influenced you so much

You will have to talk about the topic for 1 to 2 minutes You have 1 minute to think about what you are going to say You can make some notes if you wish

Test of English for International Communication (TOEICreg)

listening

Part 1 Photographs Directions For each question you will see a picture in your test book and you will hear four short statements The statements will be spoken just one time They will not be printed in your test book so you must listen carefully to understand what the speaker says When you hear the four statements look at the picture in your test book and choose the statement that best describes what you see in the picture Then on your answer sheet find the number of the question and mark your answer

[photograph of a scientist looking through a microscope]

You will hear Look at the picture marked number 1 in your test book

(A) Shes speaking into a microphone (B) Shes put on her glasses (C) She has both eyes open (D) Shes using a microscope

CHAPTER 4 Standardized Testing 101

Part 2 Question-Response Directions In this part of the test you will hear a question or statement spoken in Enshyglish followed by three responses also spoken in English The question or staten1ent and the responses will be spoken just one time They will not be printed in your test book so you must listen carefully to understand what the speakers say You are to choose the best response to each question or statement

Question 1 You will hear Ms Morikawa has worked here for a long time hasnt she

(A) At three oclock (B) No Ive lost my watch (C) More than ten years

Question 2 You will hear Which of these papers has a wider circulation

(A) The morning edition (B) Get more exercise (C) By messenger

Part 3 Short Conversations Directions In this part of the test you will hear short conversations between two people The conversations will not be printed in your test book You will hear the conversations only once so you must listen carefully to understand what the speakers say In your test book you will read a question about each conversation The question will be followed by four answers You are to choose the best answer to each question and mark it on your answer sheet

Question 1 (Man) We should think about finding another restaurant for lunch (Woman) Why The food and service here are great

(Man) Yes but the prices are going up every week

You will read Why is this man unhappy with the restaurant

(A) It is too noisy (B) It is too expensive (C) It is too crowded (D) It is too difficult to find

Question 2 (Woman A) How was Dr Borgs recent trip to Singapore (Woman B) She enjoyed the tour of the port very much (Woman A) They say its one of the most active in Asia

You will read 2 What did Dr Borg find interesting

(A) The tourist center (B) The airport (C) The musical performance (D) The harbor

Part 4 Short Talks Directions In this part of the test you vill hear several short talks Each will be spoken just one time They will not be printed in your test book so you must listen carefully to understand and remember what is said In your test book you will read two or more questions about each short talk The questions will be followed by four answers You are to choose the best answer to each question and mark it on your answer sheet

102 CHAPTER 4 Standardized Testing

You will hear Questions 1 and 2 refer to the following announcement

Good afternoon and welcome aboard Nordair Flight 857 from Copenhagen to Bangkok with intermediate stops in Dubai and Calcutta We are preparing for departure in a few minutes At this time your seat back should be returned to its full upright position and your seat belt s~ould be fastened OUf anticipated total flying time to Dubai is six hours and twenty-five minutes I hope you enjoy the flight You will hecJr Now read question 1 in your test book and answer it You will read 1 What is the final destination of the flight

(A) Bangkok (B) Copenhagen (C) Dubai (O) Calcutta

You will hear Now read question 2 in your test book and answer it You will read 2 What will happen in a few minutes

(A) The flight will land in Dubai I

(B) The passengers will board the plane (C) The plane will take off (0) The gate number will be announced

Reading In this section of the test you will have the chance to show how well you understand written English There are three parts to this section with special directions for each part

Part 4 Incomplete Sentences Directions This part of the test has incomplete sentences Four words or phrases marked (A) (8) (e) (D) are given beneath each sentence You are to choose the one word or phrase that best completes the sentence Then on your answer sheet find the number of the question and mark your answer

1 Mr Yangs trip will __ him away from the office for ten days (A) withdraw (B) continue (C) retain (0) keep

2 The company that Marie DuBois started now sells __ products throughout the world (A) its (B) it (C) theirs (D) them

3 If your shipment is not delivered __ Tuesday you can request a full refund for the merchandise (A) at (B) by (C) within (D) while

CHAPTER 4 Standardized Testing 103

Part 6 Error Recognition Directions In this part ofthe test each sentence has four words or phrases underlined The four underlined parts of the sentence are marked (A) (B) (C) (D) You are to identify the one underlined word or phrase that should be corrected or rewritten Then on your answer sheet find the number of the question and mark your answer

1 The pamphlet contains some importance information about the current exhibit ABC D

2 No matter how Jong it taking to finish the annual report it must be done properly ABC D

3 The popularity of jogging appears to have decreased since the past couple of years ABC D

Part 7 Reading Comprehension Directions The questions in this part of the test are based on a selection of reading mateshyrials such as notices letters) forms newspaper and magazine articles) and advertisements You are to choose the one best answer (A) (B) (C) or (OJ to each quesshytion Then on your ariswefsheelfindthe number of the qUestion andmcirkyour answer Answer all questions following each reading selection on thebasis of what is stated or implied in that selection

The Museum ofTechnology is a hands-on museum designed for people to experience science at w()rk~ Visitors are encouraged to use test and handle the objects o~ display Special demonstrations are scheduled for the first and second Wednesdays of each month at 1330 Open Tuesday-Friday 1200-1630 Saturday 1000-1730 and Sunday 11 00-1630

1 When during the month can visitors see special demonstrations (A) Every weekend (B) The first two Wednesdays (C) One afternoon a week (D) Every other Wednesday

Questions 2 and 3 refer to the followi ng notice

NOTICE If you are unable to work because of an extended illness or injury that is not workshyrelated you may be entitled to receive weekly benefits from your employer or the firms insurance company To claim benefits you must file a claim form within thirty days of the first day of your disability Before filing the claim you must ask your doctor to fill in the Doctors Statement on the claim form stating the period of disability

3 To whom is this notice addressed (A) Employers (8) Doctors (C) Employees (D) When paying the bill

4 When must the claim form be filed (A) On the first of the month (8) On the thirtieth of the month (C) On the first day ofdisabifity (D) Within 30 days of the start of disability

Page 32: Standardized Testing Chapter 4 Brown

CHAPTER 4 Standardized Testing 97

criticism stability skepticism success operating costs decisions design costs fall effects production costs growth decline failure recognition results

Questions 6-1 0 Look at the following list of issues (Questions 6-10) and implications (A-C) Match each issue with one implication Write the appropriate letters A-C in boxes 6-10 on your anshyswer sheet

Example The current price of one wind-generated kilowatt Answer

6 The recent installation of systems taking advantage of economies of scale

7 The potential of meeting one fifth of current U5 energy requirements by wind power

8 The level of acceptance of current wind turbine technology

9 A comparison of costs between conventional and wind power sources

10 The view of wind power in the European Union

Implications

A provides evidence against claims that electricity produced from wind power is relatively expensive

B supports claims that wind power js an important source of energy

C opposes the view that wind power technology requires further-development

General Training Reading Read the passage on Daybreak trips by coach and look at the statements below On your answer sheet write

TRUE if the statement is true FALSE jf the statement is false

NOlGIVEN if the information is not given in the leaflet

1 MiIlers Coaches owns Cambridges Cam bus fleet

2 Premier is an older company than Millers

3 Most of the Daybreak coaches are less than 5 years old

4 Daybreak fares are more expensive than most of their competitors

5 Soft drinks and refreshments are served on most longer journeys

6 Smoking is permitted at the rear of the coach on longer journeys

7 Tickets must be bought in advance from an authorised Daybreak agent

6 Tickets and seats can be reserved by phoning the Daybreak Hotline

9 Daybreak passengers must join their coach at Cambridge Drummer Street

10 Daybreak cannot guarantee return times

98 CHAPTER 4 Standardized Testing

FROM CAMBRIDGE AND SURROUNDING AREA

SPRING IS INTHEAIR

Welcome to our Spring Daybreak programme which continues the tradition of offering unbeatable value for money day trips and tours All the excursions in this brochure will be operated by Pr~mier Travel Services Limited or Millers Coaches both companies are part of the CHLGroup owners of Cambridges Cambus fleet

WERE PROUD OF OUR TRADITION

Premier was established in 1936 the Company now offers the highest standards of coaching in todays competitive operating environment Miller has an enviable reputation stretching back over the past 20 years offering coach services at realistic prices Weve traveled a long way since our early days of pre-war seaside trips Now our fleet of 50 modern coaches (few are more than five years old) operate throughout Britain and Europe but were pleased to still maintain the high standards of quality and service the trademark of our founders nearly sixty years ago

EXCLUSIVE FEATURES

Admission-inclusive fares All Daybreak fares (unless specifically otherwise stated) include admission charges to the attractions shows and exhibits we visit Many full-day scenic tours are accompanied by a fully trained English Tourist Board Blue Badge guide or local experienced driverguide Some Daybreaks include lunch or afternoon tea Compare our admission inclusive fares and see how much you save Cheapest is not the best and value for money is guaranteed If you compare our bargain Daybreak fares beware--most of our competishytors do not offer an all-inclusive fare

SEAT RESERVATIONS

We value the freedom of choice so you can choose your seat when you book The seat reservation is guaranteed a-nd remains yours at all times when aboard the coach

NO SMOKING COMFORT

With the comfort of our passengers in mind coaches on all our Daybreaks are no smokshying throughout In the interests of fellow passengers comfort we kindly ask that smokers observe our no smoking policy On scenic tours and longer journeys ample refreshment stops are provided when of course smoking is permitted

YOUR QUESTIONS ANSWERED

Do I need to book Booking in advance is strongly recommended as all Daybreak tours are subject to demand Subject to availability stand-by tickets can be purchased from the driver

What ti me does the coach leave The coach departs from Cambridge Drummer Street (Bay 12 adjacent to public toilets) at the time shown There are many additional joining points indicated by departure codes in the brochure If you are joining at one of our less popular joining points you will be adshyvised of your pick-up time (normally by telephone) not less than 48 hours before deparshyture In this way we can minimize the length of pick-up routes and reduce journey times for the majority of passengers

CHAPTER 4 Standardized Testing 99

What time do we get back An approximate return time is shown for each excursion The tim~s shown serve as a guide but road conditions can sometimes cause delay If your arrival will be later than advertised your driver will try to allow for a telephone call during the return journey

Where can I board the coach All the Daybreaks in the brochure leave from Cambridge Drummer Street (Bay 12 adjashycent to public toilets) at the time shown Many Daybreaks offer additional pick-ups for pre-booked passengers within Cambridge and the surrounding area This facility must be requested at the time of booking

Academic Writing Writing Task 1 You should spend about 20 minutes on this task

The graph below shows the different modes of transport used to travel to and from work in one European city in 1950 1970 and 1990

[graph shown here]

Write a report for a university lecturer describing the information shown below You should write at least 150 words

Writing Task 2 You should spend about 40 minutes on this task

Present a written argument or case to an educated reader with no specialist knowledge of the folowing topic

It is inevitable that as technology develops so traditional cultures must be lost Technolshyogy and tradition are incompatible-you cannot have both together

To what extent do you agree or disagree with this statement Give reasons for your answer You should write at least 250 words You should use your own ideas knowlshyedge and experience and support your arguments with examples and relevant evidence

General Training Writing Writing Task 1 You should spend about 20 minutes on this task You rent a house through an agency The heating system has stopped working You phoned the agency a week ago but it has still not been mended Write a letter to the agency Explain the situation and teil them what you want them to do about it

You should write at least 150 words You do NOT need to write your own address

Begin your letter as follows

Dear - ___-I

Writing Task 2 You should spend about 40 minutes on this task As part ofa class assignment you have to write about the following topic

100 CHAPTER 4 Standardized Testing

Some businesses now say that no one can smoke cigarettes in any of their offices Some governments have banned smoking in all public placesThis is a good idea but it takes away some of our freedom

Do you agree or disagree Give reasons for your answer You should write at least 250 words

Speaking In each ofthe three parts of the speaking module a specific function is fulfilled In Part 1 the candidates answer general questions about themselves their homes or families their jobs or studies their interests and a range ofsimilar familiar topic areas This part lasts between four and five minutes In Part 2 the candidate is given a verbal prompt on a card and is asked to talk on a particular topic The candidate has one minute to prepare before speaking at length for between one and two minutes The examiner then asks one or two wind-down questions In Part 3 the examiner and candidate engage in a discusshysion of more abstract issues and concepts which are thematically linked to the topic prompt in Part 2 The discussion lasts between four and five minutes

All interviews are recorded on audiocassette Here is a sample ofa Part 2 topic

Describe a teacher who has greatly influenced you in your education

You shou Id say

where you met them what subject they taught what was special about them

and explain why this person influenced you so much

You will have to talk about the topic for 1 to 2 minutes You have 1 minute to think about what you are going to say You can make some notes if you wish

Test of English for International Communication (TOEICreg)

listening

Part 1 Photographs Directions For each question you will see a picture in your test book and you will hear four short statements The statements will be spoken just one time They will not be printed in your test book so you must listen carefully to understand what the speaker says When you hear the four statements look at the picture in your test book and choose the statement that best describes what you see in the picture Then on your answer sheet find the number of the question and mark your answer

[photograph of a scientist looking through a microscope]

You will hear Look at the picture marked number 1 in your test book

(A) Shes speaking into a microphone (B) Shes put on her glasses (C) She has both eyes open (D) Shes using a microscope

CHAPTER 4 Standardized Testing 101

Part 2 Question-Response Directions In this part of the test you will hear a question or statement spoken in Enshyglish followed by three responses also spoken in English The question or staten1ent and the responses will be spoken just one time They will not be printed in your test book so you must listen carefully to understand what the speakers say You are to choose the best response to each question or statement

Question 1 You will hear Ms Morikawa has worked here for a long time hasnt she

(A) At three oclock (B) No Ive lost my watch (C) More than ten years

Question 2 You will hear Which of these papers has a wider circulation

(A) The morning edition (B) Get more exercise (C) By messenger

Part 3 Short Conversations Directions In this part of the test you will hear short conversations between two people The conversations will not be printed in your test book You will hear the conversations only once so you must listen carefully to understand what the speakers say In your test book you will read a question about each conversation The question will be followed by four answers You are to choose the best answer to each question and mark it on your answer sheet

Question 1 (Man) We should think about finding another restaurant for lunch (Woman) Why The food and service here are great

(Man) Yes but the prices are going up every week

You will read Why is this man unhappy with the restaurant

(A) It is too noisy (B) It is too expensive (C) It is too crowded (D) It is too difficult to find

Question 2 (Woman A) How was Dr Borgs recent trip to Singapore (Woman B) She enjoyed the tour of the port very much (Woman A) They say its one of the most active in Asia

You will read 2 What did Dr Borg find interesting

(A) The tourist center (B) The airport (C) The musical performance (D) The harbor

Part 4 Short Talks Directions In this part of the test you vill hear several short talks Each will be spoken just one time They will not be printed in your test book so you must listen carefully to understand and remember what is said In your test book you will read two or more questions about each short talk The questions will be followed by four answers You are to choose the best answer to each question and mark it on your answer sheet

102 CHAPTER 4 Standardized Testing

You will hear Questions 1 and 2 refer to the following announcement

Good afternoon and welcome aboard Nordair Flight 857 from Copenhagen to Bangkok with intermediate stops in Dubai and Calcutta We are preparing for departure in a few minutes At this time your seat back should be returned to its full upright position and your seat belt s~ould be fastened OUf anticipated total flying time to Dubai is six hours and twenty-five minutes I hope you enjoy the flight You will hecJr Now read question 1 in your test book and answer it You will read 1 What is the final destination of the flight

(A) Bangkok (B) Copenhagen (C) Dubai (O) Calcutta

You will hear Now read question 2 in your test book and answer it You will read 2 What will happen in a few minutes

(A) The flight will land in Dubai I

(B) The passengers will board the plane (C) The plane will take off (0) The gate number will be announced

Reading In this section of the test you will have the chance to show how well you understand written English There are three parts to this section with special directions for each part

Part 4 Incomplete Sentences Directions This part of the test has incomplete sentences Four words or phrases marked (A) (8) (e) (D) are given beneath each sentence You are to choose the one word or phrase that best completes the sentence Then on your answer sheet find the number of the question and mark your answer

1 Mr Yangs trip will __ him away from the office for ten days (A) withdraw (B) continue (C) retain (0) keep

2 The company that Marie DuBois started now sells __ products throughout the world (A) its (B) it (C) theirs (D) them

3 If your shipment is not delivered __ Tuesday you can request a full refund for the merchandise (A) at (B) by (C) within (D) while

CHAPTER 4 Standardized Testing 103

Part 6 Error Recognition Directions In this part ofthe test each sentence has four words or phrases underlined The four underlined parts of the sentence are marked (A) (B) (C) (D) You are to identify the one underlined word or phrase that should be corrected or rewritten Then on your answer sheet find the number of the question and mark your answer

1 The pamphlet contains some importance information about the current exhibit ABC D

2 No matter how Jong it taking to finish the annual report it must be done properly ABC D

3 The popularity of jogging appears to have decreased since the past couple of years ABC D

Part 7 Reading Comprehension Directions The questions in this part of the test are based on a selection of reading mateshyrials such as notices letters) forms newspaper and magazine articles) and advertisements You are to choose the one best answer (A) (B) (C) or (OJ to each quesshytion Then on your ariswefsheelfindthe number of the qUestion andmcirkyour answer Answer all questions following each reading selection on thebasis of what is stated or implied in that selection

The Museum ofTechnology is a hands-on museum designed for people to experience science at w()rk~ Visitors are encouraged to use test and handle the objects o~ display Special demonstrations are scheduled for the first and second Wednesdays of each month at 1330 Open Tuesday-Friday 1200-1630 Saturday 1000-1730 and Sunday 11 00-1630

1 When during the month can visitors see special demonstrations (A) Every weekend (B) The first two Wednesdays (C) One afternoon a week (D) Every other Wednesday

Questions 2 and 3 refer to the followi ng notice

NOTICE If you are unable to work because of an extended illness or injury that is not workshyrelated you may be entitled to receive weekly benefits from your employer or the firms insurance company To claim benefits you must file a claim form within thirty days of the first day of your disability Before filing the claim you must ask your doctor to fill in the Doctors Statement on the claim form stating the period of disability

3 To whom is this notice addressed (A) Employers (8) Doctors (C) Employees (D) When paying the bill

4 When must the claim form be filed (A) On the first of the month (8) On the thirtieth of the month (C) On the first day ofdisabifity (D) Within 30 days of the start of disability

Page 33: Standardized Testing Chapter 4 Brown

98 CHAPTER 4 Standardized Testing

FROM CAMBRIDGE AND SURROUNDING AREA

SPRING IS INTHEAIR

Welcome to our Spring Daybreak programme which continues the tradition of offering unbeatable value for money day trips and tours All the excursions in this brochure will be operated by Pr~mier Travel Services Limited or Millers Coaches both companies are part of the CHLGroup owners of Cambridges Cambus fleet

WERE PROUD OF OUR TRADITION

Premier was established in 1936 the Company now offers the highest standards of coaching in todays competitive operating environment Miller has an enviable reputation stretching back over the past 20 years offering coach services at realistic prices Weve traveled a long way since our early days of pre-war seaside trips Now our fleet of 50 modern coaches (few are more than five years old) operate throughout Britain and Europe but were pleased to still maintain the high standards of quality and service the trademark of our founders nearly sixty years ago

EXCLUSIVE FEATURES

Admission-inclusive fares All Daybreak fares (unless specifically otherwise stated) include admission charges to the attractions shows and exhibits we visit Many full-day scenic tours are accompanied by a fully trained English Tourist Board Blue Badge guide or local experienced driverguide Some Daybreaks include lunch or afternoon tea Compare our admission inclusive fares and see how much you save Cheapest is not the best and value for money is guaranteed If you compare our bargain Daybreak fares beware--most of our competishytors do not offer an all-inclusive fare

SEAT RESERVATIONS

We value the freedom of choice so you can choose your seat when you book The seat reservation is guaranteed a-nd remains yours at all times when aboard the coach

NO SMOKING COMFORT

With the comfort of our passengers in mind coaches on all our Daybreaks are no smokshying throughout In the interests of fellow passengers comfort we kindly ask that smokers observe our no smoking policy On scenic tours and longer journeys ample refreshment stops are provided when of course smoking is permitted

YOUR QUESTIONS ANSWERED

Do I need to book Booking in advance is strongly recommended as all Daybreak tours are subject to demand Subject to availability stand-by tickets can be purchased from the driver

What ti me does the coach leave The coach departs from Cambridge Drummer Street (Bay 12 adjacent to public toilets) at the time shown There are many additional joining points indicated by departure codes in the brochure If you are joining at one of our less popular joining points you will be adshyvised of your pick-up time (normally by telephone) not less than 48 hours before deparshyture In this way we can minimize the length of pick-up routes and reduce journey times for the majority of passengers

CHAPTER 4 Standardized Testing 99

What time do we get back An approximate return time is shown for each excursion The tim~s shown serve as a guide but road conditions can sometimes cause delay If your arrival will be later than advertised your driver will try to allow for a telephone call during the return journey

Where can I board the coach All the Daybreaks in the brochure leave from Cambridge Drummer Street (Bay 12 adjashycent to public toilets) at the time shown Many Daybreaks offer additional pick-ups for pre-booked passengers within Cambridge and the surrounding area This facility must be requested at the time of booking

Academic Writing Writing Task 1 You should spend about 20 minutes on this task

The graph below shows the different modes of transport used to travel to and from work in one European city in 1950 1970 and 1990

[graph shown here]

Write a report for a university lecturer describing the information shown below You should write at least 150 words

Writing Task 2 You should spend about 40 minutes on this task

Present a written argument or case to an educated reader with no specialist knowledge of the folowing topic

It is inevitable that as technology develops so traditional cultures must be lost Technolshyogy and tradition are incompatible-you cannot have both together

To what extent do you agree or disagree with this statement Give reasons for your answer You should write at least 250 words You should use your own ideas knowlshyedge and experience and support your arguments with examples and relevant evidence

General Training Writing Writing Task 1 You should spend about 20 minutes on this task You rent a house through an agency The heating system has stopped working You phoned the agency a week ago but it has still not been mended Write a letter to the agency Explain the situation and teil them what you want them to do about it

You should write at least 150 words You do NOT need to write your own address

Begin your letter as follows

Dear - ___-I

Writing Task 2 You should spend about 40 minutes on this task As part ofa class assignment you have to write about the following topic

100 CHAPTER 4 Standardized Testing

Some businesses now say that no one can smoke cigarettes in any of their offices Some governments have banned smoking in all public placesThis is a good idea but it takes away some of our freedom

Do you agree or disagree Give reasons for your answer You should write at least 250 words

Speaking In each ofthe three parts of the speaking module a specific function is fulfilled In Part 1 the candidates answer general questions about themselves their homes or families their jobs or studies their interests and a range ofsimilar familiar topic areas This part lasts between four and five minutes In Part 2 the candidate is given a verbal prompt on a card and is asked to talk on a particular topic The candidate has one minute to prepare before speaking at length for between one and two minutes The examiner then asks one or two wind-down questions In Part 3 the examiner and candidate engage in a discusshysion of more abstract issues and concepts which are thematically linked to the topic prompt in Part 2 The discussion lasts between four and five minutes

All interviews are recorded on audiocassette Here is a sample ofa Part 2 topic

Describe a teacher who has greatly influenced you in your education

You shou Id say

where you met them what subject they taught what was special about them

and explain why this person influenced you so much

You will have to talk about the topic for 1 to 2 minutes You have 1 minute to think about what you are going to say You can make some notes if you wish

Test of English for International Communication (TOEICreg)

listening

Part 1 Photographs Directions For each question you will see a picture in your test book and you will hear four short statements The statements will be spoken just one time They will not be printed in your test book so you must listen carefully to understand what the speaker says When you hear the four statements look at the picture in your test book and choose the statement that best describes what you see in the picture Then on your answer sheet find the number of the question and mark your answer

[photograph of a scientist looking through a microscope]

You will hear Look at the picture marked number 1 in your test book

(A) Shes speaking into a microphone (B) Shes put on her glasses (C) She has both eyes open (D) Shes using a microscope

CHAPTER 4 Standardized Testing 101

Part 2 Question-Response Directions In this part of the test you will hear a question or statement spoken in Enshyglish followed by three responses also spoken in English The question or staten1ent and the responses will be spoken just one time They will not be printed in your test book so you must listen carefully to understand what the speakers say You are to choose the best response to each question or statement

Question 1 You will hear Ms Morikawa has worked here for a long time hasnt she

(A) At three oclock (B) No Ive lost my watch (C) More than ten years

Question 2 You will hear Which of these papers has a wider circulation

(A) The morning edition (B) Get more exercise (C) By messenger

Part 3 Short Conversations Directions In this part of the test you will hear short conversations between two people The conversations will not be printed in your test book You will hear the conversations only once so you must listen carefully to understand what the speakers say In your test book you will read a question about each conversation The question will be followed by four answers You are to choose the best answer to each question and mark it on your answer sheet

Question 1 (Man) We should think about finding another restaurant for lunch (Woman) Why The food and service here are great

(Man) Yes but the prices are going up every week

You will read Why is this man unhappy with the restaurant

(A) It is too noisy (B) It is too expensive (C) It is too crowded (D) It is too difficult to find

Question 2 (Woman A) How was Dr Borgs recent trip to Singapore (Woman B) She enjoyed the tour of the port very much (Woman A) They say its one of the most active in Asia

You will read 2 What did Dr Borg find interesting

(A) The tourist center (B) The airport (C) The musical performance (D) The harbor

Part 4 Short Talks Directions In this part of the test you vill hear several short talks Each will be spoken just one time They will not be printed in your test book so you must listen carefully to understand and remember what is said In your test book you will read two or more questions about each short talk The questions will be followed by four answers You are to choose the best answer to each question and mark it on your answer sheet

102 CHAPTER 4 Standardized Testing

You will hear Questions 1 and 2 refer to the following announcement

Good afternoon and welcome aboard Nordair Flight 857 from Copenhagen to Bangkok with intermediate stops in Dubai and Calcutta We are preparing for departure in a few minutes At this time your seat back should be returned to its full upright position and your seat belt s~ould be fastened OUf anticipated total flying time to Dubai is six hours and twenty-five minutes I hope you enjoy the flight You will hecJr Now read question 1 in your test book and answer it You will read 1 What is the final destination of the flight

(A) Bangkok (B) Copenhagen (C) Dubai (O) Calcutta

You will hear Now read question 2 in your test book and answer it You will read 2 What will happen in a few minutes

(A) The flight will land in Dubai I

(B) The passengers will board the plane (C) The plane will take off (0) The gate number will be announced

Reading In this section of the test you will have the chance to show how well you understand written English There are three parts to this section with special directions for each part

Part 4 Incomplete Sentences Directions This part of the test has incomplete sentences Four words or phrases marked (A) (8) (e) (D) are given beneath each sentence You are to choose the one word or phrase that best completes the sentence Then on your answer sheet find the number of the question and mark your answer

1 Mr Yangs trip will __ him away from the office for ten days (A) withdraw (B) continue (C) retain (0) keep

2 The company that Marie DuBois started now sells __ products throughout the world (A) its (B) it (C) theirs (D) them

3 If your shipment is not delivered __ Tuesday you can request a full refund for the merchandise (A) at (B) by (C) within (D) while

CHAPTER 4 Standardized Testing 103

Part 6 Error Recognition Directions In this part ofthe test each sentence has four words or phrases underlined The four underlined parts of the sentence are marked (A) (B) (C) (D) You are to identify the one underlined word or phrase that should be corrected or rewritten Then on your answer sheet find the number of the question and mark your answer

1 The pamphlet contains some importance information about the current exhibit ABC D

2 No matter how Jong it taking to finish the annual report it must be done properly ABC D

3 The popularity of jogging appears to have decreased since the past couple of years ABC D

Part 7 Reading Comprehension Directions The questions in this part of the test are based on a selection of reading mateshyrials such as notices letters) forms newspaper and magazine articles) and advertisements You are to choose the one best answer (A) (B) (C) or (OJ to each quesshytion Then on your ariswefsheelfindthe number of the qUestion andmcirkyour answer Answer all questions following each reading selection on thebasis of what is stated or implied in that selection

The Museum ofTechnology is a hands-on museum designed for people to experience science at w()rk~ Visitors are encouraged to use test and handle the objects o~ display Special demonstrations are scheduled for the first and second Wednesdays of each month at 1330 Open Tuesday-Friday 1200-1630 Saturday 1000-1730 and Sunday 11 00-1630

1 When during the month can visitors see special demonstrations (A) Every weekend (B) The first two Wednesdays (C) One afternoon a week (D) Every other Wednesday

Questions 2 and 3 refer to the followi ng notice

NOTICE If you are unable to work because of an extended illness or injury that is not workshyrelated you may be entitled to receive weekly benefits from your employer or the firms insurance company To claim benefits you must file a claim form within thirty days of the first day of your disability Before filing the claim you must ask your doctor to fill in the Doctors Statement on the claim form stating the period of disability

3 To whom is this notice addressed (A) Employers (8) Doctors (C) Employees (D) When paying the bill

4 When must the claim form be filed (A) On the first of the month (8) On the thirtieth of the month (C) On the first day ofdisabifity (D) Within 30 days of the start of disability

Page 34: Standardized Testing Chapter 4 Brown

CHAPTER 4 Standardized Testing 99

What time do we get back An approximate return time is shown for each excursion The tim~s shown serve as a guide but road conditions can sometimes cause delay If your arrival will be later than advertised your driver will try to allow for a telephone call during the return journey

Where can I board the coach All the Daybreaks in the brochure leave from Cambridge Drummer Street (Bay 12 adjashycent to public toilets) at the time shown Many Daybreaks offer additional pick-ups for pre-booked passengers within Cambridge and the surrounding area This facility must be requested at the time of booking

Academic Writing Writing Task 1 You should spend about 20 minutes on this task

The graph below shows the different modes of transport used to travel to and from work in one European city in 1950 1970 and 1990

[graph shown here]

Write a report for a university lecturer describing the information shown below You should write at least 150 words

Writing Task 2 You should spend about 40 minutes on this task

Present a written argument or case to an educated reader with no specialist knowledge of the folowing topic

It is inevitable that as technology develops so traditional cultures must be lost Technolshyogy and tradition are incompatible-you cannot have both together

To what extent do you agree or disagree with this statement Give reasons for your answer You should write at least 250 words You should use your own ideas knowlshyedge and experience and support your arguments with examples and relevant evidence

General Training Writing Writing Task 1 You should spend about 20 minutes on this task You rent a house through an agency The heating system has stopped working You phoned the agency a week ago but it has still not been mended Write a letter to the agency Explain the situation and teil them what you want them to do about it

You should write at least 150 words You do NOT need to write your own address

Begin your letter as follows

Dear - ___-I

Writing Task 2 You should spend about 40 minutes on this task As part ofa class assignment you have to write about the following topic

100 CHAPTER 4 Standardized Testing

Some businesses now say that no one can smoke cigarettes in any of their offices Some governments have banned smoking in all public placesThis is a good idea but it takes away some of our freedom

Do you agree or disagree Give reasons for your answer You should write at least 250 words

Speaking In each ofthe three parts of the speaking module a specific function is fulfilled In Part 1 the candidates answer general questions about themselves their homes or families their jobs or studies their interests and a range ofsimilar familiar topic areas This part lasts between four and five minutes In Part 2 the candidate is given a verbal prompt on a card and is asked to talk on a particular topic The candidate has one minute to prepare before speaking at length for between one and two minutes The examiner then asks one or two wind-down questions In Part 3 the examiner and candidate engage in a discusshysion of more abstract issues and concepts which are thematically linked to the topic prompt in Part 2 The discussion lasts between four and five minutes

All interviews are recorded on audiocassette Here is a sample ofa Part 2 topic

Describe a teacher who has greatly influenced you in your education

You shou Id say

where you met them what subject they taught what was special about them

and explain why this person influenced you so much

You will have to talk about the topic for 1 to 2 minutes You have 1 minute to think about what you are going to say You can make some notes if you wish

Test of English for International Communication (TOEICreg)

listening

Part 1 Photographs Directions For each question you will see a picture in your test book and you will hear four short statements The statements will be spoken just one time They will not be printed in your test book so you must listen carefully to understand what the speaker says When you hear the four statements look at the picture in your test book and choose the statement that best describes what you see in the picture Then on your answer sheet find the number of the question and mark your answer

[photograph of a scientist looking through a microscope]

You will hear Look at the picture marked number 1 in your test book

(A) Shes speaking into a microphone (B) Shes put on her glasses (C) She has both eyes open (D) Shes using a microscope

CHAPTER 4 Standardized Testing 101

Part 2 Question-Response Directions In this part of the test you will hear a question or statement spoken in Enshyglish followed by three responses also spoken in English The question or staten1ent and the responses will be spoken just one time They will not be printed in your test book so you must listen carefully to understand what the speakers say You are to choose the best response to each question or statement

Question 1 You will hear Ms Morikawa has worked here for a long time hasnt she

(A) At three oclock (B) No Ive lost my watch (C) More than ten years

Question 2 You will hear Which of these papers has a wider circulation

(A) The morning edition (B) Get more exercise (C) By messenger

Part 3 Short Conversations Directions In this part of the test you will hear short conversations between two people The conversations will not be printed in your test book You will hear the conversations only once so you must listen carefully to understand what the speakers say In your test book you will read a question about each conversation The question will be followed by four answers You are to choose the best answer to each question and mark it on your answer sheet

Question 1 (Man) We should think about finding another restaurant for lunch (Woman) Why The food and service here are great

(Man) Yes but the prices are going up every week

You will read Why is this man unhappy with the restaurant

(A) It is too noisy (B) It is too expensive (C) It is too crowded (D) It is too difficult to find

Question 2 (Woman A) How was Dr Borgs recent trip to Singapore (Woman B) She enjoyed the tour of the port very much (Woman A) They say its one of the most active in Asia

You will read 2 What did Dr Borg find interesting

(A) The tourist center (B) The airport (C) The musical performance (D) The harbor

Part 4 Short Talks Directions In this part of the test you vill hear several short talks Each will be spoken just one time They will not be printed in your test book so you must listen carefully to understand and remember what is said In your test book you will read two or more questions about each short talk The questions will be followed by four answers You are to choose the best answer to each question and mark it on your answer sheet

102 CHAPTER 4 Standardized Testing

You will hear Questions 1 and 2 refer to the following announcement

Good afternoon and welcome aboard Nordair Flight 857 from Copenhagen to Bangkok with intermediate stops in Dubai and Calcutta We are preparing for departure in a few minutes At this time your seat back should be returned to its full upright position and your seat belt s~ould be fastened OUf anticipated total flying time to Dubai is six hours and twenty-five minutes I hope you enjoy the flight You will hecJr Now read question 1 in your test book and answer it You will read 1 What is the final destination of the flight

(A) Bangkok (B) Copenhagen (C) Dubai (O) Calcutta

You will hear Now read question 2 in your test book and answer it You will read 2 What will happen in a few minutes

(A) The flight will land in Dubai I

(B) The passengers will board the plane (C) The plane will take off (0) The gate number will be announced

Reading In this section of the test you will have the chance to show how well you understand written English There are three parts to this section with special directions for each part

Part 4 Incomplete Sentences Directions This part of the test has incomplete sentences Four words or phrases marked (A) (8) (e) (D) are given beneath each sentence You are to choose the one word or phrase that best completes the sentence Then on your answer sheet find the number of the question and mark your answer

1 Mr Yangs trip will __ him away from the office for ten days (A) withdraw (B) continue (C) retain (0) keep

2 The company that Marie DuBois started now sells __ products throughout the world (A) its (B) it (C) theirs (D) them

3 If your shipment is not delivered __ Tuesday you can request a full refund for the merchandise (A) at (B) by (C) within (D) while

CHAPTER 4 Standardized Testing 103

Part 6 Error Recognition Directions In this part ofthe test each sentence has four words or phrases underlined The four underlined parts of the sentence are marked (A) (B) (C) (D) You are to identify the one underlined word or phrase that should be corrected or rewritten Then on your answer sheet find the number of the question and mark your answer

1 The pamphlet contains some importance information about the current exhibit ABC D

2 No matter how Jong it taking to finish the annual report it must be done properly ABC D

3 The popularity of jogging appears to have decreased since the past couple of years ABC D

Part 7 Reading Comprehension Directions The questions in this part of the test are based on a selection of reading mateshyrials such as notices letters) forms newspaper and magazine articles) and advertisements You are to choose the one best answer (A) (B) (C) or (OJ to each quesshytion Then on your ariswefsheelfindthe number of the qUestion andmcirkyour answer Answer all questions following each reading selection on thebasis of what is stated or implied in that selection

The Museum ofTechnology is a hands-on museum designed for people to experience science at w()rk~ Visitors are encouraged to use test and handle the objects o~ display Special demonstrations are scheduled for the first and second Wednesdays of each month at 1330 Open Tuesday-Friday 1200-1630 Saturday 1000-1730 and Sunday 11 00-1630

1 When during the month can visitors see special demonstrations (A) Every weekend (B) The first two Wednesdays (C) One afternoon a week (D) Every other Wednesday

Questions 2 and 3 refer to the followi ng notice

NOTICE If you are unable to work because of an extended illness or injury that is not workshyrelated you may be entitled to receive weekly benefits from your employer or the firms insurance company To claim benefits you must file a claim form within thirty days of the first day of your disability Before filing the claim you must ask your doctor to fill in the Doctors Statement on the claim form stating the period of disability

3 To whom is this notice addressed (A) Employers (8) Doctors (C) Employees (D) When paying the bill

4 When must the claim form be filed (A) On the first of the month (8) On the thirtieth of the month (C) On the first day ofdisabifity (D) Within 30 days of the start of disability

Page 35: Standardized Testing Chapter 4 Brown

100 CHAPTER 4 Standardized Testing

Some businesses now say that no one can smoke cigarettes in any of their offices Some governments have banned smoking in all public placesThis is a good idea but it takes away some of our freedom

Do you agree or disagree Give reasons for your answer You should write at least 250 words

Speaking In each ofthe three parts of the speaking module a specific function is fulfilled In Part 1 the candidates answer general questions about themselves their homes or families their jobs or studies their interests and a range ofsimilar familiar topic areas This part lasts between four and five minutes In Part 2 the candidate is given a verbal prompt on a card and is asked to talk on a particular topic The candidate has one minute to prepare before speaking at length for between one and two minutes The examiner then asks one or two wind-down questions In Part 3 the examiner and candidate engage in a discusshysion of more abstract issues and concepts which are thematically linked to the topic prompt in Part 2 The discussion lasts between four and five minutes

All interviews are recorded on audiocassette Here is a sample ofa Part 2 topic

Describe a teacher who has greatly influenced you in your education

You shou Id say

where you met them what subject they taught what was special about them

and explain why this person influenced you so much

You will have to talk about the topic for 1 to 2 minutes You have 1 minute to think about what you are going to say You can make some notes if you wish

Test of English for International Communication (TOEICreg)

listening

Part 1 Photographs Directions For each question you will see a picture in your test book and you will hear four short statements The statements will be spoken just one time They will not be printed in your test book so you must listen carefully to understand what the speaker says When you hear the four statements look at the picture in your test book and choose the statement that best describes what you see in the picture Then on your answer sheet find the number of the question and mark your answer

[photograph of a scientist looking through a microscope]

You will hear Look at the picture marked number 1 in your test book

(A) Shes speaking into a microphone (B) Shes put on her glasses (C) She has both eyes open (D) Shes using a microscope

CHAPTER 4 Standardized Testing 101

Part 2 Question-Response Directions In this part of the test you will hear a question or statement spoken in Enshyglish followed by three responses also spoken in English The question or staten1ent and the responses will be spoken just one time They will not be printed in your test book so you must listen carefully to understand what the speakers say You are to choose the best response to each question or statement

Question 1 You will hear Ms Morikawa has worked here for a long time hasnt she

(A) At three oclock (B) No Ive lost my watch (C) More than ten years

Question 2 You will hear Which of these papers has a wider circulation

(A) The morning edition (B) Get more exercise (C) By messenger

Part 3 Short Conversations Directions In this part of the test you will hear short conversations between two people The conversations will not be printed in your test book You will hear the conversations only once so you must listen carefully to understand what the speakers say In your test book you will read a question about each conversation The question will be followed by four answers You are to choose the best answer to each question and mark it on your answer sheet

Question 1 (Man) We should think about finding another restaurant for lunch (Woman) Why The food and service here are great

(Man) Yes but the prices are going up every week

You will read Why is this man unhappy with the restaurant

(A) It is too noisy (B) It is too expensive (C) It is too crowded (D) It is too difficult to find

Question 2 (Woman A) How was Dr Borgs recent trip to Singapore (Woman B) She enjoyed the tour of the port very much (Woman A) They say its one of the most active in Asia

You will read 2 What did Dr Borg find interesting

(A) The tourist center (B) The airport (C) The musical performance (D) The harbor

Part 4 Short Talks Directions In this part of the test you vill hear several short talks Each will be spoken just one time They will not be printed in your test book so you must listen carefully to understand and remember what is said In your test book you will read two or more questions about each short talk The questions will be followed by four answers You are to choose the best answer to each question and mark it on your answer sheet

102 CHAPTER 4 Standardized Testing

You will hear Questions 1 and 2 refer to the following announcement

Good afternoon and welcome aboard Nordair Flight 857 from Copenhagen to Bangkok with intermediate stops in Dubai and Calcutta We are preparing for departure in a few minutes At this time your seat back should be returned to its full upright position and your seat belt s~ould be fastened OUf anticipated total flying time to Dubai is six hours and twenty-five minutes I hope you enjoy the flight You will hecJr Now read question 1 in your test book and answer it You will read 1 What is the final destination of the flight

(A) Bangkok (B) Copenhagen (C) Dubai (O) Calcutta

You will hear Now read question 2 in your test book and answer it You will read 2 What will happen in a few minutes

(A) The flight will land in Dubai I

(B) The passengers will board the plane (C) The plane will take off (0) The gate number will be announced

Reading In this section of the test you will have the chance to show how well you understand written English There are three parts to this section with special directions for each part

Part 4 Incomplete Sentences Directions This part of the test has incomplete sentences Four words or phrases marked (A) (8) (e) (D) are given beneath each sentence You are to choose the one word or phrase that best completes the sentence Then on your answer sheet find the number of the question and mark your answer

1 Mr Yangs trip will __ him away from the office for ten days (A) withdraw (B) continue (C) retain (0) keep

2 The company that Marie DuBois started now sells __ products throughout the world (A) its (B) it (C) theirs (D) them

3 If your shipment is not delivered __ Tuesday you can request a full refund for the merchandise (A) at (B) by (C) within (D) while

CHAPTER 4 Standardized Testing 103

Part 6 Error Recognition Directions In this part ofthe test each sentence has four words or phrases underlined The four underlined parts of the sentence are marked (A) (B) (C) (D) You are to identify the one underlined word or phrase that should be corrected or rewritten Then on your answer sheet find the number of the question and mark your answer

1 The pamphlet contains some importance information about the current exhibit ABC D

2 No matter how Jong it taking to finish the annual report it must be done properly ABC D

3 The popularity of jogging appears to have decreased since the past couple of years ABC D

Part 7 Reading Comprehension Directions The questions in this part of the test are based on a selection of reading mateshyrials such as notices letters) forms newspaper and magazine articles) and advertisements You are to choose the one best answer (A) (B) (C) or (OJ to each quesshytion Then on your ariswefsheelfindthe number of the qUestion andmcirkyour answer Answer all questions following each reading selection on thebasis of what is stated or implied in that selection

The Museum ofTechnology is a hands-on museum designed for people to experience science at w()rk~ Visitors are encouraged to use test and handle the objects o~ display Special demonstrations are scheduled for the first and second Wednesdays of each month at 1330 Open Tuesday-Friday 1200-1630 Saturday 1000-1730 and Sunday 11 00-1630

1 When during the month can visitors see special demonstrations (A) Every weekend (B) The first two Wednesdays (C) One afternoon a week (D) Every other Wednesday

Questions 2 and 3 refer to the followi ng notice

NOTICE If you are unable to work because of an extended illness or injury that is not workshyrelated you may be entitled to receive weekly benefits from your employer or the firms insurance company To claim benefits you must file a claim form within thirty days of the first day of your disability Before filing the claim you must ask your doctor to fill in the Doctors Statement on the claim form stating the period of disability

3 To whom is this notice addressed (A) Employers (8) Doctors (C) Employees (D) When paying the bill

4 When must the claim form be filed (A) On the first of the month (8) On the thirtieth of the month (C) On the first day ofdisabifity (D) Within 30 days of the start of disability

Page 36: Standardized Testing Chapter 4 Brown

CHAPTER 4 Standardized Testing 101

Part 2 Question-Response Directions In this part of the test you will hear a question or statement spoken in Enshyglish followed by three responses also spoken in English The question or staten1ent and the responses will be spoken just one time They will not be printed in your test book so you must listen carefully to understand what the speakers say You are to choose the best response to each question or statement

Question 1 You will hear Ms Morikawa has worked here for a long time hasnt she

(A) At three oclock (B) No Ive lost my watch (C) More than ten years

Question 2 You will hear Which of these papers has a wider circulation

(A) The morning edition (B) Get more exercise (C) By messenger

Part 3 Short Conversations Directions In this part of the test you will hear short conversations between two people The conversations will not be printed in your test book You will hear the conversations only once so you must listen carefully to understand what the speakers say In your test book you will read a question about each conversation The question will be followed by four answers You are to choose the best answer to each question and mark it on your answer sheet

Question 1 (Man) We should think about finding another restaurant for lunch (Woman) Why The food and service here are great

(Man) Yes but the prices are going up every week

You will read Why is this man unhappy with the restaurant

(A) It is too noisy (B) It is too expensive (C) It is too crowded (D) It is too difficult to find

Question 2 (Woman A) How was Dr Borgs recent trip to Singapore (Woman B) She enjoyed the tour of the port very much (Woman A) They say its one of the most active in Asia

You will read 2 What did Dr Borg find interesting

(A) The tourist center (B) The airport (C) The musical performance (D) The harbor

Part 4 Short Talks Directions In this part of the test you vill hear several short talks Each will be spoken just one time They will not be printed in your test book so you must listen carefully to understand and remember what is said In your test book you will read two or more questions about each short talk The questions will be followed by four answers You are to choose the best answer to each question and mark it on your answer sheet

102 CHAPTER 4 Standardized Testing

You will hear Questions 1 and 2 refer to the following announcement

Good afternoon and welcome aboard Nordair Flight 857 from Copenhagen to Bangkok with intermediate stops in Dubai and Calcutta We are preparing for departure in a few minutes At this time your seat back should be returned to its full upright position and your seat belt s~ould be fastened OUf anticipated total flying time to Dubai is six hours and twenty-five minutes I hope you enjoy the flight You will hecJr Now read question 1 in your test book and answer it You will read 1 What is the final destination of the flight

(A) Bangkok (B) Copenhagen (C) Dubai (O) Calcutta

You will hear Now read question 2 in your test book and answer it You will read 2 What will happen in a few minutes

(A) The flight will land in Dubai I

(B) The passengers will board the plane (C) The plane will take off (0) The gate number will be announced

Reading In this section of the test you will have the chance to show how well you understand written English There are three parts to this section with special directions for each part

Part 4 Incomplete Sentences Directions This part of the test has incomplete sentences Four words or phrases marked (A) (8) (e) (D) are given beneath each sentence You are to choose the one word or phrase that best completes the sentence Then on your answer sheet find the number of the question and mark your answer

1 Mr Yangs trip will __ him away from the office for ten days (A) withdraw (B) continue (C) retain (0) keep

2 The company that Marie DuBois started now sells __ products throughout the world (A) its (B) it (C) theirs (D) them

3 If your shipment is not delivered __ Tuesday you can request a full refund for the merchandise (A) at (B) by (C) within (D) while

CHAPTER 4 Standardized Testing 103

Part 6 Error Recognition Directions In this part ofthe test each sentence has four words or phrases underlined The four underlined parts of the sentence are marked (A) (B) (C) (D) You are to identify the one underlined word or phrase that should be corrected or rewritten Then on your answer sheet find the number of the question and mark your answer

1 The pamphlet contains some importance information about the current exhibit ABC D

2 No matter how Jong it taking to finish the annual report it must be done properly ABC D

3 The popularity of jogging appears to have decreased since the past couple of years ABC D

Part 7 Reading Comprehension Directions The questions in this part of the test are based on a selection of reading mateshyrials such as notices letters) forms newspaper and magazine articles) and advertisements You are to choose the one best answer (A) (B) (C) or (OJ to each quesshytion Then on your ariswefsheelfindthe number of the qUestion andmcirkyour answer Answer all questions following each reading selection on thebasis of what is stated or implied in that selection

The Museum ofTechnology is a hands-on museum designed for people to experience science at w()rk~ Visitors are encouraged to use test and handle the objects o~ display Special demonstrations are scheduled for the first and second Wednesdays of each month at 1330 Open Tuesday-Friday 1200-1630 Saturday 1000-1730 and Sunday 11 00-1630

1 When during the month can visitors see special demonstrations (A) Every weekend (B) The first two Wednesdays (C) One afternoon a week (D) Every other Wednesday

Questions 2 and 3 refer to the followi ng notice

NOTICE If you are unable to work because of an extended illness or injury that is not workshyrelated you may be entitled to receive weekly benefits from your employer or the firms insurance company To claim benefits you must file a claim form within thirty days of the first day of your disability Before filing the claim you must ask your doctor to fill in the Doctors Statement on the claim form stating the period of disability

3 To whom is this notice addressed (A) Employers (8) Doctors (C) Employees (D) When paying the bill

4 When must the claim form be filed (A) On the first of the month (8) On the thirtieth of the month (C) On the first day ofdisabifity (D) Within 30 days of the start of disability

Page 37: Standardized Testing Chapter 4 Brown

102 CHAPTER 4 Standardized Testing

You will hear Questions 1 and 2 refer to the following announcement

Good afternoon and welcome aboard Nordair Flight 857 from Copenhagen to Bangkok with intermediate stops in Dubai and Calcutta We are preparing for departure in a few minutes At this time your seat back should be returned to its full upright position and your seat belt s~ould be fastened OUf anticipated total flying time to Dubai is six hours and twenty-five minutes I hope you enjoy the flight You will hecJr Now read question 1 in your test book and answer it You will read 1 What is the final destination of the flight

(A) Bangkok (B) Copenhagen (C) Dubai (O) Calcutta

You will hear Now read question 2 in your test book and answer it You will read 2 What will happen in a few minutes

(A) The flight will land in Dubai I

(B) The passengers will board the plane (C) The plane will take off (0) The gate number will be announced

Reading In this section of the test you will have the chance to show how well you understand written English There are three parts to this section with special directions for each part

Part 4 Incomplete Sentences Directions This part of the test has incomplete sentences Four words or phrases marked (A) (8) (e) (D) are given beneath each sentence You are to choose the one word or phrase that best completes the sentence Then on your answer sheet find the number of the question and mark your answer

1 Mr Yangs trip will __ him away from the office for ten days (A) withdraw (B) continue (C) retain (0) keep

2 The company that Marie DuBois started now sells __ products throughout the world (A) its (B) it (C) theirs (D) them

3 If your shipment is not delivered __ Tuesday you can request a full refund for the merchandise (A) at (B) by (C) within (D) while

CHAPTER 4 Standardized Testing 103

Part 6 Error Recognition Directions In this part ofthe test each sentence has four words or phrases underlined The four underlined parts of the sentence are marked (A) (B) (C) (D) You are to identify the one underlined word or phrase that should be corrected or rewritten Then on your answer sheet find the number of the question and mark your answer

1 The pamphlet contains some importance information about the current exhibit ABC D

2 No matter how Jong it taking to finish the annual report it must be done properly ABC D

3 The popularity of jogging appears to have decreased since the past couple of years ABC D

Part 7 Reading Comprehension Directions The questions in this part of the test are based on a selection of reading mateshyrials such as notices letters) forms newspaper and magazine articles) and advertisements You are to choose the one best answer (A) (B) (C) or (OJ to each quesshytion Then on your ariswefsheelfindthe number of the qUestion andmcirkyour answer Answer all questions following each reading selection on thebasis of what is stated or implied in that selection

The Museum ofTechnology is a hands-on museum designed for people to experience science at w()rk~ Visitors are encouraged to use test and handle the objects o~ display Special demonstrations are scheduled for the first and second Wednesdays of each month at 1330 Open Tuesday-Friday 1200-1630 Saturday 1000-1730 and Sunday 11 00-1630

1 When during the month can visitors see special demonstrations (A) Every weekend (B) The first two Wednesdays (C) One afternoon a week (D) Every other Wednesday

Questions 2 and 3 refer to the followi ng notice

NOTICE If you are unable to work because of an extended illness or injury that is not workshyrelated you may be entitled to receive weekly benefits from your employer or the firms insurance company To claim benefits you must file a claim form within thirty days of the first day of your disability Before filing the claim you must ask your doctor to fill in the Doctors Statement on the claim form stating the period of disability

3 To whom is this notice addressed (A) Employers (8) Doctors (C) Employees (D) When paying the bill

4 When must the claim form be filed (A) On the first of the month (8) On the thirtieth of the month (C) On the first day ofdisabifity (D) Within 30 days of the start of disability

Page 38: Standardized Testing Chapter 4 Brown

CHAPTER 4 Standardized Testing 103

Part 6 Error Recognition Directions In this part ofthe test each sentence has four words or phrases underlined The four underlined parts of the sentence are marked (A) (B) (C) (D) You are to identify the one underlined word or phrase that should be corrected or rewritten Then on your answer sheet find the number of the question and mark your answer

1 The pamphlet contains some importance information about the current exhibit ABC D

2 No matter how Jong it taking to finish the annual report it must be done properly ABC D

3 The popularity of jogging appears to have decreased since the past couple of years ABC D

Part 7 Reading Comprehension Directions The questions in this part of the test are based on a selection of reading mateshyrials such as notices letters) forms newspaper and magazine articles) and advertisements You are to choose the one best answer (A) (B) (C) or (OJ to each quesshytion Then on your ariswefsheelfindthe number of the qUestion andmcirkyour answer Answer all questions following each reading selection on thebasis of what is stated or implied in that selection

The Museum ofTechnology is a hands-on museum designed for people to experience science at w()rk~ Visitors are encouraged to use test and handle the objects o~ display Special demonstrations are scheduled for the first and second Wednesdays of each month at 1330 Open Tuesday-Friday 1200-1630 Saturday 1000-1730 and Sunday 11 00-1630

1 When during the month can visitors see special demonstrations (A) Every weekend (B) The first two Wednesdays (C) One afternoon a week (D) Every other Wednesday

Questions 2 and 3 refer to the followi ng notice

NOTICE If you are unable to work because of an extended illness or injury that is not workshyrelated you may be entitled to receive weekly benefits from your employer or the firms insurance company To claim benefits you must file a claim form within thirty days of the first day of your disability Before filing the claim you must ask your doctor to fill in the Doctors Statement on the claim form stating the period of disability

3 To whom is this notice addressed (A) Employers (8) Doctors (C) Employees (D) When paying the bill

4 When must the claim form be filed (A) On the first of the month (8) On the thirtieth of the month (C) On the first day ofdisabifity (D) Within 30 days of the start of disability


Recommended